You are on page 1of 449

ISHANT ANAND PHYSICS CLASSES

An Ideal Destination for the Best Preparation for


JEE (Main & Adv.)/NEET

JEE NEET

CLASS XI
PHYSICS
MODULE

Branches:
• SCO - 93, HUDA Market, Sector - 3, Faridabad • SCF - 124-125, HUDA Market, Sector - 17, Faridabad
Contact : 9350626145, 8733854892
INDEX
• Chapter Utility Score

1. Physical World ..................................................................................................1-8

2. Units and Measurements ................................................................................9-36

3. Motion in a Straight Line ............................................................................... 37-64

4. Motion in a Plane ......................................................................................... 65-96

5. Laws of Motion ........................................................................................... 97-128

6. Work, Energy and Power ........................................................................... 129-160

7. System of Particles and Rotational Motion .................................................. 161-194

8. Gravitation ............................................................................................... 195-224

9. Mechanical Properties of Solids.................................................................. 225-245

10. Mechanical Properties of Fluids.................................................................. 246-277

11. Thermal Properties of Matter ..................................................................... 278-302

12. Thermodynamics ...................................................................................... 303-326

13. Kinetic Theory ........................................................................................... 327-349

14. Oscillations ............................................................................................... 350-376

15. Waves ...................................................................................................... 377-408

• Value Based Questions 409-415


• Important Formulae, Terms and Definitions (i-iv)
• Sample Paper-1 SP-1 – SP-3
• Sample Paper-2 SP-4 – SP-6
• Sample Paper-3 SP-7 – SP-9
• SOLUTIONS TO SAMPLE PAPERS 1-3 SP-10-28
1 Physical World
C ha p t e r

WHAT IS PHYSICS ?
Science refers to a system of acquiring knowledge. This system uses observation and experimentation to describe and explain natural
phenomena. The term science also refers to the organized knowledge people have gained using that system. Less formally, the word
science often describes any systematic field of study or the knowledge gained from it.
The purpose of science is to produce useful models of reality. Most scientific investigations use some form of the scientific method.
The scientific method is a logical and rational order of steps by which scientists come to conclusions about the world around them.
The Scientific method helps to organize thoughts and procedures so that scientists can be confident in the answers they find.
Scientists use observations, hypotheses, and deductions to make these conclusions.
Science as defined above is sometimes called pure science to differentiate it from applied science, which is the application of research
to human needs.
Fields of science are commonly classified into two major lines:
Natural sciences the study of the natural world, and Social sciences, the systematic study of human behavior and society. Natural
science includes physics, chemistry, biology, etc.
Physics is the study of the interactions between physical systems. The physicist attempt to describe the interaction with the
most fundamental and general law or principle possible.
Two principal thrusts in physics are (i) unification (ii) reductionism
(i) Unification : Attempts to explain diverse physical phenomena in terms of a few concepts and laws. Attempts to unify fundamental
forces of nature reflects this quests.
(ii) Reductionism : Attempts to drive the properties of a bigger more complex system from the properties and interactions of its
constituent simpler parts. For example thermodynamics was developed to explain macroscopic quantities such as temperature,
internal energy etc. of the bulk system. Later on these properties were explained in terms of molecules in kinetic theory and
statistical mechanics.

SCOPE AND EXCITEMENT OF PHYSICS


The scope of physics is truly vast. It covers a tremendous range of magnitude of physical quantities like length, mass, time, energy,
etc. Basically, there are two domains of interest : macroscopic and microscopic. The macroscopic domain includes phenomena at the
laboratory, terrestrial and astronomical scales.
The microscopic domain includes atomic, molecular and nuclear phenomena.
Classical physics deals mainly with macroscopic phenomena consisting of the study of heat, light, electricity, magnetism, optics,
acoustics, and mechanics. Since the turn of the 20th century the study of quantum mechanics and relativity has become more
important. Today, physics is often divided into branches such as nuclear physics, particle physics, quantum physics, theoretical
physics, and solid-state physics. The study of the planets, stars, and their interactions is known as astrophysics, the physics of the
Earth is called geophysics, and the study of the physical laws relating to living organisms is called biophysics.
The microscopic domain of physics deals with the constitution and structure of matter at the minute scales of atoms and nuclei.
Physics is exciting in many ways. Application and exploitation of physical laws to make useful devices is the most interesting and
exciting part and requires great ingenuity and persistence of effort.
Recently mesoscopic physics the domian intermediate between the macroscopic and the microscopic dealing with a few tens or
hundreds of atoms emerged as an exciting field of research.
2 Physics
PHYSICS, TECHNOLOGY AND SOCIETY
We hear terms like science, engineering , technology all in same context Technology
though they are not exactly same. Scientists Products and
Investigate the processes created
Science : natural world by engineers
• A body of knowledge.
• Seeks to describe and understand the natural world and its
physical properties. Society
Values
• Scientific knowledge can be used to make predictions. Needs
• Uses a process-the scientific method-to generate knowledge. Environment
Economy
Engineering :
• Design under constraint. Scientific
Knowledge/ physics Engineers
• Seeks solutions for societal problems and needs. Create the
What scientists
• Aims to produce the best solution from given resources and designed world
have learned about
constraints. the natural world
• Uses a process–the engineering design process–to produce
solutions and technologies.
Technology :
• Almost everything made by humans to solve a need is a technology.
• The body of knowledge, processes, and artifacts that result from engineering.
• Use in making pencils, shoes, cell phones, and processes to treat water.
In the real world, these disciplines are closely connected. Scientists often use technologies created by engineers to conduct
their research. In turn, engineers often use knowledge developed by scientists to inform the design of the technologies they
create.
Link between technology and physics
Technology Scientific principle(s)
Aeroplane Bernoulli’s principle in fluid dynamics
Bose-Einstein condensate Trapping and cooling of atoms by laser beams and magnetic fields
Computers Digital logic
Electric generator Faraday’s laws of electromagnetic induction
Electron microscope Wave nature of electrons
Fusion test reactor (Tokamak) Magnetic confinement of plasma
Giant Metrewave Radio Telescope (GMRT) Detection of cosmic radio waves
Hydroelectric power Conversion of gravitational potential energy into electrical energy
Lasers Light amplification by stimulated emission of radiation
Non-reflecting coatings Thin film optical interference
Nuclear reactor Controlled nuclear fission
Optical fibres Total internal reflection of light
Particle accelerators Motion of charged particles in electromagnetic fields
Photocell Photoelectric effect
Production of ultra high magnetic fields Superconductivity
Radio and television Generation, propagation and detection of electromagnetic waves
Rocket propulsion Newton’s laws of motion
Sonar Reflection of ultrasonic waves
Steam engine Laws of thermodynamics
4 Physics
Fundamental forces of nature
Name Relative Range Exchange Major role Important properties
strength particles
Gravitational force (Force of Universal attractive, weakest,
Large-scale
attraction between any two 10
–39
Infinite Gravitons long range, central,
structure
bodies by virtue of their masses) conservative force.
Weak nuclear force (Changes Very short, Govern process involving
quark types as in Beta-decay of –13
sub-nuclear Weak Nuclear neutrino and antineutrino,
nucleus 10 size (on bosons reactions operates only through a range
10
–16
m) of nuclear size.
Electromagnetic force (Force Either attractive or repulsive,
between particles with –2 Chemistry long range, central,
10 Infinite Photons
charge/magnetism) and Biology conservative force.

Strong nuclear force (Strong Basically an attractive becomes


attractive force which binds Very Short, Holding repulsive (when distance
together the protons and 1 nuclear size Gluons nuclei between nucleons < 0.5 fermi)
neutrons in nucleus) (on 10
–15
m) together strongest, short range, non-
central, non-conservative force.

Physicists are trying to derive a unified theory that would describe all the forces in nature as a single fundamental law. So far, they have
succeeded in producing a unified description of the weak and electromagnetic forces, but a deeper understanding of the strong and
gravitational forces has not yet been achieved. Theories that postulate the unification of the strong, weak, and electromagnetic forces
are called Grand Unified Theories (often known by the acronym GUTs). Theories that add gravity to the mix and try to unify all four
fundamental forces into a single force are called Superunified Theories. The theory that describes the unified electromagnetic and
weak interactions is called the Standard Electroweak Theory, or sometimes just the Standard Model.
Progress in unification of different forces/domains in nature

Name of the phys icis t Year Achievement in unification

Unified celes tial and terrestrial mechanics ; s howed that the same laws
Is sac Newton 1687
of motion and the law of gravitation apply to both the domains.

Hans Chris tian Oers ted 1820 Showed that electric and magnetic phenomena are ins eparable as pects
of a unified domain : electromagnetis m.
Michael Faraday 1830
Unified electricity, magnetism and optics ; s howed that light is an
James Clark Maxwell 1873
electromagnetic wave.
Sheld on Glas how, A bdus Salam, Showed that the 'weak' nuclear force and the electromagnetic force
1979
Steven W einberg could be viewed as different aspects of a s ingle electro-weak force.
Verified experimentally the predictions of the theory of electro-weak
Carlo Rubia, Simon Vander M eer 1984
force.

NATURE OF PHYSICAL LAWS


A physical law, scientific law, or a law of nature is a scientific generalization based on empirical observations of physical behavior.
Empirical laws are typically conclusions based on repeated scientific experiments and simple observations, over many years, and
which have become accepted universally within the scientific community. The production of a summary description of nature in the
form of such laws is a fundamental aim of science.
Physical laws are distinguished from scientific theories by their simplicity. Scientific theories are generally more complex than laws;
they have many component parts, and are more likely to be changed as the body of available experimental data and analysis develops.
This is because a physical law is a summary observation of strictly empirical matters, whereas a theory is a model that accounts for the
observation, explains it, relates it to other observations, and makes testable predictions based upon it. Simply stated, while a law notes
that something happens, a theory explains why and how something happens.
Conservation laws have a deep connection with symmetries of matter. The law of conservation of energy is thought to be valid across
all domains of nature from the microscopic to macroscopic symmetries of space and time and other types of symmetries play a central
role in modern theories of fundamental forces in nature.
Physical World 5
6 Physics

Textbook Exercises
1.1 Some of the most profound statements on the nature of life. But there is no phenomenon which can be explained
science have come from Albert Einstein, one of the on the basis that ghosts exist though they are not seen.
greatest scientists of all time. What do you think did 1.6 The shells of crabs found around a particular coastal
Einstein mean when he said. “The most incomprehensible location in Japan seem mostly to resemble the legendary
thing about the world is that it is comprehensible” ? face of a Samurai. Given below are two explanations of
Sol. The whole of physical world is very complex. Vastly this observed fact. Which of these strikes you as a
different orders of magnitudes are involved in space, time scientific explanation?
and mass. Even then, almost all the physical phenomena (a) A tragic sea accident several centuries ago
can be expressed in terms of few basic laws. This is what drawned a young Samurai. As a tribute to his
Einstein meant to say. bravery, nature through its inscrutable ways
1.2 “Every great physical theory starts as a hearsay and immortalised his face by imprinting it on the crab
ends as a dogma” Give some examples from the history shells in that area.
of science of the validity of this incisive remark. (b) After the sea tragedy, fishermen in that area, in a
Sol. Here say i.e., opinion against conventional belief and gesture to honour to their dead hero, let free any
dogma i.e., established belief. Copernicus geocentric crab shell caught by them which accidentally had
theory (Sun at the centre of solar system) started as a shape resembling the face of a samurai.
heresay and ended as dogma when explained by Tycho Consequently the particular shape of the crab shell
Brahe and Johnes Kepler. survived longer and therefore in course of time the
shape was genetically propagated. This is an
1.3 “Politics is the art of the possible”? Similarly “Science
example of evalution by artificial selection.
is the art of the soluble”. Explain this beautiful aphorism
[Note : This interesting illustration taken from Carl
on the nature and practice of science.
Sagan’s ‘The Cosmos’ highlights the fact that often
Sol. Politics is the art of possible. In politics, politicians do strange and inexplicable facts which on the first sight
not follow any principle, discipline or norm. They just appear ‘supernatural’ actually turn out to have simple
want power by any means fair or foul. Science is a scientific explanations. Try to think out other examples
systematised study of observation. A scientist potiently of this kind].
analyses these observations and come out with certain Sol. (b)
laws. Thus, science is the art of the soluble. 1.7 The industrial revolution in England and Western
1.4 Though India now has a large base in science and Europe more than two centuries ago was triggered by
technology, which is fast expanding, it is still a long way some key scientific and technological advances. What
from realising its potential of becoming a world leader were these advances ?
in science. Name some important factors, which in your Sol. Some key scientific and technological advances
view have hindered the advancement of science in India. responsible for industrial revolution are
Sol. Some of the importnat factors which have hindered the (i) Steam engine based on scientific principle. Law of
advancement of science in India are thermodynamics.
(i) Lack of infrastructure and funds for quality research (ii) Discovery of safety lamp by Humphry Davy for
work in science. mines worker to work safety.
(ii) The rural based science education is nearly non- (iii) Study of gravitation led to the study of motion and
existent so that majority of population is deprived making guns and cannons. This gave power in the
of the benefits of advancements in science and hands of western countries and they ruled over rest
technology. of the world.
(iii) Poor pay scales and other facilities to scientists. 1.8 It is often said that the world is witnessing now a second
Talented scientists and technologists prefer to industrial revolution, which will transform the society
migrate developed countries where they get better as radically as did the first. List some key contemporary
salaries and other benefits. areas of science and technology which are responsible
(iv) There is practically no co-ordination between the for this revolution.
researchers and the industrialists. The industrialists Sol. Science key contemporary areas of scien ce and
are the actual consumers of new research and technology, responsible for a new industrial revolution
technology. They prefer foreign technology rather are :
than to use country made technology. (i) Fabricatioin of super-fast computers.
1.5 No physicist has ever “seen” an electron. Yet, all (ii) Development of super-conducting materials at room
physicists believe in the existence of electrons. An temperature.
intelligent but superstitious man advances this analogy (iii) Biotechnology
to argue that ‘ghosts’ exist even though no one has (iv) Laser technology
‘seen’ one How will you refute his argument? (v) Developments in the field of space sciences.
Sol. Many phenomena based on the existence of electrons (vi) Advancements in the field of electronics, information
have been predicted, verified and observed in day-to-day and nanotechnology.
Physical World 7

Practice Questions
Very Short Answer Questions [1 Mark Qs.] 8. Which fundamental force governs the large scale motion in
the universe ?
1. How is science different from technology? 9. State one law that holds good in all natural processes.
2. What are the two basic quests in physics ?
Short Answer Questions [2 or 3 Marks Qs.]
3. What is scientific attitude ?
1. Discuss the relation of physics with other sciences.
4. What is the basic aim of science ? 2. What is physics ? Discuss the relation of physics with
5. Name the four fundamental forces of nature. technology.
3. Discuss the scope and excitement of physics.
6. What is electromagnetic force?
4. The scope of physics is truly vast. Comment.
7. Give the ratio of the the strengths of the four fundamental 5. Name three advancements made in technology on the basis
forces in nature. of physics.

HOTS/Exemplar Questions
1. Does imagination play any role in physics? [HOTS] 3. How physics is a science of excitement ? [HOTS]
2. Why do we call physics an exact science ? [HOTS]

CHAPTER TEST

Time : 30 min. Max. Marks : 15

Directions : (i) Attempt all questions


(ii) Questions 1 to 5 carry 1 mark each.
(iii) Questions 6 and 7 carry 2 marks each.
(iv) Questions 8 and 9 carry 3 marks each.

1. What is the origin of the word science ?


2. What is a theory ?
3. Name the scientist who first discovered the neutron.
4. Name one law that holds good in all natural processes.
5. What was the important discovery of C.V. Raman ?
6. How is science different from technology ?
7. How much is the range of gravitational forces ? What are the messanger particles for this force.
8. What is the basic quest of modern physicists ? Mention the various significant attempts made towards the unification of
forces ?
9. Why was science called natural philosphy in early days ?
8 Physics

Solutions
PRACTICE QUESTIONS 3. Refer to theory
4. Refer to theory
Very Short Answer Questions 5. The applications of physics have played a key role in the
1. Science is the study of natural laws while technology is the development of technology. Today we see the applications
practical application of these laws to the daily life problems. of physics in every walk of life.
2. (i) Unification and (ii) Reductionism (i) Electromagnetic waves are used in radio, television,
3. A flexible open-minded approach towards solving problems radar and wireless communication.
in which other important points of view are not neglected. (ii) Newton’s concept of gravitation is used in
4. To search for truth. It tends to analyse the natural geostationary satellites which help us in forecasting
phenomena occuring around us. weather and in geophysical survey.
5. Four fundamental forces of nature (iii) The study of thermodynamics has helped to design
(i) Gravitational force (ii) Weak nuclear force heat engines which have revolutionised the industry.
(iii) Electromagnetic force (iv) Strong nuclear force
6. It is the force due to interaction between two moving HOTS/EXEMPLAR QUESTIONS
charges. It is caused by exchange of photons (g) between
1. Yes, imagination has layed an important role in the
two charged particles.
development of physics. Huygen’s principle, Bohr’s theory,
7. FGravitation : FWeak nuclear : FElectromagnetic : FStrong necluear
Maxwell equations, Heisenberg’s uncertainty principle, etc.
= 1 : 1025 : 1036 : 1038 were the imaginations of the scientists which successfully
8. Gravitational force explained the various natural phenomena.
9. One such law is the Newton’s law of gravitation. 2. In physics, physical quantities are measured with high
precision and accuracy. Hence physics is called an exact
Short Answer Questions
science.
1. Physics is the most fundamental discipline of all sciences. 3. The study of physics is not only educative but also exciting
It has played a key role in the development of all other in many ways as follows :
sciences.
(i) Inspite of the wide range and complexity of the physical
(i) Physics in relation to mathematics : Physics is a
phenomena, it is quite exciting that these phenomena
quantitative science. Mathematics provides the
can be analysed and understood in terms of few
necessary signs and tools which the physicists use. It
universal laws and principles.
has played an important role in the development of
theoretical physics. Had Newton not invented calculus, (ii) Some other people get excitement in carrying out new
he would not have been able to discover the universal challenging experiments to unfold the mysteries of
law of gravitation. nature and in verifying or falsifying the existing
(ii) Physics in relation to chemistry : In physics, we study theories.
the structure of atom, radioactivity, X-ray diffraction,
CHAPTER TEST
etc. Such studies have enabled chemists to arrange
elements in the periodic table on the basis of their 1. From the latin word scientia which means to know.
atomic numbers. This has further helped to know the 2. A minimum number of laws in terms of which the behaviour
nature of valency and chemical bonding and to of a physical system can be explained.
understand the complex chemical structures. 3. James Chadwick in 1932.
(iii) Physics in relation to biological sciences : The
4. Newton’s law of gravitation.
developments in life sciences owe a great deal to
physics. (i) Optical microscopes are extensively used 5. Inelastic scattering of light by molecules.
in the study of biology. (ii) With the help of an electron 6. Science is the study of natural laws while technology is the
microscope, one can study the structure of cell. practical application of these laws to everyday problems.
(iii) The X-rays and neutron diffraction techniques 7. Range - Infinite ; Messenger particles – Gravitons
have helped in understanding the structure of nucleic
8. Refer to theory
acids, which helped to control vital life processes.
(iv) Radioisotopes are used in radiation therapy for 9. It is because in earlier days scientific knowledge was gained
the cure of deadly diseases like cancer. from the direct study of natural phenomena without any
2. Refer to theory experimentation.
¿¿¿
2 Units and
Measurements
C ha p t e r

Measurement of any physical quantity is the comparison of the quantity with its unit.
Unit of a physical quantity is defined as a chosen standard of measurement having the same nature as that of quantity.
PHYSICAL QUANTITIES
All quantities which can be measured and in terms of which the laws of physics can be expressed are called physical quantities.
Physical quantities are of two types: (i) Fundamental or Base quantity and (ii) Derived quantity.
(i) Fundamental quantities are those physical quantities which can be described on their own and are independent of each other.
There are seven fundamental quantities mass, length, time temperature, electric current, luminous intensity and amount of
substance.
(ii) Derived physical quantities are those quantities which are obtained from fundamental quantities such as
speed = distance/time,
acceleration = change in velocity/time
force = mass × acceleration etc.
There are two supplementary physical quantities: Plane angle and solid angle

UNITS
The reference standard used for the measurement of a physical quantity is called a unit.
In the measurement, the numerical value of a physical quantity (n) is inversely proportional to the size of the unit(u).
1
i.e., n µ or, nu = constant or n 1u1 = n2u2
u
Fundamental or Base and Derived units
Fundamental units : The units of fundamental physical quantities are called fundamental units. They are kilogram (mass), metre
(length), second (time), kelvin (temperature), ampere (electric current), Candela (Luminous intensity) and mol (amount of substance).
Derived units : The units of derived physical quantities are called derived units e.g., unit of speed metre per second = unit of
length/ unit of time; unit of acceleration metre per second 2 = unit of velocity or speed/ time etc.
System of Units
A complete set of fundamental units and derived units, is known as system of units. There are four different system of units
(i) CGS system i.e, centimetre, gram and second system.
(ii) FPS system i.e., foot, pound and second system.
(iii) MKS system i.e., metre, kilogram and second system
Above three systems on the basis of units of three fundamental units for mass, length and time.
(iv) SI system is the abbreviated form of System Internationale d unites. This system at present internationally accepted for
measurement. Because SI units used decimal system, conversions within the system are quite simple and convenient.
Definitions of 7 S.I. Fundamental Units and 2 Supplementary Units
(i) The metre (m) : It is the SI unit of length and is equal to the length of the path travelled by light in vacuum during a time interval
of 1/299,792,4 sec
(ii) The kilogram (kg) : It is the SI unit of mass and is defined as the mass of the international prototype of the kilogram which is
a standard block of Platinum Iridium alloy kept at International Bureau of Weights and Measures in Paris.
10 Physics
(iii) The second (s) : It is the SI unit of time and is defined as the duration of 9,192,631,770 periods of radiation corresponding to the
transition between two hyperfine levels of the ground state of cesium-133 atom.
(iv) The ampere (A) : It is the SI unit of electric current and is defined as the amount of current flowing through each of the two
straight parallel conductors of infinite length 1m apart in vacuum which produces a force per unit length of 2 × 10–7 N/m between
them.
1
(v) The kelvin (K) : It is the SI unit of temperature and is defined as the fraction of the thermodynamic temperature at the
273.16
triple point of water.
(vi) The candela (cd) : It is the SI unit of luminous intensity and is defined as the intensity in a perpendicular direction of 1/60,000
sq.m area of a black body at freezing point of platinum under a pressure of 101,325 N/m2.
(vii) The mole (mol) : It is the SI unit of quantity of matter and is defined as the amount of substance which contains as many
elementary particles as there are atoms in 12 g of C-12.
Two supplementary units:
(i) The radian (rad) : It is the SI unit of angle and is defined as the angle subtended at the centre of a circle by an arc of length equal
to the radius of the circle.
(ii) The steradian (sr) : It is the SI unit of solid angle and is defined as the solid angle subtended at the centre of a sphere by the
surface of a sphere equal in area to the square of the radius of the sphere.
Measurement of Length
For the measurement of length we use some direct and indirect methods. Direct methods include metre scale that is used for lengths
from 10–3 m to 102m. A vernier callipers is used for lengths to an accuracy of 10 –4 m. A screw gauge and a spherometer can be used
to measure lengths as less as to 10–5 m. To measure lengths beyond these ranges, we make use of some special indirect methods.
Measurement of Large Distances
Large distances such as the distance of a planet or a star from the earth cannot be measured directly with a metre scale. An important
method in such cases is the parallax method.
Parallax method: To measure the distance D of a far away planet ‘P’ by the parallax method, we observe it from two different positions
(observatries) A and B on the earth.
b
As the planet P is very far away, < < 1 and therefore q is very small. Wee
D
approximately take AB as an arc of length b of a circle with centre of P and the
distance D as the radius.
AP = BP so that AB = b = Dq where q is in radians.
Having determined D same way determine the size or angular diameter of the
planet.
If a = angular size of the planet then
d
a= where, d = diameter of the planet.
D
For objects nearer than about 20 meters, we have the ability to distinguish their relative distances. The brain does this by using the
fact that each of our two eyes sees objects from a slightly different point of view. Close, or cover, your eyes alternately and observe
this difference. The position of nearby objects relative to more distant objects is different for each eye. This difference, due to
different view points, is called parallax.
Some commonly used units of length in astronomical measurement
(i) Astronomical Unit (AU) is defined as the average distance between the centre of the sun and that of the earth.
1AU = 1.496 × 1011 m
(ii) Light year (ly) is the distance travelled by light in vacuum in one year.
1 light year = (3 × 108 × 365 × 24 × 60 × 60) m = 9.46 × 1015 m
(iii) Parallactic second (parsec) is defined as the distance, through which an arc of length 1 AU subtends an angle of 1 second.
1 1 p P
q = 1sec = degree = ´ radian
60 ´ 60 60 ´ 60 180
l = 1 AU = 1.496 × 1011 m
O q =1¢¢ l = 1AU
l 1.496 ´ 1011
\ r = = 16
= 3.084 × 10 m \ 1 parsec = 3.084 × 10 m 16
r=1
q 1 p parse
´ c
60 ´ 60 180
Q
1 ly = 6.3 × 104 AU \ 1 parsec = 3.26 ly
Units and Measurements 11

Units of length used in small measurements:


1 micron (1µm) = 10–6 m ; 1 nanometre (1 nm) = 10–9 m
1 angstrom (1Å) = 10–10 m ; 1 fermi (1 fm) = 10–15 m
1 × ray unit = 1 × u = 10–13 m
Some commonly used units of mass:
For heavy measurements, 1 metric ton = 1000 kg and 1 quintal = 100 kg
The largest unit of mass is Chandra Shekhar Limit (C.S.L).
1 C.S.L = 1.4 × mass of the sun.
For small measurements, 1 atomic mass unit (1 a.m.u. or 1 u) = 1.66 × 10–27 kg
Some commonly used units of time:
Solar day: It is the time taken by the earth to complete one rotation about its axis. 1 Solar day = 24 hours
Solar year: It is the time taken by the earth to complete one revolution around the sun in its orbit.
1 Solar year = 365 days (average)
Lunar month: It is the time taken by moon to complete one revolution around the earth in its orbit.
1 lunar month = 27.3 days
Shake: It is the smallest practical unit of time. 1 shake = 10–8 s
Prefixes for SI Units
In Physics we have to deal from very small (micro) to very large (macro) magnitudes. To express such large and small magnitudes
simultaneously we use following prefixes:
Prefixes for powers of ten :

Multiple of 10 Prefix Symbol Sub -multiple Prefix Symbol


1024 yotta Y 10 –1 deci d
1021 zetta Z 10 –2 centi c
1018 exa E 10 –3 milli m
1015 peta P 10–6 micro *m
1012 tera T 10 –9 nano n
10 9 giga G 10–12 pico p
10 6 mega M 10–15 femto f
10 3 kilo k 10–18 atto a
10 2 hecto h 10–21 zepto z
10 deca da 10–24 yocto y

When a prefix is placed before the symbol of unit, the combined prefix and symbol should be considered as one new symbol which
can be raised to a positive or negative power without any bracket, e.g., km3 means (103 m)3 but never 103 m3.
Illustration 1 :
Express 1 parsec in terms of metre. Write its order of magnitude.
Sol. 1 parsec =3.08 × 1016 m
Here 0.5 < 3.08 < 5
\ Order of magnitude = 16

Illustration 2 :
A planet is at a distance of 8.427 × 108 km from earth. Its angular diameter is 35.72'' of an arc. Calculate the diameter of the
planet.
Sol. Diameter of the planet, d = Dq
35.72 p
Here, D = 8.247 × 108 km, q = 35.72 ¢¢ = ´ radian
3600 180
35.72 p
\ D = (8.247 × 108) × ´ = 1.426 × 105 km.
3600 180
12 Physics
ACCURACY AND PRECISION
Accuracy refers to the closeness of observed values to its true value of the quantity while precision refers to closeness between the
different observed values of the same quantity. High precision does not mean high accuracy. The difference between accuracy and
precision can be understand by the following example : Suppose three students are asked to find the length of a rod whose length is
known to be 2.250 cm. The observations are given in the table.

Student Measurement-1 Measurement-2 Measurement-3 Average length


A. 2.25 cm 2.27 cm 2.26 cm 2.26 cm
B. 2.252 cm 2.250 cm 2.251 cm 2.251 cm
C. 2.250 cm 2.250 cm 2.251 cm 2.250 cm

It is clear from the above table, that the observations taken by student A are neither precise nor accurate. The observations of student
B are more precise. The observations of student C are precise as well as accurate.
ERRORS IN MEASUREMENTS
The result of every measurement by any measuring instrument contains some uncertainty. This uncertainty is called error. Every
calculated quantity which is based on measured values also has an error.
Error = true value – measured value
Types of Errors
There are mainly three types of errors :
1. Systematic errors : Errors with known causes and therefore can be minimised. The main sources of systematic errors are as
follows:
(i) Instrumental error; (ii) Personal error; (iii) Errror due to wrong setting of apparatus in an experiment
(iv) Error due to change in external factors like figure, temperature etc. during an experiment.
2. Random errors : Errors for which the causes are not known precisely and can be minimised by taking multiple measurements.
The mean value of all the measured values is close to the accurate value.
3. Gross errors: arises due to the carelessness of the observer and hence can be minimised by taking average of all readings.
4. Least count errors : Least count is the smallest value that can be measured by the measuring instrument. The error associated
with the resolution of the instrument is called the least count error.
ABSOLUTE ERROR, RELATIVE ERROR AND PERCENTAGE ERROR
Absolute Error
The magnitude of the difference between the individual measurement and the true value of the quantity is called the absolute error of
the measurement.
Let a1, a2, a3 ............. an are the measured values of a physical quantity a.
n
å ai
i =1
Mean of a = amean = \ Absolute error is a1 = Da1 = am - a1
n
Da2 = am - a2
................................
Dan = am - an
n
å | Dai |
| Da1 | + | Da2 | +... + | Dan | i =1
\ Mean absolute error, Damean = Þ Damean = n
n

\ Range of measured values of a will be am + Damean and amean - Damean

Relative or Fractional Error


Damean
It is the ratio of the mean absolute error to the mean value of the physical quantity i.e., da = ;
amean
Units and Measurements 13

Percentage Error
It is the relative error expressed in per cent.
Damean
Percentage error = da ´ 100% = ´ 100%
amean

COMBINATION OF ERRORS
(i) Error in sum of two quantities: If a physical quantity (x) can be expressed as the sum of two physical quantities a and b as x
= a + b then error in x will be
Dx = ± ( Da + Db ) where Da and Db are absolute errors in a and b respectively.
(ii) Error in the difference of two quantities: If x = a – b then error in x will be, Dx = ± (Da + Db)
Hence in the sum or difference of two quantities, the absolute error in final result is the sum of the absolute errors in the
individual quantities.
Dx Da Db
(iii) Error in product of two quantities: If x = a × b then the fractional error in x will be =± ±
x a b
a D x æ D a Db ö
(iv) Error in the division of two quantities: If x = then the fractional error in x will be = ±ç ± ÷
b x è a b ø

Hence relative error in the result when two quantities are multiplied or divided is the sum of the relative errors.

a m bn
(v) Error in the power of a quantity: If x = then the relative or fractional error in x will be
cr
Dx æ Da Db Dc ö
= ±çm +n +r
x è a b c ÷ø

SIGNIFICANT FIGURES
Significant figures indicate the precision of measurement which depends on the least count of the measuring instruments.
The reliable digits plus the first uncertain digit in the measurement are known as significant figures.
For example, if the length measured is 264.5 cm. here digits 2, 6 and 4 are certain digits while 5 is uncertain. Hence number of significant
figure is four.
Rules to Determine the Numbers of Significant Figures
· All non-zero digits are significant. 235.75 has five significant figures.
· All zeroes between two non-zero digits are significant. 2016.008 has seven significant figures.
· All zeroes occurring between the decimal point and the non-zero digits are not significant, provided there is only a zero to left
of the decimal point. 0.00652 has three significant figures.
· All zeroes written to the right of a non-zero digit in a number written without a decimal point are not significant. This rule does
not work if the zero is a result of measurement. 54000 two significant figures. 54000m five significant figures.
· All zeroes occurring to the right of a non-zero digit in a number written with a decimal point are significant. 32.2000 has six
significant figures.
· When a number is written in the exponential form, the exponential term does not contribute towards the significant figures.
2.465 × 105 has four significant figures.
Rules of Arithmetic Operations with Significant Figures
In arithmetical operations involving significant figures, the answer is reported in such a way that it reflects the reliability of the least
precise operation.
In Addition and Subtraction:For addition and subtraction, look at the decimal portion (i.e., to the right of the decimal point) of the
numbers only follow the following procedure :
(a) Count the number of significant figures in the decimal portion of each number in the problem. (The digits to the left of the
decimal place are not used to determine the number of decimal places in the final answer)
(b) Add or subtract in the normal fashion.
(c) Round off the answer to the least number of places in the decimal portion of any number in the problem.
In Multiplication and Division: In multiplication or division the number of significant figures in the final answer is the same as the
minimum number of the significant figures in the physical quantities being operated.
In Exponential notation: Sometimes this would required expressing the answer in exponential notation.
14 Physics
Rounding Off
Rules of Rounding off Uncertain Digits
(a) The preceding digit is raised by 1 if the uncertain digit to be dropped is more than 5 and is left unchanged if the latter is less than 5.
Example : x = 5.686 is rounded off to 5.69 (as 6 > 5)
x = 3.462 is rounded off to 3.46 (as 2 < 5)
(b) If the uncertain digit to be dropped is 5, the preceeding digit raised by 1 if it is odd and is left unchanged if it is even digit.
Example : 7.735 is rounded off to three significant figures becomes 7.74 as preceeding digit is odd.
7.745 is rounded off to 7.74 as preceeding digit is even.
Illustration 3 :
In an experiment, the refractive index of water was observed as 1.29, 1.33, 1.34, 1.35, 1.32, 1.36, 1.30 and 1.33. Calculate the
mean value, mean absolute error, relative error and percentage error in value of refractive index.
Sol. Mean value of refractive index,
1.29 + 1.33 + 1.34 + 1.35 + 1.32 + 1.36 + 1.30 + 1.33
m= = 1.33
8
Absolute error in measurement,
Dm1 = 1.33 - 1.29 = +0.04 , Dm 2 = 1.33 - 1.33 = 0.00 , Dm 3 = 1.33 - 1.34 = -0.01 , Dm 4 = 1.33 - 1.35 = -0.02 ,

Dm5 = 1.33 - 1.32 = +0.01 , Dm 6 = 1.33 - 1.36 = -0.03 , Dm 7 = 1.33 - 1.30 = +0.03 , Dm8 = 1.33 - 1.33 = 0.00
So, mean absolute error,
| 0.04 | + | 0.01| + | 0.02 | + | 0.01| + | 0.03 | + | 0.03 | + | 0 |
(Dm ) = = 0.0175 » 0.02
8
Dm 0.02
Relative error, ± =+ = ±0.015 = ±0.02
m 1.33
0.02
Percentage error, dm = ± ´ 100% = ±1.5%
1.33
Illustration 4 :
The length and breadth of a rectangle are (5.7 ± 0.1) cm and (3.4 ± 0.2) cm. Calculate area of the rectangle with error limits.
Sol. Here, l = (5.7 ± 0.1) cm, b = (3.4 ± 0.2) cm
Area A = l × b = 5.7 × 3.4 = 19.38 cm² = 19.0 cm² (rounding off to two significant figures)
DA æ Dl Db ö æ 0.1 0.2 ö æ 0.34 + 1.14 ö
=± çè + ÷ =± çè + ÷ =± çè ÷
A l b ø 5.7 3.4 ø 5.7 ´ 3.4 ø

DA 1.48 1.48 1.48


=± Þ DA = ± ´A =± ´ 19.38 = ±1.48
A 19.38 19.38 19.38
DA = ± 1.5 (rounding off to two significant figures) \ Area = (19.0 ± 1.5) sq.cm.
Illustration 5 :
A body travels uniformly a distance of (13.8 ± 0.2) m in a time (4.0 ± 0.3) s. Calculate its velocity with error limits. What is
percentage error in velocity?
Sol. Here, s = (13.8 ± 0.2) m t = (4.0 ± 0.3) s
s 13.8
Velocity, v= = = 3.45 ms–1 = 3.5 ms–1 (rounding off to two significant figures)
t 4.0
Dv æ Ds Dt ö æ 0.2 0.3 ö (0.8 + 4.14 ) Dv 4.94
=±ç + ÷ =±ç + =± Þ =± = ±0.0895
v è s t ø è 13.8 4.0 ÷ø 13.8 ´ 4.0 v 13.8 ´ 4.0
D v = ± 0.0895 × v = ± 0.0895 × 3.45 = ± 0.3087 = ± 0.31 (rounding off to two significant figures)
Hence, v = (3.5 ± 0.31) ms–1
Dv
% age error in velocity = ´ 100 = ± 0.0895 × 100 = ± 8.95 % = ± 9%
v
Units and Measurements 15

DIMENSIONS OF PHYSICAL QUANTITIES


The dimensions of a physical quantity are the powers to which the fundamental quantities mass, length and time must be raised to
represent it.
length
For example, Momentum = mass × velocity = mass ×
time
L
\ Dimension of momentum = M = [MLT -1 ]
T
Hence, the dimensions of momentum are 1 in mass, 1 in length and –1 in time

DIMENSIONAL FORMULAE AND DIMENSIONAL EQUATIONS


The expression which shows how and which fundamental quantities represent the dimensions of a physical quantity is known as
dimensional formula.
Ex. The dimensional formula of force is [MLT–2]
When a dimensional formula is equated to its physical quantity then the equation is called dimensional equation.
Ex. Dimensional equation of force :
By F = ma
Þ Dimension equation of force, F = [M1] [L1 T–2] = [MLT–2]
Ex. Dimensional equation of energy :
By E = W = force × displacement

Dimensional equation of energy, E = [M1 L1 T–2] [L1] = [M L2 T–2]


Dimensional Formula of Some Important Physical Quantities

S.No. Physical quantity Relation with other quantities Dimensional formula

Length
1. Velocity (v) [M0LT–1]
Time
Velocity
2. Acceleration (a) [M0LT–2]
Time
3. Momentum (p) Mass × velocity [MLT–1]

4. Force (F) Mass × acceleration [MLT–2]

5. Work Force × displacement [ML2T–2]

6. Power (P) Work [ML2T–3]


Time

Fr 2
7. Universal gravitational constant G= [M–1L3T–2 ]
m1m 2

8. Torque t = r ´F [ML2T–2]

9. Surface tension F [MT–2]


S=
l
W
10. Gravitational potential VG = [M0L2T–2]
m
F
11. Coefficient of viscosity h= [ML–1T–1]
dv
A
dx
12. Impulse Force×time(F×t) [MLT–1]
16 Physics
Change in length æ DL ö
13. Strain ç ÷ [M0L0T0]
Original length è L ø

Pressure æ P ö
14. Pressure gradient ç ÷ [ML–2T–2]
Distance è l ø

Arc æsö
15. Plane angle ç ÷ [M0L0T0]
Radius of circle è r ø

Angle æqö
16. Angular velocity ç ÷ [M0L0T–1]
Time ètø

Moment of inertia of body æ I ö


17. Radius of gyration çç ÷÷ [M0L1T0]
Total mass of the body è å mi ø

18. Moment of force, moment of couple Force × distance (F × s) [ML2T–2]


19. Angular frequency 2p×frequency (2pn) [M0L0T–1]
Force
20. Pressure [ML–1T–2]
Area
Output work or energy æ W ö
21. Efficiency ç ÷ [M0L0T0]
Input work or energy è Q ø
22. Angular impulse Torque×time (t × t) [ML2T–1]
Energy æ E ö
23. Planck’s constant ç ÷
Frequency è n ø
[ML2T–1]

Heat energy æ Q ö
24. Heat capacity, Entropy ç ÷ [ML2T–2K–1]
Temperature è T ø

Heat energy æ Q ö
25. Specific heat capacity ç ÷ [M0L2T –2K–1 ]
Mass ´ temperature è m ´ DT ø

Heat energy ´ thickness æ Q DT ö


26. Thermal conductivity, K ç = -KA ÷ [MLT–3K–1]
Area ´ temperature ´ time è t Dx ø

Length
27. Thermal Resistance, R [M–1L–2T3K]
Thermal conductivity × area

Volume ´ (change in pressure) æ DP ö


28. Bulk modulus (B) or (compressibility)–1 ç -V ÷ [ML–1T–2]
Change in volume è DV ø

(Energy/area) æ Q = sAtT 4 ö
29. Stefan’s constant (s) ç ÷ [ML0T–3K–4]
Time ´ (temperature) 4 èç E = Q / A.t = sT 4 ÷ø

Pressure ´ volume æ PV ö
30. Universal gas constant R ç ÷
Mole ´ temperature è nT ø [ML2 T -2 K -1mol-1 ]

Work æ W ö
31. Voltage, electric potential (V) or ç ÷ [ ML2T -3A -1 ]
Charge è q ø
electromotive force (e)
Charge æqö
32. Capacitance (C) ç ÷ [ M -1L-2T 4A 2 ]
Potential difference è V ø
Units and Measurements 17

r Electric force æ F ö
33. ( )
Electric field E
Charge çè q ÷ø
[ MLT-3A -1 ]

r Force
34. Magnetic field(B), magnetic induction, [F = IlBsin q] [ML0T–2 A–1 ]
Current ´ length
magnetic flux density

35. Magnetic flux (fm) f = BAcosq [ML2T–2 A–1 ]

Magnetic flux æ f m ö
36. Inductance [ ML2T -2 A -2 ]
Current çè I ÷ø
coefficient of self inductance (L) or
coefficient of mutual inductance (M)

37. Magnetic field strength or

Magnetic moment æ M ö
magnetic moment density (I) ç ÷ [M0L–1 T0A]
Volume èVø

(Charge)2 æ q2 ö
38. Permittivity constant in free space eo ç ÷ é M -1L-3T 4A 2 ù
4π ´ electrostatic force(distance)2 çè 4p´ F ´ r 2 ÷
ø ë û

39. Faraday constant (F), charge Avagadro constant × elementry charge [M0 L0 TA mol-1 ]

40. Mass defect, (Dm) (Sum of masses of nucleons [ML0 T 0 ]


– mass of nucleus)(MP+MN –Mnucleus)

1
41. Resonant frequency (fr) Tr [M 0 L0 T -1 ]

æ1ö
42. Power of lens (Focal length)-1 ç ÷ é M 0 L-1T 0 ù
èf ø ë û

Speed of light in vacuum


43. Refractive index Speed of light in medium [M 0 L0 T 0 ]

2p é M 0 L-1T0 ù
44. Wave number
Wavelength ë û

45. Binding energy of nucleus Mass defect × (speed of light in vacuum)2 é ML2T -2 ù
ë û

1
46. Conductance (c) é M -1L-2 A 2 T 3 ù
Resistance ë û

æ pö (Pressure) ´ (radius) 4 é M 0 L3T –1 ù


47. Fluid flow rate çè ÷ø ë û
8 (Viscosity coefficient) ´ (length)

48. Inductive reactance (Angular frequency × inductance) é ML2T -3A -2 ù


ë û

49. Capacitive reactance (Angular frequency × capacitance)–1 é ML2 T -3 A -2 ù


ë û

Torque é M 0 L2 T0 A ù
50. Magnetic dipole moment or Current ´ area ë û
Magnetic field
18 Physics
Some Physical Quantities Having Same Dimensions
S.No. Physical quantity Dimensions
1. Momentum and impulse [MLT–1]
2. Work, energy, momentum of force, torque [ML2T–2]
3. Pressure, stress, modulus of elasticity [ML–1 T–2]
4. Planck's constant, angular momentum [ML2T–1]
5. Surface tension, force constant [M1L0L–2]
6. Frequency, decay constant, angular velocity, [M0L0T–1]
7. Gravitational potential, latent heat [M0L2T–2]
8. Thermal capacity, entropy [ML2T–2K–1].

Applications of Dimensional Analysis

(i) To find the unit of a given physical quantity in a given system of units:
By expressing a physical quantity in terms of basic quantities we find its dimensions. In the dimensional formula replacing M, L,
T by the fundamental units of the required system, we get the unit of physical quantity. However, sometimes we assign a specific
name to this unit.
Example : Force is numerically equal to the product of mass and acceleration
i.e. Force = mass × acceleration

velocity displacement length


or [F] = mass × = mass × = mass × T–2] = [MLT
= [M] ×[LT T–2]
time (time) 2
(time) 2
Its unit in SI system will be Kgms–2 which is given a specific name "newton (N)
Similarly, its unit in CGS system will be g cms–2 which is called "dyne".
(ii) To convert a physical quantity from one system of unit to another :
This is based on the fact that for a given physical quantity, magnitude × unit = constant i.e., n 1u1= n2u2
So, when unit changes, magnitude will also change.
(iii To check the dimensional correctness of a given physical relation :
This is based on the principle that the dimensions of the terms on both sides of an equation must be same. This is known as the
'principle of homogeneity'. If the dimensions of the terms on both sides are same, the equation is dimensionally correct,
otherwise not.
It is not necessary that a dimensionally correct equation is also physically correct but a physically correct equation has to be
dimensionally correct.
(iv) Deducing relation among the physical quantities:
For example, the dependence of time period of a simple pendulum T on the quantities l, g and m as a product may be written as
T =klxgymz
where k is a dimensionless constant and x, y and z are the powers.
Considering dimensions on both sides.
[ M 0 L0T ] = [ L] x [ LT -2 ] y [ M ] z = Lx + y T-2y Mz
Equating dimensions on both sides
x + y = 0 ; –2y = 1 and z =0
1 1
\ x = , y = - , z =0
2 2
l l
Then, T = k l1/2 g -1/2 or, T =k or T = 2p (Q k = 2p)
g g

The value of k cannot be obtained by dimensional analysis method.


Units and Measurements 19

Limitations of Dimensional Analysis


As seen above the dimensional analysis technique is useful in several ways yet we cannot go too far, it has its own shortcomings or
limitations.
Following are the limitations of dimensional analysis.
(i) To derive a physical relation, we must first know on what factors the given physical quantity depends and even after that we can
derive only the preliminary relations of the product type, such as T = k l/g , v= E / r etc.
If the relation to be derived consists of two or more terms connected by a sum (+) or a difference (–) sign then we cannot derive
it. Thus the relations of the type : v = u + at cannot be derived by the method of dimensional analysis.
(ii) The value of a dimensionless constant appearing in a formula cannot be determined by this method.
(iii) This method may be fruitful only when the number of unknown constants is exactly the same as the number of equations
obtained to determine them.
(iv) By this method, we cannot establish the relations which involve trigonometric functions (sin q, cos q, tan q etc.), exponential
functions (ex ..., etc.) and the logarithmic functions (log x, etc.). The reason is that each of these functions is dimensionless and
acts as an unknown constant in the relation.

Illustration 6 :
Convert 1 joule into erg.

Sol. Dimensions of work done W = [ M1L2 T -2 ]


\ a = 1, b = 2, c = – 2
Here, M1 = 1 kg, L1 = 1 m , T1= 1 s, M2 = 1g, L 2 = 1cm , T2 = 1s, and n1 = 1

1 2 -2 1 2
æ 1kg ö æ 1m ö æ 1s ö æ 103g ö æ 10 2 cm ö æ 1s ö -2
\ n2 = 1ç ÷ ç ÷ ç ÷ = 1ç ÷ ç ÷ = 1× 103 × 104 = 107
è 1g ø è 1cm ø è 1s ø ç 1g ÷ ç 1cm ÷ çè 1s ÷ø
è ø è ø
\ 1 joule = 107 erg.

Illustration 7 :
Using dimensions, check the correctness of the following physical equations.

1 p Pr 4
(i) v = u + at (ii) s = ut + at 2 (iii) v 2 = u 2 + 2as (iv) T =
2 8 nl
Sol. (i) v = u + at where the symbols have their usual meanings.
v = final velocity \ [v] = LT -1 u = initial velocity \ [u] = LT -1

a = acceleration \ [at] = LT -2 T = LT -1
t = time
-1
\ Dimensions of LHS = LT and Dimensions of each term of RHS = LT -1
\ The equation v = u + at is dimensionally correct.

1 2
(ii) s = ut + at s = distance \ [s] = L, [ut] = LT -1 .T = L
2

é1 ù 1
Þ ê at 2 ú = LT -2 .T 2 = L [ is a number and hence dimensionless]
ë2 û 2
\ Dimensions of each term of the equation is L.
\ Eqn s = ut + ½at2 is dimensionally correct.
20 Physics

(iii) v 2 = u 2 + 2as

[v2] = (LT -1 )2 = L2T -2 [u2] = (LT -1 )2 = L2T -2 [2as] = LT -2 .L = L2 T -2

\ Dimensions of LHS = Dimensions of RHS


\ Eqn v2 = u2 + 2as is dimensionally correct.

p pr4
(iv) V = , V = volume of liquid coming out through a pipe in unit time
8 hl

L3
[V] = = L3 T -1 = Dimensions of LHS
T
P = pressure of the pipe \ [P] = ML–1 T–2
r = radius of the pipe \ [r] = L
h = coefficient of viscosity \ [ h ] = ML-1T -1
l = length of the pipe \ [l] = L

ML-1T -2 ´ L4
\ Dimensions of RHS = -1 -1
= L3T -1
ML T ´L

ppr 4
\ Eqn V = is dimensionally correct.
8nl

Illustration 8 :
The frequency of vibration ( n ) of a string depends upon length (l) of the string, tension (T) in the string and mass per unit
length (m) of the string. Using the method of dimension, derive a formula for the frequency of vibration of the string.

Sol. Let v µ l a T b m c \ ν = k l a Tb mc [k = constant of proportionality and hence is dimensionless].


Putting the dimension of the quantities on both sides of the equation,

M0 L0T –1 = La (MLT -2 )b (ML-1 ) c = M b + c La + b -c T -2b


Applying the principle of homogeneity of dimension,
b+c=0 Þ c=–b
1
a + b – c = 0 and – 2b = – 1 Þ b =
2
1 1 æ 1ö
\ c=– and a + - ç - ÷ = 0 Þ a = – 1
2 2 è 2ø

k T
\ n = kl -1T 1/ 2 m -1/ 2 or n =
l m
Units and Measurements 21
22 Physics

Textbook Exercises
2.1 Fill in the blanks : Sol. 1 calorie = 4.2J = 4.2 kg m2 s–2
(a) The volume of a cube of side 1cm is equal to _____ If a kg = new unit of mass
m3.
(b) The surface area of a solid cylinder of radius 2.0 cm 1
Then, 1kg = new unit of mass
and height 10.0 cm is equal to _____ (mm)2. a
(c) A vehicle moving with a speed of 18 km h–1 cover = a-1 new unit of mass
_____ m in 1s. Similarly, 1m = b-1 new unit of length
(d) The relative density of lead is 11.3. Its density is
1s = g-1 new unit of time
____ g cm–3 or _____ kg m–3 .
Sol. (a) Length, L = 1cm = 10–2 m, Now, 1 calorie = 4.2 (a-1 new unit of mass)
\ Volume, L3 = (10–2)3 = 10–6 m3 (b-1 new unit of length)2
(b) Given, r = 2 cm = 20 mm, h = 10 cm = 100 mm (g-1 new unit of time)–2
Surface area of solid cylinder = (2pr)h = 4.2 a–1 b–2g2 unit of energy.
22 2.4 Explain this statement clearly :
=2× × 20 × 100 = 1.26 × 104 mm2
7 “To call a dimensional quantity ‘large’ or ‘small’ is
meaningless without specifying a standard for
18 ´1000
(c) Speed, v = 18 kmh–1 = = 5ms–1 comparison.” In view of this, reframe the following
60 ´ 60 statements wherever necessary:
\ Distance covered in 1s = 5m (a) atoms are very small objects
(d) Relative density = 11.3,
(b) a jet plane moves with great speed
11.3 ´10 -3 kg (c) the mass of Jupiter is very large
density = 11.3 g/cm3 =
(10-2 m)3 (d) the air inside this room contains a large number of
molecules
= 11.3 × 103 kgm–3.
2.2 Fill in the blank by suitable conversion of units: (e) a proton is much more massive than an electron
(a) 1 kgm2s–2 = _____ g cm2 s–2 (f) the speed of sound is much smaller than the speed
of light.
(b) 1 m = _____ light year
Sol. Physical quantities are called large or small depending on
(c) 3 ms–2 = _____ kmh–2
the unit (standard) of measurement. For example, parsec
(d) G = 6.67 × 10–11 Nm2 kg–2 = _____ cm3 s–2g–1 is equal to 3.08 ´ 1016 m or 3.08 ´ 1012 km is certainly
Sol. (a) 1 kgm2s–2 = 1 × 103g (102 cm)2 s–2 larger than metre or kilometre.
= 107 g cm2 s –2 (a) The size of an atom is much smaller than even the
(b) 1 light year = 9.46 × 1015 m sharp tip of a pin.
1 (b) A Jet plane moves with a speed greater than that of
\1m= = 1.053 × 10–16 light year a superfast train.
9.46 ´1015
(c) The mass of Jupiter is very large compared to that
-2 of the earth.
æ 1 ö
(c) 3 ms–2 = 3 × 10–3 km ç h÷ (d) The air inside this room contains more number of
è 60 ´ 60 ø
molecules than in one mole of air.
= 3 × 10–3 × 3600 × 3600 kmh–2
(e) This is a correct statement.
= 3.888 × 10–4 kmh–2.
(d) G = 6.67 × 10–11 Nm2 kg–2 (f) This is a correct statement.
= 6.67 × 10–11 (kg m s–2) m2 kg–2 2.5 A new unit of length is chosen such that the speed of light
= 6.67 × 10–11 m3 s–2 kg–1 in vacuum is unity. What is the distance between the Sun
and the Earth in terms of the new unit, if light takes 8
= 6.67 × 10–11 × 106 cm3 s–210–3 g–1
min and 20 sec to cover this distance?
= 6.67 × 10–8 cm3 s–2 g–1
2.3 A calorie is a unit of heat or energy and it equals about Sol. Velocity of light c = 1 new unit of length s–1
4.2J, where 1J = 1 kgm2s–2. Suppose, we employ a Time taken by light of Sun to reach the Earth
system of units in which the unit of mass equals a kg, = t = 8 min 20 s = 8 × 60 + 20 = 500 s
the unit of length equals bm and the unit of time is gs. \ Distance between the Sun and Earth,
Show that a calorie has a magnitude of 4.2 a–1b –2g2 in x = c × t = 1 new unit of length s–1 × 500 s
terms of new units. = 500 new units of length.
Units and Measurements 23

2.6 Which of the following is the most precise device for (c) Large no. of observations (say 100) gives more
measuring length? reliable result, because probability of making random
(a) a Vernier callipers with 20 divisions on the sliding error in positive side of a physical quantity would be
scale coinciding with 19 main scale division same that of in negative side. Therefore, when no.
(b) a screw gauge of pitch 1 mm and 100 divisions on of observations is large random errors would cancel
the circular scale each other and hence result would be reliable.
(c) an optical instrument that can measure length within 2.9 The photograph of a house occupies an area of 1.75 cm2
a wavelength of light ? on a 35 mm slide. The slide is projected on to a screen,
Sol. (a) Least count of Vernier callipers = 1 SD – 1 VD and the area of the house on the screen is 1.55 m2. What
is the linear magnification of the projector screen
19 1 1 arrangement?
= 1 SD – SD = SD = mm = 0.005 cm
20 20 20 Sol. Area of object = 1.75 cm2
(b) Least count of screw gauge Area of image = 1.55 m2 = 1.55 × 104 cm2
pitch area of image
= \ Area magnification =
no. of division on circular scale area of object
1 1.55×104
= mm = 0.001 cm = = 8857
100 1.75
(c) Wavelength of light, l = 10–5 cm = 0.00001 cm
Since most precise device should have minimum least Linear magnification = 8857 = 94.1
count, optical instrument is the most precise one. 2.10 State the number of significant figures in the following :
2.7 A student measures the thickness of a human hair by (i) 0.007 m2 (ii) 2.64 × 1024 kg
looking at it through a microscope of magnification 100. (iii) 0.2370 g cm –3 (iv) 6.320 J
He makes 20 observations and finds that the average width (v) 6.032 Nm–2 (vi) 0.0006032 m2
of the hair in the field of view of the microscope is 3.5 Sol. No. of significant figures :
mm. What is his estimate on the thickness of hair? (i) one (ii) three
observed width ( y ) (iii) four (iv) four
Sol. Magnification m =
real width ( x ) (v) four (vi) four
y 3.5 2.11 The length breadth and thickness of a metal sheet are
\ x= = = 0.035 mm. 4.234 m, 1.005 m and 2.01 cm respectively. Give the area
m 100
2.8 Answer the following : and volume of the sheet to correct number of significant
(a) You are given a thread and a metre scale. How will figure.
you estimate the diameter of the thread? Sol. Given, length, l = 4.234 m, Breadth, b = 1.005m, thickness,
(b) A screw gauge has a pitch of 1.0 mm and 200 t = 2.01 cm = 2.01 × 10–2 m
divisions on the circular scale. Do you think it is Area of sheet = 2 (lb + bt + tl) = 2
possible to increase the accuracy of the screw gauge = [4.234 × 1.005 + 1.005 × 0.0201 + 0.0201 × 4.234]
arbitrarily by increasing the number of divisions on = 8.7209478 m2 » 8.72 m2
the circular scale? (rounding off to 3 significant digits)
(c) The mean diameter of a thin brass rod is to be Volume = lbt = 4.234 × 1.005 × 0.0201
measured by Vernier callipers. Why is a set of 100
= 0.0855289 » 0.0855 m3
measurements of the diameter expected to yield a
more reliable estimate than a set of 5 measurements (rounding off to 3 significant digits)
only? 2.12 The mass of a box measured by a grocer's balance is
Sol. (a) Meter scale can not measure small diameter of thread. 2.30 kg. Two gold pieces of masses 20.15 g and 20.17 g
No. of turns of the thread to be wound to get turns are added to the box. What is
closely one another. (a) the total mass of the box
Let l– measured length of windings on the scale (b) the difference in masses of the gold pieces to correct
which contains n no. of turns. significant figures.
l Sol. Mass of the box, m = 2.30 kg
\ Diameter of thread = .
n Mass of one gold piece, m1 = 20.17 g = 0.02017 kg
pitch Mass of second gold piece, m2 = 20.15 g = 0.02015 kg
(b) Least count =
no. of division in circular scale (a) The total mass of the box
i.e. least count decreases when no. of division on the = 2.3 + 0.0215 + 0.0217 kg = 2.3432
circular scale increases. Thereby accuracy would Since the result is correct only upto one place of
increase. but practically, it is impossible to take decimal, therefore after rounding off, total mass
precise reading due to low resolution of human eye. = 2.3 kg
24 Physics
(b) Difference in masses of the gold pieces
é a tù
and êçæ ö÷ sin ú = éê sin ùú
L T
= m2 – m1 = 20.17 – 20.15 = 0.02 g
(correct upto 2 places of decimal) ëè T ø a û ëT Lû
2.13 A physical quantity P is related to four observatives a,
= é LT -1 sin TL-1 ù
3 2 ë û
a b
b, c and d as follows : P = So, the equation is wrong.
( c d)

( ) 2 πt ö
The percentage errors of measurement in a, b, c and d æ 2πt
y = a 2 ç sin + cos
T ÷ø
are 1%, 3%, 4% and 2% respectively. What is the (d)
è T
percentage error in the quantity P? If the value of P
calculated using the above relation turns out to be 3.763, Here, [y] = [L], éë a 2 ùû = [ L ]
to what value should you round off the result?
a3b2 é 2πt 2πt ù é T Tù
and êsin + cos ú = êsin + cos ú
Sol. P = Maximum error in P, ë T T û ë T Tû
d c
Q [LHS] = [RHS]
DP Da Db Dd 1 Dc = dimensionless
=3 +2 + +
P a b d 2 c So, the equation is correct.
2 13 2.15 A famous relation in physics relates ‘moving mass’ m to
æ 1 ö æ 3 ö 1æ 4 ö
=3 ç ÷ +2 ç ÷ + 100 + ç ÷ = 100 = 0.13 the ‘rest mass’ m0 of a particle in terms of its speed v
è 100 ø è 100 ø 2 è 100 ø and the speed of light ‘c’. (This relation first arose as
a consequence of special relativity due to Albert Einstein).
DP
Percentage error in P = × 100 = 0.13 × 100 = 13% A boy recalls the relation almost correctly but forgets
P where to put the constant ‘c’. He writes:
Q 13% has two significant figures, m0
\ if P » 3.763, P would be rounded off to two significant m=
( )
1/ 2
figures, hence P = 3.8. 1 - v2
2.14 A book with many printing errors contains four different Guess where to put the missing c.
formulas for the displacement y of a particle undergoing
m0
a certain periodic motion: Sol. From the given equation, = 1 - v2
m
2 pt Left hand side is dimensionless.
(a) y = a sin (b) y = a sin vt
T Therefore, right hand side should also be dimensionless.

æaö 2 pt 2 pt ö v2
(c) y = ç ÷ sin
èT ø
t
a
( æ
(d) y = a 2 çè sin) T
+ cos
T ø
÷ It is possible only when 1- v 2 should be 1-
c2
(a = maximum displacement of the particle, -1/ 2
æ v2 ö
v = speed of the particle, T = time-period of motion). Thus, the correct formula is m = m0 çç 1 - 2 ÷÷
Rule out the wrong formulas on dimensional grounds. è c ø
Sol. According to dimensional analysis an equation must be 2.16 The unit of length convenient on the atomic scale is
dimensionally homogeneous. known as an angstrom and is denoted by Å : 1Å = 10–
10 m. The size of a hydrogen atom is about 0.5 Å. What
2 pt is the total atomic volume in m3 of a mole of hydrogen
(a) y = a sin
T atoms?
Here, [L.H.S.] = [y] = [L] Sol. Given, r = 0.5 Å = 0.5 × 10–10 m

é 2p t ù é Tù 4 3
and [R.H.S.] = ê a sin
T ú = ê Lsin T ú = [ L] Volume of each atom of hydrogen =
3
pr
ë û ë û
So, it is correct. 4
= × 3.14 (0.5 × 10–10)3 = 5.236 × 10–31 m3
(b) y = a sin vt Here, [y] = [L] 3
and [a sin vt] = [L sin (LT–1.T)] = [L sin L] No. of atoms in 1 mole of H2 = 6.023 × 1023
So, the equation is wrong. (Avogadro Number)
\ Atomic volume = 5.236 × 10 × 6.023 × 1023
–31
æaö t = 3.154 × 10–7 m3.
(c) y = ç ÷ sin Here, [y] = [L]
T
è ø a
Units and Measurements 25

2.17 One mole of an ideal gas at standard temperature and Sol. Length of base line ‘b’ = distance between Earth and Sun
pressure occupies 22.4 L (molar volume). What is the = 1 A.U. = 1.5 × 1011 m
ratio of molar volume to the atomic volume of a mole of
hydrogen? (Take the size of hydrogen molecule to be 1 1°
about 1 Å). Why is this ratio so large? Parallax angle (q) = 1" = =
60 60 ´ 60
Sol. Volume of one mole of ideal gas,
Vg = 22.4 litre = 22.4 ´ 10–3m3 p 1
= ´ radian
1Å 180 60 ´ 60
Radius of hydrogen molecule =
2 S
= 0.5 Å = 0.5 ´ 10–10 m arc b
Now, q = =
4 3 radius D q
Volume of hydrogen molecule = pr
3
l 1.5 ´1011 D D
\r=
( )
4 22 -10 3 =
= ´ 0.5 ´ 10 m3 q p 1
3 7 ´
180 60 ´ 60
= 0.5238 ´ 10–30 m3
One mole contains 6.023 ´ 1023 molecules. = 3.1 × 1016 m A b B
\ Volume of one mole of hydrogen, \ 1 parsec = 3.1 × 1016 m. Parallax method
VH = 0.5238 ´ 10–30 ´ 6.023 ´ 1023 m3 2.20 The nearest star to our solar system is 4.29 light years
= 3.1548 ´ 10–7 m3 away. How much is this distance in terms of parsecs ?
How much parallax would this star (named alpha centaur)
Vg 22.4 ´10-3 show when viewed from two locations of the Earth six
Now, = = 7.1´104
VH 3.1548 ´10-7 months apart in its orbit around the sun?
The ratio is very large. This is because the interatomic Sol. x = 4.29 l y = 4.29 × 9.46 × 1015 m
separation in the gas is very large compared to the size
of a hydrogen molecule. 4.29 ´ 9.46 ´ 1015
= parsec = 1.323 parsec
2.18 Explain this common observation clearly: If you look out 3.08 ´1016
of the window of a fast moving train, the nearby trees,
houses etc., seem to move rapidly in a direction opposite In an orbit around the sun, the distance between six
to the train's motion, but the distant objects (hill tops, the months apart locations is diameter of the Earths orbit itself
Moon, the stars etc.) seem to be stationary. (In fact, since which is 2AU.
you are aware that you are moving, these distant objects
seem to move with you). l 2AU 2 ´ 1.496 ´ 1011
Sol. The line joining a given object to our eye is known as the \ q = = = rad
r x 4.29 ´ 9.46 ´ 1015
line of sight. When a train moves rapidly, the line of sight
of a passenger sitting in the train for nearby trees' changes = 1.512 sec
its direction rapidly. As a result, the nearby trees and other 2.21 Precise measurements of physical quantities are a need
objects appear to run in a direction opposite to the train's of science. For example, to ascertain the speed of an
motion. However, the line of sight of distant and large size aircraft, one must have an accurate method to find its
objects e.g., hill tops, the Moon, the stars etc., almost positions at closely separated instants of time. This was
remains unchanged (or changes by an extremely small
the actual motivation behind the discovery of radar in
angle). As a result, the distant object seems to be
World War II. Think of different examples in modern
stationary.
2.19 The principle of parallax is used in the determination of science where precise measurements of length, time,
distances of very distant stars. The baseline AB is the mass etc., are needed. Also, wherever you can, give a
line joining the Earth's two lacations six months apart quantitative idea of the precision needed.
in its orbit around the Sun. That is, the baseline is about Sol. Extremely precise measurements are needed in modern
the diameter of the Earth's orbit » 3 × 1011 m. However, science. As an example, while launching a satellite using
even the nearest stars are so distant that with such a long a space launch rocket system we must measure time to
baseline, they show parallax only of the order of 1" a precision of 1 micro second. Again working with lasers
(second) of arc or so. A parsec is a convenient unit of we require length measurements to an angstrom unit
length on the astronomical scale. It is the distance of an (1 Å = 10–10 m) or even a fraction of it. For estimating
object that will show a parallax of 1" (second) of arc from nuclear sizes we require a precision of 10 –15 m. To
opposite ends of a baseline equal to the distance from the
measure atomic masses using mass spectrograph we
Earth to the Sun. How much is a parsec in terms of
require a precision of 10–30 kg and so on.
metres?
26 Physics
2.22 Just as precise measurements are necessary in science, Number of hair on the head
it is equally important to be able to make rough estimates
of quantities using rudimentary ideas and common area of the head
=
observations. Think of ways by which you can estimate area of cross-section of a hair
the following (where an estimate is difficult to obtain, try Taking head to be a circle of radius 8 cm, number of
to get an upper bound on the quantity): hair on head
(a) the total mass of rain-bearing clouds over India
during the Monsoon p ( 0.08 )2 64 ´ 10 -4
= = = 2.56 ´106
(b) the mass of an elephant
( ) 25 ´ 10-10
2
p 5 ´ 10-5
(c) the wind speed during a storm
(d) the number of strands of hair on your head The number of hair on the human head is of the order
(e) the number of air molecules in your classroom . of one million.
Sol. (a) The average rainfall of nearly 100 cm or 1 m is (e) Consider a class room of size 10 m ´ 8 m ´ 4 m.
recorded by meteorologists, during Monsoon, in Volume of this room is 320m3. We know that 22.4L
India. If A is the area of the country, then or 22.4 ´ 10–3 m3 of air has 6.02 ´ 1023 molecules
A = 3.3 million sq. km = 3.3 ´ 106 (km)2 (equal to' Avogadro's number).
= 3.3 ´ 106 ´ 106 m2 = 3.3 ´ 1012 m2 \ Number of molecules of air in the class room
Mass of rain – bearing clouds
6.02 ´1023
= Volume of rain water ´ density of water = ´ 320 = 8.6 ´ 1027
-3
= 3.3 ´ 1012m2 ´ 1m ´ 1000 kg/m3 22.4 ´ 10
= 3.3 ´ 1015 kg. 2.23 The sun is a hot plasma (ionised matter) with its inner
(b) Measure the depth of an empty boat in water. Let it core at a temperature exceeding 107 K, and its outer
be d1. If A be the base area the boat, then volume surface at a temperature of about 6000 K. At such high
of water displaced by boat, V1 = Ad1 . temperatures, no substance remains in a solid or liquid
Let d2 be the depth of boat in water when the phase. In what range do you expect the mass density of
elephant is moved into the boat. Volume of water the Sun to be, in the range of densities of solids, liquids
displaced by (boat + elephant), V2 = Ad2 or gases? Check if your guess is correct from the
following data : mass of Sun = 2 × 1030 kg, radius of the
Volume of water displaced by elephant,
Sun = 7 × 108 m.
V = V2 – V1 = A(d2 – d1)
Sol. M = 2 × 1030 kg, R = 7 × 108 m
If r be the density of water, then mass of elephant
= mass of water displaced by it = A(d2 – d1) r. mass M 3M
Density, r = = =
(c) Wind speed during storms can be estimated by volume 4 3 4pR 3
pR
floating a gas-filled balloon in air at a known height 3
h. When there is no wind, the balloon is at A.
Suppose the wind starts blowing to the right such 3 ´ 2 ´ 1030
that the balloon reaches position B in 1 second. = = 1.392 × 103 kg/m3.
4 ´ 3.14 ´ (7 ´ 108 )3
Now, AB = d = hq.
which is the order of density of solids and liquids not
gases.
A B
d High density of Sun is because of inward gravitational
attractions or the outer layers of the Sun.
2.24 When the planet Jupiter is at a distance of 824.7 million
km from Earth, its angular diameter is measured to be
35.72" of arc. Calculate the diameter of Jupiter?
h Sol. r = 824.7 × 106 km
wind
q 35.72 p
q = 35.72" = × radian
60 ´ 60 180

35.72 p
The value of d directly gives the wind speed. Q l = r q = 824.7 × 106 × × km
60 ´ 60 180
(d) Let us assume that the hair on our head are uniformly
distributed. Thickness of a human hair is 5 ´ 10–5 m. = 1.429 × 105 km.
Units and Measurements 27

2.25 A man walking briskly in rain with speed v must slant 2.28 The unit of length convenien on the nuclear scale is a
his umbrella forward making an angle q with the vertical. fermi : 1 f = 10–15 m. Nuclear sizes obey roughly the
A student derives the following relation between q and 1
v : tan q = v and checks that the relation has a correct following empirical relation, r = ro A3
, where r is the
limit : as v ® 0, q ® 0, as expected. (We are assuming
radius of the nucleus, A its mass number, and r0 is a
there is no strong wind and that the rain falls vertically
for a stationary man). Do you think this relation can be constant equal to 1.2 f. Show that the rule implies that
correct? If not, guess the correct relation. nuclear mass density is nearly constant for different
Sol. According to principle of homogenity of dimensional nuclei. Estimate the mass density of sodium nucleus.
equations, Compare it with average mass density of sodium atom.
Dimensions of L.H.S. = Dimensions of R.H.S. Sol. Let m = average mass of proton or neucleon
Here, v = tan q \ Nuclear mass, M = mA
i.e. L1T–1 = Dimensionless, which is incorrect.
Correcting the L.H.S., we get and radius of nucleus, r = r0 A1/ 3
v mass of nucleus
= tan q, where u is velocity of rain. Nuclear density, r =
u volume of nucleus
2.26 It is claimed that two cesium clocks, if allowed to run for
100 years, free from any disturbance, may differ by only M mA 3m
= = =
about 0.025 s. What does this imply for the accuracy of 4 3 4 4π r03
pr π( r0 A1 3 )3
the standard cesium clock in measuring a time interval 3 3
of 1 s?
Sol. Error in 100 years = 0.02 s as r0 and m are constant, hence nuclear density would
\ Error in 1 sec be constant for all nuclei.
Now, m = 1.60 × 10–27 kg
Dt 0.02 0.02
= = and r0 = 1.2 f = 1.2 × 10–15 m
t 100 ´ 365 ´ 2.5 ´ 24 ´ 60 ´ 60 3.5576 ´ 10 9
= 0.0063 × 10–9 = 0.63 × 10–11 3m 3 ´ 1.66 ´ 10-27
So, there is an accuracy of 1s is 10–11s. \ r= =
4 pr03 4 ´ 3.14 ´ (1.2 ´ 10 -15 )3
2.27 Estimate the average atomic mass density of a sodium
atom, assuming its size to be 2.5Å. Compare it with the = 2.29 × 1017 kg m–3.
density of sodium in its crystalline phase (970 kgm–3). From last question, density of sodium atom
Are the two densities of the same order of magnitude?
= 0.58 × 103 kg/m3
If so, Why?
Sol. Radius of Na atom = 2.5 Å Nuclear mass density 2.3 ´ 1017
\ = 14
4 3 Atomic mass density 0.58 ´ 103 = 3.96 ´ 10
Atomic volume = pR
3
\ Nuclear density is around 10 15 times the atomic
4 22 density of matter.
= × (2.25 × 10–10)3 = 65.25 × 10–30 m3
3 7 2.29 A LASER is a source of very intense, monochromatic and
Atomic mass unidirectional beam of light. These properties of a laser
Q
Avogadro 's No. light can be exploited to measure long distances. The
= Mass of one sodium atom distance of the Moon from the Earth has been already
determined very precisely using a laser as a source of
23
= g = 3.82 × 10–26 kg light. A laser light beamed at the Moon takes 2.56s to
6.023 ´1023 return after reflection at the Moon's surface. How much
mass is the radius of the lunar orbit around the earth?
Average mass density = Sol. Here t = 256s, c = 3 ´ 108 ms–1
Volume
Radius of the lunar orbit around the earth
3.82 ´ 10 –26 = Distance of the moon from the earth
= = 0.58 × 103 kg/m3
65.25 ´ 10 –30
c ´ t 3 ´ 108 ´ 2.56
The two densities are of same order (~103), which is due = = = 3.84 ´ 108 m.
to close packing of atoms in the crystalline phase. 2 2
28 Physics
2.30 A SONAR (Sound Navigation and Ranging) uses the dimension of time. Further, it was a very large
ultrasonic waves to detect and locate objects under water. number, its magnitude being close to the present estimate
In a submarine equipped with a SONAR, the time delay on the age of the universe ( ~15 /billion years). From the
between generation of a probe wave and the reception of table of fundamental constants in this book, try to see if
its echo after reflection from an enemy submarine is you too can construct this number (or any other
found to be 77s. What is the distance of the enemy interesting number you can think of ). If its coincidence
submarine ? (Speed of sound in water = 1450 ms–1 ). with the age of the universe were significant, what would
Sol. Here t = 77s, V = 1450 ms–1 this imply for the constancy of fundamental constants?
Distance of enemy submarine Sol. Using basic constants such as speed of light (c), charge
on electron (e), mass of electron (me), mass of proton (mv)
V ´ t 1450 ´ 77 and gravitational constant (G), we can construct the
= = = 55825m.
2 2 quantity,
2.31 The farthest objects (known as quasars) in our universe
2
are so distant that light emitted by them takes billion of æ e2 ö 1
years to reach the earth. What is the distance in km of t=ç ÷ ´
ç 4πe 0 ÷ m m 2c 3G
a quasar from which light takes 3.0 billion years to reach è ø p e
us ?
Sol. Here t = 3.0 billion years é e2 ù é 1 e2 2 ù
é 2ù
= 3.0 ´ 109 ´ 365.25 ´ 24 ´ 60 ´ 60 s Now ê 4πe ú = ê 4πe 2 r ú = ë Fr û
êë 0 úû êë 0 r úû
Speed of light, c = 3 ´ 105 kms–1
= [MLT–2 . L2] = [ML3T–2]
Distance of quasar
= ct = 3 ´ 105 ´ 3.0 ´ 109 ´ 365.25 ´ 24 ´ 60 ´ 60 2
é ML3T -2 ù
\ [t] = ë û
= 2.84 ´ 1022 km.
3
= [T]
2.32 It is a well known fact that during a total solar eclipse [ M ][ M ]2 ëé LT -1 ûù é M -1L3T -2 ù
ë û
the disc of the moon almost completely covers the disc
of the sun. From this fact and from the information that Clearly, the quantity t has the dimensions of time.
sun's angular distance q is measured to be 1920", Put G = 6.67 ´ 10–11 Nm2 kg–2,
determine the approximate diameter of the moon. Given c = 3 ´ 108 m/s
earth-moon distance = 3.8452 ´ 108 m. e = 1.6 ´ 10–19 C, me = 9.1 ´ 10–31 kg,
Sol. During total solar eclipse, the disc of the moon completely m = 1.67 ´ 10–27 kg
covers the disc of the sun, so the angular diameters of
1
both the sun and the moon must be equal. and = 9 ´109 Nm2C2
\ Angular diameter of the moon, 4pe 0
q = Angular diameter of the sun 2
é
( ) ùúû
2
9 -19
= 1920¢¢ = 1920 ´ 4.85 ´ 10–6 rad ê9 ´10 ´ 1.6 ´10
\t = ë
[Q 1¢¢ = 4.85 ´ 10–6 rad]
( ) ´ (3´10 ) ´ 6.67 ´10
2 3
1.67 ´10-27 ´ 9.1´10-31 8 -11
Earth-moon distance, s = 3.8452 ´ 108 m
Diameter of the moon, D = q ´ s = 2.13 ´ 1016 s
= 1920 ´ 4.85 ´ 10–6 ´ 3.8452 ´ 108
2.13 ´ 1016
= 3.581 ´ 106 m = 3581 km. = years = 0.667 ´ 109 years.
7
2.33 A great physicist of this century (P.A.M. Dirac) loved 3.156 ´ 10
playing with numerical values of Fundamental constants = 0.667 billion years.
of nature. This led him to an interesting observation. This time is slightly less than the age of the universe (»
Dirac found that from the basic constants of atomic 15 billion years). It implies that the values of the basic
physics (c, e, mass of electron, mass of proton) and the constants of physics should change with time because the
gravitational constant G, he could arrive at a number with age of the universe increases with time.
Units and Measurements 29

Practice Questions
Very Short Answer Questions [1 Mark Qs.] 2. Why time period of radiation from cesium -133 is chosen as
the standard of time is the definition of 'the second'?
1. How many astronomical units make one metre?
3. The density of mercury is 13.6 gm/cc. Convert it into S.I
2. How many light years make one metre? units
3. How many electrons are there in 1 kg if the mass of one 4. Explain briefly how the distance of an aeroplane is measured
electron is 9.1 × 10–31 kg? by a RADAR.
4. How many times is a millisecond larger than a microsecond? 5. Explain briefly how the depth of ocean is measured by the
5. How many times a kilogram is larger than a microgram? SONAR method.
6. How many a.m.u. make 1 kg ? 6. Explain briefly how size of moon is measured by an
7. Name two physical quantities which have units but no astronomical telescope.
dimensions. 7. How do you measure the distance of a far off star by
8. Name some physical quantities having same dimensions. spectroscopic method?
9. Calculate the dimension of kinetic energy from the formula 8. Which of the following time measuring devices is most
precise?
1
K.E. = mv2 (a) A wall clock (b) A stop watch
2
(c) A digital watch (d) An atomic clock.
10. What is the percentage error in the surface area of a sphere,
Give reason for your answer.
when the error in measuring its radius is ± 4%?
9. During a total solar eclipse the moon almost entirely covers
11. What is the percentage error in measuring the density of a
the sphere of the sun. Write the relation between the
cube if the error in measuring its mass is ±1% and in
distances and sizes of the sun and moon.
measuring its length is ±2%?
12. Why length, mass and time are chosen as base quantities æ a ö
in mechanics?
10. If ç P + ÷ (V – b ) = RT , where the symbols have their
è V2 ø
13. Define parsec and express it in metres.
æaö
14. Name the unit used to measure size of a nucleus and express usual meanings, then find the dimension of ç ÷.
it in metre. èbø
15. How can a systematic error be eliminated? 11. State the principle of homogeneity of dimensions. Test the
dimensional homogeneity of the following equation:
16. Give approximate ratio of 1 AU and 1 light year.
17. What importance is attached to the final zeroes in a number 1 2
h = h0 + v0t + gt
without any decimal point? 2
18. What do you understand by absolute error ? 12. Rule out or accept the following formulae for kinetic energy
19. Define one mole. on the basis of dimensional arguments.
20. Give two examples of non-dimensional variables.
1 2
21. Write the names of three dimensional constants. (i) K = 3/16 mv2 (ii) K = mv + ma
2
22. Write two examples of non-dimensional constants.
23. Write the number of significant figures in each of the 1 2
(iii) K = mv
following measurements : 2
(a) 1.67´ 10–27 kg; (b) 0.270 cm
24. It is said that common balance compares masses and spring
Long Answer Questions [5 Marks Qs.]
balance measures weight. If a body is 2 kg in common 1. Explain parallax method for measuring distance of a near
balance, it is 2 kg in spring balance, does it mean that mass star.
is equal to weight? 2. Explain how the height of an inaccessible object is measured
25. If g is the acceleration due to gravity and l is wavelength, by triangulation method.
then which physical quantity does lg represent? 3. Explain how do you measure the radius of an atom by
Avogadro's method.
Short Answer Questions [2 or 3 Marks Qs.] 4. The frequency of vibration (v) of a string may depend upon
length (l) of the string, tension (T) in the string and mass
1. What is the advantage of choosing the wavelength of light per unit length (m) of the string. Use method of dimensions
as a standard of length? for establishing the formula for frequency v.
30 Physics
Numerical Questions [3 or 5 Marks Qs.] 7. A capacitor of capacitance C = (2.0 ± 0.1)mF is charged to
1. The angular diameter of the sun from earth is 16 minute. If a voltage V = (20 ± 0.5) volt. Calculate the charge Q with
the distance of earth from the sun is 1.49×1011m, then what
error limits.
will be the diameter of sun?
8. The radius of a sphere is measured to be (2.1 ± 0.5) cm.
2. The moon is observed from two diametrically opposite
Calculate its surface area with error limits.
points A and B on earth. The angle q subtended at the
9. Two resistances r1= (5.0 ± 0.2)W and r2= (10.0 ± 0.1) W are
moon by the two directions of observation is 1°54' .
connected in parallel. Find the value of equivalent resistance
Calculate the distance of moon from earth. [Given the
with limits of percentage error.
diameter of earth = 1.276 × 107m]
10. Calculate the focal length of a spherical mirror from the
3. The velocity of sound in air is 332m/s. If the units of length
following data :
is km and unit of time is hour, using dimension calculate
the value of velocity? Object distance u = (50.1 ± 0.5) cm, image distance = v
= (20 ± 0.2) cm.
4. A body has an acceleration of 10 km/h 2. Find its value in
c.g.s system. 11. A physical quantity Q is given by
5. Convert a power of one mega watt on a system whose A2 .B3 2
fundamental units are 10 kg, 1 dm and 1 minute. Q=
C +4 D1 2
6. The resistance R is the ratio of potential difference V and
The percentage error in A,B, C, D are 1%, 2%, 4%, 2%
current I. What is the % error in R if V = (100 ± 5) volt and
respectively. Find the percentage error in Q.
I = (10 ± 0.2)A ?

HOTS/Exemplar Questions
Very Short Answer Questions [1 Mark Qs.] 3. Time for 20 oscillations of a pendulum is measured as
t1 = 39.6s; t2 = 39.9s; t3 = 39.5 s. What is the precision in the
1. What will be the dimensions of (a × b) in the relation measurements? What is the accuracy of the measurement?
a - x2 [Exemplar]
P= where P is pressure, x is distance and t is time.
bx 4. Give an example of
[HOTS] (a) a physical quantity which has a unit but no
2. Why do we have different units for the same physical dimensions.
quantity? [Exemplar] (b) a physical quantity which has neither unit nor
3. Why length, mass and time are chosen as base quantities dimensions.
in mechanics? [Exemplar] (c) a constant which has a unit.
(d) a constant which has no unit. [Exemplar]
Short Answer Questions [2 or 3 Marks Qs.]
Long Answer Questions [5 Marks Qs.]
1. A function f (q) is defined as :
1. The nearest star to our solar system is 4.29 light years away.
q 2 q3 q 4 How much is this distance in terms of parsecs? How much
f (q ) = 1 - q + - + ....
2! 3! 4! parallax would this star (named Alpha Centauri) show when
viewed from two locations of the Earth six months apart in
Why is it necessary for q to be a dimensionless quantity?
its orbit around the Sun ? [HOTS]
[HOTS]
2. If velocity of light c, Planck’s constant h and gravitational
2. The liquid drop of density r, radius r and surface tension s constant G are taken as fundamental quantities then express
oscillates with time period T. Find T2 in terms of r, r and s. mass, length and time in terms of dimensions of these
[HOTS] quantities. [Exemplar]
Units and Measurements 31

CHAPTER TEST

Time : 45 min. Max. Marks : 24

Directions : (i) Attempt all questions


(ii) Questions 1 to 5 carry 1 mark each.
(iii) Questions 6, 7 and 8 carry 3 marks each.
(iv) Questions 9 and 10 carry 5 marks each.

a 2b3
1. A physical quantity A is related to four observables a, b, c and d as follows A = . The percentage errors of measurement
c d
in a, b, c and d are 1%, 3%, 2% and 2% respectively. What is the percentage error in A?
2. Calculate the number of astronomical units in one metre.
3. Name the quantities represented by the dimensional formula [M1L2T–1], [M1L2T–2].
4. Write the number of significant figures for the following: 1.0076, 1000, 0.00900, 2007.
5. If y = x5 then the relative error in y will be how many times the relative error in x?
6. Convert 1 newton into 1 dyne dimensionally.
1 mgl
7. Test if the following equation is dimensionally correct. n = where n = frequency, I = moment of inertia
2p I
8. Check the correctness of the following relations using dimension :

2GM
(i) V= Here, V is escape velocity, M = mass and R = radius of the earth, a = universal gravitational constant.
R3
2Scos θ
(ii) h= Here, h is height, S is surface tension, q is angle of contact, r is radius, d is density and g is acceleration
r dg
due to gravity
(iii) F = 6 πηrv Here, F is Viscous force acting on a body of radius r moving with a velocity v in a medium of coefficient
of viscosity h
9. The diameter of a wire as measured by a screw gauge was found to be 1.328, 1.330, 1.325, 1.326, 1.334 and 1.336 cm. Calculate
(i) Mean value of diameter
(ii) Absolute error in each measurement,
(iii) Mean absolute error,
(iv) Fractional error and
(v) Percentage error
10. Experiments show that frequency (n) of a tuning fork depends on length (l) of the Prong, density (d) and Young’s modulus
(Y) of its material. On the basis of dimensional analysis, derive an expression for the frequency of tuning fork.
32 Physics

Solutions
PRACTICE QUESTIONS If we take arithmetic mean a as the true value, then the
absolute errors in the individual measured values will be
Very Short Answer Questions D a1 = a - a1 Da2 = a - a2
1. 1 m = 6.67× 10–12 AU . D a3 = a - a 3 Dan = a - an
2. 1m = 1.057 × 10–16 ly. 19. One mole is that amount of a substance which contains as
3. No. of electrons in 1 kg = 1.1 × 1030 many elementary entities as there are atoms in 12 g of carbon-
4. 1 millisecond = 10–3s and 1 microsecond = 10–6s 12 isotope. These entities may be atoms, molecules, ions,
\ 1 millisecond is 103 times larger than a microsecond. etc.
5. 1 kg = 103g and 1 mg = 10–6g 20. Specific gravity and strain.
\ 1 kg is 109 times larger than a microgram 21. Gravitational constant, Planck’s constant and Boltzman’s
6. 1amu = 1.66 × 10–27 kg constant.
22. p, e and all trigonometric functions.
\ 1 kg = 1 23. (a) Three : 1,6,7
amu = 0.6 × 1027 amu
1.66 ´ 10-27 (b) Three : 2,7,0
7. Angle and Solid angle. 24. No, mass and weight are two different physical quantities.
8. Plank's constant, angular momentum have same dimension If mass is 2 kg in common balance, it is 2 kilogram force (kg
although they are totally different physical quantities. f) in spring balance.
Frequency and velocity gradient are also having same 25. l g = L.LT 2 = LT -1
dimension.
9. [K.E.] = M[LT–1]2 = ML2T–2. \ l g represent velocity..

10. A = 4πr 2 Short Answer Questions

DA Dr 1. (a) Wavelength is not affected by any external condition


\ =2 = 2 ´ (±4%) = 8%. like temperature, pressure, place, time etc.
A r
(b) Wavelength can be accurately defined
M M Dd DM 3DL (c) It is easily reproducible.
11. Density = d = = = +
V L3 2. The time period is chosen as the standard because
d M L
(a) It is accurately defined.
Dd æ DM 3DL ö (b) It is not affected by the change of place, time and
´100% = ç + ÷ ´100% external conditions like pressure, temperature etc.
d è M L ø
(c) It is easily reproducible.
= ± (1% + 3 ´ 2%) = ± 7%
13.6 ´ 103 kg
12. Because all other quantities of mechanics can be expressed 3. Density d = 13.6 gm/cc = = 13.6 × 103kg/m3
(10 -2 )3 m3
in terms of length, mass and time through simple relations. 4. From RADAR station, radio waves are transmitted in space
13. One parsec is defined as the distance at which an arc of in all directions. When they are intercepted by the aeroplane,
length one astronomical unit subtends an angle of one they are reflected back and received by the receiver at the
second of an arc. RADAR station. The time interval (t) between the
1 parsec = 3.08 ´ 1016 m. transmission and reception of the wave is noted.
14. The unit used to measure nuclear size is fermi. \ Using the formula, Distance = speed × time; we get
1 fermi = 10–15 m. t
15. The errors which tend to occur in one direction, positive or = x = c´
2
negative, are called systematic errors. Such an error can be 5. SONAR is the abbreviation of Sound Navigation and
eliminated by detecting the source of error and the rule Ranging. Ultrasonic waves from a transmitter just under
governing this error. water level is sent in a given direction. It goes though water
1AU 1.496×1011 m and is reflected back from the ocean bed. When it reaches
16. = » 10-5 to the top of water level it is received by a receiver and the
1light year 9.46×1015m
time difference (t) between the transmitted and reflected
17. All such zeros are not significant. For example 86,400 has
wave is noted.
three significant figures.
18. The magnitude of the difference between the true value of \ The depth of ocean (x) is given by x = v ´ t where n is
the quantity measured and the individual measured value 2
is called absolute error. the speed of ultra sonic wave through water.
Units and Measurements 33

6. The two diametrically opposite points P and Q on moon is


a
focussed by an astronomical telescope from a point (O) on 10. Energy. i.e., dimensions of are [ML2T–2]
b
the Earth. The angle ÐPOQ by which the telescope rotates
is measured as q. 11. Dimensions for all terms should be same.
[h] = L ; [h 0] = L
P Q [v0t] = LT T2 = L
–2

é1 2ù -2 2
êë 2 gt úû = LT T = L
S
q So the equation is dimensionally correct.
12. Dimension of L.H.S. and R.H.S. are same in (i) and (iii). So
O
Earth they are accepted
Let the average distance of moon from the surface of earth
1 2
be S. Q S >> PQ (diameter of moon) (ii) K= mv + ma
2
\ PQ ~ length of circular arc of radius S = Sq
\ Knowing S and Q, PQ can be calculated. é1 2ù
and ê mv ú ¹ [ ma ] . So it is wrong.
7. Stars which are at a distance more than 100 light years from ë2 û
the earth are chosen in this method. The intensity of light
Long Answer Questions
(I1) coming from the far off star is compared with the intensity
of (I2) of a near by bright star on a photographic plate. If r1 1. The parallax method is used to find the distance of a star
which is at a distance of less than 100 light years away from
and r2 be the distances of the far off star and the nearby the earth.
I1 r22 PQ is the diameter of the earth's orbit around the sun (O)
star respectively from the earth then we can write, = and is chosen as the base line. N is a nearly star which is at
I 2 r12
a distance d (= PN) from the earth which has to be measured.
as intensity is inversly proportional to the square of the F is a far off star taken as the reference star whose direction
distance. practically remains same at all position of the earth in its
orbital motion.
I2
\ r1 = r2
I1 N
\ Knowing r2 and measuring I1 and I2 , r1 can be F F
calculated. q1 q2
8. An atomic clock is the most precise time measuring device
because atomic oscillations are repeated with a precision
of 1s in 1013 s.
q1 q2
9. The distances of Sun and Moon are related as
Rs R P Q
Þ = es (In similar DSsOE and DMOME) O
Rm Rem
arc
Q q=
radius F or the position of the earth PV
θ= ( πRs2 / Res 2 )( πR m
2
/ Rem 2
) ÐFPN = q1 is measured with an astronomical telescope
S by rotating it through q1 while focussing the far off star
and then the nearly star N.
M Ð FPN = Ð PNO = q1
Rs Rm After 6 months, the earth moves to the point Q and from
Os Lm q E there ÐFQN = q2 is measured by the telescope.

Ð FQN = Ð QNO = q2 ,
on Rem
Mo
Ð PNQ = q1 + q 2 which is the angle subtended by N on
Res
Sun the orbital diameter of earth.
34 Physics

Arc PQ Using Avogadro's hypothesis,


Q Angle = \ q1 + q2 =
radius PN
4 3 Nm 2 2m
pr ´ = V =
PQ 3 M 3 3d
\ PN = 1
q1 + q 2 M æ M ö3
r3 = Þ r =ç ÷
2π Nd è 2πNd ø
\ PQ
d= 4. Let v = Kla Tb mc ........... (i)
θ1 + θ 2
K is dimensionless constant a, b, c are the powers of l, T
PQ = 2PO = 2AU = 2 × 1.5 × 1011 m = 3 × 1011 m and m respectively,
Dimensions of
3 ´ 1011
\ d= m l = [L], v = [T–1] , T = [M1L1T–2], m = [M1L–1]
q1 + q2
Putting values in equation (i)
2. Let PQ be an inaccessible object of height h which has to [M0L0T–1] = [L]a [MLT–2]b [ML–1]c
be measured. = LaMbLbT–2b McL–c
Angle q1 and q2 are measured from point S and R with the = Mb+c La + b – c T–2c
help of a sextant. Applying principle of homogeneity of dimensions we get,
QS b+c=0
In DPQS , cotq1 =
PQ a+b–c=0
–2b = –1, b = 1/2.
QR P Solving we get, c = -1/ 2, a = -1
In DPQR, cot q2 =
PQ
h Putting these values of a, b, c in equation. (i)
\ cot q1 - cot q2 v = Kl–1 T1/2 m–1/2
q1 q2
S Q
x R K T
QS QR QS – QR x \ Frequency, v =
= - = = l m
PQ PQ PQ h
Numerical Questions
x
\ h= 16 p
cos q1 - cos q 2 1. Here, q = 16 min = ´ radian,
60 180
Knowing x and measuring q1 and q2, h can be calculated. r = 1.49× 1011m, use l = rq = 6.93 × 108 m
3. According to Avogadro's hypothesis, volume of all atoms 2. Given,: q = 1°54' = 3.32 × 10–2 rad,
l
2 l = 1.276 × 107 m, use l = rq, r = = 3.84 ×108 m
in any amount of mass of a substance is rd of the volume q
3 3. Here, n1 = 332, n2 = ?, M1 = 1 kg, M2 = 1 kg, L1 = 1m, L2 =
1 km, T1 = 1sec, T2 = 1 hour
of that mass of the substance.
a b c
Let m = mass of substance; V = volume occupied by the éM ù éL ù éT ù
use n2 = n1 ê 1 ú ê 1 ú ê 1 ú , a = 0, b = 1, c = –1
substance; d = density of the substance ë M 2 û ë L 2 û ë T2 û

m Þ V= m Solving we get, velocity 1195 km/h


\ d =
V d 4. Ans : 0.0772 cm/s2
5. Ans : 2.16 × 1012 in new units
Let M = atomic weight of the substance;
N = Avogadro's number. DR æ DV DI ö
6. Using, ´ 100 = ± ç + ÷ ´ 100
R è V I ø
Nm
\ Number of atoms in mass m of the substance = æ 5 0.2 ö
M =ç + ´ 100 = 7%
è 100 10 ÷ø
Nm 4 Nm
Volume of no. of atoms = pr 3 ´ 7. Ans : Q = (40 ± 3) ´ 10-6 C
M 3 M
Units and Measurements 35

22 The time period of oscillation of the liquid drop is given by


8. As we know, S = 4 pr 2 = 4 ´ ´ (2.1)2 = 55.4
7 m
Ds Dr T = 2p
=2 0.5 k
Þ Þ Ds = 2 ´ 55.4 ´ = 26.4
s r 2.1 where m is inertia factor and k is spring factor.
Error = (55.4 ± 26.4) cm2
æ 4 3 ö éQfor unit length ù
9. rp = 3.3 W ± 7 % ç pr r ÷ ê ú
æ m ö 3
or T 2 = 4 p 2 ç ÷ = 4p 2 è ø
ê k=F ú
èkø s
1 1 1
= + Þ f =
uv êë and s = F úû
10. As we know, = 14.3cm
f u v v+u
Df é Du Dv Du + Dv ù r 3r
Þ = ±ê + + or T2 µ
f ë u v u + v úû s
Þ D f = 0.4 cm 3. (a) Precision is given by the least count of the instrument.
For 20 oscillations, precision = 0.1 s
Error = (14.3 ± 0.4) cm
For 1 oscillation, precision = 0.005 s
% error in Q = 2 æç ´ 100 ö÷ + æç ´ 100 ö÷
dA 3 dB
11. 39.6 + 39.9 + 39.5
è A ø 2è B ø (b) Average time t = s = 39.6s
3
æ dC ö 1 æ dD ö
+ 4ç ´ 100 ÷ + ç ´100 ÷ 39.6
è C ø 2 è D ø Period = = 1.98s
20
3 1
= 2 ´1 + ´ 2 + 4 ´ 4 + ´ 2 = 2 + 3 + 16 + 1 = 22% Max. observed error = (1.995 – 1.980)s = 0.015s.
2 2 4. (a) Angle or solid angle
(b) Relative density, etc.
HOTS/EXEMPLAR QUESTIONS
(c) Planck’s constant, universal gravitational constant, etc.
Very Short Answer Questions (d) Raynold number

1. Q x2 is subtracted form a, Long Answer Questions

\ [a] = [x2] = L2 1. Distance = 4.29 light years


= (4.29 × 365.25 × 86400) × 3 × 108m
é a - x2 ù L2
Also ê ú = [P] Þ = ML-1T -2 4.29 ´ 365.25 ´ 864 ´ 3 ´ 1010
êë bx úû [b ].L = parsec
3.08 ´ 1016
L2
Þ [b] = = M -1L2T 2 [Q1 parsec = 3.08 ´ 1016 m]
ML-1T -2 .L
= 1318656.9 × 10–6 parsec = 1.319 parsec.
\ 2 -1 2 2
[a × b] = L M L T = M L T -1 4 2
Required parallax = 2q, where q is the annual parallax.
2. Because, bodies differ in order of magnitude significantly in Since parsec is the distance corresponding to an annual
respect to the same physical quantity. For example, parallax of one second of arc, therefore q is 1.319 second of
arc.
interatomic distances are of the order of angstroms, inter-
\ Required parallax = 2 × 1.319 second of arc = 2.638 second
city distances are of the order of km, and interstellar distances
of arc.
are of the order of light year.
2. M, L, T, in terms of new units become
3. Because all other quantities of mechanics can be expressed
in terms of length, mass and time through simple relations. ch hG hG
M® ,L ® ,T ®
3
G c c5
Short Answer Questions

1. Since f (q) is a sum of different powers of q, it has to be CHAPTER TEST


dimensionless.
a 2 b3
4 1. Relation given, A =
2. Volume of liquid drop, V = pr 3 c d
3
4 3 DA æ Da Db Dc 1 Dd ö
pr r Relative error in A, =±ç2 +3 + + ÷
\ mass of liquid drop, m = A
3 è a b c 2 d ø
36 Physics
9. (i) Mean value of diameter
DA æ Da Db Dc 1 Dd ö
% error ´100 = ± ç 2 +3 + + ÷ ´100 1.328 + 1.330 + 1.325 + 1.326 + 1.334 + 1.336
A è a b c 2 d ø =
6
æ 1ö
= ± ç 2 ´ 1 + 3 ´ 3 + 2 + 2 ´ ÷ % = ±14% 7.979
è 2ø = = 1.3298 = 1.330
6
2. 1 AU = 1.5 × 1011 m
(ii) Absolute errors
1 DD1 = | 1.330 – 1.328 | = 0.002
\ 1m= AU = 6.67 ×10–12 AU
1.5 ´ 1011 DD2 = | 1.330 – 1.330 | = 0
3. [M1L2T–1] = [M1L2T–2] × [T] = J-s DD3 = | 1.330 – 1.325 | = 0.005
\ This represents Planck’s constant DD4 = | 1.330 – 1.326 | = 0.004
[M1L2T–2] represents work or energy DD5 = | 1.330 – 1.334 | = 0.004
4. No. of significant figures in 1.0076 = 5 (1, 0, 0, 7, 6) DD6 = | 1.330 – 1.336 | = 0.006
No. of significant figures in 1000 = 1 (1) (iii) Mean absolute error,
No. of significant figures in 0.009900 = 4 (0, 0, 9, 9)
No. of significant figures in 2007 = 4 (2, 0, 0, 7) 0.002 + 0 + 0.005 + 0.004 + 0.004 + 0.006
DD =
6
5. 5 times, as Dy = 5Dx
u x 0.021
= = 0.0035 = 0.004
a b c 6
æM ö æL ö æT ö
6. n2 = n1 ç 1 ÷ ç 1 ÷ ç 1 ÷
è M 2 ø è L 2 ø è T2 ø DD 0.004
(iv) Fractional error = =±
Dmean 1.330
[F] = [M1L1T -2 ]
a =1 ,b =1 c = –2 = ± 0.003
1 1 -2 é103 g ù é102 cm ù
(v) Percentage error = ± 0.003 ´ 100% = ± 0.3%
é1kg ù é 1m ù é1s ù
n1 = 1; n2 = 1 ê ú ê ú ê ú =ê úê ú
ë 1g û ë1cm û ë1s û êë 1g úû êë 1cm úû 10. Let n µ l a d b y c
= 103 × 102 = 105
n = k l ad b yc ;
\ 1N = 105 dyne
[M0L0T–1] = [L]a [ML–3]b [M1L–1T–2]c
1 mgl [M0L0T–1] = Mb+c La – 3b – c T –2c
7. n= ;
2p I Applying principle of homogeneity of dimensional
Dimension of LHS = [ n ] = [M0L0T–1] equations.
Dimension of RHS b + c = 0, a –3b – c = 0, –2c = –1
1 1
é 1 mgl ù M1/ 2 [LT -2 ]1/ 2 L1/ 2 c= b = -c = -
=ê ú= 2 2
ë 2p I û [ML2 ]1/ 2
æ 1ö 1 3 1
M1/ 2 LT -1 a = 3b + c = 3 ´ ç - ÷ + = - + = -1
RHS = = T -1 è 2ø 2 2 2
M1/ 2 L a = –1
\ [LHS] = [RHS]
It is dimensionally correct \ n = kl -1d -1/ 2 y1/ 2
8. (i) Incorrect
k y
(ii) Correct n=
l d
(iii) Correct

¿¿¿
3 Motion in a
Straight Line
C ha p t e r

MOTION
An object is said to be in a state of motion if its position changes with respect to time in a given frame of reference. An object in
motion in a reference frame can be at rest in another reference frame.
An object is said to be in a state of rest, if its position does not change with time with respect to its surroundings.
Rest and motion are relative terms.
Frame of reference : The set of coordinate axes attached to a specified body, relative to which the position of a particle is described
is known as reference frame. To express a physical quantity the reference frame from which it is observed should be specified.
If the size of an object is smaller than the distance moved by it in a fixed interval of time, the object is called point object. It is a
mathematical concept used to simplify the problem in mechanics.
Motion in one dimension or straight line or rectilinear motion: An object is said to be in one-dimensional motion if only one of its
position coordinates changes with time.
Examples: A stone is thrown up or is made to fall vertically down, a weight is hanging from a spring balance and bobbing up and
down.
PATH LENGTH OR DISTANCE AND DISPLACEMENT
Path Length or Distance
The length of the actual path between initial and final positions of a particle in a given interval of time is called distance covered
by the particle. Distance is the actual length of the path. It is the characteristic property of any path i.e. path is always associated
when we consider distance between two positions.
Characteristics of distance
(i) It is a scalar quantity.
(ii) It depends on the path.
(iii) It never reduces with time.
(iv) Distance covered by a particle is always positive and can never be negative or zero.
Displacement
The shortest distance from the initial position to the final position of the particle is called displacement.
®
Position vector of A w.r.t. O = OA Y
® A
ˆ ˆ
Þ rA = x1 i + y1 j + z1 k ˆ r
® rA
Position vector of B w.r.t. O = OB B
rB
®
Þ rB = x 2 ˆi + y 2 ˆj + z 2 kˆ X
®
ˆ ˆ ˆ
Displacement = AB = (x 2 - x1 ) i + (y 2 - y1 ) j + (z2 - z1 ) k O
r
D r = Dx ˆi + Dy ˆj + Dz kˆ
Characteristics of displacement Z
(i) It is a vector quantity.
(ii) The displacement of an object in a given time interval may be +ve, –ve or zero.
(iii) The actual distance travelled by a particle in the given interval of time is always equal to or greater than the magnitude of the
displacement and in no case, it is less than the magnitude of the displacement, i.e. Distance ³ | Displacement |
The unit of both is same metre in SI system.
38 Physics
UNIFORM AND NON-UNIFORM MOTION
When the magnitude of the displacement equals the actual distance travelled by the object. i.e velocity (dx/dt) is constant it covers
equal distance in equal interval of time and undergoes uniform motion.
When an object undergoes uniform motion, no force is required. The average and instantaneous velocity in a uniform motion are
always equal. The velocity of uniform motion is equal to the slope of position-time graph. The velocity-time graph for a uniform
motion is a straight line parallel to time-axis.
Non-uniform motion: When the velocity of the object changes with time i.e dx/dt ¹ 0, then unequal distance is travelled in equal
intervals of time or vice versa and the motion is said to be non-uniform. In non-uniform motion object has variable velocity, where
either its speed or direction of motion or both change with time. The velocity-time graph may not be straight line. e.g., when we are
travelling in a bus, which starts from rest and picks up its speed gradually with time.
AVERAGE VELOCITY AND AVERAGE SPEED
Velocity
It is defined as the rate of change of displacement.
(i) It is a vector quantity
(ii) Its direction is same as that of displacement.
Average Velocity
It is the ratio of the total displacement (s) to the total time interval (t) in which the displacement occurs.

r total displacem ent


i.e., Average velocity, vav =
total time taken
Average velocity is a vector quantity. Its direction is same as that of displacement.
r r
If at any time t1 position vector of the particle is r1 and at time t2 position is r2 then for this interval,
r r
r r2 - r1
Average velocity, vav =
t 2 - t1

Speed
It is the distance covered by the particle in one second.
Average speed
It is the ratio of total distance (d) travelled by the particle to the total time taken (t) in which this distance is travelled.
total distance travelled
i.e., Average speed, uav =
total time taken
If motion takes place in same direction then average speed and average velocity are same.
In fig., a particle goes from A to C. Distance, speed and time taken are shown.
v1 v2
S S
S1 = v1t1 Þ t1 = 1 ; S 2 = v2 t2 Þ t2 = 2 A S1 S2 C
v1 v2 B
t1 t2

S1 + S 2 v1t1 + v2 t2
vav = = ...(1)
t1 + t2 t1 + t2

vav = S1 + S 2 ...(2)
S1 S2
+
v1 v2
v1 + v2
1. If t1 = t2 = t, then vav = i.e., average speed is equal to arithmetical mean of individual speeds.
2
2v1 v2
2. If S1 = S2 = S, then vav = v + v i.e., average speed is equal to harmonic mean of individual speeds.
1 2
Motion in a Straight Line 39

INSTANTANEOUS VELOCITY AND INSTANTANEOUS SPEED


Instantaneous Velocity
It is defined as the rate of change of particle’s position with time.
r r
r lim Dr dr
Instantaneous velocity, v = Dt®0 =
Dt dt
r
where the position rr of a particle at any instant changes by D r in a small time Dt.
The magnitude of velocity is called speed i.e. speed = | velocity | i.e. v = | vr |.
In straight line motion, there is no change in direction so vr and v both have same meaning.
Instantaneous Speed
It is the speed of a particle at particular instant of time or position.

lim Ds = ds
Instantaneous speed v = Dt®0
Dt dt
(i) If during motion, velocity remains constant throughout a given interval of time, the motion is said to be uniform and for uniform
motion vr = constant = vr av.
However converse may or may not be true i.e., If vr = vr av, the motion may or may not be uniform.
(ii) If velocity is constant, speed ( = | velocity | ) will also be constant. However converse may or may not be true.
i.e. if speed = constant, velocity may or may not be constant as velocity has a direction in addition to magnitude which may or
may not change, e.g. in case of uniform rectilinear motion,
r r
v = constant and so speed | v | = constant
while in case of uniform circular motion, v = constant but | vr | ¹ constant due to change in direction.
POSITION-TIME AND VELOCITY-TIME GRAPH
ds
(i) As by definition v = , the slope of displacement versus time graph
dt

ds
gives velocity i.e., v = = tan q = slope of s-t curve.
dt
ds
(ii) As v = Þ ds = vdt
dt
From fig, vdt = dA, so dA = ds \ s = ò dA = ò vdt
Area under velocity versus time graph with proper algebraic sign
gives displacement while without sign gives distance.
ACCELERATION
Acceleration is defined as the rate of change of velocity of a body.
i.e., Acceleration, a = Dv
Dt
• It is a vector quantity. Its SI unit is ms–2
INSTANTANEOUS ACCELERATION AND AVERAGE ACCELERATION
Instantaneous Acceleration
It is defined as the acceleration of a body at particular instant.
r r
Dv dv
Instantaneous acceleration ainst = lim =
Dt®0 Dt dt
The time rate of change of velocity at any instant of time is known as instantaneous acceleration or simply acceleration.
dv
i.e., a= .
dt
Actually, acceleration at an instant is defined as the limit of the average acceleration as the time interval dt around that instant becomes
infinitesimally small.
æ Dv ö dv
Acceleration, a =lim
Dt ® 0 ç =
è Dt ÷ø dt
40 Physics
Average Acceleration
It is the ratio of total change in velocity to the total time taken in which this change in velocity takes place.
total change in velocity Dv v v + Dv
Average acceleration, aav = =
total time taken Dt t t + Dt
To find average acceleration, we need to know only the total change in velocity from initial position to final position. We need not
consider how motion takes place between these two points.
r r
If velocity of the particle at an instant t1 is v1 and at instant t2 is v2 , then the average acceleration is mathematically given by (fig.).

r r uur
r v2 - v1 Dv
Average acceleration, aav = =
t2 - t1 Dt

Illustration 1 :
On a 60 km track, a train travels the first 30 km at a uniform speed of 30 km h–1. How fast the train travels the next 30 km
so as to have an average speed of 40 km h–1 for the entire trip?
Sol. Here, average speed = 40 km h–1; Total distance travelled = 60 km
s s 30
s1 = 30 km and v1 = 30 km h–1; s2 = 30 km and v2 = ?, t1 = 1 = 30/30 = 1h and t2 = 2 =
v1 v2 v2

Total time taken = t1 + t2 = 1 + 30


v2
Average speed = total distance travelled/ total time taken
60 1 + 30
Now, 40 = or, 1.5 = or,, v2 = 30/ 0.5 = 60 km h–1
æ 30 ö v2
ç1 + ÷
è v2 ø
Illustration 2 :
Two buses started simultaneously towards each other from towns A and B, which are 480km apart. The first bus travelling
from A to B took 8 hours and the second bus travelling from B to A took ten hours to cover this distance. Determine when did
the buses meet after starting and at what distance from town A. Assume that both the buses travelled with constant speed?
Sol. Let, t = time taken by both the buses to meet from A.
480
Speed of bus from A = = 60km h–1
8
480
Speed of bus from B = = 48km h–1
10
\ Distance travelled by A = 60t and that of B = 48t
480
\ 60t + 48t = 480 or, t = = 4.4h
108
480
Distance of the meeting from A = s = 60 × t = 60 × = 266.7 km
108
KINEMATIC EQUATIONS FOR UNIFORMLY ACCELERATED MOTION
For objects in uniformly accelerated rectilinear motion, five quantities, displacement (s), time taken (t), initial velocity (u),
final velocity (v) and acceleration (a) are related by a set of simple equations called kinematic equations of motion.
1
(i) v = u + at (ii) s = ut + at2 (iii) v2 – u2 = 2as
2
Motion in a Straight Line 41

Graphical Method to Prove above Equations


Fig. below represents a velocity-time graph BC, in which AB represents the initial velocity u, CE represents the final velocity v, such
that the change in velocity is represented by CD, which takes place in time t, represented by AE.
(i) To prove v = u + at
We know,
C
CD CE - DE

Velocity (in m/s)


Acceleration = slope of the graph line BC = =
BD BD
v - u ì Q DE = AB = u ü at
or a = í ý
t î Q BD = AE = t þ B
v
or v – u = at D
or v = u + at u u
1 2
(ii) To prove s = ut + at A
2 t E
Distance travelled = area of trapezium ABCE Time (in second)
1 1 1
Þ s= (AB + CE) × AE Þ s = (u + v) t = (u + u + at) × t [ Q v = u + at]
2 2 2
1 æ at ö 1
Þ s= (2u + at) × t = ç u + ÷ ´ t or, s = ut + at2
2 è 2 ø 2
(iii) To prove v2 – u2 = 2as
From the velocity-time graph
Distance covered = area of trapezium ABCE
1
Þ s = (AB + CE) × AE
2
1
Þ s = (u + v) × t ..... (i)
2
From the velocity-time graph
change in velocity v,u
Acceleration = Þ a<
time t
v ,u
Þ t< ..... (ii)
a
Substituting the value of t in eqn. (i)
∋v ∗ u ( ∋v , u ( v 2 , u 2
s< ´ <
2 a 2a
Þ 2as = v2 – u2 or, v2 – u2 = 2as
Distance travelled by a particle in nth second
1
S n = un + a ( 2 n - 1)
2
Illustration 3 :
A 100 m sprinter increases her speed from rest uniformly at the rate of 1ms–2 upto three quarters of the total run and covers
the last quarter with uniform speed. How much time does she take to cover the first half of the second half of the run?
Sol. Assuming initial distance, s = 0 when t = 0.
Let, sprinter takes times t1 to cover first 50 m, t2 to cover first 75 m and t3 to cover the whole 100 m.
In time t1, distance travelled, s1 = 50 m; in time t2, distance travelled, s2 = 75 m and in time t3, distance travelled, s3 = 100 m, a =
1ms–2.
1
Now, s1 = ut + at 2
2 1
1
\ 50 = 0 + × 1 × t12 or, t1 = 10s
2
Again, v22 = v12 + 2as = 0 + 2 × 1 × 75 = 150, or, v2 = Ö150 = 12.25 ms–1.
42 Physics
Now, v2 = u + at = 0 + 1 × t2 or, t2 = v2 = 12.25s
Time required to cover the last quarter is,
25
t3 – t2 = = 2.04 s \ t3 = t2 + 2.04 = 12.25 + 2.04 = 14.29 s
12.25
We know, t1 = 10 s
\ Time taken to cover the first half of the second half of the run,
t3 – t1 = 14.29 – 10 = 4.29s
Illustration 4 :
A bullet moving with a velocity of 10 m/s hits the wooden plank, the bullet is stopped when it penetrates the plank 20 cm deep.
Calculate retardation of the bullet.
20
Sol. Given : v0 = 10 m/s, v = 0 and s = 20 cm = = 0.2m
100
Using v² – v0² = 2ax
-100
0 - (10) 2 = 2a (0.2) Þ =a
2 ´ 0.02
or a = – 2500 m/s² \ Retardation = 2500 m/s²
Illustration 5 :
Two trains A and B, 100 km apart, are travelling towards each other with starting speeds of 50 km/h for both. The train A is
accelerating at 18 km/h² and B is decelerating at 18 km/h². Find the distance from the initial position of A of the point when the
engines cross each other.
Sol. Such problems can be tackled by keeping in mind, that for any event, to occur, time remains same for both bodies.
Let P be the point, where the two engines cross each other. If t hour be the time to occur this event, then total distance covered
by the two trains should be equal to 100 km.(fig.)
i.e., AP + BP = 100
1 2
As we know, S = ut + at
2
1 1
50t + ´ 18t 2 + 50t - ´ 18t 2 = 100 Þ 100t = 100 Þ t = 1 hour
2 2
1
\ x = AP = 50 (1) + ´18(1) Þ x = 50 + 9 = 59 km.
2

MOTION UNDER GRAVITY (FREE FALL)


The most important example of motion in a straight line with constant acceleration is motion under gravity. In case of motion under
gravity,
(i) the acceleration is constant,
i.e., a = g = 9.8 m/s2 and directed vertically downwards.
(ii) the motion is in vacuum, so viscous force or thrust of the medium has no effect on the motion.
Body falling freely under gravity : Taking initial position as origin and downward direction of motion as positive, we have
u=0 [as body starts from rest]
a = +g [as acc. is in the direction of motion]
So if the body acquires velocity v after falling a distance h in time t, equations of motion, viz.
1
v = u + at ; s = ut + at2 and v2 = u2 + 2as
2
reduces to v = gt ....(1)
1 2
h=gt ....(2)
2
v2 = 2gh ....(3)
These equations can be used to solve most of the problems of freely falling body.
Motion in a Straight Line 43

RELATIVE VELOCITY
Relative velocity of an object A with respect to another object B, when both are in motion is the time rate at which object A changes
its position with respect to object B.
® ®
If v A and v B be the respective velocities of object A and then relative velocity of A w.r.t. B is
® ® ®
v AB = v A – vB
Similarly, relative velocity of B w.r.t. A is
® ® ®
v BA = v B – v A
Special Cases
(a) When the two objects move with equal velocities
i.e., vA = vB or vB – vA = 0
(b) When the two objects move with unequal velocities
(i) When vA > vB, then vB – vA is negative.
(ii) When vB > vA , then vB – vA is positive.
(c) When two trains A and B move with same velocity v but in opposite direction
The relative velocity of train A w.r.t. train B
® ® ®
ˆ - v(-ˆi)
v AB = vA – v B = v(i) ˆ
= 2v(i)
Relative velocity of train B w.r.t. A
® ® ®
v BA = v B – vA = v(-ˆi) - v(i)
ˆ = 2v(-ˆi)
(d) When the two bodies moving in directions inclined to each other
The magnitude of the relative velocity vAB is given by

vAB = v 2A + vB2 + 2v AvB cos (180° - q) = v 2A + vB2 - 2v AvB cos q


Let a be the angle made by vAB with vA, then
vB sin(180° - q) vB sin q
tan a = =
v A + vB cos(180° - q) v A - vB cos q

æ vB sin q ö
or a = tan–1 ç v - v cos q ÷
è A B ø
®
Ða gives the direction of the relative velocity with v A .
Relative Velocity of Rain w.r.t. the Moving Man
® ®
A man walking west with velocity vm , represented by OA .

® A vm O –vm C
Let the rain be falling vertically downwards with velocity vr , W E

®
represented by OB as shown in fig. vr vrm

® ® ®
The relative velocity of rain w.r.t. man v rm = v r - v m , will be represented by D
B
®
diagonal OD of rectangle OBDC.

\ `vrm = v 2r + v m
2
+ 2v r v m cos90º = v 2r + v 2m

® BD v æ vm ö
If q is the angle which v rm makes with the vertical direction then, tan q = = m or q = tan–1 ç v ÷
OB vr è r ø
Here angle q is from vertical towards west and is written as q, west of vertical.
44 Physics
River-Boat Problems
Minimum distance of approach C vr B
Let d = width of river, vr = velocity of river water,
vm = velocity of swimmer
® ® ®
The swimmer should swim in a direction such that resultant v of v m and v r is along vr
v d
AB which is the shortest path. vm
v d
sin q = r ; v = v 2 - v 2 ; t =
vm m r
v 2 -v 2m r
A
2
or, d = - vm - vr2 t

d
Time of crossing the river, t = v cos q
m

Illustration 6 :
A body is dropped from a height of 300m. Exactly at the same instant another body is projected from the ground level vertically
up with a velocity of 150ms–1. Find when and where will they meet? Take g = 9.8ms –2.
Sol. Let, s = distance at which two bodies meet after t sec.
First body is travelling the distance s in t time, starting with zero velocity, and
Second body is travelling the distance (300 – s) in t time, starting with the velocity, 150 ms–1.
1 2 1
Then for 1st body, s = gt and for second body (300 – s) = 150t – gt2
2 2
We get from above equations, 300 = 150t, t = 300/150 = 2 s
Putting the value of t we get,
1 2 1
s= gt = × 9.8 × (2)2 = 19.6 m
2 2
Illustration 7 :
A balloon is rising up with a velocity of 9.8 ms–1 and a bag is dropped from it when its height from the ground is 39.2 m.
Calculate the time taken by the bag to reach the ground.
Sol. Here, v = – 9.8 ms–1 at t = 0.
s = 39.2 m and g = 9.8 ms–2
1 2 1
Now, s = ut + gt or, 39.2 = – 9.8t + 9.8t2
2 2
\ t = 4s., i.e. time taken by the bag to reach the ground = 4s.
Illustration 8 :
Two railway tracks are parallel to North-South direction. Train A is moving with a speed of 40 ms–1 from North to South along
one track, while train B is moving with a speed of 30 ms–1 from South to North. Calculate (i) relative velocity of B w.r.t. A and
(ii) relative velocity of ground w.r.t. A.
Sol. Consider the direction from North to South as positive.
\ vA = + 40 ms–1 and vB = – 30 ms–1
(i) Relative velocity of B w.r.t. A
= vB – vA = – 30 – 40 = – 70 ms–1
Thus train B appears to move from South to North with speed of 70 m s –1 for an observer at A.
(ii) Velocity of ground, vg = 0
\ Relative velocity of ground w.r.t. A
= vg – vA = 0 – 40 = – 40 ms–1
Thus, the ground will appear to move from south to north with speed of 40 ms–1 w.r.t. A.
Motion in a Straight Line 45
46 Physics

Textbook Exercises
3.1 In which of the following examples of motion can the body 3.3 A woman starts from her home at 9.00am, walks with a
be considered approximately a point object: (a) a railway speed 5km/h on straight road up to her office 2.5 km away,
carriage moving without jerks between two stations (b) a
stays at the office up to 5.00pm and returns home by an
monkey sitting on the top of a man cycling smoothly on a
circular track, (c) a spinning cricket ball that turns auto with a speed of 25 km/h. Choose suitable scales and
sharply on hitting the ground, (d) a tumbling beaker that plot the x-t graph of her motion.
has slipped off the edge of a table? Sol. Time taken in reaching office = distance/ speed = 2.5/5
Sol. (a) The carriage can be considered a point object if the = 0.5 hr.
distance between the two stations is very large
Time taken in r eturn ing from office = 2.5/25
compared to the size of the railway carriage.
(b) The monkey can be considered as a point object if the = 0.1 hr. = 6 minutes
cyclist describes a circular track of very large radius It means the woman reaches the office at 9.30am and returns
because in that case the distance covered by the cyclist home at 5.06 pm. The x-t graph of this motion will be as
is quite large as compared to the size of monkey. The shown below:
monkey can not be considered as a point object if the
cyclist describes a circular track of small radius because
in that case the distance covered by the cyclist is not 3

x (in km)
very large as compared to the size of the money.
(c) The cricket ball can not be considered as a point object 2
because the size of the spinning cricket ball is quite
appreciable as compared to the distance through which
the ball may turn on hitting the ground. 1
(d) A beaker slipping off the edge of the table can not be
considered as a point object because the size of the
beaker is not negligible as compared to the height of 9.00 11.00 3.00 5.00 5.06
1.00
the table. t (in hour )
3.2 The position-time (x-t) graphs for two children A and B 3.4 A drunkard walking in a narrow lane takes 5 steps forward
returning from their school O to their homes P and Q and 3 steps backward, followed again by 5 steps forward
respectively are shown in Figure. Choose the correct and 3 steps backward, and so on. Each step is 1m long and
entries in the brackets
x below:
requires 1s. Determine how long the drunkard takes to
Q fall in a pit 13m away from the start:
Sol. For covering a distance of 8 m in forward direction the
drunkard will have to move 32 steps in all.
P Now he will have to cover 5 meters more to reach the pit,
for which he has to take only 5 forward steps.
A B Therefore, he will have to take = 32 + 5 = 37 steps to move
13 meters. Thus he will fall into the pit after taking 37 steps
i.e. after 37 seconds from the start.
O t
3.5 A jet airplane travelling at the speed of 500kmh–1 ejects
(a) (A/B) lives closer to the school than its products of combustion at the speed of 1500kmh–1
(B/A). relative to the jet plane. What is the speed of the latter
(b) (A/B) starts from the school earlier than (B/A). with respect to an observer on the ground?
(c) (A/B) walks faster than (B/A). Sol. Let, v1, vg and v0 be the velocities of jet, ejected gases i.e.
combustion products and observer on th e ground
(d) A and B reach home at the (same/different) time.
respectively.
(e) (A/B) overtakes (B/A) on the road (once/twice).
Let jet be moving towards right (+ve direction).
Sol. (a) Because OP < OQ, hence A lives closer to the school \ ejected gases will move towards left
than B. (–ve direction).
(b) When x = 0, t = 0 for A, while t ¹ 0 for B. Therefore A \ According to the statement v1 = 500 kmh –1
starts from the school earlier than B. As oberver is at ground i.e. at rest
(c) B walks faster, since the slope of B is more than the \ v0 = 0
slope of A. Now relative velocity of plane w.r.t. the observer
(d) A and B reach home at the same time. v1 – v0 = 500 – 0 = 500 kmh –1 ...(i)
(e) B overtakes A once during the journey.
Motion in a Straight Line 47

Relative velocity of the combustion products w.r.t. jet plane 3.8 On a two lane road, car A is travelling with a speed of
vg – v1 = – 1500 kmh–1 (given) ...(ii) 36kmh–1. Two cars B and C approach car A in opposite
–ve sign indicates that the combustion products move in a directions with a speed of 54kmh–1 each. At a certain
direction opposite to that of jet. instant, when the distance AB is equal to AC, both being 1
\ Adding equations (i) and (ii), we get the speed of km, B decides to overtake A before C does. What minimum
combustion products w.r.t. observer on the ground i.e. acceleration of car B is required to avoid an accident ?
(v1 – v0) + (vg – vi) = vg – v0 = 500 + (–1500) Sol. Velocity of car A = 36kmh–1 = 10ms–1;
or vg0 =vg– v0 = – 1000 kmh–1 Velocity of car B or C = 54kmh –1 = 15ms–1
–ve sign shows that relative velocity of the ejected gases Relative velocity of B w.r.t. A = 15 – 10 = 5ms–1
w.r.t. observer is towards left i.e. –ve direction i.e. in a
Relative velocity of C w.r.t. A = 15 + 10
direction opposite to the motion of the jet plane.
= 25ms–1
3.6 A car moving along a straight highway with speed 126km/
Time available to B or C for crossing
h is brought to a stop within a distance of 200m. What is
A = 1000/25 = 40s.
the retardation of the car (assumed uniform) and how long
does it take for the car to stop? If car B accelerates with acceleration a, to cross A before
car C does then, u = 5 ms –1 , t = 40s,
Sol. Here, u = 126 km/h
s = 1000m
= 126 × 1000/ (60 × 60) ms–1 = 35 ms–1
v = 0, s = 200m 1 2
Using s = ut + at we have
We know,v2 = u2 + 2as 2
Þ 0 = (35)2 + 2 a × (200)
or, a = – (35)2/ (2 × 200) 1
1000 = 5 × 40 + × a × (40)2 or, a = 1m/s–2.
= – 49/16 = – 3.06 ms–2 2
As v = u + at Þ 0 = 35 + (– 49/16) t 3.9 Two towns A and B are connected by a regular bus service
t = 35 × 16/ 49 = 80/7 = 11.43 s with a bus leaving in either direction every T min. A man
3.7 Two trains A and B of length 400m each are moving on cycling with a speed of 20kmh–1 in the direction A to B
two parallel tracks with a uniform speed of 72 kmh–1 in notices that a bus goes past him every 18 min in the
the same direction, with A ahead of B. The driver of B direction of his motion, and every 6 min in the opposite
decides to overtake A and accelerates by 1ms–2. If, after direction. What is the period T of the bus service and with
50s, the guard of B just crosses past the driver of A, what what speed (assumed constant) do the buses ply on the
was the original distance between them (the guard of B road?
and the driver of A)? Sol. Let, v kmh–1 = constant speed with which the buses ply
Sol. Originally, both the trains have the same velocities. So between the towns A and B.
the relative velocity of B w.r.t. A is zero. Relative velocity of the bus (for motion A to B) w.r.t. the
cyclist (i.e. in the direction in which the cyclist is going = (v
A
G D – 20)kmh–1
B 400m
400m Relative velocity of the bus (for motion B to A) w.r.t. the
cyclist = (v + 20) kmh–1
1 2 The distance travelled by the bus in time T (minutes) = vT
We know, s = ut + at
2
vT
For train A, u = 72kmh–1 =(72 × 1000)/(60 × 60) As per question, ( v - 20) = 18,
= 20ms–1, t = 50s, a = 0, s = sA
1 or, vT = 18v – 18 × 20 . ... (i)
\ sA = 20 × 50 + × 0 × (50)2 = 1000m
2 vT
and, = 6, or, vT = 6v + 20× 6 ... (ii)
For train B, u = 72kmh–1 =(72 × 1000)/(60 × 60) ( v + 20)
= 20ms–1, t = 50s, a = 1ms–2, s = sB
Solving (i) and (ii), we get, v = 40 kmh–1
1 Putting this value of v in (i) we get,
As, \ sB = 20×50 + × 1 × 502 = 2250m
2 40T = 18 × 40 – 18 × 20 = 18 × 20 or, T = 9 mins.
Taking the guard of the train B in the last compartment of Putting this value of T we get
the train B, it follows that original distance between then is v9 = 18v – 360
= sB – sA = 2250 – 1000 = 1250 m or v = 40 kmh–1
48 Physics
3.10 A player throws a ball upwards with an initial speed of Sol. (a) True: when a body is thrown vertically upwards in the
29.4 ms–1. (a) What is the direction of acceleration during space, then at the highest point, the body has zero
the upward motion of the ball? (b) What are the velocity speed but has downward acceleration equal to the
and acceleration of the ball at the highest point of its acceleration due to gravity.
motion. (c) Choose the x = 0, t = 0 be the location and time (b) False: because velocity is the speed of body in given
at its highest point, vertically downward direction to be direction. When speed is zero, the magnitude of
the positive direction of x-axis and give the signs of velocity of body is zero, hence velocity is zero.
(c) True: when a particle is moving along a straight line
position, velocity and acceleration of the ball during its
with speed, its velocity remains constant with time.
upwards, and downward motion. (d) To what height does
Therefore, acceleration (= change in velocity/time) is
the ball rise and after how long does the ball returns to the
zero.
player’s hands. (g = 9.8ms–2 and air resistance is (d) False: True only when the chosen position direction
negligible). is along the direction of motion.
Sol. (a) Since the ball is moving under the effect of gravity, 3.12 A ball is dropped from a height of 90m on a floor. At each
the direction of acceleration due to gravity is always collision with the floor, the ball loses one tenth of its speed.
vertically downward. Plot the speed-time graph of its motion between t = 0 to
(b) At the highest point, the vertical velocity of the ball 12s (g = 10 ms–2).
becomes zero and acceleration is equal to the Sol. u = 0, a = 10ms–2, s =90 m; using v2 – u2 = 2as
acceleration due to gravity = 9.8ms–2 in vertically We get, t = Ö(2s/a) = Ö(2 × 90/10) = 3Ö2s = 4.24s
downward direction. Now, v = Ö(2sa) = Ö(2 × 10 × 90) = 30Ö2ms–1
(c) When the highest point is chosen at the location x = 0 Rebound velocity of ball, u ' = 9/10v
and t = 0 and vertically downward direction to be the = (9/10) × 30Ö2 = 27Ö2ms–1
positive direction of x-axis and upward direction as Time to reach the h ighest point is, t = u ' /a
negative direction of x-axis. =27Ö2/ 10 = 2.7Ö2 = 3.81s
During upward motion, position sign is negative, sign Total time, t + t ' = 4.24 + 3.81= 8.05s
of velocity is negative and sign of acceleration is The ball will take further 3.8s to fall back to floor, where its
velocity before striking the floor = 27Ö2ms–1
positive.
Velocity after striking the floor = (9/10) × 27Ö2 = 24.3Ö2 ms–1
During downward motion, position sign is positive, Total time elapsed before upward motion of ball
sign of velocity is negative and sign of acceleration is = 8.05 + 3.81 = 11.86s
positive. Thus the speed-time graph of this motion is shown below.
(d) During upward motion Speed (m/s)
u = –29.4 ms–1, a = 9.8 ms–2, v = 0
30 2
As v2 – u2 = 2as
Þ0 – (29.4) = 2 × 9.8 × s 27 2

-(29.4) 2
Þ s= = – 44.1 m 24 2
2 ´ 9.8
Also v = u + at Þ v – u = at
0 4.24 8.05 11.86 t ( s)
Þ 0 – (–29.4) = 9.8 t
3.13 Explain clearly, with examples the distinction between :
29.4 (a) Magnitude of displacement (sometimes called
or t= =3s distance) over an interval of time, and the total length
9.8
of path covered by a particle over the same interval
Total time = 3 + 3 = 6 s (b) Magnitude of average velocity over an interval of time,
[Q time of ascent = time of descent] and the average speed over the same interval. (Average
3.11 Read each statement below carefully and state with reasons speed of a particle over an interval of time is defined
and examples, if it is true or false: as the total path length divided by the time interval).
A particle in one-dimensional motion Show in both (a) and (b) that the second quantity is
(a) with zero speed at an instant may have non-zero either greater than or equal to the first. When is the
acceleration at that instant equality sign true? [For simplicity, consider one-
(b) with zero speed may have non-zero velocity dimensional motion only].
(c) with constant speed must have zero acceleration Sol. (a) Suppose a particle goes from point A to B along a
(d) with positive value of acceleration must be speeding straight path and returns to A along the same path.
up
Motion in a Straight Line 49

The magnitude of the displacement of the particle is Sol. Instantaneous velocity is the velocity of a particle at a
zero, because the particle has returned to its initial particular instant of time. In the case of small interval of
position. The total length of path covered by the time, the magnitude of the displacement is effectively equal
particle is AB + BA = 2AB. Thus, the second quantity to the distance travelled by the particle in the same interval
is greater than the first. This displacement may be of time. Therefore, there is no distinction between
zero but distance will not be zero. instantaneous velocity and speed.
(b) Suppose, in the above example, the particle takes time 3.16 Look at the graphs, given below, carefully and state the
t to cover the whole journey. Then, the magnitude of
reasons which of these graphs cannot possibly represent
the average velocity of the particle over time-interval
one-dimensional motion of a particle.
t is v
x
Magnitude of displacement 0
= = =0
Time-interval t
While the average speed of the particle over the same t t
time-interval is
Total path length 2AB
= =
Time-interval t (a) (b)
Again, the second quantity (average speed) is greater Total path
than the first (magnitude of average velocity). Speed length
Note : In both the above cases, the two quantities are equal
if the particle moves from one point to another along a t t
straight path in the same direction only.
3.14 A man walks on a straight road from his home to a market
2.5km away with a speed of 5 km/h. Finding the market (c) (d)
closed, he instantly turns and walks back with a speed of
(a) This graph does not represent one dimensional motion
7.5 km/h. What is the (a) magnitude of average velocity
because at the given instant of time, the particle will
and (b) average speed of the man, over the interval of time
have two positions, which is not possible in one
(i) 0 to 30min (ii) 0 to 50min (iii) 0 to 40 min?
Sol. (i) Time taken by the man to go to market from his home, dimensional motion.
(b) This graph does not represent one dimensional motion
1
t1 = distance/ speed = 2.5/5 = h because at the given instant of time, the particle will
2 have velocity in positive as well as in negative
Time taken by the man to go his home from the market, direction, which is not possible in one dimensional
t2 = distance/ speed = 2.5/7.5 = 1/3 h motion.
1 (c) It also does not represent one-dimensional motion
\ Total time taken = t1 + t2 = + 1/3 = 5/6 h = 50 min.
2 because this graph tells that the particle will have the
(ii) Time interval 0 to 30 min negative speed. Never happens.
(a) Average velocity = displacement/time (d) This graph does not represent one dimensional motion
= 2.5/(1/2) = 5 km/h because here the total path length decreases after
(b) Average speed = distance / time = 2.5/(1/2) certain time but total path length of a moving particle
= 5 km/h can never decrease with time.
(iii) Time interval 0 to 50 min 3.17 Figure below shows x-t plot of one-dimensional motion of a
Total distance travelled = 2.5 + 2.5 = 5 km
particle. Is it correct to say from the graph that the particle
Total displacement = zero
moves in a straight line for t < 0 and on a parabolic path for
(a) Average velocity = displacement / time = 0
t > 0? If not, suggest a suitable physical context for this
(b) Average speed = distance / time = 5/(5/6) = 6 km/h
graph.
(iv) Time interval 0 to 40 min x
Distance moved in 30min
(from home to market) = 2.5 km
Distance moved in 10min (from market to home) with a
speed 7.5km/h = 7.5 × 10/60 = 1.25 km 0
So displacement = 2.5 – 1.25 = 1.25 km t
Distance travelled = 2.5 + 1.25 = 3.75 km t<0 t>0
(a) Average velocity = 1.25 / (40/60) = 1.875km/h
(b) Average speed = 3.75 / (40/60) = 5.625km/h
3.15 We have carefully distinguished between average speed Sol. No, because the x-t graph does not represent the
and magnitude of average velocity. No such distinction is trajectory of the path followed by a article.
necessary when we consider instantaneous speed and From the graph, it is noted that at t = 0, x = 0
magnitude of velocity. The instantaneous speed is always Context: The above graph can represent the motion of a
equal to the magnitude of instantaneous velocity. Why? body falling freely from a tower under the gravity.
50 Physics
3.18 A police van moving on a highway with a speed of 30kmh–1 Sol. In the SHM acceleration a = – w2x where w is angular
fires a bullet at a thief’s car speeding away in the same frequency (= constant).
direction with a speed of 192kmh–1. If the muzzle speed of (i) At time t = 0.3s, x is negative, the slope of x-t plot is
the bullet is 150ms–1, with what speed does the bullet hit also negative, hence position and velocity are
the thief’s car? (Note, obtain that speed which is relevant negative. Since, a = – w2x, acceleration is positive.
for damaging the thief’s car?) (ii) At time t = 1.2s, x is positive, the slope of x-t plot is
Sol. Muzzle speed of bullet, vB = 150ms–1 also positive, hence position and velocity are positive.
= 540kmh–1 Since, a = – w2x, acceleration is negative.
Speed of police van, vP = 30kmh –1 (iii) At time t = – 1.2s, x is negative, the slope of x-t plot is
Speed of thief car, vT = 192kmh–1 also negative, But since x and t are negative here,
Since the bullet is sharing the velocity of the police van, its hence velocity is positive. Finally, acceleration is
effective velocity is positive.
vB = vB + vP = 540 + 30 = 570kmh–1 3.21 Figures gives the x-t plot of a particle in one-dimensional
The speed of the bullet w.r.t the thief’s car moving in the motion. Three different equal intervals of time are shown.
same direction In which interval is the average speed greatest, and in
v BT = vB – vT = 570 – 192 = 378 kmh–1 which is it the least? Give the sign of average velocity for
378 ´ 1000 each interval.
= = 105ms -1
60 ´ 60
3.19 Suggest a suitable physical situation for each of the x
following graphs?
Sol. Figure (a): The x-t graph shows that initially x is zero i.e. at
rest, then it increases with time, attains a constant value
and again reduces to zero with time, then it increases in
opposite direction till it again attains a constant value i.e. 3
comes to rest. The similar physical situation arises when a 1 2 t
ball resting on a smooth floor is kicked which rebounds
from a wall with reduced speed. It then moves to the
opposite wall, which stops it.
x x a

Sol. Greatest in 3, least in 2; v > 0 in 1 and 2, v < 0 in interval 3.


A
t t t 3.22 Figure gives a speed-time graph of a particle in motion
B along a constant direction. Three equal intervals of time
are shown. In which interval is the average acceleration
(a) (b) (c) greatest in magnitude? In which interval is the average
Figure (b): The velocity changes sign again and again speed greatest? Choosing the positive direction as the
with passage of time and every time some speed is lost. constant direction of motion, give the signs of v and a in
The similar physical situation arises when a ball is thrown the three intervals. What are the accelerations at the
up with some velocity, returns back and falls freely. On points A, B, C and D?
striking the floor, it rebounds with reduced speed each
time it strikes against the floor. D
Figure (c): Initially body moves with uniform velocity. Its
acceleration increases for a short duration and then falls to
B
Speed

zero and thereafter the body moves with a constant velocity.


The similar physical situation arises when a cricket ball
moving with a uniform speed is hit with a bat for very short
interval of time. A C
3.20 Figure below shows x-t plot of a particle executing one-
dimensional simple harmonic motion. Give the signs of 1 2 3 t
position, velocity and acceleration variables of the particle
at t = 0.3s, 1.2s, 1.2s. x Sol. The acceleration is greatest in magnitude in interval 2 as
the change in speed in the same interval is maximum in this
interval.
The average speed is greatest in interval 3 (peak D is at
- 2 –1 1 2 3 maximum on speed axis).
t
0 The sign of v and a in the three intervals are :
+ve in 1, 2 and 3; +ve in 1
+ve in 2, a = 0 in 3.
Motion in a Straight Line 51

3.23 A three-wheeler starts from rest, accelerates uniformly (a) speed of the child running in the direction of motion
with 1 ms–2 on a straight road for 10s, and then moves of the belt?
with uniform velocity. Plot the distance covered by the (b) speed of the child running opposite to the direction of
vehicle during the nth second (n = 1, 2 3...) versus n. What motion of the belt? and
do you expect this plot to be during accelerated motion : a (c) time taken by the child in case (a) and (b)?
straight line or a parabola? Which of the answers alter if motion is viewed by one
of the parents ?
1
Sol. Since S
n th
= u+ a (2n - 1)
2
when u = 0, a = 1 ms–2
1 1
\ Sn th = 0 + (2 n - 1) = (2 n - 1)
2 2
\ For n = 1, 2, 3 .............
1 10
S1 = (2 × 1 – 1) = 0.5 m
2 8 Sol. Let us consider left to right to be the positive direction of
6 x-axis.
1
S2 = (2 × 2 – 1) = 1.5 m 4 (a) Here, velocity of belt, vB = + 4 km/h
2 Speed of the child w.r.t. belt, vC = + 9 km/h
2
1 Speed of the child w.r.t. stationary observer, while
S3 = (2 × 3 – 1) = 2.5 m O 2 4 6 8 10 running in direction of motion of the belt
2
v¢C = vC + vB = 9 + 4 = 13 km/h
1 1 (b) Here, vB = + 4 km/h; vC = – 9 km/h
S4 = (2 × 4 – 1) = 3.5 m; S5 = (2 × 5 – 1) = 4.5 m Speed of the child w.r.t. stationary observer, while
2 2
running in opposite direction of motion of belt
1 1 v¢C = vC + vB = – 9 + 4 = 5 km/h
S6 = (2 × 6 – 1) = 5.5 m; S7 = (2 × 7 – 1) = 6.5 m
2 2 \ Velocity of the child in any direction (from father to
mother or from mother to father) will be 9 kmh–1
1 1
S8 = (2 × 8 – 1) = 7.5 m; S9 = (2 × 1 – 9) = 8.5 m
2 2
vC
1
S10 = (2 × 10 – 1) = 9.5 m
2 + ve direction
F of motion of belt M
3.24 A boy standing on stationary lift (open from above) throws
a ball upwards with the maximum initial speed he can,
equal to 49ms–1. (a) how much time does the ball take to vB
return to his hands? (b) If the lift starts moving up with a Belt
uniform speed of 5ms–1, and the boy again throws the ball
up with the maximum speed he can, how long does the ball 5 –1.
or ms
take to return to his hands? 2
Sol. (a) Here, v(0) = 49 ms–1, a = – 9.8ms–2. \ Time taken by the child in each case (a) and (b) each is
We know, v(t) = v(0) + at t = 50/(5/2) = 20s
\ 0 = 49 – 9.8t, or, t = 49/9.8 = 5s If the motion is observed by one of the parents, answer to
This is the time taken by the ball to reach the case (a) or case (b) will get altered. It is so because speed
maximum height. The time of decent is also 5s. So, of child w.r.t. either of mother or father is 9 km/h
the total time after which the ball comes back is 5s + But answer (c) remains unaltered due to the fact that parents
5s = 10s. and child are on the same belt and as such all are equally
(b) The uniform velocity of the lift does not change the affected by the motion of belt.
relative motion of ball and lift. So, the ball would take 3.26 Two stones are thrown up simultaneously from the edge of
the same total time i.e. it would come back after 10s. a cliff 200 m high with initial speeds of 15 ms–1 and 30
ms–1. Verify that the graph shown in Fig. correctly
3.25 On a long horizontally moving belt, a child runs to and fro
represents the time variation of the relative position of the
with a speed of 9 km/h (w.r.t. belt) between his father and
second stone with respect to the first. Neglect air
mother located 50 m apart on the moving belt. The belt
resistance and assume that the stones do not rebound after
moves with a speed of 4 kmh–1.For an observer on a
hitting the ground. Take g = 10 ms–2. Give the equations
stationary platform outside, what is the for the linear and curved parts of the plot.
52 Physics
To find s1: Let, u1 = velocity of particle after 2s and a1 =
acceleration of the particle during the time interval zero to
5s.
x2 – x1 (m) 120 Then u = 0, v = 12m/s, a = a1 and t = 5s.
We have, a1 = (v – u)/ t = (12 – 0)/5 = 2.4 ms–2
100 \ u1 = u + a1 t = 0 + 2.4 × 2.4 = 4.8ms–2
Thus for the distance travelled by particle in 3 s (i.e. time
80 interval 2s to 5s), we get,
u1 = 4.8ms–1, t1 = 3s, a1 = 2.4 ms–2
60 1 1
s1 = u 1t 1 + a 1 t 1 2 = 4.8 × 3 + × 2.4 × 32
2 2
40 = 25.2m
To find s2: Let, a2 = acceleration of the particle during
20 the motion, t = 5s to t = 10s.
We have, a1 = (0 – 12)/(10 – 5) = – 2.4 ms–2
2 4 6 8 10 t(S) Taking motion of the particle during the motion, t = 5s to
t = 10s, we get,
Sol. For first stone, x (0) = 200 m, v (0) = 15 ms –1 , u2 =12 ms–1, t2 = 1s, a2 = – 2.4 ms–2
a = – 10 ms–2 1 1
s2 = u2t2 + a2t22 = 12 × 1 + × (– 2.4) × 12
1 2 2 2
x1 (t) = x (0) + v (0) t + at = 10.8 m
2 \ Total distance travelled, S = 25.2 + 10.8
x1 (t) = 200 + 15t – 5t2 = 36 m
When the first stone hits the ground, x1 (t) = 0 Average speed = 36/(6 – 2) = 36/4 = 9 ms–1
\ –5t2 + 15t + 200 = 0 3.28 The velocity-time graph of a particle in one-dimensional
On simplification, t = 8s motion is shown below. Which of the following formulae
For second stone, x (0) = 200 m, v (0) are correct for describing the motion of the particle over
= 30 ms–1, a = – 10 ms–2 the time interval from t1 to t2?
x2 (t) = 200 + 30t – 5t2
When this stone hits the ground, x2 (t) = 0 v
\ –5t2 + 30t + 200 = 0
Relative position of second stone w.r.t. first is given by
x2 (t) – x1 (t) = 15t
Since there is a linear relationship between x2 (t) – x1 (t)
and t, therefore the graph is a straight line.
For maximum separation, t = 8 s
So maximum separation is 120 m
After 8 second, only the second stone would be in motion.
So, the graph is in accordance with the quadratic equation.
3.27 The speed- time graph of a particle moving along a fixed t1 t2 t
direction is as shown in the figure. Obtain the distance
travelled by the particle between (a) t = 0 to 10 seconds &
(b) t = 2 to 6s. What is the average speed of the particle 1
(a) x(t2) = x (t1) + v (t1) (t2 – t1) + a (t2 – t1)2
over the intervals in(a) & (b)? 2
(b) v (t2) = v (t1) + a (t2 – t1)
(c) aaverage = [x (t2) – x (t1)]/ (t2 – t1)
Speed (ms–1)

12 (d) aaverage = [v (t2) – v (t1)]/ (t2 – t1)


1
(e) x (t 2 ) = x (t 1 ) + v av (t 2 – t 1 ) + a av .
2
(t2 – t1)2
(f) x (t2) – x (t1) = Area under the v – t curve bounded by
0 5 10 t
t-axis and the dotted lines.
Sol. Distance travelled by the particle between 0 to 10s = Area Sol. (c), (d), (f).
1 As it is evident from the shape of v-t graph that
of DOAB = × 10 × 12 = 60 m.
2 acceleration of the particle is not uniform between time
Average speed = 60/10 = 6m/s intervals t1 and t2. (since the given v-t graph is not
Let s1 and s2 be the distances covered by the particle in straight). The equations (a), (b) and (e) represent uniform
the time interval 2s to 5s and 5s to 6s, then the total distance
acceleration.
covered in the given time interval is s1 + s2
Motion in a Straight Line 53

Practice Questions
Very Short Answer Questions [1 Mark Qs.] Short Answer Questions [2 or 3 Marks Qs.]
1. Are rest and motion absolute or relative terms? 1. The displacement of a body is given to be proportional to
2. Can an object be at rest as well as in motion at the same the cube of time elapsed. What will be the acceleration of
time? the body?
3. What is uniform motion? 2. Draw the position time graph for particle moving with
4. What do you get from area under (v-t) graph? positive and negative velocities.
3. A stone released with zero velocity from the top of the
5. What will be nature of x-t graph for a uniform motion?
tower reaches the ground in 4second. What is the
6. How can the distance travelled be calculated from v-t graph?
approximate height of the tower?
7. Consider that the acceleration of a moving body varies 4. A particle moves along x-axis in such a way that its
with time. What does the area under acceleration-time graph coordinate x varies with time according to the equations =
for any time interval represent? 2 – 5t + 6t2. What is the initial velocity and acceleration of
8. A ball is thrown straight up. What is its velocity and the particle?
acceleration at the top? 5. A body moving along X direction has at any instant its x
9. Give an example which shows that a negative acceleration coordinate given by x = a + bt + ct 2 . What will be
can be associated with a speeding up object. acceleration of the particle?
10. What is the ratio of the time taken to go up and come down 6. What is the ratio of distances travelled by a body falling
by a body thrown vertically up? freely from rest in the first, second and third seconds.
11. What can you say about the nature of acceleration, 7. A stone is thrown upwards with a velocity v from the top of
associated with a mass whose v-t graph is shown? a tower. It reaches the ground with a velocity 3v. What is
–1
the height of the tower?
v (ms ) 8. A motor car moving with a uniform velocity of 20 ms–1
comes to a stop, on the application of brakes after travelling
10 a distance of 10m. What is its acceleration?
9. A wooden block of mass 10g is dropped from the top of a cliff
100m high. Simultaneously, a bullet of mass 10g is fired from the
foot of the cliff upwards with a velocity 100ms–1. After what
0 t(s) time, the bullet and the block meet ?
10 20
10. A car A travelling with a speed of 60km/hr on a straight road is
–10 ahead of another car B travelling with a speed of 40km/hr.
What would be relative velocity of A with respect to B? Would
it be changed if B is ahead of A?
12. What does the speedometer record – the average speed or 11. A bird is tossing (flying to and fro) between two cars moving
instantaneous speed ? towards each other on a straight road. One car has a speed
13. Can a body moving with a uniform velocity be in equilibrium? of 18 m/h while the other has the speed of 27 km/h. The bird
14. Two particles A and B are moving along the same straight starts moving from first car towards the other and is moving
line with B ahead of A. Velocity remaining unchanged, what with the speed of 36 km/h and when the two cars were
would be the effect on the magnitude of relative velocity, if separated by 36 km. What is the total distance covered by
A is ahead of B? the bird? What is the total displacement of the bird?
15. Under what condition the average velocity of a body is
equal to its instantaneous velocity? Long Answer Questions [5 Marks Qs.]
16. When is the magnitude of average velocity same as that of 1. Establish the three equations of uniformly accelerated
average speed? motion graphically.
17. Identify the type of motions? (a) a carom coin is striking 2. Establish the three equations of uniformly accelerated
against the side of a board and not rebounding smoothly motion by calculus method.
but hoping up as it rebounds, (b) a car going along a zigzag 3. Establish the three equations of uniformly accelerated
path on a road. motion algebraically.
18. Can a particle have varying speed but a constant velocity? 4. (a) Derive an equation for the distance covered by a
19. What is the acceleration of a particle moving with uniform uniformly accelerated body in nth second of its
velocity? motion.
(b) A body travels half its total path in the last second of
20. Under what condition will the distance and displacement
its fall from rest. Calculate the time of its fall.
of a moving object have the same magnitude?
5. Explain the importance of position time graph.
54 Physics
6. A car accelerates from rest at a constant rate a, for sometime 14. The velocity-time graph of a body moving in a straight line
after which it decelerates at a constant rate b to come to is as shown in the following figure. Find the displacement
rest. If the total time elapsed is t s, then calcualte and the distance travelled by the body in 6 seconds.
(a) maximum velocity reached. 15. The velocity-time graph of a particle moving along a straight
(b) total distance travelled. line is as shown in the following graph. Calculate the distance
covered between t = 0 to 10 s. Also find displacement in that
Numerical Questions [3 or 5 Marks Qs.] time.
1. A car travels along a straight line with speed 40 km/h from A v(m/s)
to B and returns back to A with speed 60 km/h. Find the 20
average speed of the car and its average velocity. 10
2. A car travelled the first third of a distance x at a speed of 10
km/h, the second third with a speed of 20 km/h and the last 0 t(s)
third at speed of 60km/h. Determine the average speed of 2 4 6 8 10
–10
the car over the entire distance x.
3. A table clock has its minute hand 9 cm long. Find the average –20
velocity of the tip of the minute hand (a) between 3 am to
16. Figure given below shows the variation of velocity of a
3.30 am and (b) between 3 am to 3 pm.
particle with time.
4. A body moves on three quarters of circle of radius r. Find
the values of the displacement and the distance travelled Y
by the body.
C
8
6

velocity
(m/s)
4
A 2
O
X
1 2 3 4 5 6 7
time (sec)
B
5. The displacement x (in m) of a body varies with time t (in s) Find the following :
2 (i) Displacement during the time intervals
as x = - t 2 + 16t + 2 . How long does the body take to (a) 0 to 2 sec, (b) 2 to 4 sec. and (c) 4 to 7 sec.
3
(ii) Accelerations at –
come to rest? (a) t = 1 sec, (b) t = 3 sec. and (c) t = 6 sec.
6. A body starting from rest has an acceleration of 20 m/s2. (iii) Average acceleration –
Calculate the distance travelled by it in 10th second. (a) between t = 0 to t = 4 sec.
7. A body covers 12m in 2nd second and 20 m is 4th second. (b) between t = 0 to t = 7 sec.
Find what distance the body will cover in 4 second after the (iv) Average velocity during the motion.
5th second. 17. Figure shows the acceleration-time graph for a particle
8. Two cars start off to race with velocities 2m/s and 4m/s travel moving along a straight line. Find the following : (Assume
in straight line with uniform acceleration 2m/s2 and 1m/s2 the particle starts from rest.)
respectively. What’s the length of the path if they reach the
final point at the same time?
6
9. A body falling from rest was observed to fall through 78.4 m
in 2s. Find how much time had it been falling before it was 5
observed?
10. A stone is dropped from the top of a cliff and is found to 4
acc.
travel 44.1 m in the last second before it reaches the ground. (m/s² ) 3
Find the height of the cliff.
11. The height y and the distance x along the horizontal, for a 2
body projected in the vertical plane are given by; y = 8t – 1
5t2 and x = 6t. What is the initial velocity of the body?
12. The displacement of a particle along x-axis is given by x = 4 1 3 24 5 6
+ 8t + 14t2. Obtain its velocity and acceleration at t = 2s. time(sec)
13. A particle moves along a straight line such that its
(i) Velocity at the end of (a) 3 sec. (b) 6sec.
displacement ‘s’ at any time ‘t’ is given by s = (t3 – 6t2 + 3t
(ii) Displacement at the end of (a) 4 sec. (b) 10 sec.
+ 4) m. Find the velocity when acceleration is zero.
(iii) Does the direction of motion change during the motion?
Motion in a Straight Line 55

18. The velocity-time graph of a particle moving along a straight 19. On a straight road, A is moving towards north with a velocity
line is given by graph shown 2 m/s and B is moving towards south with a velocity 5 m/s.
What is the relative velocity of (i) A w.r.t. B, and (ii) B w.r.t.
A?
20 – 2t 20. Two cars A and B are running at velocities of 60 km/hr and
v 45 km/hr respectively. Calculate the relative velocity of car
A with respect to B, if
(i) they both are travelling eastwards and
t
t 20 – t 20 (ii) car A is travelling eastwards while car B is travelling
The rate of acceleration and deceleration is same and it is westwards.
equal to 4 m/s2. If the average velocity during the motion is 21. New Delhi is at a distance of 400 km from Jallandhar. A sets
15 m/s and total time of motion is 20 seconds then find out from Jallandhar at a speed of 30 km h –1 and B sets out at
(a) the value of t (b) the maximum velocity of the particle the same time from New Delhi at a speed of 20 km h–1.
during the journey. (c) the distance travelled with uniform When will they meet each other?
velocity. 22. Two passenger trains, each 100 m long, are running on
parallel tracks. One overtakes the other in 20s and one across
the other in 10s. Calculate the velocities of two trains.

HOTS/Exemplar Questions
Very Short Answer Questions [1 Mark Qs.] (a) Plot qualitatively velocity vs time graph.
(b) Plot qualitatively acceleration vs time graph.
1. Can the direction of velocity of an object change, when 5. A man runs across the roof-top of a tall building and jumps
acceleration is constant ? [HOTS]
horizontally with the hope of landing on the roof of the next
2. Is it possible for a body to be accelerated without speeding
building which is of a lower height than the first. If his
up or slowing down ? If so, give an example. [HOTS]
3. Give examples of a one-dimensional motion where speed is 9 m/s, the (horizontal) distance between the two
(a) the particle moving along positive x-direction comes buildings is 10 m and the height difference is 9 m, will he be
to rest periodically and moves forward. able to land on the next building? (take g = 10 m/s2)
(b) the particle moving along positive x-direction comes [Exemplar]
to rest periodically and moves backward. [Exemplar]
4. A uniformly moving cricket ball is turned back by hitting it Long Answer Questions [5 Marks Qs.]
with a bat for a very short time interval. Show the variation
of its acceleration with time. (Take acceleration in the 1. A car, starting from rest, accelerates at the rate f through a
backward direction as positive). [Exemplar] distance s, then continues at constant speed for some time
t and then decelerates at the rate f / 2 to come to rest. If the
Short Answer Questions [2 or 3 Marks Qs.] 1 2
total distance is 5s, then prove that s = ft . [HOTS]
1. Can a body have zero velocity and still be accelerating ? 2
[HOTS] 2. It is a common observation that rain clouds can be at about
2. Two balls of different masses (one lighter and other heavier) a kilometer altitude above the ground.
are thrown vertically upwards with the same speed. Which (a) If a rain drop falls from such a height freely under
one will pass through the point of projection in their gravity, what will be its speed? Also calculate in km/h.
downward direction with the greater speed ? [HOTS]
(g = 10 m/s2)
3. A rubber ball is dropped from a height of 5 metre on a plane
where the acceleration due to gravity is not known. On (b) A typical rain drop is about 4 mm diameter. Momentum
bouncing, it rises to a height of 1.8 m. On bouncing, by is mass x speed in magnitude. Estimate its momentum
what factor the ball loses its velocity ? [HOTS] when it hits ground.
4. A ball is dropped and its displacement vs time graph is as (c) Estimate the time required to flatten the drop.
shown Fig. (displacement x is from ground and all quantities (d) Rate of change of momentum is force. Estimate how
are +ve upwards). [Exemplar] much force such a drop would exert on you.
(e) Estimate the order of magnitude force on umbrella.
Typical lateral separation between two rain drops is
5 cm.
(Assume that umbrella is circular and has a diameter of 1m
and cloth is not pierced through !!)
[Exemplar]
56 Physics

CHAPTER TEST

Time : 45 min. Max. Marks : 24

Directions : (i) Attempt all questions


(ii) Questions 1 to 5 carry 1 mark each.
(iii) Questions 6, 7 and 8 carry 3 marks each.
(iv) Questions 9 and 10 carry 5 marks each.

1. Displacement of a body is given by


x = 1 + 2t + 6t2. Find the value of instantaneous acceleration.
2. A ball is thrown up in air. What is the acceleration and velocity at the instant it reaches its highest point?
3. Two masses in the ratio 1 : 2 are thrown vertically up with the same speed. What is the effect on the time by the mass?
4. Define uniform and non-uniform motion.
5. Differentiate between distance and displacement.
6. The velocity-time graph of a particle moving along a straight line is shown in the following figure by curve OABCD. Calculate
the distance covered by the particle between (i) t = 0 to 18s, (ii) t = 2s to 12s and the maximum value of acceleration during this
interval.
v(m/s)
20 A

15
B C
10
5
D
t(s)
0 2
4 6 8 10 12 14 16 18
7. A car runs at a constant speed on a circular track of radius 200 m, taking 62.8 s on each lap. Find the average velocity and
average speed on each lap.
8. Two straight lines drawn on the same displacement - time graph make angles 30° and 60° with time axis respectively in the
figure. Which line represents greater velocity? What is the ratio of the velocity of line A to the velocity of line B?
B
Displacement

60°
30°
Time
9. From top of a tower 200 m in height, a ball is dropped and at the same time another ball is projected vertically upwards from the
ground with a velocity of 50 m/s. Find when and where the two balls will meet.
10. Draw velocity-time graph of uniformly accelerated motion in one dimension. From the velocity time graph of uniform
accelerated motion deduce the equations of motion

1 2
(i) s = ut + at (ii) v2 – u2 = 2as
2
Motion in a Straight Line 57

Solutions
PRACTICE QUESTIONS Short Answer Questions

d dv
Very Short Answer Questions 1. As, s = kt3 ‘v’ velocity = 3kt2, acceleration = 6kt,
dt dt
1. Yes, both rest and motion are relative terms. An object at which shows the acceleration is increasing with time.
rest in one frame of reference can be in motion in another 2. The x-t graph for an object moving with a positive
frame of reference. velocity (a).
2. Yes. A body may be at rest relative to one object and at the The x-t graph for an object moving with a negative
same time it may be in motion relative to another object. velocity (b).
3. If a body has equal displacements in equal intervals of
time, the motion is called uniform motion. The velocity of x x
motion will be a constant in this case.
4. Area under v-t graph gives displacement.

v t t
B(1)
A C (a) (b)
x0 3. Here, u = 0, a = 10 ms–2, t = 4s,
1 2
't' Q s = ut + at
2
0 D
1
5. It will be a straight line, inclined with time axis. Therefore, h = × 10 × (4)2 = 80m
2
6. The area under v-t graph for a given time interval is 4. Here, s = 2 – 5t + 6t2.
numerically equal to the distance covered by the body in Therefore, after differentiating, we get the velocity, v = ds/
that time interval. dt = – 5 + 12t
7. The area under acceleration-time graph for any time interval For initial velocity, t = 0, \ v = – 5 ms–1
represents the change of speed of the body during that 5. Differentiating the given equation, dx/dt = b + 2ct
time interval. Differentiating again, d2x/dt2 = 2c
8. At the highest point, the velocity of the ball is zero and its Therefore, acceleration = 2c.
acceleration is equal to acceleration due to gravity acting 6. Distance travelled in first second of motion
in the downword direction. 1 1 2
= × 10 × 1× 1 = 5 m [Q s = ut + at ]
9. An object in simple harmonic motion speeds up while 2 2
moving from an extreme position to the mean position but Distance travelled in first two seconds of motion
its acceleration is negative.
1
10. 1 : 1 in the absence of air resistance. = × 10 × 2 × 2 = 20 m
2
11. The slope is same. So acceleration is constant.
Distance travelled in first three seconds of motion
12. Instantaneous speed.
1
13. Yes, since the net force acting on it is zero. = × 10 × 3 × 3 = 45 m
2
14. No effect.
Distance travelled in 2nd second = (20 – 5) m = 15 m
15. When the body is moving with constant velocity, the
Distance travelled in 3rd second = (45 – 20) m = 25 m
average velocity of a body is equal to its instantaneous
The required ratio will be 5 : 15 : 25 = 1 : 3 : 5.
velocity.
7. We know, v2 – u2 = 2as.
16. When the body is moving with a constant velocity, the From this relation, we get, 9v2 – v2 = 2gh.
magnitude of average velocity is same as that of average Therefore, h = 8v2/2g = 4v2/g
speed. 8. Here, u = 20 ms–1, s = 10m
17. (a) Three dimensional motion (b) Two dimensional motion We know, v2 – u2 = 2as.
18. No since both speed and velocity remain constant in Therefore, a = – 400/(2 × 10) ms–2 = – 20 ms–2
uniform motion. 9. Let the block and the bullet meet at a height x from the
19. Zero. ground. Considering downward direction as +ve and the
upward direction as negative, we get
20. If the object is moving along a straight line.
for block, u = 0, s = 100 – x, a = + g
58 Physics
For bullet, u = – 100 ms–1, a = + g, s = – x For velocity-distance equation : v2 – u2 = 2as
Displacement s = area of trapezium OACD
1 2
Putting in s = ut + at 1
2 = (OD + AC) × OA
2
1 2 1 1
We get, – x = – 100t + gt and 100 – x = gt2 = (OD + OE) × (OE – OD)/a
2 2 2
Therefore, – x = – 100t + 100 – x = (OE2 – OD2)/2a = (v2 – u2)/2a
Therefore, t = 1s. or, v2 – u2 = 2as
10. Relative velocity of A with respect to B is 20km/hr and it
æ v - u OE - OD ö
will not change if B is ahead of A. çQ OA = t = = ÷
11. Relative speed of cars = 45 km/h, time required to meet è a a ø
2. Velocity-time equation : v = u + at
36 km Let a body is moving in a straight line with a uniform
= = 0.80h
45 km / h acceleration ‘a’.
Thus, distance covered by the bird = 36 km/h × 0.8 h u- velocity of the body at an instant t = 0
= 28.8 km. v- velocity of the body at the instant t = t
We define acceleration a = dv/dt, or, dv = adt
Long Answer Questions v t
1. Consider an object travelling along a straight line with initial
Integrating both sides, ò u dv = aò 0 dt
velocity u and uniform acceleration a. The final velocity of or, (v – u) = a(t – 0)
the object in time t is v and the distance travelled in that or, v = u + at
time is s. The graph is as shown in figure. 1 2
Distance-time equation: S = ut + at
2
Let, a body is moving in a straight line with a uniform
v C
acceleration (‘a’).
Velocity

E u- velocity of the body at an instant t = 0


u B v- velocity of the body at the instant t = t
D s- distance travelled by the body in time t
The velocity of the body is given by,
A v = ds/dt, or, ds = vdt
O t Time Since, v = u + at
s t
For velocity time equation : v = u + at.
From the graph.
On integrating both sides we have, ò0 ds = ò0 (u + at )dt
a = BC/DB = (AC – AB)/OA = (OE – OD)/OA = (v – u)/t or, [ s ]0s = [ut + 1/ 2at 2 ]t0
Therefore, v = u + at.
1 2
1 or, S = ut + at
For distance-time equation : s = ut + at2 2
2 Velocity-distance equation : v2 – u2 = 2as
Area under v-t curve gives displacement. Let a body is moving in a straight line with a uniform
\ Here displacement S = area of trapezium OACD = area acceleration (a).
of W OABD + area of DDBC At Initial position,
u- velocity of the body at an instant t = 0, when distance
1 travelled = 0
= OA × OD + DB × BC
2 At final position,
v- velocity of the body at the instant t = t when distance
1 travelled = s
= OA × OD + OA× (AC – AB)
2 The, acceleration a = dv/dt = dv/ds × ds/dt = (dv/ds) × v
1 or a ds = v dv
= OA × OD + OA × (OE – OD) s v
2 On integrating both sides we have, ò0 ads = òu vdv
1 s v
= ut + t (v – u) or, a[ s] = [1/ 2v2 ]
2 0 u
1
1 2 or, as = [v2 – u2 ]
or, s = ut + at (Q v – u = at) 2
2
v2 – u2 = 2as
Motion in a Straight Line 59

3. Velocity time equation : v = u + at 5. (i) The importance of position time graph is that its slope
From definition, a = [v – u]/ (t – 0); gives the velocity of the object in uniform motion.
where, v- velocity at time t, u - velocity at time
x¢ B
t=0 E
\ v = u + at x D q x¢– x
C E¢

Position
1 2
Distance time relation : s = ut + at t¢–1
2 x0
Average velocity over the interval t,
O C¢ t t¢
1 1 1 Time A X
va = [v + u ] = [{u + at } + u] = u + at
2 2 2 Let us consider the position time graph of an object moving
1
with uniform velocity represented by the line DB making
va = u + at ...(1) angle q with the time axis. Let the coordinates of D and B be
2 (x, t) and (x¢, t¢) respectively. Let BA and DC¢ be
Velocity distance equation : v2 – u2 = 2as perpendiculars drawn from B and D respectively on the
Now the distance travelled by the object with the average time axis and BE and DC be perpendiculars on y-axis from B
speed is, and D.
s = (t – 0)va ...(2) Now, BE¢ = CE = x¢ – x and C¢ A = t¢ – t = DE¢
Combining the equations (1) and (2), we have,
x¢ - x BE ¢
Then velocity = = = tan q
1 1 t¢ - t DE ¢
s = t (u + at) Þ s = ut + at2
2 2 So, velocity v = slope of position time graph.
From above relations, by eliminating t, we can also derive,
v2 – u2 = 2as B
x + Dx
4. (a) For a body having a uniform acceleration a in a straight A Dx
line, starting with an initial velocity u, the displacement x
Dt C
in ‘n’ seconds is given by,
x
1 2
Sn = nu + an
2 t¢
O t
In (n – 1) seconds, t t¢ + Dt
1 (ii) The position time graph for a stationary object is a
Sn–1 = (n – 1) u + a (n – 1)2
2 straight line parallel to the time axis. Here the slope of
curve is zero, means the object is stationary as v = 0.
\ Displacement in nth second = Sn – Sn–1 (iii) In case of variable velocity the position time curve is
a not a straight line. In this case the slope of curve gives
Snth = u + (2n – 1) average velocity.
2
(b) Let S be the complete length of fall and t be the time ( x + Dx ) - x Dx
vaverage = =
taken for it, then (t + D t ) - t Dt
= slope or chord AB when DT ® 0.
1 2
S= gt (Q u = 0) ...(i) then the slope of curve gives the instantaneous
2 velocity.
S Dx
Also, is covered in the last second. Instantaneous velocity = Lt
2 Dt ® 0 Dt

S g Thus position time graph gives information about


\ =0+ (2t – 1) ...(ii) velocity.
2 2
6. (a) The given situation is shown in fig. below :
Using eqn. (i) and (ii), solve for t,
v A
1 2
S = g (2t – 1) = gt ,
2
i.e., 4tg – 2g = gt2 Þ gt2 – 4tg + 2g = 0 Þ t2 – 4t + 2 = 0
v(ms–1 )

a b
4 ± 16 - 8 4 ± 2 2
i.e., t = =
2 2
D B
t=2± 2 O t1 t2 t (s)
60 Physics
Let t1 and t2 be the times of acceleration and deceleration 3. (a) Between 3 am to 3.30 am the minute hand rotates
respectively of the car. through 180°
If t = total time of journey, \ Displacement = 2r = 2 × 4 = 8 cm.
then t = t1 + t2 ...(i) Time = 30 min = 30 × 60s.
Also v = maximum velocity reached
(i) For accelerated motion, 8
\ Average displacement = 30 ´ 60 = 4.4 ´ 10–3 cm/s
using equations, v = u + at, we get
v = 0 + at1 (b) Displacement between 3am to 3.30 pm
or v = at1 ...(ii) Time taken = 12 hours 30 minutes
(ii) For decelerated motion, (12 × 60 + 30)60 s = 45000s
here, u = v, final velocity is zero.
\ 0 = v + (–b) t2 8
\ Average velocity = cm/s = 1. 8 ´ 10–4 cm/s
v = bt2 ...(iii) 45000
\ From (ii) and (iii), we get
at1 = bt2 4. Here, Displacement AB = r 2 + r 2 = 2r
t2 a 3 3pr
or = ...(iv) Distance = AB = ´ 2pr =
t1 b 4 2
Adding 1 on both sides of equation (iv), we get 2 2
5. x=- t + 16t + 2 Þ dx = - 2 ´ 2t + 16
t2 a t1 + t 2 a +b t a +b 3 dt 3
+ 1= + 1 or = or =
t1 b t1 b t1 b dx 2
At rest =0 \ - ´ 2t + 16 t = 12 s
b dt 3
\ t1 = t and v = at1 = ab t
a+b a+b a
6. Distance travelled during n th second = u + (2n - 1)
(b) Total distance travelled 2
1 20
= area of DOAB = (OB ´ AD) S = 0+ (19)metre
2 10th 2
1 1 1 ab 2 1
= (t1 + t 2 )v = vt = t 7. Here, s = u + a (2 ´ 2 - 1)
2 2 2 ( a + b) 2
1
Numerical Questions Þ 12 = u + a ´ 3 ...(i)
2
1
x 20 = u + a (2 ´ 4 - 1)
1. Time take to go from A to B = 2
40
1
Þ 20 = 4 + a ´ 7 ...(ii)
x 2
Time taken to come back from B to A = Solve (i) and (ii) to get u and then
60
When AB = BA = x we get S9– S5 = 136 m
1 1
x+x 8. Here, s = 2t + ´ 2t 2 Also, s = 4t + ´ 1 ´ t 2 ; Solving we
\ Average speed = = 48 km/h 2 2
x x get t = 4s;
+
40 60
1
Q Displacement of car is zero. \ s = 2´ 4 + ´ 2 ´ 42 = 24 m
\ Average velocity is zero. 2
9. Let the body be falling for t seconds and distance travelled
x/3 in that time is s m.
2. Time taken in 1st one third distance =
10
\ Distance travelled in (t + 2) s = (s + 78.4)m.
x/3 1
Time taken in 2nd one third distance = s = ´ 9.8 ´ (t + 2)2
20 2
Solve this eq. to get t.= 3 sec.
x/3 10. Let h be the height of the cliff and t be the time taken by the
Time taken in 3rd one third distance =
60 stone to fall from top to bottom of the cliff.
x
Hence, u = 0, a = 9.8 m/s2, Sn = 44.1m
Average speed = = 18 km/h. 1
x x x 9.8
+ + s = u + a (2n - 1) Þ 44.1 = (2t - 1)
30 60 90 2 2
Motion in a Straight Line 61

Þ t = 5s 16. (i) (a) Displacement between t = 0 sec. to


by using the value of t, we get h = 122.5 m t = 2 sec.
1
Þ ´ 2 sec ´ 8 m/s = 8m
dx
11. Here, v y = dy = 8 - 10t and v x = =6 2
dt dt (b) Between t = 2 sec. to t = 4 sec.
Þ 2 sec ´ 8 m/s = 16m
\ Velocity = v = v x2 + v 2y
(c) Between t = 4 sec. to t = 7 sec.
2 2
Þ v = 6 + (8 - 10t ) 1
Þ ´ 3 sec ´ 8 m/s = 12m
2
At t = 0, initial velocity = u = 6 2 + 82 (ii) Acceleration = slope of v-t curve
= 10 m/s (a) At t = 1 sec,
12. Here, x = 4 + 8t + 14t 2
8 m/sec
dx slope = m/sec = 4 m/s 2
u= = 0 + 8 + 28t and v|t = 2 = 8 + 28 × 2 = 64 m/s 2 sec
dt
(b) At t = 3 sec, slope = 0
dv (c) At t = 6 sec,
a= = 28 m / s 2
dt
8 2
slope = - = -2 m/s 2
13. 3 2
Here, s = (t - 6t + 3t + 4) ; 3 3
(iii) Average acceleration
ds dv
v= = 3t 2 - 12t + 3 ; a = = 6t - 12
dt dt Total change in velocity
=
When a = 0 Þ 6t – 12 = 0 Þ t = 2s Total change in time
\ v = 3 × 22 – 12 × 2 + 3 (a) Between t = 0 to t = 4 sec.,
= 12 – 24 + 3 = – 9 m/s
8m/s
Average acceleration = = 2m/s 2
14. v(m/s) 4
4
(b) Between t = 0 to t = 7 sec.,
2
t(s) 0
1 2 3 4 5 6 Average acceleration = =0
–2 7
–4
Total displacement
(iv) Average velocity =
Distance travelled in 6 s = Area under v-t graph = 4 × 2 + 2 × Total time
2 + 2 × 2 = 16 m.
Displacement = 4 × 2 + (– 2) × 2 + 2 × 2 8 + 16 + 12 36 1
= = = 5 m/s
= 12 – 4 = 8 m. 7 7 7
17. The equation for the above curve is given by : a = 2t for
15. v(m/s)
0£t£3
20
10 [Q Equation of a straight line passing through the origin is
given by y = mx where m is the slope]
0 t(s) and a = 12 – 2t, for 3 < t £ 6
2 4 6 8 10
–10 (i) (a) Thus, a = 2t Þ dv = 2t dt
–20 v v
Integrating both sides we have ò dv = ò 2t dt
0 0
Distance = Area of v-t graph between t = 0 to 10 s
1 1 ét2 ù
3
= ´ 6 ´ 20 + ´ 20 ´ 2 + 10 ´ 2 = 60 + 20 + 20 = 100 m;
2 2 Þ v=2ê ú Þ v = 9 m/s
ëê 2 ûú 0
1 1
Displacement = × 6 × 20 – × 20 + 2 + 10 × 2 (b) Again, from a = 12 – 2t
2 2
Integrating both sides we have
= 60 – 20 + 20 = 60 m.
62 Physics
v 6 2t (40 - 2t )
15 =
ò dv = ò (12 - 2t ) dt 20
9 3
Solvin g quadratic equation, 150 = 40t – 2t 2
6 Þ t = 5 sec. (another solution not acceptable think why!)
Þ v - 9 = éë12t - t ùû
2
3 Maximum velocity = 4t = 4 × 5 = 20 m/s
distance travelled with uniform velocity
Þ v – 9 = 12 × 3 – (36 – 9) Þ v = 18 m/s
= (20 – 2t) V = (20 – 2 × 5) × 20 = 200m
ds 19. Let northward direction is taken as positive. So southward
(ii) From v = \ ds = v dt direction is negative.
dt
r r r
(a) Displacement at the end of 4 sec. (i) \ V AB = VA - VB = ( +2) - ( -5) = 2 + 5
s 4 3 4 = + 7 m/s (+ means towards north)
Þ ò ds = ò vdt Þ ò vdt + ò vdt
0 0
N
0 3
VA
3 4
Þ ò t 2 dt + ò (12t - t 2 + 9) dt
0 3

3 4 4
VB
é t3 ù ét2 ù é t3 ù
ê ú + 12 ê ú - ê ú + 9 [t ]3
4
Þ 3 2 3
ëê ûú 0 ëê ûú 3 ëê úû 3
r r r
1 1 V BA = V B - V A
(ii)
Þ (27 - 0) + 6 (16 - 9) - (64 - 27) + 9 (1) = (–5) – (+ 2) = – 5 – 2
3 3
= – 7 m/s (– means towards south)
Þ 9 + 42 – 12.33 + 9 = 47.67 m 20. Let us take east direction as positive, then the west direction
(b) The particle is under acceleration only during first is negative,
6 sec, thereafter it continues to move with uniform (i) VA = + 60 km/hr
velocity.
(ii) VB = + 45 km/hr
\ Displacement between 0 to 3 sec.
The relative velocity of the car A w.r.t. B is
3 VA – VB
S1 (say) = ò t 2 dt = 9 m ... (1) \ VAB = 60 – 45 = 15 km/hr
0 \ VAB is 15 km/hr eastwards
displacement between 3 to 6 sec., (ii) VA = + 60 km/hr VB = – 45 km/hr
6 VAB = VA – VB = 60 – (–45)
S 2 (say) = ò (12t - t 2 + 9) dt = 105 km/hr eastwards
3 21. Here, vA = 30 km/h, vB = – 20 km/hr
Relative speed of one w.r.t. another
6
6 é t3 ù vAB = vA – vB = 30 – (–20) = 50 km h–1.
6 ét 2 ù - ê ú + 9 [t ]3
6
Þ
ë û3 ê 3 ú Time taken to cover a distance of 400 km with a speed of 50
ë û3
km h–1 = 400/50 = 8 h.
1 Hence they will meet after 8 hours from the start.
Þ 6 [36 - 9] - [216 - 27] + 9 [6 - 3] 22. Let u and v be the velocity of trains A and B. While
3
overtaking, the relative velocity of train A w.r.t. B = u – v.
Þ 162 – 63 + 27 = 126 m ... (2)
While crossing, the relative velocity of train A w.r.t. B = u +
displacement between 6 to 10 sec.
v
S3 (say) = 18 × 4 = 72 m Total distance to be travelled by train A while crossing =
\ Total displacement = 9 + 126 + 72 = 207 m 100 + 100 = 200.
(iii) Evidently, the direction of motion does not change.
18. v = 0 + ut 200
\ 20 = or u – v = 10
u-v
1
Total displacement = (20 - 2t + 20) ´ 4t = 2t ( 40 – 2t)
2 200
and 10 = or u + v = 20
u+v
total displacement
Average velocity = On solving, u = 15 ms–1 and v = 5 ms–1.
total time
Motion in a Straight Line 63

In this time, the distance moved horizontally is


HOTS/EXEMPLAR QUESTIONS
x - x o = vox t = 9m / s ´ 1.34s = 12.06m
Very Short Answer Questions Yes-he will land.
1. Yes. For an object thrown vertically upwards, the direction 9 m/s
of velocity changes during its rise and fall. But acceleration
acts always downwards and remains constant.
2. Yes. An object in uniform circular motion is accelerating but
its speed neither decreases nor increases. 9m
3. (i) x(t) = t – sin t
(ii) x(t) = sin t
4.

10 m

Long Answer Questions

1. For accelerated motion, u = 0, a = f, s = s


As, v2 – u2 = 2as
Short Answer Questions v12 , 02 = 2 fs or v1 = 2 fs
1. Yes. A body thrown vertically upwards has zero velocity at For uniform motion, u = v1 = 2 fs , t = t
its highest point but has acceleration equal to the
acceleration due to gravity. Distance travelled, s2 = ut = 2 fs t
2. In case of motion under gravity, the speed with which a
body returns back is always equal to the speed with which For decelerated motion, u = 2 fs , a = – f /2, v = 0
it is thrown up. Since expression for final speed does not As v2 – u2 = 2as,
involve mass, both the balls will acquire the same speed.
\ 02 – ( 2 fs )2 = 2 × (– f / 2) s3
3. Downward motion
Distance travelled, s3 = 2 s
v2 - 02 = 2 ´ 9.8 ´ 5 Þ v = 98 = 9.9 Given, s + s2 + s3 = 5 s
Also for upward motion
or, s+ 2 fs t + 2s = 5 s
02 - u 2 = 2 ´ ( -9.8) ´1.8 Þ u = 3528 = 5.94
or, 2 fs t = 2 s
9.9 - 5.94
Fractional loss = = 0.4
9.9 1 2
or, s= ft
4. The ball is released and is falling under gravity. Acceleration is – g, 2
except for the short time intervals in which the ball collides with
2. (a) v = 2gh = 2 ´ 10 ´ 1000 = 141 m / s = 510km / h
v 4p 3 4p
( )
3
2 ´ 10-3 (10) = 3.4 ´ 10-5 kg
a 3
(b) m= r r=
3 3
P = mv » 4.7 × 10–3 kg m/s » 5 × 10–3 kg m/s.
0 t (c) Diameter » 4mm
t
–g Dt » d / v = 28ms » 30ms

DP 4.7 ´ 10 -3
ground, and when the impulsive force acts and produces a large (d) F= = » 168N » 1.7 ´ 102 N
Dt -6
28 ´ 10
acceleration.
5. Suppose that the fall of 9 m will take time t. Hence 2 2
(e) Area of cross-section = pd / 4 » 0.8m
gt 2 With average separation of 5 cm, number of drops that will
y = yo = voy -
2 0.8m 2
fall almost simultaneously is » 320.
Since voy = 0, (5 ´ 10-2 ) 2
2 ( y - yo ) Net force » 54000 N (Practically drops are damped by air
2 ´ 9m
t= ® = 1.8 » 1.34 seconds viscosity).
g 10 m / s 2
64 Physics
CHAPTER TEST For the ball thorwn upwards,
u = 50 m/s, a = – 9.8 m/s2, s = (200 – x)m, t = t
1. Given, x = 1 + 2t + 6t2;
dx 1 2
v= = 0 + 2 + 12t \ s = ut + at
dt 2
dv 1
Instantaneous acceleration = = 12 m/s2 Þ 200 - x = 50 t + (-9.8) t 2 ..... (2)
dt 2
2. At the highest point, velocity is zero. Solving (1) and (2) we get, t = 45, x = 78.4
Accleration is due to gratvity (g = 9.8 m/s2) in the downward
direction. 10. Consider an object moving along a straight line with uniform
3. Mass does not influence time. acceleration a. Let u be the initial velocity at t = 0 and v be
4. Refer to theory the final velocity after time t.
5. Refer to theory From graph OA = ED = u OC = EB = v OE = t = AD
6. (i) Distance covered in 0 to 18s = area under v–t graph
Velocity
1 1 1 v
= ´ 8 ´ 20 + ´ (10 + 20) ´ 2 + ´ 4 ´ 10 = 170 m C B
2 2 2
(ii) Distance travelled in 2 to 12s
1 1 Au D
= ´ 6 ´ (5 + 20) + ´ 2 ´ (10 + 20) + 2 + 10 = 125 m
2 2
Max. acceleration is calcuated from the slope of the graph
which is in AB part of the graph.
20 - 10
O
E Time
Slope of AB = – = – 5 m/s2
10 - 8
7. Displacement in a complete loop = 0 1 2
(i) s = ut + at
2
Distance = 2pr = 2 × 3.14 × 200 Area under velocity time graph for a given time interval
represents the distance covered by a uniformly accelerated
displacement object in a given time interval.
Average velocity = =0 From graph, acceleration, a = slope of velocity time graph
time
AB.
2 ´ 3.14 ´ 200 DB DB
Average speed = = 20 m/s \ a= = or DB = at
62.8 AD t
1 Distance travelled by object in time t is
8. Velocity of line A, vA = tan 30º = and of line B, s = area of trapezium OABE
3
= area of rectangle OADE + Area of triangle ADB
vB = tan 60º = 3 Þ line B has greater velocity
1
V 1 = OA ´ OE + DB + AD
Velocity ratio A = 2
VB 3
1 1
9. Let the two balls meet after time t at a distance x metre from = ut + at × t = ut + at2
top. The distacne of ball from ground at the meeting point 2 2
= 200 – x. (ii) v2 – u2 = 2as
Distance travelled by an object in time interval t is
u=0 s = area of trapezium OABE
x,t 1
= (EB + OA) × OE
200m 2
1
= (AB + ED) × OE (Q OA = ED)
2
Acceleration, a = slope of velocity time graph AB
u=50m/s
DB EB - ED EB–ED
For the ball dropped down, a= = Þ OE =
AD OE a
u = 0, a = 9.8 m./s2, s = x, t = t
1 (EB – ED)
1 s= (EB + ED) ×
s = ut + at 2 2 a
2
1 1
1 = (EB2 – ED2) = (v2 – u2)
Þ x = 0 ´ t + ´ 9.8 ´ t 2 = 4.9t2 ..... (i) 2a 2a
2 or, v2 – u2 = 2as
¿¿¿
Motion in a Plane 65

4 Motion in a Plane
C ha p t e r

An object moving in a plane is said to have two dimensional motion. The two dimensional motion is equal to vector sum of two one
dimensional motions along mutually perpendicular direction.

SCALARS AND VECTORS


A scalar quantity is a physical quantity that has only magnitude. Examples mass, length, time, distance, speed, temperature, energy,
work done, power etc.
A vector quantity is a physical quantity that has both magnitude and direction and obeys the triangle law or parallelogram law of
vector addition. Examples weight, displacement, velocity, force etc.
Representation of a vector : A vector is represented by a directed line segment i.e., a straight line with B
an arrowhead over it. The length of the line gives the magnitude and the direction of arrow gives the
r uuur B)
direction of the vector. In the figure, the line AB represents a vector a or AB, whose direction is from a (A
r
A to B and magnitude is a which is its length AB.
A
Position and Displacement Vectors
Consider a body moving in X-Y plane. Let P and Q be the positions of the body at time t1 and t2
y
respectively. P Q
The lines joining P and Q with the origin of coordinate system O are called the position vectors of the
r r'
points P and Q. The line joining P and Q is called displacement vector.
uuur uuur uuur
Here, OP and OQ are position vectors of P and Q respectively and PQ is the displacement vector.. O x
uuur r r
PQ = r ¢ - r
The magnitude of displacement is either less or equal to the path length.

Different Types of Vectors


r r
(i) Equal vectors: Two vectors a an d b are said to be equal if and only if they have same magnitude i.e.
r r r r
| a | = | b | and same direction. The equality is usually indicated by a = b . Graphically, equal vectors superimpose each other..
(ii) Negative vectors: A negative vector of a given vector is having the same magnitude as the given vector and opposite direction.
r r r r
a is said to be the negative of b if a = – b .
(iii) Unit vectors: A unit vector of a given vector is that vector having unit magnitude and acting in the same direction of the given
vector. It is equal to the vector divided by its magnitude.
r r r r r
A vector a can be expressed as a = | a | â Thus unit vector â = a / a
(iv) Co-initial vectors: Two or more vectors are said to be co-initial if they are having common staring point.
(v) Co-linear vectors: Two or more vectors having equal or un-equal magnitudes would be co-linear if they act along the same line
or along parallel lines in the same direction.
(vi) Coplanar vectors: Two or more vectors would be coplanar if they are lying on the same plane.
r
(vii) Zero or null vector : A vector having zero magnitude and arbitrary direction is called zero vector. It is written as, | 0 | = 0
Displacement of a stationary particle, velocity of a stationary body and acceleration of a body in uniform motion are few
examples of zero vector.
66 Physics
Properties of zero vector :
· The addition or subtraction of zero vector from a given vector does not change the given vector.
r r
A + 0 = A, A – 0 = A
· The multiplication of a non zero real number with a zero vector gives a zero vector.
r
n0 =0
ur ur ur ur
· If N1 and N2 are two different non-zero real numbers and N1 ¹ N2 then N1 A = N2 B can hold if both A and B are zero
vectors.
(viii) Free Vectors: Displacing a vector parallel to itself leaves the vector unchanged such vectors are called free vectors.
(ix) Localised Vectors: Vectors having fixed line and locations.
Multiplication of Vectors by Real Numbers
r r r
The process of multiplication of a vector by a real number will give another vector. If one vector a is given by a = 2 b , the meaning
r r r r r r
is that the magnitude of a is twice the magnitude of b (i.e. | a | = 2½ b |) however the direction of a and b are same. It is represented
r r
mathematically by, | l a | = l | a |.
When l > 0, the vector and its resultant, after multiplication by a scalar l, will have the same direction. However, when l < 0, the
direction of the resultant vector, after multiplication with a scalar, will be opposite.

ADDITION OF VECTORS (GRAPHICAL METHOD)


Triangle Law of Vector Addition
According to this law if two vectors are represented by the two sides of a triangle taken in order, both in magnitude and direction,
then their resultant is represented by the third side of the triangle, both in magnitude and direction, taken in opposite order.
uuur uuur uuur ur ur ur A
OQ = OP + PQ Þ R = A + B Q P Q

r
The magnitude of resultant vector R = A2 + B 2 + 2 AB cos q B R =A+ B
B
B R =A+ B
ur ur ur
If R is the resultant of two vectors A and B , then aA q
A O P S
O
R = A2 + B 2 + 2 AB cos q
ur ur
where q ® angle between A and B .
ur ur B sin q
If R makes an angle a with A , then tan a =
A + B cos q
Q S
Parallelogram Law of Vector Addition
According to this law if two vectors are acting simultaneously from a
point and represented both in magnitude and direction by two R
adjacent sides of a parallelogram, then their resultant is represented B B
by the diagonal of the parallelogram, both in magnitude and direction,
drawn from the same point.
uur uuur uuur ur ur ur O P
OS = OP + OQ Þ R = A + B A A

Polygon Law of Vector Addition


C S
According to this law, if a number of vectors are represented in
magnitude and direction by the sides of polygon taken in same order, Q
then their resultant is represented in magnitude and direction by the A + B+ C
closing side of the polygon taken in opposite order. B
uur uuur uuur uur
OS = OP + PQ + QS
Vector addition (analytical method) : It is much easier to add vectors O P
A
by combining their respective components than using the graphical
method.
Motion in a Plane 67

r r
If A= Ax iˆ + Ay ˆj + Az kˆ and B = Bx iˆ + B y ˆj + Bz kˆ
r
Let R = Rx iˆ + R y ˆj + Rz kˆ represents the sum
r r r
i.e. R = A + B = ( Ax iˆ + Ay ˆj + Az kˆ) + ( Bxiˆ + B y ˆj + Bz kˆ)
Then Rx = Ax + Bx, Ry = Ay + By and Rz = Az + Bz
Properties of vector addition :
uur uur ur ur
· Vector addition is cummutative, A + B = B + A
uur uur uur ur ur uur
· Vector addition is associative, ( A + B ) + C = A + ( B + C )
Subtraction of Vectors
ur ur ur ur ur ur ur ur
Subtraction of vector B from vector A is defined as the addition of the vector- B to vector- A . Therefore, A – B = A + (– B )
ur ur ur ur uuur uur uur
In DOPS, A - B = A + ( - B ) = OP + PS = OS [By triangle law]
r ur Q
If the angle between A and B is q, then
ur ur B B
angle between A and – B is (180° – q)
A P q
\ R = A2 + B 2 + 2 A.B cos(180° - q) and, O b
A -B
tan f = B sin(180° - q) / { A + B cos(180° - q)} A + (- B )
uur uur uur uur ur S
If A and B are two vectors, q is angle between them and R = A - B , then
B sin(180° - q)
R = A2 + B 2 + 2 AB cos (180° - q ) ; tan b = A + B cos (180° - q )
ur uur
where b is the angle made by R with A .
Properties of vector subtraction :
ur ur ur ur
· Vector subtraction is not commutative i.e. A - B ¹ B - A
ur ur ur ur ur ur
· Vector subtraction is not associative, i.e. A - ( B - C ) ¹ ( A - B) - C
RESOLUTION AND COMPONENTS OF VECTOR IN A PLANE
Resolution is the process of splitting a single vector into two or more vectors in different directions, which produce the same effect
as the given single vector. The splitted parts are called the components of the given vector.
When a vector is splitted into two components at right angles to each other, then the components are called rectangular components
of the given vectors.
r r
a and b are two non zero vectors shown in the figure.
Q

b Q
A A S
A
P' nb P'
P ma
a P
ur r
Let us consider another rvector A shown as PQ. Now through P draw a straight line parallel to a and through tail Q draw a
straight line parallel to b . Let these straight lines intersect at point Q.
ur uuur uuur uuuur
A = PQ = PP ' + P ' Q
uuur uuuur r r
Now since PP ' and P ' Q are vector parallel to vectors a and b we can write,
uuur r uuuur r
PP ' = m a and P ' Q = n b
ur uuur uuur uuuur ur r r
Thus, A = PQ = PP ' + P ' Q or,, A = m a + n b
68 Physics
Rectangular Components of a Vector in a Plane
If a vector is splitted into two component vectors at right angles to each other, the component vectors are called rectangular
components of the vector.
Let, iˆ, ˆj are the unit vectors along OX and OY respectively then
ur ur
A x = Ax$i ; A y = Ay $j (Here, i = j = k = 1 since these are unit vectors) Y
uuur uuur uur
From triangle law of vector addition, OR = OP + PR
ur ur ur R
Therefore, A = A x + A y Q
A
Let, ÐPOR = q j
We may write, Ax = OP and Ay = OQ = PR q
O i P X
\ OP = OR cos q or, Ax = A cos q and PR = OR sin q or, Ay = A sin q

Moreover, OR2 = OP2 + PR2 \ A2 = Ax2 + Ay2 or, A = Ax2 + Ay2 and, tan q = PR/OP = Ay /Ax

Vector A can be resolved into three components along x–, y–, and z-axes in three dimensions. If a, b, and g are the angles between A

and the x-, y-, and z-axes, respectively (fig.) we have y

Ax = A cos a, Ay = A cos b, Az = A cos g

In general, we have A = Ax iˆ + Ay ˆj + Az kˆ
Ay
Ay
The magnitude of vector A is A
b
g a Ax
A= Ax2 + Ay2 + Az2 x

SCALAR PRODUCT OR DOT PRODUCT OF Az


TWO VECTORS Az
® ® Ax
If q is the angle between A and B . z
® ® ® ®
Then A (B cos q) = A • B , A and B are the magnitudes of vector A and B .
The quantity AB cos q is a scalar quantity.
® ®
B cos q is the component of vector B in the direction of A .
Hence, the scalar product of two vectors is equal to the product of the magnitude of one vector and the component of the second
vector in the direction of the first vector.
® ® ® ® ® ®
Ex. : [1] W = F • S [2] P = F • V [3] f = B • A
Properties of Scalar Product
(a) The scalar product is commutative.
® ® ® ®
A • B = B• A
® ® ® ®
A • B = A (B cos q) = (A cos q) B = B • A
(b) The scalar product is distributive.
® ® ® ® ® ® ®
A • (B + C) = A • B + A • C
(c) The scalar product of two mutually perpendicular vectors is zero.
® ®
A • B = AB cos 90° = 0 ( Q cos 90° = 0)
® ® ® ®
A • B = 0 for A ^ B
iˆ • ˆj = 0, ˆj • kˆ = 0, kˆ • iˆ = 0
Motion in a Plane 69

(d) The scalar product of two parallel vectors is equal to the product of their magnitudes.
® ®
A • B = AB cos 0° = AB (Q cos 0° = 1)
(e) The scalar product of a vector with itself is equal to the square of
the magnitude of the vector.
® ®
A • A = A A cos 0° = A2
iˆ • iˆ = 1, ˆj • ˆj = 1, kˆ • kˆ = 1
CROSS PRODUCT OR VECTOR PRODUCT OF TWO VECTORS
® ®
Cross product of A and B inclined to each other at an angle q is defined as :
® ®
A B sin q nˆ = A ´ B
® ®
n̂ ^ to plane of A and B .
® ®
Direction of n̂ is given by right hand thumb rule. Curl the fingers of your right hand from A to B . Then the direction of the erect
® ®
thumb will point in the direction A × B .
Properties of Vector Product
(a) Thevector product is not commutative.
® ® ® ®
i.e. A´ B¹B´A
® ® ® ®
A ´ B =-B ´ A
(b) The vector product is distributive.
® ® ® ® ® ® ®
i.e. A ´ (B + C) = A ´ B + A ´ C
(c) The magnitude of the vector product of two vectors mutually at right angle is equal to the product of the magnitudes of the
vectors.
® ® ® ®
A ´ B = ABsin 90° nˆ = AB nˆ , | A ´ B |= AB
(d) The vector product of two parallel vectors is a null vector (zero vector).
® ®
A ´ B = AB(sin 0°) nˆ = 0
(e) The vector product of a vector by itself is a null vector (zero vector).
® ®
A ´ A = AA sin(0°)nˆ = 0

(f) The vector product of unit orthogonal vectors ˆi, ˆj and kˆ have the following relations in the right-handed coordinate system.

(a) iˆ ´ ˆj = kˆ ˆj ´ iˆ = - kˆ
ˆj ´ kˆ = iˆ kˆ ´ ˆj = -iˆ
kˆ ´ iˆ = ˆj iˆ ´ kˆ = - ˆj

(b) iˆ ´ iˆ = 0 ˆj ´ ˆj = 0 kˆ ´ kˆ = 0
Position, Velocity and Acceleration Vectors
y
(a) Position vectors : A vectors which gives the position of a point with reference
to the origin of the co-ordinate system. n of v
Directio
A position vector r can be expressed as P'
Dy P
r = xiˆ + yjˆ + zkˆ Dr
r'
where x, y and z are the components of r and iˆ, ˆj and k̂ are
r
unit vectors along x-, y- and z-axes, respectively.
Displacement between instants t and t¢ is Dx
O x
70 Physics
Dr = r¢ – r ... (i)
and is directed from P to P¢.
We can write Eq. (i) in component form :

Dr = ( x¢iˆ + y ¢ ˆj ) - ( xiˆ + yjˆ) = iˆDx + ˆj Dy


where Dx = x¢ – x, Dy = y¢ – y
r
(b) The average velocity (v ) of an object is the ratio of the dispalcement and the corresponding time interval

r Dr Dxiˆ + Dyjˆ ˆ Dx ˆ Dy r r
i.e., v = = =i + j or v = vx iˆ + v y ˆj
Dt Dt Dt Dt
Dr
Since, vr = , the direction of the average velocity is the same as that of Dr
Dt
(c) The velocity (instantaneous velocity) is given by the limiting value of the average velocity as the time interval approaches zero
uur
r Dr dr
i.e., v = lim =
Dt ® 0 Dt dt
y
The direction of velocity at any point on the path of an object is tangential to
the path at that point and is in the direction of motion.
We can express v in a component form :
The magnitude of velocity vector v is then
v
v = v 2x + v 2y vy j
q
And the direction of v is given by the angle q : vx i
vy æ yö
v
tan q = , q = tan -1 ç ÷ O x
vx è vx ø
vx , v y and angle q are shown in fig. for a velocity vector v.
(d) Acceleration : The average acceleration a of an object for a time interval Dt moving in x-y plane is the change in velocity
divided by the time interval :
Dv D (vx iˆ + v y ˆj ) Dv x ˆ Dv y ˆ
i.e., ar = = = i+ j
Dt Dt Dt Dt
or, ar = a x iˆ + a y ˆj.
In one dimension, the velocity and the acceleration of an object are always along the same straight line (either in the same
direction or in the opposite direction). However, for motion in two or three dimensions, velocity and acceleration vectors may
have any angle between 0° and 180° between them.
Illustration 1 :
® ®
Given a = 3iˆ + 2jˆ - kˆ and b = ˆi + ˆj + 3kˆ .
® ® ® ®
Determine : (i) a + b and (ii) a - b
® ®
Sol. a + b = (3iˆ + 2jˆ - k) ˆ = 3iˆ + 2ˆj - kˆ + ˆi + ˆj + 3kˆ = 4iˆ + 3jˆ + 2kˆ
ˆ + (iˆ + ˆj + 3k)
® ®
a - b = (3iˆ + 2jˆ - k) ˆ = 3iˆ + 2jˆ - kˆ - ˆi - ˆj - 3kˆ = 2iˆ + ˆj - 4kˆ
ˆ - (iˆ + ˆj + 3k)

Illustration 2 :
→ ®
Given : B = 2iˆ + 2jˆ . Find out magnitude and direction of B .
Sol. Bx = 2, By = 2
®
B = (2)2 + (2) 2 = 2 2
®
If B makes Ð f with x-axis, then
By 2
tan f = = = 1 \ Ðf = 45°
Bx 2
Motion in a Plane 71

Illustration 3 :
r r ® ® ® ® ® ®
If A = 4 ˆi – 3 ˆj and B = 6iˆ + 8 ˆj then, obtain the scalar magnitude and directions from x axis of A, B , A + B and A - B .
®
Sol. Magnitude of A = (4)2 + (-3)2 = 5
® 3 æ 3ö
Direction of A , tan q = - \ q = tan -1 ç - ÷
4 è 4ø
®
Magnitude of B = 62 + 82 = 10
® 8 4 æ4ö
Direction of B tan q = = \ q = tan -1 ç ÷ = 53°
6 3 è3ø
® ®
A + B = 4 ˆi - 3jˆ + 6 ˆi + 8 ˆj = 10 ˆi + 5 ˆj
5 1
tan q = = = 0.5 or q = tan –1(1/ 2)
10 2
® ®
Magnitude and direction of A + B

® ®
| A + B | = (10)2 + (5)2 = 11.2 q = tan–1 (0.5) = 26.5° approx.
® ®
ˆ = 4iˆ - 6iˆ - 3jˆ - 8jˆ = – 2iˆ -11jˆ
A - B = 4iˆ - 3jˆ - (6iˆ + 8j)
® ®
Magnitude and direction of A – B
® ®
| A - B | = (-2) 2 + (-11)2 = 4 + 121 = 5 5

(-11) 11 æ 11ö
tan q = = \ q = tan -1 ç ÷
(-2) 2 è 2ø

MOTION IN A PLANE WITH CONSTANT ACCELERATION

If an object is moving in a plane with constant acceleration a = |a| = ax2 + a 2y and its position vector at time t = 0 is r0, then at any
other time t, it will be at a point given by :
1 2
r = r0 + v0t + at
2
And its velocity is given by : v = v0 + at where v0 is the velocity at time t = 0
In component form :
1 1
x = x0 + vox t + axt2 and y = y0 + voy t + a t2
2 2 y
vx = vox + axt and vy = voy + ayt Q

RELATIVE VELOCITY IN TWO DIMENSIONS R Q


VA
The relative velocity of an object A with respect to the object B
r r VA
moving with velocity v A and v B is represented by,, 180 - q b
q Oq
r r r r r O P' P
v AB = { v A + (– v B)} = ( v A – v B) VB P - VB
The magnitude of relative velocity of the object A with respect to object B is given as,

v AB = v A2 + vB 2 + 2v A .v B cos(180 - q) = v A2 + vB 2 - 2v A .vB cos q


If vAB makes angle q with vA, then,
tan b = vB sin(180 - q) / {v A + vB cos(180 - q) = vB sin q / (v A - vB cos q)
72 Physics
Illustration 4 :
A particle starts from the origin at t = 0 s with a velocity of 10.0 ĵ m/s and moves in the xy-plane with a constant acceleration
of (8.0iˆ + 2.0jˆ) m/s2
(a) At what time is the x-coordinate of the particle is 16 m? What is the y-coordinate of the particle at the time ?
(b) What is the speed of the particle at the time ?
Sol. (a) Given : ux = 0; uy = 10 m/s
and a x = 8.0 m/s2; ay = 2.0 m/s2
Let at time t the x-coordinate is 16 m. We have
1 1
x = ux + a t2 or 16 = 0 + ´ 8 ´ t 2
2 x 2
After simplifying, t = 2s
The y coordinate at t = 2s is
1 1
y = uyt + ay t2 =10 × 2 + × 2 × 22 = 24 m
2 2
(b) The velocity vx = ux + axt = 0 + 8 × 2 = 16.0 m/s
and vy = uy + ayt = 10 + 2 × 2 = 14 m/s
The speed att = 2 Is
v= vx2 + v 2y = 162 + 142 =21.26 m/s.

Illustration 5 :
Rain is falling vertically with a speed of 21 ms–1 (approx). A man rides a bicycle with a speed of 12 ms–1 in east to west direction.
Find the direction in which he should hold his umbrella?
Sol. The velocity of rain as experienced by man is the velocity of rain relative to the velocity of bicycle. vb
N
r r r
i.e., v rb = v r - v b
q
r W E vr vrb
v 12 1 1
From fig, tan q = r b = = = or, q = 30°
v r 21 1.73 3
S –vb
Therefore, man should hold his umbrella at an angle of about 30° with the vertical towards the west.
PROJECTILE MOTION
An object that is in flight after being projected is called a projectile. If an object is projected with initial velocity v0 making an angle
q0 with x-axis and if we assume its initial position to coincide with the origin of the coordinate system, then the position and velocity
of the projectile at time t are given by :
x = (v0 cos q0) t
y = (v0 sin q0) t – (1/2) g t2 y
vy = 0
vx = vox = v0 cos q0 v = v0x i
vy = v0 sin q0 – g t q = 0
The path of a projectile is parabolic and is given by : v
vy j
gx 2 v0x i
y = (tan q 0 ) x - 2 v0x i v
2(v cos q )
0 0 v j
y
The maximum height (hm) that a projectile attains is v0
2 2 v0y j
(v0 sin q0 ) 2 n 0 sin q0
hm = = q0 v0x i
2g 2g
The time taken to reach this height is O q = – q0 x
v0x i
v0 sin q0 vy j = –v0y j v
tm =
g
And total time of flight (tf )
2v0 sin q0
tf =
g
The horizontal distance travelled by a projectile from its initial position to the position it passes y = 0 during its fall is called the range.
R of the projectile.
v02
Horizontal range, R = sin 2q0
g
Motion in a Plane 73

UNIFORM CIRCULAR MOTION


When a point object is moving on a circular path with a constant speed, i.e. it covers equal distances on the circumference of the
circle in equal intervals of time, then the motion of the object is said to be a uniform circular motion.
Let us consider, a point object moving along a circular path.
For uniform circular motion, when the object completes one revolution, the angle traced at its axis of circular motion is 2p radians,
which implies, when t = T, q = 2p radians
\ Angular velocity w = q/ t = 2p/T = 2pn (since T = 1/n) where n = frequency of a point object, T = time period,
Relation between linear velocity and angular velocity: v = Rw where R is the radius of circular path.
Angular acceleration of an object in circular motion is defined as the time rate of change of its angular velocity.
Dw d w
If Dw be the change in angular velocity of the object between the time interval Dt, the angular acceleration a = Lt =
Dt ® 0 Dt dt
The S.I. unit of angular acceleration is rad s–2
r r ur
Relation between linear acceleration and angular acceleration: a = α× R
The acceleration of an object moving with speed v in a circle of radius R has a magnitude v2/R and is always directed towards the
centre. This acceleration is called centripetal acceleration ac.
v2 2 2 4p 2 R
Centripetal acceleration ac = = w R = R ( 2 pv ) = 4 p 2 R n 2 =
R T2
Illustration 6 :
A bullet is fired at an angle 30° with the horizontal hits the ground 3 km away. By adjusting its angle of projection, can one
hope to hit a target 5 km away? Assume the muzzle speed to be fixed and neglect air resistance.
2q
Sol. Horizontal range, R = u2 sin
g
Initially, R = 3 km = 3000 m and q = 30°
u 2 sin 60° u2 3000 3000 ´ 2
3000 = Þ = =
g g sin 60° 3
Maximum horizontal range, Rmax = u2 /g = 2000 Ö3 = 3464 m = 3.4 km
The bullet can never hit a target 5 km away by changing the angle of projection.
Illustration 7 :
A helicopter on a flood relief mission flying horizontally with a speed u at an altitude h, has to drop a food packet for a victim
standing on the ground. At what distance from the victim should the food packet be dropped?
Sol. In fig. H represents position of the helicopter and V that of the victim. For vertical motion
of the packet H u
1 2 2h
h = 0 + gt or t =
2 g
Horizontal distance covered by the food packed in time t,
2h h D
x = ut = u
g
The distance of the point of projection from the food packet is
2 v
2 2u h x
D = h2 + x 2 = h + .
g
Illustration 8 :
A body of mass 2 kg is attached to a string 3 m long and then whirled round in a horizontal circle making 60 revolutions per
minute. Find (i) the angular velocity and (iii) the linear velocity (ii) the centripetal acceleration.
60 ´ 2p
Sol. (i) Angular velocity, w = 60 rev min–1 = = 2p rad s–1
60 rad s -1
(ii) Linear velocity, v = rw = 2p × 3 ms–1 = 6p ms–1
v2 (6p)2 12p 2
(iii) Centripetal acceleration = = = = 118.3 ms–2
r 3ms -2 60 ms -2
74 Physics
Motion in a Plane 75

Textbook Exercises
4.1 State, for each of the following physical quantities, if it is Sol. (a) True, because magnitude is pure number
a scalar or a vector : volume, mass, speed, acceleration, (b) False, because each component is a vector.
density, number of moles, velocity, angular frequency, (c) True only, if the particle moves along a straight line in
displacement, angular velocity. the same direction, otherwise false.
Sol. Scalars: Volume, mass, speed, density, number of moles, (d) True, because the total path length is either greater
angular frequency.
than or equal to the magnitude of the displacement
Vectors: Acceleration, velocity, displacement, angular
vector.
velocity.
4.2 Pick out the two scalar quantities in the following lists: (e) True, as they cannot represent the three sides of a
force, angular momentum, work, current, linear triangle taken in the same order.
momentum, electric field, average velocity, magnetic 4.6 Establish the following vector inequalities geometrically
moment, relative velocity. or otherwise:
Sol. Work and current are the scalar qualities in the given list. r r r r
4.3 Pick out the only vector quantity in the following lists: (a) A + B £ A + B
temperature, pressure, impulse, time, power, total path- r r r r
length, energy, gravitational potential, coefficient of (b) A+ B ³ A - B
friction, charge.
Sol. Since, Impulse = change in momentum = force x time. As r r r r
(c) A- B £ A + B
momentum and force are vector quantities, hence impulse
is a vector quality. r r r r
4.4 State with reasons, whether the following algebraic (d) A- B ³ A - B
operations with scalar and vector physical quantities are
When does the equality sign above apply?
meaningful. ur ur
(a) adding any two scalars, (b) adding a scalar to a vector of Sol. Consider two vectors A and B be represented by the
the same dimensions, (c) multiplying any vector by any uuur uuur
scalar, (d) multiplying any two scalars, (e) adding any two sides OP and OQ of a parallelogram OPSQ. According to
vectors, (f) adding a component of a vector to the same ur ur
vector. parallelogram law of vector addition, ( A + B ) will be
uuur
Sol. (a) No, because only the scalars of same dimensions can represented by OS as shown in the adjoining figure.
be added. ur ur ur ur
(b) No, because a scalar cannot be added to a vector. Thus, OP = | A |, OQ = PS = | B |and OS = | A + B |.
(c) Yes, multiplying a vector with a scalar gives the scalar ur ur ur ur
(number) times the vector quantity which makes sense (a) To prove | A + B | £ | A | + | B |
and one gets a bigger vector. For example, when
r Q S
acceleration A is multiplied by mass m, we get a force
r r A +B
F = m A.
(d) Yes, two scalars multiplied yield a meaningful result, B
for example multiplication of rise in temperature of O A
water and its mass gives the amount of heat absorbed P
by that mass of water.
–B A –B
(e) No, because the two vectors of same dimensions can
be added.
(f) Yes, because both are vectors of the same dimensions. T R
4.5 Read each statement below carefully and state with reasons,
if it is true or false: We know that the length of one side of triangle is
(a) The magnitude of a vector is always a scalar always less than the sum of the lengths of the other
(b) Each component of a vector is always a scalar. two sides. Hence from DOPS, we have
(c) The total path length is always equal to the magnitude OS < OP + PS or, OS < OP + OQ or,
of the displacement vector of a particle. ur ur ur ur
(d) The average speed of a particle (defined as total path | A + B |£| A |+| B | …… (1)
length divided by the time taken to cover the path) is ur ur
either greater or equal to the magnitude of average If the two vectors A and B are acting along the
velocity of the particle over the same interval of time. same straight line and in the same direction then,
ur ur ur ur
(e) Three vectors not lying in plane can never add up to | A + B |=| A |+| B | …… (2)
give a null vector.
76 Physics
ur ur ur ur ur
(b) | A + B | ³ | | A | – | B || (c) The magnitude of A can never be greater than the
From DOPS, we have, ur ur ur
sum of the magnitudes of B , C and D .
OS + PS > OP or, OS > | OP – PS | or, ur ur ur ur ur
OS > | OP – OQ | …… (3) (d) B + C must lie in the plane of A + D , if A and D
(Since PS = OQ) ur uur
uur uur are not collinear and in the line of A and D , if they
The modulus of (OS - PS ) has been taken because are collinear.
the LHS is always positive but the RHS may be ur ur ur ur
Sol. (a) False, because A + B + C + D can be zero in many
negative if OP < PS. Thus from (3) we have, ur ur ur ur
ur ur ur ur ways other than A , B , C and D must each be null
| A + B | > | | A | – | B || …… (4)
ur ur vector.
If the two vectors A and B are acting along a straight ur ur ur ur ur ur
(b) True, since A + B + C + D = 0, therefore, A + C
line in opposite directions, then ur ur ur ur ur ur
ur ur ur ur = – ( B + D ) or, | A + C | = | B + D |
| A + B | = | | A | – | B || …… (5) ur ur ur ur
Considering (4) and (5) together, we get, (c) True, since A + B + C + D = 0, therefore,
ur ur ur ur ur ur ur ur
| A + B | ³ | | A | – | B || A =– ( B + C + D )
ur ur ur ur ur
(c) | A – B | £ | A | + | B | It means the magnitude of A is equal to the
ur ur ur ur ur ur ur
In Fig, | A | = OP and | – B | = OT = PR and ( A - B ) magnitude of vector ( B + C + D ). Since the sum of
ur ur ur
= OR the magnitude of B , C and D may be equal or
From DOPS we note that OR < OP + PR. ur
ur ur ur ur greater than the magnitude of A , hence the magnitude
or | A – B | < | | A | + | – B || ur
ur ur ur ur of A can never be greater than the sum of the
or, | A – B | < | | A | + | B || …… (6) ur ur ur
If the two vectors are acting along the straight line magnitude of B , C and D .
but in opposite direction, then, ur ur ur ur ur ur ur
ur ur ur ur (d) True, since A + B + C + D = 0, therefore, A + ( B + C )
| A – B | = | | A | + | B || …… (7) ur ur
+ D = 0.The resultant sum of the three vectors A ,
Considering (6) and (7) together, we get, ur ur ur ur ur
ur ur ur ur ( B + C ), D can be zero only if ( B + C ) lies in the
| A – B |£| A |+ | B | ur
ur
ur ur ur ur plane of A and D and these three vectors are
(d) | A – B | ³ | | A | – | B ||
represented by the three sides of the triangle taken in
In Fig, from DOPS we have, ur ur ur ur
OR + PR > OP or, OR > | OP – PR | one order. If A and D are collinear, then ( B + D )
or, OR > | OP – OT | …… (8) ur ur
(Since OT = PR) must be in the line of A and D , only then vector sum
uuur uuur of all the vectors will be zero.
The modulus of (OP - OT ) has been taken because 4.8 Three girls skating on a circular ice ground of radius
LHS is positive and RHS may be negative if 200 m start from a point P on the edge of the ground and
OP < OT. reach a point Q diametrically opposite to P following
From (8), different paths as shown in Fig. What is the magnitude of
ur ur ur ur the displacement vector for each? For which girl is this
| A – B |> | A |– | B | …… (9) equal to the actual length of path skate?
ur ur
If the two vectors A and B are along the same straight Q
line in the same direction then,
ur ur ur ur
| A – B |= | A |– | B | …… (10)
Considering (9) and (10) together, we get,
ur ur ur ur B
| A – B | ³ | | A | – | B || A
ur ur ur ur
4.7 Given A + B + C + D = 0, which of the following
statements are correct?
(a) A, B, C and D must be a null vector
ur ur
(b) The magnitude of A + C equals the magnitude of P
ur ur uuur
B + D. Sol. Displacement for each girl = PQ .
Motion in a Plane 77

\ Magnitude of the displacement for each girl Let the motorist start from A.
= PQ = diameter of circular ice ground = 2 × 200 = 400 m. Third turn
For Girl B, the magnitude of displacement The motorcyclist will take the 3rd turn at D. Displacement
= length of path skate. vector at D = AD
4.9 A cyclist starts from the centre O of a circular park of Magnitude of this displacement = 500 + 500 = 1000 m
radius 1 km, reaches the edge P of the park, then cycles Total path length from A to D = AB + BC + CD
along the circumference, and returns to the centre along = 500 + 500 + 500 = 1500 m
QO as shwon in Fig. If the round trip takes 10 min, what is Sixth turn
the (a) net displacement, (b) average velocity, and (c) The motorcyclist will take the 6th turn at A.
average speed of the cyclist. \ Displacement vector is null vector.
Magnitude of this displacement = 500 + 500 = 1000 m
Q Total path length = AB + BC + CD + DE + EF
= 500 + 500 + 500 + 500 + 500 + 500 = 3000 m
Eighth turn
The motorcyclist takes the 8th turn at C.
\ Displacement vector = AC, which is represented by
O the diagonal of the parallelogram ABCG .
\ [(500)2 + (500)2 + 2 ´ (500) ´ (500) cos 60°]
2 2
= [(500) + (500) + 250000] = 866.03m
tan b = 500 sin 60°/{500 + 500 cos 60°]
Sol. (a) Since both the initial and final positions are the same
therefore the net displacement is zero. = (500 3 / 2) /{500(1 + 1/ 2)} = 1/ 3
(b) Average velocity is the ratio of net displacement and = tan 30° or, b = 30°
total time taken. Since the net displacement is zero 4.11 A passenger arriving in a new town wishes to go from the
therefore the average velocity is also zero. station to a hotel located 10 km away on a straight road
distance covered from the station. A dishonest cabman takes him along a
(c) Average speed =
time taken circuitous path 23 km long and reaches the hotel in
OP + Actual distance PQ + QO 28 minutes. What is (a) the average speed of the taxi, (b)
= the magnitude of average velocity? Are the two equal?
10 minute
Sol. Here, actual path length travelled, s = 23 km, Displacement
1 = 10 km, Time taken, t = 28 min = 26/60 h.
1 km + ´ 2p ´ 1 km + 1 km
4 (a) Average speed of the taxi = actual path length/ time
=
10 taken = 23/28/60 = 49.3km/h
60h (b) Magnitude of average velocity
= displacement/ time taken = 10/(28/60) = 21.4 km/h
æ 22 ö -1 50 The, average speed is not equal to the magnitude of
= 6 ç 2 + ÷ km h = 6 ´ = 21.43 km h–1.
è 14 ø 14 average velocity. The two are equal for the motion of
4.10 On an open ground, a motorist follows a track that turns taxi along a straight path in one direction.
to his left by an angle of 60° after every 500 m. Starting 4.12 Rain is falling vertically with a speed of 30 ms–1. A woman
from a given turn, specify the displacement of the motorist rides a bicycle with a speed of 10 ms–1 in the north to
at the third, sixth and eighth turn. Compare the magnitude south direction. What is the direction in which she should
of the displacement with the total path length covered by hold her umbrella ?
the motorist in each case. Sol. The rain is falling along OA with speed 30 ms–1 and woman
Sol. In this question, the path is a regular hexagon ABCDEF of rider is moving along OS with 10 ms–1. i.e. OA = 30 ms–1
side length 500 m. In Fig, and OB = 10 ms–1.

E 60°
Speed rain

D
60°

60° q
F G C O Speed woman
C N S
B
60° q
60° 60° 500 m
b
D A
A B
500 m
78 Physics
The woman rider can protect herself from the rain if she
51sin135° 51sin 45°
holds her umbrella in the direction of relative velocity of tan b = 72 + 51cos135° = 72 + 51( - cos 45°)
rain w.r.t. woman. To do so apply equal and opposite
velocity of woman on the rain i.e. impress the velocity
10 ms–1due north on rain which is represented by OC. æ 1 ö
51 ´ ç
Now the relative velocity of rain w.r.t. woman will be è 2 ÷ø
represented by diagonal OD of parallelogram OADC. = = 1.0039
æ 1 ö
If ÐAOD = q, then in DOAD, 72 - 51 ç
è 2 ÷ø
tan q = AD/OA = OC/OA = 10/30 = 0.3333 = tan18° 26 '
b = 18°26¢ with vertical in forward direction. \ b = tan–1 (1.0039) = 45.1°
4.13 A man can swim with a speed of 4.0 km/h in still water. Angle w.r.t. east direction = 45.1° – 45° = 0.1°
How long does he take to cross a river 1.0 km wide if the It means the flag will flutter almost due east.
river flows steadily at 3.0 km/h and he makes his strokes 4.15 The ceiling of a long hall is 25m high. What is the
normal to the river current? How far down the river does maximum horizontal distance that a ball thrown with a
he go when he reaches the other bank? speed of 40ms–1 can go without hitting the ceiling of the
Sol. Time to cross the river, t = width of river/ speed of man hall ?
1 Sol. Given, u = 40 ms–1, H = 25 m
= 1km/4 km/h = h = 15 m. Let, q = the angle of projection with the horizontal direction
4
to have the maximum range, with maximum height = 25 m
Distance moved along the river in time
Maximum height, H = 25 = u2 sin2q/ 2g
1 = (40)2 sin2q/ (2 × 9.8)
t = vr × t = 3 km/h × h = 750 m
4 sin q = (25 × 2 × 9.8/402)½ = 0.5534 = sin 33.6°
4.14 In a harbour, wind is blowing at the speed of 72 km/h and or, q = 33.6°
the flag on the mast of a boat anchored in the harbour Horizontal range, R = u2sin 2q/ g = (40)2
flutters along the N-E direction. If the boat starts moving sin (2 × 33.6°)/ 9.8 = 150.5 m
at a speed of 51 km/h to the north, what is the direction of 4.16 A cricketer can throw a ball to a maximum horizontal
the flag on the mast of the boat. distance of 100m. How much high above the ground can
Sol. When the boat is anchored in the harbour, the flag flutters the cricketer throw the same ball?
along the N-E direction. It shows that the velocity of wind Sol. Let u = velocity of projection of the ball. The ball will cover
is along the north-east direction. When the boat starts maximum horizontal distance when angle of projection with
moving, the flag will flutter along the direction of relative horizontal, q = 45°. Then Rmax = u2/g
r Now, 100 = u2/g
velocity of wind w.r.t. boat. Let vwb be the relative velocity
r r In order to study the motion of the ball along vertical
of wind w.r.t. boat and b be the angle between vwb and vw . direction, consider a point on the surface of earth as the
(see Fig. below) origin and vertical upward direction as the positive direction
r r r of Y axis. Taking motion of the ball along vertical upward
Now, vwb = vw + (– vb )
direction, we get, uy = u, ay = – g, vy = 0,
we know, vy = uy + ayt
N Þ 0 = u + (–g)t or, t = u/g
Again, y = uyt + ½ ayt2
NE
vb \ y = u (u/g)+ ½ (–g)u2/g2 = u2/g – ½ u2/g
vw = u2/g =100/2 = 50 m
45° 4.17 A stone tied to the end of a string 80cm long is whirled in
b
W E a horizontal circle with a constant speed. If the stone makes
O
14 revolutions in 25 s what is the magnitude and direction
136°
of acceleration of the stone?
–vb vwb
135°

Sol. Given, r = 80 cm = 0.8 m, n =14/25 s–1,


w = 2pn = 2 × 22/7 × 14/25 = 88/25 rad.s–1
Centripetal acceleration, a = v2/r = (250)2/1000
S Again, a/g = (250)2/1000 × 1/9.8 = 6.38
4.18 An aircraft executes a horizontal loop of radius 1.00 km
r
Here, | vw | = 72 km/h with a steady speed of 900 km/h. Compare its centripetal
r acceleration with the acceleration due to gravity.
| – vb | = 51 km/h Sol. Here r = 1 km = 103 m,
r r
Angle between vw and - vb is 135° i.e., 5
v = 900 km h–1 = 900 × = 250 ms–1
q = 135°. Then 18
Motion in a Plane 79

Integrating the above relation with the change of time,


v 2 (250) 2 from 0 to t, velocity changes from u to v, we get,
Centripetal acceleration = ac = = = 62.5 ms–2
r 103 ur uur
v - u = (8.0iˆ + 2.0 ˆj )t or,
ac 62.5 ur uur
Now, = = 6.38.
g 9.8 v = u + (8.0iˆ + 2.0 ˆj )t
4.19 Read each statement below carefully and state, with ur d ur ur
reasons, if it is true or false : v = (r ) or, dr = v dt
dt
(a) The net acceleration of a particle in circular motion uur
is always along the radius of the circle towards the so, dr = (u + 8.0tiˆ + 2.0tjˆ ) dt
centre. Integrating the above relation with the change of time, from
(b) The velocity vector of a particle at a point is always 0 to t, displacement changes from 0 to r, we get,
ur r
along the tangent to the path of the particle at that r = (ut + 1 2 8.0t 2iˆ + 1 2 2.0t 2 ˆj )
point.
(c) The acceleration vector of a particle in uniform or xiˆ + yjˆ = 10tjˆ + 4t 2iˆ + t 2 ˆj
circular motion averaged over one cycle is a null
vector. = 4t 2 iˆ + (10t + t 2 ) ˆj
Sol. (a) False, the net acceleration of a particle in circular motion Here, it is given, x = 4t2 and y = 10t + t2
is along the radius of the circle towards the centre
1
only in uniform circular motion. æ xö 2
(b) True, because while leaving the circular path, the Þ t = ç ÷ = 2 s.
è 4ø
particle moves tangentially to the circular path.
(c) True, the direction of acceleration vector in a uniform 1
circular motion is directed towards the centre of æ 16 ö 2
circular path. It is constantly changing with time. The (a) At x = 16 m, t = ç ÷ = 2 s. and,
è 4ø
resultant of all these vectors will be a zero vector.
4.20 The position of a particle is given by y = 10 × 2 + 22 = 24 m
r (b) Velocity of the particle at time t is
r = 3.0t$i - 2.0 t 2 $j + 4.0 $
k m, where t is in seconds and the
r v = 10 ˆj + 8tiˆ + 2tjˆ , when, t = 2 s, then,
coefficients have the proper units for r to be in meters.
r v = 10 ˆj + 8 ´ 2tiˆ + 2 ´ 2tjˆ = 16iˆ + 14 ˆj
(a) find the v and a of the particle? (b) what is the
magnitude and direction of velocity of the particle at ur 2 2 -1
t = 2.0 s? Speed = | v | = 16 + 14 = 21.26 ms
r
Sol. (a) Velocity, v = dr/dt 4.22 iˆ and ĵ are unit vectors along x and y axis repectively..
d What is the magnitude and direction of the vectors
= (3.0t iˆ - 2.0t 2 ˆj + 4.0t 2 kˆ) = 3.0iˆ - 4.0 t ˆj
dt
iˆ + ˆj and iˆ - ˆj ? What are the components of a vector
r d
Acceleration, a = dv/dt = (3.0 iˆ - 4.0t ˆj) = 4.0 ˆj
dt A = 2iˆ + 3jˆ along the direction of iˆ + ˆj and iˆ - ˆj ?
(b) At time t = 2s,
r Sol. (a) Magnitude of (iˆ + ˆj ) = i + ˆj = (1) 2 + (1) 2 ) = 2
v = 3.0iˆ - 4.0× 2 ˆj = 3.0iˆ - 8.0 ˆj
v = [ ( 3) + ( -8) ] ms -1 = 73 = 8.54 ms -1
2 2
Let the vector (iˆ + ˆj ) make an angle q with the
If q is the angle which v makes with x-axis, then
tan q = vy / vx = –8/3 = –2.667 = tan 69.5°. direction of iˆ , then
\ q = 69.5° below the x-axis. cos q = (iˆ + ˆj ).i/ iˆ + ˆj .i = 1/( 2) (1) = 1/ 2
4.21 A particle starts from the origin at t = 0 s with a velocity of
= cos 45° or, θ = 45°
10.0 ĵ m/s and moves in the x-y plane with a constant
2 2
Magnitude of (iˆ - ˆj ) = iˆ - ˆj = (1) + (-1) = 2
acceleration of 8.0iˆ + 2.0 ˆj m/s–2. (a) At what time is the
x-coordinate of the particle 16 m? What is the y-coordinate Similarly, if vector (iˆ - ˆj ) makes an angle q with the
of the particle at that time? (b) What is the speed of the
particle at that time? direction of iˆ , then
uur
Sol. Given, u = 10.0 ˆjms -1at t = 0 ; cos q = (iˆ - ˆj ).i / iˆ - ˆj .i = 1/( 2 ) (1) = 1/ 2
ur d ur = cos 45° or, q = 45°
a = (v ) = (8.0iˆ + 2.0 ˆj )ms -2
dt Here q = – 45° with iˆ
80 Physics
ur 4.24 Read each statement below carefully and state, with
(b) Now, A = 2iˆ + 3 ˆj reasons and examples, if it is true or false:
ur A scalar quantity is one that
To find the component of A along the vector ( iˆ + ˆj ), (a) is conserved in a process
(b) can never take negative values
we have to find out the unit vector (iˆ + ˆj ) .
(c) must be dimensionless
(d) does not vary from one point to another in space
(iˆ + ˆj ) iˆ + ˆj iˆ + ˆj (e) has the same value for observers with different
then, aˆ = =
orientations of axes.
12 +12 2
Sol. (a) False, because kinetic energy is a scalar but does not
ur remain conserved in an inelastic collision.
Magnitude of the component of A along (b) False, because potential energy in a gravitational field
uur may have negative values.
(iˆ + ˆj ) = ( A.a) aˆ = 5/ 2 (i + j )/ 2 = 5/2(i + j)
(c) False, because mass, length, time, speed, work, etc.,
ur all have dimensions.
Therefore, component of A along
uur (d) False, because speed, energy, etc., vary from point to
(iˆ + ˆj ) = ( A.a) aˆ = 5/ 2 (iˆ + ˆj )/ 2 = 5/2(iˆ + ˆj ) point in space.
(e) True, because a scalar quantity will have the same
Let b̂ is unit vector along the direction value for observers with different orientations of axes
since a scalar has no direction of its own.
(iˆ - ˆj ) 4.25 An aircraft is flying at a height of 3400 m above the ground.
If the angle subtended at a ground observation point by the
= (iˆ - ˆj ) / iˆ - ˆj = (iˆ - ˆj ) / (12 + ( -1) 2 ) aircraft positions 10.0 s apart is 30°, what is the speed of
the aircraft?
= (iˆ - ˆj ) / 2 Sol. In Fig, O is the observation point at the ground. A and B are
ur the positions of air craft for which ÐAOB = 30°. Draw a
Therefore, component of A along perpendicular OC on AB. Here OC = 3400 m and ÐAOC =
ur ÐCOB = 15°.
(iˆ - ˆj) will be = ( A.bˆ) bˆ
A C B
= {(2iˆ - 3 ˆj )1/ 2}{1/ 2 (iˆ - ˆj}) {1/ 2 (iˆ - ˆj})

= 1/ 2(2 - 3)1/ 2 (iˆ - ˆj ) = 1/ 2 (iˆ - ˆj )


4.23 For any arbitrary motion in space, which of the following 3400m
15°
relations are true:
30°

æ 1ö
(a) vaverage = ç ÷ [v (tl) + v (t2)]
è 2ø
(b) vaverage = [r (t2) – r (tl)]/(t2 – t1) O
In DAOC, AC = OC tan 15° = 3400 × 0.2679 = 910.86 m.
(c) v (t) = v (0) + a t
AB = AC + CB = AC + AC = 2 AC = 2 × 910.86 m
(d) r (t) = r (0) + v (0) t + (1/2) a t2
distance AB
(e) aaverage = [v (t2) – v (tl)]/(t2 – t1) Speed of the aircraft, v =
time
(The 'average' stands for average of the quantity over the
2 ´ 910.86
time interval t1 to t2) = = 182.17 ms–1 = 182.2 ms–1
10
Sol. (b) and (e) are true; others are false because relations (a),
(c) and (d) hold only for uniform acceleration.

4.26. A vector has magnitude and direction. Does it have a (ii) A vector can vary with time. As an example, velocity
location in the space? Can it vary with time? Will two and acceleration vectors may vary with time.
r r r
equal vectors a and b at different locations in space r
(iii) Two equal vectors a and b having different locations
necessarily have identical physical effects? Give examples may not have same physical effect. As an example,
in support of your answer. two balls thrown with the same force, one from earth
Sol. (i) Besides having magnitude and direction, each vector and the other from moon will attain different 'maximum
has also a location in space. heights'.
Motion in a Plane 81

4.27 A vector has both magnitude and direction. Does it mean Sol. Velocity of plane,
that anything that has magnitude and direction is 5
necessarily a vector? The rotation of a body can be specified vp = 720 ´ ms–l = 200 ms –l
18
by the direction of the axis of rotation and the angle of
rotation about the axis. Does that make any rotation a Velocity of shell = 600 ms–l ;
vector? 200 1
Sol. No. finite rotation of a body about an axis is not a vector sin q = =
600 3
because finite rotations do not obey the laws of vector
addition. æ 1ö
or q = sin –1 ç ÷ = 19.47°
4.28 Can you associate vectors with (a) the length of a wire bent è 3ø
into a loop (b) a plane area (c) a sphere? Explain. This angle is with the vertical.
Sol. (a) We cannot associate a vector with the length of a wire Let h be the required minimum height. Using equation
bent into a loop. This is because the length of the v2 – u2 = 2a s, we get
loop does not have a definite direction. (0)2 – (600 cos q)2 = – 2 × 10 × h
(b) We can associate a vector with a plane area. Such a
vector is called area vector and its direction is 600 ´ 600(1 - sin 2 q)
or, h =
represented by a normal drawn outward to the area. 20
(c) The area of a sphere does not point in any definite æ 1ö 8
direction. However, we can associate a null vector = 30 ´ 600 ç1 - ÷ = ´ 30 ´ 600 m = 16 km.
è 9ø 9
with the area of the sphere. We cannot associate a
vector with the volume of a sphere. 4.31 A cyclist is riding with a speed of 27 km/h. As he approaches
4.29 A bullet fired at an angle of 30° with the horizontal hits a circular turn on the road of radius 80 m, he applies
the ground 3.0 km away. By adjusting its angle of brakes and reduces his speed at the constant rate of 0.50
projection, can one hope to hit a target 5.0 km away? m/s every second. What is the magnitude and direction of
Assume the muzzle speed to be fixed, and neglect air the net acceleration of the cyclist on the circular turn?
resistance. 5
Sol. Here v = 27 km/h = 27 × m/s = 7.5 m/s, r = 80 m
Sol. Here R = 3 km = 3000 m, q = 30°, g = 9.8 m s–2 18
and tangential acceleration at = – 0.50 m/s2
u 2 sin 2q
As range, R = v2 (7.5)2
g \ Centripetal acceleration ac = =
r 80
u 2 sin 2 ´ 30° u 2 sin 60 = 0.70 ms–2 (radially inwards).
Þ 3000 = = Thus, as shown in fig. above, two accelerations are acting
9.8 9.8
r
in mutually perpendicular directions. If a be the resultant
3000 ´ 9.8 acceleration, then
Þ u2 = = 3464 × 9.8
3 r
| a | = at2 + ac2 = (0.5) 2 + (0.7) 2
2
= 0.86 ms–2
u 2 sin 2q¢
Also, R¢ =
g
v
3464 ´ 9.8 ´ sin 2q
Þ 5000 =
9.8
ac
O A
5000
i.e. sin 2q¢ = = 1.44 b
3464
which is impossible because sine of an angle cannot be at
a
more than 1. Thus this target cannot be hoped to be hit.
4.30 A fighter plane flying horizontally at an altitude of 1.5 km
with speed 720 km/h passes directly overhead an anti-
aircraft gun. At what angle from the vertical should the ac 0.7
gun be fired for the shell with muzzle speed 600 ms–1 to and tan b = = = 1.4
at 0.5
hit the plane? At what minimum altitude should
the pilot fly the plane to avoid being hit? (Take g = 10 ms–2) Þ b = tan–1 (1.4) = 54.5° from the direction of negative of
the velocity.
82 Physics
4.32 (a) Show that for a projectile the angle between the
velocity and the x-axis as a function of time is given u 2 sin 2 q u 2 sin 2 q
(b) Since, hmax = and R =
by 2g g
æ voy - gt ö
q (t) = tan -1 ç ÷
è vox ø u 2 sin 2 q
(b) Show that the projection angle q0 for a projectile hmax 2g tan q
Þ = 2 = (As sin 2q = 2 sin q cos q)
launched from the origin is given by R u sin 2q 4
æ 4h ö g
q0 = tan -1 ç m ÷ y
è R ø
where the symbols have their usual meaning.
tan q hmax
Sol. (a) Let a projectile fired at an angle q with x-axis. Þ =
4 R uy
As q depends on t, q (t), at any instant
u oy A q
vy voy - gt 4hmax ux
tan q (t) = = or tan q =
vx vox R
(Since, vy = voy – gt and vx = vox) q(t)
-1 æ 4h ö u ox
-1 æ voy - gt ö or q = tan ç max ÷ . O B X
Þ q (t) = tan ç ÷ è R ø
è vox ø

Practice Questions
ur ur ur
Very Short Answer Questions [1 Mark Qs.] 16. If the magnitude of vectors A, B, C are 3, 4 and 5 units
ur ur ur ur
1. “All the physical quantities having magnitude and direction respectively, and if A + B = C , find the angle between A
can be vectors”. Comment. ur
2. Out of the following, which is not a vector: gravitational and C ?
field, magnetic field, electric potential and magnetic moment. ur ur
17. Wh at would be nature of vectors A an d B i f,
3. Can we add three vectors not lying in the same plane to get ur ur ur ur
a null vector? A+ B = A- B ?
4. Can the scalar product of the vectors be zero even when ur
18. The vector A has x, y and z components of 8, 12 and
neither of them is a zero vector? ur
5. Is pressure a vector? Give reason. – 4 units respectively. Obtain a vector expression for A in
6. Can two vectors of different magnitudes be combined to unit vector notation.
give zero resultant? 19. What is the maximum value of the difference of two vectors
7. State the essential condition for the addition of vectors. r r
a and b ?
8. When is the magnitude of the resultant of two vectors
equal to either of them? 20. Give an example of a physical quantity which a scalar but is
9. A bullet is dropped from a certain height and at the same a resultant of two vectors.
time, another bullet is fired horizontally from the same 21. A boat is moving with a velocity -3.0iˆ - 4.0 ˆj with respect
height. Which one will hit the ground earlier and why?
to the ground. The water in the river is moving with a
10. A stone tied to the end of a string is whirled in a circle. If
the string breaks, the stone flies away tangentially. Why? velocity 3.0iˆ + 4.0 ˆj with respect to the ground. What is
11. For uniform circular motion, does the direction of centripetal the relative velocity of boat with respect to water?
acceleration depend upon the sense of rotation? ur
12. Why does a tennis ball bounce higher on hills than in 22. What is the angle made by vector A = 2.0$i + 2.0$j with
plains? x-axis ?
13. Does a uniform acceleration in two-dimensional motion ur ur ur
necessarily imply motion along a straight-line path? 23. If the magnitude of vectors A, B, C are 12, 5 and 13 units
ur ur ur ur ur ur
14. What is the property of two vectors A and B such that respectively, and if A + B = C , find the angle between A
ur ur ur ur
A + B = C and A – B = C ? and B .
ur ur ur ur
15. Under what condition the magnitude of the sum of two 24. Find the vector C such that 2 A + 7 B + 4 C = 0, if
vectors is equal to the magnitude of difference between ur ur
them. k and B = -3$i + 6$
A = 3$i + 5$j - 2$ k
Motion in a Plane 83

25. What will be the effect on horizontal range of a projectile 7. Is it important in the long jump that how much height you
when its initial velocity is doubled, keeping the angle of take for jumping? What factors determine the span of a
projection identical? jump?
26. What will be the effect on maximum height of a projectile if 8. Show that two dimensional uniform velocity motion is
its angle of projection is changed from 30° to 60° keeping equivalent to two one dimensional uniform velocity motion
the same initial velocity of projection? along two coordinate axes.
27. Is the maximum height attained by projectile is largest when 9. A projectile is fired with a velocity ‘u’ making an angle q
its horizontal range is maximum? with the horizontal. Show that its trajectory is a parabola.
28. If the velocity at the maximum height of a projectile is half 10. What is a projectile? Find its horizontal range and show it
its initial velocity of projection u, find out its range on the is maximum for an angle of 45°.
horizontal plane. 11. Justify the statement that a uniform circular motion is an
accelerated motion.
29. Write down the expression for the equation of trajectory of
12. Explain subtraction of two vectors with suitable
a projectile. illustrations.
30. What is the nature of the trajectory of a particle having a 13. What do you mean by zero vector? How can they be
uniformly accelerated motion in a plane? obtained ? Give examples and properties of zero vectors.
31. If the two bodies are projected at an angle q and (90°– q) to 14. Derive a relation between linear and angular velocity.
the horizontal with the same speed, then find out the ratio 15. Derive a relation between linear and angular acceleration.
of their time of flight. Long Answer Questions [5 Marks Qs.]
32. If the two bodies are projected at the same initial velocity
at angles q and (90°– q) to the horizontal. What is the ratio 1. State triangle law of vector addition. Find analytically the
magnitude and direction of the resultant of two vectors.
of their (i) maximum height attained by them and (ii) of
What will be the resultant when (a) two vectors are parallel
horizontal ranges?
to each other (b) two vectors are opposite to each other
33. In a projectile motion the range R is ‘n’ times that of its and (c) two vectors are perpendicular to each other.
maximum height H, find its angle of projection. 2. Find the relation for the velocity and time, displacement
34. If the greatest height to which a man can throw a body is and time, when an object is moving with uniform
‘h’, what will be the greatest distance up to which he can acceleration in two dimensions.
throw the body? 3. Find the time of flight, maximum height, horizontal range of
35. Define angular speed. What is its SI unit ? projectile projected from ground with speed u and making
36. Two cars are going around two concentric circular paths at an angle q with the horizontal direction.
the same angular speed. Does the inner or the outer car 4. Define centripetal acceleration? Find its magnitude and
have the larger linear speed? direction in case of uniform circular motion.
37. Among the angular velocities of hour hand and that of
earth, which will be greater? Numerical Questions [3 or 5 Marks Qs.]
38. The range of a particle when launched at angle of 15° with 1. A boy throws a ball in air at 60° to the horizontal along a
the horizontal is 1.5 km. What is the range of the particle road with a speed of 10 m/s (36 km/h). Another boy sitting
when launched at angle of 45° with the horizontal. in a passing by car observers the ball. Sketch the motion of
Short Answer Questions [2 or 3 Marks Qs.] the ball as observed by the boy in the car, if car has a speed
of (18 km/h). Give explanation to support your diagram.
1. What do you understand by scalar and vector? Give five 2. A fighter plane is flying horizontally at an altitude of 1.5 km
examples of each. wih speed 720 km/h. At what angle of sight (w.r.t. horizontal)
2. What do you understand by position vector and when the target is seen, should the pilot drop the bomb in
displacement vector. Distinguish them with examples. order to attack the target?
r r r r 3. Two equal forces act at a point. The square of their resultant
3. Do a + b and a - b lie in the same plane. Give reason.
r r is 3 times their product. Find the angle between them.
4. The resultant of two vectors P and Q is perpendicular
r r 4. At what angle two forces (A + B) and (A – B) act so that
to P and its magnitude is half that of Q . What is the angle
resultant is
r r
between P and Q ?
(i) 3A2 + B 2 and (ii) 2( A2 + B 2 )
5. A skilled gunman always keeps his gun slightly tilted above
the line of sight while shooting. Why? 5. The resultant of two equal forces acting at right angles to
6. A ball of mass m is thrown vertically up. Another ballof each other is 1414 N. Find the magnitude of each force.
mass 2 m is thrown at an angle q with the vertical. Both of 6. Two forces whose magnitude are in ratio of 3 : 5 act at an
them remain in air for the same period of time. What is the angle of 60°and have a resultant equal to 35N. Find the
ratio of the heights attained by the two balls? magnitude of the forces.
84 Physics
7. Two forces acting on a particle in opposite directions have uur uur
15. Find the angle between the vectors A and B , where
a resultant of 10 N. If they act at right angles to each other, uur uur
resultant is 50 N. Find the two forces. A = iˆ + 2 ˆj - kˆ and B = -iˆ + ˆj - 2kˆ .
uur uur uur uur uur uur
8. A train is moving with a velocity of 30 km/h due east and a 16. Three vectors A , B , C are such that A = B + C and their
car is moving with a velocity 40 km/h due north. What is the magnitudes are 5, 4, 3 respectively. Find the angle between
velocity of car as appears to a passenger in the train? uur uur
N A and C .

D 40 km/h
q B
A q
O A E C
W C
–30 km/h 30 km/h

S B
9. A force is inclined at 50° to the horizontal. If its rectangular
17. A ball is thrown horizontally from the top of a tower with a
component in the horizontal direction be 50 N, find the
speed of 50 m/s. Find the velocity and position at the end of
magnitude of the force and its vertical component.
3 second. [g = 9.8m/s2]
10. One of the rectangular components of a force of 50 N is 25
18. A ball is projected from ground with a velocity of 19.6 m/s at
N. Find the angle it makes with this component and
an angle of 30° with the horizontal. Calculate its time of
magnitude of other component.
flight and horizontal distance it travels.
11. Calculate the vector which has to be added to the resultant
ur ur 19. A particle is projected with a velocity u so that its horizontal
of A = 2iˆ - 4 ˆj - 6kˆ and B = 4iˆ - 3 ˆj - 3kˆ gives unit vector range is twice the greatest height attained. Find the
along z-direction. horizontal range of it.
12. Find a unit vector parallel to the resultant of the vector 20. Calculate the angular velocity of the minute’s hand of a
ur ur
A = 2iˆ - 4 ˆj - 2kˆ and B = 3iˆ - ˆj + kˆ clock.
uur uur 21. The angular velocity of a particle moving in a circle of radius
13. Given A = 2iˆ + 5 ˆj - 4kˆ and B = -3iˆ + 2 ˆj + kˆ . 50 m is increased in 5 minutes from 100 revolutions per
uur uur uur uur
Find C such that A + B - C = 0 minute to 400 revolution per minute. Find angular
uur uur uur acceleration.
14. Vectors A = 3iˆ + 5 ˆj + 5kˆ and B = iˆ - 6 ˆj. Find C such that
uur uur uur 22. A body is moving in a circle of radius 100 cm with a time
2 A + 6B + 5C = 0 period of 2s. Find the acceleration.

HOTS/Exemplar Questions
Very Short Answer Questions [1 Mark Qs.] y

1. Whey you toss a ball upward, by how much does its upward
speed decrease each second? [HOTS] B
2. Consider a batted baseball following a parabolic path on a C
day when the sun is directly overhead. How does the speed
H .
of ten ball’s shadow across the field compare with the ball’s
horizontal component of velocity? [HOTS] A
3. The boy on the tower throws a ball 20 meters downrange as
shown. What is his pitching speed? [HOTS] x
Short Answer Questions [2 or 3 Marks Qs.]
5m 1. Someone standing at the edge of a cliff throws a ball straight
up at a certain speed and another ball straight down with
the same initial speed. If air resistance is negligible, which
20 m ball has the greater speed when it strikes the ground below?
4. A football is kicked into the air vertically upwards. What is [HOTS]
its (a) acceleration, and (b) velocity at the highest point? 2. How does the vertical distance a projectile falls below an
[Exemplar] otherwise straight-line path compare with the vertical
5. A particle is projected in air at some angle to the horizontal, distance it would fall from rest in the same time? [HOTS]
moves along parabola as shown in Fig. where x and y 3. A fighter plane is flying horizontally at an altitude of 1.5 km
indicate horizontal and vertical directions, respectively. with speed 720 km/h. At what angle of sight (w.r.t. horizontal)
Show in the diagram, direction of velocity and acceleration when the target is seen, should the pilot drop the bomb in
at points A, B and C. [Exemplar] order to attack the target? [Exemplar]
Motion in a Plane 85

Long Answer Questions [5 Marks Qs.] speed of 2 m/s; so that its distance from the hill can be
adjusted. What is the shortest time in which a packet can
1. A stair case contains three steps, each 10cm high and 20cm reach on the ground across the hill? Take g = 10 m/s2.
wide. What should be the minimum horizontal velocity of a [Exemplar]
ball rolling off the uppermost plane so as to hit directly the 4. A river is flowing due east with a speed 3 m/s. A swimmer
lowest plane (take g = 10m/s) [HOTS] can swin in still water at a speed of 4 m/s (Fig.).
N
E

B
2. A small body of mass m slides without friction from the top
of the hemispherical cup of radius R as shown in the 3 m/s
following figure. If it leaves the surface of the cup at a vertical
distance h below the highest point, then find a relation
between h and R. [HOTS] A

h (a) If swimmer starts swimming due north, what will be his


Q
resultant velocity (magnitude and direction)?
(b) If he wants to start from point A on south bank and
reach opposite point B on north bank.
R (i) which direction should he swim?
3. A hill is 500 m high. Supplies are to be sent across the hill (ii) what will be his resultant speed?
using a canon that can hurl packets at a speed of 125 m/s (c) From two different cases as mentioned in (a) and (b)
over the hill. The canon is located at a distance of 800 m above, in which case will he reach opposite bank in
from the foot of hill and can be moved on the ground at the shorter time. [Exemplar]
86 Physics

CHAPTER TEST
Time : 45 min. Max. Marks : 24

Directions : (i) Attempt all questions


(ii) Questions 1 to 5 carry 1 mark each.
(iii) Questions 6, 7 and 8 carry 3 marks each.
(iv) Questions 9 and 10 carry 5 marks each.

1. A body is moving on a curved path with a constant speed. What is the nature of its acceleration?
2. Under what condition the three vectors cannot have zero resultant?
uur
3. Find a vector A and its magnitude with initial point P (1, 2, –1) and terminal point Q (3, 2, 2).
4. Define centripetal force.
5. Define uniform circular motion.
6. If both the speed of a body and radius of its circular path are doubled, what will happen to the centripetal acceleration?
7. What are the two angles of projection of a projectile projected with velocity 30 m/s, so that the horizontal range is 45 m. [Take
g = 10 m/s2]
8. Two forces 5 kg wt. and 10 kg wt. are acting with an inclination of 120° between them. What is the angle which the resultant
makes with 10 kg wt.?
D C
9. A man wants to reach from A to the opposite corner of the square C figure. The sides of the
Q
square are 100 m. A central square of 50 m × 50 m is filled with sand. Outside this square, he
can walk at a speed 1 m/s. In the central square, he can walk only at a speed of v m/s (v < 1).
What is smallest value of v for which he can reach faster via a straight path through the sand
than any path in the square outside the sand ? P
50 mm R
A B
r r 100 mm
10. State parallelogram law of vector addition. Show that resultant of two vectors A and B inclined at an angle q is
R= A2 + B 2 + 2 AB cos q.
Motion in a Plane 87

Solutions
ur ur
PRACTICE QUESTIONS 16. Here, A2 + B2 = C2 therefore, angle between A and B is
ur ur
Very Short Answer Questions 90°. Let, q is the angle between the B and C , then cos q =
1. No, In addition of these two characteristics, quantities must 3/5 or, q = cos–1(3/5)
ur ur ur ur ur ur
follow the laws of vector addition and satisfy the 17. Here, A + B = A - B or, B = 0 i.e. B is a null vector..
commutative property of vector addition so that it can be ur
classified as a vector. e.g. angular displacement of a body. 18. A = 8 iˆ + 12 ĵ – 4 k̂
r r
2. Electric potential 19. The magnitude c of ( a – b ) is,
3. No. c2 = a2 + b2 + 2ab cos (p – q) = a2 + b2 – 2ab cos q
4. Yes, if the angle between the vectors is 90°. c is maximum when cos q is –1, i.e. cmax = a + b.
ur
5. No, pressure is always taken to be normal to the plane of 20. Work, which is a scalar product of two vectors force F and
the area on which it is acting. As this direction is unique, it r
displacement S .
does not need any specification. So pressure is not a vector. ur ur
6. No, two vectors of different magnitudes cannot be W = F. S
combined to give zero resultant. 21. Here, velocity of boat with respect to ground,
r
7. The essential condition for the addition of vectors is that v B = –3.0i$ – 4.0 $j
they must represent the physical quantities of same nature. Velocity of water with respective to ground,
8. When two vectors of equal magnitude are inclined to each r
vW = 3.0iˆ + 4.0 ˆj
other at an angle of 120°, the magnitude of their resultant is
equal to that of the either vector. Relative velocity of boat with respect to water, is
r r r
a2 = a2 + a2 + 2a2 cos2 q v BW = (v B - v W ) = –3.0 iˆ – 4.0 $j - (3.0$i + 4.0 ˆj )

1 vBW = - 6.0iˆ - 8.0 ˆj


Þ cos q = – Þ q = 120°
2 ur
22. Here, Ax = 2, Ay = 2. Let q be the angle which A makes with
9. Since the heights of both bullets from the ground is the x-axis then,
same, so the time taken by both of them to reach the ground tan q = Ax / Ay = 2/2 = 1, q = 45°
will be the same. 23. Here, A2 + B2 = C2 i.e (12)2 + 52 = (13)2 therefore, angle
ur ur
10. The instantaneous velocity of the stone going around the between A and B is 90°.
circular path is always along tangent to the circle. When ur ur ur
the string breaks, the centripetal force ceases. Due to inertia, 24. Since, 2 A + 7B + 4C = 0 ,
the stone continues its motion along tangent to the circular ur 1
therefore, C = – (2A + 7B)
path. 4
11. The direction of centripetal acceleration does not depend 1
= – {2( 3iˆ + 5jˆ - 2kˆ ) + 7( -3jˆ + 6kˆ )}
on the clockwise or anticlockwise sense of rotation of the 4
body. It always acts along the radius towards the centre of ur 1
C = - {6i$- 11j $ + 38k}
$ = -1.5iˆ - 2.75 ˆj - 9.5kˆ
the circle. 4
25. It will be four times the initial horizontal range.
1
12. Maximum height attained by a projectile µ
g é u2 ù
êQ Rmax = or R µ u 2 ú
As the value of g is less on hills than on plains, so a tennis ëê 2g ûú
ball bounces higher on hills than on plains.
26. It will be three times the initial vertical height.
13. No
14. Two vectors are parallel and acting in the same direction é u 2 sin 2 q h sin 2 30° æ 1 ö ù
2
ê hm = Þ m = =ç ú
(i.e. q = 0°) ê 2g hm2 sin 2 60° è 3 ÷ø ú
ur ur ur ur ë û
15. Here, vectors | A + B | = | A - B | or,, A2 + B2 + 2AB cos q = 27. Maximum height will be attained at q = 90°, whereas
A2 + B2 – 2AB cos q or, 4AB cos q = 0, or, cos q = 0° or 90° horizontal range will be maximum when q = 45°.
i.e. the two vectors are perpendicular to each other.
88 Physics

1 Y
28. u cos q = u/2 i.e. cos q = or, q = 60°.
2
P Q
\ R = u2/g sin 2(60°) = u2/g (Ö3/2) = Ö3 u2/2g
29. The equation of trajectory of a projectile having initial r r'
speed v at angle q with the horizontal is,
y = x tanq – g (x/v cos q)2/2 O X
30. Parabolic. uuur uuur
31. T1 = 2usin q/g and T2 = 2u sin(90° – q)/g Here, OP and OQ are position vectors of P and Q
\ T1/ T2 = sin q/ sin (90°– q) = sin q/ cos q = tanq respectively and PQ is the displacement vector.
r r
32. Horizontal range, R = (u2/g) sin 2q and Maximum height, H 3. Yes, because a + b is represented by the diagonal of the
r r
= u2 sin2q /2g parallelogram drawn with a and b as adjacent sides. The
(i) when q = q, R1 = (u2/g) sin 2q and H1 = u2 sin2q/2g r r
diagonal passes through the common tail of a and b .
(ii) when q = (90°– q), R2 = (u2/g) sin 2(90°– q) = u2/g sin 2q r r
However, a - b is represented by the other diagonal of
and H2 = u2 sin2 (90° – q)/2g = u2 cos2 q/2g the same parallelogram not passing through the common
\ R1/ R2 = 1 and H1/ H2 = u2 sin2 q/2g/ u2 cos2 q/2g r r r r r r
tail of a and b . Thus both a + b and a - b lie in plane of
= sin2q/ cos2q = tan2q the same parallelogram.
2
33. Here, R = nH so, (u2/g) sin 2q = nu2 sin2q /2g, æQö
4. Clearly, P 2 + ç ÷ = Q2
\ tan q = 4/n, q = tan–1 (4/n) è2ø
34. We know, maximum height, H = u2 sin2q /2g, if q = 90°,
sin q = 1 (maximum), then, Hmax = u2/2g = h
Similarly, maximum horizontal range, Rmax = u2/g = 2h Q
Q
35. The angular displacement per unit time is known as angular 2 b
speed. Mathematically, angular speed = angular
displacement / time taken. The SI unit is radians per second P
(rad s–1).
36. The linear speed is given by v = r w. Since the angular 3 2 3
or P2 = Q or P = Q
speed w is same for both the cars, the linear speed v is 4 2
more for the outer car for having higher radius r. Q/2 Q/2 1
\ tan q = = =
37. For hour hand of watch, time period, Th = 12h for earth, P 3Q / 2 3
Te = 24h, or q = 30°
Angular velocity, w = 2p/T r r
Angle between P and Q ,
\ wh/ we = Te /Th= 24/12 = 2 b = 180° – q = 180° – 30° = 150°.
\ wh= 2we 5. When a bullet is fired from a gun with its barrel directed
38. Here, (u2/g) sin2(15°)= 1.5, or, u2/g(1/2) = 1.5 or, u2/g = 3km towards the target, it starts falling downwards on account
of acceleration due to gravity. Due to which the bullet hits
Horizontal range, R = (u2/g) sin 2(45°)= (u2/g) sin (90°)=
below the target. Just to avoid it, the barrel of the gun is
(u2/g) = 3km lined up little above the target, so that the bullet, after
Short Answer Questions travelling in parabolic path hits the distant target, as shown
in Fig.
1. Scalar is the physical quantity, which has only magnitude.
Path of bullet
The examples of scalar quantities are mass, length, time,
distance, speed, temperature, energy, work done, power. Line of sight Target
Vector is the physical quantity, which has both magnitude
and direction. The examples are weight, displacement, Gun
velocity, force. 6. Time of flight of the ball thrown vertically upwards,
2. Consider a body is moving in X-Y plane. Let P and Q be the 2u
T1 = 1
positions of the body at time t1 and t2 respectively. g
The lines joining P and Q with the origin of coordinate Time of flight of the ball thrown at an angle q with the
system O are called the position vectors of the points P
2u2 cos q
and Q. The line joining P and Q is called displacement vertical, T2 =
g
vector.
Motion in a Plane 89

9. Let a body is projected with initial velocity u, making an


2u1 2u2 cos q
As T1 = T2 \ = angle q with the horizontal. The velocity u has two
g g rectangular components :
or u1 = u2 cos q (i) The horizontal component u cos q, which remains
constant throughout the motion.
u12 u22 cos2 q (ii) The vertical component u sin q, which changes with
Now, H1 = and H2 = time under the effect of gravity. This component first
2g 2g
decreases, becomes zero at the highest point A, after
which it again increases, till the projectile hits the
H1 u12 u 2 cos 2 q
\ = = 2 = 1:1. ground.
H 2 u22 cos 2 q u22 cos 2 q
Y vy
7. Yes, for the longest jump the player should throw himself
at an angle of 45° with respect to horizontal. The vertical v A
height for this angle is x b vx Path of
P(x,y) Max projectile
2 u sin q
45° u u y height = hm
H = u2 sin2 =
2g 4g q vx = u cos q
X
where u is the velocity of projection. If the vertical height O u cos q B q
R v
is different from u2/4g, then the angle will be different from
vy = u sin q
45° and the horizontal distance covered will also be less.
Clearly, the span of jump depends upon Trajectory of a projectile fired at an angle q with the
(i) the initial velocity of the jump, horizontal.
(ii) the angle of projection. Let the body reaches the point P (x, y) after time t.
8. Consider an object moving with uniform velocity vr in \ The horizontal distance covered by the body in time t,
XY-plane. x = Horizontal velocity × time = u cos q t
x
or t =
r (0 ) u cos q
For vertical motion : u = u sin q, a = – g, so the vertical
distance covered in time t is given by
r (t )
1 2
r r s = ut + at
Let r (0) and r (t ) be its position vectors at times t = 0 and 2
t = t respectively. x 1 x2
r r or y = u sin q. - g. 2
r (t ) - r (0) u cos q 2 u cos2 q
Then vr =
t -0
r r r or y = x x tan q -
g
x2
or r (t ) = r (0) + vt ...(1) 2 2
2u cos q
In terms of rectangular components, we can write
or y = px – qx2
r vx2 + v 2y where p and q are constants.
v = vx iˆ + v y ˆj , where v =
Thus y is a quadratic function of x. Hence the trajectory of
r a projectile is a parabola.
r (0) = x (0)iˆ + y (0) ˆj
10. Horizontal range. It is the horizontal distance travelled by
rˆ(t ) = x(t )iˆ + y (t ) ˆj the projectile during its time of flight.
Substituting these values in equation (1), we get So Horizontal range = Horizontal velocity × time of flight
x(t )iˆ + y (t ) ˆj = x(0)iˆ + y (0) ˆj + (v i + v ˆj ).t
x y
2u sin q u 2
or R = u cos q ´ = .2 sin q cos q
x(t )iˆ + y (t ) ˆj = [ x (0) + v x t ]iˆ + [ y (0) + v y t ] ˆj ...(2) g g

Equating the coefficients of iˆ and ĵ on both sides, we get


u 2 sin 2q
x (t) = x(0) + vxt or R= [Q 2sin q cos q = sin 2q]
y (t) = y (0) + vyt g
The above two equations represent uniform motions along Clearly, R will be maximum when
x-axis and y-axis respectively. Thus equation (2) shows
that a uniform motion in two-dimensions can be expressed sin 2q = 1= sin 90º or 2q = 90° or q = 45°
as the sum of two uniform motions along two mutually Thus the horizontal range of a projectile is maximum when
perpendicular directions. it is projected an angle of 45° with the horizontal.
90 Physics
11. In uniform circular motion, the speed of the body remains 14. Let us consider, a point object moving along a circular
the same but the direction of motion changes at every point. path, where
Fig. shows the different velocity vectors at different w = angular velocity of a point object and
positions of the particle. At each position, the velocity v = linear velocity of that point object
r r r = radius of the circular path with centre at O.
vector v is perpendicular to the radius vector r . uuur r
Let, at any instant, t, the object is at P, where OP = r and
v
r r uuur r
at instant, t + Δt is at Q, where OQ = r + Δr
where, Ð POQ = Dq
r
O Q
v

v
r + D r Dq
Thus the velocity of the body changes continuously due P
O r
to the continuous change in the direction of motion of the
body. As the rate of change of velocity is acceleration, so
a uniform circular motion is an accelerated motion.
ur ur The object describes an arc PQ of length Dl in time interval
12. Subtraction of vector B from vector A is defined as the
ur ur Dt.
addition of the vector - B to vector A . Therefore, v = Dl/Dt or, Dl= vDt …(a)
Again w =Dq/Dt or, Dq = w Dt …(b)
Since the angle = arc/radius,
\ Dq = Dl/r or, w Dt = vDl/r, from …(a) and …(b)
B we have, v = wr
Q
B 15. Let, w = angular velocity and v = linear velocity of a point
A qP object
O q We know, v = wr, differentiating both sides, dv/ dt = d/dt
A –B
A + (– B ) (wr) = (dw/dt) r = a r
S Therefore, a = a × r and in vector notation,
a=a×r
Therefore,
ur ur ur ur where a is the linear acceleration and a is the angular
A - B = A + ( - B) acceleration.
ur ur ur ur uuur uuur uuur
In DOPS, A - B = A + (- B) = OP + PS = OS [by triangle Long Answer Questions
law] ur ur
ur ur ur 1. Let, A , B be the two vectors represented both in
If the angle between A and B is q, then angle between A magnitude and direction by the two sides OP and PQ, taken
ur
and – B is (180 – q) in same order, of a triangle OPQ, as shown in the adjoining
figure. According to the triangle law of vector addition, the
Therefore, \ R = A 2 + B 2 + 2 A.B cos(180 - q ) and, third side of the triangle, taken in opposite order i.e. OQ
ur ur uuur uuur uuur
\ tan f = B sin(180 - q) /{ A + B cos(180 - q} represents the vector sum of A and B . i.e. OQ = OP + PQ
13. A vector having zero magnitude and arbitrary direction is ur ur ur
or, R = A + B
called zero vector. It is written as, | 0 | = 0
In order to find the magnitude and direction of the resultant
Displacement of a stationary particle, velocity of a stationary
body and acceleration of a body in uniform motion are few vector R, we draw perpendicular QN on the line OP
examples of zero vector. produced.
Two properties:
1. The addition or subtraction of zero vector from a Q
given vector does not change the given vector.
ur r ur ur r ur
A + 0 = A, A – 0 = A R= A+ B
2. The multiplication of a non zero real number with a B
zero vector gives a zero vector.
r r f q
n0= 0 O P N
A
Motion in a Plane 91

From right-angled triangle OQN, The situation is clear from the graphical representation:
(OQ)2 = (OP)2 + (PQ)2 = (OP + PN)2 + (NQ)2 Y
= (OP)2 + (PN)2 + 2OP.PN + (NQ)2
= (OP)2 + {(PN)2 + (NQ)2} + 2OP.PN ...(i)
B P (x,y)
In right angled triangle PNQ, (PN)2 + (NQ)2
= (PQ)2 ...(ii)
PN = PQ cos q
r
Therefore, equation (i) gives,
(OQ)2 = (OP)2 + (PQ)2 + 2OP.PQ cos q ...(iii) O
But, OQ = R,OP = A and PQ = B A X
From (iii) we get,
R2 = A2 + B2 + 2A.B cos q
OA = x and OB = y is magnitude of the position vector in X
\R = A2 + B 2 + 2 A.B cos q and Y axes respectively.
ur r
Let f be the angle made by resultant vector R with the Let us consider, r (t) - position vector of the particle at
ur QN time t
vector A , then in right angled triangle D ONQ, tan f = r
ON r (t ') - position vector of the particle at time t¢
QN Thus the displacement vector from time t to time t¢ is given
=
(OP + PN ) by
In right angled triangle PQN, r r r
D r = r (t ') – r (t) = [ iˆ x (t ') + ĵ y (t ') ]
QN = PQ sin q and PN = PQ cos q
PQ sin q B sin q = [ iˆ x(t) + ĵ y(t)] = [x (t ') – x(t)] iˆ + [y (t ') – y(t)] ĵ
\ tan f = = Case of uniform velocity:
(OP + PQ cos q) ( A + B cos q)
This is defined when a moving object covers equal
B sin q displacement in equal intervals of time. The body moves in
\ tan f =
( A + B cos q) this case, same distance and in the same direction in every
Cases: unit of time. Mathematically, it is given by,
ur ur r r r
I. If A and B are parallel to each other i.e. q = 0°, then v = [ r (t ') – r (t)] /( t ' – t)
r r r
R= A2 + B 2 + 2 A.B = A + B, which is the maximum Equation of motion: r (t) = r (o) + v t .... (1)
ur ur r r r
value of the resultant to two vectors A and B . r (t ') = r (t) + v ( t ' – t)
Moreover, tanf = 0 i.e. q = 0°, i.e. the resultant of two r
vectors is the same direction of the two vectors. The velocity is expressed as v = vx iˆ + vy ĵ
II. If A and B are acting on the opposite to each other i.e. where, vx, vy - magnitude of the velocity along the X and Y
q = 180°, then axis respectively

R= A2 + B 2 - 2 A.B = A - B , which is the minimum iˆ , ĵ - unit vectors along the X and Y axis respectively
ur ur denoting the directions
value of the resultant to two vectors A and B . Again,
Magnitude of the velocity, i.e. speed is given by,
f = 0°, i.e. the resultant of two vectors is in the direction
of the greater vector. r
Speed = v = | v | = (vx 2 + v y 2 )
III. If A and B are perpendicular to each other i.e. q = 90°,
then From equation (1) we can write, x(t) iˆ + y(t) ĵ

R = A2 + B 2 + 2 A.B = A + B = x(0) iˆ + y(0) ĵ + (vx iˆ + vy ĵ )t


Here, tan f = B/A, i.e. f = tan–1(B/A). Thus the component can be equated as,
2. Let the position P of a particle in two dimensions is given x(t) = x(0) + vxt
r y(t) = y(0) + vyt
ˆ + ˆjy
by, r = ix
Case of uniform acceleration:
where iˆ ˆj - position vectors for denoting direction of the This case arises when moving object changes its velocity
particle along X and Y axes vector by equal amount in equal intervals of time. That
x, y- position coordinate of the particle means acceleration is not function of time. Mathematically
Magnitude of the vector i.e. Length of the vector given by,
r r r
uuur uuur 2 2 a = [ v (t ') – v (t)] /( t ' – t)
OP = | OP | = x + y
92 Physics
Equation of motion: Time of flight:
r r r Time of flight = Total time to reach the maximum height and
v (t) = v (0) + a t ... (1)
r r r return to ground
or, v (t ') = v (t) + a ( t ' – t) Since the time to go up is equal to time to come down (t),
r \ T = 2t
The acceleration is expressed as a = ax iˆ + ay iˆ At the highest point, velocity is zero, therefore, using the
where, ax, ay - magnitude of the acceleration along the X component of velocity equation, we get,
and Y axis respectively v = u + at; or, 0 = u sin q – gT/2 or, T = 2u sin q/ g
iˆ , ĵ - unit vectors along the X and Y axis respectively Maximum Height:
Taking the vertical upward motion in consideration, we get,
denoting the directions
r y = H, t = u sin q/ g, uy = u sin q, ay = – g,
Each component of a is constant and the components of Putting these values in the y component of the equation of
r motion, we get,
v (t) changes with time by,,
H = u sin q × (u sin q/g) – ½ g (u sin q/g)2
vx(t) = vx(0) + axt
\ H = u2 sin2 q/ 2g
vy(t) = vy(0) + ayt
Horizontal Range:
Magnitude of the acceleration is given by,
Range (R) is the horizontal distance from the launch point
r
a = | a | = (a x2 + a y 2 ) to the point at which the particle returns to the launching
plane.
Equation of motion: R = u cos q T = u cos q(2u sin q/g) = u2/g sin 2q
Over the time interval t to t¢ the average velocity is given R = (u2/g) sin 2q
by, 4. When an object moves in a circular path with constant
r r r r r r
v avg = 1/2 ( v (t) + v (t ') ) = 1/2[ v (t) + v (t) + a ( t ' – t)] speed, the direction of velocity changes at every point in
r r r its path, due to which an acceleration will be produced
Now, r (t ') = r (t) + v avg ( t ' – t) towards the center of the circle which is called centripetal
r r r r acceleration.
= r (t) + v ( t ) ( t ' – t) + 1/2 a ( t ' – t)2
This is a generalize relation, if we put t ' ® t and t ® 0 we
B v2
can write the components as,
x(t) = x(0) + vx(0)t + 1/2ax t2 Q A
A'
y(t) = y(0) + vy(0)t + 1/2ay t2 r2
Each of these components is just like uniformly accelerated v1 B'
motion in one dimension. O Dq P v1
3. Let, a projectile is projected with an angle q with the r1 v2 Dq
horizontal with initial velocity u.
R- range of the projectile P'
H- maximum height of the projectile
u cos q - horizontal component of the velocity
u sin q - vertical component of the velocity Let, m – mass of the particle; v- constant speed; w- uniform
angular speed; r- radius of the circular path
At any time t, the particle is at P with position vector
Y
uuur ur
OP = r1
and at time t + Dt, the particle is at Q with position vector
u sin q

uuur uur
H OQ = r2
u ÐPOQ = Dq
ur uur
q r1 + r2 = r
u cos q X Angular speed = w = Dq /Dt
R uur uur
Then the X and Y components of the equations of motion Let, v1 and v2 are the velocity vectors of the particles
are: at P and Q respectively.
x = (u cos q)t ……(1) uuur uuur
y = (u sin q)t – 1/2 gt2 ……(2) Since the speed is uniform, | PA |=| QB |= v
From equation (1) the value of t can be substituted in To find the change in the velocity in the given time interval,
equation (2) and the path of the particle in projectile motion uur uur uur uur
draw P 'A ' and P 'B ' as the velocity vectors v1 and v2
is given by the equation, y = (tan q)x – gx2/ 2(u cos q)2
Which is the equation of parabola. So the path of the particle ÐA ' P ' B ' = Dq
in projectile motion is parabolic.
Motion in a Plane 93

From triangle law of vectors,


uur uur uur 2H æHö
, f = tan -1 ç ÷
P 'A '+ A 'B ' = P 'B ' 2. Here, R = v0
uur uur uur uur uur g èRø
or, A 'B ' = P 'B '- P 'A ' = v2 – v1 = Dv
æ 1 gH ö
As Dt ® 0, A ' B ' would be arc of A ' B ' of circle of radius = tan -1 ç ÷ = 23°12¢
çv 2 ÷ø
P' A' = v è 0
r r
\ Dq = A ' B '/ P ' A ' = | D v | / | v | 3. Here, R2 = 3A2 and P = Q = A
r r r r
wDt = | Dv | / | v | or | D v | / | D t | Using, R 2 = P 2 + Q 2 + 2 PQ cos q
r
= | v | w = (wr )w = w 2 r [ \ v = wr] Þ 3 A2 = A2 + A2 + 2 A2 cos q
r r
When Dt ® 0 , then | D v | / | D t | would be magnitude of
Þ A2 = 2 A2 cos q
centripetal acceleration at P and given as,
r r 1
| a | =| D v | / | Dt | = w2r = (v/r)2r = v2/r Þ cos q = = cos 60° Þ q = 60o
r 2
Thus, | a | = w2r = v2/r
This gives the magnitude of the acceleration of a particle in 4. (i) 3A2 + B 2
uniform circular motion. = ( A + B)2 + ( A - B )2 + 2( A + B )( A - B) cos q
Direction of acceleration. As Dt tends to zero, the angle
Dq also approaches zero. In this limit, as AB = AC, so ÐABC = A2 + 2 AB + B 2 + A2 - 2 AB + B 2 + 2( A2 - B 2 ) cos q
r
= ÐACB = 90°. Thus the change in velocity Dv and hence
r 3 A2 + B 2 = 2 A2 + 2 B 2 + 2( A2 - B 2 )cos q
the acceleration a is perpendicular to the velocity vector
r r ( A2 - B 2 ) = 2( A2 - B 2 ) cos q
v1 . But v1 is directed along tangent at point P, so
r 1
acceleration a acts along the radius towards the centre of Þ cos q = = cos 60o Þ q = 60o
the circle. Such an acceleration is called centripetal 2
æ v2 ö (ii) 2( A2 + B 2 ) = ( A + B )2 + ( A - B) 2
acceleration. Its magnitude remains constant ç = ÷ but + 2( A + B ) ( A - B) cos q
è r ø
its direction continuously changes and remains Solve to get q = 90 o
perpendicular to the velocity vector at all positions.
5. Here, q = 90o , A = B, R = 1414
v
\ From R 2 = A2 + B 2 + 2 AB cos q
Þ 14142 = A2 + A2 + 2 A2 cos 90o
1414 2 = 2A2 Þ A = 1000 N
a
a 6. Given : A = 3x, B = 5x, q = 60o , R = 35
O Use R 2 = A2 + B 2 + 2 AB cos q , Find x, Then A = 15 N and
v
B = – 5N
a
7. R = A – B =10; R = A2 + B 2 = 50
v Solve to get A and B 30 N, 40N respectievely
Centripetal acceleration 8. N
Numerical Questions 40 km/h
D
q B
1. Since the speed of car matches with the horizontal speed
of the projectile, boy sitting in the car will see only vertical E
component of motion as shown in Fig. (b). W C O A
– 30 km/h 30 km/h

S
Relative velocity of car w.r.t. train = OD
= OC 2 + OB 2 = 402 + 302 = 50 km/h
BD 30
tan q = = = 0.75 = tan 36o ,52 '
OB 40
(a) (a) (b) \q = 36o ,52 ' west of north.]
94 Physics
9. Here, R cos 50° = 50 Þ R × 0.6428 = 50 vx = ux + axt = 50 m/s; vy = uy + ayt = 29.4 m/s ;
Þ R = 77.7N
R sin 50° = 77.7 × sin 50° = 59.59 N v= vx2 + v 2y = 502 + (29.4) 2 = 58 m/s
10. R = 50 N, R cos q = 25
vy 29.4
1 tan q = = = 0.588 = tan 30°27’
Þ cos q = = cos 60o Þ q = 60o , vx 50
2
3 2u sin q 2 ´ 19.6 ´ sin 30°
R sin 50o = 50 × sin 60o = 50 = 25 3 N 18. Time of flight, T = = = 2s;
2 g 9.8
ur ur
11. A + B = 6iˆ - ˆj - 3kˆ ; Required vector u 2 sin 2q (19.6) 2 ´ sin 2 ´ 30°
Range, R = = = 33.95 m
= k̂ – ( 6iˆ - ˆj - 3kˆ ) = -6iˆ + ˆj + 4kˆ g 9.8
uur uur
12. From question, A + B = 6iˆ - ˆj - 3kˆ ; u 2 sin 2q u 2 sin 2 q
19. Range, R = 2H Þ = 2´
uur uur g 2g
| A + B | = 52 + 32 + ( -1)2
Þ 2 sin q cos q= sin q Þ tan q = 2
2
= 25 + 9 + 1 = 35 1
2
uur uur \ sin q= and cos q= \
A +B 5iˆ + 3 ˆj - kˆ 5 5
\ Unit vector = uur uur =
|A +B | 35
2u 2 2 1 4u 2
uur uur uur R2 = . ´ =
13. A + B = C = -iˆ + 7 ˆj - 3kˆ g 5 5 5g
uur uur uur 20. Here, T = 60 min = 60 × 60 s, q = 2p rad.
14. Here, 2 A + 6B + 5C = 0
uur uur uur q
w= =
2p
=
p
rad/s]
Þ 5C = -2 A - 6 B
t 60 ´ 60 1800
uur 6 2 w - w0
Þ C = - (iˆ - 6 ˆj ) - (3iˆ + 5 ˆj + 5kˆ ) 21. Angular acceleration, a =
5 5 t
6 36 ˆ 6 ˆ ˆ (400 - 100) ´ 2p p
= - iˆ + j - i - 2i - 2kˆ = = rad/s2
5 5 5 60 ´ 5 ´ 60 30
1 2 p 2p
= – (12iˆ - 26 ˆj - 10kˆ) ] 22. Here, T = 2s, r = 100 cm; w = = = p rad/s;
5 T 2
uur uur v = w2r = p 2 ×100 = (3.14)2 ×100 = 987.7 cm/s2
uur uur A.B
15. As we knlow, A . B = AB cos q Þ cos q =
AB HOTS/EXEMPLAR QUESTIONS
(iˆ + 2 ˆj - kˆ).( -iˆ + ˆj - 2kˆ) 1
Very Short Answer Questions
Þ cos q = =
12 + 22 + ( -1) 2 ( -1)2 + 12 + (-2)2 2
1. When you toss a ball upward, acceleration due to gravity
uur uur uur
16. Q A = B + C Þ A2 = B2 + C2 here q = 90° acts in the downward direction. So the speed of the ball
As 5, 4, 3 forms a Pythagorian triplet. i.e. 52 = 42 + 32 decreases by 9.8 m/s in each second.
2. They are the same.
3. Use the equation for speed as a ‘guide to thinking’
A q d
v=
C t
d is 20 m; but we don’t know t the time the ball takes to go
20 meters. But while the ball moves horizontally 20 meters, it
B falls a vertical distance of 5 meters, which takes 1 second,
so t = 1 s.
uur uur 3
Angle between A and C , cos q = = 0.6 d 20m
5 v= = = 20m / s
= cos 53°8' Þ q = 53°8' t 1s
4. acceleration - g.
17. Given : u = 50 m/s, t = 3s, x = ut = 150 m;
velocity - zero.
1 2
y= gt = 44.1m
2
Motion in a Plane 95

Short Answer Questions N = 0, where the body leaves contact with the sphere.
1. If air resistance is negligible, both balls will strike the ground v2 æR -h ö
with the same speed. It is because the ball thrown upwards \ = g cos q = g ç ÷
R è R ø
reaches its maximum height with zero speed and again falls
down with an increment of 10 m/s in the speed each second. but v2 = 2gh, 2gh = g(R – h)
Thus, it reaches the point from where it was thrown up with
or h = R/3
the same initial speed of throw. In this way, both the balls in
3. The minimum vertical velocity required for crossing the hill is
a way, are thrown downward from the same point with the
same initial speed. Hence, both of them will strike the ground given by v 2^ ³ 2gh = 10, 000
with the same speed.
v ^ > 100 m / s
2. The vertical distance a projectile falls below an otherwise
As canon can haul packets with a speed of 125 m/s, so the maximum
straight line path is the same vertical distance it would fall
from rest in the same time because the distance covered in value of horizontal velocity, vP will be

1 2 vP = 1252 - 1002 = 75 m/s


vertical direction in both the cases is given by d = gt ,
2 The time taken to reach the top of the hill with velocity v^ is
where t is the elapsed time, and g is the acceleration due to
gravity. 1 2
given by gT = h Þ T = 10s .
2
2H æHö æ 1 gH ö
3. R = vo , f = tan -1 ç ÷ = tan -1 ç ÷ = 23°12¢ In 10s the horizontal distance covered = 750 m.
g èRø çv 2 ÷ø So cannon has to be moved through a distance of 50 m on the
è o
ground.
Long Answer Questions So total time taken (shortest) by the packet to reach ground across
50
1. For body to hit the lowest plane with minimum velocity, it the hill = s + 10s + 10s = 45s
should be able to just clear the second step 2
4. (a) 5 m/s at 37° to N.
Let R = Range for minimum velocity
h = height for minimum velocity (b) (i) tan
-1
( 3/ 7 ) of N (ii) 7 m/s
v = minimum velocity (c) In case (i) he reaches the opposite bank in shortest time.

t = time of motion CHAPTER TEST


R = u × t Þ t = R/u ......(1) 1. The acceleration is uniform and directed towards the centre
of the path. It is called centripetal acceleration.
1 2
h= gt ......(2) 2. When the three vectors are not lying in a plane their resultant
2 can not be zero.
From (1) & (2), uur uuur
3. A = QP = (3iˆ + 2 ˆj + 2kˆ) - (iˆ + 2 ˆj - kˆ) = 2iˆ + 3kˆ
1 R 2
gR 2 uur
h= g´ Þh= ......(3) \ | A | = 22 + 32 = 13
2
2 u 2u 2
4. Refer to theory.
R = 2 × 20 cm = 40 cm = 0.4 m 5. Refer to theory.
h = 2 × 10 cm = 20 cm = 0.2m v2
6. Centripetal acceleration = a =
r
1 R2 1 0.4 ´ 0.4
\u2 = g´ Þ u 2 = ´ 10 ´ =4
2 h 2 0.2 (2v )2 4v 2 2v 2
a' = = = = 2a
2r 2r r
Þ u = 2m/s
2. Consider the figure \ acceleration will be double.

N u 2 sin 2q
7. R=
h Q mv
2
g
R
q
os q
(R – h) gc (30)2 sin 2q 45 ´ 10 1
m mg mg sin q 45 = Þ sin 2q = =
q 10 30 ´ 30 2
sin q = sin 30°
mv2 q = 15° and 75°
N+ = mg cos q
R
96 Physics
8. Here, A = 5 kg wt, B = 10 kg wt, q = 120° 10. Parallelogram law of vector addition : Let the two vectors
r r
A and B inclined to each other at an angle q be
3 represented both in magnitude and direction by the

5sin120° 2 1 uuur uuur
tan a = = = = tan 30° adjacent sides OP and OQ of the parallelogram OPSQ.
10 + 5cos120° 1 3
10 - 5 ´
2 Then according to the parallelogram law of vector addition,
r r
\ a = 30° the resultant of A and B is represented both in magnitude
uuur
and direction by the diagonal OS of the parallelogram.
9. Consider the straight line path APQC through the sand.
Q S
D C
Q
B
A +B
R= B

P q
50 mm R b q
A B O A P N
100 mm
ur
Time taken to go from A to C via this path Magnitude of resultant R . Draw SN perpendicular to OP
produced.
AP + QC PQ Then ÐSPN = ÐQOP = q, OP = A, PS = OQ = B, OS = R
Tsand = +
1 v From right angled DSNP, we have
25 2 + 25 2 50 2 = 50 2 é 1 ù SN
= + êë v + 1úû = sin q or SN = PS sin q = B sin q
1 v PS
Ths shortest path outside the sand will be ARC. . PN
= cos q or PN = PS cos q = B cos q
Time taken to go from A to C via this path PS
AR + RC Using Pythagoras theorem in right-angled DONS, we get
= Toutside = s = 2 752 + 252 s = 2 ´ 25 10 s OS2 = ON2 + SN2 = (OP + PN)2 + SN2
1
or R2 = (A + B cos q)2 + (B sin q)2
é1 ù = A2 + B2 (cos2 q + sin 2 q) + 2AB cos q
For Tsand < Toutside, 50 2 ê + 1ú < 2 ´ 25 10
ëv û = A2 + B2 + 2 AB cos q

1 or R = A2 + B 2 + 2 AB cos q
Þ +1 < 5 ur ur
v
Direction of resultant R . Let the resultant R make angle b
ur
1 1 with the direction of A . Then from right angled DONS, we
Þ < 5 - 1 or v > = 0.81m/s
v 5 -1 get
SN SN B sin q
tan b = = or tan b = .
ON OP + PN A + B cos q

¿¿¿
5 Laws of Motion
C ha p t e r

To put a stationary body in motion or to stop a moving body we require some external agency may be in contact with the body e.g.,
frictional force, may not be in contact with the body e.g., gravitational force, magnetic force, etc. But no any external agency is required
to maintain a body in uniform motion.
The agency, push or pull, which brings (i) change in the state of rest or of motion (ii) change in shape of an object is called force.
ARISTOTLE’S FALLACY
According to Aristotelian law, an external force is required to keep a body in motion. However an external force is required to overcome
the frictional forces in case of solids and viscous forces in fluids which are always present in nature.

NEWTON’S FIRST LAW OF MOTION OR GALILEO’S LAW OF INERTIA


Every body continues to be in its state of rest or of uniform motion in a straight line unless compelled by an external force to change
its state. This fundamental property of the body is called inertia.
This law is known as Newton’s first law of motion or law of inertia. This law defines the force and states that the force is a factor which
can change the state of an object.
Definition of force from Newton’s first law of motion “Force is the push or pull which changes or tends to change the state of rest or
of uniform motion of a body”.
Inertia
Inertia is the property of a body due to which it opposes the change in its state. Inertia of a body is measured by mass of the body. It
is directly proportional to the mass of the body
i.e., Inertia µ mass
Types of Inertia
1. Inertia of rest : It is the inability of a body to change by itself, its state of rest.
Ex. • When we shake a branch of a mango tree, the mangoes fall down.
• When a bus or train starts suddenly the passengers sitting inside tends to fall backwards.
• The dust particles in a blanket fall off when it is beaten with a stick.
• When a stone hits a window pane, the glass is broken into a number of pieces whereas, if the high speed bullet strikes the
pane, it leaves a clean hole.
2. Inertia of motion : It is the inability of a body to change by itself, its state of uniform motion.
Ex. • When a bus or train stops suddenly, a passenger sitting inside lean forward.
• A person jumping out of a running train may fall forward.
• A ball thrown upwards in a running train continues to move along with the train.
3. Inertia of direction : It is the inability of a body to change by itself, its direction of motion.
Ex. • When a straight running car turns sharply, the person sitting inside feels a force radially outwards.
• Rotating wheels of vehicle throw out mud, on the inner side of mudguard over the wheels.
• A body released from a balloon rising up, continues to move in the direction of balloon.
98 Physics
NEWTON’S SECOND LAW OF MOTION
The rate of change of momentum of a body is directly proportional to the external unbalanced force applied on it.
r dpr r r
dp
i.e., F µ or, F = k
dt dt

r dpr d r r
r dv r dm
If k = 1 then, F = = (mv) or F=m +v
dt dt dt dt

If m = constant then, If vr = constant then,


r
dm dv
=0 =0
dt dt
r r r
dv r dm
i.e., F = m = ma i.e., F = v
dt dt
r r
or, F = ma
• The change in momentum always takes place in the direction of force.
• The second law of motion is a vector law.
• This law gives the magnitude of force.
Definition of one newton : If an object of mass one kilogram has an acceleration of 1 m/s2 relative to an inertial reference frame, then
the net force exerted on the object is one newton.
Consequences of Newton’s Second Law of Motion
F
Concept of inertial mass : From Newton’s second law of motion, m =
a
i.e., the magnitude of acceleration produced by a given body is inversely proportional to mass i.e., greater the mass, smaller is the
acceleration produced in the body. Thus, mass is the measure of inertia of the body. The mass given by above equation is therefore
called the inertial mass.
Momentum
r r r r
The linear momentum of a body (p) is defined as the product of the mass of the body (m) and its velocity (v) . i.e., p = mv .
It is a vector and points in the same direction as the velocity vector. In terms of momentum, Newton's second law can be expressed as :
r r r
F = (dp / dt) . When a resultant force F acts on a body, it causes the linear momentum of that body to change with time.
Linear momentum depends on frame of reference, e.g., linear momentum of a body at rest in a moving train, is zero relative to a man
sitting in the train, while is non zero for a man standing on the ground.
The S.I. unit of momentum is kg m s–1
Impulse
If a large force acts on a body or particle for a smaller time, then the impulse = product of force and time.
r r
Suppose a force F acts for a short time dt then impulse = F dt
t2
r
For a finite interval of time t1 to t2 the impulse = ò F dt
t1
If constant force acts for an interval Dt then
r
Impulse = F. Dt

Applications of Concept of Impulse :


• Bogies of a train are provided with buffers.
• A cricket player draws his hands back while catching a ball.
• A person jumping on hard cement floor receives more injuries than a person jumping on muddy or sandy road.
• Cars, buses, trucks, bogies of the train etc. are provided with a spring system to avoid severe jerks.
• Chinawares are wrapped in straw or paper.
Laws of Motion 99

NEWTON’S THIRD LAW OF MOTION


To every action there is always an equal and opposite reaction.
r r
The magnitude of the two forces F a (the action) and F r (the reaction) are equal. This is always true. No matter how much the masses
may differ, and how many other forces are present.
r r r r
| F a| = | F r | or F a = – F r,
It is not important which force we call the action and which the reaction, but it is significant that they appear and disappear
simultaneously; i.e. the time interval during which they act on each other is the same.
When we hold the apple of 1 kg in our hand, gravity exerts an acting force of 9.8 N on the apple pointing downwards. Since we
hold the apple, it exerts this force on our hand. We call this as action force. Our hand exerts a reacting force of 9.8 N on the apple. This
force points upwards.
Other Examples of action and reaction:
Action: the tyres of a car push on the road. Reaction: the road pushes on the tyres.
Action: while swimming, you push the water backwards. Reaction: the water pushes you forward.
Action: a rocket pushes out exhaust. Reaction: the exhaust pushes the rocket forward.
Illustration 1 :
A truck starts from rest and rolls down a hill with constant acceleration. It travels a distance of 400m in 20s. Find its
acceleration and also find the force acting on it if its mass is 7metirc tonnes.
Sol. Given, s = 400m, u = 0, t = 20s
1 1
s = ut + at2 i.e. 400 = 0 + a(20)2 = 200a
2 2
\ a = 2ms–2.
The force acting on the truck,
F = ma = 7 × 103 × 2 = 14 × 103N
Illustration 2 :
A bullet of mass 0.04 kg moving with speed of 90 ms–1 enters a heavy wooden block and is stopped after a distance of 60 cm.
What is average resistive force exerted by the block on the bullet?
Sol. Given m = 0.04 kg, u = 90 ms–1, v = 0, s = 60 cm = 0.6 m
Now, v2 = u2 + 2as i.e., 0 = (90)2 + 2 a (0.6)
\ a = – 6750 ms–2.
\ Retarding force, F = ma = 0.04 × 6750 = 270N
Illustration 3 :
Two blocks made of different metals identical in shape and size are acted upon by equal forces, which cause them to slide on a
horizontal surface. The acceleration of the second block is found to be 5 times that of the first, what is the ratio of the mass of
the second to first?
Sol. Let F be resultant force on each block.
For 1st block, F1 = m1a1
For 2nd block, F2 = m2a2
or, m1a1/ m2a2 = 1, i.e., m2 / m1 = a1/ a2 ….(i)
Here, a2 = 5a1 ….(ii)
From (i) and (ii), m2 / m1 =1 : 5

Illustration 4 :
Two billiard ball, each of mass 0.05 kg, moving in opposite directions with speed of 6 ms–1, collide and rebound with the same
period. What is the impulse imparted to each ball due to the other?
Sol. Before collision, mass of each ball = 0.05 kg, speed of ball = 6 ms–1,
\ momentum, p = mv = 0.05 × 6 = 0.3 kg ms–1.
After collision, mass of each ball = 0.05 kg, speed of ball = – 6 ms–1,
\ momentum = 0.05 × (–6) = – 0.3 kg ms–1.
(Change in momentum after collision = p – p )
2 1.
\ change of momentum = impulse = – 0.3 – 0.3 = – 0.6 kg ms–1
Hence an impulse of 0.6 kg ms–1 will be imported to each ball in opposite directions.
100 Physics
CONSERVATION OF MOMENTUM
Newton’s second and third laws of motion lead to a consequence – The principle of conservation of momentum.
Principle of conservation of linear momentum : When no external force is acting on a body or system then total momentum remains
n
conserved, i.e. If F = 0 then, å pi = constant or p = constant.
i =1

Principle of conservation of momentum for a two particle system, m1u1+ m2u2 = m1v1 + m2v2 .
where m1, m2 ® mass of the particles; u1, u2 ® initial velocities; v1, v2 ® final velocities of the particles.
A rocket sent up in space will acquire a velocity (v) given by, v = –u loge(Mi/M), where u–exhaust speed of burnt gases, Mi- initial
mass of the rocket and its contents, M- mass of rocket and its contents at any instant.
æ dM ö
· The acceleration associated with a rocket is a = - ç u /M, where dM/dt is the rate at which the fuel is consumed.
è dt ÷ø

Velocity of a rocket at any time is given by v = u log e æç 0 ö÷ , where u ® velocity of exhaust gases relative to the rocket.
m
·
è mø
m0 ® critical mass of the rocket and m ® mass of the rocket at time t.

Thrust on the rocket = F = -u çæ


dm ö dm
· , where ® rate of combustion of fuel at time t.
è dt ÷ø dt
Recoiling of a gun is also based on the principle of conservation of momentum.
EQUILIBRIUM OF A PARTICLE
A particle is said to be in equilibrium if the net external force on the particle is zero. F2
r r r r F3
If F1, F2 , F3 ..... Fn forces act on a particle then the particle will be in equilibrium if a
r r r b g
F1 + F2 + F3 + ... = 0 . F1
According to Lami’s theorem under equillibrium F1/sin a = F2/sin b = F3/sin g, where a, b, g are angles indicated.
Inertial and Non-inertial Frame of Reference
Inertial frame of reference : It is that frame of reference, which has no acceleration, and where Newton’s laws of motion hold good.
Non-inertial frame of reference : Non-inertial frame of reference is that frame of reference, which has definite acceleration, and where
Newton’s laws are to be modified.
Illustration 5 :
A 75.0 kg man standing in a stationary 50.0 kg boat throws a 10.0 kg anchor horizontally with a speed of 2.00 m/s. How fast
will the system (man + boat) recoil ?
Sol. The given velocity 2.00 m/s is the velocity of the anchor w.r.t. boat; we denote it as vAB.
Velocity of anchor w.r.t. water is vAW.

From law of conservation of momentum


mBvBW + mAvAW = mBv’BW + mAv’AW .......... (1)
Note that we have used all the velocities relative to single frame of reference, i.e., water.
r r r
Now v AB = vAW = vBW
r r r
or v AW = vAB + vBW
r r r
Similarly, v 'AW = v' AB + v'BW
r r
On substituting expression for v'AW and v AW in equation (1), we obtain
r r r
0 = m B v'BW + m A (v'AB + v'BW )

r æ mA ö r
or v'BW = - ç v'AB
è m B + m A ÷ø
Laws of Motion 101

æ 10.0 ö
= -ç (2.00) = –0.148 m/s
è 125.0 + 10.0 ÷ø
The anchor travels with velocity relative to water.
r r r
v AW = vAB + v'BW = 2.00 + (– 0.148) = 1.85 m/s

COMMON FORCES IN MECHANICS


In mechanics, we encounter non-contact force e.g., gravitational forces and contact forces e.g., friction – the component parallel to the
surfaces in contact, normal reaction – the component of contact force normal to the surfaces in contact, buoyant force equal to the
weight of the fluid displaced, viscous force, air resistance, tension in string, force due to spring – spring force, F = –kx where
x = elongation or compression in string, k = force constant and –(ve) sign indicates that the force is opposite to the displacement from
the unstretched state. These contact forces of mechanics fundamentally arise from electrical forces.
FRICTION
Two surfaces which can move one across the other without encountering any resistance are in frictionless contact, or smooth
contact. Conversely two objects whose relative surface movement is resisted have friction between them and their contact is rough.
Whenever a body moves or tends to move over the surface of another body, a force comes into play which acts parallel to the surface
of contact and opposes the relative motion. This opposing force is called friction.
The results of experimental investigation into the behaviour of frictional forces confirm that:
(a) frictional force opposes the movement of an object across the surface of another with which it is in rough contact.
(b) the direction of the frictional force is opposite to the direction of motion.
(c) the magnitude of the frictional force is only just sufficient to prevent movement and increases as the tendency to move
increases, up to a limiting value. When the limiting value is reached, the frictional force cannot increase any further and motion
is about to begin (limiting equilibrium). When the frictional force, F, reaches its limit, its value then is related to the normal
reaction N in the following way : N
F = µN
The constant µ is called the coefficient of friction and each
pair of surfaces has its own value for this constant.
In limiting equilibrium F = µN
In general F £ µN
F
STATIC FRICTION
When there is no relative motion between the contact surfaces, frictional force is
called static frictional force. It is a self-adjusting force, it adjusts its
value according to requirement.
In the graph shown static frictional force is equal to applied force. Hence one
can say that the portion of graph ab will have a slope of 45°. ( f s £ m s N )

Limiting Friction
This is the maximum frictional force that can exist at the contact surface. This frictional force acts when body is about to move.
Laws of limiting friction:
1. The magnitude of the force of limiting friction (fL) between any two bodies in contact is directly proportional to the normal
reaction (N) between them i.e., fL µ N
2. The direction of the force of limiting friction is always opposite to the direction in which one body is on the verge of moving over
the other.
3. The force of limiting friction is independent of the apparent contact area, so long as normal reaction between the two bodies in
contact remains the same.
4. Limiting friction between any two bodies in contact depends on the nature of material of the surfaces in contact and their
roughness and smoothness.

KINETIC FRICTION
Once relative motion starts between the surfaces in contact, the frictional force is called as kinetic frictional force.
The magnitude of kinetic frictional force is also proportional to normal force. fk = mkN
From the previous observation we can say that mk < ms
Although the coefficient of kinetic friction varies with speed, we shall neglect any variation i.e., one relative motion starts a constant
frictional force starts opposing its motion.
102 Physics
Angle of Friction (l)
At a point of rough contact, where slipping is about to occur, the two forces acting on each object are the normal reaction N and
frictional force µN.
The resultant of these two forces S makes an angle l with the normal, where
mN
tan l = =m
N
The angle l is called the angle of friction.
Angle of Repose ( a )
It is the angle that an inclined plane makes with the horizontal when a body placed on it is in limiting equilibrium.
Let us consider a block is placed on a rough inclined plane of inclination a. If block is just about to slide, then we have
flim = Mg sin a …(i)
and N = Mg cos a …(ii)
Dividing equation (i) by (ii), we get
flim
= tan a
N
f
Also we have, m s = lim
N
\m s = tan l
As m s = tan l
\a = l

ROLLING FRICTION
When a body rolls on a surface, the resistance offered by the surface is called rolling friction. If a
body rolls over the surface of another body, then both the rolling body and the surface on which
it rolls are compressed by a small amount.
The rolling body has to continuously detach itself from the surface on which it rolls. This is
opposed by the adhesive force between the two surfaces in contact and hence a force originates
which retards the rolling motion.
This retarding force is called the rolling friction. It is denoted by fr.
For the same magnitude of normal reaction, the sliding friction is much greater than the rolling Cause of rolling friction
friction.
Ex. : The sliding friction of steel on steel is approximately 100 times more than the rolling friction of steel on steel.
If mr is the coefficient of rolling friction, then mr < mk <ms

ADVANTAGES AND DISADVANTAGES OF FRICTION


Friction has advantages as well as disadvantages. In other words, friction is not desirable but
without friction, we cannot think of survival, so we can say that “friction is a necessary evil”.
In some cases friction acts as a supporting force and in some cases it acts as opposing force.
Supporting: Walking process can only take place because there is friction between the shoes You push
Ground
and ground. backward
pushes
Opposing: When a block slides over a surface the force' of friction acts as an opposing force on ground
you forward
in the opposite direction of the motion
Both Supporting and Opposing :
(i) Pedaling: In the cycling, rear wheel move by the force
communicated to it by pedaling, while front wheel moves by
itself, therefore, like in walking force of friction on rear wheel is
in forward direction and acts as a supporting force. As front
wheel moves by itself, force of friction on front wheel is in the
backward direction. f1 f2
f1 f 2
(ii) Non-Pedaling : When pedaling is stopped, both the wheels moves by themselves so the force of friction on both the wheels is
in backwards direction.
Laws of Motion 103

Advantages of Friction
(i) The force of friction helps us to move on the surface of earth. In the absence of friction, we cannot think of walking on the
surface.
(ii) The force of friction between the tip of a pen and the surface of paper helps us to write on the paper.
It is not possible to write on the glazed paper as there is no force of friction.
(iii) The force of friction between the tyres of a vehicle and the road helps the vehicle to stop when brake is applied. In the absence
of friction, the vehicle skid off the road when brake is applied.
(iv) Moving belts remain on the rim of a wheel because of friction.
(v) The force of friction between a chalk and the blackboard helps us to write on the board.

Disadvantages of Friction
(i) A significant amount of energy of a moving object is wasted in the form of heat energy to overcome the force of friction.
(ii) The force of friction restricts the speed of moving vehicles like buses, trains, aeroplanes, rockets etc.
(iii) The efficiency of machines decreases due to the presence of force of friction.
(iv) The force of friction causes lot of wear and tear in the moving parts of a machine.
(v) Sometimes, a machine gets burnt due to the force of friction between different moving parts.

Methods of Reducing Friction


Friction can be reduced by
• polishing the surface. (but extra polishing increase friction)
• lubrication.
• proper selection of material.
• avoiding moisture.
• streamlining the shape.
• using ball bearings or roller bearings.
Illustration 6 :
A box of mass 4 kg rests upon an inclined plane. The inclination of the plane to the horizontal is gradually increased. It is found
that when the slope of the plane is 1 in 3, the box starts sliding down the plane. (Given g = 9.8ms–2). (i) Find the coefficient of
friction between the box and the plane. (ii) What force applied to the box parallel to the plane will just make it move up the plane?
Sol. (i) Here, m = 4 kg, sin q = 1/3, g =9.8 ms–2,
When the box just begins to slide, the forces are in equillibrium
N
\ f = mg sin q and R = mg cos q
f
f
m= = tan q
R
1
tan q = = 0.35, 3
32 - 12 q mg sin q
µ = tan q = 0.35 mg cos q
mg
(ii) Minimum force applied to the box to move it up the plane q
= mg sin q + f = mgs in q + mR = mg (sin q + m cos q) [Q R = mg cos q]
æ1 1 8ö
= 4 × 9.8 ç + ´ = 26.13 N
è2 8 3 ÷ø

Illustration 7 :
m a
A block of mass 1 kg placed on an inclined plane as shown in figure.
(a) What must be the frictional force between block and inclined plane if the block is
not to slide along the inclined plane when the inclined plane is accelerating to the
right at 3 m/s2 ?
(b) What is the least value of m can have for this to happen?
(cos 37° = 0.8 ; sin 37° = 0.6)
104 Physics

Sol. N = m (g cos 37° + a sin 37°) = 1 (9.8 × 0.8 + 3 × 0.6) ma cos 37° N
and mg sin 37° = ma sin 37° + f f
(a) f = mg sin37° – ma sin37° or, f = 1 (9.8 × 0.8 – 3 × 0.8) = 5.48 ma
ma sin 37° mgsin 37°
f 5.48
(b) f = µ N \ m= = = 0.57 mg cos 37° f
N 9.64 mg37°

Illustration 8 :
For the block and surface shown in fig. µs = 0.5, µk = 0.3, mass M = 50 kg and F = 600 N Then what is its acceleration?
(Take g = 10 m/s2)

F
M

Sol. The forces acting on the block are shown in fig. Since
Fs = µsN = 0.5 × 50 × 10 = 250 N
Since, F > fs max so the block is in motion.

Net force 600 - 0.3(50 ´ 10)


Acceleration of block = = = 9 m/s2
Total mass 50

CIRCULAR MOTION
v2
When a body moves along a circular path of radius R with a velocity v and the acceleration directed towards the centre then
R
mv 2
according to Newton’s second law of motion, the force fc = where m is the mass of the body..
R
v2
This force is directed towards the centre of the circular path and is called centripetal force and the acceleration is called
R
centripetal acceleration.
The centripetal force for
(i) a stone rotated in a circle by a string is provided by the tension in the string.
(ii) motion of a planet around the sun is the gravitational force on the planet due to the sun.
(iii) the circular motion of a car on a flat and banked road is provided by the force of friction.
Centrifugal force is pseudo force that is equal and opposite to the centripetal force. It is directed away from the centre along the
radius. The centrifugal force cannot balance the centripetal force because they act on the different bodies.

Motion of a Car on a Level Road


When a car takes a turn on a level road, the portion of the turn can be approximated by an arc of a circle of radius r (see fig). If the car
makes the turn at a constant speed v, then there must be some centripetal force acting on the car. This force is generated by the
friction between the tyres and the road. (car has a tendency to slip radially outward, so frictional force acts inwards)
µS = coefficient of static friction
N
N = mg is the normal reaction of the surface
The maximum safe velocity is v f
mv2 f car
= mS N = µSmg r
r
v2 v
or mS = or v = ms rg
rg mg
Free body
It is independent of the mass of the car. diagram of car
The safe velocity is same for all vehicles of larger and smaller mass.
Laws of Motion 105

Motion of Car on a Banked Road


If a cyclist takes a turn, he can bend from his vertical position.
This is not possible in the case of car, truck or train.
The tilting of the vehicle is achieved by raising the outer edge of the circular track,
slightly above the inner edge. This is known as banking of curved track.
The angle of inclination with the horizontal is called the angle of banking.
If a driver moves with slow velocity, then friction does not play any role in R
negotiating the turn.
The various forces acting on the vehicle are :
(i) Weight of the vehicle (mg) in the downward direction.
(ii) Normal reaction (N) perpendicular to the inclined surface of the road.
Resolve N in two components.
(a) N cos q, vertically upwards which balances weight of the vehicle.
N cos N
\ N cos q = mg .....(i)
(b) N sin q, in horizontal direction which provides necessary centripetal force.

mv2
\ N sin q = .....(ii)
r
b N sin
On dividing eqn. (ii) by eqn. (i)

mv 2
N sin q
= r
N cos q mg
W = mg
v2 -1
æ v2 ö
or tan q = \ q = tan ç ÷
rg è rg ø
Where m is the mass of the vehicle, r is radius of curvature of the road, v is speed of the vehicle and q is the banking angle
æ hö
çè sin q = ÷ø .
b
i.e. Angle of banking does not depend on the mass of the vehicle.
Q v2 = gr tan q \ v = gr tan q (maximum safe speed)
This gives the maximum safe speed of the vehicle. In actual practice, some frictional forces are always present. So, the maximum safe
velocity is always much greater than that given by the above equation.
If the driver moves faster than the safe speed mentioned above, a friction force must act parallel to the road, inwards towards the
centre of the turn. N cos q N
In this case forces acting on the vehicle are :
(i) Weight of the vehicle (mg) in the downward direction.
q
(ii) Normal reaction perpendicular to the inclined plane of the road.
(iii) The frictional force f between the tyres and the road.
N cos q and N sinq are the two rectangular components of N. b N sin q
f cos q and f sin q are the two rectangular components of f. h
The car does not have any vertical motion. f cos q
\ mg + f sin q = N cos q
q
or mg = N cos q – f sin q
f sin q
But f = µN
where µ £ µs. W = mg
\ mg = N cos q – µ N sin q … (i)
The forces N sin q and f cos q together provide the necessary centripetal force.
mv 2
\ = N sin q + f cos q
r
mv 2
or = N sin q + µN cos q … (ii)
r
106 Physics
Dividing eqn (ii) by eqn (i) we get
mv 2
r = N sin q + µN cos q or
v2 sin q + µcos q
=
mg N cos q - µN sin q rg cos q - µsin q
v2 cos q (tan q + µ) tan q + µ tan q + µ
or = or v2 = rg or v = rg
rg cos q (1 - µ tan q) 1 - µ tan q 1 - µ tan q
The best speed to negotiate a curve is obtained by putting µ = 0.
\ v = rg tan q
With this speed, there will be minimum wear and tear of the tyres.
Bending of cyclist
In order to take a circular turn of radius r with speed v, the cyclist should bend himself through an angle q from the vertical such that
v2
tan q =
rg
Motion in a Vertical circle
For motion in a vertical circle,
mv2
(a) Tension in the string at any position, T = + mg cos q .
r
(b) At the highest point of vertical circle q = 180°, TH = mvH2/r + mg.
(c) Velocity at highest point, vH ³ gr .
(d) Velocity at lowest point, for looping the loop, vL ³ 5 gr .
(e) At the lowest point of vertical circle q = 0°, TL = mvL2/r + mg.
(f) Minimum velocity, when string is horizontal, q = 90°, vL = Ö3 gr.
(g) Height through which a body should fall for looping the vertical loop, h = 5r/2.
(h) Difference in tension at lowest and highest point, TL – TH = 6mg.
SOLVING PROBLEMS IN MECHANICS
While solving a typical problem in mechanics, one should use the following steps :
(i) Draw a diagram showing schematically the various parts of the assembly of bodies, the links, supports, etc.
(ii) Choose a convenient part of the assembly as one system.
(iii) Draw a separate diagram which shows this system and all the forces on the system by the remaining part of the assembly.
Include also the forces on the system by other agencies. Do not include the forces on the environment by the system. A
diagram of this type is known as ‘a free-body diagram’.
(iv) In a free-body diagram, include information about forces (their magnitudes and directions) that are either given or you are sure
of (e.g., the direction of tension in a string along its length). The rest should be treated as unknowns to be determined using
laws of motion.
(v) If necessary, follow the same procedure for another choice of the system. In doing so, employ Newton’s third law. That is, if in
the free-body diagram of A, the force on A due to B is shown as F, then in the free-body diagram of B, the force on B due to
A should be shown as – F.
Motion in a Lift
Apparent weight in a lift when it is
· Stationary or in uniform motion: Apparent weight = Actual weight
· Lift accelerated up by ‘a’: Apparent weight = Actual weight + Ma = M (g + a)
· Lift accelerated down by ‘a’: Apparent weight = Actual weight – Ma = M (g – a)
· In free fall: a = g and Apparent weight = M (g – g) = 0
· When a > g while falling: Apparent weight is negative. Therefore the mass M is said to be in contact with the roof.
Illustration 9 :
At what angle should a highway be banked for cars travelling at a speed of 100 km/h if the radius of the road is 400m and no
frictional forces are involved?
Sol. The banking should be done at an angle q such that
250 250
´
v2 625
tan q = = 9 9
or tan q = = 0.19
rg 400 ´ 10 81 ´ 40
or q = tan–1 0.19 » 0.19 radian » 0.19 × 57.3º » 11º
Laws of Motion 107

Illustration 10 :
A circular racetrack of radius 300 m is banked at an angle of 15°. If the coefficient of friction between the wheels of a race-car
and the road is 0.2, what is the (a) optimum speed of the race-car to avoid wear and tear on its tyres, and (b) maximum
permissible speed to avoid slipping ?
Sol. On a banked road, the horizontal component of the normal force and the frictional force contribute to provide centripetal force
to keep the car moving on a circular turn without slipping. At the optimum speed, the normal reaction’s component is enough to
provide the needed centripetal force, and the frictional force is not needed.
(a) The optimum speed v0 is given by
v0 = (R g tan q)1/2. Here R = 300 m, q = 15°, g = 9.8 m s-2;
Solving we get v0 = 28.1 m s-1.

æ ms + tan q ö
(b) The maximum permissible speed vmax is given by v max = ç Rg = 38.1 ms -1
è 1 - m s tan q ÷ø

Illustration 11 :
A mass of 6kg is suspended by a rope of length 2m from a ceiling. A force of 50N in the horizontal direction is applied at the mid-
point of the rope. What is the angle the rope makes with the vertical in equilibrium? Take g = 10ms–2. Neglect the mass of the
rope.
Sol. Let q - angle rope makes with vertical in equilibrium, T = tension in the rope. q
T1 T1cos q
P - mid-point of the rope where three forces are acting 1m
q T3 = 50 N
Force equations: P
T1sin q = T3 = 50 N and T1sin q
T1cos q = T2 = 6 × 10 = 60 N T2
1m
tan q = T1sin q/ T1cos q = T3/ T2 = 50/60 = 5/6 T2
q = 39.8°
Illustration 12 :
6 kg wt
Two blocks of mass 2.9 kg and 1.9 kg are suspended from a rigid support S by two inextensible wires each of lengths 1m. The
upper wire has negligible mass and the lower wire has uniform mass of 0.2 kgm–1. The whole system of blocks, wire and support
have an upward acceleration of 0.2 ms–2. (g = 9.8 ms–2). (i) find the tension at the mid-point of the lower wire (ii) find the tension
at the mid-point of the upper wire.
Sol. T1 – tension in the upper wire, which will be same through the wire due to its negligible mass
T2 – tension in the lower wire,
The force equation of mass m1,
S
T1 – T2 – m1g = m1a...........(i) and,
T2 – (m2 + m1)g = (m2 + m3)a............(ii), 1m
T1
where m3 = mass of the lower wire = 0.2 kg
From equation (i) and (ii), m1 = 2.9kg
T1 – (m1 + m2 + m3)g = (m1 + m2 + m3)a T2
or, T1 = (m1 + m2 + m3)(g + a) = (2.9 + 1.9 + 0.2) (9.8 + 0.2) = 50 N
1m T3
\ Tension at the mid-point of the lower wire be, T3.
The force-equation of mass m2,
m 2 = 1.9 kg
T3 – (m2 + m3/2)g = (m2 + m3/2)a,
or, T3 = (m2 + m3/2)(g + a) = (1.9 + 0.2/2)(9.8 + 0.2) = 20 N
\ Tension at the mid-point of the lower wire = 20 N.
108 Physics
Laws of Motion 109

Textbook Exercises
5.1 Give the magnitude and direction of the net force acting on (d) As the stone is lying on the floor of the train, its
(a) a drop of rain falling down with a constant speed, (b) a acceleration is same as that of the train.
cork of mass 10g floating on water, (c) a kite skillfully \ force acting on the stone,
held stationary in the sky, (d) a car moving with a constant F = ma = 0.1 × 1 = 0.1 N.
velocity of 30km/h on rough road (e) a high speed electron It acts along the direction of motion of the train.
in space far from all material objects, and free of electric 5.4 One end of a string of length l is connected to a particle of
and magnetic fields. mass m and the other to a small peg on a smooth horizontal
Sol. (a) As the raindrop is falling with a constant speed, its table. If the particle moves in a circle with speed v the net
acceleration a = 0, hence net force F = ma = 0. force on the particle (directed towards the centre) is :
(b) As the cork is floating on water, its weight is being
balanced by the upthrust (= weight of water displaced), mv 2 mv 2
(i) T, (ii) T – (iii) T + (iv) 0
hence net force on the cork = 0. l l
(c) As the kite is held stationary, net force on the kite is T is the tension in the string. [Choose the correct
zero, in accordance with Newton’s 1st law. alternative].
(d) As the velocity of car is constant, the acceleration Sol. (i) T
a = 0 hence F = ma = 0. The net force T on the particle is directed towards the
(e) As no fields (gravitational, electrical or magnetic) act centre. It provides the centripetal force required by
on the electron, net force = 0. the particle to move along a circle.
5.2 A pebble of mass 0.05 kg is thrown vertically upwards. 5.5 A constant retarding force of 50N is applied to a body of
Give the direction and magnitude of the net force on the mass 20kg moving initially with a speed of 15 ms–1. How
pebble, (a) during its upward motion, (b) during its downward long does the body take to stop?
motion, (c) at the highest point, where it is momentarily at Sol. Here, F = –50N, m = 20 kg, u =15 ms–1 and v = 0;
rest. Do your answers change if the pebble was thrown at Now, F = ma, a = F/m = –50/20 = –2.5 ms–2
an angle of 45° with the horizontal direction? Ignore air From relation, v = u + at, 0 = 15 – 2.6t, t = 15/2.5 = 6s
resistance. 5.6 A constant force acting on a body of mass 3.0kg changes
Sol. (a) Ignore air resistance. When the pebble is moving its speed from 2.0 m/s–1 to 3.5 m/s–1 in 25s. The direction
upward, the acceleration g is acting downward, so the of the motion of the body remains unchanged. What is the
force is acting downward is equal to magnitude and direction of the force?
F = mg = 0.05 kg × 10 ms–2 = 0.5 N. Sol. Here, m = 3.0 kg, u =2.0 ms–1 and v = 3.5 ms–1, t = 25s;
(b) In this case also F = mg = 0.05 × 10 = 0.5 N.(downwards). Now, F = ma = m (v – u )/t = 3.0 (3.5 – 2.0 )/ 25 = 0.18 N
(c) The pebble is not at rest at highest point but has \ the force is along the direction of motion.
horizontal component of velocity. The direction and 5.7 A body of mass 5 kg is acted upon by two perpendicular
magnitude of the net force on the pebble will not alter forces 8N and 6N. Give the magnitude and direction of the
even if it is thrown at 45° because no other acceleration acceleration of the body.
except ‘g’ is acting on pebble. Sol B 6N C
5.3 Give the magnitude and direction of the net force acting on F2
a stone of mass 0.1 kg (a) just after it is dropped from the
window of a stationary train, (b) just after it is dropped q
from the window of a train running at a constant velocity of
O F1 = 8N A
36 km/hr. (c) just after it is dropped from the window of a
train accelerating with 1ms–2 (d) lying on the floor of a Here m = 5 kg, F1 = OA = 8 N and F2 = OB = 6N
train which is accelerating with 1 ms–2 the stone being at From the adjoining figure,
rest relative to the train. Neglect air resistance throughout.
Sol. (a) Mass of stone = 0.1 kg Resultant force, F = F12 + F2 2 = 8 2 + 6 2 = 10 N
Net force, F = mg = 0.1 × 10 = 1.0 N. tan COA = tan q = AC/OA = OB/OA = 6/8 = 0.75
(vertically downwards). \ q = 36°52'
(b) When the train is running at a constant velocity, its which is the direction of the resultant force, and hence the
acceleration is zero. No force acts on the stone due to direction of the acceleration.
this motion. Therefore, the force on the stone is the \ a = F/m = 10/5 = 2 ms–2
same (1.0 N). 5.8 The driver of a three wheeler moving with a speed of 36
(c) The stone will experience an additional force F¢ (along km/h sees a child standing in the middle of the road and
horizontal) i.e., brings his vehicle to rest in 4.0s just in time to save the
F¢ = ma = 0.1 × 1 = 0.1 N child. What is the average retarding force on the vehicle?
As the stone is dropped, the force F¢ no longer acts The mass of the three-wheeler is 400 kg and the mass of
and the net force acting on the stone the driver is 65 kg.
F = mg = 0.1 × 10 = 1.0 N. Sol. Here, u = 36 km/h = 10 m/s, t = 4s,
(vertically downwards). m = 400 + 65 = 465 kg
110 Physics
Now, retarding force, F = ma = m (v – u )/t = 465 (0 – 10 )/ 4 stone = acceleration due to gravity = 10 ms–2 in the
= –1162.5 N. The negative sign shows that the force is a vertically downward direction. The path of the stone
retarding force. would be parabolic.
5.9 A rocket with a lift off mass 20,000 kg is blasted upwards 5.12 A bob of mass 0.1kg hung from the ceiling of a room by a
with an initial acceleration of 5.0 ms–2. Calculate the initial string 2m long is set into oscillation. The speed of the bob
thrust (force) of the blast. at its mean position is 1m/s–1. What is the trajectory of the
Sol. Here, m = 20000 kg, a = 5 ms–2 bob, if the string is cut when the bob is (a) at one of its
The thrust should be such that it overcomes the force of extreme positions (b) at its mean position?
gravity besides giving it an upward acceleration of 5 ms–2. Sol. (a) At each extreme position, velocity of the bob is zero.
Thus the force should produce a net acceleration of At the extreme position, it is only under the influence
9.8 + 5.0 = 14.8 ms–2. of gravity. Hence the bob will fall vertically downwards.
As thrust = force = mass × acceleration. (b) At each mean position, velocity of the bob is 1 ms–1
\ force, F = 20000 ×14.8 = 2.96 ×105N. along the tangent to the arc, which is in the horizontal
direction. If the string is cut at mean position, the bob
5.10 A body of mass 0.40 kg moving initially with a constant will behave as horizontal projectile. Hence, it will follow
speed of 10 m/s–1 to the north is subject to a constant force a parabolic path.
of 8.0 N directed towards the south for 30s. Take the instant 5.13 A man of mass 70 kg stands on a weighing scale in a lift,
the force is applied to be t = 0, and the position of the body at which is moving (a) upwards with a uniform speed of
that time to be x = 0, predict its position at t = –5s, 25s, 100s? 10 ms–1, (b) downwards with a uniform acceleration of
Sol. Here, m = 0.4 kg, u = 10 ms–1 due north 5 ms–2, (c) upwards with a uniform acceleration of 5 ms–2.
F = – 8 N, (negative sign shows the force directed opposite). What would be the readings on the scale in each case
F -8 (d) what would be the reading if the lift mechanism failed
Therefore, a = = = –20 ms–2 (0 £ t £ 30s) and it hurted down freely under gravity?
m 0.4
Sol. Given m = 70 kg, a = 10 ms–2
When t = –5s, x = ut = 10(–5) = –50 m (as a = 0) The weighing machine gives the reading of the reaction
When t = 25s, x = ut + ½ at2 = 10 × 25 force, R, which is apparent weight.
+ ½ (–20)(25)2 = – 6000 m (a) When lift is moving upwards with a uniform speed,
Upto t = 30s, motion is under acceleration, i.e. acceleration = 0,
x = ut + ½ at2 = 10 × 30 + ½ (–20)(30)2 = – 8700 m \ R = mg = 70 × 9.8 = 700 N
At t = 30s, v = u + at = 10 – 20 × 30 = – 590 m/s (b) When lift is moving downwards with acceleration
During t =30 to 100s, x2 = vt = – 590 × 70 = – 41300m = 5ms–2,
[as the force is removed a = 0] \ R = m(g – a) = 70 × (10 – 5) = 350 N
Total distance, x1 + x2 = –(8700 + 41300) m = –50 km (c) When lift is moving upwards with acceleration = 5 ms–2,
5.11 A truck starts from rest and accelerates uniformly at \ R = m(g + a) = 70 × (10 + 5) = 1050 N
2.0 ms–2. At t = 10s, a stone is dropped by a person standing (d) When lift is coming down freely under gravity,
on the top of the truck (6m high from the ground). What acceleration, a = g,
are the (a) velocity and (b) acceleration of the stone at \ R = m(g – g) = 0
t = 11s? (Neglect air resistance.) 5.14 Figure shows the position-time graph of a particle of mass
Sol. Given, u = 0, t = 10s, a = 2ms–2 4 kg. What is the (a) force on the particle for t < 0, t > 4s,
vx A 0 < t < 4s? (b) impulse at t = 0 and t = 4s? (Consider one-
O
q dimensional motion only).
vy
Sol. (a) When t < 0. As this part is horizontal, thus it can be
A C concluded that distance covered by the particle is zero
and hence force on the particle is zero.
The velocity of the truck when the stone is dropped,
v = u + at = 0 + 2 × 10 = 20 ms–1. x(m)
(a) When the stone is dropped, horizontal velocity,
vx= v = 20 ms–1; vx remains constant as air resistance is
3 A
neglected
In vertical direction, a =g = 9.8 ms–2, u = 0,
t =11–10 = 1s.
vy= v = u + gt = 0 + 10 × 1 = 10 m/s.
0 4 t (s)
Resultant velocity,
When 0 < t < 4s. As OA has a constant slope, hence in
this intervel, particle moves with constant velocity
v= v x 2 + v y 2 = 202 + 102 = 22.3 ms–1
æ3 -1 ö
Let q be angle which vx makes with resultant velocity, çè = 0.75 ms ÷ø . Hence force on the particle is zero.
4
q = tan–1 (vx/ vy) = tan –1 (10/20) = tan–1 (10.5) = 26° 34¢ When t > 4s. As this portion shows that particle always
(b) When the stone is dropped from the car, horizontal remains at a distance of 3 m from the origin i.e., the
force on the stone is zero. Only acceleration of the particle is at rest. Hence force on the particle is zero.
Laws of Motion 111

(b) Impulse at t = 0. Here u = 0, v = 0.75 ms –1 , 5.17 A nucleus is at rest in the laboratory frame of reference.
M = 4 kg Show that if it disintegrates into two smaller nuclei the
\ Impulse = total change in momentum products must move in opposite directions.
= Mv – mu r r
Sol. Let m1, m2 be the masses of products and v1, v2 be their
= M (v – u) = 4 (0.75 – 0) = 3 kg ms–1 respective velocities. Therefore, total linear momentum after
Impulse at t = 4s. Here u = 0.75 ms–1, v = 0 r r
disintegration = m1v1 + m2v2 . Before disintegration, the
\ Impulse = M (v – u) = 4 (0 – 0.75) nucleus is at rest.
= – 3 kg ms–1 . Therefore, its linear momentum before disintegration is zero.
5.15 Two bodies of masses 10 kg and 20 kg respectively kept on According to the principle of conservation of linear
a smooth, horizontal surface are tied to the ends of a light
momentum.
string. A horizontal force F = 600 N is applied to (i) A, r
r r r mv
(ii) B along the direction of string. What is the tension in m1v1 + m2 v2 = 0 or v2 = – 1 1
the string in each case? m2
r r
600N Negative sign shows that v1 and v2 are in opposite
Sol. Acceleration = = 20 ms–2 directions.
10 kg + 20 kg
5.18 Two billiard balls each of mass 0.05 kg moving in opposite
(i) When force is applied on 10 kg mass directions with speed 6 ms–1 collide and rebound with the
600 – T = 10 × 20 or T = 400 N same speed. What is the impulse imparted to each ball due
600 N A B to the other?
T T
10 kg 20 kg Sol. Given, initial momentum i ® of each ball before collision
= 0.05 (6) = 0.3kg ms–1,
When ball rebound after collision, speed would be reversed,
A B 600 N final momentum of each ball = 0.05(– 6) = – 0.3 kg ms–1,
T
10 kg 20 kg Impulse change in momentum
= final momentum – initial momentum
(ii) When force is applied on 20 kg mass, = (– 0.3 – 0.3) = – 0.6 ms–1 [–ve sign shows the two impulses
600 – T = 20 × 20 or T = 200 N are opposite in direction]
5.16 Two masses 8 kg and 12 kg are connected at the two ends 5.19 A shell of mass 0.020 kg is fired by a gun of mass 100 kg.
of a light inextensible string that goes over a frictionless If the muzzle speed of the shell is 80 ms–1 what is the
pulley. Find the acceleration of the masses and the tension recoil speed of the gun?
in the string, when the masses are released. Sol. Given, Mass of shell, m = 0.02 kg, Mass of gun, M = 100 kg
Sol. For block m2 ® m2 g – T = m2 a ...(i) and Muzzle speed of shell v = 80 ms–1
and for block m1 ® T – m1 g = m1 a ...(ii) According to the principle of conservation of linear
Adding (i) and (ii), we obtain momentum, mV + Mv = 0
(m2 – m1) g = (m2 + m1) a \ v = – mV/ M = – 0.02 × 80/100 = 0.016 ms–1 = – 0.16 cm/s
æ m - m1 ö (Negative sign shows that the gun moves in opposite
or a = ç 2 g direction of motion of shell.
è m2 + m1 ÷ø 5.20 A batsman deflects a ball by an angle of 45° without changing
12 - 8 4 ´ 10 its initial speed which is equal to 54 km/h. What is the
= ´ 10 = = 2ms -2 impulse imparted to the ball? (Mass of the ball is 0.15 kg.)
12 + 8 20
Substituting value of a in equation (ii), we obtain Sol. A D u cos a B
T = m1 (g + a) = 8 × (10 + 2) = 8 × 12 = 96 N. 22.5°
A u cos a P2
a 22.5° u
u a

X Y
O u sin a
T T a u
u sin a
a
T Let, XY– direction of the bat AO–direction of the ball before
the strike
OB – direction of the ball after the strike
a OD – normal to the bat XY
m1 = 8 kg T \ a = ÐDOA = 45/2 = 22.5°
Let, u– initial velocity of the ball along AO,
m1g m2 = 12 kg u cos a – component of the velocity along DO
u sin a– component of the velocity along XY
From the figure, it is clear that velocity along horizontal
m2g direction is just changing its direction not the magnitude.
112 Physics
\ Impulse on the ball = mu cos a – (– mu cos a) 5.23 Explain why (a) a horse cannot pull a cart and run in empty
= 2 mu cos a space, (b) passengers are thrown forward from their seats
= 2 × 0.15 × 15 cos 22.5° when a speeding bus stops suddenly, (c) it is easier to pull
= 4.16 kgms–1. a lawn mower than to push it? (d) a cricketer moves his
5.21 A stone of mass 0.25 kg tied to the end of string is whirled hands backwards while holding a catch ?
round in a circle of radius 1.5 m with a speed of 40 rev/min. Sol. (a) When a cart is pulled, the horse pushes the ground
in a horizontal plane. What is the tension in the string? with a force at an angle. The ground offers an equal
What is the maximum speed with which the stone can be reaction in opposite direction, through the feet of the
whirled around if the string can withstand a maximum horse. The horizontal component of this reaction helps
tension of 200N ? the cart to move in forward direction. In empty space,
Sol. Given, m = 0.25 kg, r = 1.5 m, reaction force would be zero, so horse cannot pull the cart.
n = 40 rpm = 40/60 rps (b) If the speeding bus stops suddenly, the lower part of
T = mrw2 the passengers, in contact with bus, would be at rest.
2 However, upper part of the body would still be in
40 4 æ 4ö
w = 2pn = 2p = p = 0.25 × 1.5 × ç ÷ × p2 = 6.6 N motion. Therefore, passengers would move forward.
60 3 è 3ø This occurs due to inertia of motion.
Tmax = mvmax2/r (c) When a lawn mower is pulled, the force acts along the
vmax = Ö( Tmaxr/ m) = Ö( 200 × 1.5/ 0.25) handle, which has two components. The upwardly
= Ö1200 = 34.6 ms–1 directed vertical component reduced the weight of
5.22 If, in exercise, the speed of the stone is increased beyond mower, however, horizontal component of the force
the maximum permissible value, and the string breaks helps to push forward the mower. Whereas, when a
suddenly, which of the following correctly describes the lawn mower is pushed, force is applied downwards
trajectory of the stone after the string breaks : and thereby, vertical component, directed downward,
(a) the stone moves radially outwards, increased the weight of mower, which creates
(b) the stone flies off tangentially from the instant the difficulties in pushing the mower. Therefore, pushing
string breaks, of mower is difficult than pulling the same.
(c) the stone flies off at an angle with the tangent whose (d) When a cricketer holds a catch, the impulse received
magnitude depends on the speed of the particle? at hand = F × t = change in linear momentum of the ball
Sol. (b) The velocity is tangential at each point of circular = constant. By moving the hand backward, the cricketer
motion. At the time the string breaks, the particle increases the time of impact and reduces the force, so
continues to move in the tangential direction according as the reaction, thereby reduces the chances of hurting
to Newton’s first law of motion. severely.

5.24 Figure shows the position-time graph of a body of mass = (4 × 10–4) – (– 4 × 10–4) = 8 × 10–4 kg ms–1
0.04 kg. Suggest a suitable physical context for this motion. Time between two consecutive impulses is 2s i.e, the ball
What is the time between two consecutive impulses received receives an impulse every 2s.
by the body? What is the magnitude of each impulse? 5.25 Figure shows a man standing stationary with respect to a
x(m) horizontal conveyor belt that is accelerating with 1 ms–2.
What is the net force on the man? If the coefficient of static
friction between the man’s shoes and the belt is 0.2, up to
2 what acceleration of the belt can the man continue to be
stationary relative to the belt? (mass of the man = 65 kg.)

2 4 6 8 10 12 14 16 t (s)
Sol. This graph can be of a ball rebounding between two walls
situated at position 0 cm and 2 cm. The ball is rebounding
from onewall toanother, timeand again every 2s with uniform
velocity.
displacement 2
Impulse. Here velocity = = = 0.01 ms–1 Sol. Here acceleration of conveyor belt a = ms –2 ,
time 100 ´ 2
ms = 0.2 and mass of man m = 65 kg. As the man is in an
Initial momentum = mu = 0.04 × 0.01 = 4 × 10–4 kg ms–1
accelerating frame, he experiences a pseudo force Fs = ma
Final momentum = mv = 0.04 × (– 0.01) = – 4 × 10–4 kg ms–1
as shown in Fig. (a). Hence to maintain his equilibrium, he
Magnitude of impulse = Change in momentum exerts a force F = – Fs = ma = 65 × 1 = 65 N in forward
Laws of Motion 113

direction i.e., direction of motion of belt. Sol. Given, speed of water v = 15 ms–1; Area of cross section, a
\ Net force acting on man = 65 N (forward) = 10–2 m2 and
As shown in Fig. (b), the man can continue to be stationary Volume of water coming out per second = a × v = 10–2 ×15
with respect to belt, it force of friction = 0.15 m2 s–1
ms N = ms mg = mamax amax = ms . g = 0.2 × 10 = 2 ms–2 \ mass of the water hitting the wall per sec = m
= volume × (density of water) = (0.15) × (103) = 150 kg–1.
N = mg
N = mg \ force excreted on the wall = mv /t = 150 × 15/ 1 = 2250 N
man 5.29 Ten one rupee coins are put on top of each other on a table.
Each coin has a mass m. Give the magnitude and direction of
(a) the force on the 7th coin (counted from the bottom) due to
Fs = ma all the coins on its top, (b) the force on the 7th coin by the 8th
coin and (c) the reaction of the 6th coin on the 7th coin.
ma us N Sol. (a) Force on the 7th coin = weight of the 3 coins lying
above 7th coin, F = 3 mg N (acts vertically downwards)
(b) Eight coin is under the weight of 2 coins above it and
mg its own weight, force on 7th coin due to 8th coin = sum
mg
(a) (b) of the 3 coins = 2 mg + mg = 3 mg N
5.26 A stone of mass m tied to the end of a string is revolves in a (c) Sixth coin is under the weight of 4 coins above it,
vertical circle of radius R. The net force at the lowest and therefore, reaction = – F = – 4 mg N (negative sign
highest points of the circle directed vertically downwards shows the reaction is vertically upwards)
are : (Choose the correct alternative). 5.30 An aircraft executes a horizontal loop at a speed of 720
km/h with its wings banked at 15°. What is the radius of
Lowest Point Highest Point the loop?
(a) mg – T1 mg + T2 Sol. Given,
(b) mg + T1 mg – T2 a = 15°, v = 720 km/h = 200 m/s, g = 10 ms–2. \ tana = v2/rg,
(c) mg + T1 – (mv12)/R mg – T2 + (mv12)/R r = v2/g tan a = (200)2/ (10 × tan 15°) = 15.23 m
(d) mg – T1 – (mv12)/R mg + T1 + (mv12)/R 5.31 A train runs along an unbanked circular track of radius
T1 and v1 denote the tension and speed at the lowest point. 30m at a speed of 54 km/h. The mass of the train is 106 kg.
T2 and v2 denote corresponding values at the highest point. What provides the centripetal force required for this
purpose, the engine or the rails? What is the angle of
Sol. The net force at the lowest point is (mg – T1) and the net
banking required to prevent wearing out the rails?
force at the highest point is (mg + T2). Therefore, alternative Sol. Here, lateral thrust exerted by the rails to wheel is providing
(a) is correct. the necessary centripetal force and the train would exert an
Since mg and T1 are in mutually opposite directions at lowest equal and opposite thrust (by Newton’s 3rd law) on the rails
point and mg and T2 are in same direction at the highest \ tan a = v2/rg,
point. Given, r = 30 m, v = 54 km/h = 15 m/s, g = 10 ms–2.
5.27 A helicopter of mass 1000 kg rises with a vertical a = tan–1 v2/ rg tan–1 (15)2/ (30 × 10) » 37°
acceleration of 15 ms–2. The crew and the passengers 5.32 A block of mass 25 kg is raised by a 50 kg man in two
weigh 300 kg. Give the magnitude and direction of the (a) different ways as shown in Fig. What is the action on the
force on the floor by the crew and passengers (b) action of floor by the man in the two cases? If the floor yields to a
the rotor of the helicopter on the surrounding air, (c) force normal force of 700 N, which mode should the man adopt to
on the helicopter due to the surrounding air. lift the block without the floor yielding?
Sol. Given, Mass of the helicopter, m 1 = 1000 kg,
Mass of the crew and passengers, m2 = 300 kg, upwards
acceleration a = 15 ms–2 and g = 10 ms–2.
(a) Force on the floor of helicopter by the crew and
passengers = apparent weight of crew and passengers
= m2 (g + a) = 300 (10 + 15) = 7500 N
(b) Action of the rotor of helicopter is downwards, when
helicopter rising up, therefore, force of action
= (m2 + m1) (g + a) = (1000 + 300) (9.8 + 15) = 32500 N
(vertically downwards)
(c) Force on the helicopter due to the surrounding air is
the reaction and would equal to the force of action
(since action = reaction), therefore, force of reaction 25 kg 25 kg
= 32500 N (vertically upwards) (a) (b)
5.28 A stream of water flowing horizontally with a speed of Sol. In 1st case, man applies an upward force of 25 kg wt., (same
15 ms–1 pushes out of tube of cross sectional area as the weight of the block) According to Newton’s third law
10–2 m2 and hits at a vertical wall nearby. What is the of motion, there will be a downward reaction on the floor.
force exerted on the wall by the impact of water, assuming The action on the floor by the man
it does not rebound? = 50 kg wt.+ 25 kg wt. = 75 kg wt.= 75 × 10 = 750 N.
114 Physics
In case II, the man applies a downward force of 2 kg wt. Sol. Here, Mass of A, mA = 5 kg and Mass of B, mB = 10 kg;
According to Newton’s third law, the reaction is in the Coefficient of friction, m = 0.15; Force acting on A, F = 200N
upward direction. (a) reaction of the partition = – 200 N
In this case, action on the floor by the man Q (FBlock wall = – Fwall Block)
= 50 kg wt.– 25 kg wt = 25 kg wt.= 25 × 10 = 250 N. (b) Since A presses B with a force of 200N towards right
Therefore, the man should adopt the second method. \ B presses A with a force of 200 N towards right.
5.33 A monkey of mass 40 kg climbs on a rope (fig.) which can When the wall of the partition is removed, the block
stand a maximum tension of 600 N. In which of the following system will move due to the applied force & kinetic
cases will the rope break: the monkey friction will come into the picture.
(a) climbs up with an acceleration of 6 ms–2
\ 200 – m ( m A + mB ) g = (m A + mB )a
(b) climbs down with an acceleration of 4 ms–2
(c) climbs up with a uniform speed of 5 ms–1 200 – µ(m A - mB ) g
(d) falls down the rope nearly freely \a=
mA + mB
under gravity? (Ignore the mass of
the rope). 200 – 0.15(5 + 10)10 117.5
= = =11.8 ms–2
5 + 10 15
Sol. (a) When the monkey climbs up with an acceleration, then If the bodies are in motion, this force will overcome the
T – mg = ma force of friction (f) & the reaction of block B on A (fBA).
where T represents the tension (figure a). So under equillibrium
\ T = mg + ma = m (g + a) 200 – f = mAa + FBA
or T = 40 kg (10 + 6) ms–2 = 640 N FBA = 200 – µmAg – mAa = 200 – (µg – a)mA
But the rope can withstand a maximum tension of 600 = 200 – (0.15 × 10 – 11.8) 5 = 200 + 51.5 = 251.5 N
N. So the rope will break. 5.35 A block of mass 15 kg is placed on a long trolley. The
T T coefficient of static friction between the block and the trolley
is 0.18. The trolley accelerates from rest with 0.5 ms–2 for
a
m 20s and then moves with a uniform velocity. Discuss the
motion of the block as viewed by (a) a stationary observer
on the ground (b) an observer moving with the trolley.
Sol. Given, m = 15 kg, m = 0.18, a = 0.5 ms–2, t = 20 s,
Force acting on the block due to the motion of the trolley, F¢
m a = ma = 15 × 0.5 = 7.5 N, which acts in the forward direction.
Force due to the liming friction on the block,
mg mg F = mR = mmg = 0.18 × 15 × 9.8 = 26.46 N, which opposes the
(a) (b) motion of the block
(b) When the monkey is climbing down with an Q F ¢ < F , the block will not move
acceleration, then
\ (a) for stationary observer, the block would be at rest with
mg – T = ma (figure (b))
Þ T = mg – ma = m (g – a) respect to the trolley.
or T = 40 kg (10 – 4) ms–2 = 240 N (b) When the observer moves with an acceleration with
The rope will not break. the trolley, he would be in a non-inertial frame, where
(c) When the monkey climbs up with uniform speed, then law of inertia would not be valid. The box will be at rest
T = mg = 40 kg × 10 ms–2 = 400 N [Q a = 0] relative to the observer.
The rope will not break. 5.36 The rear side of a truck is open and a box of 40 kg mass is
(d) When the monkey is falling freely, it would be a state placed 5 m away from the open end as shown in Fig. The
of weightlessness. So, tension will be zero and the coefficient of friction between the box and the surface below
rope will not break.
it is 0.15. On a straight road, the truck starts from rest
5.34 Two bodies A and B of masses 5 kg and 10 kg in contact
with each other rest on a table against a rigid wall. The and accelerates with 2 m s–2. At what distance from the
coefficient of friction between the bodies and the table is starting point does the box fall off the truck? (Ignore the
0.15. A force of 200N is applied horizontally at A. What are size of the box).
(a) the reaction of the partition (b) the action-reaction forces
between A and B? What happens when the partition is
removed? Does the answer to (b) change, when the bodies
are in motion? Ignore difference between ms and mk.
Sol. Force experienced by box, F = ma = 40 × 2 = 80 N
Frictional force Ff = mmg = 0.15 × 40 × 10 = 60 N.
A B Net force = F – Ff = 80 – 60 = 20 N.
Backward acceleration produced in the box,
200 N
Laws of Motion 115

20 æ Net force ö The minimum speed required to perform a vertical loop is


a= ç ÷ø Þ a = 0.5 ms–2 given by equation (1) when R = 0.
40 è m
If t is time taken by the box to travel s = 5 metre and fall off 2
mvmin
the truck, then from \ mg = or v2min. = gr
r
1 1
s = ut+ at2 5 = 0 × t + × 0.5 t2 or V = gr = 10 ´ 25 ms -1 = 15.8 ms–1
2 2
5.39 A 70 kg man stands in contact against the inner wall of a
5´2 hollow cylindrical drum of radius 3 m rotating about its
t= = 4.47 s.
0.5 vertical axis with 200 riv/min. The coefficient of friction
If the truck travels a distance x during this time, then again between the wall and his clothing is 0.15. What is the
from minimum rotational speed of the cylinder to enable the
1 1 man to remain stuck to the wall (without falling) when the
S = ut + at2 x = 0 × 4.47 + × 2 (4.47)2 = 19.98 m. floor is suddenly removed?
2 2
1 Sol. Given, m = 70 kg, r = 3 m, n = 200 rpm = 200/60 rps,
5.37 A disc revolves with a speed of 33 rev/min and has a and m = 0.15
3
w
radius of 15 cm. Two coins are placed at 4 cm and 14cm
away from the center of the record. If the coefficient of
friction between the coins and the record is 0.15, which of
f = mN
the coins will revolve with the record?
Sol. When the frictional force is sufficient to provide the N
centripetal force the coin revolves with the disc. Coin would mg
slip the record, if this force is not sufficient.
Force of friction, f = mR = mmg,
Centripetal force, mv2/r = m r w2
In order to prevent slipping, mmg ³ m r w2, i.e. mg ³ r w2
Horizontal force provided by the wall on the man N =
1
Case 1: Given, r = 4 cm = 0.04 m, n = 33 rev/min centripetal force = m r w2. Frictional force, acting vertically
3 upwards, opposes the weight of the man.
1 If floor is removed, man would stuck the wall, when
w = 2pn = 2 × (22/7) × ( 33 rev/min) = 3.49s–1.
3 mg = f < mN i.e. g < mrw2.
\ r w2 = (.04) (3.49)2 = 0.49 ms–2 \ Minimum angular speed of rotation of the cylinder is
and mg = 0.15 × 9.8 = 1.47 ms–2 w = Ö(g/mr) = Ö[10/(0.15 × 3)] = 4.7rad/s.
Q mg ³ r w2, hence first coin will rotate with the record. 5.40 A thin circular loop of radiusR rotates about its vertical
Case 2: Given, r = 14cm = 0.14m, diameter with an angular frequency w, Show that a small
and w = 3.49s–1. bead on the wire loop remains at its lowermost point for w
\ r w2 = (.14) (3.49)2 = 1.7 ms–2 and mg £ Ög/R. What is the angle made by the radius vector joining
= 0.15 × 9.8 = 1.47 ms–2 the center to the bead with the vertical downward direction
\ mg is less than rw2, hence second coin will not rotate for w = Ö2g/R? Neglect friction.
with the record. Sol. q - angle between radius vector joining the bead to the
5.38 You may have seen in a circus a motorcyclist driving in center and vertical downward direction.
vertical loops inside a ‘death well’ (a hollow spherical
N – normal reaction.
chamber with holes, so the spectators can watch from
outside). Explain clearly why the motorcyclist does not drop mg = N cos q …(i)
down when he is at the uppermost point, with no support and m r w2 = N sin q N cos q
or, m (R sin q) w2 = N sin q O N
from below. What is the minimum speed required at the q
Þ Rw2 = N…(ii)
uppermost position to perform a vertical loop if the radius rR q
of the chamber is 25 m? From (i) and (ii), N sinq P
Sol. When the motorcyclist is at the highest point of the death- mg = mRw2 cos q
well, the normal reaction R on the motorcyclist by the ceiling Þ cos q = g/ Rw2 mg
of the chamber acts downwards. His weight mg also acts Since cos q £1, bead will emain its lower most point.
downwards. These two forces are balanced by the outward
Þ g/ Rw2£1 or, w£Ö(g/R)
centrifugal force acting on him.
1
mv 2 If, w = Ö(2g/R), cosq = g/R (2g/R) =
\ R + mg = … (1) 2
r i.e. q = 60°
Here v is the speed of the motorcyclist and m is the mass of
the motorcyclist (including the mass of the motor cycle).
Because of the balancing of the forces, the motorcyclist
does not fall down.
116 Physics

Practice Questions
Very Short Answer Questions [1 Mark Qs.] 22. “Friction is a self-adjusting force.” Is this statement correct.
23. What provides the centripetal force in the following cases?
1. State Newton’s first law of motion. (i) Electron revolving around the nucleus.
2. Two objects having different masses have same (ii) Earth revolving around the sun.
momentum. Which one of them will move faster. (iii) Car taking a turn on a banked road.
3. What is the effect on the acceleration of a particle if the 24. (a) A body is moving with uniform velocity. Can it be said
net force on the particle is doubled? to be in equilibrium? Why?
4. Why does a heavy rifle not kick as strongly as a light rifle (b) Why Newton’s second law of motion is not applicable
using the same cartridges? to the motion of a rocket?
5. On what factors does the thrust on a rocket depend? 25. Name the forces which are in equilibrium in each of the
6. What do you mean by normal reaction? following situations :
7. At which place on earth, the centripetal force is maximum? (a) a book resting on a table.
8. What is the angle between frictional force and (b) a cork floating on water.
instantaneous velocity of the body moving over a rough (c) a pendulum bob suspended from the ceiling with the
help of a string.
surface?
26. A force of 5N changes the velocity of a body from 10 ms–1
9. Is earth an inertial frame of reference?
to 20 ms–1in 5s. How much force is required to bring about
10. What provides the centripetal force in the following cases? the same change in 2s.
(i) Electron revolving around the nucleus. 27. Sand is thrown on the tracks covered with snow why?
(ii) Earth revolving around the sun. 28. Why is it easier to maintain the motion than to start it?
11. Can a body in linear motion be in equilibrium? 29. Why are the wheels circular in shape?
12. Why are curved roads generally banked? 30. Rubber tyres are preferred to steel tyres, why?
13. A person sitting in the compartment of a train moving with 31. What is the angle of friction between two surfaces in contact,
a uniform speed throws a ball in the upward direction. What if coefficient of friction is 1/Ö3?
path of the ball will appear to him? What to a person
standing outside?
Short Answer Questions [2 or 3 Marks Qs.]
14. The length of an ideal spring increases by 0.1cm when a 1. (a) An astronaut accidently gets separated out of his small
body of 1kg is suspended from it. If this spring is laid on a spaceship accelerating in inter-steller space at a
frictionless horizontal table and bodies of 1 kg each are constant rate 100 ms–2. What is the acceleration of the
suspended from its ends, then what will be the increase in astronaut at the instant after he is outside the
its length? spaceship?
15. The two ends of a spring-balance are pulled each by a (b) How is it that a stone dropped from a certain height
force of 10 kg-wt. What will be the reading of the balance? falls much more rapidly as compared to a parachute
16. A retarding force is applied to stop a motorcar. If the speed under similar conditions?
of the motorcar is doubled, how much more distance will it 2. Explain how proper inflation of tyres saves fuel?
cover before stopping under the same retarding force? 3. In a circus in the game of swing, the man falls on a net after
17. A lift is accelerated upward. Will the weight of person inside leaving the swing but he is not injured, why?
4. (a) A meteorite burns in the atmosphere before it reaches
the lift increase, decrease or remain the same relative to its
the Earth’s surface. What happens to its momentum?
real weight? If the lift is going with uniform speed, then?
(b) By accident a person fell on a floating ice slab in a
18. A ball of 0.5 kg mass moving with a speed of 10m/s
pond. He has nothing with him to get out and the ice
rebounds after striking normally a perfectly elastic wall. slab is big enough so that he cannot put his hands or
Find the change in momentum of the ball. feet in water. How can he save himself by coming out
19. A body is suspended from the ceiling of a transparent of the ice slab?
cabin falling freely towards the earth. Describe the motion 5. A block of mass M is placed on a frictionless, inclined plane
of the body as observed by an observer (a) sitting in the of angle q, as shown in the figure. Determine the acceleration
cabin, (b) standing on the earth. of the block after it is released. What is force exerted by the
20. Calculate the net force acting on a body of mass 10 kg, incline on the block?
moving with a uniform velocity of 2 ms–1.
21. (a) According to Newton’s third law of motion, every
force is accompanied by an equal (in magnitude) and
opposite (in direction) force called reaction, then how M
can a movement takes place?
(b) You can move a brick easily by pushing it with your
foot on a smooth floor, but, if you kick it, then your
foot is hurt. Why?
q
Laws of Motion 117

6. Show that the total linear momentum of an isolated system 7. Consider a mass ‘m’ attached to a string of length ‘l’
of interacting particles is conserved. performing vertical circle. Find an expression for the
7. State the law of conservation of momentum. Establish the (i) velocity at any point,
same for a ‘n’ body system. (ii) tension at any point,
8. State the laws of limiting friction. Hence define coefficient
of friction. Numerical Questions [3 or 5 Marks Qs.]
9. Vehicles stop on applying brakes. Does this phenomenon
violate the principle of conservation of momentum? 1. A force produces an acceleration of 16 m/s2 in a body of mass
10. Four blocks of the same mass m connected by cords are 0.5 kg, and an acceleration of 4 m/s2 in another body. If both
pulled by a force F on smooth horizontal surface, as shown the bodies are fastened together, then what is the acceleration
in figure. Determine the tensions T1, T2, and T3 in the cords. produced by that force?
11. Two bodies of masses M and m are allowed to fall freely 2. A 20 gm bullet moving at 300 m/s stops after penetrating 3
from the same height. If air resistance for each body is same, cm of bone. Calculate the average force exerted by the bullet.
then will both the bodies reach the earth simultaneously? 3. A stone of mass 5 kg falls from the top of a cliff 50 m high
12. A bird is sitting on the floor of a closed glass cage in the and buries 1 m deep in sand. Find the average resistance
hands of girls. Will the girl experience any change in the offered by the sand and the time it takes to penetrate.
weight of the cage when the bird (i) starts flying in the cage 4. A force of 50 N is inclined to the vertical at an angle of 30°.
with a constant velocity ? (ii) flies upwards with acceleration?
Find the aceeleration it produces in a body of mass 2 kg
(iii) flies downwards with acceleration ?
which moves in the horizontal direction.
13. The driver of a truck travelling with a velocity v suddenly
notices a brick wall in front of him at a distance d. It is better 5. A gun weighing 10 kg fires a bullet of 30 g with a velocity of
for him to apply brakes or to make a circular turn without 330 m/s. With what velocity does the gun recoil? What is
applying brakes in order to just avoid crashing into the the resultant momentum of the gun and the bullet before
wall? Why? and after firing?
14. Derive a formula for the acceleration of a body moving down 6. An elevator weighs 4000 kg. When the upward tension in
on a rough inclined plane. the supporting cable is 48000 N, what is the upward
15. Derive a relation between angle of friction and angle of acceleration? Starting from rest, how far does it rise in 3
repose. seconds?
16. Derive an expression for the acceleration and the tension in 7. A machine gun has mass of 20kg. It fires 25g bullet at the
the string for a system of two masses connected by a string
rate of 600 bullets per minutes with a speed of 200 m/s.
passing over a smooth frictionless pulley.
17. Why does water from a bucket not fall even when it is upside Calculate (i) recoil velocity of the gun and, (ii) force required
down while rotating in a vertical circle? to keep the gun in position.
18. Derive the maximum angle by which a cyclist can bend while 8. A ball of mass 100 g falls from a height of 5 m and rebounds
negotiating a curved path. to a height of 1.25 m. Calculate the impulse and average
force between the ball and the ground, if the time during
Long Answer Questions [5 Marks Qs.]
which they are in contact is 0.1s [g = 10 m/s2]
1. State Newton’s second law of motion. How does it help to 9. A ball of mass 20g hits a wall at an angle of 45° with a
measure force. Also state the units of force. velocity of 15 m/s. If the ball rebounds at 90° to the direction
2. State Newton’s third law of motion. Discuss its of incidence, calculate the impulse received by the ball.
consequences.
3. A uniform rod is made to lean between a rough vertical wall 10. A rocket of initial mass 6000 kg ejects mass at a constant
and the ground. Show that the least angle at which the rod rate of 16 kg/s with constant relative speed of 11 km/s. What
can be leaned without slipping is given by is the acceleration of the rocket one minute after blast?
11. A horizontal force of 1.2 kg is applied on a 1.5 kg block,
-1 æ 1 - m1m 2 ö
q = tan ç
è 2m 2 ÷ø
which rests on a horizontal surface. If the coefficient of
friction is 0.3, find the acceleration produced in the block.
where m1 and m2 stand for the coefficient of friction between 12. A body weighing 20 kg just slides down a rough inclined
(i) the rod and the wall and (ii) the rod and the ground. plane that rises 5 m in every 12 m. What is the coefficient of
4. Name a varying mass system. Derive an expression for friction ?
velocity of propulsion of a rocket at any instant 13. A block of mass 2 kg rests on a plane inclined at 30° with the
5. Obtain an expression for minimum velocity of projection of horizontal. The coefficient of friction between the block and
a body at the lowest point for looping a vertical loop.
the surface is 0.7. What will be the frictional force acting on
6. (i) Define friction.
the block?
(ii) Show that kinetic friction is less than the static friction.
14. What is the smallest radius of a circle at which a cyclist can
(iii) Establish that static friction is a self-adjustable force.
travel if its speed is 36 km/h, given that he bends by an
(iv) Write the laws of limiting friction.
angle 45° while turning [g = 10 m/s2]
118 Physics
15. A stone tied to one end of a string 70 cm long is whirled in a 17. A plane describes a vertical circle when looping. Find the
horizontal circle with a constant speed. If the stone makes 10 radius of the greatest possible loop if the velocity of the
revolutions in 20s, what is the direction and magnitude of the plane at the lowest point of its path is 50 m/s. [g = 10 m/s2]
acceleration of the stone? 18. A particle moves in a circle of radius 20 cm. It’s linear speed
16. Find the force required to move a train of mass 5000 quintals is given by v = 2t where t is in second and v in metre/s. Find
up an incline of 1 in 50 with an acceleration of 2 m/s2. [Take the radial and tangential acceleration at t = 3s.
force of friction = 0.2 N/quintal and g = 10m/s2]

HOTS/Exemplar Questions
Very Short Answer Questions [1 Mark Qs.] 3. A person in an elevator accelerating upwards with an
acceleration of 2 ms–2, tosses a coin vertically upwards
1. Is it possible to have motion in the absence of a force? Is it
possible to have force in the absence of motion? [HOTS] with a speed of 20 m s–1. After how much time will the coin
2. When the pellet fired into the spiral tube emerges, which fall back into his hand? (g = 10 ms–2) [Exemplar]
path will it follow? (Neglect gravity) [HOTS]
4. A block of mass M is held against a rough vertical wall by
pressing it with a finger. If the coefficient of friction between
the block and the wall is m and the acceleration due to gravity
is g, calculate the minimum force required to be applied by
the finger to hold the block against the wall? [Exemplar]

Long Answer Questions [5 Marks Qs.]


1. Two masses each equal to m are lying on X-axis at (–a, 0) and
A C
(Top view)
B (+ a, 0) respectively as shown in fig. They are connected by
a light string. A force F is applied at the origin and along the
3. At the moment an object that has been tossed upward into Y-axis. As a result, the masses move towards each other.
the air reaches its highest point, is it in equilibrium? Defend What is the acceleration of each mass? Assume the
your answer. [HOTS] instantaneous position of the masses as (– x, 0) and (x, 0)
respectively.
4. A person driving a car suddenly applies the brakes on seeing
F
a child on the road ahead. If he is not wearing seat belt, he
falls forward and hits his head against the steering wheel.
Why? [Exemplar]
5. Why are mountain roads generally made winding upwards
(–a, 0) (a, 0)
rather than going straight up? [Exemplar] –X X
m O m
[HOTS]
Short Answer Questions [2 or 3 Marks Qs.] 2. A 0.30 kg hockey puck slides on the horizontal frictionless
surface of an ice rink. It is struck simultaneously by two
1. In the absence of air friction, it is claimed that all objects fall different hockey sticks. The two constant forces that act on
with the same acceleration. A heavier object is pulled to the the puck as a result of the hockey sticks are parallel to the
Earth with more force than a light object. Why does the ice surface and are shown in the pictorial representation in
heavier object not fall faster? [HOTS] figure. The force F1 has a magnitude of 5.0 N. and F2 has a
2. Note the readings on the scales. Rustam the painter has a magnitude of 8.0 N. Determine the acceleration of the puck
weight of 600 N and carries a 100-N bucket of point. What is while it is in contact with the two sticks.
the weight of the scaffold? y
F2

700 N 500 N F1= 5.0 N


F2= 8.0 N
60°
x
20°

F1
[HOTS] [HOTS]
Laws of Motion 119

3. A helicopter of mass 2000 kg rises with a vertical acceleration 4. A cricket bowler releases the ball in two different ways.
of 15 ms–2. The total mass of the crew and passengers is (a) giving it only horizontal velocity, and
500 kg. Give the magnitude and direction of the (g = 10 ms–2)
(b) giving it horizontal velocity and a small downward
(a) force on the floor of the helicopter by the crew and velocity.
passengers.
The speed vs at the time of release is the same. Both
(b) action on the rotor of the helicopter on the surrounding
are released at a height H from the ground. Which one
air.
will have greater speed when the ball hits the ground?
(c) force on the helicopter due to the surrounding air. Neglect air resistance. [Exemplar]
[Exemplar]

CHAPTER TEST

Time : 45 min. Max. Marks : 24

Directions : (i) Attempt all questions


(ii) Questions 1 to 5 carry 1 mark each.
(iii) Questions 6, 7 and 8 carry 3 marks each.
(iv) Questions 9 and 10 carry 5 marks each.

1. A stone is tied to one end of a string and rotated in a vertical circle. What is the difference in tension of the string at lowest
and highest points of the vertical circle?
2. Can a body remain in rest when external forces are acting on it?
3. An athlete runs a certain distance before taking a long jump. Why?
4. A bullet of mass 50 g moving with a speed of 500 m/s is brought to rest in 0.01s. Find the impulse.
5. A cricket ball is rolled on ice with a velocity of 5.6 m/s and comes to rest after travelling 8 m. Find the coefficient of friction.
[g = 9.8 m/s2]
6. State the principle of conservation of linear momentum. Explain, the recoiling of a gun when a bullet is fired from it with this
principle.
7. A block of mass M is supported by a cord C from a rigid support, and another cord D is attached to the bottom of the block.
If you give a sudden jerk to D, it will break. But if you pull on D steadily, chord C will break. Why ?

8. When a horse pulls a cart, the cart also pulls it with an equal and opposite force by Newton’s third law. Explain how the cart
moves forward.
9. Two objects of masses m1 and m2 are tied at the two ends of a light inextensible string passing over a frictionless pulley. Find
expressions for the acceleration of the system and the tension in the string.
10. An object is first sliding down an inclined plane of inclination q with some constant velocity. It is next projected up the plane
with an initial velocity u. Calculate the distance upto which it will rise before coming to rest. What will happen to the object
after it has come to rest?
120 Physics

Solutions
PRACTICE QUESTIONS 22. This statement is correct only so long as the friction is
static friction. Upto the limiting friction, the force of friction
Very Short Answer Questions is equal (in magnitude) and opposite to the applied force.
23. It is provide by-
1. Everybody continues to be in state of rest or of uniform (i) Electrostatic force of attraction between the electron
motion in a straight line unless compelled by some external and the nucleus.
force to act otherwise. (ii) Gravitational force of attraction between Earth and Sun.
2. Object with smaller mass as momentum r = mv
(iii) A component of the reaction of the road.
E 24. (a) Yes, it can be said to be in equilibrium when it moves
3. Since, a = . On doubling the force, the acceleration will
m with uniform velocity as no acceleration i.e. no net
also be doubled. force acts on the body.
(b) Newton’s second law i.e. F = ma is applicable only if
mv
4. The recoil speed of rifle V = is inversely proportional the mass (m) of the body remains constant. In case of
M the rocket, the mass continuously decreases and hence
to its mass. So far a heavy rifle the kick is less stronger. F = ma is not applicable.
5. The upward thrust on a rocket depends on exhaust speed 25. (a) Gravitational force on the book and a force of reaction
of the gases w.r.t. the rocket and rate at which mass of the of the table.
exhaust gases escapes. (b) Gravitational force on the cork and an upward thrust
6. It is the reaction due to the surface on which the body or buoyant force of water.
moves. It acts perpendicular to the surface of contact.
(c) Gravitational force on the bob and the tension in the
mv 2 string.
7. Since F = , so at the pole, the value of F is maximum. 26. F1 = dp/dt1 and F2 = dp/dt2
r
\ F2 = F1 dt1/dt2 = 5 × 5/2 = 12.5N
8. The angle is 180°, because force of friction always opposes
the relative motion. 27. Sand increases the friction
9. Since earth rotates on its own axis and also revolve around 28. As the dynamic friction is less than the force of limiting
the sun, there will be acceleration associated. So earth friction.
cannot be taken as inertial frame of reference. 29. Wheels convert the sliding friction to rolling friction.
10. (i) Electrostatic force. 30. Coefficient of friction between the rubber and road is smaller
(ii) Gravitational force. than that between the steel and road.
11. Yes, provided the vector sum of the forces acting upon the
body is zero. 1 1
31. Here, m = , tan q = \ q = 30°.
12. Curved roads are generally banked so as to help in providing 3 3
centripetal force needed for motion of vehicles on the curved
road. Short Answer Questions
13. It will appear to fall in vertically downward direction to the 1. (a) According to Newton’s first law of motion, the moment
person in the compartment and parabolic to a person he is out of the spaceship, there is no external force on
standing outside.
the astronaut, thus his acceleration is zero. Here we
14. It would increase in length by 0.1cm only.
are assuming that he is out of the gravitational field of
15. 10 kg-wt.
heavenly bodies i.e. there are no nearby stars to exert
16. Since distance µ (velocity) 2, motorcar will cover the
gravitational force on him and the small spaceship exerts
distance four times longer than before.
17. Apparent weight will increase. When lift is going up with negligible gravitational attraction on him.
uniform speed, apparent weight will be same as real weight. (b) As the surface area of a parachute is much larger as
18. Change in linear momentum = – mv – mv = – 2mv = –2 × 0.5 compared to the surface area of a stone, so the air
× 10 = – 10 kgms–1. resistance, i.e. fluid friction in case of parachute is much
19. (a) The body will appear stationary in air, (b) The body larger than in case of stone. Hence the parachute falls
will appear falling freely under the acceleration in air. slowly.
20. In uniform velocity, 2. When the tyres are properly inflated, the area of contact
acceleration = 0, F = ma = 0. between the tyres and the ground is reduced which in turn
21. (a) As the action and reaction never act on the same body, reduces the rolling friction. As a result of this, there is less
so the motion is possible. dissipation of energy against friction. So the automobiles
(b) As Ft remains constant, so if t is reduced, then F will cover greater distance for the same quantity of fuel
be increased and hence hurt our foot. consumed. Hence proper inflation of tyres leads to saving
the fuel.
Laws of Motion 121

3. When the man falls on the net it is depressed where man r r r r r r


( p¢ A - p A ) + ( p¢ B - p B ) = FAB Dt + FBA Dt
falls on it and thus the time of contact is increased. Due to
this, force of reaction on the man is reduced to a great extent. r r
= – FBA Dt + FBA Dt = 0
Because the increase in time reduces the impulse which is
r r r r
equal to the changes in momentum (FDt = mDv). So, F is Þ pA¢ + pB¢ = pA + pB
quite less and the man is not injured by the net. In fact F
Which shows that the total final momentum of the isolated
pushes him up once and again he falls on the net.
system is exactly same as its initial momentum. Thus, it is
4. (a) A meteorite while travelling towards earth shares its
proved that total momentum of an isolated system remains
momentum with atmospheric particles and the
remaining momentum is imparted to the Earth. conserved.
(b) The person should know how to generate a force so 7. When no external force acts on a system the momentum will
that the ice slab is thrown away opposite to the direction remain conserved. Consider a system of a, n bodies of
of the force alongwith. This, he can do by spitting masses m1, m2 , m3...., mn. If p1, p2, p3, ...., pn are the
forcefully or sneezing etc. again and again till the slab momentum associated then, the rate of change of momentum
reaches the shore of the lake from where he can get with the system,
out from the slab by the same technique. dp dp dp dp dp
5. When the block is released, it will move down the incline. = 1 + 2 + 3 + ..... n
dt dt dt dt dt
Let its acceleration be a. As the surface is frictionless, so d
the contact force will be normal to the plane. Let it be N. = (p + p + p + .... + pn)
dt 1 2 3
Here, for the block we can apply equation for motion along
the plane and equation for equilibrium perpendicular to the dp
If no external force acts, =0
plane. dt
i.e., Mg sin q = Ma Þ a = g sin q \ p = constant, i.e., p1 + p2 + p3 + .... + pn = constant.
Also, Mg cos q – N = 0 Þ N = Mg cos q. 8. The laws of limiting friction are as follows :
(1) The value of limiting friction depends on the nature of
the two surfaces in contact and on the state of their
smoothness.
(2) The force of friction acts tangential to the surfaces in
contact in a direction opposite to the direction of
N relative motion.
(3) The value of limiting friction is directly proportional to
the normal reaction between the two given surfaces.
(4) For any two given surfaces and for a given value of
Mg cos q
normal reaction the force of limiting friction is
Mg Mg sin q independent of the shape and surface area of surfaces
r in contact. Coefficient of limiting friction for two given
6. Consider two bodies A and B, with initial momenta p A and
surfaces in contact is defined as the ratio of the force
r
p B respectively. Let the two bodies collide, get apart and of limiting friction f1 between them and the force of
r r normal reaction N.
have final momenta p¢A and p ¢B respectively. By the second f1
\ ml =
law of motion : N
Change in momentum of body A, 9. Vehicle stops on applying brakes, which is in accordance
r r r
pA¢ - pA = FAB Dt ...(i) with the law of conservation of momentum. When brakes
r are applied, opposition force acts on the vehicle. In pursuit,
where FAB is the force acting on A due to action of B for a car will be at rest. Loss of momentum of the vehicle is exactly
time Dt. equal to the impulse of the applied force.
r r 10. Let a be the common acceleration of the whole system.
Similarly, change in momentum of body B, pB¢ - pB =
r T3 T2 T1 F
FAB Dt ...(ii) m m m m
Here time Dt, the time for which two bodies A and B are in \ F = (m + m + m + m )a = 4ma, or, a = F/4m
contact and interact, is same for both the forces. Force equation, F – T1 = ma, T1 – T2 = ma, T2 – T3 = ma,
Moreover, from third law of motion T3 = ma, solving these equations, we get,
r r
FAB = – FBA 3 1 1
Hence, adding (i) and (ii), we obtain T1 = F, T2 = F and T3 = F
4 2 4
122 Physics
11. No. Net force on the body of mass M is (Mg – F) 16. Let m 1 an d m 2 be the masses
\ a = (Mg – F)/ M = (g –F/M), thus acceleration would connected by a massless string
be greater if the mass will be higher and it will reach the over a smooth frictionless pulley. If
earth earlier. ‘T’ be the tension in string and ‘a’
12. In closed glass cage, air inside is bound with the cage. be the ‘acceleration’ with which the a T a
masses move m
Therefore, (i) no change in weight of the cage, when bird
flies with constant velocity, (ii) cage will be heavier, when then m2
bird flies with upward acceleration , (iii) cage will be lighter, For, mass m1, m1g – T = m1a........(i)
when bird flies with downward acceleration. For, mass m2, T – m2g = m2a........(i)
From (1) and (2), m1g– m2g = (m1+ m2) a m1
13. While applying brakes, Let FB be the force required to stop m g
the truck in distance d. or, a = (m1 – m2)g/ (m1 + m2) 2

\ FB x d = ½ mv2 or, FB = mv2/ 2d Again from (1) and (2),


(m1g – T)/ (T– m2g) = m1a/ m2a = m1/ x2 m1g
For taking a turn of radius d, the force required, FT = mv2/d
= 2FB or, FB = ½ FT, which means it is better to apply brakes. or, m1m2g – Tm2 = m1T – m1m2g,
14. Acceleration of a body sliding down an inclined plane. As or, 2m1m2g = T (m1+ m2)
shown in Fig. consider a body of weight mg placed on an or, T = 2m1m2g/ (m1+ m2)
17. When bucket containing water is rotated in vertical circle
inclined plane. Suppose the angle of inclination q be greater
with velocity at the lowest point,
than the angle of repose. Let a be the acceleration with
vL = Ö5gr, water would not spill even at the heighest point,
which the body slides down the inclined plane.
when the bucket is upside down. If the bucket is whirled
R slowly, so that mg > mvH2/r, its weight provides necessary
centripetal force (mvH2/r) and rest of the water (mg – mvH2/r)
causes some water to accelerate downwards and spill.
a fk 18. Let the cyclist OC of mass ‘m’ bends at an angle q with the
vertical. In order to take a circular turn of radius ‘r’ with
q q speed ‘v’. The horizontal component R sin q of the normal
g sin mg cos q reaction ‘R’ provides the necessary centripetal force while
m
mg the vertical component R cos q balances the weight of the
q
cycle & cyclist.
R
The weight mg has two rectangular components : In equilibrium,
(i) mg cos q perpendicular to the inclined plane. It balances R cos q = mg and Rcosq
R sin q = mv2/r (centripetal force) Y C
the normal reaction R. Thus Rsin q
R sin q /R cos q = tan q = v2 /rg
R = mg cos q
(ii) mg sin q down the inclined plane. mg
If fk is the kinetic friction,t hen the net force acting q
down the plane is
F = mg sin q – fk O X
But fk = mk R = mk mg cos q
\ ma = mg sin q – mk mg cos q
Hence a = g (sin q – mk cos q).
15. If a- angle of repose,
R = mg sina Long Answer Questions
F = mg cosa 1. Newton’s second law of motion states that the rate of change
F/R = mgcosa/ mgsina = tana = m of momentum of a rigid body is directly proportional to the
By definition, coefficient of friction, applied force on it.
m = tan q The law implies that when a bigger force is applied on a
\ q = a, which means angle of repose is equal to the body of given mass, its linear momentum changes faster
angle of friction. and vice-versa. The momentum will change in the direction
of the applied force.
R
Let, m = mass of a body,
F B r
mgsina v = velocity of the body
a \ The linear momentum of the body
a mgcosa r r
A C p = mv ...(i)
r
mg Now, suppose F = external force applied on the body in
the direction of motion of the body.
Laws of Motion 123

r The most important consequence of the third law of motion


Dp = a small change in linear momentum of the body in a
is the law of conservation of linear momentum and its
small time Dt. application in collision problems.
r Dv
Dp Since F12 = – F21 and F = m
Rate of change of linear momentum of the body = Dt
Dt
According to Newton’s second law, Dv Dv
\ m1 1 = – m2 2
r
Dp r r Dpr Dt Dt
a F or F a Here Dt is the time for which the bodies come in contact
Dt Dt during impact. This is same for the two bodies of masses m1
r and m2 and having velocity Dv1 and Dv2 respectively.
r Dp
or F = k ...(ii) Therefore,
Dt m1 Dv1 = m2 Dv2
where k is a constant of proportionality. or m1 Dv1 + m2 Dv2 = 0
r Let u1, u2 and v1, v2 be the initial and final velocities of the
Dp
Taking the limit DT ® 0, the term becomes the derivative two masses before and after collision, then,
Dt m1 (v1 – u1) = – m2 (v2 – u2)
r
or differential coefficient of p w.r.t. time t. It is denoted by or m1u1 + m2u2 = m1v1 + m2v2
r Momentum before impact = Momentum after impact.
dp (This is known as the law of conservation of momentum).
. 3. Figure shows the various forces acting on the rod AB when
dt
r r it is leaning between the wall and the ground without
dp slipping.
\ F =k
dt Since the rod is in equilibrium, the net force as well as the
r net torque on it must each be zero. Considering the forces
r d r dv acting on the rod : we have
Using eqn (i) F = k (mv ) = km
dt dt R1 + (– F¢ ) = 0 or R1 = m2R2
r r and R2 + F + ( – W) = 0 or R2 + m1R1 = W
F = kma ...(iii)
r F = m 1R1
r dv
where a = represents acceleration of the body..
dt
A R1
The value of constant of proportionality k depends on the
units adopted for measuring the force.
Now, putting k = 1
r r
F = ma , This gives mean of measuring force. C
Units of Force : Force in SI units is measured in ‘Newton’ or Rod R2
r r Wall
N. From the relation F = ma , we can see that a newton
force is that force which produces 1 ms–2 acceleration in a
body of mass 1 kg. N q
1 newton = 1 kilogram × 1 metre/second2 O B Ground
Þ 1 N = 1kg × 1 ms–2 = 1 kg ms–2 F¢ = m 2R2
In CGS system, force is measured in ‘dyne’. W
1 dyne = 1 gram × 1 cm s–2 = 1g cm s–2 Considering next the moments of all forces about A, we
Since 1 N = 1kg ms–2 = 1000 g × 100 cm s –2 have
= 105 g cm s–2 = 105 dyne. R2 × O B = W × O N + m2 R2 × O A
Þ 1 N = 105 dyne. AB cos q
or 1 dyne = 10–5 N. or R2 × A B cos q = W × + m 2 R2 ´ AB sin q
2. Newton’s third law of motion states that for any action, 2
there is equal and opposite reaction. æ Wö
So, if a body applies a force F12 on body 2 (action), then or çè R2 - ÷ø cos q = (m2 R2) sin q
2
body 2 also applies a force F21 on body 1 but in opposite
direction, then æ Wö
çè R2 - ÷ø
F21 = – F12 or tan q = 2
In terms of magnitude m 2 R2
|F21| = |F12| Now, W = R2 + m1 R1 = R2 + m1 (m2 R2)
It is very important to note that F12 and F21 though are = R2 (1 + m1 m2)
equal in magnitude and opposite in direction yet act on
W R
different point or else no motion will be possible. \ R2 – = R2 – 2 (1 + m1 m2)
For example, hands pull up a chest expander (spring) and 2 2
spring in turn exerts force on the arms. A football pressed R2 R2 R
reacts on the foot with the same force and so on. = - m1m 2 = 2 (1 - m1m 2 )
2 2 2
124 Physics
R2 (1 - m1m 2 ) (1 - m1m 2 ) Therefore, for completing the loop,
\ tan q = = Tmin = (mvH2 /r – mg) ³ 0 i.e. mvH2 /r ³ mg
2 m 2 R2 2m 2
ì1 - m1m 2 ü or, vH ³ gr .........(i)
or q = tan -1 í ý which is the minimum velocity at the highest point.
î 2m 2 þ
4. The varying mass system is a rocket, which is moving up. By principle of conservation of energy,
Let, m0- initial mass of the rocket alongwith its fuel Mechanical energy at L = Mechanical energy at H
1 mv 2 = 1 mv 2 + mg (2r) ..........(ii)
v0- initial velocity of the rocket, m- mass of the rocket left 2 L 2 H
v- velocity acquired by the rocket From relations (i) and (ii),
As the exhaust gases are escaping, m < m0 and v > v0 1 mvL2 ³ 1 2 m(gr) + 2mgr Þ vL ³ 5gr
2
Now let, dm- small decrease in mass = mass of the exhaust
6. (i) Firction–It is an opposing force acting tangentially on
gases that escape
a body.
dv- small increase in velocity of the rocket
(ii) Plotting a graph between applied force and frictional
vg- velocity of the exhaust gas w.r.t. earth (opposite to the
force, we have
rocket motion, i.e. negative)
OL = static friction = F1
According to the law of conservation of momentum,
OP = limiting friction = F2
mv = (m – dm) (v + dv) + dm (– vg)
MN = kinetic friction = F3.
mv = mv + m(dv) – (dm) v – dmdv – (dm) vg
F1 goes on increasing with applied force at L the static
After simplification and neglecting smaller terms,
friction is maximum beyond L the frictional force
mdv = dm (v + vg)
decreases slightly. As the portion MN of the curve is
If, u – relative velocity of exhaust gases w.r.t. the rocket ,
– u = v + vg parallel to OX therefore, kinetic friction does not change
Now we have, mdv = – udm, or, dv = – u dm/m with applied force. Also kinetic friction is always slightly
less than limiting friction as once motion actually starts
Integrating with a limit,
irregularities of one surface are not able to get locked
m = m0 and v = v0 to m = m and v = v, we have, into irregularities of another surface.
v m (iii) Magnitude and direction of the static friction force
ò dv = ò udm / m or, v = v0 + u loge(mo/m) Þ v = u loge(mo/m) adjust themselves according to applied force. When
v0 m0 we change the applied force, force of static friction
5. Let a body of mass ‘m’ is looping a verticle circle of radius changes accordingly in the direction opposite to the
‘r’. applied force.
vH H
P L
M N
Frictional force

Q
O
r q T F1 F2 F1
P
q
L
mg sin q mg mg cosq X
O Applied force
At only instant
(iv) Laws of limiting friction :
let, q be the angular position of the body rotating in the (1) Magnitude of force of limiting friction (F) between
vertical circle any two bodies in contact is directly proportional
mg – weight of the body acting vertically downwards to normal reaction (R), i.e, F µ R.
T – tension in the string, with which the body is tied, acting (2) Direction of force of limiting friction is always
along PO opposite to the one in which the body starts
mg sin q– component of the weight, acting opposite to T moving over another.
Force equation for the body at P, (3) As long as normal reaction between two bodies
T – mgcosq = necessary centripetal force = mv2 /r in contact remains same the force of limiting
or, T = mv2 /r + mg cos q friction is independent of area of contact.
When cos q = –1 (minimum), i.e. q =180°, (4) Force of limiting friction only depends upon the
T = minimum, i.e. material and nature of the surfaces in contact.
Tmin = TH = mv2 /r – mg 7. Consider the mass m attached to a string of length l. Let the
When Tmin ³ 0, the body will move along the vertical lower most and top most point be marked A and B
respectively. Consider any point P where the length l has
circle however when Tmin £ 0, the string will be slacked and turned by q from the vertical line through A. Centripetal
the body will fall down from vertical circle and from highest force is provided by the tension and mg cos q acting in
point. opposite directions.
Laws of Motion 125

B 4. Horizontal component of force = F sin q


F cos q
50 N

O
q = 30°
q TP
T mP
F sin q
mg cos q
mg
A
From the diagram, OA = OP = l, OT = l cos q, AT = l (1 – cos q) F sin q 50sin 30°
(i) If vA and vP are velocity at A and P respectively, using Þ a= = = 12.5m / s 2
m 2
u2= u2 + 2as we have,
30
vP2 = vA2 – 2g (AT) 5. Given : M = 10 kg, m = kg., V = ?, v = 330 m/s
= vA2 – 2gl (1 – cos q) 1000
From conservation of momentum, MV + mv = 0
Velocity at any point ®
30
Þ 10 × V = - ´ 330 Þ V = – 0.99 m/s
P = vP = v 2A - 2 gl (1 - cos q). 1000
Resultant momentum before and after firing is zero as no
mvP2 external force is acting on it.
(ii) At P TP – mg cos q = , 6. Here, m = 4000 kg, T = 48000 N, a = ?
l
T 48000
T = m (g + a) Þ g + a = = = 12
mvP2 m 4000
TP = mg cos q + 2
a = 12 – g = 12 – 9.8 = 2.2 m/s ; u = 0, s = ?, t = 3s
l
m 1 2 1 2
\ TP = mg cos q + [vA2 – 2gl (1 – cos q)] Þ s = ut + at = 0 + ´ 2.2 ´ 3 = 9.9 m
l 2 2
By knowing vA and q one can find velocity and tension 7. (i) Recoil velocity of the gun,
at any point. - mv - 0.025 ´ 600
V= = = 0.75 m/s
Numerical Questions M 20
(ii) Force required to keep the gun in position
1. Here, a1 = 16 m/s2, m1 = 0.5 kg,
F = m1a1 = 16 × 0.5 kg = 8 N æ ö
ç v-0 ÷
F 8 F = Ma = m ç ÷ = 15 × 600 = 9000N
m2 = = = 2 kg ; m1 + m2 = 0.5 + 2 = 2.5 kg, F = 8 N;
4 çç 1 ÷÷
a2 è 600 ø
F 8 8. For u = 0, v2 – u2 = 2gh thus
a= = = 3.2 m/s2 Velocity on striking the ground
m1 + m2 2.5
= v = 2gh = 2 ´10 ´ 5 = 10 m/s
20
2. Given : m = kg, u = 300 m/s, v = 0, s = 3 cm = 3 × 10–2 m Velocity on rebounding = v ' = 2gh '
1000
v2 - u2 0 - (300)2 -300 ´ 300
= 2 ´ 10 ´ 1.25 = 5 m/s
\a= = = = – 1.5 × 106 m/s2 I = change in momentum = mv – (–mv’)
2s 2 ´ 3 ´ 10-2 2 ´ 3 ´ 10-2
100
20 = m (v + v¢ ) = (10 + 5) = 1.5 N-s
\ F = ma = ´ (-1.5 ´ 106 ) = – 3 × 104 N 1000
1000
1.5
I=F×t Þ F= = 15N
3. Here, u = 2 gh = 2 ´ 9.8 ´ 50 = 980 0.1
9. mu mu sin 45°
v2 - u 2
v = 0, s = 1 m, a = ?, a = = – 490 m/s2;
2s 45°
– mu cos 45° 45°
F = ma = 5(– 490) = – 2450 N
N O
v = u + at Þ 0 = 980 - 490t Þ t = 0.064 s mu cos 45° 45°

mu mu sin45°
126 Physics

20 17. As we know, VL = 5 gr
Given : m = kg = 0.02 kg;
100
Þ 50 = 5 ´ 10 ´ r Þ r = 50 m
Change in momentum along horizontal direction
= – mu cos 45° – (mu cos 45°) 18. Given : v = 2t = 2 × 3 = 6 m/s, r = 20 cm = 0.2 m;
2
Radial acceleration = ar = v = 6 ´ 6 = 180 m/s 2
1
= – 2 mu cos 45 ° = – 2 × 0.02 × 15 × = 4.8 kg m/s
2 r 0.20
Impulse received by the ball = 4.8 kg m/s dv d
Tangential acceleration = aT = = (2t ) = 2 m/s 2
dm dt dt
10. Here, m0 = 6000 kg, = 16 kg/s , u = 11 km/s
dt
= 11000 m/s, t = 1 min = 60 s, a = ? HOTS/EXEMPLAR QUESTIONS

Mass left after 1 minute = m0 – æç


dm ö Very Short Answer Questions
t
è dt ÷ø
= 6000 – 16 × 60 = 5040 kg. 1. Motion requires no force. Newton’s first law says an object
in motion continues to move by itself in the absence of
du dm external forces. It is possible for forces to act on an abject
F= m =u
dt dt with no resulting motion if the forces are balanced.
2. While in the tube the pellet is forced to curve, but when it
du u (dm / dt )
Þ a= = gets outside, no force is exerted on the pellet and due to
dt m law of inertia it follows a straight-line path. Hence ‘B’ path
1100 ´ 16 will follow.
Þ a= = 34.92 m/s2
5040 3. When an object reaches at the highest point of its path,
11. Given : F = 1.2 kgwt = 1.2 × 9.8 N, m = 1.5 kg, when it is tossed upwards in air, its velocity is zero. The
m = 0.3, a = ?, f = m R = m mg = 0.3 × 1.5 × 9.8 force due to gravity acts in the downward direction
which balances upward force with which it is thrown. So
Net force = F – f = 1.2 × 9.8 – 0.3 × 1.5 × 9.8 = 9.8 × 0.75; it is in equilibrium.
4. The only retarding force that acts on him, if he is not using
net force 9.8 ´ 0.75 a seat belt comes from the friction exerted by the seat.
a= = = 4.9 m/s2
mass 1.5 This is not enough to prevent him from moving forward
5 when the vehicle is brought to a sudden halt.
12. Here sin q = = 0.4167
12 5. f = m R = m mg cos q is the force of friction, if q is angle
Þ q = 24.626°; m = tan q = tan 24.626° = 0.458 made by the slope. If q is small, force of friction is high and
there is less chance of skidding. The road straight up would
13. Max. static frictional force = f s = m s R = m s mg cos q
have a larger slope.
fs = 0.7 × 2 × 9.8 cos 30° = 1.4 × 9.8 × 0.866 = 11.9 N
But here, static frictional force = mg sin q Short Answer Questions
= 2 × 9.8 × sin 30° = 9.8 N 1. It is indeed true that the heavier object is pulled with a
larger force. The strength of the force is determined by the
v2 36 ´1000
m/s gravitational mass of the object. The resistance to the
14. As we know, tan q = , v = 36 km/h =
rg 3600 force and, therefore, to the change in motion of the object,
= 10 m/s; is represented by the inertial mass. Thus, if an object has
twice as much mass as another, it is pulled to the Earth
v2 10 ´ 10 with twice the force, but it also exhibits twice the resistance
r= = = 10 m
g tan q tan 45° ´ 10 to having its motion changed. These factors cancel, so
20 that the change in motion, the acceleration, is the same for
15. Here, r = 70 cm, T = = 2s, all objects, regardless of mass.
10
2
2.
2 æ 2p ö 4p2r 22 22 70 700 N 500 N
a = w r = ç ÷ r = 2 = 4 ´ ´ ´ = 691.43 cm/s2
èT ø T 7 7 22
1
16. Here, m = 5000 quintals = 5 × 105 kg, sin q = , a = 2 m/s2,
50
F = 0.2 × 5000 = 103 N
Total force = mg sin q + F + ma 100 N
1
= 5 × 105 × 10 × + 103 + 5 × 105 × 2
50 600 N ?
= 1.101 × 106 N
Laws of Motion 127

The scaffold is in equilibrium, so we see that in accord


ΡFy 5.2 N
with SF = 0 , the net force on the scaffold is zero. That ay < < < 17 m / s 2
means that the sum of the upward forces must be equal in m 0.30kg
magnitude to the sum of the downward forces. Note the The acceleration has a magnitude of
scale readings add to 1200 N, the total force upward. The
total force downward must then also be 1200 N. We see a < (29) 2 ∗ (17) 2 m / s 2 < 34 m / s 2
that Burl and his bucket of point weight 700 N. So the
weight of the scaffold is 1200 N - 700 N = 500 N. and its direction is

2V 2 ´ 20 40 10 æ 17 m / s 2 ÷ö
ay ç ÷÷ < 30°
3. t= = = = = 3.33 s π < tan,1 < tan ,1 çç
g + a 10 + 2 12 3 ax çè 29 m / s 2 ÷÷ø
4. If F is the force of the finger on the book, F = N, the normal
reaction of the wall on the book. The minimum upward relative to the positive x-axis.
frictional force needed to ensure that the book does not 3. (a) F - (500 × 10) = (500 × 15) or F = 12.5 × 103 N, where F is
fall is Mg. The frictional force = mN. Thus, minimum value the upward reaction of the floor and is equal to the force
downwards on the floor, by Newton’s 3rd law of motion.
Mg (b) R - (2500 × 10) = (2500 × 15) or R = 6.25 × 104 N, action
of F = of the air on the system, upwards. The action of the rotor
m
on the surrounding air is 6.25 × 104 N downwards.
Long Answer Questions (c) Force on the helicopter due to the air = 6.25 × 10 4 N
upwards.
1. F 1 2
4. For (a) v z = gH v z = 2gH
2
A

T T
Speed at ground = vs2 + v z2 = vs2 + 2gH
q
B C é1 2 ù
For (b) also ê mvs + mgH ú is the total energy of the ball
(–x, 0) O (x, 0) ë2 û
when it hits the ground.
From figure F = 2 T cos q or T = F/(2 cos q) So the speed would be the same for both (a) and (b).
The force responsible for motion of masses on X-axis is T
sin q CHAPTER TEST
F 1. Difference in tension between highest and lowest point =
\ m a = T sin q = ´ sin q TL – TH = 6 mg = 6 × weight of the stone.
2 cos q
2. Yes, if the forces are balanced, then the body will be in
F F OB F x equilibrium.
= tan q = ´ = ´
2 2 OA 2 (a - x 2 )
2 3. An athlete runs a certain distance before taking a long jump
as he gathers inertia of motion which helps him to take
longer jump. Also inertia at rest is over come by running.
F x
So, a = ´
2m 50
(a - x 2 )
2
4. Given : m = kg , u = 500 m/s, v = 0, t = 0.01 s
1000
2. The puck is modeled as a particle under a net force. We Impulse = Change in momentum
first find the components of the net force. The component
= m(v – u) = 0.05 (0 – 500) = – 25 Ns
of the net force in the x direction is
5. Here, u = 5.6 m/s, v = 0, s = 8 m
ΡFx < F1x ∗ F2 x < F1 cos 20° ∗ F2 cos 60° g = 9.8 m/s2
< (5.0 N )(0.940) ∗ (8.0 N )(0.500) < 8.7 N v2 – u2 = 2as
02 – (5.6)2 = 2 × a × 8
The component of the net force in the y direction is
5.6 ´ 5.6
ΡFy < F1y ∗ F2 y < ,F1 cos 20° ∗ F2 sin 60° Þ a=- = – 1.96 m/s2
2´8
< ,(5.0 N )(0.342) ∗ (8.0 N )(0.866) < 5.2 N m= =
a 1.96
= 0.2
g 9.8
Now we use Newton’s second law in component form to
find the x and y component of acceleration 6. Principle of conservation of momentum : Refer to theory.
Explanation : Recoiling of the gun
ΡFx 8.7 N Formula MV + mv = 0
ax < < < 29 m / s 2
m 0.30kg
128 Physics
7. When a sudden jerk is given to D, the upper portion of the 10. Let the mass of the object be m.
system is not able to share the force in short time and the Case (a). The force acting on it when it is moving down. As
block tends to remain at rest (inertia of rest). So the chord D the object slides down with a constant velocity, the net
breaks. force acting on the object is zero.
When the chord D is pulled steadly, the force gets sufficient
F
time to reach the chord C which ultimately breaks. R
8. Weight (W1) of the cart is balanced by the reaction force (R1)
due to ground on the cart. Weight (W2) of the horse is
balanced by the reaction force (R2) and the component R
sin q of the reaction force R. q
g sin q
m
mg cos q
R1 R2
R q
R sin q
T T H q
C Rcos q mg
\ F = mg sin q and R = mg cos q
F q W2
f But R = mk R
F = mk mg cos q
W1 where mk is coefficient of kinetic friction.
F is the force with which the horse pushes the ground at an \ mk mg cos q = mg sin q
angle q . or mk = tan q ...(i)
Case (b). When the body is projected upwards with an initial
\ The ground also gives an equal and opposite reaction
velocity u, it experiences a downward acceleration given by
force (R). The component R cos q in the forward direction
should be more than the frictional force (f ) for the system to R
move.
9. Let there be two objects masses m1 and m2 (m1 < m2) tied at
the two ends of a light inextensible string passing over a
friction less pully.
qF mg cos q
g sin
m mg
q
m 2 > m1

mg sin q + m k mg cos q
a
T a= = g sin q + mk g cos q
T m
a = g (sin q + mk cos q)
From (i), mk = tan q.
m1
\ a = g (sin q + tan q × cos q) = g (2 sin q)
m2 or a = 2g sin q ...(ii)
m1g Suppose y is the distance up the plane which the object
travels before it stops.
m2g
\ 02 – u2 = 2 × (– 2g sin q) × y

For m1, the force equation is T – m1g = m1a ..... (1) u2


For m2, the force equation is, m2g – T = m2a ..... (2) or y=
4 g sin q
Adding equations (1) and (2) and simplifying we get,
Once the object comes to rest, the frictional force to be
æ m - m2 ö overcome is that of static friction. The object will therefore,
Acceleration, a = ç 1 g
è m + m ÷ø
1 2
slide down the plane if its inclination is tan–1 (ms), where ms
is the coefficient of static friction.
substituting the value of ‘a’ in equation (1) we get,
As ms > mk and the plane has an inclination of tan–1 mk, the
2m1m2 g object will stay at rest, and will not slide down the plane
Tension, T = again.
m1 + m2
¿¿¿
6 Work, Energy and
C ha p t e r Power

THE SCALAR PRODUCT


r r r r
The scalar product or dot product of any two vectors A and B is defined as A. B = AB cos q
r r r r
where q is the angle between A and B . The dot product of A and B is a scalar quantity.
Properties of scalar product :
r r r r
(i) The dot product follows commutative law i.e. A . B = B. A
r r r r r r r
(ii) It obeys distributive law i.e. A .( B + C ) = A . B + A.C
r r B
(iii) Also A. A = A2 = Ax2 + Ay2 + Az2
q
r r r r
(iv) A .B = 0 if A and B are perpendicular. A
For unit vectors iˆ, ˆj, kˆ
iˆ.iˆ = ˆj. ˆj = kˆ.kˆ = 1; iˆ. ˆj = ˆj.kˆ = kˆ.iˆ = 0

WORK
Work done by a force on a body is defined as the product of force and the displacement of the body in the direction of force.
uur uur
Work done (W) by a constant force ( F ) producing a displacement ( S ) is F
uur uur
W = F . S = F S cos q q
uur uur
where q is the angle between F and S .
S
The S.I. unit of work is joule and its C.G.S unit is erg. 1 joule = 107 erg.
It is a scalar quantity.
There are three types of work done:
(i) Positive work: When force and displacement are in same direction, work done by the force is positive. i.e. W is positive
p
when 0 < q <
2
(ii) Negative work: When force and displacement are in opposite direction, work done by the force is negative. i.e. W is negative
p
when <q<p
2
(iii) Zero work done: When force and displacement are perpendicular to each other, work done by the force is zero. i.e.
p
W = 0 when q =
2
x2
Work done by a variable force W = ò F ( x) dx, where x1 and x2 are initial and final positions.
x1

For a variable force W = ò F .ds Þ Area under the force displacement curve gives the work done.
1 2
Work done by a stretched spring W = ò kx dx = 2
kx where k is spring constant.
130 Physics
Conservative and Non-conservative Force
A force is said to be conservative if work done doesn’t depend on the path followed but depends only on initial and final positions
of the object. Ex. Gravitational force.
A force is said to be non-conservative if work done depends on the length of the path but not path followed. Ex. Frictional force.
ENERGY
Energy of a body is the capacity of doing work. Energy can exist in various forms such as mechanical energy (energy due to
motion or position), chemical energy (energy due to chemical bonding of atoms), heat energy (energy due to random motion of
its molecules), nuclear energy (energy due to binding of nucleus) and electrical energy (energy due to flow of charge).
The unit of energy is same as that of work. In S.I. system joule and erg in C.G.S. system.
Kinetic Energy
It is the energy possessed by the body due to its motion.
1 2
K.E. = mv where v is the velocity of the body
2
r r
Consider a body of mass m which initially is at rest. A constant force F is applied on the body which produces a displacement d s in
its direction. The work done by the force on the body is

W= ò Fds = ò (ma)ds
æ dv ö æ ds ö
= ò m çè dt ÷ø ds = ò m çè dt ÷ø dv
v 1 æ ds ö
ò0 mvdv = 2 mv çèQ
2 = v÷
= ø
dt
This work done appears as the kinetic energy (K) of the body. Thus kinetic energy
1 2
K = 2 mv .
Kinetic energy can also be obtained by simple use of equation of motion as (provided force applied is constant).
v2 = u2 + 2as = 0 + 2as
F æ Fö
or v2 = 2 s çèQ a = ÷ø
m m
1
or Fs = mv 2
2
1 2
\ W=K= mv .
2
Relation between Kinetic energy and momentum (P) K.E. = P2 2m
Potential Energy
Potential Energy (P.E.) is the energy possessed by the body due to its position or shape.
Gravitational P.E. = mgh (due to change in position)
where h is the height through which the body is raised.
1 2
or P.E. = kx (due to change in shape) where, k is spring constant.
2
The Conservation of Mechanical Energy
The total mechanical energy of a system remains conserved when conservative forces work on the system.
i.e., Total mechanical energy (E) = kinetic energy (K) + potential energy (U).
Mass-energy equivalence: According to this theorem mass and energy are inter-convertible.
E = mc2, where c = 3 × 108 ms–1 is velocity of light in vacuum or air.
When a body moves with a velocity v camparable to the velocity of light its mass is given by
m0
m= where m0 is the rest mass.
v2
l- 2
c
Work, Energy and Power 131

VARIOUS FORMS OF ENERGY : THE LAW OF CONSERVATION OF ENERGY


Energy is of many types – mechanical energy, sound energy, heat energy, light energy, chemical energy, atomic energy, nuclear energy
etc.
In many processes that occur in nature energy may be transformed from one form to other. Mass can also be transformed into energy
and vice-versa. This is according to Einstein’s mass-energy equivalence relation, E = mc2.
In dynamics, we are mainly concerned with purely mechanical energy.
Law of Conservation of Energy :
The study of the various forms of energy and of transformation of one kind of energy into another has led to the statement of a very
important principle, known as the law of conservation of energy.
"Energy cannot be created or destroyed, it may only be transformed from one form into another. As such the total amount of energy
never changes".
NOTIONS OF WORK AND KINETIC ENERGY: THE WORK-ENERGY THEOREM
It states that the work done by the net force acting on a body is equal to the change produced in the kinetic energy of the
body i.e., Kf – Ki= W
Work-Energy Theorem for a Constant Force
Suppose a constant force F acting on a body of mass m produces acceleration a in it. After covering distance s, suppose the
velocity of the body changes from u to v.
We use the equation of motion, v2 – u2 = 2as
1 1 2 1 2
Muiltiplying both sides by m, we get mv - mu = mas
2 2 2
By Newton’s second law, ma = F, the applied force.
1 2 1 2
\ mv - mu = Fs = W
2 2
i.e., Kf – Ki= W
Change in K.E. of the body = Work done on the body by the net force.
This, proves the Work – Energy theorem for a constant force.
Work-Energy Theorem for a Variable Force
r uur
Suppose a variable force F acts on a body of mass m and produces displacement ds in its own direction (q = 0°).
The small work done is
r uur
dW = F .ds = Fds cos 0° = Fds
dv
According to Newton’s second law of motion, F = ma = m
dt
dv é ds ù
\ dW = m
dt
.ds = mv dv êëQ dt = v úû
If the applied force increases the velocity from u to v, then the total work done on the body will be
v v v
é v2 ù 1 1
W = ò dW = ò mv dv = m ò v dv = m ê ú = mv 2 - mu 2
u u
2
ëê ûúu 2 2
or W = Kf – Ki = Change in K.E. of the body
This proves the Work-Energy theorem for a variable force.
POWER
In many cases it is useful to know not just the total amount of work being done, but how fast the work is done.
We define power as the rate at which work is being done.
Work done Total change in kinetic energy
Average power, Pav = =
Time taken to do work Total time in change
132 Physics
DW W2 - W1
If DW is the amount of work done in the time interval Dt then, Instantaneous power, P = =
Dt t 2 - t1
When work is measured in joule and t is in second, the unit for power is the joule per second, which is called watt. For
motors and engines, power is usually measured in horsepower (hp)
The definition of power is applicable to all types of work like mechanical, electrical, thermal etc.
dW
Instantaneous power, P =
dt
rr d r r r dxr r r
W = F.x \ P= (F.x) = F. = F.v
dt dt
where v is the instantaneous velocity of the particle and dot product is used as only that component of force will contribute to power
which is acting in the direction of instantaneous velocity.
® ®
If velocity vector makes an angle q with the force vector, then P = F . v = Fv cos q

dK
P= (time rate of change of kinetic energy)
dt
Power is a scalar quantity.
The SI unit of power is J/s or watt
1W = 1 J/s and 1Horse power [1HP] = 746 W, 1calorie = 4.2J and one kilo watt hour,. 1 kW h = 3.6 × 106 J
Illustration 1 :
Calculate work done to move a body of mass 10 kg along an inclined plane (q = 30°) with constant velocity through a distance
of 10 m.
Sol. Here the motion is not accelerated, the resultant force parallel to the plane must be zero. N F
So, F – Mg sin 30° = 0
or F = Mg sin 30° ; d = 10 m 10
m

W = F d cos q 0
0
= (Mg sin 30°) d cos 0° (Q q = 0° so cos 0° = 1) si n3 Mgcos30
0

Mg
1 30
0

= 10 × 10 × × 10 × 1 = 500 J
2
Mg
Illustration 2 :
Calculate work done from the graph.
A
Sol. Work done = area of triangle (OAB) – area of rectangle (BCDE) 10 –

1 5–
W= × 20 × 10 – 10 × 5 = 50 J
2
F orce F

B E
0 | X
10 20 30

–5 –
C D
Displacement
–10 –
Illustration 3 :
Determine the average force necessary to stop a bullet of mass 20 g and speed 250 ms–1 as it penetrates a wooden block to a
distance of 12 cm.
Sol. If F newton be the retarding force, then the work done by force is given by
W = F × S = F × 0.12 joule
1 20
Loss of kinetic energy = ´ × 250 × 250 joule = 625 joule
2 1000
(This kinetic energy is consumed in stopping the bullet and is converted into heat energy)
625
Applying work-energy theorem, F × 0.12 = 625 or F = N = 5.2 ´ 103 N
0.12
It is interesting to note that the retarding force is nearly 30,000 times the weight of the bullet.
Work, Energy and Power 133

Illustration 4 :
A cyclist climbing up an inclined road maintains a constant speed of 18 km/h. The total mass of the cycle plus the cyclist
is 120 kg. If the angle of inclination is 15°, what is the minimum power expended by the cyclist?
Sol. Given : m = 120 kg, v = 18 km/h = 5m/s R
Force opposite to the direction of motion of the cyclist = mgsinq = 120 × 9.8 × sin 15°

F
= 120 × 9.8 × 0.2588 = 304.35N 15º
Work F .ds v q
\ Power expended by the cyclist Q Power = = = F .u sin 15º 120 g
Time dt g
m 15º mg
= 304.35 × 5 = 1521.75 W
COLLISIONS
When exchange of momentum takes place between two physical bodies only due to their mutual interaction force, it is defined as
collision between two bodies.
Momentum can be transferred from one body to another by any mutual interaction force (only internal forces), does not matters
whether there is physical contact or not.
The case of collision in which physical contact takes place is known as "Impact".
A process in which there is a change in velocity, momentum, mechanical energy and direction of motion due to mutual interaction is
called collision.

u1 u2 v1 v2

m1 m2 m1 m2 m1 m2

1. For collision, it is not necessary for particles to come in contact. For example, an alpha particle coming towards a nucleus and
going away in some direction.
2. Before collision particle come closer and after collision they recede or stick together.
3. In all collisions, linear momentum is conserved.
r r r r
i.e., m1u1 + m 2 u 2 = m1v1 + m 2 v2
Newton's Law of Collision
According to Newton's experimental law of impact, the ratio of relative speed of separation after collision and the relative speed of
approach before collision is a constant. This constant is called coefficient of restitution or coefficient of resilience for a given pair of
materials. It is denoted by e.
Relative velocity of separation after collision µ relative velocity of approach before collision
r r r r
v 2 - v1 µ u1 - u 2
r r r r
v 2 - v1 = e (u1 - u 2 )
r r
v -v (velocity of separation after collision)
Coefficient of restitution, e = r2 r1 =
u1 - u 2 (velocity of approach before collision )

The value of ‘e’ lies between 0 and 1. It is zero when the materials stick together and approaches unity for highly elastic materials.
PERFECTLY ELASTIC COLLISION (IDEALLY ELASTIC)
Collision between two bodies is said to be elastic if both the bodies come to their original shape and size after the collision, i.e., no
fraction of mechanical energy remains stored as deformation potential energy in the bodies. Thus in addition to the linear momentum,
kinetic energy also remains conserved before and after collision.
The total kinetic energy of the bodies involved before collision is equal to the total kinetic energy after collision.
In this case :
1. Total kinetic energy is conserved.
2. Linear momentum is conserved.
3. Total mechanical energy is conserved.
4. It takes place under influence of conservative forces.
r r r r
5. e = 1 so, v 2 - v1 = u1 - u 2
134 Physics
Examples of perfectly elastic collision :
Collisions between atomic, nuclear and fundamental particles only may be truly elastic.
Collisions between gross bodies, for example, ivory or glass balls are approximately elastic.
Perfectly Elastic or Head on Collision (Collision in One Dimension)
Collision of two elastic bodies moving in a straight line, known as head-on collision.
Before and after collision particles move along a straight line.
u1 u2 v1 v2
m1 m2 m1 m2

Before collision After collision


Let two particles of masses m1 and m2 are moving along a straight line with constant velocities u1 and u2 respectively.
No external force or forces acting on them so they collide due to difference of their velocities only.
Let after collision their velocities are v1 and v2 respectively.
According to law of conservation of momentum
m1v1 + m2v2 = m1u1 + m2u2 .........(1)
or m1(v1 – u1) = –m2(v2 – u2) ........(1a)
According to law of conservation of kinetic energy
1 1 1 1
m1v12 + m 2 v 22 = m1u12 + m 2 u 22 ........(2)
2 2 2 2
or m1(v12 – u12) = –m2(v22 – u22) . .......(2a)
n n
Dividing eq . (2a) by eq . (1a)
v1 + u1 = v2 + u2
or, v1 – v2 = – (u1 – u2) ..........(3)
|Relative velocity of separation after collision| = |Relative velocity of approach before collision|
From eqn. (3) v2 = u1 – u2 + v1
Putting this value of v2 in eqn. (1), we get

æ m - m2 ö æ m2 ö
v1 = ç 1 ÷ u1 + 2 ç u2
è m1 + m 2 ÷ø
+ ........(4)
è 1
m m 2ø

æ 2m1 ö æ m - m1 ö
v2 = ç ÷ u1 + ç 2 u2
è m1 + m 2 ÷ø
........(5)
è m1 + m2 ø

Special Cases
Case-1 : If , m1 = m2 when both the colliding bodies are of the same mass.
From eqns. (4) and (5) we get, v1 = u2 and v2 = u1.
Thus, if masses of two bodies are equal, velocities after collision are interchanged. If the second particle is at rest, after collision
first comes to rest and second moves with the velocity of the first.
If target body m2 is at rest i.e. u2 = 0
then v1 = 0 and v2 = u1
So, maximum transfer of energy and momentum takes place.
There is hundred per cent transfer of energy.
When a body suffers an elastic collision with another body of the same mass at rest, it is stopped dead whereas the second body
starts moving with the same velocity as that of the first.
Case-2 : If , m1 >> m2, and m1 is moving with velocity u1 and m2 is at rest, u2 = 0
m1 m1
m2 m2
u1 u2 = 0 u1 2u1

Before collision After collision


Work, Energy and Power 135

Neglecting m2 in comparison with m1, m1 + m2 » m1 and m1 – m2 » m1.


m1
From eqn. (4) v1 » m u1 or v1 = u1
1
So, the velocity of the massive body after collision is nearly equal to its velocity before collision. In other words, massive body
loses only a very small fraction of its kinetic energy.
2m1
From eqn. (5), v2 » u1 or v2 = 2u1
m1
So, the velocity of the light body after collision is nearly double the velocity of the massive body before collision.
When a massive body suffers an elastic collision with a stationary light body, there is practically no change in the velocity of the
massive body but the light body acquires a velocity which is nearly double the initial velocity of massive body.
Case-3 : If, m2 >> m1, and m1 is moving with velocity u1 and m2 is at rest, u2 = 0 m2
Neglecting m1 in comparision with m2, m1 + m2 » m2 and m1 – m2 » – m2.
Massive Massive
body body
- m2 m1 v1 = –u1
From eqn. (4) v1 » u1 or v1 = – u1 u1
m2 v2 = 0
2m1 2m1 Light body
From eqn. (5) v2 » u1 = u1
m1 + m 2 m2
So, the velocity of the massive body after collision is very small. After collision
Before collision
2m1
or v2 » 0 (Q m » 0 )
2
So, the velocity of the light body after collision is nearly equal (in magnitude) and opposite to its velocity before collision. In
other words, the light body transfers only a small fraction of its kinetic energy to the massive body.
When a light body suffers an elastic collision with a stationary massive body, the velocity of the light body is reversed and the
massive body remains practically at rest.
Ex. A rubber ball thrown against a wall.
Kinetic energy transferred from projectile to target
u1 u2=0 v1 v2
m1 m2 m1 m1

From laws of conservation of energy,


1 1 1
m1u12 + 0 = m1v12 + m 2 v 22
2 2 2
1 2 1 2
Change in KE of mass m1 is m1u1 - m1v1
2 2
2 2
Change in energy DK æv ö æ m - m2 ö
= = 1- ç 1 ÷ = 1- ç 1
Initial KE Ki è u1 ø è m1 + m 2 ÷ø
4m1m 2 4m1m 2
= =
2
(m1 + m 2 ) (m1 - m 2 ) 2 + 4m1m 2
æ DK ö
For maximum transfer of KE çè K ÷ø = 1
i max
This is possible when m1 – m2 = 0 i.e. m1 = m2
Transfer of KE is maximum when two particles are of equal masses.
ELASTIC COLLISION IN TWO DIMENSIONS (OBLIQUE COLLISION)
If the colliding bodies do not move along the same straight line path, then the
collision is said to be an oblique collision.
Particles move before and after collision in a plane.
Conserving momentum along
X-axis, m1u1 cosa1+ m2u2 cosa2 = m1v1 cosb1 + m2v2 cosb2 ........(1)
Y-axis, –m1u1 sina1 + m2u2 sina2 = m1v1 sinb1 – m2v2 sinb2 ........(2)
136 Physics
y
For elastic collision
m1 v1
1 1 1 1 m1
m1u12 + m 2 u 22 = m1v12 + m 2 v 22 ........(3) u1
2 2 2 2
With the help of these three eqns. we can determine unknown quantity.
x
Special Cases
(i) Glancing collision. For each collisions, u2
m2
u1 = v1 and v2 = 0
m2 v2
1 2
K.E. of the target particle = m2 v2 = 0
2
Hence in a glancing collision, the incident particle does not lose any kinetic energy and is scattered almost undeflected.
(ii) Head-on collision. In such a collision, the target particle moves in the direction of the incident particle.
(iii) Elastic collision of two identical particles. As the two particles are identical, so m1 = m2 = m (say). By conservation of K.E. for
elastic collision,
1 1 1
mu12 = mv12 + mv22 or u12 = v12 + v22
2 2 2
By conservation of linear momentum,
r r r r r r
mu1 = mv1 + mv2 or u1 = v1 + v2
r r r r r r r r r r r r r r
\ u1 × u1 = ( v1 + v2 ) × ( v1 + v2 ) = v1 × v1 + v1 × v2 + v2 × v1 + v2 × v2
r r
or u12 = v12 + v22 + 2v1 × v2
r r éëQ v12 + v22 = u12 ùû
or u12 = u12 + 2v1 × v2
r r
or v1 × v2 = 0.
r r
This shows that the angle between v1 and v2 is 90°. Hence two identical particles move at right angles to each other after
elastic collision in two dimensions.

PERFECTLY INELASTIC COLLISION


It is also known as absolutely non-elastic collision.
An inelastic impact means an encounter between two bodies whereby these two bodies stick together after collision.
In this case :
1. Maximum loss in kinetic energy takes place.
2. Linear momentum is conserved.
r r
3. e = 0; v 2 = v1 i.e. particle stick after collision.
Examples of perfectly inelastic collision :
(i) Collision between a bullet and its target when the bullet remains embedded in the target.
(ii) Collision between clay spheres.
(iii) Collision between oppositely charged ions resulting in the formation of a molecule.
(iv) Capture of an electron by a positive ion.
(v) Jumping of a man on to a moving trolley, etc.
The term completely inelastic collision does not mean that all the initial kinetic energy is lost. The kinetic energy loss in a completely
inelastic collision is the maximum possible consistent with momentum conservation. An indeterminate amount of kinetic energy may
be lost to heat, sound or other forms of energy.
Perfectly Inelastic Collision (Particles Stick After Collision)
Collision is said to be perfectly inelastic if both the particles stick together after collision and move with same velocity, say V.
V
m1 m2
u1 u2
m1 + m2
Before collision After collision
Let two bodies of masses m1 and m2 are moving with velocities u1 and u2 before collision.
Total momentum before collision = m1 u1 + m2 u2.
After collision the two bodies stick together and have a common mass (m1 + m2).
Total momentum after collision = (m1 + m2) . V
Work, Energy and Power 137

Applying law of conservation of momentum,


m1 u1 + m2 u2 = (m1 + m2) . V
m u + m2 u 2
or V= 1 1
m1 + m 2
(a) If the motion of the bodies colliding head on, is along a straight line, then after impact the bodies will follow the direction of the
body having the originally larger momentum.
(b) If the momentum of the bodies are equal in magnitude,
i.e., m1u1 = – m2u2.
V = 0, i.e., the colliding bodies come to a standstill.
(c) An inelastic collision is accompanied by a transformation of energy.
Kinetic energy of the system before collision is
1 1
Ki = m1u12 + m 2 u 22
2 2
Kinetic energy of the system after collision is
1
K f = (m1 + m2 ) V 2
2
1é 2 2 (m u + m 2 u 2 )

Loss of kinetic energy during collision is DK = Ki – Kf = ê m1u1 + m 2 u 2 - 1 1 ú
2 êë (m1 + m 2 ) úû

1 é m1m 2 ù
or DK = ê (u1 - u 2 ) 2 ú
2 ë (m1 + m 2 ) û
Since (u1 – u2)2 is +ive, so DK is also positive.
As m1 and m2 are positive and square of a real quantity (number) is necessarily positive. Thus in an inelastic collision there is
always a loss of kinetic energy.
(i) In case of mechanical collisions, the loss in energy is converted into heat, sound etc.
(ii) In case of atomic collisions, it is converted into potential energy of the orbital electrons resulting into excitation of the
atoms.
Special Cases
If the target is initially at rest, i.e., u2 = 0, then
1 é m1m 2 ù
Loss of kinetic energy, DK = ê .u12 ú
2 ë m1 + m 2 û
1
But as K i = m1u12
2
DK m2
So, =
K i (m1 + m 2 )
Now if the target is massive, that is, m2 > m1.
DK
= 1 or DK = K
Ki i

If a light moving body strikes a heavy target at rest and sticks to it, practically all its kinetic energy is lost.

INELASTIC COLLISION
In an inelastic collision, the colliding bodies do not return to their original shape and size completely after collision. Some part of the
mechanical energy of the system goes to the deformation potential energy, and some part of energy may be lost to heat, sound or other
forms of energy. Thus, only linear momentum remains conserved in case of an inelastic collision.
In case of inelastic collision :
1. Kinetic energy is not conserved. Loss in K.E takes place
2. Linear momentum is conserved
3. Total energy is conserved
4. It takes place under influence of non-conservative force.
r r r r
5. 0 < e < 1 v 2 - v1 = e (u1 - u 2 )
138 Physics
Examples of inelastic collision :
Collision between two vehicles, collision between a ball and floor.
Inelastic Collision (Head on) Particles do not Stick After Collision
Particles do not stick after collision in an inelastic collision.
Let two particles of masses m1 and m2 are moving with velocities u1 and u2 respectively.
The final velocities of m1 and m2 after collision are v1 and v2 and coefficient of restitution is e.
On application of law of conservation of momentum,
m1 v1 + m2 v2 = m1 u1 + m2 u2 ......(1)
According to Newton's law of impact
v2 – v1 = e (u1 – u2) ......(2)
From eqns. (1) and (2)
(m1 - em 2 ) m (1 + e) m (1 + e) (m - em1 )
v1 = u1 + 2 u 2 and v 2 = 1 u1 + 2 u2
m1 + m 2 m1 + m 2 m1 + m 2 m1 + m 2
1 1 1 1
Loss in kinetic energy, DK = Ki – Kf = m1u12 + m 2 u 22 - m1v12 - m 2 v 22
2 2 2 2
m1m 2
Þ DK = (u1 - u 2 )2 (1 - e2 )
2(m1 + m 2 )

Illustration 5 :
Two equal spheres of mass m are in contact on a smooth horizontal table. A third identical sphere impinges symmetrically on
them and is reduced to rest. Prove that e = 2/3 and find the loss in K.E.
Sol. Let u be the velocity of sphere A before impact. As the spheres are identical, the triangle ABC formed by joining their centres is
equilateral. The spheres B and C will move in direction AB and AC after impact making an angle of 30° with the original line of
motion of sphere A.
Let v be the speed of the other spheres after impact
From momentum conservation, v
mu = v cos 30° + mv cos 30°
u=v 3 …(i) u B
From newton’s experimental law, for an oblique collision, we have to A C
take components along the normal, i.e., along AB for spheres A and B.
Hence, v
vB – vA = – e(uB – uA)
Þ v – 0 = – e (0 – u cos 30°)
v = e u cos 30° …(ii)
Combining Eqs. (i) and (ii), we get e = 2/3.
1 2 æ 1 2ö
Loss in KE = mu – 2 è mv ø
2 2
2
1 æ uö 1
= mu 2 – m ç ÷ = mu 2
2 è 3ø 6

Illustration 6 :
A block of mass 1.2 kg moving at a speed of 20 cm/s collides head on with a similar block kept at rest. The coefficient of
restitution is 0.6. Find the loss in kinetic energy during collision.
Sol. Suppose the first block moves at a speed v1 and the second at v2 after collision. Since the collision is head on, the two blocks
move along the original direction of motion of first block. Using the principle of conservation of momentum,
(1.2 × 0.2) = 1.2v1 + 1.2v2 ; v1 + v2 = 0.2 .... (i)
By Newton’s law of restitution,
v2 – v1 = – e (u2 – u1)
v2 – v1 = – 0.6 (0 – 0.2) ; v2 – v1 = 0.12 ... (ii)
Adding equations (i) and (ii), 2v2 = 0.32
v2 = 0.16 m/s or 16 cm/s ; v1 = 0.2 – 0.16 = 0.04 m/s = 4 cm/s
1 1 1
Loss in K. E. = ´ 1.2 ´ (0.2) 2 - ´ 1.2 ´ (0.16) 2 - ´ 1.2 ´ (0.04) 2
2 2 2
= 0.6 [0.04 - 0.0256 - 0.0016] = 0.6 ´ 0.0128 = 7.7×10–3 J
Work, Energy and Power 139

Illustration 7 :
A body of mass 2 kg makes an elastic collision with another body at rest and continues to move in the original direction
with a speed equal to one-third of its original speed. Find the mass of the second body.
u
Sol. m1 = 2 kg, u1 = u, u2 = 0, v1 =
3
In an elastic collision linear momentum and kinetic energy remain conserved.
\ By the principle of conservation of linear momentum, m1u1 + m2u2 = m1v1 + m2v2

u 4
2u + 0 = 2 ´ + m2 v2 Þ m2v2 = u
3 3
Since kinetic energy also remains conserved.
1 1 1 1
From the principle of conservation of KE., m1u12 + m2 u 22 = m1v12 + m2 v22
2 2 2 2
1 1 u2 1 16 2
´ 2 ´u2 + 0 = ´ 2´ + m2 v22 Þ m2 v22 = u
2 2 9 2 9
4 4 4
Dividing, v2 = u \ m2 ´ u = u \ m2 = 1 kg
3 3 3
140 Physics
Work, Energy and Power 141

Textbook Exercises
6.1 The sign of work done by a force on a body is important 6.3 Given below are examples of some potential energy
to understand. State carefully if the following quantities functions in one dimension. The total energy of the
are positive or negative: particle is indicated by a cross on the ordinate axis. In
(a) Work done by a man in lifting a bucket out of a well each case, specify the regions, if any, in which the particle
by means of a rope tied to the bucket. cannot be found for the given energy. Also, indicate the
(b) Work done by gravitational force in the above case. minimum total energy the particle must have in each case.
(c) Work done by friction on a body sliding down an Think of simple physical contexts for which these
inclined plane. potential energy shapes are relevant.
(d) Work done by an applied force on a body moving on V(x) V(x)
a rough horizontal plane with uniform velocity.
V0
(e) Work done by the resistive force of air on a vibrating V0
pendulum in bringing it to rest.
E
Sol. (a) It is positive, as force and displacement are in same E
direction. a x
(b) It is negative, as the bucket is moving against the V(x) =V(a) 0 a b x
direction of gravitational force. (a) –V1
(b)
(c) It is also negative, as the friction is always opposite V(x)
to the direction of motion. V(x)
(d) It is positive as the force and displacement are in same V0
V3
direction.
(e) It is negative as the resistive force acts against the V0 E
direction of motion. V –a/2–a/2 x
E 1 –b/2 b/2
6.2 A body of mass 2 kg initially at rest moves under the a c b d x –V1
action of an applied horizontal force of 7N on a table with (c) (d)
coefficient of kinetic friction = 0.1. Calculate the Sol. We know that the total energy of the body is given by
(a) Work done by the applied force in 10s. 1
(b) Work done by friction in 10s E = K.E. + P.E. or K.E. = E – P.E. = mv 2
2
(c) Work done by the net force on the body in 10s So, K.E. of the body can never be negative. Thus, P.E.
(d) Change in K.E. of the body in 10s and interpret your cannot be greater than E.
results. (a) In the region between x = 0 and x = a, P.E. is zero,
Sol. Given : m = 2kg, u = 0, F = 7N, m = 0.1, t = 10s, W = ? so K.E. is positive. In the region x > a, the P.E. (V0)
Acceleration produced by applied force has a value greater than E. So K.E. will be negative
F 7 in this region. Thus, the particle can’t be present in
= a1 = = = 3.5 m/s2.
m 2 the region x > a.
Force of friction = f = mR = mmg = 0.1 × 2 × 9.8 = 1.96 N The minimum total energy that the particle can have
Retardation produced by friction in this case is zero.
- f -1.96 (b) In region x < a and x > b, the P.E. is V0 which is
= a2 = = = – 0.98 m/s2 greater than the total energy of the particle. So K.E.
m 2
Net acceleration = a = a1 + a2 = 3.50 – 0.98 = 2.52 m/s2 will be negative in this region. Thus the particle cannot
Distance moved by the body in 10 seconds be present in the region x < a and x > b. In the region
1 1 between x > a and x < b, the P.E. is negative. This
S = ut + at 2 = 0 + ´ 2.52 ´ (10) 2 = 126 m. means that there is a positive value of K.E. so the
2 2
particle can be present in the region between x > a
(a) Work done by the applied force = W = FS = 7 × 126
and x < b.
= 882 J
(b) Work done by the frictional force = – f ×s The minimum total energy that the particle can have
= – 1.96 × 126 = – 246.9 J in this case is –V1.
(c) Work done by the net force = Net force × distance (c) Here in all the regions i.e. – ¥ < x < a, a < x < c,
= (7– 1.96) × 126 = 635 J c < x < b, b < x < d and d < x < ¥, the P.E. is greater
(d) Velocity at the end of 10 s than the total energy.
is v = u + at = 0 + 2.52 × 10 = 25.2 m/s So, the particle cannot be present in the region
1 1 – ¥ < x < ¥.
Final K.E. = mv 2 = ´ 2 ´ (25.2) 2 = 635 J The minimum total energy that the particle can have
2 2
in this case is V1.
Initial K.E. = 0 (d) In this case, the P.E. of the particle is more than the
\ Change in KE = 635 – 0 = 635 J total energy
This shows that the change in K.E. is equal to the work
done by the net force.
142 Physics
b a a b Sol. (a) The heat energy required for burning of the casing
(E) in the regions - < x < - and <x< .
2 2 2 2 of a rocket in flight is obtained from the rocket itself.
So, K.E. of the particle will be negative in this regions, The energy is obtained at the expense of the mass
b a of the rocket and its kinetic and potential energy.
thus it will not be present in the regions - , x < -
2 2 (b) The gravitational force on the comet due to Sun is
a b a conservative force. Since the work done by a
and <x< .
2 2 conservative force over a closed path is always zero
The minimum total energy that the particle can have
in this case is –V1. (irrespective of the nature of path). Hence the work
done by the gravitational force over every complete
6.4 The potential energy function for a particle executing
1
orbit of the comet is zero of the comet. .
linear simple harmonic motion is given by V ( x ) = kx 2 , (c) As the satellite comes closer and closer to Earth, its
2
potential energy decreases. Since according to the
where k is the force constant of the oscillator. For
k = 0.5 N/m, the graph of V (x) versus x is shown in the law of conservation of energy, the sum of kinetic and
figure. Show that a particle of total energy 1 joule moving potential energy must remain constant, so the kinetic
under this potential must turn back, when it reaches energy and hence velocity of the satellite increases.
x = ± 2m. However, the total energy of the satellite
Sol. At any instant, the total energy of an oscillator is equal continuously decreases some-what due to the loss
to the sum of K.E. and P.E. of energy against friction i.e. atmospheric resistance.
y (d) In the case shown in fig. (i), the man applies the force
1 1 on the 15 kg mass in vertically upward direction but
E = mv 2 + kx 2 v( x)
2 2 walks 2m in the horizontal direction so the angle
When v = 0 between F and S is 90°, hence work done W = F.S
x¢ x is zero.
1 2 2m 2m
E= kx . O In fig. (ii), the force applied on the mass of 15 kg in
2
the vertically upward direction (= weight of the body)
At this instant the particle will comeback.
and the mass is also lifted along the vertical so q = 0
1 1 1 2 \ W = mgS = 15 ´ 9.8 ´ 2 = 294 J
As E = 1 J and k = N /m1 = ´ x
2 2 2 Thus work done in second case is greater.
Þ x 2 = 4 Þ x = ±2 m 6.6 Underline the correct alternative:
(a) When a conservative force does positive work on a
6.5 Answer the following:
body, the potential energy of the body increases/
(a) The casing of a rocket in flight burns up due to
decreases/remains unaltered.
friction. At whose expense is the heat energy
(b) Work done by a body against friction always results
required for burning obtained? The rocket or the
in a loss of its kinetic/potential energy.
atmosphere?
(c) The rate of change of total momentum of a many
(b) Comets move around the sun in highly elliptical
particle system is proportional to the external force/
orbits. The gravitational force on the comet due to
sum of the internal forces on the system.
the sun is not normal to the comet's velocity in
(d) In an inelastic collision of two bodies, the quantities
general. Yet the work done by the gravitational force
over every complete orbit of the comet is zero. Why? which don't change after the collision are the total
kinetic energy/total linear momentum/total energy
(c) An artificial satellite orbiting the earth in very thin
of the system of two bodies.
atmosphere loses its energy gradually due to
Sol. (a) Potential energy of the body decreases. The
dissipation against atmospheric resistance, however
small. Why then does its speed increase progressively conservative force does positive work on a body,
as it comes closer and closer to the earth? when it displaces the body in the direction of the
force. The body therefore approaches the centre of
(d) In Fig. given below (i) the man walks 2m carrying
a mass of 15 kg in his hands. In Fig. (ii), he walks force thus, decreasing x. Hence P.E. decreases.
the same distance pulling the rope behind him. The (b) Work is done by a body against friction at the expense
rope goes over a pulley, and a mass of 15 kg hangs of its kinetic energy. Hence work done by a body
at its other end. In which case is the work done against friction results in a loss of its kinetic energy.
greater? (c) Internal forces cannot change the total or net
momentum of a system. Hence the rate of change of
total momentum of many particle system is
proportional to the external force on the system.
15kg 15kg (d) In an elastic collision of two bodies the quantities
which do not change after the collision are total linear
(i) (ii)
Work, Energy and Power 143

momentum and total energy of the system of two (i) t1/2 (ii) t
bodies (if the system is isolated). The total K.E. of (iii) t3/2 (iv) t2
the system, is not conserved as it may change to an Sol. v = u + at = 0 + at = at
equivalent amount of energy in some other form. Power = P = F × v = ma × at = ma2t
6.7 State if each of the following statements is true or false. Q m and a are constants, P µ t \ Ans (ii)
6.10 A body is moving unidirectionally under the influence of
Give reasons for your answer.
a source of constant power. Its displacement in time t is
(a) In an elastic collision of two bodies, the momentum
proportional to
and energy of each body is conserved. (i) t1/2 (ii) t
(b) Total energy of a system is always conserved, no (iii) t3/2 (iv) t 2
matter what internal and external forces on the body Sol. P = F × v = [MLT–2] [LT–1] = [ML2T–3] = constant
are present.
L2
(c) Work done in the motion of a body over a closed loop \ L2T–3 = constant Þ = constant Þ L2 µ T3
is zero for every force in nature. T3
(d) In an inelastic collision, the final kinetic energy is Þ L2 µ T3/2
always less than the initial kinetic energy of the \ Displacement is proportional to t3/2.
system. 6.11 A body constrained to move along the z–axis of a
coordinate system is subjected to a constant force
Sol. (a) False. The total momentum and total energy of the uur
system are conserved but not of each body. F = (-iˆ + 2 ˆj + 3kˆ) N Where
(b) False. The external forces on the system may change iˆ , ˆj & k̂ are unit vector along the x–,y– & z– axis
(increase or decrease) the total energy of the system. respectively. What is the work done by this force in
(c) False. Work done in the motion of a body over a moving the body over a distance of 4m along the z-axis?
uur uur
closed loop is zero only when the body moves under Sol. F = (-iˆ + 2 ˆj + 3kˆ) N and S = 4kˆ m
the action of conservative force (like gravitational or Since the displacement is in the direction of applied force
electrostatic forces). It is not zero when the forces are q = 0º
non-conservative e.g. frictional forces etc. uuruur
W = F .S = (-iˆ + 2 ˆj + 3kˆ).(4kˆ) = 12 J
(d) True. Usually but not always because in an inelastic 6.12 An electron and a proton are detected in a cosmic ray
collision, some kinetic energy usually changes into experiment, the first with kinetic energy 10 keV and the
other forms of energy. second with 100 keV. Which is faster, the electron or
6.8 Answer carefully, with reasons:
proton? Obtain the ratio of their speeds.
(a) In an elastic collision of two billiard balls, is the total
Take mass of electron = 9.11 × 10–31 kg. Mass of proton
kinetic energy conserved during the short time of
= 1.67 × 10–27 kg and 1 keV = 1.6 × 10–16J.
collision of the balls (i.e., when they are in contact)?
(b) Is the total linear momentum conserved during the 1
Sol. Kinetic energy, Ee = me ve2 = 10 × 1.6 × 10–16;
short time of an elastic collision of two balls? 2
(c) What are the answers to (a) and (b) for an inelastic 1
Ep = m p v 2p = 100 × 1.6 × 10–16.
collision? 2
(d) If the potential energy of two billiard balls depends
only on the separation distance between their centres, me ve2 1
Dividing, =
is the collision elastic or inelastic? (Note, we are m p v 2p 10
talking here of potential energy corresponding to the
force during collision, not gravitational potential ve 1 mp 1.67 ´10-27
energy). Þ = . =
vp 10 me 10 ´ 9.11´10-31
Sol. (a) No, the total kinetic energy is not conserved during
the given elastic collision because a part of the kinetic
1.67
energy is used in deforming the balls in that short = 102 = 13.53
interval for which they are in contact during collision 91.1
and gets converted into potential energy. In an elastic [ ve = 13.53 vp Þ the electron is faster than the proton.
collision, the K.E. before and after collision is same. 6.13 A rain drop of radius 2 mm falls from a height of 500
(b) Yes, the total linear momentum is conserved during m above the ground. It falls with decreasing acceleration
the short time of an elastic collision of two balls.
due to viscous resistance of air until half of its original
(c) In an inelastic collision, total K.E. is not conserved
during collision and even after collision. The total height. It attains its maximum (terminal) speed and moves
linear momentum is however conserved during as well with uniform speed thereafter. What is the work done by
as after collision. the gravitational force on the drop in the first half and
(d) The collision is elastic. second half of the journey? [Density of H2O = 103 kg/
6.9 A body is initially at rest. It undergoes one dimensional m3 ]. What is the work done by the resistance force in
motion with constant acceleration. The power delivered to the entire journey if its speed on reaching the ground is
it at time t is proportional to 10 m/s?
144 Physics
Sol. Here, r = 2 mm = 2 ×10–3 m, r = 103 kg/m3
500
Density of water r H2 O = 103 kg/m3
Distance moved in half of the journey = S = = 250 m. \ Mass of water pumped = Vol × density
2 = 30 × 103 kg
4
Mass of the drop = Vol. × density = pr 3r W mgh
3 Output power = =
t t
4 22
= ´ ´ (2 ´10 -3 )3 ´ 103 = 3.35 × 10–5 kg
3 7 30 ´ 103 ´ 9.8 ´ 40
= = 13070 W
Work done = W = F × S = 3.35 × 10–5 × 9.8 × 250 = 0.082 J 900
Work done by the gravitational force remains same weather
Output power 13070
the drop falls under decreasing acceleration or with uniform Input power = =
Efficiency 31/100
speed. So, the work done in second half of the journey
= 43567 W = 43.567 kW
is also 0.082J. 6.16 Two identical balls bearings in contact with each other
Energy of the drop on reaching the ground and resting on a frictionless table are hit head on by
= E = mgh = 3.35 × 10–5 × 9.8 × 500 = 0.164 J another ball bearing of the same mass moving initially
(considering no resistive forces). with a speed v. If the collision is elastic, which of the
following is a possible result after collision?
1 2 1 1 2 3
Kinetic energy = mv = ´ 3.35 ´ 10-5 ´10 2 = 1.675 × I v
2 2
1 v=0 v/2 2
–3
10 J = 0.001675 = 0.1623 J 2 3 1 2 3
II v
= Work done by the resistive force. v
6.14 A molecule in a gas container hits the wall with speed III v=0 v
200 m/s at an angle 30° with the normal and rebound with 1 2 3 3
the same speed. Is momentum conserved in the collision? v/3
Is collision elastic or inelastic? Sol. Let m ® mass of each ball bearing.
A Before collision, total KE of the system
1 2 1 2
= mv + 0 = mv
2 2
After collision, KE of the system in case I is,
30° 2
N O 1 ævö 1
30° E1 = (2 m ) ç ÷ = mv 2
2 2
è ø 4
1
In case II is, E2 = mv 2
2
B 2
Sol. Momentum is conserved in all types of collisions. 1 ævö 1
In case III is, E3 = (3m ) ç ÷ = mv 2
Let m ® mass of the molecule, M ® mass of the wall 2 è3ø 6
K.E is conserved in case II.
1 2 1 2 \ The only possibility is case II.
\ K.E. after collision = m(200) + M(0) = 2 × 104 mJ. 6.17 The bob A of a simple pendulum released from 30° to the
2 2 vertical hits another bob B of the same mass at rest on
a table. How high does the bob A rise after the collision?
1 Neglect the size of the bobs and assume the collision to
K.E. before collision = m ´ (200)2 = 2 ´104 mJ
2 be elastic.
= K.E. after collison. O
Q K.E. before collison = K.E. after collision
\ The collision is elastic. 30º
6.15 A pump on the ground floor of a building can pump up
water to fill a tank of volume 30 m3 in 15 minutes. If the
tank is 40 m above the ground and the efficiency of the
pump is 30%, how much electric power is consumed by A
the pump? B m
Sol. Volume of water = 30m3, t = 15 min = 15 × 60 = 900 s, h
= 40m, Sol. When two bodies of same mass undergo elastic collision
their velocities are interchanged. So after collision, A will
Output power
Efficiency = h = = 30% not rise but it will come to rest and B will move with the
Input power velocity of A.
Work, Energy and Power 145

6.18 The bob A of a pendulum is released from a horizontal Sol. Let, r be the density of air, then distance covered by wind
position A as shown in the figure. If the length of the in time ‘t’ is = vt.
pendulum is 1.5 m, what is the speed with which the bob
(a) Vol. of wind flowing/s = Av,
arrives at the lower most point B, given that it dissipates
5% of its initial energy against air reristance? Mass of wind flowing/s = Av r
Sol. Here, h = 1.5 m, v = ?, Energy dissipated = 5% (b) Mass of wind flowing in t sec = Av r t ;
1.5 m 1 1 1
O A KE of air = mv 2 = ( Avrt )v 2 = Av 3rt
2 2 2
(c) Electrical energy produced
1.5 m 25 1 1
= ´ KE of air = ´ Av 3rt
100 4 2
B output power
Q B is the lowest position of the bob, its P.E. at B is zero. [Given that efficiency = ]
Input power
At A, it is mgh.
While going from A to B, P.E. is converted to K.E. Energy 1 3
= 95% (mgh). If v the velocity at B, then K.E. = Av rt
Energy 8 1
\ Power = = = Av3r
1 2 95 Time t 8
mv = (mgh);
2 100
1
95 19 = ´ 30 ´103 ´ 1.2 = 4500 W = 4.5 kW
v= ´ 2gh = ´ 2 ´ 9.8 ´ 1.5 = 5.285 m/s 8
100 20
6.22 A person trying to lose weight lifts a 10 kg mass, to a
6.19 A trolley of mass 300 kg carrying a sand bag of 25 kg height of 0.5 m each time for 1000 times. Assume that
is moving uniformly with a speed of 27km/h on a PE lost each time she lowers the mass is dissipated
frictionless track. After a while, sand starts leaking out (a) How much work does she do against the gravitational
of a hole on the trolley’s floor at the rate of 0.05 kg/s. force?
What is the speed of the trolley after the entire sand bag (b) Fat supplies 3.8 × 10 7 J of energy/kg which is
is emptied? converted to mechanical energy with a 20%
Sol. Since the trolley carrying the sand bag is moving uniformly, efficiency rate. How much fat will the dieter use up?
so the external force on the system is zero. When the sand Sol. (a) m = 10kg, h = 0.5 m, n = 1000
leaks out, it doesn’t apply any external force on the trolley. Work done against the gravitational force = n (mgh)
So the speed of the trolley doesn’t change i.e. it remains = 1000 × 10 × 9.8 × 0.5 = 49000 J.
27 kmh–1. (b) Mechanical energy supplied by 1 kg of fat with 20%
6.20 A particle of mass 0.5 kg travels in a staright line with
20
velocity v = ax3/2, where a = 5m–1/2s–1. What is the work efficiency = 3.8 × 107 × = 0.76 × 107J
done by the net force during its displacement from 100
x = 0 to x = 2m? or 0.76 × 107 J energy is supplied by = 1 kg fat.
Sol. Here, m = 0.5 kg, v = ax3/2, a = 5m–1/2s–1. \ Fat used up by dieter in the exercise
Initial velocity at x = 0 is v1 = a × 0 = 0
1
Final velocity at x = 2 is v2= a.23/2 = 5 × 23/2 = 7
´ 49000 = 6.45 ´ 10-3 kg
0.76 ´10
Work done = Inerease in KE =
1
2
(
m v22 - v12 ) 6.23 A family uses 8 kW of power. Direct solar energy is
incident on the horizontal surface at an average rate of
1 200 W per square meter. If 20% of this energy can be
= ´ 0.5[(5 ´ 23/ 2 )2 - 0] = 50 J converted to useful electrical energy, how large an area
2
6.21 The blades of a windmill sweep out a circle of area A is needed to supply 8 kW?
(a) If the wind flows at a velocity v perpendicular to the Sol. Let A ® Area of the surface in square metre.
circle, what is the mass of air passing through it 1 m2 receives power = 200 W
in time t? 20
(b) What is KE of the air? \ Useful electrical energy produced = 100 (200) = 40 W
(c) Assume that the wind mill converts 25% of the 40 W power is produced by an area of 1m2
Þ 8 kW power will be produced by an area
wind’s energy into electrical energy and that A= 30
m2, v = 36 km/h and the density of air is 1.2 kg/m3. 1
= ´ 8000 = 200 m 2
What is the electrical power produced? 40
146 Physics

6.24 A bullet of mass 0.012 kg and horizontal speed 70 m/s


strikes a block of wood of mass 0.4 kg and instantly 2 ´ 10 1
tB = . = 2 2s.
comes to rest with respect to the block. The block is 10 sin 30º
suspended from the ceiling by a wire. Calculate the height
2 ´ 10 1 2 2
to which the block rises. Also estimate the amount of heat tC = . = s.
10 sin 60º 3
produced in the block
Sol. Given : m1 = 0.012 kg, u1 = 70 m/s, m2 = 0.4 kg, u2 =0 6.26 A 1kg block situated on a rough incline is connected to
a spring of spring constant 100 N/s as shown in the fig.
As the bullet gets embedded in the block, the two behave
The block is released from rest with the spring in the
as one body.
unstretched position. The block moves 10 cm down the
Let v = velocity of the combination.
incline before coming to rest. Find the coefficient of
Applying law of coservation of linear momentum,
friction between the block and the incline. Assume that
(m1+ m2)v1 = m1u1+ m2u2 = m1u1 [ Q u2 = 0 ]
the spring is of negligible mass and the pulley is
m1u1 0.012 ´ 70 0.84 frictionless.
\ v= = = = 2.04 m/s
m1 + m2 0.012 + 0.4 0.412 k = 100 N/m
Let h = height to which the block rises.
\ PE of the combination = KE of the combination
F
1 m = 1 kg
( m1 + m2 ) gh = ( m1 + m2 )v 2
2
37º
v 2 2.04 ´ 2.04
Þ h= = = 0.212m Sol. From figure, R = mg cosq and F = mr = mmg cosq
2g 9.8 ´ 2
Heat produced = energy lost
= initial KE of the bullet – final KE of the combination
1 1 F R
= m 1u12 - ( m1 + m2 )v 2
2 2 q
mgcos q
1 2 1 2 mg mg sin q
= ´ 0.012 ´ (70) - (0.412)(2.04) q = 37°
2 2
Net force on the block in the downward direction
= 29.4 – 0.86 = 28.54 J
= mgsinq - F = mgsinq – mmgcosq = mg (sinq - mcosq)
6.25 Two inclined frictionless tracks, one gradual and the Distance moved = x = 10 cm = 0.1 m
other steep meet at A from where two stones are allowed In equilibrium, work done = PE of the spring
to slide down from rest, one on each track (Fig.). Will the
1 2
stones reach the bottom at the same time? Will they reach = mg (sinq - mcosq) x = kx
2
there with the same speed? Explain, Given q1 = 30º,
2mg (sinq - mcosq) = kx
q2 = 60º, and h = 10m, what are the speeds and times taken 2 × 1 × 10(sin37° - m cos 37°) = 100 × 0.1
by the two stones? 20(0.601 – m × 0.798) = 10 Þ m = 0.126
A 6.27 A bolt of mass 0.3 kg falls from the ceiling of an elevator
moving down with an uniform speed of 7m/s. It hits the
h floor of the elevator (length of the elevator = 3m) and does
q1 q2 not rebound. What is the heat produced by the impact?
Would your answer be different if the elevator were
B C
stationary?
1 2 Sol. Given : m = 0.3 kg, v = 7 m/s, h = 3 m
Sol. mv = mgh, v = 2 gh
2 Q The relative velocity of the bolt w.r.t. the elevator is
zero.
= 2 ´ 10 ´ 10 ms -1 = 14.14 ms-1
\ Only PE of the ball is converted into heat energy by
1 the impact. Amount of heat produced = PE lost by
vB = vc 14.14 ms–1 , l = ( g sin q ) t 2 the ball = mgh = 0.3 × 9.8 × 3 = 8.82 J
2
The answer will not change if the elevator was stationary
h h 1 2h 1 as the relative velocity of the bolt w.r.t. the elevator would
sin q = , l= = g sin qt 2 or t = . still be zero.
l sin q 2 g sin q
Work, Energy and Power 147

6.28 A trolley of mass 200 kg moves with an uniform speed Sol. The potential energy of a system of two masses varies
of 36 km/h on a frictionless track. A child of mass 20 1
kg runs on the trolley from one end to the other (10 m inversely as the distance (r) between them i.e, V ( r ) µ .
r
away) with a speed of 4 m/s relative to the trolley in a When the two billiard balls touch eachother, P.E. becomes
direction opposite to the trolley’s motion and jumps out zero i.e., at r = R + R = 2R; V (r) = 0. Out of the given
of the trolley. What is the final speed of the trolley? How
graphs, curve (v) only satisfies these two conditions.
much has the trolley moved from the time the child begins
Therefore, all other curves cannot possibly describe the
to run?
elastic collision of two billiard balls.
Sol. Mass of the trolley = m1 = 200kg, mass of the child = 20 kg,
speed of trolley = v = 36 km/h = 10 m/s 6.30 Consider the decay of a free neutron at rest: n ® p + e–.
Before the child starts running, Show that the two body decay of this type must necessarily
momentum of the system = p1 = (m1 + m2) v = (200 + 20)10 give an electron of fixed energy, and therefore, cannot
= 2200 kgm/s. account for the observed continuous energy distribution
When the child starts running in the opposite direction in the b-decay of a neutron or a nucleus, Fig.
of the trolley suppose v ' is the final speed of the trolley
(w.r.t. earth)

No. of b particles per


\ Speed of the child relative to earth = v ' – u

unit energy interval


\ Momentum of the system when the child is running
= p2 = 200 v ' + 20( v ' – u) = 220 v ' = 80
2280
v' = = 10.36 m/s.
220
Time taken by the child to run a distance of 10 m over the
10
trolley = = 2.5 s
4
Distance moved by the trolley in that time
= 10.36 ´ 2.5 = 25.9 m. K.E. of b-particle emitted
6.29 Which of the following potential energy curves in Fig.
cannot possibly describe the elastic collision of two billiard Sol. Let the masses of the electron and proton be m and M
balls? Here r is distance between centres of the balls. respectively. Let v and V be the velocities of electron and
proton respectively. Using law of conser vation of
momentum.
Momentum of electron + momentum of proton
V(r) V(r) = momentum of neutron
m
\ mv + MV = 0 Þ V = - v
M
2(R) r 2(R) r Clearly, the electron and the proton move in opposite
(i) (ii) directions. If mass Dm has been converted into energy in
the reaction, then
1 2 1
mv + MV 2 = Dm ´ c 2
V(r) 2 2
V(r)
1 2 1 é mù 2
or mv + M ê- ú v = Dmc 2
2 2 ë Mû
2(R) r 2(R) r
1 2é mù
(iii) (iv) or mv ê1 + ú = Dmc 2
2 ë M û

2M Dmc 2
or v2 =
V(r) V(r) m ( M + m)
Thus, it is proved that the value of v2 is fixed since all the
quantities in right hand side are constant. It establishes
2(R) r 2(R) r that the emitted electron must have a fixed energy and thus
(v) we cannot account for the continuous energy distribution
(vi)
in the b-decay of a neutron.
148 Physics

Practice Questions
Very Short Answer Questions [1 Mark Qs.] 2. Calculate the work done to stretch a spring from x to 2x.
Given the spring constant is k.
1. A man raises a mass ‘m’ to a height ‘h’ and then shifts it 3. Two inclined frictionless tracks are gradual and the other
horizontally by a length ‘x’. What is the work done against steep meet at A from where two stones are allowed to slide
the force of gravity? down from rest on each track. Will the stones reach the
2. Name and define SI unit of work.
bottom at the same time and at same speed. Why?
3. Give an example in which a force does work on a body
but fails to change its kinetic energy. A
4. An artificial satellite is at a height of 36,500 km above
earth’s surface. What is the work done by earth’s
gravitational force in keeping it in its orbit ?
5. Name the process in which (i) momentum is conserved but
K.E. is not conserved and (ii) momentum changes but K.E.
B C
does not change.
6. What is Einstein’s mass-energy equivalence relationship? 4. What is an elastic collision? What will happen when
7. A mass m collides with another mass 2m and sticks to it. (i) a heavy body collides on a light mass at rest.
What is the nature of collision? (ii) a light body collides on a heavy mass at rest.
8. Two equal masses, one at rest and another moving, 5. Two identical masses, one at rest and other moving,
undergo elastic oblique collision. If one mass goes at an undergo oblique elastic collision. Show that they will move
angle p/3 with its original direction of motion, what is the at right angles to each other after collision.
direction of motion of the other? 6. Show that in case of a freely falling body the total energy
9. Two bodies m1 and m2 (m1 > m2 ) have equal kinetic remains constant at every point in its paths.
energies. Which will have more momentum? 7. A motor pump is used to deliver water at a certain rate
10. Ten identical balls are placed in contact on a smooth from a given pipe. To obtain ‘n’ times water from the same
surface. If an eleventh identical ball moving with a speed pipe in the same time by what amount (a) the force and
‘u’ collides on the first, what will be the resultant motion (b) power of the motor should be increased.
of the system? 8. Two masses, one n times heavier than the other are
11. Can a constant velocity be maintained in a body moving dropped from same height. How do their momentum
on a rough surface without doing any work on it? compare just before they hit the ground?
12. Does the work done in raising a box on to a platform 9. Two masses, one n times heavier than the other have equal
depends upon how fast it is raised up? If not, why? kinetic energy. Find the ratio of their momentum.
13. A man rowing a boat upstream is at rest with respect to 10. Show that coefficient of restitution for one dimensional
the shore, is he doing work? elastic collision is equal to one.
14. A light body and a heavy body have the same momentum. 11. How do we calculate work done by a force? Write any two
Which one will have greater kinetic energy? conditions under which work done by a force is zero.
15. A spring is cut into two equal halves. How is spring 12. Derive an expression for the kinetic energy of a body of
constant of each half affected? mass m moving with velocity v by calculus method.
16. What is one electron-volt ? 13. What is meant by mass-energy equivalence?
17. Draw the variation of potential energy stored in a spring Discuss its significance in physics.
as a function of extension. 14. Prove that bodies of identical masses exchange their
18. The earth moving round the sun in circular orbit is acted velocities after head-on elastic collision.
upon by a force and hence work must be done on the earth 15. Consider a one-dimensional motion of a particle with total
by the force. Explain. energy E. There are four regions A,B, C and D in which
19. If one of the two colliding particles is initially at rest, is the relation between potential energy V, kinetic energy (K)
it possible for both of the particles to be at rest after and total energy E is as given below:
collision ? Region A : V > E
Region B : V < E
Short Answer Questions [2 or 3 Marks Qs.] Region C : K > E
Region D : V > K
1. Two springs A and B with constant kA and kB (kA > kB) State with reason in each case whether a particle can be
are given. In which of the spring more work is to be done, found in the given region or not.
if 16. The bob A of a pendulum released from horizontal to the
(a) they are stretched by the same amount, vertical hits another bob B of the same mass at rest on
(b) they are stretched by same force? a table as shown in Fig.
Work, Energy and Power 149

5. The relation between the displacement x and the time t for a


A body of mass 2 kg moving under the action of a force is
m
given by x = t3/3, where x is in metre and t is in second.
1m Calculate work done by the body in first 2 second.
6. A body of mass 50 kg has a momentum of 1000 kg m/s.
B m Calculate its K.E.
7. The masses of 1 g and 4 g are moving with equal kinetic
If the length of the pendulum is 1m, calculate energies. Calculate the ratio of the magnitudes of their linear
(a) the height to which bob A will rise after collision. momenta.
(b) the speed with which bob B starts moving. 8. A bullet weighing 10 g is fired with a velocity of 800 m/s.
Neglect the size of the bobs and assume the collision After passing through a mud wall 1 m thick, its velocity
to be elastic. decreases to 100 m/s. Find the average resistance offered
17. A raindrop of mass 1.00 g falling from a height of 1 km by the mud wall.
hits the ground with a speed of 50 ms–1.Calculate 9. A circle draws a tractor of mass 1000 kg at a steady rate of
20 m/s on a level road. The tension in the compling is 2000 N.
(a) the loss of P.E. of the drop.
Calculate the power spent on the tractor.
(b) the gain in K.E. of the drop.
10. Water falling from a 50 m high fall is to be used for generating
(c) Is the gain in K.E. equal to loss of P.E.? electric power. If 1.8 × 105 kg of water falls per hour and half
If not why. the gravitational potential energy can be converted into
Take g = 10 ms–2 electric energy, how many 100 watt lamps can be lit?
Long Answer Questions [5 Marks Qs.] 11. A body of mass 2 kg makes an elastic collision with another
body at rest and continues to move in the original direction
1. Derive an expression for the work done by a variable force. with a speed equal to half its original speed. Find the mass
2. Explain how potential energy can be stored in a spring. of the second body.
Obtain an expression for it and discuss graphically its 12. Suppose the average mass of raindrops is
nature of variation. 3.0 ´ 10–5kg and their average terminal velocity 9 m s–1.
3. A balloon filled with helium rises against gravity increasing Calculate the energy transferred by rain to each square metre
its potential energy. The speed of the balloon also of the surface at a place which receives 100 cm of rain in a
increases as it rises. How do you reconcile this with the year.
law of conservation of mechanical energy? You can 13. An engine is attached to a wagon through a shock absorber
neglect viscous drag of air and assume that density of air of length 1.5m. The system with a total mass of 50,000 kg is
is constant. moving with a speed of 36 km h –1 when the brakes are
4. State and prove work energy theorem. applied to bring it to rest. In the process of the system
5. Discuss elastic collision in one dimension. Obtain being brought to rest, the spring of the shock absorber gets
expressions for velocities of the two bodies after an elastic
compressed by 1.0 m. If 90% of energy of the wagon is lost
collision.
due to friction, calculate the spring constant.
Numerical Questions [3 or 5 Marks Qs.] 14. The human heart forces 4000 cm 3 of blood per minute
through the arteries under pressure of 130 mm. The density
1. A man moves on a straight horizontal road with a block of of blood is 1.03 g cm–3. What is the horse power of the heart?
mass 2 kg in his hand. If he moves a distance of 40 m with an 15. A block of mass 1 kg is pushed up a surface inclined to
acceleration of 0.5 m/s2. Calculate work done by the man on horizontal at an angle of 30º by a force of 10 N parallel
the block during motion. to the inclined surface (Fig. ). The coefficient of friction
2. A body of mass 0.3 kg is taken up an inclined plane of between block and the incline is 0.1. If the block is pushed
length 10 m and height 5 m, and then allowed to slide down up by 10 m along the incline.
to the bottom again. The coefficient of friction between the
body and the plane is 0.15. What is the
F
(a) Work done by the gravitational force over the round
trip? m
(b) Work done by the applied force on the upward journey?
(c) Work done by the frictional force over the round trip.
(d) Kinetic energy of the body at the end of the trip?
ur
3. A particle moves from a point r1 = (2iˆ + 3 ˆj) to another point 30º
uur uur Calculate
r2 = (3iˆ + 2 ˆj) during which a certain force F = (5iˆ + 5 ˆj )
acts on it. Calculate work done by the force on the particle (a) work done against gravity
during this displacement. (b) work done against force of friction
4. A particle moves along the x-axis from x = 0 to x = 5 m under (c) increase in potential energy
the influence of a force given by f = 7 – 2x + 3x2. Calculate (d) increase in kinetic energy
the work done. (e) work done by applied force.
150 Physics

HOTS/Exemplar Questions
Very Short Answer Questions [1 Mark Qs.] 4. A ball of mass m, moving with a speed 2v0, collides
inelastically (e > 0) with an identical ball at rest. Show that
1. The figure shows three paths connecting points a and b. A (a) For head-on collision, both the balls move forward.
r
single force F does the indicated work on a particle moving (b) For a general collision, the angle between the two
along each path in the indicated direction. On the basis of velocities of scattered balls is less than 90°. [Exemplar]
r
this information, is force F conservative? 5. Suppose the average mass of raindrops is 3.0 × 10–5 kg and
their average terminal velocity 9 ms–1. Calculate the energy
transferred by rain to each square metre of the surface at a
place which receives 100 cm of rain in a year. [Exemplar]

Long Answer Questions [5 Marks Qs.]


1. (a) A block with a mass of 2kg rests on a horizontal
[HOTS] frictionless surface and is attached to a wall with
2. Can an object possess work? [HOTS] horizontal ideal inertialess spring under no initial
compression. (The spring constant is 250 N/m). A bob
3. A body falls towards earth in air. Will its total mechanical
of puff having a mass of 0.50 kg and a horizontally
energy be conserved during the fall? Justify. [Exemplar] directed velocity v collides with the block and sticks
4. Why is electrical power required at all when the elevator is to it. Determine v if the spring undergoes a maximum
descending? Why should there be a limit on the number of compression of 10cm.
passengers in this case? [Exemplar] (b) Mountain roads rarely go straight up the slope but
wind up gradually; why? [HOTS]
Short Answer Questions [2 or 3 Marks Qs.] 2. (i) Two smooth tracks of equal length have bump’s—A
up and B down, both of the same curvature. If two balls
1. How is it possible that a flock of birds in flight can have a start simultaneously with the same initial speed, along which
momentum of zero but not have zero kinetic energy? [HOTS] track will the ball complete the journey first.
2. In many situations, friction forces tend to reduce the kinetic
energy of an object; however, friction forces can sometimes A
increase an object’s kinetic energy. Describe a situation in B
which friction causes and increase in kinetic energy. [HOTS]
3. Three baseballs are thrown from the top of the cliff along
paths A, B, and C. If their initial speeds are the same and air (ii) If the initial speed is 2 m/s, and the speed of the ball at
resistance is negligible. which ball will strikes the ground the bottom of the curve on Track B is 3 m/s, then what is the
below with the greatest speed? [HOTS] speed of the ball at the top of the curve on Track A? [HOTS]
A 3. A rocket accelerates straight up by ejecting gas downwards.
In a small time interval Dt, it ejects a gas of mass Dm at a
relative speed u. Calculate KE of the entire system at t + Dt
and t and show that the device that ejects gas does work =
C B æ1ö 2
ç ÷ Dmu in this time interval (neglect gravity). [Exemplar]
2
è ø
h
4. A balloon filled with helium rises against gravity increasing
its potential energy. The speed of the balloon also increases
as it rises. How do you reconcile this with the law of
conservation of mechanical energy? You can neglect
viscous drag of air and assume that density of air is constant.
[Exemplar]
Work, Energy and Power 151

CHAPTER TEST

Time : 45 min. Max. Marks : 24

Directions : (i) Attempt all questions


(ii) Questions 1 to 5 carry 1 mark each.
(iii) Questions 6, 7 and 8 carry 3 marks each.
(iv) Questions 9 and 10 carry 5 marks each.

1. Does the work done in moving a body depend on how fast or how slow the body is moved ?
2. Define power. Write its unit.
3
3. A mass is moving in a circular path with constant speed. What is the work done in th of a rotation?
4
4. Write work-energy theorem.
5. States law of conservation of energy.
6. The momentum of a body is increased by 50%. What is the percentage change in its K.E.?
7. A bullet of mass 50 g moving with a velocity of 400 m/s strikes a wall and goes out from the other side with a velocity of 100
m/s. Calculate work done in passing through the wall.
8. A mass m is placed on a platform from a height ‘h’. The platform is attached to a sping whose other end is fixed to the
ground. Find the compression in the spring, if the spring constant is k.
9. A small block of mass m slides along the frictionless loop-to-loop track shown in the Figure. (a) If it starts from rest at P what
is the resultant force acting on it at Q? (b) At what height above the bottom of loop should the block be released so that the
force it exerts against the track at the top of the loop equals its weight?

5R

R
Q

10. Define the terms elastic collision and inelastic collision. What is the difference between an inelastic collision and a
perfectly inelastic collision?
152 Physics

Solutions
PRACTICE QUESTIONS 18. It is wrong. Since force and displacement are perpendicular
to each other work done is zero.
Very Short Answer Questions 19. This is not possible as law of conservation of momentum
will not be obeyed.
1. Work done against gravity is mgh, as no work is done in
the horizontal displacement. Short Answer Questions
2. The SI unit of work is joule (J). One joule of work is said to
1. Let F is the force applied. W is the work done and ‘x’ is
be done when a force of one newton displaces a body through
a distance of one metre in the direction of applied force.. the distance moved on applying force.
3. When a body is pulled on a rough horizontal surface with
constant velocity, work is done by the applied force on 1 2
F = – kx and energy = W = kx
the body but K.E. of the body remains unchanged. 2
4. Zero.
5. (i) Inelastic collision. WA k A
(ii) Uniform circular motion. (a) For same stretch, =
WB k B
6. E = mc2.
7. Inelastic, as the two bodies are moving together after
collision. Q k A > kB \ WA > WB
p p
The direction of the other mass is at an angle æç - ö÷
(b) For same force,
8.
è2 3ø
i.e., p or 30°. WA F 2 2k B k
= . = B \ WA < WB
6 WB 2k A F 2 kA
p2
9. Energy (E) = \ p = 2mE 2. From the definition of work W = F.ds.
2m
Heavier the mass, more will be momentum. Here F = kx & ds = dx
\ m1 will have more momentum.
10. When the eleventh ball hits the first ball, the tenth ball 2x
1 1 3 2
starts moving with the same speed due to transfer of
momentum in equal proportion.
W = ò kx.dx =
2
k[(2 x )2 - x 2 ] = k 3x 2 = kx
2 2
x
11. No, work has to be done to compensate the energy loss
due to friction. 3. Let AM = h and AB = s, a = acceleration of the stone
12. Since, the gravitational force is conservative in nature, so falling through AB.
work done against it depends only on initial and final
points, not on time. A
13. Yes, he is doing work to oppose the river current. h
\ a = g sin q = g .
s
p2 1 q qmg cos q
14. K.E = Q p is same K .E µ gs
in
2m m 1 1 m mg
\ Light body will have greater kinetic energy. Q s = ut + at 2 = at 2 h
2 2 q
15. F = Kl \ K= F B M C
l 1 gh 2
When it is cut into 2 equal halves, length of each half will = t [Q u = 0]
l 2 s
be . \ Spring constant will be doubled.
2
16. 1 eV = 1.6 ´ 10–19 joule. It is the energy required to transfer 2
electron (charge = 1.6 ´ 10–19 C) through a potential \ t=s Þ t µs
gh
difference of 1 volt.
17. Energy \ The stone falling through AB will take more time than
that falling through AC.
1 2 v2 – u2 = 2gh
kx
2
Q u = 0, \ v = 2 gh
Q Both the stones are falling through the same vertical
height h, therefore they will reach the bottom with the
same velocity.
–x x
Work, Energy and Power 153

4. Elastic collision is defined as that collision in which both P.E. = mg (h – x)


momentum and kinetic energy before and after collision are \ T.E. = K.E. + P.E. = mgx + mg(h – x) = mgh
conserved, If two masses m1 and m2 moving with velocities At C :
u1 and u2 collides, velocities after collision is given by
It touches the ground.
( m1 - m2 )u1 + 2m2 u2
v1 = \ P.E. = 0
( m1 + m2 )
If v is the final velocity, then v 2 = 2 gh
(m2 - m1 )u2 + 2 m1u1
and v2 =
(m1 + m2 ) 1 2 1
\ K.E. = mv = ´ m ´ 2 gh = mgh \
2 2
mu
(i) If m1 >> m2 and u2 = 0, v1 = 1 1 = u1 and v2 = 2u1 T.E. = mgh
m1 Total energy remains constant at every point in the path
\ The first body continues to move with the same of a freely falling body.
velocity and the second body acquires double the 7. Let A = Cross-sectional area of the pipe, v = velocity of
initial velocity of the first. flow of water, r = density of water.
(ii) If m1 >> m2 and u1 = 0 dm
2m2 u2 mu \ Mass of water flowing out/s = = Avr
v 1= » 0 and v 2 = - 1 2 = -u2 dt
m1 m1 To get n times water in same time,
\ After collision, the heavy body remains at rest and '
the lighter body rebounds with the same velocity. æ dm ö æ dm ö
5. Let the mass of each body be m. ç ÷ = nç ÷ = nAvr
è dt ø è dt ø
A 'V ' r = n Avr Þ V¢ = nm
v1 sin q1v1 m
dm
u=0 (a) F =v
q1 dt
m m v1 cos q1 + v2 cos q2
u q2
æ dm ö æ dm ö
v2 v 'ç ÷' nv ç n ÷
v2 sin q2 m m F' è dt ø è dt ø
\ = = \ F ' = n2F
F æ dm ö æ dm ö
Using momentum conservation principle, x component of vç ÷ vç ÷
è dt ø è dt ø
the equation is,
mu = mv1 cosq1 + mv2 cosq2 P ' F '´ v ' n2 F ´ nv
Þ u = v1cosq1 + v2cosq2 ...(1) (b) = = = n3 \ P ' = n3P
p F ´v F ´v
y component of the equation is
8. When dropped from same height, the two masses hit
0 = v1sinq1 – v2sinq2 ...(2)
ground with same velocity.
According to conservation of K.E.,
1 1 1 P2 m2
v= 2 gh \ = =n
mu 2 = mv12 + mv22 P1 m1
2 2 2
Þ u2 = v 2 1 + v 2 2 ...(3)
Squaring and adding (1) and (2), we have P2
9. K.E. = = constant.
u2 = v12 + v22 + 2v1v2 (cosq1cosq2- sinq1sinq2) 2m
Þ u2 = v12 + v22 + 2v1v2 cos (q1 + q2 ) P2 m2
Using (3), We get, 2v1v2 cos (q1 + q2) = 0 \ P = 2 m ´ K .E \ = = n
P2 m1
p
Þ cos (q1 + q2) = 0 Þ q1 + q2 = 10. As momentum is conserved in any collision, so
2
m1u1 + m2u2 = m1v1 + m2v2
\ The masses move at right angles after the collision. or m1(u1– v1) = m2(v2– u2) ...(i)
6. At A: Also, kinetic energy is conserved in an elastic collision,
K.E. = 0 (Since the body is at rest), m so
P.E. = mgh A
\ T.E. = K.E. + P.E. = mgh 1 1 1 1
m1 u12 + m 2 u22 = m1v12 + m2 v22
AT B: x 2 2 2 2

( ) ( )
Let v1 be the velocity of the body. h
\ Using the relation, v2 – u2 = 2as B or m1 u12 - v12 = m2 v22 - u22 ...(ii)
v12 – 0 =2gx or v12 = 2gx On dividing (ii) by (i), we get
h-x
1 2 1 u1 + v1 = v2 + u2
\ K.E. = mv1 = ´ 2mgx = mgx C
or v2 – v1 = u1 – u2
2 2
154 Physics
Hence, the coefficient of restitution for one dimensional 13. Einstein’s mass-energy equivalence: mass and energy are
elastic collision will be equivalent and related by the relation as
E = mc2
v2 - v1
e= = 1. where c, is the speed of light in vacuum is approximately
u1 - u2 3 ´ 108 m/s.
11. Measurement of work done when force and displacement Applications of mass-energy equivalence:
are inclined to each other at an angle q.
F sinq
(i) ( )
0
Annihilation of matter. When an electron -1 e and

F
F a positron ( e) come close to each other, they
0
1

q annihilate (destroy) each other forming two g-rays


(electromagnetic radiation) of total energy given by
F cosq
Einstein’s mass-energy relation.
s
(ii) Pair production. When a g-ray photon of energy 1.02
Fig. Work done, when force is inclined to displacement. MeV passes close to a massive nucleus, it
Work done = Component of force in the direction of materialises into a pair of particles-an electron and a
displacement ´ magnitude of displacement positron. Thus energy gets converted into matter.
W = F cos q ´ s (iii) Energy generation in the sun and stars. The energy
or W = Fs cos q generated in the sun and stars is due to the
r r conversion of mass into energy.
or W = F . S
14. Here, m1= m2 = m (say), Initially u1= v, u2 = – v
Thus work done is th e dot product of force and
As the collision is perfectly elastic, velocities after the
displacement vectors. Hence work is a scalar quantity.
collision will be
Work done by a force is zero if
m1 - m2 2m2
p p v1 = .u1 + .u2
(i) q= Þ i.e. W = Fs cos = 0 as in the m1 + m2 m1 + m2
2 2
gravitational force of earth does no work in the moon m-m 2m
in its orbit. = .v + ( -v ) = 0 – v = – v
m+m m+m
(ii) s = 0 i.e. applied force do not displace the object as
in the case of man pushing a brick wall. 2m1 m - m1
v2 = .u1 + 2 .u2
12. Kinetic energy by calculus method. Consider a body of m1 + m2 m1 + m2
r
mass m initially at rest.A force F applied on the body
r 2m m-m
= .v + . ( - v ) = v + 0 = v.
produces a displacement ds in its own direction (q = 0º). m+m m+m
The small work done is Thus the two bodies exchange their velocities after the
r uur
dW = F .ds = Fds cos 0º = Fds head-on elastic collision.
According to Newton’s second law of motion, 15. Region A : No, as K.E. will become negative.
Region B : Yes, total energy can be greater than PE for
dv non zero K.E.
F = ma = m
dt Region C : Yes, KE can be greater than total energy if its
PE is negative.
dv é ds ù Region D : Yes, as P.E. can be greater than K.E.
\ dW = Fds = m .ds = mv dv êQ = vú
dt ë dt û 16. (a) Ball A transfers its entire momentum to the ball on
The total work done to increase its velocity from 0 to v the table and does not rise at all.
is given by (b) v = 2 gh = 4.42 m/s
v v v 17. (a) Loss of P.E. = mgh
é v2 ù 1 2
W = ò dW = = 1 ´ 10–3 ´ 10 ´ 10–3 = 10J
ò mv dv = m ò v dv = m ê ú = mv
0 0 ëê ûú 0 2
2 (b) Gain in K.E.
This work done appears as the kinetic energy (K) of the 1 2 1
=mv = ´ 10 -3 ´ 2500 = 1.25J
body. 2 2
1 2 (c) No, because a part of PE is used up in doing work
\ K= mv against the viscous drag of air.
2
Work, Energy and Power 155

Long Answer Questions


F Fs
1. Let the force be a function of distance (F(x)) and it is
s =
–k -kxm2
x Area =
displacing the body from A to B.
2
(d) xm
C O B x

S R

D A
F(x)
(a) The spring force Fs is zero when the displacement x
from the equilibrium position is zero.
xA xB
(b) For the stretched spring x > 0 and Fs < 0
A PQ B (c) For the compressed spring x < 0 and Fs > 0.
Distance
(d) The plot of Fs versus x.
Let us divide the entire displacement AB into a large no. Let the spring be further displaced by a small amount dx
of infinitesimal small displacements. Consider one such against the restoring force.
small displacement PQ = dx and assume that over this small \ Work done against the restoring force
displacement the force remains constant. = dW = – Fx.dx = kx.dx.
\ Small amount of work done in moving the body from \ Total work done in producing displacement x from 0
P to Q = dW = Fdx = PS × PQ x
= Area of the strip PQRS. = W = ò dW = ò kxdx
\ Total work done in moving the body from A to B x =0

xB xB 1 2 1 2
Þ W = kx \ P.E. = kx
ò
= W = dw = ò Fdx = ò Area of PQRS 3.
2 2
Let m, V, rHe denote respectively the mass, volume and
xA xA
density of helium balloon and rair be density of air
= Area of ABCDA Volume V of balloon displaces volume V of air.
\ Work done by a variable force is calculated by the So, V (rair – rHe) g = ma (1)
area under the force– displacement graph. Integrating with respect to t,
2. When a spring is compressed or extended a restoring force V (rair – rHe) gt = mv
is developed in it due to its elasticity in a direction 1 2 1 V2
opposite to the applied force. Some work has to be done Þ
2
mv = m
2 m2
( rair - rHe )2 g 2t 2
against the restoring force, which is stored as potential
energy of the spring. 1 2
V ( r air - r He ) g 2t 2
2
= (2)
Let x ® compression or extension of the spring and 2m
F ® restoring force 1 2
If the balloon rises to a height h, from s = ut + at ,
F µ - x or F = – kx [k = spring constant]. 2
Fs = 0
x=0 1 2 1 V ( rair - rhe ) 2
(a) we get h = at = gt (3)
2 2 m
x=0 x From Eqs. (3) and (2),
Fs is negative
x is positive 1 2 é
2
(
mv = V ra - r He
ë ) g ùû
é 1
( ) 2ù
ê 2 m V ra - r He gt ú
ë û
(b)
= V (ra – rHe) gh
x x Rearranging the terms,
Fs Fs is positive
1 2
x is negative Þ mv + V r He gh = V rair hg
(c) 2
KEballoon + PEballoon = change in PE of air.
x x
So, as the baloon goes up, an equal volume of air comes
down, increase in PE and KE of the baloon is at the cost
x=0 of PE of air [which comes down]
156 Physics
4. According to work-energy theorem work done by a force m1u1 + m2u2 = m1 v1 + m2u1 – m2u2 + m2v 1
in displacing a body is equal to the change in its kinetic v1 (m1 + m2) = (m1 – m2)u1 + 2m2u2
energy.
Let m = mass of the body, u = initial velocity, F = force ( m1 - m 2 ) 2 m2 u2
Þ v1 = .u1 +
applied in the direction of motion ( m1 + m 2 ) ( m1 + m 2 )
ds = small displacement of motion
Small amount of work done by the force = dW 2m2u2 + ( m1 - m2 ) u1
uur uur =
m1 + m2
= F .ds = Fds cos0° = Fds = mads
dv é dv ù Putting the value of v1 in equation (3) we get
dW = m .ds êQ acceleration = a = ú
dt ë dt û
(m1 - m2 ) 2 m2 u2
ds v2 = u1 - u2 + .u1 +
= m.dv.v [Q =v] m1 + m2 ( m1 + m2 )
dt
\ Total work done by the force to change the velocity é m1 - m 2 ù é 2 m2 ù
of the body from u to v = u1 ê1 + ú + u2 ê - 1ú
ë m1 + m 2 û ë m1 + m 2 û
v
é v2 u2 ù é m1 + m2 + m1 - m2 ù é 2 m2 - m1 - m 2 ù
W = ò dw = m ò vdv = m ê 2 - 2 ú = u1 ê
m1 + m 2
ú + u2 ê
m1 + m 2
ú
u êë úû ë û ë û
1 1 2 m1u1 (m - m1 )u 2 2m1u1 + ( m 2 - m1 ) u 2
= mv 2 - mu 2 = Change in K.E. of the body. v2 = + 2 =
2 2 m1 + m2 m1 + m2 m1 + m 2
5. Two masses m1 and m2 are moving with velocities u1 and
u2 along the same direction. If u1 > u2, relative velocity Numerical Questions
of approach, before collision = u1 – u2. After collision
v2 > v1 . 1. Given : m = 2 kg, s = 40 m, a = 0.5 m/s2
F = ma = 2 × 0.5 = 1 N; W = FS cos q = 1 × 40 × cos 0° = 40 J
m1 u1 m2 u2 2. (a) The displacement over the round trip is zero
Before Collision \ work done = force × displacement is also zero.
m1 m2
m1 m2 During (b) Work done by the applied force
v1 v2
Collision = (mg sin q + f )s = (mg sin q + mmg cos q)s
After Collision R
Relative vcelocity of separation = v2 – v1.
According to law of conservation of linear momentum, 10 f
q q 5m
Total linear momentum before collision = Total linear
sin gcos
momentum after collision mg m
m1u1+ m2u2 = m1v1 + m2v 2 q
Þ m2(v2 – u2) = m1 (u1 – v1 ) ... (1)
Total K.E. is also conseverd,
mg 102 - 52
1 1 1 1 æ 5 5 3ö
m1u12 + m2 u22 = m1v12 + m1v22 = 0.3 × 9.8 × çç + 0.15 ´ ÷10
2 2 2 2 è 10 10 ÷ø
1 2 2 1 2 2
m2 (v2 - u2 ) = m1 (u1 - v1 ) = 0.3 × 9.8 (5 + 0.15 × 1.73 × 5)
2 2
Þ m2 (v22 - u22 ) = m1 (u12 - v12 ) ... (2) = 0.3 × 9.8 (5 + 0.2595 × 5)
= 0.3 × 9.8 (5 + 1.2975) = 0.3 × 9.8 × 6.2975 = 18.5 J.
m2 (v22 - u22 ) m1 (u12 - v12 ) (c) Work done by the frictional force, f × s
Dividing (2) by (1), =
m2 ( v 2 - u 2 ) m1 (u1 - v1 )
= – mmg cos q´ s
(v2 + u2 )(v2 - u 2 ) (u1 + v1 )(u1 - v1 )
Þ =
( v2 - u 2 ) (u1 - v1 ) 5 3
= – 0.15 × 0.3 × 9.8 × ´ 20 = –7.6J
or v2 + u2 = v1 + u1 or v2 – v1 = u1 – u2 10
(d) K.E of the body at the end of the trip = work done by
\ Relative velocity of seperation = Relative velocity of
the net force
approach. uur ur
v2 = u1 – u2 + v 1 ... (3) 3. Displacement = r2 - r1 = iˆ - ˆj ;
uur uur
\ m1u1 + m2u2 = m1v 1 + m2 (u1 – u2 + v1) Work done, W = F .S = (5iˆ + 5 ˆj).(iˆ - ˆj) = 5 - 5 = 0
Work, Energy and Power 157

x2 12. Given : m = 3.0 ´10–5 kg r = 10–3 kg/m2 v = 9 m/s


5 A = 1m2 h = 100 cm
4. Work done, W = ò f .dx = ò0 (7 - 2 x + 3x2 ).dx Þ n = 1m3 M = r v = 10–3 kg,
x1
1 1 2
5 E = Mv2 = ´103 ´ ( 9 ) = 4.05 ´ 104 J.
é 2x2 x3 ù 2 2
= ê7 x - + 3. ú
êë 2 3 úû 1 2 1
0 13. Kinetic energy, KE = mv @ ´ 5 ´104 ´102 = 2.5 ´ 105 J
2 2
= 7(5 – 0) – (52 – 02) + (53 – 03) = 135J 10% of this is stored in the spring.
1 2
2 kx = 2.5 ´104 ; x = 1 m
5. Given : x = t3/3 Þ v = dx = 3t = t 2 2
dt 3 k = 5 ´ 104 N/m.
dv 14. Pressure = 130 mm of Hg = 13 cm of Hg
Þ a= = 2t
dt = hrg = 13 ´ 1.03 ´ 980 dyne cm–2
\ F = ma = 2 × 2t = 4t
Volume = 4000cm3 ; time, t = 1 min = 60s
2 2
dx W pressure× volume
W = ò fdx = ò F .dt = ò F.v.dt Power, P = =
dt t t
0 0
13 ´1.03 ´ 980 ´ 4000
= = 874813.33 erg s–1
2 2 2 ù 60
t4
= ò 4t ´ t 2 dt = ò 4t 3 dt = 4 = 2 4 = 16 J ú
4 ú = 874813.33 ´ 10–7 Js–1 or W
0 0 0 úû
874813.33 ´ 10-7
2 = hp = 1.17 ´ 104 hp.
p 1000 ´1000 746
6. Kinetic energy, K.E. = = = 104 J
2m 2 ´ 50 15. (a) Wg = mg sinq d = 1 ´ 10 ´ 0.5 ´ 10 = 50 J
(b) Wr = m mg cosq d = 0.1 ´10 ´ 0.866 ´ 10 = 8.66 J
p2 p2 p 2
7. Kinetic energy, K.E. = = const. Þ 1 = 2 (c) DU = mgh = 1 ´ 10 ´ 5 = 50 J
2m 2 m1 2 m2 (d) a = {F – (mg sinq + m mg cosq )}
p1 m1 1 1 = [10 –5.87] = 4.13 m/s2
\ = = =
p2 m2 4 2 v = u + at or v2 = u2 + 2ad
8. –2
Given : m = 10 g = 10 kg., u = 800 m/s, s = 1 m, v = 100 m/s, 1 1
DK = mv 2 - mu 2 = mad = 41.3J
F=? 2 2
Q work done = change in K.E. (e) W = F d = 100 J

1 2 1 2 HOTS/EXEMPLAR QUESTIONS
f×s= mu – mv
2 2 Very Short Answer Questions

1 r
Þ F×1= × 10–2 (8002 – 1002) Þ F = 3150 N. 1. The force F is not conservative. Work done during the
2 round trip in the smaller loop is not zero.
9. Given : m = 1000 kg, v = 20 m/s, 2. No, unlike energy, work is not something an object has.
Tension = Force applied = 2000 N Work is something an object does to some other object. An
object can do work only if it has energy. Or, stated another
Q P = F × v = 2000 × 20 = 40000 W = 40 kW way, an object spends energy when it does work on
10. Given : h = 50 m, m = 1.8 × 105 kg, t = 1 h = 3600 s something else.
energy mgh 3. No, because resistive force of air also acts on the body
Total power generated = = = 12250 W
time 2t which is a non-conservative force. So the gain in KE would
be smaller than the loss in PE.
12250
No. of 100 W bulbs can be lit = < 122.5 ; 122 4. To prevent elevator from falling freely under gravity.
100
11. Given : m1 = 2 kg, u1 = u, m2 = ?, u2 = 0, v1 = u/2 Short Answer Questions

( m1 - m2 )u1 + 2m2 u 2 u (2 - m2 )u + 2m2 ´ 0 1. Momentum is a vector quantity having both magnitude and
v1 = , = direction whereas kinetic energy is a scalar quantity having
m1 + m2 2 2 + m2
only magnitude. Momentum that is directional is capable of
2 being cancelled entirely.
Þ m2 = = 0.67 kg .
3
158 Physics
The vector sum of the momenta of a flock of birds in flight
\ p2 > p12 + p22
can be zero because of birds flying in different directions in
the flock. Thus p, p1 and p2 are related as shown in the figure.
Each flying bird has some kinetic energy and the algebraic q is acute (less than 90°) ( p2 = p12 + p22 would give
addition of the kinetic energies of all birds in the flock cannot q = 90°)
be zero. 5. m = 3.0 × 10–5 kg r = 10–3 kg/m2 v = 9 m/s
2. If a crate is located on the bed of a truck, and the truck A = 1m2 h = 100cm Þ n = 1 m3
accelerates to the east, the static friction force exerted on
the crate by the truck acts to the east to give the crate the M = rv = 10-3 kg ,
same acceleration as the truck (assuming the crate doesn’t 1 1 2
slip). Another example is a car that accelerates because of E= Mv 2 = ´ 103 ´ ( 9 ) = 4.05 ´10 4 J
2 2
the friction forces exerted on the car’s tires by the road.
These forces act in the direction of the car’s motion and the Long Answer Questions
sum of these forces causes an increase in the car’s kinetic
1. (a) Let common velocity (Puff + Block) = v1
energy.
3. \ (M + m) v1 = mv or 2.5 v1 = 0.5 v1 = 0.5v
A v1 = (0.5/2.5) v
v
1 1
v Further ( M + m ) v12 = kx 2
v 2 2
1 1
(2.5) v12 = ´ (250)(0.1) 2
B
C 2 or
(mgh + 1/2 mv ) initial 2 2
2
h = (1/2 mv¢ )final
250
v12 = ´ (0.1) 2 or v1 = 1m/sec
2.5
2.5
v¢ v¢ v¢ \v = v1 = 5m / sec
0.5
The speed of impact for each ball is the same. With respect
to the ground below, the initial kinetic + potential energy of (b) When a road goes straight up, the slope is large and
each ball is the same. This amount of energy becomes the hence the frictional force (= m mg cos q) is small.
kinetic energy at impact. So for equal masses, equal kinetic Now due to insufficient friction, the wheels of the
energies mean the same speed. vehicle slip on the road. Secondly, for a large slope
4. (a) For head on collission: more power will have to be used (because it would
Conservation of momentum Þ 2mv0 = mv1 + mv2 take less time).
Or 2v0 = v1 + v2
2. (i) Although both balls have the same speed on the level
v -v parts of the tracks, the speeds along the curved parts
and 2 e = 2 1 Þ v2 = v1 + 2v0 e
2v0 differ. The speed of the ball everywhere along curve B
\ 2v1 = 2v 0 - 2ev0 \ v1 = v0 (1 - e ) is greater than the initial speed, whereas everywhere
Since e < 1 Þ v1 has the same sign as v0, therefore the along curve A it is less. So the ball on Track B finishes
ball moves on after collission. first.
(b) Conservation of momentum Þ p = p1 + p2 (ii) Does the gain in speed at B’s bottom equal the loss at
A¢ s top? No; Speed isn’t conserved : energy is. The
p2 p2 p2 loss in kinetic energy at the top of A will be equal to
But KE is lost Þ > 2 + 2
2m 2m 2m the gain in kinetic energy at the bottom of B if there is
p1
enough energy to begin with.
p
é1 2ù
There isn’t because the initial KE ê m2 ú is less than
Before After ë2 û

p2 1
the gain in K.E at the bottom of B [ m (32 – 22). At 2
2
m/s, the ball will not even make it to the top of A’s
p p2 curve. At the top of the curve on track A the speed of
the ball will be less than 1 m/s.
q
p1
Work, Energy and Power 159

1 1 6. When momentum increases by 50%, velocity also increases


3. ( KE )t + Dt = ( M - Dm )( v + Dv)2 + Dm ( v - u )
2
by 50%
2 rocket 2 gas
50 3
1 1 \ New velocity = v + v= v
2 2 100 2
= Mv + MvDv - Dmvu + Dmu
2 2 2
1 1 æ3 ö 9 æ1 2ö
1 \ K.E. = mv 2 = m ç v ÷ = ´ ç mv ÷
( KE )t = Mv 2 2 2 è2 ø 4 è2 ø
2
9
1 Þ It becomes times
( KE )t +Dt - ( KE )t = ( MDv - Dmu ) v + Dmu 2 4
2
æ9 ö
1
= Dmu 2 = W ç 4 - 1÷ T
2 % increase in K.E. = è ø ´100 , where T is initial K.E.
T
(By Work – Energy theorem)
5
æ Mdv æ dm ö ö = ´100 < 125 %
Since ç =ç ÷ ( u ) ÷ Þ ( MDv - Dmu ) = 0 4
è dt è dt ø ø 7. Work done in passing through the wall = loss in K.E. of
4. Let m, V, rHe denote respectively the mass, volume and
1
density of helium balloon and rair be density of air bullet = m (v12 – v22)
Volume V of baloon displaces volume V of air. 2
So, V ( rair - rHe ) g = ma ...(1) 1
= (50 × 10–3) (4002 – 1002) = 3.75 × 103 J.
Integrating with respect to t, 2
8. Drop in potential energy = Energy stored in spring
V ( rair - rHe ) gt = mv
1 2
or, mg ( h + x ) = kx h m
1 1 V2 2
mv 2 = m 2 ( rair - rHe ) g 2 t 2
2
Þ or, 2 mgh + 2 mgx = kx2
2 2 m
or, kx2 – 2 mgx – 2 mgh = 0
1 2
V ( rair - rHe ) g 2 t 2
2
= ...(2) k
2m 2mg ± 4m 2 g 2 - 4k .2mgh
x=
1 2 2k
If the balloon rises to a height h, from s = ut + at .
2
mg mg 2kh
Þ x= ± 1-
1 1 V (rair - rhe ) 2 k k mg
We get h = at 2 = gt ...(3)
2 2 m 9. (a) Point Q is at a height R above the ground. Thus, the
From Eqs. (3) and (2). difference in height between points P and Q is 4R.
Hence, the difference in gravitational potential energy
1 é 1 ù
mv 2 = éë V ( ra - rHe ) g ùû ê V (ra - rHe ) gt 2 ú of the block between these points = 4 mgR.
2 ë 2m û Since the block starts from rest at P its kinetic energy
= V ( ra - rHe ) gh at Q is equal to its change in potential energy. By the
Rearranging the terms, conservation of energy.
1 2
1 \ mv = 4mgR v2 = 8gR
Þ mv2 + VrHe gh = Vrair hg 2
2 AT Q, the only forces acting on the block are its weight
Þ KEbaloon + PEbaloon = change in PE of air. mg acting downward and the force N of the track on
So, as the balloon goes up, an equal volume of air comes block acting in radial direction. Since the block is
down, increase in PE and KE of the balloon is at the cost of moving in a circular path, the normal reaction provides
PE of air (which comes down). the centripetal force for circular motion.
CHAPTER TEST mv 2 m ´ 8 gR
N= = = 8 mg
1. No, work done in moving a body doesn’t depend on the R R
speed of the body as work done = Force × displacement is The loop must exert a force on the block equal to eight times
independent of the path. the block’s weight.
2. Refer to theory (b) For the block to exert a force equal to its weight against
3. Since centripetal force and displacement are perpendicular, the track at the top of the loop,
work done is zero. mv '2
4. Refer to theory = 2mg or v 2 = 2 gR
R
5. Refer to theory
160 Physics
1 Characteristics of inelastic collisions :
\ mgh = mv '2 (i) The momentum is conserved.
2
(ii) Total energy is conserved.
v '2 2 gR (iii) The kinetic energy is not conserved.
h= = =R
2g 2g (iv) Some or all of the forces involved ar e non-
The block must be released at a height 3R above the conservative.
bottom of the loop. (v) A part of the mechanical energy is converted into
10. Elastic collision. If there is no loss of kinetic energy during heat, light, sound, etc.
a collision, it is called an elastic collision. Examples. Collision between two vehicles, collision
Characteristics of elastic collisions : between a ball and floor.
(i) The momentum is conserved. Perfectly inelastic collision. If two bodies stick together
(ii) Total energy is conserved. after the collision and_ move as a single body with a
(iii) The kinetic energy is conserved. common velocity, then the collision is said to be
(iv) Forces involved during the collision are conservative. completely inelastic collision.
(v) The mechanical energy is not converted into heat, Characteristics of perfectly inelastic collision
light, sound, etc.
(i) Momentum is conserved
Examples. Collision between subatomic particles, collision
(ii) The loss of kinetic energy is maximum.
between glass balls, etc.
Examples. Mud thrown on a wall and sticking to it, a man
Inelastic collision. If there is a loss of kinetic energy during
jumping into a moving trolley, a bullet fired into a wooden
a collision, it is called an inelastic collision.
block and remaining embedded in it, etc.

¿¿¿
System of Particles
7 and Rotational
C ha p t e r Motion

RIGID BODY
Rigid body is defined as a collection of large number of point masses (or particles) in which the separation between any two
constituent particles is fixed. There will be no change in the shape or volume of rigid bodies under an external force. No real
body is truly rigid, since it deforms under the influence of forces.
Different types of motion for a rigid body: A rigid body can have
(i) translational motion wherein all the particles of body have same velocity at the given instant e.g., sliding motion of a block
down an inclined plane.
(ii) rotational motion wherein every paticle of the rigid body moves in a circle that lies in a plane perpendicular its centre on
the axis e.g., motion of a ceiling fan.
(iii) combination of translational and rotational motion e.g., rolling motion of a cylinder down an inclined plane.
CENTRE OF MASS
It is an imaginary point at which the whole mass of a body is supposed to be concentrated. A single force of appropriate magnitude
applied at centre of mass of a system of particles can produce the same effect as that of many forces acting on individual point
masses, constituting the system.
Characteristics of Centre of Mass
(i) It need not hold mass physically, e.g. for a hollow sphere, centre of mass is at the geometrical centre of the sphere although
there is no mass present physically at the centre.
(ii) Location of centre of mass depends on the distribution of masses and their individual location. For regular geometrical shaped
bodies, having uniform distribution of mass, the centre of mass is located at their centres.
(iii) When no external force acts on a body, centre of mass has constant velocity and constant angular momentum. Acceleration
is zero.
Position of Centre of Mass (C.M.) of a System of Particles
Y
(i) Position vector of C.M. of a two particles system is given by M2
ur ur C.M.
uur M1 r1 + M 2 r 2
RCM = r2 M1
M1 + M 2
ur ur r1
uur r1 + r 2
If M1 = M2 then RCM = X
2
\ Centre of mass of two particles of equal masses lies at the mid-point of the R CM
line joining the two masses.
If the masses are unequal, C.M. lies towards the heavier mass.
ur ur Z
If C.M. is the origin then M1 r1 + M2 r 2 = 0
(ii) For a n particles system, the position vector of the C.M. is given by
n ur
uur
ur ur ur
M1 r1 + M 2 r 2 + .................. + M n r n
å Mi r i
i =1
RCM = = n
M1 + M 2 + ........... + M n
å Mi
i =1
162 Physics
n ur
If C.M. is the origin, then å Mi r i = 0.
i =1
If the system has continuous distribution of mass, treating the mass element dm at position rr as a point mass and replacing
r 1 r 1 1 1
summation by integration rCM =
M ò rdm so that XCM =
M ò Xdm , YCM = ò Ydm and ZCM =
M Mò
Zdm
Position of centre of mass of regular shaped objects:
Name of the object Position of C.M.
Circular ring At the centre of the ring
Circular disc At the centre of the disc
Hollow sphere/solid sphere At the centre of the sphere
Plane lamina in the form of a triangle At the centroid of the triangle
Plane lamina in the shape of a
At the point of intersection of its two diagonals
parallelogram/square
Hollow/solid cylinder At the mid point of the axis of the cylinder
Cone At a height 3h /4 from the vertex of the cone
[h = height of the cone]

MOTION OF CENTRE OF MASS


The centre of masses of a system of particles moves as if all the masses of the system was concentrated at the centre of mass
and all the external forces were applied at that point.
As for a system of particles, position of centre of mass is
r r r
r m r + m 2 r2 + m3 r3 + ....
rCM = 1 1
m1 + m 2 + m3 + ....
r r r
dr dr dr
m1 1 + m 2 2 + m3 3 + ....
d r dt dt dt
So, (rCM ) =
dt m1 + m 2 + m3 + ....
Velocity of CM
r r n r
r m v + m2 v2 + .... 1 r é dr r ù
vCM = 1 1
m1 + m2 + ......
=
M
å mi vi Q
êë dt = v úû
i =1

r d r
Similarly, acceleration of CM a CM = vCM
dt

r r n r
r m a + m2 a2 + .... 1 r é r dv ù
aCM = 1 1
m1 + m2 + ......
=
M
å mi a1 êëQ a =
dt úû
i =1

Illustration 1 :
Calculate the position of the centre of mass of a system consisting of two particles of masses m1 and m2 separated by a
distance L.
Sol. Treating the line joining the two particles as x axis
m1 ´ 0 + m 2 ´ L (0,0) (L,0)
x=
m1 + m 2
yCM = 0, ZCM = 0 L
Therefore, the position of the system of particles
m2 L
x CM =
m1 + m 2
System of Particles and Rotational Motion 163

Illustration 2 :
Find the centre of mass of a uniform disc of radius 'a' from which a circular a
section of radius b has been removed. The centre of the hole is at a distance m2
O bm
c from the centre of the disc. 1
X
O2 O1
x1

x2 c
Sol. Let the circular disc of radius ‘a’ is made up of the circular section of radius b and
remainder. Further let the line of symmetry joining the centres O and O1 be the x-axis
with O as origin.
The centre of mass of the disc of radius ‘a’ will be given by
m1x1 + m2 x 2
XCM = ........ (i)
m1 + m 2
while YCM and ZCM will be zero (as for all points on x-axis y and z = 0). If s is the density of the material of disc.
m1 = pb2s and x1 = c
m2 = p (a2 – b2) s and x2 = ?
M = m1 + m2 = pa2s and XCM = 0
So substituting these values in equation (i)

pb2 sc + p (a 2 - b2 ) sx 2
0= ....... (ii)
pa 2 s
or x2 = – cb2/(a2 – b2)
i.e., centre of mass of the remainder (say O2) is at a distance cb2/(a2 – b2) to the left of O on the line joining the centres O and O1.
VECTOR PRODUCT OF TWO VECTORS
r r r
A vector product of two vectors A and B is a vector C such that is defined as
ur uur uur
C = A × B = AB
uur uur ur uur uur
sin q n̂ where n̂ is the unit vector along the direction of A × B ´ C . A × B is C =A×B
uur uur uur ur uur ur ur uur
perpendicular to both A and B . e.g. angular momentum L = r × p ; Torque t = r × F .
ur ur
If we take a right handed screw with its head lying in the plane of A and B and the screw
ur uv
perpendicular to this plane, and if we turn the head in the direction from A to B , then
ur
the tip of the screw advances in the direction of C . This is right handed screw rule.
Rule for defining the direction of the vector product. B

A
iˆ ˆj kˆ
ur ur
A ´ B = Ax Ay Az
Bx By Bz

Properties Of Vector Product


r r r r
Vector product is not commutative i.e., A ´ B ¹ B ´ A
Vector product is distributive w.r.t. vector addition
r r r r r r r
i.e., A ´ ( B + C ) = A ´ B + A ´ C
Since magnitude of A × A = A2 sin 0° = 0
r r
A ´ A = 0 null vector
iˆ ´ ˆj = kˆ
164 Physics
ANGULAR VELOCITY AND ITS RELATION WITH LINEAR VELOCITY
dq
Angular velocity w = . It is a vector quantity. Angular velocity (w) related with linear velocity (v) as
dt
v = rw where r is the radius of circular path
v=w×r
Linear velocity in translational motion is analogous to angular velocity in rotational motion.
Angular acceleration
dw
It is the time rate of change of angular velocity i.e., a =
dt
TORQUE AND ANGULAR MOMENTUM
Torque or couple is the moment of force. It is the cross product of the force with the perpendicular distance between the axis
of rotation and the point of application of force with the force.
v uuv uuv
Torque = t = r ´ F = rF sin q ; Its S.I. unit is N m
The rotational analogue of force is torque or couple. Couple or Torque is a pair of equal and oposite forces with different lines of action.
A couple produces rotation without translation.
Angular momentum is the moment of linear momentum. It is the product of the linear momentum and the perpendicular distance
of the mass from the axis of rotation.
r r r r
Angular momentum, L = mvr = r ´ p = r P sin q, where r = position relative to origin p = linear momentum at position.
Its S.I unit is kg m2/s
uur
r ur dL
Relation between torque (t) and angular momentum ( L ), t =
dt
Principle of Conservation of Angular Momentum
If the external torque acting on a system of particles is zero, then the total angular momentum of the system is conserved.
uuv uuv uuv
If t = 0 then L1 + L2 + ..... + Ln = constant
Q L = Iw = constant, where I is moment of inertia
\ I1w1 = I2w2
EQUILIBRIUM OF A RIGID BODY
A rigid body is said to be in mechanical equilibrium, if both its linear momentum and angular momentum are not changing with
time, or equivalently, the body has neither linear acceleration nor angular acceleration. This means
(1) The total force, i.e. the vector sum of the forces, on the rigid body is zero.
n
F1 + F2 + .... + Fn = å Ft = 0 ...(i)
t= 1
If the total force on the body is zero, then the total linear momentum of the body does not change with time. Eq. (i) gives
the condition for the translational equilibrium of the body.
(2) The total torque, i.e. the vector sum of the torques on the rigid body is zero,
n
t 1 + t 2 + ... + t n = å tt = 0 ...(ii)
t =1
If the total torque on the rigid body is zero, the total angular momentum of the body does not change with time. Eq. (ii)
gives the condition for the rotational equilibrium of the body.
Principle of Moments for a Lever
According to this principle,
Load × load arm = effort × effort arm
Load Effort arm
Mechanical advantage of lever, M. A. = Effort = Load arm

Centre of Gravity: The centre of gravity of an extended body is that point where the total gravitational torque on the body is
zero. In unifom gravity or gravity free space the centre of gravity of the body coincides with the centre of mass.
System of Particles and Rotational Motion 165

Illustration 3 :
A car of mass 900 kg is travelling around a circular path of radius 300 m with a steady speed of 72 km/h. Calculate
its angular momentum.
72 ´ 1000
Sol. Here, mass, m = 900 kg, v = 72 km/h = = 20m/s, r = 300m
3600
Angular momentum L = mvr = 900 × 20 × 300 = 5.4 × 106 kgm2/s
Illustration 4 :
A force of (2i + 3j + 4k) N acts at point (2, 3, 1). What is torque about origin?
Sol. t = r ´ F = (2i + 3j + k) ´ (2i + 3j + 4k) = 6 k – 8j – 6k + 12i + 2j – 3i = (9i – 6j) Nm.

Illustration 5 :
A point mass 2 kg is moving with a velocity (2i + 3j) m/s. This is at point P (2, 3, 1) m. What is the angular
momentum about point O having coordinate (1, 2, 3) m?
Sol. L = r ´ p = i+j –2k
r = OP = (2 –1)i + (3–2)j + (1–3)k
\ L = (i+j–2k) ´ (4i + 6j) = 12i – 8j + 2k kg m2/s.
MOMENT OF INERTIA
It is defined as the sum of the product of the constituent masses and the square of their perpendicular distances from the axis
of rotation.
For an n-particles system having mass points m1, m2, m3, ..............., mn at perpendicular distances r1, r2, ............, rn
n
2 2
Moment of inertia, I = m1r1 + m2r2 + ............ + mnrn = 2
å mi ri2
i =1
Its S.I. unit is kgm2. It is a tensor quantity. Moment of inertia is the rotational analogue of mass in rotational motion.
Radius of Gyration
It is the root mean square of the perpendicular distances of the constituent masses. It is the perpendicular distance of the point
where the whole mass is concentrated form the axis of rotation.
n
å ri2
r12 + r22 + .................. + rn2 i =1
Radius of gyration k = =
n n
Moment of Inertia I = Mk2

THEOREMS OF PERPENDICULAR AND PARALLEL AXES z


Perpendicular Axes Theorem
If Ix and Iy are the moments of inertia about two perpendicular axes of any body,
the moment of inertia I about a 3rd perpendicular axis (z) is the sum of two x
moments of inertia, where all 3 axes are meeting at one point and x and y axes
are in the plane of the body and z-axis is perpendicular to the plane of the body.
i.e., Iz = Ix + Iy y
A C
Parallel Axes Theorem
The moment of inertia (I) about an axis parallel to the axis through C.M., is the
sum of the moment of inertia about the axis through the C.M. and the product
of the total mass (M) and the square of the perpendicular distance (r) between O h
the two axes concerned. (C.M.)
ICD = IAB + Mr2, where AB || CD.

B D
166 Physics
Moment of inertia of some regular shaped objects
Name of the object Axis Moment of inertia
(a) Rod (i) about an axis passing through M l 2/12
its C.M. and ^ to its length
(ii) about an axis passing through M l 2/3
one edge of the rod
(b) Ring (i) about an axis passing through MR2
C.M & ^ to its plane
(ii) about any diameter MR2/2
3
(iii) about a tangent in the plane of the ring MR2
2
(iv) about a tangent ^ to the plane of the ring 2MR2
(c) Disc (i) about an axis passing through C.M. and MR2/2
^ to its plane
(ii) about its diameter MR2/4
(d) Solid cylinder about its axis MR2/2
(e) Hollow cylinder about its axis MR2
2
(f) Hollow sphere about its diameter MR2
3
2
(g) Solid sphere about its diameter MR2
5
Illustration 6 :
Three point masses M1, M2, M3 are placed at the vertices of an equilateral triangle of length 'l'. What is the M.I. of
the system about an axis along the median of the triangle passing through M1?
P
3
M1
Sol. M.I of the system about PM = å M i ri2
i=1
l l
2 2
ælö æ lö l2
= M1 × 0 + M2 × ç ÷ + M 3 ´ ç ÷ = (M 2 + M 3 )
è 2ø è 2ø 4 M2 M3
l /2 l /2
Q R
Illustration 7 : M

Calculate the M.I of a circular disc about a transverse axis through the centre of the disc. Given, diameter of the disc
is 20 cm, thickness = 5 cm and density of the material of the disc = 7 g/cm3.
Sol. Given R = 20/2 = 10 cm, x = 5 cm, r = 7g/cm3
22
Mass of the disc = M = pR2xr = ´ 10 2 ´ 5 ´ 7 = 11000 g
7
Moment of inertia of the disc.
MR 2 1 2
I = = ´11000 ´ 10 = 5.5 × 105 gcm2
2 2
Illustration 8 :
A rod of mass M and length L is bent from centre at 90º. What is the moment of inertia for an axis ^ and passing
from centre?
Sol. Find MOI for individual rods and add them.
The axis of each rod for end becomes
2 2
M æ Lö M æ Lö
çè ÷ø ç ÷
= 2 2 2 è 2ø ML2
+ = .
3 3 12
ML2
If the rod was bent at angle q then also the result remains .
12
System of Particles and Rotational Motion 167

DYNAMICS OF ROTATIONAL MOTION ABOUT A FIXED AXIS


Angle traced per unit time about a fixed point or axis is called angular velocity.
dq
In rotational motion, angular velocity w = ,
dt
d w d 2q
Angular acceleration a = = 2
dt dt
Work done by torque in rotational motion W = tq
1 2
K.E. of rotation = Iw
2
Power P = t w
ur uur
Relation between torque (t) and angular acceleration (a), t = I a
uur uur
Relation between angular momentum (L) and angular velocity (w), L = I w
ur ur uur
Angular velocity of a rigid body about its axis of rotation is also expressed in terms of cross product as v = r × w , where,
ur ur
v ® linear velocity, r ® radius vector of the particle from the axis of rotation.
1
w 2 = w1 + at ; q = w1t + at 2 ; w 2 2 - w12 = 2aq
2
Here, w 2 and w 1 are final and initial angular velocities, q = distance, a = angular acceleration

ROLLING MOTION
When a body has both translational and rotational motion, it is said to be rolling.
1 2 1 2
K.E. of a rolling body = K.E. rotational + K.E. translational = mv + Iw
2 2
The velocity of a rolling body on an inclined plane of angle of inclination q (without slipping) is given by,

2 gh
v= where k ® radius of gyration, h ® height of inclined plane, v ® velocity on reaching the ground r ® radius
k2
1+
r2
of the rolling body.
g sin q
Acceleration a =
1+ k2 / r2
Comparison of linear motion and rotational motion
Linear Motion Rotational Motion
(i) Internal mass (m) (i) Moment of inertia (I)
(ii) Displacement (Dr) (ii) Angular displacement (Dq)
r dq
(iii) Linear velocity vr = dr (iii) Angular velocity w =
dt dt
r r
r dv d 2 r d w d 2q
(iv) Linear acceleration a = = 2 (iv) Angular acceleration a = =
dt dt dt dt 2
r r r r r r
(v) Linear momentum p = mv (v) Angular momentum L = r ´ p = I w
r r r r
r r dp mdv r r r r dL dw
(vi) Force F = ma = = (vi) Torque t = r ´ F = I a = =I
dt dt dt dt
r r r
(vii) Impulse = F .Dt = Dp (vii) Angular impulse = rt.Dt = DL
r 1 2 1 2
(viii) Kinetic energy E = mv (viii) Rotational kinetic energy Erot = I w
2 2
168 Physics
r r r r
(ix) Work W = F .D r (ix) Work W = t.D q
r r r r
r t×q r r
(x) Power P = dW = F × Dr = F × vr (x) Power P = = t×w
dt Dt Dt

(xi) Relation between p and E, p = 2 m E (xi) Relation between Erot I and L, L = 2 IErot
(xii) Equations of motion (xii) Equations of rotational motion
(a) v = u + at (a) w2 = w1 + at

1 2 1
(b) s = ut + at (b) q = w1t + at 2
2 2
(c) v2 = u2 + 2as (c) w22 = w12 + 2 a q
1 1
(d) Snth = u + a (2n–1) (d) Snth = w1 + a (2n–1)
2 2
Illustration 9 :
The motor of an engine is rotating about its axis with an angular velocity of 120 rpm. It comes to rest in 10s, after being
switched off. Assuming constant deceleration, calculate the number of revolutions made by it before coming to rest.
120 ´ 2 p
Sol. Here, w1 = 120 rpm = = 4 p rad/s; w2 = 0, t = 10s, q = ?
60
w 2 - w1 4p 2p
w2 = w1 + at Þ a = = - = - rad/s2
t 10 5
w 2 2 - w12 0 - 16 p 2
w22 = w12 + 2aq \ q= = = 20p rad.
2a 2 ´ ( -2 p / 5)
q 20 p
\ No. of revolutions made = = = 10
2p 2p
Illustration 10 :
A fly wheel rotating at the rate of 120 r.p.m slows down at a constant rate of 2 rad/s2. What time is required to stop the
fly wheel and how many rotations does it make in this process?
120 ´ 2 p
Sol. Here, w1 = 120 rpm = = 4p rad/s.
60
a = – 2 rad/s2, w2 = 0, t = ? q = ?
w2 - w1 0 - 4p
w2 = w1 + at Þ t = = = 2p sec.
a -2
1 2 1
Q q = w1t + a t = 4p.2p + (-2).(2p) 2 = 4p2 rad.
2 2
4 p2
Q No. of rotations made = = 2p ; 6 .
2p
Illustration 11 :
A solid cylinder of mass 2 kg and radius 0.2m is rotating with angular velocity 3 rad/sec. A particle
of mass 0.5 kg moving with velocity 5 m/s strike at the circumference of the cylinder. and stick on
it. 0.2m
Then calculate :
(a) Angular velocity after sticking the particle.
(b) Loss in kinetic energy due to collision.
Sol. (a) By law of conservation of angular momentum
Iw + mvR = (I + mR 2 ) w¢
System of Particles and Rotational Motion 169

1
Iw + mvR MR 2 w + mvR
Þ w¢ = Þ w¢ = 2
I + mR 2 1
MR 2 + mR 2
2

1
´ 2 ´ (0.2)2 ´ 3 + 0.5 ´ 5 ´ 0.2
2
1 = 10.3 rad/s
´ 2 ´ (0.2) 2 + 0.5 ´ (0.2) 2
2
1 2 1
(b) Now, (K.E.)i = Iw + mv 2
2 2

1 1 1
= ´ ´ 2 ´ (0.2)2 ´ (3)3 + ´ (0.5) ´ (5) 2 = 6.43 J
2 2 2

1 1
(K.E.)f = (I + mR 2 )w¢ 2
2 2

1 æ1 ö
= ´ ç ´ 2 ´ (0.2)2 + (0.5) ´ (0.2)2 ÷ (10.3)2 = 3.18 J
2 è2 ø
Þ Eloss = KEi – KEf = 6.43 – 3.18 = 3.25 J
170 Physics
System of Particles and Rotational Motion 171

Textbook Exercises
7.1 Give the location of the centre of mass of a B
(i) sphere, (ii) cylinder, (iii) ring, and (iv) cube, each of C
uniform mass density. Does the centre of mass of a body
necessarily lie inside the body? b
Sol. In all the four cases, as the mass density is uniform, centre q
of mass is located at their respective geometrical centres. O N a A
No, it is not necessary that the centre of mass of a body
In D OBN,
should lie on the body. For example, in case of a circular
ring, centre of mass is at the centre of the ring, where there BN BN
sin q = = \ BN = b sin q
is no mass. OB b
7.2 In the HCl molecule, the separation between ur ur 2(OA ´ BN )
the nuclei of two atoms is about 1.27Å | a ´ b | = ab sin q = OA × BN =
2
(1Å = 10–10 m). Find the approximate location of the C.M.
D
= 2 (area of OAB)
of the molecule, given that a chlorine atom is about 35.5
times as massive as a hydrogen atom and nearly all the 1 ur ur
\ Area of D OAB = | a ´ b |
mass of an atom is concentrated in its nucleus? 2
ur ur ur
Sol. Let m ® mass of H-atom 7.5 Show that a .( b ´ c ) is equal in magnitude to the volume
\ 35.5 m ® mass of Cl-atom ur ur
of the parallelopiped formed on the three vectors a , b
m 35.5 m r
x 1.27 - x and c .
C.M. Sol. B
1.27 Å
Let C.M. is situated at a distance x from b
H-atom. a A
O
\ Distance of C.M. from Cl atom
c
= (1.27 – x) Å C
Considering the C.M. as the origin, the sum of ur ur
moments of both the atoms about the origin will be b ´ c = bc sin 90° n̂ = bc n̂ where n̂ is the unit vector
uuur ur
zero. along OA perpendicular to the plane containing b and
\ mx – (1.27 – x) 35.5m = 0 ur
[both have opposite directions] c.
ur ur ur ur
x = 35.5 (1.27 – x). a .(b ´ c ) = a .bcnˆ = (a) (bc) cos 0°
x + 35.5x = 35.5 × 1.27 ur
36.5x = 35.5 × 1.27 (Q n̂ || a ) = abc = volume of the parallelopiped.
35.5 ´ 1.27 7.6 Find the components along the x, y, z-axes of the angular
Þ x= = 1.235Å r r
36.5 momentum l of a particle, whose position vector is r with
r
\ The C.M. of the molecule is located at a distance of components x, y, z and momentum is p with components
1.235Å from the H-atom. px, py and pz. Show that if the particle moves only in the
7.3 A child sits stationary at one end of a long trolley moving x-y plane the angular momentum has only a
uniformly with a speed v on a smooth horizontal floor. If z-component.
r
the child gets up and runs about on the trolley in any Sol. We know that angular momentum l of a particle having
manner, what is the speed of the C.M. of the system? r r
Sol. When the child gets up and runs on the trolley, the forces position vector r and momentum p is given by
r r r
acting on the system (child + trolley) are internal. Since, l =r ´p
no external force is acting on the system, the speed of the
C.M. of the system remains unchanged. But rr = [ xiˆ + yjˆ + zkˆ], where x, y, z are the components
7.4 Show that the area of the triangle contained between the of
r r r r r r
vectors a and b is one half of the magnitude of a × b . r and p = [ p x iˆ + p y ˆj + p z kˆ ]
uuur r uuur r
Sol. Let OA = a and OB = b and ÐAOB = q . Complete the r r r
\ l = r ´ p = [ xiˆ + yjˆ + zkˆ ] ´ [ p x iˆ + p y ˆj + p z kˆ ]
parallelogram OACB. Join BA. Draw BN ^ OA.
172 Physics
T1 sin q1 = T2 sin q2
iˆ ˆj kˆ
or (l x iˆ + l y ˆj = l z kˆ) = x y z T1 sin q2 sin53.1° 0.7407
= = = = 1.3523
px py pz T2 sin q1 sin36.9° 0.5477

= ( yp z – zp y )iˆ + ( zp x – xp z ) ˆj + ( xp y – yp x )kˆ Let C be the position of centre of gravity of the rod from
the left and at a distance d.
From this relation, we conclude that
For rotational equilibrium of the rod about C, the moment
lx = ypz – zpy, ly = zpx – xpz of the vertical forces must be equal and opposite.
and lz = xpy – ypx
T1 cos q1 T2 cos q 2
If the given particle moves only in the x–y plane, then z T1 T2
r q q2
= 0 and pz = 0 and hence, = ( xp y – yp x ) kˆ, which is only q1 1 q2
r l 2m
the z-compo-nent of l .
It means that for a particle moving only in the x–y plane, T1 sin q1 C T2 sin q 2
the angular momentum has only the z-component. d
7.7 Two particles, each of mass m and speed v, travel in
opposite directions along parallel lines separated by a W
distance d. Show that the vector angular momentum of the T1 cos q1 × d = T2 cos q2 (2 – d)
two particle system is the same whatever be the point Þ T1 cos 36.9° × d = T2 cos 53.1°(2 – d)
about which the angular momentum is taken.
Sol. Angular momentum about A, T1 cos 36.9° 2 - d
´ =
LA = mv × 0 + mv × d = mvd T2 cos 53.1° d
Angular momentum about B, 0.8366 2
LB= mv × d + mv × 0 = mvd Þ 1.3523 ´ = -1
0.6718 d
Angular momentum about C, 2 1.3523 ´ 0.8366
= + 1 Þ d = 0.745m
v m d 0.6718
B 7.9 A car weighs 1800 kg. The distance between its front and
y
back axles is 1.8 m. Its centre of gravity is 1.05 m behind
C d the front axle. Determine the force exerted by the level
d–y ground on each front wheel and each back wheel.
R2 R1
A
m v
Lc = mv × y + mv × (d – y) = mvd C
In all the three cases, the direction of angular momentum
is the same. 1.05 m

r r r 1.8 m
\ L = LB = LC .
Sol. m = 1800 kg.
7.8 A non-uniform bar of weight W is suspended at rest by Let R1, R2 = Reaction forces due to ground on each front
two strings of negligible weight as shown in the figure. wheel and each back wheel.
The angles made by the strings with the vertical are 36.9° \ R1 + R2 = mg = 1800 × 9.8 ..... (1)
and 53.1° respectively. The bar is 2m long. Calculate the Distance between the front and back axles = 1.8 m
distance 'd' of the centre of gravity of the bar from its Distance of centre of gravity from the front axle = 1.05 m
left end. For rotational equilibrium of the car about the centre of
gravity,
R1 × 1.05 = R2(1.8 – 1.05) = R2 × 0.75
R1 0.75 5
= = ..... (2)
53.1° R2 1.05 7
36.9° 2m
5
W
From (1) and (2), R2 + R2 = 1800 ´ 9.8
d 7
1800 ´ 9.8 ´ 7
Þ R2 = = 10290 N.
12
Sol. Let q1 = 36.9° and q2 = 53.1°
T1 and T2 are tensions in the two strings. For equilibrium 5 5
R1 = R2 = ´ 10290 = 7350 N
of the rod along the horizontal, 7 7
System of Particles and Rotational Motion 173

7.10 (a) Find the M.I. of a sphere about a tangent to the


a 2 I1 MR 2 5
sphere, given the M.I. of the sphere about any of its \ = = =
a1 I 2 2 2
2 MR 2
diameters to be MR 2 , where M is the mass and 5
5 Þ \ a 2 > a1 [a2 = 2.5a1]
R is the radius of the sphere. Q w = w0 +at, where w0 and t are constants
(b) What is the moment of inertia of a uniform circular \ w2 > w1
disc of radius R and mass M about an axis (i) passing \ Angular speed of the solid sphere is greater than that
through its centre and normal to the disc, (ii) of the cylinder.
passing through a point on its edge and normal to 7.12 A solid cylinder of mass 20 kg rotates about its axis with
the disc? The moment of inertia of the disc about any angular speed 100 rad s–1. The radius of the cylinder is
1 0.25m. What is the K.E. associated with the rotation of
of its diameters is given to be MR 2 . the cylinder? What is the magnitude of angular
4
momentum of the cylinder about its axis?
2 Sol. M = 20 kg., R = 0.25 m, w = 100 rad/s
Sol. (a) Given, IAB = MR 2 , ICD = ? M.I of the solid cylinder
5
P 1 2 1 2
A C A = MR = ´ 20 ´ (0.25) = 0.625 kgm2
2 2
1 1
K.E. of rotation = Iw2 = ´ 0.625 ´ (100) 2 = 3125 J.
O D C 2 2
R O Angular momentum = L = Iw = 0.625 × 100
= 62.5 Js.
B D B 7.13 A child stands at the centre of a turn table with his two
Q arms outstretched. The turn table is set rotating with an
(a) (b) angular speed of 40 rpm. How much is the angular speed
By parallel axes theorem, ICD = IAB + MR2 of the child, if he folds his hands back reducing the M.I.
to 2/5 time the initial value? Assume that the table rotates
2 7
Þ MR 2 + MR 2 = MR 2 without friction (b) Show that the child's new K.E. of
5 5 rotation is more than the initial K.E. of rotation. How do
(b) Consider 2 diameters AB and CD symmetrical to each you account for this increase in K.E.?
other
2
1 Sol. Here, w1 = 40 rev/min, I2 = I1
I AB = ICD = MR 2 (Given) 5
4
Q No external torque is acting on the system
(i) By ^ axes theorem, M.I. of the disc about an axis
Q L = Iw = constant
passing through its centre and normal to its plane
\ I1w1 = I2w2
1 1
IXY = IAB + ICD = 2 ´ MR 2 = MR 2
4 2 I1 5
w2 = w1 = ´ 40 = 100 rpm ;
(ii) By parallel axes theorem, M.I of the disc about an axis I2 2
passing through its edge and ^ to its plane
1
IPQ = IXY + MR2 I 2 w 22 æ I 2 ö æ w2 ö
2
Final K.E. 2
1 3 = = ç ÷ç ÷
MR 2 + MR 2 = MR 2 Initial K.E. 1 è I1 ø è w1 ø
= I1w12
2 2 2
7.11 Torques of equal magnitude are applied to a hollow
cylinder and a solid sphere, both having the same mass 2
2 æ 100 ö 5
and radius. The cylinder is free to rotate about its = ´ç ÷ = = 2.5
5 è 40 ø 2
standard axis of symmetry, and the sphere is free to rotate
about its axis passing through its centre. Which of the \ Final K.E. = 2.5 (Initial K.E.)
two will acquire a greater angular speed? This increase in K.E is due to the fact that child spends
Sol. I1,M.I of a hollow cylinder about its axis = MR2 his internal energy in folding his hands which is converted
I2,M.I of a solid sphere about an axis passing through its into K.E.
7.14 A rope of negligible mass is wound around a hollow
2
centre = MR 2 cylinder of mass 3 kg and radius 40 cm. What is the
5 angular acceleration of the cylinder, if the rope is pulled
Q Applied torque on the cylinder and the sphere is same with a force of 30 N? What is the linear acceleration of
\ t = I1a1 = I2a2
the rope? Assume that there is no slipping.
174 Physics
Sol. M.I of the hollow cylinder about its axis = MR2 = 3 × (0.4)2 7.18 A solid sphere rolls down two different inclined planes
= 0.48 kgm2 of the same heights but different angles of inclination.
[Q M = 3kg., R = 40 cm = 0.4m] Will it reach the bottom with the same speed in each case?
F = 30 N \ t = F × R = 30 × 0.4 = 12 N.m Will it take longer to roll down one plane than the other?
If so, which one and why?
t 12 Sol. Let v = speed of the solid sphere at the bottom of the
But t = Ia Þ a = = = 25 rad/s2
I 0.48 inclined plane.
Linear acceleration = a = Ra = 0.4 × 25 = 10m/s2 \ P.E. at the top of inclined plane = Total K.E. at the
7.15 To maintain a rotor at a uniform angular speed of 200 bottom of the plane
rad/s, an engine needs to transmit a torque of 180N-m. 1 1
What is the power of the engine required? Mgh = Mv 2 + I w 2 ;
2 2
Sol. w = 200 rad/s, t = 180N-m, P = tw = 180 × 200 = 36000
W = 36kW 2
M.I. of solid sphere = MR 2
7.16 From a uniform disk of radius R, a circular section of 5
radius R/2 is cut out. The centre of the hole is at R/2 from
the centre of the original disc. Locate the C.M of the 1 1 2
\ Mgh = Mv 2 + ´ MR 2 w 2
resulting flat body. 2 2 5
Sol. Let mass/unit area of the disc = m
1 1
= Mv 2 + Mv 2 [Q v = rw]
2 5
x R/2
P 7 10
O O' Mgh = Mv 2 Þ v= gh
10 7
Q The velocity is independent of the angle of inclination
\ Mass of original disc = M = pR2m \ Both the spheres will reach the bottom at the same
Mass of the cut out portion speed.
2 As the height is same, so the time taken by them will be
æ Rö pR2 M
= M ' = pç ÷ m = m= same.
è 2ø 4 4 7.19 A hoop of radius 2m weighs 100 kg. It rolls along a
M is concentrated at O and M' is concentrated at O'. After horizontal floor so that its centre of mass has a speed of
the removal of M', the system can be considered as two 20 cm/s. How much work has to be done to stop it?
masses one M at O and another – M' at O'. Sol. M = 100 kg, R = 2m, v = 20 cm/s = 0.2 m/s
If x be the distance of C.M. from O, then Work required to stop the hoop = Total energy
M R
- ´ 1 2
M ´ O - M '´ R/ 2 4 2 =-R W= Mv 2 + I w 2 [Q I = MR2]
x= =
2 5
M -M' M 6
M-
4
\ C.M. is at a distance R/6 to the left of O. 1 2 1 2 1 2
= Mv 2 + MR 2 w 2 = Mv + Mv [Q v = rw]
7.17 A metre stick is balanced on a knife edge at its centre. 2 5 2 2
When two coins, each of mass 5 g are put one on top of = Mv2 = 100 (0.2)2 = 4J.
the other at the 12 cm mark, the stick is found to be 7.20 The oxygen molecule has a mass of
balanced at 45 cm. What is the mass of the metre stick? 5.3 × 10–26 kg and a M.I of 1.94 × 10–46 kg m2 about an
Sol. Let m be the mass concentrated at C. axis through its centre perpendicular to the lines joining
A C' C B the two atoms. Suppose the mean speed of such a molecule
in a gas is 500 m/s and that its K.E. of rotation is two
12 cm 50 cm
45 cm thirds of its K.E of translation. Find the average angular
10 g velocity of the molecule.
mg Sol. Let m/2 = mass of each oxygen atom
For equilibrium about C¢, the moment of the forces should X
be equal.
10g (45 – 12) = mg (50 – 45)
O O
10 ´ 33 m/2 R R m/2
Þ 10 × 33 = m × 5 Þ m = = 66g
5 Y
System of Particles and Rotational Motion 175

2R = separation between them. 1 2 1 2


Total mass of the molecule Total K.E = P.E Þ mv + I w = mgh
2 2
= m = 5.3 × 10–26 kg
M.I. = I = 1.94 × 10–46 kg m2; v = 500 m/s Þ 1 mv2 + 1 æç 1 mr 2 ö÷ w2 = mgh éê Icyl = 1 mr 2 ùú
2 2è2 ø ë 2 û
m 2 m 2
I XY = r + r = mr2
2 2 3 2
Þ mv = mgh [using v = rw]
4
I XY 1.94 ´10 - 46
\ r= = = 0.61 × 10–10m 3v 2 3 ´ 52
m 5.3 ´10 -26 Þ h= = = 1.913 m.
4 g 4 ´ 9.8
Given that,
Let, s = distance moved up by the cylinder on the inclined
1 2 2 1 2 1 2 2 1 2 plane.
I w = ´ mv Þ (mr )w = mv
2 3 2 2 3

2 v 2 500 s
w= . = ´ h
3 r 3 0.61´ 10 -10 5m/s
12
= 6.7 × 10 rad/s q
7.21 A solid cylinder rolls up an inclined plane of angle of
inclination 30°. At the bottom of the inclined plane, the h h 1.913
\ sin q = Þ s= = = 3.826 m
C.M. of the cylinder has a speed of 5 m/s s sin q sin 30°
(a) How far will the cylinder go up the plane? Time taken to return to the bottom = t
(b) How long will it take to return to the bottom?
Sol. q = 30°, v = 5 m/s æ k2 ö
2s ç1 + ÷÷ æ 1ö
Let h be the height on the plane upto which the cylinder ç 2 ´ 3.826 ç1 + ÷
will go up. è r2 ø = è 2ø
= = 1.53s.
\ From conservation of energy, g sin q 9.8sin 30°

7.22 As shown in Fig. the two sides of a step ladder BA and \ W × (MB) = NC × (CB)
CA are 1.6 m long and hinged at A. A rope DE, 0.5m is
KB ´ BF 0.5 ´ 1.2
tied half way up. A weight 40 kg is suspended from a point But MB = = = 0.375m
F, 1.2 m from B along the ladder BA. Assuming the floor BA 1.6
to be frictionless and neglecting the weight of the ladder, Substituting this value in (i), we get
find the tension in the rope and forces exerted by the floor W ´ (MB) 392 ´ 0.375
on the ladder. (Take g = 9.8 m/s2) NC = = = 147 N
(CB ) 1
(Hint: Consider the equilibrium of each side of the ladder
separately.)
A A

F F q
W W
D E D E
NB H T
NC

B C B M K C
Sol. The forces acting on the ladder are shown in Fig.
Again considering equilibrium at point C in similar manner,
Here, m = 40 kg Þ weight = 40 × 9.8 N = 392 N, AB = AC we have
1 W × (MC) = NB × (BC)
= 1.6 m, BD = × 1.6 m = 0.8 m,
2 W ´ ( MC ) W ´ ( BC – BM )
BF = 1.2 m and DE = 0.5 m, \ NB = =
( BC ) ( BC )
In the Fig. DADE and DABC are similar triangles, hence,
393 ´ (1 – 0.375)
AB 0.5 ´ 1.6 = = 245N
BC = DE × = = 1.0m 1
AD 0.8
176 Physics
Now, it can be easily shown that tension in the string T (ii) Initial K.E. of the system
= NB – NC = 245 – 147 = 98N. 1 1
7.23 A man stands on a rotating platform, with his arms = E1 = I1w12 + I 2 w 2 2
2 2
stretched horizontally holding a 5 kg weight in each hand. Final K.E. of the system
The angular speed of the platform is 30 revolutions per
minute. The man then brings his arms close to his body 1
( I1 + I 2 )w 2
= E2 =
with the distance of each weight from the axis changing 2
from 90 cm to 20 cm. The M.I. of the man together with \ Change in K.E. of the system = E1 – E2
the platform may be taken to be constant and equal to 7.6 2
kgm2. What is his new angular speed? (Neglect friction). 1 1 1 æI w +I w ö
= I1w12 + I 2 w 2 2 – ( I1 + I 2 ) ç 1 1 2 2 ÷
Is K.E. conserved in the process? If not, from where does 2 2 2 è I1 + I 2 ø
the change come about?
Sol. Let the initial moment of inertia = I1 = 7.6 + 2 × 5(0.9)2 1 1
= I1w12 + I 2 w 2 2
= 15.7 kg m2 2 2
Final moment of inertia = I2 = 7.6 + 2 × 5(0.2)2 = 8 kgm2
w1 = 30 rpm and w2 = ? 1 ( I12 w12 + I 2 2 w 22 + 2 I1I 2 w1w 2 )

Q No external torque is acting is acting on the system 2 ( I1 + I 2 )
I1
\ L = Iw = constant Þ I1w1 = I2w2 Þ w 2 = w1 ( I1I 2 (w1 - w2 )2
I2 = >0
( I1 + I 2 )
15.7 ´ 30 \ E1 – E2 > 0 Þ E1 > E2
= = 58.88 rpm.
8 \ E2 < E1 Proved.
7.24 A bullet of mass 10 g and speed 500 m/s is fired into a \ The loss in K.E. is due to friction between the two
door and gets embedded exactly at the centre of the door. discs as they are in contact with each other.
The door is 1 m wide and weighs 12 kg. If it is hinged 7.26 (a) Prove the theorem of perpendicular axes.
at one end and rotates about a vertical axis practically (b) Prove the theorem of parallel axes.
without friction, find the angular speed of the door just Sol. (a) Consider a plane lamina in XY plane
after the bullet embeds into it. (Hint: The M.I. of the door IX = M.I. of the lamina along x axis
about the vertical axis at one end is ML2/3)
Sol. Angular momentum imparted by the bullet to the door = L IY = M.I. along Y axis and IZ = M.I. along Z axis
According to the theorem of perpendicular axes, IZ
1
= mvr = 10 × 10–3 × 500 × = 2.5 kgm2/s = IX + IY
2
ML2 12 ´ 1 Z
M.I. = I = = = 4 kgm2.
3 3
L 2.5
Also, L = Iw \ w = = = 0.625 rad/s.
I 4 xi
O
7.25 Two discs of moments of inertia I1 and I2 about their X
ri
respective axes (normal to the disc and passing through yi
the centre) and rotating with angular speeds w1 and w2 mi
are brought into contact face to face with their axes of
rotation coincident. Calculate, (i) angular speed of the two Y
disc system, (ii) show that K.E. of the combined system
is less than the sum of the initial kinetic energies of the Let the lamina is made up of n no. of particles of mass
two discs. How do you account for this loss of energy? m1, m2........., mn with perpendicular distance r1, r2.........,
Take w 1 ¹ w 2 rn from Z-axis respectively.
Sol. (i) Total angular momentum of both the discs = L1 Let xi and yi are the distances of a particle of mass
= I1w1 + I2w2 mi from X and Y axes respectively.
Let w is the angular velocity of the combined system \ IX =m1y12 + m2y22 + .....+ mnyn2
\ Final angular momentum of the system = L2 n
= (I1 + I2)w [I1 + I2 = Total M.I of the system]
Q No external torque is acting on the system = å mi yi 2
i =1
\ L = Iw = constant (I1 + I2)w = I1w1 + I2w2
Þ IY = m1x12 + m2x22 + .....+ mnxn2
I w +I w
Þ w= 1 1 2 2 ..... (1) n
I1 + I 2
= å mi xi 2
i =1
System of Particles and Rotational Motion 177

IZ = IX = m1r12 + m2r22 + .....+ mZrn2 7.27 Prove the result that the velocity v of translation of a
rolling body (like a ring, disc, cylinder or sphere) at the
n
bottom of an inclined plane of a height h is given by v2
= å mi ri 2
i =1 2 gh
= .
æ k2 ö
ç1+ 2 ÷
n n
IX + IY = å mi yi 2 + å m i xi 2 è R ø
i =1 i =1 Sol. Using dynamical consideration (i.e. by consideration forces
n n
and torques). Note k is the radius of gyration of the body
= å mi ( xi 2 + yi 2 ) = å mi ri 2 about its symmetry axis, and R is the radius of the body.
The body starts from rest at the top of the plane.
i =1 i =1
From the principle of conservation of energy, K.E. of
translation + K.E. of rotation = P.E. at the top of the plane
A C
1 2 1 2
Þ mv + I w = mgh [ Q I = Mk 2 ] from the
2 2
h ri definition of radius of gyration.
O 1 2 1 2 2
C.M. mi Þ mv + mk w = mgh
2 2
1 2 1 v2
Þ mv + mk 2 2 = mgh [Q v = Rw]
B D 2 2 R
1 2æ k2 ö
Q xi2 + yi2 = ri2 \ I Z = I X + IY Þ mv ç 1 + 2 ÷ = mgh
2 è R ø
(b) Consider a rigid body made up of n no. of particles
of masses m1, m2, ....., mn at perpendicular distances 2 gh
Þ v2 =
r1, r2, ......., rn respectively from the axis CD passing (1 + k 2 / R 2 )
through the centre of mass of the body. 7.28 A disc rotating about its axis with angular speed w0 is
Let ri be the perpendicular distance of mass mi from placed lightly (without any translational push) on a
CD. perfectly frictionless table. The radius of the disc is R.
n What are the linear velocities of the points A, B and C
I CD = å mi ri 2 on the disc as shown in the figure.
i =1
A
The perpendicular distance of the particle mi from AB
= ri + h w0
n R C
\ I AB = å mi (ri + h)2 2
i =1
f
n B
= å mi (ri 2 + h2 + 2ri h) Sol. For point A, VA = Rw0 along AX
i =1
For point B, VB = Rw0 along BX'
n n n
Þ I AB = å mi ri + å mi hi 2 2
+ å 2hmi ri For point C, VC =
R
2
w 0 parallel to AX
i =1 i =1 i =1
n
I AB = I CD + h 2 å mi w0
A
X
i =1 C
Q The body is balanced about its centre of mass R/2
\ Algebraic sum of moments of the weight of all O
n w0
particles about C.M is zero å mi ri = 0] X'
B
i =1 f
n
\ IAB = ICD + Mh2 where å mi = M (Total mass of 7.29. Explain why friction is necessary to make the disc roll
in the direction indicated. Shown in the solution above
i=1
the body) figure.
178 Physics
(a) give the direction of frictional force at B, and the (b) What is the work done against friction during
sense of frictional torque, before perfect rolling rolling?
begins. (c) If the inclination q of the plane is increased, at what
(b) What is the force of friction after perfect rolling value of q does the cylinder begin to skid and not
begins? roll perfectly?
Sol. To roll a disc, we require a torque, which can be provided Sol. Given: m = 10 kg, r = 15 cm = 0.15m, q = 30°, µs = 0.25
only by a tangential force. As force of friction is the only Acceleration of the cylinder when rolling down the inclined
tangential force in this case, it is necessary. plane = a
(a) As frictional force at B opposes the velocity of point
B, which is to the left, the frictional force must be to 2 2 9.8
= g sin q = 9.8 ´ sin 30° = m/s2
the right. The sense of frictional torque will be 3 3 3
perpendicular to the plane of the disc and outwards. (a) Force of friction = f = mg sin q – ma
(b) As frictional force at B decreases the velocity of the
point of contact B with the surface, the perfect rolling æ 9.8 ö
= m(g sin q – a) = 10ç 9.8 sin 30° - ÷
begins only when velocity of point B becomes zero. è 3 ø
Also, force of friction would becomes zero at this = 16.4 N
stage. (b) Work done against the friction during rolling is zero
7.30 A solid disc and a ring, both of radius 10 cm are placed as the point of contact is at rest.
on a horizontal table simultaneously with initial angular
speed equal to 10p rad/s. Which of the two will start to 1
(c) For rolling without slipping, µ = tan q
roll earlier? The coefficient of kinetic friction is µk = 0.2 3
Sol. Q Frictional force causes the acceleration of the C.M.
Þ tan q = 3 × 0.25 = 0.75 Þ q = 37°
\ µk × mg = ma
\ a = µkg ..... (1) Beyond this q, it will not roll perfectly.
Q The initial velocity of the C.M. is zero 7.32 Read each statement below carefully, and state, with
\ Using v = u + at Þ v = µkgt ..... (2) reasons, if it is true or false:
Torque due to friction causes retardation, (a) During rolling, the force of friction acts in the same
t = F × R = – Ia Þ µkmg × R = – Ia direction as the direction of motion of the CM of the
body.
- µk mgR
a= (b) The instantaneous speed of the point of contact
I during rolling is zero.
µ mgRt (c) The instantaneous acceleration of the point of contact
Q w = w0 + at Þ w = w 0 - k .
I during rolling is zero.
For rolling, v = Rw (d) For perfect rolling motion, work done against
æ µ mgRt ö friction is zero.
µk gt = R ç w 0 - k ÷ø (e) A wheel moving down a perfectly frictionless inclined
è I
For a ring, I = mR 2 plane will undergo slipping (not rolling) motion.
\ µRgt = Rw0 – µkgt Sol. (a) True. When a body rolls without slipping, the force
of friction acts in the same direction as the direction
æ 2µ gt ö w R
Þ w0 = ç k ÷ Þ t = 0 . of motion of the centre of mass of rolling body.
è R ø 2µk g (b) True. This is because rolling body can be imagined
to be rotating about an axis passing through the point
mR 2 of contact of the body with the ground. Hence its
For a disc, I =
2 instantaneous speed is zero.
\ µkgt = Rw0 – 2µkgt (c) False. This is because when the body is rotating, its
Rw 0 instantaneous acceleration is not zero.
3µRgt = Rw0 Þ t = (d) True. For perfect rolling motion as there is no relative
3µ R g
motion at the point of contact, hence work done
Q Time taken by the disc is less than the ring.
against friction is zero.
\ The disc will begin to roll earlier than the ring.
7.31 A cylinder of mass 10 kg and radius 15 cm is rolling (e) True. This is because rolling occurs only on account
perfectly on a plane of inclination 30°. The coefficient of of friction which is a tangential force capable of
static friction is µs = 0.25. providing torque. When the inclined plane is perfectly
(a) How much is the force of friction acting on the smooth, it will simply slip under the effect of its own
cylinder? weight.
System of Particles and Rotational Motion 179

7.33 Separation of Motion of a system of particles into motion (a) Momentum of ith particle
of the centre of mass and motion about the centre of mass: P = miV i¢
r r r r
(a) Show p = pi + mi V where pi is the momentum of = mi(Vi¢ + V) (since Vi = vi + V)
r r or P = miVi¢ + miV
the ith particle (of mass mi) and pi¢ = mi vi ' . Note P = Pi + miV
r (b) Kinetic energy of the system of particles.
vi¢ is the velocity of the ith particle relative to the
1
centre of mass.
Also, prove using the definition of the centre of mass
K =
1
å mi v = å miVi .Vi
2 i
2
r
å p¢i = 0 1
= å mi (Vi¢ + V ) (Vi + V )
2 i
1
(b) Show K = K ¢ + MV 2
2
1
where K is the total kinetic energy of the system of = å mi (Vi2 + V 2 + Vi .V )
2 i
particles, K¢ is the total kinetic energy of the system
when the particle velocities are taken with respect
1 1
to the centre of mass and MV2/2 is the kinetic = å
2 i
miV 2 + å miVi¢2 + å miVi .V
2 i
energy of the translation of the system as a whole i
(i.e., of the centre of mass motion of the system).
r r r r r 1
= MV 2 + K ¢
(c) Show L = L '+ R ´ MV 2
r r r
where L ' = å ri¢ ´ pi¢ is the angular momentum of where M = å mi = total mass of the system
the system about the centre of mass with velocities
1
taken relative to the centre of mass. Remember K¢ = å mi v1¢2
r r r 2 i
ri ¢ = ri - R ; rest of the notation is the standard
= kinetic energy of motion about the centre of mass
notation used in the chapter.
r uuur ur 1
Note L ' and MP ´ V can be said to be angular or Mv 2 = kinetic energy of motion of centre of mass.
momenta respectively, about and of the centre of mass 2
of the system of particles. (Proved)
r r
dL¢ r d p¢ dVi
(d) Show
dt
= åri' ´
dt
Since å miVi ¢.V = å mi
i i dt
.V

r
dL¢ d
Further, show that:
dt
= t¢ext
dt
( å miVi ¢ ) .V = 0
=

where t¢ext is the sum of all external torques acting (c) Total angular momentum of the system of particles.
on the system about the centre of mass.
[Hint: Use the definition of centre of mass and
L = ri × p = (ri¢ + R) ´ åmV
i i
Newton’s Third Law. Assume the internal forces i
between any two particles act along the line joining = ( ri¢ + R ) ´ å mi (Vi¢ + V )
the particles.] i

Sol. Here ri = ri¢ + R and Vi = Vi¢ + V æ ö


= å ( R ´ miV ) + å ri' ´ miV ¢i + çç å mi r¢i ÷÷
where ri¢ and vi¢ denote the radius vector and velocity of i i è i ø
the ith particle referred to centre of mass O¢ as the new origin
and V = R¢ is the velocity of centre of mass relative to O. ´V + R ´ å mV
i i
i
Y i
æ ö
ri
r¢i = å (R ´ miV ) + å ri¢´ miVi + çç å mi r¢i ÷÷
CM i i è i ø

dæ ö
R
X ´V + R ´ çç å mi ri ¢ ÷÷
O dt è i ø
180 Physics
The last two terms vanish for both contain the factor
dPi
å r ¢i ´ å ri ´ Fiext
åmi r¢i which is equal to = dt = = t¢ext

åmi ri = åmi (r¢i – R) = MR - MR = 0 Since å


dr ¢i
dt
´ Pi = å
dr ¢i
dt
´ mvi = 0
i i
from the definition of centre of mass. Also Total torque t = å ri ´ Fiext
å(R ´ mV
i ) = R × MV
= å ( ri + R ) ´ Fiext
i
so that L = R × MV + r¢i Pi
or L = R × MV + L¢ = å r ¢i ´ Fiext + R ´ å Fi ext

where L¢ = å r ¢i ´ Pi = t¢ext + t0(ext)


i where text is the total torque about the centre of mass as origin
(d) From previous solution and t ext
0 , that about the origin O.

L¢ = å ri' ´ pi t¢ ext = å r¢i ´ Fiext


i

dP ¢i d dL¢
dL¢
= å r ¢i ´
dPi dr ¢
+ å i ´ Pi = å ri ´ dt
= å
dt i
( ri ´ Pi ) =
dt
dt dt dt
System of Particles and Rotational Motion 181

Practice Questions
Very Short Answer Questions [1 Mark Qs.] 11. State the condition for translational equilibrium of a body.
12. State the condition for rotational equilibrium of a body.
1. Should the centre of mass of a body necesssarily lie inside
13. When the earth shrinks, without reducing its mass, what
the body? Explain.
change will be there in the duration of a day?
2. What physical quantities do the following represent?
14. Why are the spokes fitted in a cycle wheel?
(a) Moment of linear momentum
15. Why ships which have been emptied of cargo are filled
(b) Rate of change of angular momentum
with ballast (sand/water)?
(c) The product of moment of inertia and angular
acceleration? 16. Explain how a cat is able to land on its feet after a fall taking
3. Which component of linear momentum does not contribute advantage of the principle of conservation of angular
to angular momentum? momentum?
ur ur ur ur 17. A rifle barrel has a spiral groove which imparts spin to the
4. Calculate the angle between P and Q , if P and Q denote bullet. Why?
the adjacent sides of a parallelogram drawn from a point 18. Why are we not able to rotate a wheel by pulling or
1 pushing along its radius?
and the area of the parallelogram is PQ .
2 Short Answer Questions [2 or 3 Marks Qs.]
5. Why the moment of inertia of a solid disc is smaller than
that of a ring of same size and mass? 1. Prove that the centre of mass of an isolated system moves
6. With reference to figure of a cube of edge a and mass m, with a uniform velocity along a straight line.
state whether the following are true or false. (O is the 2. Derive a relation between torque and moment of inertia of
centre of the cube.) a body.
z z¢¢ 3. Derive a relation between the angular momentum and
H moment of inertia of a rotating body.
G
4. A particle is moving in, uniform circular motion with an

angular momentum L. If the frequency of rotation is
D C
O doubled and its K.E. becomes half, how its angular
momentum will change?
E 5. A stone of mass m tied to the end of a string is rotating
F
in a horizontal circle (neglect force of gravity). The length
Y of the string is reduced gradually keeping its angular
A a B momentum about the centre of the circle constant. Then
x the tension in the string is given by T = Arn where A =
(a) The moment of inertia of cube about z-axis is Iz = Ix constant and r is the instantaneous radius of the circle.
+ Iy Show that n = – 3
(b) The moment of inertia of cube about z¢ is 6. The length of a second’s hand of a clock is 10 cm. Calculate
ma 2 the speed of the tip of the hand.
Iz = Iz + 7. Why do we prefer to use a stearing wheel with a long arm?
2
8. The moments of inertia of two rotating bodies X and Y are
(c) The moment of inertia of cube about z¢¢ is
IX and IY (IX > IY) and their angular momenta are equal.
ma 2 Which one has greater K.E.?
= Iz + 9. A ring, a disc and a sphere all of the same radius and mass
2
roll down an inclined plane from the same height h. Which
(d) Ix = Iy of the three reaches the bottom (a) first (b) last?
10. How does an ice-skater, a ballet doncer or an acrobat take
7. Two solid spheres of same mass are made up of different advantage of the principle of conservation of angular
metals (r1 > r2). Which of them has a larger M.I. about momentum?
its diameter? 11. If the ice on the polar caps of the earth melts how will it
8. Which one has a larger M.I. and why? affect the duration of the day?
(a) a hollow sphere 12. Using expression for power P = tw and rotational K.E.
(b) a solid sphere of same mass and radius and why? 1
9. A big and a small sphere roll down from rest through an = Iw2, derive the relation M = Ia, where the symbols
2
inclined plane. Which one goes to the bottom first? have their usual meaning.
10. Why the spin angular velocity of a neutron star increases 13. If the earth were to shrink to half its radius what would
tremendously as it is formed from a star collapsing under be the duration of the day?
its own gravitational pull?
182 Physics
14. A thin uniform circular disc of mass M and radius R is 5. A uniform circular disc and a uniform circular ring, each has
rotating in a horizontal plane about an axis passing through a mass of 10 kg and diameter 1m. Calculate their moment of
its centre and perpendicular to its plane with an angular inertia about a transverse axis through their centre.
velocity w. Another disc of same dimension but of mass 6. Calculate moment of inertia of earth about a tangent, taking
M/4 is placed gently on the first coaxially. Show that the it to be a sphere of mass 1025 kg and diameter 12800 cm.
4 7. Three particles, each of mass m, are situated at the vertices
angular velocity of the system becomes w .
5 of an equilateral triangle PQR of side a. Find the M.I. of the
15. Two circular discs of same mass and thickness are made system about the line PX perpendicular to PR in the plane
up of two different metals with densities dX and dY (dX > of PQR as shown in the following figure.
dY). Their moments of inertia about the axes passing
through their centers of gravity and perpendicular to their X
Q
planes are IX and IY. Which one is greater, IX or IY?
16. The moments of inertia of two rotating bodies X and Y are
IX and IY (IX > IY) and their angular momenta are equal. Which
one has a greater K.E.?
Long Answer Questions [5 Marks Qs.]
1. Derive an expression for the position vector of centre of P a R
mass of a two particle system.
Y
8. Calculate K.E. of rotation of a circular disc of mass 2 kg and
m1 A radius 0.2 m rotating about an axis passing through its centre
f1 and perpendicular to its plane. The disc is making 30/ p
C.M.
r1 rpm.
r B
m2 9. The Sun rotates around itself once in 27 days. If it were to
r2 f2 expand to twice its present diameter, what would be its new
X period of revolution ?
O
10. A 20 kg flywheel in the form of a uniform circular disc 2 m in
radius is making 60 rpm. Calculate its angular momentum.
Z
11. If a constant torque of 500 N-m turns a wheel of moment of
2. Derive an expression for the kinetic energy of rotational inertia 100 kg m2 about an axis passing though its centre,
of a body. Hence define its moment of inertia?
find the gain in angular velocity in four seconds.
3. Derive an expression for the moment of inertia of a uniform
thin circular ring about an axis passing through its centre 12. A flywheel of moment of inertia 5 kg m2 is rotating at a
and perpendicular to its plane. speed of 60 rad/s. It comes to rest in 5 minutes, because of
4. Derive an expression for the moment of inertia of a uniform friction on the axes. Find (a) Average torque of the friction;
linear disc about a diameter of the disc. (b) Total work done by the friction (c) Angular momentum of
5. Derive an expression for the acceleration and the the wheel in one minute before it stops rotating.
coefficient of friction of a solid cylinder rolling without 13. A flywheel of mass 20 kg and radius 20 cm is rotating at an
slipping down an inclined plane. angular speed of 50 rpm when its motor is turned off.
6. Derive the equations of rotational motion by calculus
Neglecting friction at the axle, calculate the force that must
method.
be applied tangentially to the wheel to bring it to rest in 10
revolutions.
Numerical Questions [3 or 5 Marks Qs.] 14. A flywheel of mass 1000 kg and radius 1 m is rotating at the
1. Two bodies of masses 1 kg and 2 kg are lying in xy plane at rate of 420 rpm. Find the constant retarding torque required
(–2, 3) and (1, 2) respectively. What are the coordinates of to stop the wheel in 14 rotations, assuming mass to be
the centre of mass? concentrated at its rim.
2. Two particles of masses 1 kg and 3 kg are located at 15. A disc of mass 200 kg and radius 0.5 m is rotating at the rate
of 8 revolutions per second. Find the constant torque
(2iˆ + 5 ˆj + 13kˆ) m and (-6iˆ + 4 ˆj - 2kˆ)m respectively. Find required to stop the disc in 11 rotations.
the position of their centre of mass.
16. A particle starts rotating from rest according to the formula
3. In CO molecule, the centre of mass is at a distance of 0.068
nm from the C atom. What is the separation between the 3t 3 t 2
q= - . Calculate the angular velocity and angular
nuclei of carbon and oxygen ? 90 2
4. The moment of inertia of a circular ring about an axis passing acceleration at the end of 2 seconds.
through the center and perpendicular to its plane is 400 g 17. A flywheel rotating at the rate of 120 r.p.m slows down at a
cm2. If the radius of the ring is 10 cm, calculate the mass of constant rate of 2 rad/s2. What time is required to stop the
the ring. flywheel and how many rotations does it make in the process?
System of Particles and Rotational Motion 183

18. The spin drier of a washing machine revolving at 15 rps revolutions made by it before coming to rest.
slows down to 5 rps, while making 50 revolutions. Find (a) 20. On applying a constant torque, a wheel at rest, turns through
angular acceleration and (b) time taken. 400 radian in 10 s. Find angular acceleration. If same torque
19. The motor of an engine is rotating about its axis with an continues to act, what will be the angular velocity of the
angular velocity of 120 rpm. It comes to rest in 10s, after wheel after 20s from start ?
being switched off. 21. A uniform circular disc of mass 200 g and 4 cm radius is
Assuming constant decelaration, calculate the number of rotated about one of its diameters at an angular speed of 10
rad/s. Find the angular momentum about the axis of rotation?

HOTS/Exemplar Questions
Very Short Answer Questions [1 Mark Qs.] 2. The 40 kg woman stands on the end of a 4 meter long
uniform plank. If the maximum overhand for balance is 1
1. Both torque and work are products of force and distance. meter, estimate the mass of the plank. [HOTS]
How are they different?
[HOTS]
2. Two spheres, one hollow and one solid, are rotating with
the same angular speed about their centres. Both spheres
have the same mass and radius. Which one, if either, has
the higher rotational kinetic energy?
4m
[HOTS]
3. Why does a solid sphere have smaller moment of inertia 1m
than a hollow cylinder of same mass and radius, about an
axis passing through their axes of symmetry? [Exemplar]
4. A uniform sphere of mass m and radius R is placed on a
rough horizontal surface (Fig.). The sphere is struck 3. The vector sum of a system of non-collinear forces acting
horizontally at a height h from the floor. Match the following: on a rigid body is given to be non-zero. If the vector sum of
all the torques due to the system of forces about a certain
point is found to be zero, does this mean that it is necessarily
zero about any arbitrary point? [Exemplar]
4. (n – 1) equal point masses each of mass m are placed at the
vertices of a regular n-polygon. The vacant vertex has a
position vector a with respect to the centre of the polygon.
(a) h = R/2 (i) Sphere rolls without Find the position vector of centre of mass. [Exemplar]
slipping with a constant
velocity and no loss of Long Answer Questions [5 Marks Qs.]
energy. 1. Consider the diatomic oxygen molecule O2, which is rotating
(b) h=R (ii) Sphere spins clockwise, in the xy plane about the z axis passing through its center,
loses energy by friction. perpendicular to its length. The mass of each oxygen atom
(c) h = 3R/2 (iii) Sphere spins anti- is 2.66 × 10–26 kg, and at room temperature, the average
clockwise, loses energy separation between the two oxygen atoms is
by friction.
(d) h = 7R/5 (iv) Sphere has only a d < 1.21´10,10 m.
traslational motion, (a) Calculate the moment of inertia of the molecule about
looses energy by the z-axis. [HOTS]
friction. [Exemplar] (b) A typical angular speed of a molecule is 4.60 × 1012 rad/
Short Answer Questions [2 or 3 Marks Qs.] s. If the oxygen molecule is rotating with this angular
speed about the z-axis, what is its rotational kinetic
1. Two point masses 4 kg and 6 kg are moving along the same energy? [HOTS]
straight line with speed 3 m/s and 2 m/s respectively. Find 2. A rigid sculpture consists of a thin hoop (of mass m and
the velocity of their centre of mass if both the masses are radius R = 0.15 m) and a thin radial rod (of mass m and
moving in (a) the same direction (b) opposite direction. length L = 2.0R), arranged as shown in Fig. The sculpture
[HOTS] can pivot around a horizontal axis in the plane of the hoop,
passing through its center.
184 Physics
(a) Does the law of conservation of angular momentum
apply to the situation? why?
(b) Find the angular speed of the two-disc system.
(c) Calculate the loss in kinetic energy of the system in
the process.
(d) Account for this loss. [Exemplar]
4. A uniform disc of radius R, is resting on a table on its rim.
The coefficient of friction between disc and table is m(Fig.).
Now the disc is pulled with a force F as shown in the figure.
What is the maximum value of F for which the disc rolls
without slipping?
[Exemplar]
In terms of m and R, what is the sculpture’s rotational inertia
I about the rotation axis?
3. Two discs of moments of inertia I 1 and I 2 about their F
respective axes (normal to the disc and passing through the
centre), and rotating with angular speed w1 and w2 are
brought into contact face to face with their axes of rotation
coincident.

CHAPTER TEST
Time : 45 min. Max. Marks : 25
Directions : (i) Attempt all questions.
(ii) Questions 1 to 5 carry 1 mark each.
(iii) Questions 6,7 and 8 carry 3 marks each.
(iv) Questions 9 and 10 carry 5 marks each.

1. Define centre of mass. 6. Write theorem of perpendicular and parrallel axis.


2. Write an expression for the velocity of the centre of mass of 7. An electron of mass 9 × 10–31 kg revolves in a circle of
particles. radius 0.53Å around the nucleus of hydrogen with a velocity
3. The variation of angular position q, of a point on a rotating of 2.2 × 106 ms–1. Show that angular momentum of electron
rigid body, with time t is shown in fig. Is the body rotating h
is , where h is Planck’s constant.
clock-wise or anti-clockwise? 2p
8. Show that the centre of mass of an isolated system moves
q with a uniform velocity along a straight line path.
t1 t2 t3
9. Two blocks of masses m1 and m2 connected by a weightless
O t
spring of stiffness k rest on a smooth horizontal plane. Block
2 is shifted a small distance x to the left and released. Find
the velocity of the centre of mass of the system after block
1 breaks off the wall.
4. A disc of metal is melted and recast in the form of a solid 10. Derive an expression for the moment of inertia of a thin
sphere. What will happen to the moment of inertia about a uniform circular rod about an axis passing through its
vertical axis passing through the centre? centre and perpendicular to its length.
5. A ring and a circular disc of different materials have equal
masses and equal radii. Which one will have a larger moment
of inertia about an axis passing through its centre of mass
and perpendicular to its plane ?
System of Particles and Rotational Motion 185

Solutions
PRACTICE QUESTIONS Short Answer Questions
1. The equation of motion of a system of particles is given
Very Short Answer Questions r
d 2 r ur
1. No, it may lie outside the body. In case of semicircular ring, by M = f , where M is the total mass of the system
it is at the centre which is outside the ring. dt 2
r
2. (a) Angular momentum (b) Torque concentrated at the C.M. with position vector r
r
(c) Torque (t = I a) d æ dr ö r
3. Radial component as it is perpendicular to angular \ M ç ÷= f
dt è dt ø
momentum.
ur ur d r r
1 Þ M (vCM ) = f
4. Area of a parallelogram = | P ´ Q | = PQ sin q = PQ dt
2 r
r dr
1 where vCM = = velocity of C.M.
\ sin q = = sin 30° Þ q = 30° dt
2 For an isolated system, no external force acts from outside
ur ur
\ Angle between P and Q is 30° . and all internal forces cancel out in pairs.
r r
5. Since the entire mass of the ring is concentrated at its dv
\ f =0 \ M CM = 0
circumference, so, it is at maximum distance from the centre dt
but for the disc it is uniformly distributed over its area.
d r
6. (a) False, perpendicular axis theorem is applicable only Q M ¹0 \ (vCM ) = 0
to a lamina. dt
(b) True r
Þ vCM = constant
(c) False, z and z¢¢ are not parallel axes.
\ The C.M. of an isolated system moves with a constant
(d) True. velocity in a straight line.
7. Q The 2nd sphere is made up of a metal of lower density 2. Consider a rigid body consisting of n particles of masses
so its size will be bigger than the first. So the 2nd sphere m1, m2, ........., mn at perpendicular distances r1, r2, .............,
will have large M.I. rn respectively from the axis of rotation.
8. The hollow sphere has a large M.I. as its mass is
distributed on its surface and away from the axis of rotation.
9. Since the acceleration due to gravity is independent of
mass so both of them will reach the bottom simultaneously. r1
10. As the star collapses under its own gravitational pull, size
m1
of the star decreases. So its M.I. decreases. As its angular
rn mn
momentum remains constant ( L = I w) , so, its angular
velocity increases. m 2 r2
11. For translational equilibrium of a body net force acting on
it i.e., the vector sum of all the forces acting on the body
Let a ® angular acceleration of all the particles.
must be zero.
a1 ,a2, ..................., an ® linear accelerations of the masses.
12. For rotational equilibrium of a body the vector sum of a1 = r1a, a2 = r2a ............... , an = rna
torques of all the forces acting on the body about the Force on m1 = f1 = m1a1 = m1 r1a
reference point must be zero. Moment of this force about the axis of rotation = f1 × r1
13. L is conserved. If the earth shrinks, duration of the day = m1r1a × r1 = m1r12a
decreases. Total moment of all the forces acting on the particles of
14. To increase moment of inertia with less mass. the system = Torque acting on the body
15. Centre of mass is lowered to have more stability by loading = m1r21a + m2r22a + ............. + mnr2na = (m1r12 + m2r22
ballast. + ............. + mnr2n)a
16. While falling a cat stretches its body along with the tail
n n
so that its moment of inertia (I) increases. As no external
torque acts, L = Iw = constant. As I increase w decreases \ å mi ri ´ a = Ia, where I = å mi ri2 = M.I. of the
and it lands gently on its feet. i =1 i= 1
17. Angular momentum gained by the bullet provides better system of particles
accuracy. \ t = Ia
18. Torque to rotate is not created. 3. Consider a rigid body consisting of n particles of masses
m1, m2, ..........., mn with perpendicular distances r1, r2
..............., rn, respectively, from the axis of rotation.
186 Physics

æ 2p ö -2 2 p
6. As we know, v = r w = r ç ÷ = 10 ´10 ´
è T ø 60
p
r1 = ´ 10 -2 m/s.
3
m1
7. It produces torque required for rotational motion.
rn mn r r ur
Q t = r ´ F , stearing wheel with a long arm has large
m 2 r2 value of r. So, it will produce a larger torque with the
application of a small force.
1 2
8. Angular momentum L = I w and K.E. = I w
2
Let w ® angular velocity of all particles.
v1, v2 ............... vn ® linear velocities of the particles. 1 I 2 w 2 L2
= =
v1 = r1w, v2 = r2w ..............., vn = rnw 2 I 2I
\ Linear momentum of mass m1 = p1 = m1v1 = m1r1w 1
Angular momentum of m1 = r1 × p1 = m1r12w Q L is constant. \ K.E. µ
I
\ Total angular momentum of the body = m1r21w + m2r22w as IX > IY \ (K.E.)X < (K.E.)Y
+ ................. + mnr2nw \ K.E. of Y is greater than K.E. of X.
= (m1r21 + m1r22 +............... + mnr2n)w 9. Acceleration of an object down an inclined plane = a
g sin q
æ n ö =
çç å mi ri2 ÷÷ w = Iw, 1 + I / mr 2
è i =1 ø g sin q g
For ring, I = mr 2 \ aring = = sin q
1+ 1 2
æ n ö
where ç I =
ç å mi ri2 ÷÷ 2
For disc, I = mr \ adisc =
g sin q 2
1
= g sin q
è i =1 ø 2 1+
3
\ L = Iw 2
4. Angular momentum, 2 2
For sphere, I = mr
2
L = mvr = m(r w)r = mr (2pn) ............(i) 5
[w = 2pv] g sin q 5
\ asphere = = g sin q
1 1 1 2 7
1+
K .E = mv 2 = m(r 2 w 2 ) = mr 2 (4p 2 n 2 ) ; 5
2 2 2
asphere is maximum. So it will reach the bottom first and aring
K .E = 2p 2 mr 2 n 2 ........... (ii) [v = rw] is minimum so it will reach the bottom at last.
K .E K .E 10. Since, no external torque is acting on them, so, the angular
Dividing (ii) by (i), = pn Þ =L momentum (L = Iw) is constant. So, when they stretch their
L pn
New angular momentum hands, (I = Mr2), M.I. increases. So, their angular velocity
(w) decreases. When they fold their hands, r decreases.
( K .E.) ' ( K .E.) / 2
= = So M.I. decreases and angular velocity increases.
pn ' p.2 n 11. When ice on the polar caps of the earth melts, mass
1 K .E 1 concentrated near the axis of rotation spreads out.
= . = L \ M.I. (I = Mr2) increases. As no external torque is
4 pn 4
acting on the earth, so its angular momentum (L = Iw)
mv 2 remains constant. Therefore, as I increases w will decrease
5. For circular motion, T =
r æ 2p ö
and length of the day T ç = will increase.
Angular momentum = L = mvr = constant = è w ÷ø
K (say)
12. Power, P = rate of doing work = rate of change of K.E.
K
\ V= d æ 1 2ö 1 dw
mr \ P= çè I w ÷ø = I.2w
dt 2 2 dt
m K2 K 2 -3
\ T= . = r = Ar n é dw ù
r m2r 2 m Þ P = I.wa êë a = dt úû
K2 But P = tw `(given)
where A = = constant Comparing we get, M = Ia
m
Comparing we get, n = – 3 13. Q The earth is a solid sphere.
2
\ Its M.I. is I = MR2. If it shrinks, then its radius will
5
System of Particles and Rotational Motion 187

æ Rö Long Answer Questions


decrease, ç R ' = ÷ M.I. will also decrease.
è 2ø 1. Let m1, m2 ® masses of the 2 particles of the system,
As no external torque acting on the earth, so its angular r r
r1 , r2 ® position vectors of the particles.
momentum (L = Iw) is constant. r r
According to the law of conservation of an gular v1 , v2 ® velocities of the particles.
momentum, Y
I I m1 A
I1w1 = I 2 w 2 Þ 1 = 2 f1
T1 T2 C.M.
r1
I2 r B
or T2 = T1 m2
I1
r2 f2
2
2 2 æ Rö X
I1 = MR 2 , I 2 = M ç ÷ O
5 5 è 2ø
1
\ T1 = ´ 24 = 6 hr
4 Z
1 r r
14. Initial angular momentum of the disc = L = Iw = MR 2 w r dr r dr
2 \ v1 = 1 and v 2 = 2
dt dt
é 1 2ù
uur uur
êëQ I disc = 2 MR úû Let f1 = external force on m1, f2 = external force on m2.
uur uur
Final angular momentum of the combination should remain F 12 ® force on m1 due to m2, F 21 ® force on m2 due
same as no external torque is acting on it. to m1
M.I. of the combination r r
Linear momentum of m1 = p1 = m1v1
1 1æMö 5 From Newton's 2nd law, total force applied on a particle
= MR 2 + ç ÷ R 2 = MR 2 is equal to its rate of change of linear momentum.
2 è
2 4 ø 8 ur
d p1 uur ur
\ 5 MR 2 ´ w ' = 1 MR 2 w \ = f1 + F 12
8 2 dt
d r uur ur
4 Þ (m1 v1 ) = f1 + F 12
\ w' = w dt
5 ur
15. Let rX and rY be the radii of the discs X and Y. d p 2 ur ur d r ur ur
and = f 2 + F 21 Þ (m2 v 2 ) = f 2 + F 21
Q Their mass is same. dt dt
2 2 d r d r
\ m = prX ´ t ´ d X = m = prY ´ t ´ dY \ (m1 v1 ) + (m2 v 2 )
dt dt
ur ur ur ur
rX2 dY
[t = thickness, m = mass] Þ = = f 1 + F 12 + f 2 + F 21
rY2 dX
d r r r r r
(m1v1 + m2 v2 ) = f1 + f2 = f
1 2 dt
IX mrX r 2 d r r
= 2 = X = Y as d X > dY \ [as F12 = - F21 ]
IY 1
mr 2 rY
2 dX r r
d é dr1 dr2 ù r
2 Y m + m = f
dt êë dt dt úû
1 2
IY > I X
d éd r r ù r
1 2 Þ ê (m1r1 + m2 r2 )ú = f
16. K .E. = Iw and L = Iw dt ë dt û
2
2 é r r
d m r + m1r2 ù r
L2 Þ (m1 + m2 ) 2 ê 1 1 ú= f
\ K .E = dt ë m1 + m2 û
2I r r
Q L is same for both the bodies, m1 r 1 + m2 r 2 r
If we substitute = r CM then we get
(K.E.) X IY m1 + m2
\ =
( K.E.)Y I X r
As IX > IY d 2 r CM ur
(m1 + m2 ) = f which is the equation of motion of
\ (K.E.)Y > (K.E.)X dt 2
the centre of mass.
188 Physics
\ Position vector of the C.M. of a 2 particle system = 3. Let M ® mass and R ® Radius of the ring.
r r r Length of the ring = circumference = 2 p R
m1r1 + m2 r2
r CM = Mass/unit length = M/2 p R
m1 + m2
2. Let us consider an n particle system of masses X
m1, m2 ..............., mn at the perpendicular distances
r1 , r2 ..............., rn respectively, about the axis of rotation.
Z O dx
R

v2
r1 m2
m1 r2 Y
v1
M
Mass of a small length element dx =
.dx
O Y 2 pR
\ M.I. of this element about the axis XY
M MR
X = dI = .dx.R 2 = .dx
2 pR 2p
The angular momentum of each point mass is same as the \ Moment of inertia of the entire ring =
body is rigid and all its constituent particles take same time
2p r
to complete one revolution. MR
r r r
If v1 , v2 ..............., vn be their linear velocities respectively,
I=
ò dI =
2p ò dx
0
then v1 = r1w,
MR 2 pR MR MR
v2 = r2w,............., vn = rn w [ x] = (2 pR - 0) = .2 pR
2p 0 2p 2p
1 1 1 \ I = MR2
K.E. of m1 = m1v12 = (r1w 2 ) = m1r12 w 2 , K.E. of 4. Consider a disc of uniform thickness, radius R and mass
2 2 2
M.
1 Surface area of the disc = pR2
m2 = m2 r22 w 2 ,
2 M
Mass/unit area =
1 pR 2
K.E. of mn= m n rn2 w 2
2
X
\ Total K.E. of rotation of the body
1 1 1
= m 1r12 w2 + m 2 r22 w2 + .......... + m n rn2 w 2
2 2 2 R X dx
1 2
= w [m1r12 + m2r22 + .............. + mnrn2]
2
n Y
1
= w2
2
å 1
mi ri 2 = w2 .
2
Consider a small element of the disc, in the form of a circular
i =1 strip of radius x and width dx.
\ Length of the element = its circumference = 2px
n Surface area of the element = 2pxdx
Where I = å mi ri2 = M.I of the body
Mass of this element =
M
´ 2pxdx =
2Mxdx
2 =1
pR 2 R2
2 × K.E. of rotation 2Mxdx
\ I= M.I. of this element about XY axis = ´ x2
ω 2 R2
If w = 1, then I = 2 × K.E. of rotation.
2Mx3dx
\ M.I. of the rotating body can also be defined as twice dI =
the K.E. of rotation when the body is moving with unit R2
angular velocity. M.I. of the whole circular disc
System of Particles and Rotational Motion 189

R
For a solid cylinder, M.I. about its axis
2M
ò dI = ò x dx
3
=I = Mr 2 mg sin q 2
R2 0 I= \ a= = g sin q
2 mr 2 3
m+
R 2
2m é x 4 ù 2M é R 4 ù MR 2
= 2 .ê ú = 2 ê - 0ú = Ia I 2
R êë 4 úû 0 R êë 4 úû 2 \ Frictional force = F = = . g sin q
2
r r2 3
A
mr 2 2 1
= . g sin q = mg sin q
2 3 3
2r
1
D C mg sin q
O F
\ Coefficient of friction = m = = 3
R mg cos q

1
B Þ m= tan q
3
To calculate M.I. of the disc about its diameter, consider 6. There are 3 equations of rotational motion
AB and CD are two perpendicular diameters. (i) w = w0 + at
Applying perpendicular axes theorem, IAB + ICD = IXY
Q The diameter are symmetrical, 1 2
(ii) q = w0t + at
\ IAB = ICD 2

MR 2 MR 2 (iii) w2 – w 02 = 2aq
\ 2IAB = IXY = ; IAB =
2 4 Here w ® angular velocity of a body at time t, w0 ® initial
5. Let M ® mass of the solid cylinder R ® Radius of the angular velocity (at t = 0)
cylinder, q ® Angle of inclination of the plane, a - Angular q ® angular displacement at time t, a ® uniform angular
acceleration of the cylinder. acceleration.
R 1st equation:
a
F From the definition of angular acceleration,
q a = Rate of change of angular velocity
sin
mg q w t
mg cos q dw
q
mg Þ a=
dt
Þ dw = a dt Þ ò dw = ò adt Þ w – w0
w0 0
Various forces acting on the cylinder are: = a (t – 0) Þ w = w0 +at
R = mg cos q, F = frictional force acting upwards 2nd equation:
\ Net force in the downward direction = f = ma = mg From the definition of angular velocity, w = rate of change
sin q – F of angular displacement
The torque required for the rolling motion of the cylinder
is due to frictional force which acts tangential to the dq
w= Þ d q = wdt = (w0 + at)dt
surface. dt
Ia q
\ t = r × F Also t = Ia = [Q a = ra] t

Ia
r
Ia
Þ ò ò
d q = (w 0 + at )dt
0 0
\ = rF \ F=
r r2
t t

\ ma = mg sin q -
Ia
r2
ò
Þ q - 0 = w 0 dt + a t dt
ò
0 0
æ I ö
Þ a ç m + 2 ÷ = mg sinq æ t2 ö
è r ø q = w0 (t – 0) + a ç - 0÷
è 2 ø
mg sin q
Þ a=
I 1
m+ 2 Þ q = w 0 t + at 2
r 2
190 Physics
3rd equation:
2
1 1 æ 1ö
dq d w d w dq d w Idisc = MR2 = ´ 10 ´ ç ÷ = 1.25 kg m 2
Q w= and a = = . = .w 2 2 è 2ø
dt dt d q dt dq
\ w dw = a dq 6. M.I. of a solid sphere about an axis passing through its

w q 2
centre = MR 2
ò wd w = ò adq 5
w0 0 \ By parallel axes theorem, M.I. of the sphere about a
2 7 2
2 w 2 tangent = MR 2 + MR2 = MR
w q w - w 20 5 5
Þ = aq0 Þ = aq
2 2
w0 7
= × 1025 × (6.4 × 106)2 = 5.74 × 1038 kg m2
5
\ w2 - w02 = 2aq
7. Moment of inertia in the plane PQR
Numerical Questions I PQR = m 1 r 1 2 + m 2 r 2 2 + m 3 r 3 2 = m × 0 + m
1. Coordinate 2
æaö ma 2 5
× ç ÷ + m × a2 = + ma2 = ma2.
m1 x1 + m2 x2 1 ´ (-2) + 2 ´ 1 -2 + 2 è2ø 4 4
x= = = = 0;
m1 + m2 1+ 2 3 8. Kinetic energy,
1 2 30
m y + m2 y2 1 ´ 3 + 2 ´ 1 7 K.E. = Iw here, w = rpm
y= 1 1 = = . 2 p
m1 + m2 1+ 2 3
2. Given: 30 ´ 2p
uv = rad/s = 1 rad/s
p ´ 60
r1 = 2iˆ + 5 ˆj + 13kˆ ;
uuv MR 2 2 ´ (0.2)2
r2 = -6iˆ + 4 ˆj - 2kˆ Idisc = = = 0.04 kg m2;
uv uuv 2 2
uuv m1 r1 + m2 r2 1
r = K.E. = × 0.04 × 12 = 0.02 J.
m1 + m2 2
9. Since no torque is acting on the sun from outside
1(2iˆ + 5 ˆj + 13kˆ) + 3( -6iˆ + 4 ˆj - 2kˆ)
=
1+ 3 \ I w = const. Þ I1 w1 = I2 w 2
2iˆ + 5 ˆj + 13kˆ - 18iˆ + 12 ˆj - 6kˆ 2p 2p I I
= Þ I1. = I2 Þ 1= 2
4 T1 T2 T1 T2
-16iˆ + 17 ˆj + 7kˆ 17 ˆ 7 ˆ 2
= = -4iˆ + j+ k Q I= MR 2
4 4 4 5
3. Here, m1 = 12, m2 = 16, x1 = 0.068 nm 2 2
If x2 be the distance of C.M from O atom MR12 MR22 R12 T1
\ 5 5 =
m1x1 = m2x2 = Þ
T1 T2 R2 2 T2
m1 x1 12 ´ 0.068 Here, T1 = 27 days, T2 = ?, R1 = R, R2 = 2R
Þ x2 = = = 0.051 nm
m2 16
\ Separation between the nuclei of C and O R2 27
\ = Þ T2 = 27 × 4 = 108 days.
2 T2
= x1 + x2 = 0.068 + 0.051 = 0.119 nm (2 R )
4. M.I. of the ring, I = MR2
Here I = 400 g cm2, R = 10 cm. 1
10. Angular momentum L = Iw = MR2 w
400 2
\ M= =4g
10 ´10 60 ´ 6p
Here, M = 20 kg, R = 2m, w = 60 rpm = = 2 p rad/s
5. Moment of inertia, 60
2
æ1ö 1
Iring = MR2 = 10 × ç ÷ = 2.5 kg m2. \ L= × 20 × 22 × (2 p )2 = 160 p kg m2s–1
è2ø 2
System of Particles and Rotational Motion 191

11. Given: t = 500 N–m, I = 100 kg m2, q = 14rotations = 14 ´ 2p rad = 28p rad, w = 0
Q t =Ia

t 500 w 2 - w02
Use a = and t = Ia
\ a= = = 5 rad/s2 2q
I 100
by solving we get
w - w0
a= Þ w – w0 = a t = 5 × 4 = 20 rad/s required torque = 1000 Nm
t 15. Given, M = 200 kg R = 0.5 m;
w - w 0 0 - 60 1 1 1
12. (a) a = = = - rad/s2; I= MR2 = × 200 × (0.5)2
t 5 ´ 60 5 2 2
w0 = 8 rev/s = 8 × 2 p /60 rad/s
æ 1ö
t = I a = 5 ´ ç - ÷ = -1 Nm and w = 0, 8 = 11 × 2 p rad.
è 5ø
w2 - w0 2
(b) w = tq But w 2 = w 0 2 + 2Rq Use a = and then t = I a
2q
Solving we get t = 457.01 N-m
02 - 60 2 60 ´ 60 ´ 5
Þq= = = 9000 rad.
æ 1ö 2 3 2
2´ç- ÷ 16. Given q = 3t - t ;
è 5ø 90 3
\ W = – 1 × 9000 = – 9000 J = – 9 kJ.
3q 3.3t 2 2t
Negative sign indicates work is done by the wheel. w= = - = t2 - t
dt 90 2
\ work done by the friction = 9 kJ.
1 Þ w|t=2 = 2 = 22 – 2 = 4 – 2 = 2 rad/s
(c) w = w 0 + at = 60 - ´ 4 ´ 60 = 60 - 48
5
dw d 2
a= = (t - t ) = 2t - 1
dt dt
\ L = Iw= 5 × 12 = 60 kg m2/s
13. Goven: M = 20 kg. r = 20 cm = 20 × 10–2 m; w 0 = 50 rpm \ a |t = 2 = 2 = 2 × 2 – 1 = 4 – 1 = 3 rad/s2.

50 ´ 2p 5 120 ´ 2p
= = p rad / s 17. Here, w 0 = 120 rpm = = 4 p rad / s ;
60 3 60

q = 10 rev. = 10 × 2 p = 20 p rad a = – 2 rad/s2 w = 0, t = ?


0 = 4 p – 2 × t Þ t = 2 p sec.
w 2 - w 0 2 = 2aq
1 1
æ5 ö
2 q = w 0 t + at 2 = 4p ´ 2p - ´ 2. ´ (2 p) 2
2 2 2
0 - ç p ÷ = 2 ´ a ´ 20p
è3 ø
-5 / 3p ´ 5 / 3p 5p = 8p2 - 4p2 = 4p2 rad = 2p(2p)rad. = 2p rev..
Þ a= =-
2 ´ 20p 72 18. Given: w 0 = 15 rps = 15 × 2 p = 30 p rad/s;
1 w = 5 rps = 5 × 2 p = 10 p rad/s;
\ t = I a = F ´ R Þ MR 2 a = F ´ R
2 q = 50 rev = 50 × 2 p = 100 p rad.
æ -5p ö
20 ´ 20 ´ 10-2 ´ ç w 2 w 0 2 (10p)2 - (30p ) 2
MRa è 72 ÷ø (a) a= =
\ F= = 2q 2 ´ 100p
2 2
= -4p rad / s 2
-1
20 ´10
´ 5 ´ 3.14 31.4
=- =- = -0.4 N (b) w = w 0 + at
72 72
14. Here, moment of inertia, w - w0 10 p - 30 p
Þ t= = = 5s .
I = MR2 = 1000 × 12 = 1000 kg m2, a - 4p
420 ´ 2 p
w0 = 420 rpm = = 14p rad/s
60
192 Physics

120 ´ 2 p \ L = Iw = 8 × 10–5 × 10 = 8 × 10–4 J - s


19. Given: w0 = 120 rpm = = 4 p rad / s ,
60 HOTS/EXEMPLAR QUESTIONS
w = 0, t = 10s
Very Short Answer Questions
w - w 0 0 - 4p -2p
a= = = rad / s 2 1. Torque and work have two major differences. The primary
t 10 5
difference is that the vector displacement in the expression
for work is directed parallel to the force, but the position
w 2 - w0 2 02 - (4p )2 vector in the torque expression is perpendicular to the
q= = = 20p rad
2a æ -2p ö force. The second difference depends on whether there is
2´ç
è 5 ÷ø motion or not is the case of work, work is done only if the
force succeeds in causing a displacement of the point of
20 p application of the force. On the other hand, a force applied
= = 10 rev.. at a perpendicular distance from a rotation axis results in a
2p
torque whether there is motion or not.
20. Here, q = 400 rad, t = 10 s, w 0 = 0, 2. The hollow sphere has the higher rotational kinetic energy.
Its mass is located near the spherical surface, and the
1 2 1
q = w0 t + at = 0 + ´ a ´ 102 mass of the solid sphere is located throughout the volume.
2 2 This results in a higher moment of inertia for the hollow
400 = 50 a Þ a = 8 rad/s2
1 2
sphere (rotational K.E. = I w ).
w = w0 + a t = 0 + 8 × 20 = 160 rad/s 2
1 1 3. I = å m i r12 . All the mass in a cylinder lies at distance R from
21. M.I of the disc about its diameter = MR2 = × 0.2 ×
4 4 the axis of symmetry but most of the mass of a solid sphere lies at
(0.04)2 = 8 × 10–5 kg -m2. a smaller distance than R.
w = 10 rad/s 4. (a) ® iii; (b) ® iv; (c)® ii; (d)® (i)

Short Answer Questions


1. Given (a) m1 = 4 kg, v1 = 3 m/s, m2 = 6 kg, v2 = 2 m/s
m1v1 + m 2 v 2 4 ´ 3 + 6 ´ 2 24
Using, v cm = ; v cm = = = 2.4 m / s
m1 + m 2 4+6 10
(b) In this case, v1 = 3 m/s, v2 = –2 m/s
4´3- 6´ 2
v cm = =0 m/s
4+6
2.
Centre of gravity of plank
4m
2m 1m 1m

Weight of plank Weight of woman

The mass of the plank is about 40 kg. The plank tends to rotate like a seesaw about a pivot point at the edge of the building. Her
weight multiplied by 1 meter produces a torque that tends to rotate the system clockwise. The counterbalancing torque is
produced by the weight of the plank multiplied by the distance from the pivot point to the plank’s center of gravity. Note that
this distance is also 1 meter. So both the woman and the plank weigh the same . Their masses are equal.
System of Particles and Rotational Motion 193

3. No. Given å Fi ¹ 0 3. (a) Yes, because there is no net external torque on the system.
i External forces, gravitation and normal reaction, act through
The sum of torques about a certain point ‘0 the axis of rotation, hence produce no torque.
å ri ´ Fi = 0 (b) By angular momentum conservation
i Iw = I1w1 + I2w2
The sum of torques about any other point O’,
I1w1 + I2 w2
å ( ri - a ) ´ Fi = å ri ´ Fi - a ´ å Fi \ w=
I1 + I2
i i i
Here, the second term need not vanish.
4. Let the C.M. be ‘b’. Then, 1 (I w + I w ) 2
(c) Kf = ( I1 + I2 ) 1 1 2 22
( n - 1) mb + ma
= 0Þ b= -
1
a
2 (I + I )
1 2
mn n -1 2
1 ( I1w1 + I 2 w2 )
=
Long Answer Questions 2 I1 + I2
1. (a) We model the molecule as a rigid body, consisting of
two particles (the two oxygen atoms), in rotation. Ki =
1
2
(
I1w12 + I2 w 22 )
Because the distance of each particle from the z-axis
is d/2, the moment of inertia about the z-axis is I1I 2
DK = K f - Ki = ( w1 - w2 )2
2 ( I1 + I 2 )
æ d ö2 æ d ö2 md 2
I < Ρ mi ri 2 < m çç ÷÷ ∗ m çç ÷÷ <
i çè 2 ø÷ èç 2 ø÷ 2
(d) The loss in kinetic energy is due to the work against the
friction between the two discs.
4. Let the acceleration of the centre of mass of disc be ‘a’, then
(2.66´10,26 kg )(1.21´10,10 m)2
< Ma = F-f
2 The angular acceleration of the disc is a = a/R. (if there is no
sliding).
< 1.95´10,46 kg.m 2 Then

æ1 F
(b) Using the formula 2ö
ç MR ÷ a = Rf
1 2 è2 ø f
KR = Iw
2 Þ Ma = 2f
1 Thus, f = F/3. Since there is no sliding,
< (1.95´10,46 kg .m2 )(4.60 ´1012 rad/s) 2
2 Þ f £ mmg
= 2.06 × 10–21 J Þ F £ 3mMg
2. A key idea here is that we can separately find the rotational
inertias of the hoop and the rod and then add the results CHAPTER TEST
to get the sculpture’s total rotational inertia I. The hoop 1. An imaginary point at which the whole of the mass is said
1 2 to be concentrated is called centre of mass.
has rotational inertia Ihoop = mR about its diameter..
2 2. Velocity of centre of mass of particles system
The rod has rotational inertia Icom = mL2/12 about an axis r r
Vcm = m1v 2 + m 2 v 2 / m1 + m2
through its center of mass and parallel to the sculpture’s
rotation axis. To find its rotational inertia Irod about that 3. Positive slope indicates anticlockwise rotation which is
rotation axis, we use the parallel-axis theorem : traditionally taken as positive.

2
1
2 mL2 æ Lö 4. Moment of inertia will decrease, because Id = mr2 and Is
I rod = Icom + mhcom = + mç R + ÷ 2
12 è 2ø
2
= 4.33 mR2. = mr2, the radius of sphere formed on recasting the disc
5
where we have used the fact that L = 2.0R and where the
will also decrease.
perpendicular distance between the rod’s center of mass
5. A ring has a longer moment of inertia because its entire
and the rotation axis is h = R + L/2. Thus, the rotational
mass is concentrated at the rim at maximum distance from
inertia I of the sculpture about the rotation axis is
the axis.
1
I = I hoop + I rod = mR 2 + 4.33mR 2 6. Refer to theory
2 7. Here, m = 9 × 10–31 kg,
= 4.83 mR2 » 4.8 mR 2 r = 0.53Å = 0.53 × 10–10 m
194 Physics
v = 2.2 × 106 m/s The distances x1 and x2 are measured from the wall.
Angular momentum,
dxcm m1 dx1 m2 dx2
L = mvr = +
dt m1 + m2 dt m1 + m2 dt
= 9 × 10–31 × 2.2 × 106 × 0.53 × 10–10m
L = 1.0494 × 10–34 Js dx1
At start =0
h -34 dt
6.6 ´ 10
Also, = = 1.05 ×10–34 Js
2p 2 ´ 22 / 7 dxcm m2 m2 x k
Hence, L = h/2p \ = v2 =
dt m1 + m2 m1 + m2 m2
8. From Newton’s 2nd law of motion, the equation of motion
of C.M is
x km2
uur uur Velocity of centre of mass of system =
d 2 r uur d æ d r ö uur m1 + m2
M 2 = f Þ M ç = f
dt dt è dt ÷ø 10. Let M ® mass and l ® length of the cylindrical rod.
uur M
d r uur Mass/unit length =
But = V CM = velocity of centre of mass of the system, l
dt Consider a small element of the rod dx at a distance x from
d uur uur its center.
M
dt
(
V CM = f ) X
For an isolated system, external force = 0
dx
d uur
\ M
dt
( )
V CM = 0 Q M ¹ 0 O x

l /2 l /2
uur uur
\ d V
dt
(
CM = 0 Þ V CM) = constant Y
M
Q Velocity of C.M is constant \ Mass of the element = dx
l
\ C.M moves with uniform velocity along a straight line
path. M
M.I. of the element about axis XY = dx ´ x 2
1 2 l
9. We know that the potential energy of compression = kx Q The element can be anywhere along the length of the
2
rod.
k \ Total M.I. of the rod about XY = I
m1 m2 +l / 2
M 2
= ò l
x dx
When the block m1 breaks off from the wall, the spring has -l / 2
its unstretched length and the kinetic energy of the block l/2
m2 is given by M é x3 ù M éæ l ö 3 æ l ö 3 ù
` = .ê ú = êç ÷ - ç - ÷ ú
l êë 3 úû 3l êëè 2 ø è 2ø ú
û
1 1 2 kx 2 k -l / 2
m2 v22 = kx 2 Þ v2 = Þ v2 = x
m2 m2
2 2 M é l 3 l 3 ù M 2l 3 Ml 2
= .ê + ú = ´ =
m1 x1 + m2 x2 3l ëê 8 8 ûú 3l 8 12
For centre of mass , xcm =
m1 + m2
¿¿¿
8 Gravitation
C ha p t e r

All material objects attracted towards the earth. A falling apple led Issac Newton to realize this phenomenon. He said not only
the earth, every object in this universe attracts every other object. This phenomenon of attracting objects towards each other
is called gravitation.
KEPLER'S LAWS OF PLANETARY MOTION
Kepler gave three laws of planetary motion.
(i) Law of orbits : All planets revolve round the sun in elliptical orbits with the Sun at one of its foci.

Y
B Planet

2b
P O A X
S S¢
(Perihelion) (Aphelion)

C
2a
OP = OA = Semi-major axis

(ii) Law of areas : The line joining the Sun and the planet sweeps out equal areas in equal intervals of time.

v
dA P¢
i.e., = constant dA vdt
dt v
S r P
F
1 r(vdt) 1 L
= = rv = [since L = mvr]
2 dt 2 2m

(iii) Law of period : The square of the time period of revolution is directly proportional to the cube of the mean orbital radius
of the satellites. Considering that the satellites/planets revolve in elliptical orbit the orbital radius can be equivalent to the
semi-major axis of the ellipse traced out by the satellite. Satellite
2 3
T1 a
i.e., T2 µ a3 or, = 1 r
2
T2 a23
The position of a planet nearest to the sun is known as perigee. Perigee a Apogee
In this position the speed of planet is maximum. KE = max KE = min
PE = min Focus (sun) PE = max
The position of a planet at the maximum distance from sun is
b
known as apogee. In this position the speed of the planet is a = semi b = semi
minimum. major axis minor axis

Illustration 1 : r min r max

The period of revolution of planet A around the sun is 8 times that of B. The distance of A from the sun is how many times greater
than that of B from the sun?
196 Physics
Sol. According to Kepler’s law of periods,
2 3
æ TA ö æR ö
çç ÷÷ = çç A ÷÷
è TB ø è RB ø

2 3
æ 8 TB ö æR ö
\ çç ÷÷ = çç A ÷÷ [Q TA = 8 TB]
T
è B ø è RB ø
3
æ RA ö
or, 64 = çç ÷÷
è RB ø

3
3æR ö RA
or, (4) = çç A ÷÷ or, 4 = \ RA = 4 RB
è RB ø RB

NEWTON'S UNIVERSAL LAW OF GRAVITATION


According to Newton’s universal law of gravitation, every body in this universe attracts every other body with a force which
is directly proportional to the product of their masses and inversely proportional to the square of the distance between them.
Let there are two bodies, masses m 1 and m2 separated by a distance r, then the gravitational force of attraction (F) between them

1 m1m2
F µ m1m2 and F µ 2
Þ F µ
r r2

m1m2
\ F=G r m2
r2 m1

Here, G = Universal gravitational constant = 6.67 × 10–11 Nm2/kg2


The value of G is independent of the nature and size of the bodies as well as the nature of the medium between them. Hence
G is called universal gravitational constant.
r mm
In vector from F = - G 1 2 2 rˆ
r
r r r
where r̂ is the unit vector from m1 to m2 and r = r2 - r1
Characteristics of Gravitational Force
(i) Gravitational force is attractive in nature and acts along the line joining the two masses. So it is a central force.
(ii) It doesn't depend on the nature, size of the objects and the medium separating them.
(iii) It is independent of the presence of other masses.
(iv) It is the longest range force and least in the magnitude in the universe.
(v) It is strictly valid for point objects and spherically symmetrical objects.

Illustration 2 :
Calculate the force exerted by the earth on the moon. Take, mass of the earth 6 × 1024 kg, mass of the moon
7.4 × 1022 kg. Distance of the moon from the earth is 3.84 × 105 km.
GM1M 2
Sol. According to Newton’s universal law of gravitation, Force F =
R2
Here, G = 6.67 × 10–11
M1 = 6 × Nm2/kg2, 1024 kg; M2 = 7.4 × 1022 kg.
5 8
R = 3.84 × 10 km = 3.84 × 10 m

6.67 ´10 -11 ´ 6 ´10 24 ´ 7.4 ´ 10 22


F= = 20.2 × 1019 N.
(3.84 ´108 ) 2
Gravitation 197

Illustration 3 :
The mass of planet Jupiter is 1.9 × 1027 kg and that of the Sun is 1.99 × 1030 kg. The average distance of the Sun from
Jupiter is 7.8 × 1011m. Calculate the gravitational force exerted by the Sun on Jupiter.
Sol. Here, m1 = 1.9 × 1027kg, m2 = 1.99 × 1030 kg, r = 7.8 × 1011m, G = 6.67 × 10–11 Nm2/kg2
According to Newton’s universal law of gravitation, force

Gm1m2 6.67 ´10 -11 ´ 1.9 ´10 27 ´ 1.99 ´1030


F= = = 4.15 × 1023 N.
r2 (7.8 ´ 1011 ) 2

INERTIAL AND GRAVITATIONAL MASS


Inertial mass is defined as the ratio of force applied to a body and the acceleration produced in it.
i.e, Inertial mass m = F/a
Gravitational mass is the ratio of gravitational force to the acceleration due to gravity.
gravitational force
i.e, Gravitational mass, m= They are equal for a particular body.
g
Mass and weight: Mass is the amount of matter present in a body. Weight is the gravitational pull with which a body is pulled
by the earth towards its centre.
g Re2
Mass and density of the earth: Mass of the earth, Me = = 6 × 1024 kg, Radius of the earth, Re = 6400 km;
G
3g
Density of the earth, r = = 5.5 × 103 kg/m3
4pReG

ACCELERATION DUE TO GRAVITY OF THE EARTH


When a body is dropped from a certain height above the ground, it begins to fall towards the earth under gravity. The acceleration
produced in the body due to gravity is called the acceleration due to gravity. It is denoted by g. Its value close to the Earth's surface
is 9.8 m/s2.
Suppose that the mass of the Earth is M, its radius R, then the force of attraction acting on a body of mass m close to the surface of
GMm
Earth is F=
R2
According to Newton's second law, the acceleration due to gravity
F GM
g= or g= at the surface of the Earth
m R2
This is the relation between acceleration due to gravity (g) and universal gravitational constant (G).
This expression is free from mass of the body, m. If two bodies of different masses are allowed to fall freely they will have the same
acceleration, i.e., if they are allowed to fall from the same height, they will reach the earth simultaneously.
Acceleration due to gravity, g in terms of density of planet
Let r = density of the planet

4 3
Mass of the planet, M= pR r
3
4
G ´ pR 3r 4
Aceeleration due to gravity, g = GM = 3 or g= pG Rr
R2 R2 3

VARIATION IN ACCELERATION DUE TO GRAVITY


The value of ‘g’ acceleration due to gravity, varies from place to place on the surface of Earth. It also varies as we go above or below
the surface of the Earth.
198 Physics
Variation in acceleration due to gravity with the Height (Altitude) above the earth's surface
If the value of acceleration due to gravity at the surface of earth is g and at a height 'h' above the surface of earth is g' then
GM e
g= … (i)
Re2

GMe
g' = … (ii)
(R e + h)2
From eqns (i) and (ii)

g
g¢ =
2
æ h ö or,, g' = g æç 1 - 2 h ö÷ (if h << Re)
1+
èç R e ø÷ è Re ø

æ 2h ö
i.e., The decrease in the value of g on going up a height 'h' above the surface of earth by a factor 1 -
çè R ÷ø
e
2gh
Change in aceeleration due to gravity, Dg =
Re
Variation in acceleration due to gravity with the depth below the earth’s surface
If the value of acceleration due to gravity at the surface of earth is g and at a depth h from the earth's surface is g' then,
GM e
g= … (i) d
Re2 P

GMe Re–d
Re
g' = (R e - d) … (ii) O
R e3 r M'
From eqn s (i) and (ii)
Re – d = r
R -d æ d ö
g' = g e or, g' = g ç 1 - ÷
Re Re ø
è
æ d ö
Thus the value of g decreases by a factor ç1 - R ÷ as we go down below the surface of the earth.
è eø
The graph showing variation of acceleratiodue to gravity (g) with (i) height above the earth’s surface and (ii) depth below the
earth’s surface.
's
th
ar ce
ce th's

e a
g At urf
rfa ar

s
su e e
ov
Ab

r=R r

Inside Distance from the Pole


gP
the earth centre of the earth

Variation in acceleration due to gravity due to shape of the earth Rp


RE Equator
The earth is elliptical in shape. It is flatter at the poles and bulged out gE
O
at the equator. Now, we know that
g µ 1/R2, RP < Re \ gE < gP
Therefore the value of g at the equator is minimum and the value of g at the poles is maximum.
Gravitation 199

Variation in acceleration due to gravity due to axial rotation (latitude) of the earth
If the observed value of g at the latitude l is represented by gl ,then w
gl = ge – Rew2 cos2 l P
r FcF
where w is the angular velocity of the earth.
Fg
At equator, l = 0º; cos l = 1 Re
q
\ gl = ge – Re w2 C
At poles, l = 90º ; cos l = 0
\ gl = ge i.e., gp > ge
Illustration 4 :
Calculate the height above the Earth’s surface at which the value of acceleration due to gravity reduces to half its value on the
Earth’s surface. Assume the Earth to be a sphere of radius 6400 km.
2
gh æ R ö g
Sol. We know that =ç ÷ ; But g h =
g è R + h ø 2
2
1 æ R ö R 1
\ =ç ÷ Þ =
2 è R+hø R+h 2

R+h h
or, = 2 Þ = 2 - 1 = 0.414
R R
h = 0.414 ´ R = 0.414 ´ 6400 km or h = 2649.6 km
\ At a height of 2649.6 km from the Earth’s surface, the acceleration due to gravity will be half of its value on the surface of the
earth.
Illustration 5 :
If the radius of the earth were to shrink by one per cent, its mass remaining the same, the value of g on the earth’s surface would
(a) increase by 0.5% (b) increase by 2% (c) decrease by 0.5% (d) decrease by 2%.
GM dg dR
Sol. (b) As we know, g = Þ = -2
R 2 g R

dR dg
Q = -1% \ = 2% i.e., the value of g increases by 2%
R g

GRAVITATIONAL FIELD AND FIELD INTENSITY


Gravitational field of a body is the region or space around a body upto which the gravitational force due to that particular
body can be experienced. Theoretically it is extended upto infinity.
Gravitational field intensity at a particular point in the gravitational field is the gravitational force experienced by unit mass placed
at that point in the field.
GM
i.e., Gravitational field intensity E=
r2
where r is the distance of the unit mass from the centre of mass M.
GRAVITATIONAL POTENTIAL AND GRAVITATIONAL POTENTIAL ENERGY
Gravitational Potential
The gravitational potential at a point in the gravitational field of a body is defined as the amount of work done in displacing a body
of unit mass from infinity to that point in the field.
r
Q P
dr

M
200 Physics
Consider a body of unit mass placed at a distance r from the centre of a mass M. Then the gravitational force acting on the unit mass
is given by

GM ´1 GM
F= 2
= … (i)
r r2

The direction of this force is towards the centre of the body of mass M. Let the unit mass be displaced from point P to Q, through a
distance dr towards mass M, then the work done is given by
r uur
dW = F .dr = Fdr cos q = Fdr cos 0°

GM
Þ dW = F dr = dr (From eqn. (i)) …(ii)
r2

Total work done in displacing the unit mass from infinity (r = ¥) to the point P whose distance from mass M is r can be calculated by
integrating eqn. (ii) between limits r = ¥ to r

r
GM
\ ò dW = ò r2
dr
¥

r
r
GM
r é r -1 ù
-2
or W = ò r2
dr Þ W = GM ò r dr = GM ê ú
ë -1 û ¥
¥ ¥

r
é1ù é1 1 ù
or W = -GM ê ú = -GM êë r - ¥ úû
ë r û¥

GM é 1 ù
Þ W = –
r êëQ ¥ = 0úû

This work done is equal to the gravitational potential

–GM
Gravitational potential, V =
r
Gravitational Potential Energy
Gravitational potential energy of any object at any point in gravitational field is equal to the work done in bringing it from
infinity to that point.

-GMm
Gravitational potential energy U =
r
i.e., Gravitational potential energy = gravitational potential × mass

Illustration 6 :
Calculate the gravitational potential on the surface of the earth from the following data. Radius of the earth = 6.37 ×
108 cm, Mean density of the earth = 5.5 g/cm3 and G = 6.67 × 10–11 Nm2/kg2.
Sol. As we know, gravitational potential

-GM -G æ 4 3 ö -4pGR 2 r
V= = çè pR r÷ø =
R R 3 3
-4 ´ 3.14 ´ 6.67 ´ 10-11 ´ (6.37 ´ 106 ) 2 ´ 5.5 ´103
V= = – 6.22 × 107 J/kg.
3
Gravitation 201

Illustration 7 :
What is the potential energy of system shown in figure?

Sol. There are total 10 pairs of masses. 4 are corner pairs and 2 are diagonal pairs and 4 pairs between mass at center and
corners.

4Gm 2 2Gm 2 4Gm 2


\ Potential energy, U= - - -
l 2 l l/ 2

ESCAPE SPEED
Escape speed is the minimum speed that should be given to the body to enable it to escape away from the gravitational field
of earth.
If the mass of the planet is M and its radius is R, then the escape speed from its surface will be
ve = (2GM / R)

or ve = (2 gR ) [since GM = gR2]
Escape speed from the surface of earth is 11.2 km/sec.
• The value of escape velocity does not depend upon the mass of the projected body, instead it depends on the mass and
radius of the planet from which it is being projected.
• The value of escape velocity does not depend on the angle and direction of projection.
• The minimum energy needed to escape a body from the surface of the planet = GMm/R.
• If the velocity of a satellite orbiting the earth is increased by 41.4% , then it will escape away from the gravitational field of the
earth.
SATELLITES
Just as the planets revolve around the sun, in the same way few celestial bodies revolve around these planets. These bodies are called
'Satellites'. For example moon is the natural satellite of Earth. Artificial satellites are launched from the Earth. Such satellites are used for
telecommunication, weather forecast etc. The path of these satellites are elliptical with the centre of Earth at a focus. However, the
difference in major and minor axes is so small that they can be treated as nearly circular for not too sophisticated calculations.
Let us derive certain characteristics of the motion of satellites by assuming the orbit to be perfectly circular, and mass of the satellite
is much smaller than the Earth's mass.
Orbital velocity (v0) : Let a satellite of mass m revolves around the Earth in circular orbit of radius r with speed v0. The
gravitational pull between satellite and earth provides the necessary centripetal force.

mv02
Centripetal force required for the motion =
r
GMm
Gravitational force =

mv02 GMm GM
= 2 or v02 =
r r r
GM
or v0 = = gR ... (1)
r

g GM
or v0 = R Q g= and r = (R + h)
R+h R2
202 Physics

Relation between escape speed (ve) and orbital speed (v0) : ve = 2v0
(i) Value of orbital velocity does not depend on the mass of satellite but it depends on the mass and radius of the planet around
which the rotation is taking place.
(ii) The orbital velocity for a satellite near the surface of earth is 7.92 km/sec.
(iii) Greater is the height of the satellite, smaller is the orbital velocity.
(iv) The direction of orbital velocity is along the tangent to the path.
(v) The work done by the satellite in a complete orbit is zero.
Period of revolution (T) :
circumference of the orbit 2pr 2p
Period of revolution T = or T= = [as v0 = rw]
orbital velocity v0 w

2 p r 2p ( R + h) 2pr
Þ T= = or T=
v0 v0 GM / r

r3
T = 2p ...(2)
GM
4p2 3 GMT 2
On squaring eqn. (2), T2 = r T² µ r³ or r3 =
GM 4 p2
1
æ GM 2 ö 3
or r = ç T ÷ ...(3)
è 4 p² ø
1
æ GM ö3
or r = ç ´ R ²T ² ÷
è 4 p² R ² ø
1
æ gR²T² ö 3
or r = ç ...(4)
è 4 p ² ÷ø 1
æ gR 2T 2 ö 3
and R + h = çç 2 ÷ ÷
è 4p ø
1
æ gR 2T 2 ö 3
\ Height of satellite h = çç 2 ÷
-R
÷
è 4p ø
Energy of satellite : A satellite revolving around a planet has both kinetic and potential energy.
(i) Kinetic energy : The kinetic energy of the satellite is due to motion of the satellite.
mv02 GMm GMm 1 2 GMm
= or mv02 = or mv0 =
r r2 r 2 2r
GMm
Kinetic energy of orbiting satellite, K =
2r
(ii) Potential energy : As for external point a spherical mass behaves as whole of its mass is concentrated at its centre;
GMm
potential energy of the satellite. U = -
r
The negative sign is because of zero potential energy at infinity.
(iii) Total energy : Total energy of orbiting satellite,
GMm é GMm ù GMm GMm
E=K+U = + ê- = -
2r ë r úû 2r r
GMm
or E =-
2r
Gravitation 203

GEO-STATIONARY SATELLITE
A satelite which appears to be stationary for a person on the surface of the Earth is called geostationary satellite.
It is also known as Communication Satellite or Synchronous Satellite.
(i) The orbit of the satellite must be circular and in the equatorial plane of the Earth.
(ii) The angular velocity of the satellite must be in the same direction as the angular velocity of rotation of the earth
i.e., from west to east.
(iii) The period of revolution of the satellite must be equal to the period of rotation of earth about its axis.
i.e. 24 hours = 24 × 60 × 60 = 86400 sec.
(iv) Height from the surface of the earth is nearly 35600 km.
1 1
1
4p ² æ GMT 2 ö 3 æ GM R 2T 2 ö 3
T2 = r³ or r =ç ÷ =ç ´ ÷ = é9.8 ´ (6.38 ´ 106 ) 2 ´ (86400)² ù 3
GM ç 4 p2 ÷ ç R2 2 ÷
4p ø ê 4p ² úû
è ø è ë
= 42237 × 103 m = 42,237 km. » 42000 km. approximately.
h = r – R = 42000 – 6400 = 35600 km.
(v) The orbital velocity of this satellite is nearly 3.08 km/sec.
(vi) The relative velocity of geostationary satellite with respect to earth is zero.
This type of satellite is used for communication purposes.
The orbit of a geostationary satellite is called 'Parking Orbit'.
POLAR SATELLITE
Polar Satellites go around the poles of the earth in north-south direction and the earth rotates around its axis in east-west direction.
The altitude of polar satellite is around 500 to 800 km and its time period is around 100 minutes.
These satellites can view polar and equatorial regions at close distance with good resolution. Polar satellites are extremely useful for
remote sensing, meterology as well as environmental studies of the earth. PSLV series satellites are polar satellites of India.
WEIGHTLESSNESS
A situation wherein the effective weight of the object becomes zero. An astronaut experiences weightlessness in space satellite
because the astronaut as well as the satellite are in a free fall state towards the earth. When the object is free fall, it is weightless and
this phenomenon is called weightlessness.
Illustration 8 :
Calculate the escape velocity of a body from the surface of the moon. Consider the moon to be a uniform sphere of radius
1.74 × 106 m and mass 7.36 × 1022 kg. (G = 6.67 × 10–11Nm2/kg2)
2GM
Sol. As we know, escape velocity ve =
R

2 ´ 6.67 ´10 -11 ´ 7.36 ´10 22


= = 2.38 × 103 m/s
1.74 ´ 106

Illustration 9 :
Calculate the time period of a satellite which revolves round a planet in an orbit just above the surface of the planet using
the data: G = 6.67 × 10–11 Nm2/kg2 and mean density of the planet = 8 × 103 kg/m3.
Sol. Centripetal force = gravitational force

mv 2 GMm mw 2 R 2 GMm GM G 4 3 4
i.e., = Þ = [v = r w] Þ w2 = 3
= . pR r = pGr
3 3
R R 2 R R2 R R 3

2p 4p 2 4 3p
Q w= \ = pGr \ T=
T T 2 3 Gr

3 ´ 3.14
T= = 4205 s.
6.67 ´ 10-11 ´ 8 ´ 103

Illustration 10 :
204 Physics
An artificial satellite is moving in a circular orbit around the earth with a speed equal to half the magnitude of escape
velocity of the earth.
(a) Determine the height of the satellite above the earth’s surface.
(b) If the satellite is stopped suddenly in its orbit and allowed to fall freely onto the earth, find the speed with which it
hits the surface of the earth.
1 GM 1 2GM
Sol. (a) Velocity of the satellite = × escape velocity Þ =
2 R+h 2 R
GM 1 2GM
Squaring both sides, = ´ Þ R + h = 2R Þ h = R = 6400km.
R+h 4 R
(b) Change in P.E. of the satellite on hitting the surface of the earth
-GMm æ GMm ö -GMm GMm
= P.E. at earth's surface – P.E. at height h = -ç- ÷ = +
R è R + hø R R+h

GMm GMm GMm GMm -GMm


=– + (as h = R) = - + =
R R+ R R 2R 2R
Decrease in P.E. = Increase in K.E.

1 2 GMm 2 GM gR 2
\ mv = Þ v = = = gr [Q GM = gR 2 ]
2 2R R R

v= gR = 9.8 ´ 6400´ 103 = 7.92×103 m/s


Gravitation 205
206 Physics

Textbook Exercises
8.1 Answer the following: R1 = 1A.U., R2 = ?
(a) You can shield a charge from electrical forces by 2/3 2/3
putting it inside a hollow conductor. Can you shield æT ö æ1/ 2 ö
R2 = R1 çç 2 ÷÷ = 1ç ÷ = 0.63 AU
a body from the gravitational influence of nearby è T1 ø è 1 ø
matter by putting it inside a hollow sphere or by
8.4 Io one of the satellites of Jupiter, has an orbital period
some other means?
of 1.769 days and the radius of the orbit is 4.22 × 108
(b) An astronaut inside a small spaceship orbiting
m. Show that mass of Jupiter is about one-thousandth
around the Earth cannot detect gravity. If the space
times that of the sun. (Take 1 year = 365.25 mean solar
station orbiting around the Earth has a large size,
day)
can he hope to detect gravity?
Sol. For the satellite of Jupiter, T1 = 1.769 days
(c) If you compare the gravitational force on the Earth
due to the Sun to that due to the Moon you would find = 1.769 × 24 × 60 × 60 s, R1 = 4.22 × 108 m
that the Sun's pull is greater than the Moon's pull. 4p 2 R13
(You can check this yourself using the data available \ Mass of Jupiter = M1 =
in the succeeding exercises). However, the tidal effect GT12
of the Moon's pull is greater than the tidal effect of
4 ´ (3.14) 2 ´ (4.22 ´ 108 )3
Sun. Why? Þ M1 =
Sol. (a) No. Gravitational forces are independent of medium. 6.67 ´ 10 -11 ´ (1.769 ´ 24 ´ 60 ´ 60) 2
A body cannot be shielded from the gravitational ..... (1)
influence of nearby matter. For earth around the sun, T2 = 1 year
(b) Yes. If the size of the spaceship is extremely large, = 365.25 × 24 × 60 × 60s.
then the gravitational effect of spaceship may become R2 = 1A.U = 1.496 × 1011 m
measurable. The variation in g can also be detected. 4p 2 R2 2
(c) Tidal effect depends inversely on the cube of the Mass of the sun = M2 =
GT2 2
distance, unlike force which depend inversely on the
square of the distance. Since the distance of moon 4(3.14)2 ´ (1.496 ´ 1011 )3
from the ocean water earth is very small as compared ...... (2)
6.67 ´ 10-11 ´ (365.25 ´ 24 ´ 60 ´ 60) 2
to the distance of sun from the ocean water earth. Dividing (1) by (2),
Therefore, the tidal effect of Moon's pull is greater
3
than the tidal effect of sun. M1 R13T22 æ 4.22 ´ 108 ö æ 365.25 ö
2
8.2 Choose the correct alternative: = 3 2 =ç ÷ ´
M 2 R2T1 ç 1.496 ´1011 ÷ çè 1.769 ÷ø
(a) Acceleration due to gravity increases/decreases with è ø
increasing altitude. M1 1 1
(b) Acceleration due to gravity increases/decreases with = »
M 2 1046 1000
increasing depth (assume the Earth to be a sphere
of uniform density). 1
\ M1 = M2
(c) Acceleration due to gravity is independent of the 1000
mass of the Earth/mass of the body. 1
\ Mass of Jupiter is times the mass of the sun.
æ 1 1ö 1000
(d) The formula - GMm ç - ÷ is more/less 8.5 Let us consider that our galaxy consists of
è r2 r1 ø 2.5 × 1011 stars each of one solar mass. How long will
accurate than the formula mg (r2 – r1) for the this star at a distance of 50,000 ly from the galactic
difference of potential energy between two points r2 centre take to complete one revolution? Take the diameter
and r1 distance away from the centre of the Earth. of the Milky way to be 105 ly. G = 6.67 × 10–11 Nm2/kg2
Sol. (a) decreases (b) decreases Sol. r = 50,000 ly = 50, 000 × 9.46 × 1015 m
(c) mass of the body (d) more. = 4.73 × 1020 m
8.3 Suppose there existed a planet that went around the sun M = 2.5 × 1011 solar mass
twice as fast as the earth. What would be its orbital size = 2.5 × 1011 × 2 × 1030 kg = 5 × 1041kg
as compared to that of the earth?
4p 2 r 3
Q M =
T12 R13 GT 2
Sol. From Kepler's 3rd law, = ; Here,
T22 R23 1/ 2
æ 4p 2 r 3 ö
T1 = 1 year, T2 = 1/2 year, \ T= ç ÷
è GM ø
Gravitation 207

1/ 2 Sol. (a) We know that the legs carry the weight of the body
é 4 ´ (3.14) 2 (4.73 ´10 20 )3 ù in the normal position due to gravity pull. The
= ê -11 41
ú
êë 6.67 ´10 ´ 5 ´10 úû astronaut in space is in weightlessness state. Hence,
16 swollen feet may not affect his working.
= 1.12 × 10 s.
(b) In the conditions of weightlessness, the face of the
8.6 Choose the correct alternative:
astronaut is expected to get more supply. Due to it,
(a) If the zero of potential energy is at infinity, the total
the astronaut may develop swollen face. As eyes,
energy of an orbiting satellite is negative of its
ears, nose, mouth etc are all embedded in the face,
kinetic/potential energy. hence, swollen face may affect to great extent the
(b) The energy required to launch an orbiting satellite seeing/hearing/ smelling/ eating capabilities of the
out of Earth's gravitational influence is more/less astronaut in space.
than the energy required to project a stationary object (c) Headache is due to mental strain. It will persist
at the same height (as the satellite) out of Earth's whether a person is an astronaut in space or he is
influence. on earth. It means headache will have the same effect
Sol. (a) If the zero of potential energy is at infinity, the total on the astronaut in space as on a person on earth.
energy of an orbiting satellite is negative of its kinetic (d) Space also has orientation. We also have the frames
energy. of reference in space. Hence, orientational problem
(b) The energy required to launch an orbiting satellite out will affect the astronaut in space.
of Earth's gravitational influence is less than the 8.10 In the following two exercises, choose the correct answer
energy required to project a stationary object at the from among the given ones. The gravitational intensity
same height (as the satellite) out of Earth's influence. at the centre of a hemispherical shell of uniform mass
8.7 Does the escape speed of a body from the Earth depend density has the direction indicated by the arrow (see Fig.)
on (a) the mass of the body, (b) the location from where (i) a, (ii) b, (iii) c, (iv) 0.
it is projected, (c) the direction of projection, (d) the height g b
of the location from where the body is launched? P C a
d e f c
2GM
Sol. The escape speed ves = = 2 gR . Hence,
R
(a) The escape speed of a body from the Earth does not
depend on the mass of the body.
(b) The escape speed does not depend on the location
from where a body is projected. Sol. At all points inside a hollow spherical shell, potential is
(c) The escape speed does not depend on the direction same. So, gravitational intensity, which is negative of
of projection of a body. gravitational potential gradient, is zero. Due to zero
(d) The escape speed of a body depends upon the height gravitational intensity, the gravitational forces acting on
of the location from where the body is projected, any particle at any point inside a spherical shell will be
because the escape velocity depends upon the
symmetrically placed. It follows from here that if we remove
gravitational potential at the point from which it is
the upper hemispherical shell, the net gravitational force
projected and this potential depends upon height
acting on a particle at P will be downwards. Since
also.
gravitational intensity is gravitational force per unit mass
8.8 A comet orbits the Sun in a highly elliptical orbit. Does
therefore, the direction of gravitational intensity will be
the comet have a constant (a) linear speed (b) angular
along c. So, option (iii) is correct.
speed (c) angular momentum (d) kinetic energy
(e) potential energy (f) total energy throughout its orbit? 8.11 For the above problem, the direction of the gravitational
Neglect any mass loss of the comet when it comes very intensity at an arbitrary point P is indicated by the arrow
close to the Sun. (i) d, (ii), e, (iii) f (iv) g.
Sol. (a) The linear speed of the comet is variable in Sol. Using the explanation given in the solution of the previous
accordance with Kepler's second law. When comet problem, the direction of the gravitational field intensity
is near the sun, its speed is higher. When the comet at P will be along e. So, option (ii) is correct.
is far away from the sun, its speed is very less. 8.12 A rocket is fired from the earth towards the sun. At what
(b) Angular speed also varies slightly. point on its path is the gravitational force on the rocket
(c) Comet has constant angular momentum. zero? Mass of sun = 2 × 1030 kg, Mass of earth = 6 ×
(d) Kinetic energy does not remain constant. 1024 kg. Neglect the effect of the other planets. Orbital
(e) Potential energy varies along the path. radius of the earth = 1.5 × 1011 m
(f) Total energy throughout the orbit remains constant. Sol. Ms = 2 × 1030 kg. Me = 6 × 1024 kg r = 1.5 × 1011m
8.9 Which of the following symptoms is likely to afflict an Let at a distance x from the earth the gravitational force
astronaut in space (a) swollen feet, (b) swollen face, on the rocket due to sun and the earth are equal and
(c) headache, (d) orientational problem. opposite.
208 Physics
\ Distance of the rocket from the sun = r – x 8.15 A body weighs 63N on the surface of the earth. What is
If m is the mass of the rocket then the gravitational force on it due to the earth at a height
equal to half the radius of the earth?
GM s m GM e m ( r - x) 2 Ms
= Þ = Sol. Weight of the body = mg = 63N
2 2 2 Me
(r - x ) x x Acceleration due to gravity at a height 'h'

10 3 é R2 ù gR 2
r-x Ms 2 ´ 1030 = g' = gê ú =
4
= g
Þ = Þ = 2 2
x Me 6 ´ 1024 3 êë ( R + h) úû æ Rö 9
çè R + ÷ø
2
r 103 r 103 10 3 + 3
Þ Þ = +1= \ Gravitational force on the body at a height h = mg'
-1 =
x 3 x 3 3
4 4
= m´ ´ g = ´ 63 = 28N
11 9 9
3r 1.732 ´ 10 ´ 1.5
\ x= = 8.16 Assuming the earth to be a sphere of a uniform mass
3
10 + 3 10 3 + 1.732
density, how much would a body weigh half way down to
= 2.59 × 108 m the centre of earth if it weighed 250 N on the surface?
8.13 How will you ‘weigh the sun’, that is estimate its mass? [g on the surface of the earth = 9.8 m/s2]
The mean orbital radius of the earth around the sun is
1.5 × 108 km. æ dö
Sol. Acceleration due to gravity at a depth 'd' is g ' = g ç1 - ÷
Sol. The mean orbital radius of the Earth around the Sun è Rø
R = 1.5 × 108 km = 1.5 × 1011 m
Time period, T = 365.25 × 24 × 60 × 60 s æ dö
\ Weight = mg' = mg ç1 - ÷
Let the mass of the Sun be M and that of Earth be m. è Rø
According to law of gravitation
æ R / 2ö
= 250ç1- = 125N
R ÷ø
Mm
F= G ...(i) è
R2
8.17 A rocket is fired vertically with a speed of
Centripetal force,
5 km/s from the earths surface. How far from the earth
mv 2 does the rocket go before returning to the earth? Mass
F= = m.R.w2 ...(ii) of the earth = 6 × 1024 kg,
R
Mean radius of the earth = 6.4 × 106 m
From eqn. (i) and (ii), we have
G = 6.67 × 10–11 Nm2/kg2
GMm Sol. Let, v = velocity of the rocket when it is fired from the earth.
= mRw2 h = height at which its velocity becomes zero.
R2
Total energy of the rocket at the surface of the earth
mR 4p 2 é 2p ù 1 2 æ GMm ö
= êQ w = T ú = K.E. + P.E. = mv + ç - ÷
T 2 ë û 2 è R ø
At the highest point v = 0
4 p2 R 3
\ M= -GMm
GT 2 \ K.E. = 0 and P.E. =
R+h
4 ´ (3.14) 2 ´ (1.5 ´ 1011 )3 -GMm
= \ T .E =
6.67 ´ 10 -11 ´ (365.25 ´ 24 ´ 60 ´ 60) 2 R+h
= 2.009 × 1030 kg = 2.0 × 1030 kg. Using law of conservation of energy,
8.14 A Saturn year is 29.5 times the earth year. How far is
1 2 GMm GMm
the Saturn from the Sun if the Earth is 1.5 × 108 km away mv - = -
from the sun? 2 R R+h
Sol. Ts = 29.5 Te, Re = 1.5 × 108 km; 1 2 é1 1 ù éR + h- Rù
v = GM ê - ú = GM ê ú
Ts 2 Rs3 2 ëR R + hû ë R( R + h) û
From Kepler's 3rd law, =
Te 2 Re3 1 2 gR 2 .h
Þ v = [Q gR2 = GM]
2 /3 2/3 2 R( R + h )
æT ö æ 29.5Te ö
\ Rs = Re ç s ÷ = 1.5 × 108 ç
è Te ø è Te ÷ø 1 2
v =
gRh
Þ v2(R + h) = 2gRh
2 R+h
= 1.43 × 109 km
Gravitation 209

Þ v2R + v2h = 2gRh Sol. Mass of each star = M = 2 × 1030kg. Radius = R = 104
Þ h(v2 – 2gR) = – v2R km = 107 m
Initial distance between 2 stars = r
Þ h (2gR – v2) = v2R
= 109km = 1012m
v2R
Þ h= -GM 2
2 Initial P.E. of the system = ;
2 gR - v r
Total K.E. of the system
(5 ´103 ) 2 ´ 6.4 ´10 6
= 1 1
2 ´ 9.8 ´ 6.4 ´10 6 - (5 ´103 ) 2 = Mv2 + Mv2 = Mv 2
2 2
= 1.6 × 106m v = speed of collision of each star.
8.18 The escape velocity of a projectile on the surface of earth Distance between the centres of the stars when they are
is 11.2 km/s. If a body is projected out with thrice of this about to collide = r' = R + R = 2R
speed, find the speed of the body far away from the earth.
Ignore the presence of other planets and sun. \ -GM 2
Final P.E. of the system =
Sol. ve = 11.2 km/s, velocity of projection = v = 3ve 2R
Let m and vo be the mass and the velocity of the projectile Q Loss in P.E. = Gain in K.E. \
far away from the earth
then using law of con servation of en ergy, GM 2 æ GM 2 ö
MV 2 = - -ç- ÷
1 1 2 1 r è 2R ø
mvo2 = mv - mve2
2 2 2 é 1 1 ù
2 × 1030 × V 2 = – GM 2 ê - ú
12
vo = v 2 - ve2 = (3ve )2 - ve2 = 8ve ë10 2 ´ 10 7 û
Þ 2 × 103 × v 2
= 2 2 ´ 11.2 = 2 × 1.414 × 11.2 = 31.68 km/s
8.19 A satellite orbits the earth at a height of 400 km above é 2 - 105 ù
= – 6.67 × 10–11 × (2 × 1030)2 ê ú
the surface. How much energy must be expended to rocket êë 2 ´ 1012 ûú
the satellite out of the gravitational influence of the earth?
Mass of the satellite is 200 kg, mass of the earth 1.334 ´ 10 43
= 6 × 1024 kg, radius of the earth = 6.4 × 10 6 m, v2 =
2 ´ 10 30
G = 6.67 × 10–11 Nm2/kg2
Þ v = 2.583 × 106 m/s.
Sol. Total energy of the satellite at a height
8.21 Two heavy spheres each of mass 100 kg and radius 0.1m
GMm 1 2 are placed 1 m apart on a horizontal table. What is the
h= - + mv gravitational field and potential at the mid point of the line
( R + h) 2
joining the centres of the spheres? Is an object placed at
GMm 1 GM GMm that point in equilibrium? If so, is the equilibrium stable
= - + m = - or unstable.
R+h 2 R+h 2( R + h)
Energy expended to rocket the satellite out of the earth's Sol. O
M M
GMm
gravitational field = – (T.E. of the satellite) = r
2( R + h)
Gravitational field at the mid point O of the line joining the
6.67 ´10 -11 ´ 6 ´10 24 ´ 200 two spheres is equal and opposite
= = 5.9 × 109 J.
2(6.4 ´10 6 + 4 ´ 105 ) GM GM
Net field = - 2
+ =0
8.20 Two stars each of 1 solar mass (= 2 × 10 30 kg) are ( r / 2) ( r / 2) 2
approaching each other for a head on collision. When they \ Net field zero.
are at a distance109 km, their speeds are negligible. What GM GM -4GM
Potential at O = V = - - =
is the speed with which they collide? The radius of each r/2 r/2 r
star is 104 km. Assume the stars to remain undistorbed
until they collide. Use the known value of G. - 4 ´ 6.67 ´10 -11 ´100
= = – 2.7 × 10–8 J/kg
1
210 Physics

8.22 A geostationary satellite orbits the earth at a height of


é 2 ´1030 6.4 ´ 10 23 ù
nearly 36,000 km from the surface of the earth. What = 6.67 × 10–11 × 1000 ê + ú
11
is the potential due to earth's gravity at the site of this êë 2.28 ´10 3395 ´ 103 úû
satellite? (Consider the P.E. at infinity to be zero.) Mass
of the earth = 6 × 1024 kg Radius = 6400 km é 20 6.4 ù
Sol. Gravitational potential at a height h above the surface of = 6.67 × 10–8 ê + ú × 10
18
ë 2.28 33.95 û
-GM = 5.98 × 1011J.
the earth = V =
R+h 8.25 A rocket is fired ‘vertically’ from the surface of Mars with
a speed of 2km/s. If 20% of its initial K.E. is lost due
- 6.67 ´10 -11 ´ 6 ´ 10 24
= = – 9.4 × 106 J/kg. to martian atmospheric resistance, how far will the rocket
(6.4 ´10 6 + 36 ´10 6 ) go from the surface of Mars before returning to it ? Mass
8.23 A star 2.5 times the mass of the sun and collapsed to a of Mars = 6.4 × 10 23 kg; radius of Mars
size of the 12 km rotates with a speed of 1.5 revolution = 3395 km. G = 6.67 × 10–11 Nm2/kg2
per second. Will an object placed on its equator remain
Sol. Let m = mass of the rocket; M = mass of Mars; R = Radius
struck to its surface due to gravity? (Mass of the sun
of Mars
= 2 × 1030 kg)
v = initial velocity of the rocket.
Sol. The object will remain struck to the surface of star due to
Total initial energy of the rocket = K.E. + P.E.
gravity, if the acceleration due to gravity is more than the
centrifugal acceleration due to its rotation. 1 2 GMm
= mv -
GM 2 R
Acceleration due to gravity = g = Q 20% of the K.E. is lost
R2 \ Energy available to the rocket
6.67 ´ 10 -11 ´ 2.5 ´ 2 ´10 30 80 1 2 GMm 2 GMm
= 2
= 2.3 × 1012 m/s2 = ´ mv - = 0.4 mv -
(12000) 100 2 R R
Centrifugal acceleration = rw2 = r (2 p)2 = 12000 × (2 × 3.14 If the rocket reaches at a height h above the surface of
× 1.5)2 = 1.1 × 106 m/s2 -GMm
Here g > rw2 Mars, at this point K.E. = 0 and P.E. =
R+h
\ The objected will remain struck with the surface. \ From law of conservation of energy,
8.24 A spaceship is stationed on Mars. How much energy must
be expended on the spaceship to rocket it out of the solar GMm GMm
0.4 mv 2 - = -
system? Mass of the space ship = 1000 kg, Mass of the R R+h
sun = 2 × 1030 kg, Mass of the Mars = 6.4 × 1023 kg,
GM GM 1
Radius of Mars = 3395 km, Radius of the orbit of Mars = - 0.4v 2 = [GM - 0.4 Rv 2 ]
= 2.28 × 1011m G = 6.67 × 10–11 Nm2/kg2. R+h R R
Sol. Let R' = Radius of the Mars, R = Radius of the orbit of R+h R
Mars, M = Mass of the sun =
M' = Mass of the Mars, m = Mass of the GM GM - 0.4 Rv 2
space-ship. GMR
\ P.E. of space-ship due to gravitational attraction of Þ R+h =
GM - 0.4 Rv 2
-GMm
the Sun = GMR
R Þ h= -R
P.E. of space-ship due to gravitational attraction of Mars GM - 0.4 Rv 2
-GM ' m é GM ù
= h = Rê - 1ú = 3395 × 103
R' 2
ë GM - 0.4Rv û
Q The K.E. of space-ship is zero.
\ Total energy of the ship é ù
ê -11 23 ú
ê 6.67 ´ 10 ´ 6.4 ´ 10
-GMm GM ' m æ M M 'ö - 1ú
= - = -Gm ç + ê 6.67 ´ 10-11 ´ 6.4 ´ 10 23 - 0.4 ´ 3395 ´ 103 ´ (2 ´ 103 ) 2 ú
R R' è R R ' ÷ø ê ú
êë úû
Energy required to rocket out the space ship from the solar
system = – (total energy) = 42
= 3395 × 103 × 0.1458
é æ M M 'ö ù æ M M 'ö Þ h = 495 × 103 m = 495 km.
= – ê -Gm ç + = Gm ç +
ë
÷
è R R'øû ú è R R ' ÷ø
Gravitation 211

Practice Questions
Very Short Answer Questions [1 Mark Qs.] Short Answer Questions [2 or 3 Marks Qs.]
1. The mass of two bodies are doubled and the distance is 1. A planet reduces its radius by 1% keeping its mass
halved, how will the gravitational force change? unchanged. How does its acceleration due to gravity
2. If two masses are equal and made of same material, how change?
will the force of attraction vary with separation? 2. Prove that the acceleration due to gravity varies with
3. What is the height at which the value of g is the same
as at a depth of R/2?
æ 2h ö
4. A mass is dropped into a hole bored along the diameter height as g' = g ç1 - ÷
è Rø
of earth. What is the type of motion with the mass?
5. If the earth stops rotating about its own axis, what is the 3. Derive an expression to find the variation of acceleration
effect on the value of g on the surface of earth? due to gravity at a depth 'd' below the surface of the earth.
6. Can a pendulum undergo simple harmonic motion in a
satellite? 4. What is the angular velocity at any point on the equator
7. Two artificial satellites, one close to the surface and the so that the body feels weight lessness?
other away are revolving around the earth. Which has 5. Show that the gravitational potential at a distance r from
larger speed?
8. Which has a longer time period, a satellite revolving close GM
a mass M is given by V = – .
or away from the surface of the earth? r
9. Will 1 kg sugar weigh more at poles or at the equator?
10. A satellite revolving around earth loses height. How will 6. Derive an expression for the escape velocity from the
its time period be changed? surface of the earth.
11. What is the amount of work done in bringing a mass from 7. The value of 'g' at the moon is 1/6th of the value of 'g'
the surface of Earth on one side to a point diametrically at the surface of the earth and the diameter of the moon
opposite to the other side?
is 1/4th of the diameter of the earth. Compare the ratio of
12. What would happen to an artificial satellite, if its orbital
velocity is slightly decreased due to some defects in it? the escape velocities.
13. What is 1 kg-wt? 8. The radius of the earth is reduced by 4%. The mass of
14. Define gravitational field intensity at any point? Write its the earth remains unchanged. What will be the change in
S.I. units. escape velocity?
15. What is the reason for absence of atmosphere in some
9. If a planet existed where mass and radius were both half
planets ?
16. Define potential difference between a pair of points in a of those of the earth, what would be the value of the
gravitational field. acceleration due to gravity on its surface as compared to
17. Why does a person in a satellite feels weightlessness? what it is on the earth's surface?
18. If the choice of origin is shifted what is the change in (i) 10. The mass and diameter of a planet are twice of those of
Gravitational potential (ii) Potential energy? the earth. What will be the period of oscillation of a simple
19. Can a satellite be in an orbit in a plane not passing through
pendulum on this planet, if it is a second's pendulum on
the earth’s centre? Explain your answer.
20. If earth be at one half its present distance from the sun, the earth?
how many days will there be in a year? 11. A high jumper can jump 1.5 m on earth. With the same
21. The masses M in the two figures are identical. Which of effort, how high will he be able to jump on a planet whose
the two systems has the smaller gravitational potential
energy ? Why ? 1 1
density is rd and radius th of that of the earth?
3 4
M
12. Objects at rest on the earth's surface move in circular paths
with a period of 24 hours. Are they in orbit in the same
r sense as that of a satellite of the earth? Why not? What
would be the length of the day if such objects have to
r r put in a time orbit?
M O M
M r M M 2r 13. While approaching a planet circling a distant star, a space
(a) (b) traveller determines the radius of a planet is half of that
212 Physics
of the earth. After landing on its surface, he finds its 4. A body weighs 36 kg on the surface of the earth. How much
acceleration due to gravity is twice as that of the earth. would it weigh on the surface of a planet, whose mass is 1/
Find the ratio of the mass of planet and that of the earth. 4 and radius is 1/2 of that of the earth ?
14. State universal law of gravitation. Establish the relationship 5. At what height the acceleration due to gravity above the
Me = gRe2/G where Me and Re are the mass and radius of earth’s surface would be half of its value on the surface of
the earth ? [Radius of the earth = 6400 km]
the earth respectively.
15. Objects at rest on the earth’s surface move in circular paths 6. Calculate the weight of a person at a height 6000 km above
the surface of the earth if his weight is 50 kgwt. on the
with a period of 24 hours. Are they in orbit in the same surface of the earth. [Given : Radius of the earth = 6000 km]
sense that an earth’s satellite is in orbit? Why not? What
7. At what height above the surface of the earth the
would the length of the day have to be to put such objects acceleration due to gravity will be
in a true orbit? (i) one third
16. Discuss the variation of ‘g’
(ii) one fourth of the acceleration due to gravity at the
(i) due to height (ii) due to depth. surface of the earth ? [Radius of the earth = 6400 km]
17. What are Geostationary Satellites? Calculate the height of 8. At what height above the surface of the earth does the
the orbit above the surface of the earth in which a satellite, acceleration due to gravity (a) reduces to 64% and
if placed, will appear stationary. (b) reduces by 64% of its value of the surfaces of earth.
18. State and explain Kepler’s laws of planetary motion. Derive Radius of earth = 6400 km.
Kepler’s third law. 9. How deep below the surface of earth does the acceleration
19. For a satellite orbiting in an orbit, close to the surface of due to gravity (i) reduces to 36% and (ii) reduces by 36% of
earth, to escape, what is the percentage increase in the its value on the surface of earth, radius of earth = 6400 km
kinetic energy required? 10. At what height from the surface of earth, the acceleration
due to gravity is the same as that at a depth 160 km below
Long Answer Questions [5 Marks Qs.] the surface of earth. [Radius of the earth = 6400 km]
1. Estimate the acceleration due to gravity at any point on 11. Two bodies of masses 100 kg and 1000 kg are at a distance
the surface of the earth, due to its rotation. How this value 1 m apart. Calculate the gravitational field intensity at the
changes from the equator to the poles? mid point of the line joining them.[G = 6.67 × 10–11 Nm2/kg2]
2. Derive expressions for orbital velocity, time period and 12. What is the minimum energy required to launch a satellite
altitude of a satellite. of mass m from the surface of earth of mass M, radius R in a
3. (a) Deduce Kepler's 3rd law from Newton's law of circular orbit at an altitude 3R.
gravitation. 13. A remote sensing satellite of the earth revolving in a circular
(b) Deduce Newton's law of gravitation from Kepler's orbit at a height of 250 km above earth’s surface. What is
laws. the (a) orbital velocity, and (b) Time period of revolution of
4. Six point masses of mass m each are at the vertices of a the satellite ?
regular hexagon of side l. Calculate the force on any of [Radius of the earth = 6.4 × 106 m, acceleration due to gravity
the masses. at the surface of the earth = 9.8 m/s2]
5. A satellite is to be placed in equatorial geo-stationary orbit
around earth for communication. 14. Calculate the escape velocity of the moon. [Given : Radius
(a) Calculate height of such a satellite. of the moon = 1.7 × 106 m and acceleration due to gravity at
(b) Find out the minimum number of satellites that are the surface of the moon = 1.63 m/s2]
needed to cover entire earth so that at least one 15. The distance of two planets from the sun are 1013 and 1012
satellites is visible from any point on the equator. metre respectively. Find the ratio of time periods and speed
of the two planets.
[M = 6 × 1024 kg, R = 6400 km, T = 24h, 16. A mass m is splaced at P a distance h along the normal
G = 6.67 × 10–11 SI units] through the centre O of a thin circular ring of mass M and
radius r (fig.)
Numerical Questions [3 or 5 Marks Qs.]
1. Gravitational force between a point mass m and M separated
r
by a distance is F. If a point mass 3 m is placed next to m,
what will be the (a) force on M due to m and (b) total force P
on M? o h m
2. If the distance of the earth from the sun were half the present
value, how many days will make one year?
[Given 1 year = 365 days]
3. If the radius of the earth shrinks by 5%, mass remaining M
If the mass is removed further away such that OP becomes
constant, then how would the value of acceleration due to 2h, by what factor the force of gravitation will decrease, if
gravity change ? h=r?
Gravitation 213

17. A satellite orbits the earth at a height of 500 km from its 18. A body of mass 100 kg falls on the earth from infinity. What
surface. Compute its (i) kinetic energy, (ii) potential energy, will be its velocity on reaching the earth? What will be its
and (iii) total energy. Mass of the satellite = 300 kg, Mass of K.E.? Radius of the earth is 6400 km and g = 9.8 ms–2. Air
the earth = 6.0 × 1024 kg, radius of the earth = 6.4 × 106 m, friction is negligible.
G = 6.67 × 10–11 Nm2 kg–2.

HOTS/Exemplar Questions
Very Short Answer Questions [1 Mark Qs.] 3. An object of mass m is raised from the surface of the earth
to a height equal to the radius of the earth, that is, taken
1. If you were in a car that drove off the edge of a cliff, why from a distance R to 2R from the centre of the earth. What is
would you be momentarily weightless? Would gravity still the gain in its potential energy? [Exemplar]
be acting on you? [HOTS] 4. Two identical heavy spheres are separated by a distance 10
times their radius. Will an object placed at the mid point of
2. If the moon didn’t exist, would ocean tides occur on the the line joining their centres be in stable equilibrium or
earth? If so, how often? [HOTS] unstable equilibrium? Give reason for your answer.
3. How is the gravitational force between two point masses [Exemplar]
affected when they are dipped in water keeping the
separation between them the same? [Exemplar] Long Answer Questions [5 Marks Qs.]
4. What is the angle between the equatorial plane and the
1. A playful astronaut releases a bowling ball, of mass m = 7.20
orbital plane of (a) Polar satellite? (b) Geostationary satellite?
[Exemplar] kg into circular orbit about earth at an altitude h of 350 km.
(a) What is the mechanical energy E of the ball in its orbit?
Short Answer Questions [2 or 3 Marks Qs.] (b) What is the mechanical energy E0 of the ball on the
1. If there is an attractive force between all objects, why do we launchpad at Cape Canaveral?
not feel ourselves gravitating toward massive buildings in (c) From there to the orbit, what is the change DE in the
our vicinity? [HOTS] ball’s mechanical energy? [HOTS]
2. A satellite is to be placed in equatorial geostationary orbit
2. A rocket coasts in an elliptical orbit around the Earth. To around earth for communication.
attain escape velocity using the least amount of fuel in a (a) Calculate height of such a satellite.
brief firing time, should it fire off at the apogee or at the (b) Find out the minimum number of satellites that are
perigee? [HOTS] needed to cover entire earth so that at least one satellite is
visible from any point on the equator.
Apogee [M = 6 × 1024 kg, R = 6400 km, T = 24h, G = 6.67 × 10– 11 SI
units]. [Exemplar]
3. Earth’s orbit is an ellipse with eccentricity 0.0167. Thus,
earth’s distance from the sun and speed as it moves around
the sun varies from day to day. This means that the length
of the solar day is not constant through the year. Assume
that earth’s spin axis is normal to its orbital plane and find
out the length of the shortest and the longest day. A day
should be taken from noon to noon. Does this explain
variation of length of the day during the year? [Exemplar]
Perigee
214 Physics

CHAPTER TEST

Time : 45 min. Max. Marks : 24

Directions : (i) Attempt all questions


(ii) Questions 1 to 5 carry 1 mark each.
(iii) Questions 6, 7 and 8 carry 3 marks each.
(iv) Questions 9 and 10 carry 5 marks each.

1. A spring balance is suspended inside an artificial satellite revolving around the earth. If a body of mass 5 kg is suspended
from it, what would be its reading?
2. If the earth stops rotating about its axis, by what value the acceleration due to gravity at equator will change ?
3. Earth is continuously pulling moon towards its centre. Why does not moon fall onto earth?
4. What are the factors affecting acceleration due to gravity?
5. What is weightlessness?
6. Two satellites A and B go around a planet P in circular orbits having radius 4R and R respectively. If the speed of the satellite
A is 3V, find the speed of satellite B.
7. The radius of a planet is double than that of the earth but their average densities are the same. If the escape velocity at the
planet and at the earth are Vp and Ve respectively, then prove that Vp = 2Ve
8. A geostationary satellite is orbiting the earth at a height of 6R above the surface of the earth; R being the radius of the earth.
What will be the time period of another satellite at a height 2.5 R from the surface of the earth ?
9. Three point masses, each of m, are placed at the vertices of an equilateral triangle of side a. What is the gravitational
field and potential due to this system at the centroid of the triangle?

A
a
FA
F E
G
FB FC

B a C
D
2
10. Find the potential energy of a system of four particles each of mass m placed at the vertices of a square of side L. Also
obtain the potential at the centre of the square.
Gravitation 215

Solutions
9. The value of 'g' is larger at poles than at the equator.
PRACTICE QUESTIONS \ 1 kg sugar will weigh more at poles when weighed
by a spring balance calibrated at the equator.
Very Short Answer Questions
10. Time period of satellite is given by
1. According to Newton’s law of gravitation,
( R + h)3
GM1M 2 T = 2p
F= GM
R2 Therefore, T will decrease if h decreases.
G (2 M 1 ) (2 M 2 ) 11. Since gravitational potential difference is zero therefore the
New force F ¢ = work done is zero.
( R / 2) 2 12. It will fall onto the Earth.
13. 1 Kg. wt = 9.8 newton.
G M1M 2
= 16 = 16F 14. Gravitational field intensity is defined as the force
R2 experienced per unit mass kept at that point. S.I. unit is
\ Force will become 16 times. N/Kg.
15. Very less escape velocity of 2.38 km/sec.
2
æ4 ö 16. Work done in carrying unit mass from one point to the
G ç p r 3r÷
GM2 è3 ø other is called gravitational potential difference, i.e.,
2. As we know, F = = µ r4
r2 r2 æ1 1ö
-GM ç - ÷ .
\ Force of attraction is directly proportional to fourth è r1 r2 ø
power of separation.
17. Since no reaction is experienced by the person, he feels
3. Variation in acceleration due to gravity, weightlessness.
æ 2h ö æ dö 18. (i) Remains dependent on the choice of origin.
g' = g ç1 - ÷ and g' = g ç1 - ÷ (ii) Potential energy is independent since it depends only
è Rø è Rø
on separation.
19. The centripetal force required for the orbital motion of the
R æ 2h ö æ R / 2ö
d= ; Here g ç1 - ÷ = g ç1 - ÷ satellite is provided by the gravitational force of attraction.
2 è Rø è R ø Gravitational force is a central force, i.e., it passes through
the centre of mass of the earth and the satellite. Hence the
2h 1 1 1 2h plane of orbit of the satellite has to pass through the
1– =1– = Þ1– =
R 2 2 2 R earth’s centre.
20. According to Kepler’s third law of planetary motion
1 2h R
Þ = Þh= T 2 µ r3 ,
2 R 4
4. Simple harmonic motion. 3
r T12 æ 1 ö
æ Rw 2 ö Since r ® , 2 = ç ÷
2 T è 2ø
5. If the earth stops rotating w = 0; g' = g ç1 - cos 2 a ÷
è g ø 3/ 2 3
æ 1ö æ 1 ö
Þ g' = g \ T1 = ç ÷ T = ç T
\ It increases. è 8ø è 2 2 ÷ø
6. No, since there is no gravity inside the satellite T
i.e. T1 = .
l 16 2
\ g = 0 and T = 2p = ¥.
g 21. The first system has the outer most masses closer to each
other than the second system. Hence the gravitational P.E.
of the second system is lesser than that of the first system.
GM
7. As orbital speed, v = ; therefore higher the value Short Answer Questions
R+h
of h, lesser will be the speed.
1
\ The satellite close to the surface will have larger 1. When mass is same, g µ
speed. R2

8. Q T 2 µ R3
Dg DR
\ =2
g R
\ The satellite away from the surface of the earth will
have a longer time period. Dg DR
Þ × 100 = 2 × 100 = 2 × 1% = 2%.
g R
216 Physics
2. Let M ® mass of the earth; R ® radius of the earth. 4. At any point on the surface of the earth the acceleration
B due to gravity is
æ Rw 2 ö
g' = g ç1 - cos 2 a ÷ [a = latitude]
h è g ø
For equator a = 0
A æ Rw 2 ö
For weightlessness g' = 0; g ç1 - ÷ =0
R è g ø

g
O Þw= .
R
5. The gravitational force on a unit mass due to mass M at
GM GM
Acceleration due to gravity at its surface = g = 2 a distance x is given by F =
R x2
Acceleration due to gravity at a height h above the surface To move the unit mass through a small distance dx, the
GM GM
= g' = work done = dW = Fdx = dx
2
( R + h) x2
\ To bring the unit mass from infinity to a point at a
g' GM R2 R2 distance r from M,
Dividing, = × =
g ( R + h)2 GM ( R + h)2 r
GM
R2 æ hö
-2 the total work done = W = ò dW = ò x2
dx
Þ g' = g = g ç1 + ÷ ¥
2 è Rø
2æhö r
R ç1 + ÷ é 1ù é1 1 ù
è Rø = GM ê - ú = – GM ê - ú
ë x û¥ ër ¥û
h
Expanding binomially and neglecting higher powers of x dx
R M
as h << R. O B A
æ 2h ö GM GM GM
g' = g ç1 - ÷ =– + =–
è Rø r ¥ r
3. At the surface, acceleration due to gravity, \ Gravitational potential at that point
4 GM
G. p R 3 r = – .
GM 3 4 pRrG r
g= = Þ g=
2 2 3 6. Let the escape velocity from the surface of the earth be
R R
r
[ = Mean density of the earth] ve.
B 1
\ K.E. of the body thrown with this velocity = mve 2
A d 2
To escape from the earth's gravitational field the body must
R– d have a K.E. greater than or equal to its P.E. at the surface
O of the earth.
GMm
P.E. at the surface = –
R
At a depth ‘d’ below the surface, acceleration due to gravity
To escape K.E. should be greater P.E.,
GM '
g= where M' = Mass of the sphere of radius 1 GMm
(R - d )2 mve 2 ³
2 R
(R – d) \ Minimum velocity
4 2GM é GM ù
G. p ( R - d )3 r ve = = 2gR ê g = 2 ú
3 4p ( R - d ) r G
g' = = R ë R û
2 3
(R - d )
\ Escape velocity = 2gR
g' R-d æ dö
Þ = Þ g' = g çè1 - ÷ø 2GM
g R R 7. Escape speed, ve = = 2gR = g´D
R
Gravitation 217

[D = Diameter = 2R] \ P.E. will be same


(ve )moon ( g ´ D) moon 1 1 1 ge he
= = ´ = . \ Mgphp = Mgehe Þ hp =
(ve )earth ( g ´ D)earth 6 4 4.9 gp
4
2GM G. pRe3re
8. Escape velocity = v = GM e 3 4pGRe re
R But ge = = 2 =
Re2 Re 3
2GM
Þ v2 = .......(i) ge Re re
R \ =
v2 = (2GM)R–1 gp R pr p
Differentiating both sides, we get,
1 1
dv 2GM dv GM Rp = × Re and r p = × re
2v =– Þv = ......(ii) 4 3
dR R 2 dR R2
ge
1 dv 1 \ = 12 \ hp = 12 × 1.5 = 18 m.
Dividing (ii) by (i), =– gp
v dR 2R
12. If an object is at rest on the surface of the earth then it
dv dR is not in orbital motion w.r.t. the earth. In order that this
Þ =–
v 2R object has an orbital motion close to the earth's surface
dv -dR its orbital velocity and time period will be, v = gR and
=
v 2R
2pR R
1 - dR T= = 2p
dv v g
Þ × 100 = × 100
v 2 R Putting the value of R and g we get,
T = 1.4 hr.
dv 1 \ Length of the day will be 1.4 hr. for those objects if
Þ × 100 = × 4% = 2%
v 2 they are put in a true orbit.
\ If the radius decreases by 4%, escape velocity will GM e m
increase by 2%. 13. For earth, mge =
Re2
GM GM ' 1 1 GM p m
9. g= 2
and g' = 2 where M' = M and R' = R For the planet, mgp =
R R' 2 2
Rp2
g' 1
\ = × 22 = 2 æMpö æR 2ö gp
g 2 Dividing, ç
e
.ç 2 ÷ = ;
÷ çè R p ÷ø
Þ g' = 2g. è Me ø ge
It will be twice the value of g.
Re
GM GM e Given that gp = 2ge and Rp =
10. Q g= \ ge = 2
2
R Re 2
Mp 2 1
GM ' p \ = = \ Mp : Me = 1 : 2.
and gp = Me 4 2
Rp2 14. Universal law of gravitation. Every body in this universe
Here, Mp = 2Me and Rp = 2Re attracts every other body with a force which is directly
proportional to the product of their masses and is inversely
gp 1 proportional to square of distance between them.
\ =
ge 2 m1m2

l r2
Time period of a simple pendulum = T = 2p
g Gm1m2
Te gp 1 1 F=
\ = = = r2
Tp ge 2 2
Consider earth to be a spherical body of mass Me, Radius
\ Tp = 2 Te = 2 ×2 Re with centre O. Suppose a body of mass m is placed on
surface of earth where acceleration due to gravity is ‘g’.
[Te = 2s as it is a second's pendulum] If density of earth is uniform, its mass can be supposed
Þ Tp = 2 2 sec. to be concentrated at its centre O.
11. Let he ® height the man jumps on the earth, Let F be force of attraction between earth and body.
hp ® height the man jumps on the planet According to Newton’s law of gravitation,
Q the effort is same.
218 Physics

GM e m the sun in an elliptical orbit with the sun at one focus


F= of the ellipse.
R2 (ii) Second law (Law of areas). The line joining the planet
e
to the sun sweeps out equal areas in equal intervals
From gravity pull F = mg of time.
m (iii) Third law (Law of periods). The square of the time
GM e m period (T2) of any planet about the sun is proportional
\ mg = mg Re
R2 to the cube of the planet’s mean distance from the
e sun (R3).
GM e Me O
g= T 2 µ R3
2 Assume that the planet moves in a circular orbit of
R
e radius R. The gravitational force between the sun and
gRe2 the planet provides the necessary centripetal force,
Me = therefore,
G
15. The objects on the earth’s surface are not in orbital motion mv 2
GMm
w.r.t. the earth. In order that on object has an orbital motion =
R2 R
close to the earth’s surface, its orbital velocity, and period Where M, m, R and v respectively denote the mass
of motion must be of the sun, mass of the planet, radius of the circular
v0 = gR ; orbit and orbital speed of the planet.
2 pR
2 pR R But v =
T= v = 2p = 1.4 hr. T
0 g
Therefore, the length of the day has to be 1.4 hr in case GM 4p 2 R2 4p 2 R
\ = =
the objects on the earth’s surface are in the true orbital R2 T2 ´ R T2
motion like that of the earth satellite.
æ 2h ö æ dö T2 4p 2
16. (i) g = g ç1 - ÷ (ii) g¢ = g çè1 - ÷ø or 3 = = Constant
è Rø R R GM
\ T 2 µ R3 .
19. Orbital velocity for close to earth orbits = gR . Escape
g
velocity required = 2 gR .
ve - v0
% Increase required = ´ 100
v0
O R ¥ gR ( 2 - 1)
The value of g decreases both on moving up and on = ´100 = 41%
moving down from the surface of earth. This can be shown gR
graphically.
17. A satellite which revolves around the earth with the same Long Answer Questions
angular speed in the same direction as is done by earth
around its axis is called geostationary or geosynchronous
satellite. 1. Let us consider a body of mass m at a point P on the
Velocity of such a satellite relative to earth is zero. surface of the earth.
T = 24 h Ð POE = l = latitude of point P,WE = equator.
height of geostationary satellite N

1/3
æ T 2 R2 g ö
h = çç 2 ÷
÷ -R R' l
P V
Fc
è 4p ø mg H
R = 6.4 × 106 m W l E
g = 9.8 m/s2 O
T = 24 hr = 24 × 60 × 60 sec
1/ 3
æ (86400)2 ´ (6.4 ´ 106 )2 ´ 9.8 ö
h= ç ÷ -6.4 ´ 106
è 4 ´ (3.14) 2 ø S
Due to earth's rotation the particle moves with an angular
h = 3.6 × 107 m = 36000 km.
18. Kepler’s laws of planetary motion. velocity w in a circle of radius PM.
(i) First law (Law of orbits). Each planet revolves round PM = Rcosl = R¢(say)
Gravitation 219

The various forces acting on the particle P are: Time period =


(a) weight mg along PO.
Distance travelled is one revolution
(b) Centrifigual force Fc trying to lift the mass away. Fc T=
can be resolved into the vertical and horizontal orbital velocity
components V and H respectively.
Q R¢ = R cos l, Fc = mw2R¢, V = Fc cos l 2πr 2pr R+h
= T= =
Since direction of H is almost perpendicular to the direction v R g
of mg.
Hence component of H along the direction of mg is 2π ( R + h)3
negligible. T= ....... (2)
R g
If g¢ be the final gravitational acceleration at point P (due
to the rotation of earth).
4 3
Then mg¢ = mg – v = mg – Fc cosl Mass of the earth = M = pR r ;
3
= mg – mw2 R¢ cosl
GM G 4 pGRr
\ g¢ = g - w2 R¢ cos l = g – w2Rcos2l g= 2
= 2
. 4 pR 3 r =
g¢ = g – Rw2cos2l R R 3 3
At the poles, l = 90° \ cosl = 0
gpoles = g 2p 3( R + h)3 3p ( R + h) 3
T= = ;
\ Acceleration due to gravity is maximum at poles and R 4pGRr Gr R 3
minimum at equator due to rotation of the earth.
At the equator l = 0 \ cos l = 1 3p
If h << R, then T = ;
æ Rw 2ö Gr
gequator = g ç 1 - ÷ = g – w2R
è g ø
2 p R3 R
2. Let M = Mass of the earth; R = Radius of the earth; m From (2) T = = 2p
R g g
= Mass of the satellite
v = orbital velocity of the satellite; h = altitude of the
satellite 4 p 2 ( R + h )3
Altitude: Squaring eqn. (2) we get T 2 =
r = R + h = orbital radius of the satellite. The centripetal R2 g
force required by the satellite is provided by the
gravitational force of attraction between the earth and the T 2 R2 g
satellite Þ ( R + h)3 =
4p 2
2
mv GMm GM
\ = Þ v= 1/ 3
r r2 r æ T 2 R2 g ö
Þ R+h = ç ÷
GM è 4p 2 ø
But g = \ GM = gR2
R2
1/3
æ T 2 R2 g ö
gR 2 Þ h=ç ÷ -R.
\ v= ç 4p 2 ÷
R+h è ø
3. (a) Let m = mass of the planet, M = mass of the sun, r
= radius of the orbit of the planet w = constant angular
h velocity of the planet. The centripetal force on the
Earth planet required for its orbital motion is provided by
r Satellite the gravitational force of attraction between the
R planet and the sun.

GMm
\ mw 2 r =
r2
g æ 4p 2 ö GMm é 2p ù
v=R .....(1) Þ m çç 2 ÷÷ = 2
R+h T r êQ w = T ú
è ør ë û
If h << R then v = gR
220 Physics

æ 4p 2 ö 3 é 4p 2 ù
2 2 3 êQ = constant ú Gm 2 Gm2
\ T =ç ÷ r \ T µ r = 3= .
è GM ø êë GM ûú 3l 2 3l 2

2p Gm2
(b) w= = angular velocity of the planet .
T Force due to mass M at D =
4l 2
\ Centripetal force on the planet
æ 2p ö 4p 2 mr
2
Gm2 é 1 1ù
= F = mw2r = mr ç ÷ = \ Total Force = 2 ê
1+ + ú.
èT ø T2 l ë 3 4û
2 3
According to Kepler's 3rd law, T µ r
1/ 3
2 3 æ GMT 2 ö
\ T = kr 5. (a) r = ç q
[k = constant] 2 ÷
è 4p ø
E
4p 2 mr 4p 2 m
\ F= = ´ 1/ 3
kr 3 k r2 æ GMT 2 ö
\h= ç 2 ÷ –R
m è 4p ø
\ Fµ
r2 = 4.23 × 107 – 6.4 × 106 = 3.59 × 107 m.
Q The gravitational force of attraction between the
sun and the planet in motion also depends on M. æ ö
-1 æ R ö -1 1
(b) q = cos ç ÷ = cos ç ÷
è R + hø ç1+ h ÷
4p 2 è r ø
\ F is also proportional to M. \ µM
k
-1 æ 1 ö
4p 2 GMm = cos ç ÷ = 81° 18¢
Þ = GM \ F= è 1 + 5.61 ø
k r2
\ 2q = 162° 36¢
4. AB = BC
360°
3 » 2.21; Hence minimum number = 3.
(AC) = 2 AG = 2.l. = 3l 2q
2
Numerical Questions
l l
AD = AH + HJ + JD = + l + . = 2l 1. (a) Force on M due to m remains same as
2 2
GMm
F=
F A R2
(b) Total force on M = force on M due to m
l + force on M due to 3m
H
GMm GM .3m GMm
E G B = 2
+ 2
= (1 + 3)
r r r2
J GMm
=4 = 4F
r2
D C
GMm mv 2
2. Force (F) = =
AE = AC = 3l , AF = l R2 R
Force along AD due to m at F and B
GM 2 pR R3
Þ v= ; T= = 2p = 365
2é 1 ù1 2é 1 ù1 Gm 2 R v GM
= Gm ê 2 ú + Gm ê 2 ú = 2
ël û 2 ël û 2 l
( R / 2)3 T 365
Force along AD due to masses at E and C T ' = 2p = = = 129 days.
GM 8 8
1 Gm2 3. Acceleration due to gravity
2
= Gm cos (30°) + cos (30°)
3l 2 3l 2 GM
g= , If R decreases, g increases.
R2
Gravitation 221

Taking logarithm on both sides, 64 64


log g = log G + log M – 2 log R \ d= ´R= ´ 6400 km
100 100
dg 2dR
Differentiating, = 0+ 0- . When radius shrinks by æ d ö 64 d 64
(b) g¢ = g ç1 - ÷ = g
è R ø 100 Þ R = 1 - 100
g R
dR 5
5%, then =- 36 36 ´ 6400
R 100 Þ d= ´R= = 2304 km.
100 100
dg 2dR 10. As we know,
\ ´ 100 = - ´ 100
g R æ dö
g ' = g ç1 - ÷
æ -5 ö è Rø
= -2 ç ´ 100 = 10%
è 100 ÷ø
æ 2h ö
= g ç1 - ÷
GM G M /4 GM GM è Rø
4. As we know, g = ; g' = = =
R2 ( R / 2) 2 R2 R2 d 2h d 160
4. Þ = Þ h= = = 80 km
4 R R 2 2
\ g'=g 11. Gravitational field intensity
\ weight remains same. Gm2 Gm1
2 I= -
2
æ hö ( r / 2) ( r / 2) 2
5. With height value of g, g ' = g / ç1 - ÷
è Rø
é 1000 100 ù
2 1 R 2 = 6.67 × 10–11 ê - ú
\ g' R êë (0.5)
2
(0.5) 2 úû
= Þ =
g ( R + h) 2 2 ( R + h) 2
6.67 ´ 1011 ´ 900
Þ R+h= 2R = = 2.4 × 10–7 N/kg
0.25
Þ h = ( 2 - 1) R = (1.414 – 1) × 6400 = 2651 km 12. Energy of the satellite on the surface of the earth =
mgR 2 50 ´ 60002 50 GMm
6. Weight W = mg ' = = = E1 = -
( R + h) 2
(6000 + 6000) 2 4 R
Energy of the satellite at a distance (3R + R) = 4R from the
= 12.5 kgwt
GMm 1 2
7. (i) 4685 km (ii) 6400 km centre = E2 = - + mv
4R 2
64 gR 2 GMm 1 GM GMm
8. (a) g ' = g= =- + m =-
100 ( R + h)2 4R 2 4R 8R
\ Energy required to launch the satellite
8 R
Þ = Þ 8R + 8h = 10 R Þ 8h = 2R GMm æ GMm ö
10 R + h = E2 – E1 = - -ç-
è ÷
R 6400
8R R ø
Q h= = = 1600 km
4 4 GMm GMm 7GMm
= - =
2 R 8R R
36 gR
(b) g ' = 66% g = g=
100 ( R + h) 2 13. (a) Orbital velocity, V = R
g
R+h
6 R 4R 2
\ = Þ h= = R
10 R + h 6 3 9.8
= 6.4 × 106 = 7756.6 m/s
2 ´ 6400 6.4 ´10 + 0.25 ´ 106
6
= = 4266.7 km.
3
9. With depth acceleration due to g ravity g varies as 2p( R + h)
(b) Time period of revolution T =
V
æ d ö 36
(i) g ' = g ç 1 - ÷ = g
è R ø 100 2 ´ 3.14(6400 + 250) ´ 103
= = 5373 s.
æ dö 36 d 36 64 7756.6
Þ çè1 - ÷ø = Þ = 1- =
R 100 R 100 100
222 Physics
14. Escape velocity
HOTS/EXEMPLAR QUESTIONS
6
Ve = 2 g M RM = 2 ´ 1.63 ´ 1.7 ´ 10 Very Short Answer Questions
= 2.354 × 103 m/s 1. I would be momentarily in weightless situation because
the car is falling as fast as the person sitting in the car.
T12 R13 Thus, all the effects of gravity are eliminated. This gives
15. From Kepler’s law, =
T22 R23 the sensation of weightlessness. Gravity still acts on
me but it is not felt as weight as there is no supporting
force.
T12 (1013 )3
Þ = = (10)3 2. Yes, the earth’s tides would be due only to the sun. They
T22 (1012 )3 would occur twice per day (every 12 hours instead of every
12.4 hours) due to the earth’s daily rotation.
T1 3. It remains same as the gravitational force is independent of
Þ = (10)3/ 2 = 10 10
T2 the medium separating the masses.
4. (a) 90° (b) 0°
V1 2pR1 / T1 R T
Ratio of speeds = = 1´ 2 Short Answer Questions
V2 2pR2 / T2 R2 T1
1. Gravity pulls us to massive buildings and to everything
1013 1 1 else in the universe. Physicist Paul A.M. Dirac, winner of
= ´ =
1012 10 10 10 the 1933 Nobel prize for physics, put it this way. “Pick a
16. Only the horizontal component (i.e. along the line joining m flower on earth and you move the farthest star” How much
and O) will survive. The horizontal component of the force we are influenced by buildings or how much interaction
on any point on the ring changes by a factor : there is between flowers and stars is another story. The
forces between us and buildings are relatively small
é 2r ùé m ù 4 2 because their masses are small compared with the mass of
ê 2 2 3/ 2
úê 2 ú
2 3/ 2 = . earth. The forces due to the stars are also small because of
ëê (4r + r ) ûú ëê (r + r ) ûú 5 5
their great distances from us. These tiny forces escape
17. Here h = 500 km = 500 × 103 m, our notice when they are overwhelmed by the
m = 300 kg, M = 6.0 × 1024 kg overpowering attraction to earth.
R = 6.4 × 106 m, G = 6.67 × 10–11 Nm2 kg–2
2. In accord with the work-energy formula Fd = DKE , for a
(i) Kinetic energy
constant thrust F, the maximum change in KE will occur
é when d is maximum. The rocket will travel the greatest
1 2 1 GM GM ù
= mv = m. êQ v = ú distance d during the brief firing time where it is traveling
2 2 R+h êë R + h úû fastest –at the perigee.

1 300 ´ 6.67 ´ 10-11 ´ 6.0 ´ 1024 Fd = D KE


= ´ d
2 6.4 ´ 106 + 500 ´ 103
= 8.7 × 109 J
(ii) Potential energy
GMm 6.67 ´ 10-11 ´ 6.0 ´ 1024 ´ 300
=- =
( R + h) 6.4 ´ 106 + 500 ´ 103
= – 17.4 × 109 J.
(iii) Total energy d
= K.E. + P.E. = 8.7 × 109 – 17.4 × 109 Fd = D KE
= – 8.7 × 109 J.
This can be seen also by considering the relative KEs
18. A body thrown up with escape velocity ve reaches infinity.
Hence a body falling on the earth from infinity should come given to the exhaust gases at the perigee and apogee. At
back with velocity ve given by the apogee where the rocket coasts slower, much more
KE of the system goes to gases, whereas at the perigee
ve = 2 gRe = 2 ´ 9.8 ´ 6400 ´ 103 most of the KE is associated with the rocket. (If the orbital
= 11.2 × 103 ms–1 = 11.2 kms–1. speed = rocket exhaust speed, the gases are motionless
with respect to the Earth and the rocket gets 100% of the
1 2 1
K.E. = mve = ´ 100 ´ (11.2 ´ 103 )2 = 6.27 × 109 J. KE.)
2 2 3. mgR/2
Gravitation 223

4. Consider moving the mass at the middle by a small amount h to DE = E - E0 = (–214 MJ) – (–451MJ) = 237 MJ.
10R
1/ 3
æ GMT 2 ö
r=ç ÷
the right. M m R M 2. (a) ç 4 p2 ÷
è ø
h
1/ 3
GMm GMm æ GMT 2 ö
\ h = ç ÷ -R
Then the forces on it are: to the right and to ç 4p 2 ÷
(R - h) 2 ( R + h )2 è ø
the left. The first is larger than the second. Hence the net force will = 4.23 × 107 – 6.4 × 106
also be towards the right. Hence the equilibrium is unstable. = 3.59 × 107 m

æ R ö
(b) q = cos -1 ç ÷ q
Long Answer Questions èR+hø E
1. (a) The key idea here is that we can get E from the orbital
energy, given by (E = –GMm/2r), If we first find the
orbital radius r. That radius must be -1 æ 1 ö
= cos ç h÷
r = R + h = 6370 km + 350 km = 6.72 × 106 m, ç1+ ÷
è rø
in which R is the radius of Earth. Then, the mechanical
energy is
-1 æ 1 ö
GMm = cos ç ÷ = 81°18¢
E=- 1 +
è 5.61 ø
2r
\ 2q = 162°36¢
(6.67 ´ 10-11 N .m2 / kg 2 )(5.98 ´1024 kg )(7.20kg ) 360°
=- » 2.21 ; Hence minimum number = 3.
6
(2)(6.72 ´10 m) 2q
3. Angular momentum and areal velocity are constant as earth orbits
8 the sun.
= –2.14 ´ 10 J = –214 MJ.
2 2
(b) The key idea here is that, on the launchpad, the ball At perigee rp wp = ra wa at apogee.
is not in orbit and thus (E = – GMm/2r) does not If ‘a’ is the semi-major axis of earth’s orbit, then rp = a(1 – e) and
apply. Instead, we must find E0 = K0 + U 0 , where ra = a(1 + e).
wp 2
K 0 is the ball’s kinetic energy and U 0 is the æ 1+ eö
\ =ç ÷ , e = 0.0167
gravitational potential energy of the wa è 1 - e ø
ballearth system. To find U 0 , wp
\ = 1.0691
wa
GMm
U0 = - Let w be angular speed which is geometric mean of wp and wa and
R corresponds to mean solar day,

1 4
(6.67 ´ 10 -11 N .m 2 / kg 2 )(5.98 ´ 1024 kg )(7.20kg ) æ wp öæ w ö
=– P s A
6.37 ´ 106 m \ çç w ÷ç
÷ w ÷ = 1.0691 rp ra
è øè a ø 2 3
= –4.51´108 J = -451MJ .
wp w
The kinetic energy K0 of the ball is due to the ball’ss \ = = 1.034
w wa
motion with earth’s rotation. You can show that K 0
If w corresponds to 1° per day
is less than 1 MJ, which is negligible relative to U 0 . (mean angular speed), then
Thus, the mechanical energy of the ball on the wp = 1.034 per day and wa = 0.967 per day. Since 361° = 14 hrs:
launchpad is mean solar day, we get 361.034° which corresponds to 24 hrs
8.14¢¢ (8.1¢¢ longer) and 360.967° corresponds to 23 hrs 59 min
E0 = K 0 + U 0 » 0 - 451MJ = – 451 MJ
52¢¢ (7.9¢¢ smaller).
(c) The increase in the mechanical energy of the ball This does not explain the actual variation of the length of the day
from launch pad to orbit is during the year.
224 Physics
CHAPTER TEST uuur uuur uuur Gm Gm 3Gm
| FA | = | FB | = | FC | = 2 = =
1. The reading of the spring balance inside the satellite will be r ( a / 3) 2
a2
zero as acceleration due to gravity doesn’t exist in the
Resultant of FB and FC, FBC
satellite.
2. If the earth stops rotating about its axis, the acceleration = FB 2 + FC 2 + 2 FB FC cos120°
due to gravity increases by Rw 2 where R is the radius of
2 2 .2
the earth and w is the angular velocity of rotation of the æ 3Gm ö æ 3Gm ö æ 3Gm ö 2GM
earth.
= çè 2 ÷ø + çè 2 ÷ø + 2 çè 2 ÷ø cos120° =
a a a a2
3. Because the gravitational pull of the earth on the moon
provides the necessary centripetal force for its orbital 3Gm 3Gm
motion. \ Resultant of FBC and FA = -=0
2
a a2
4. Factors effecting acceleration due to gravity are
(i) Rotation of earth about its own axis (as they are equal and opposite)
(ii) Effect of altitude Gravitational potential at G due to masses at A, B and C
(iii) Effect of depth -Gm 3Gm
5. Weightlessness is a situation in which the effective weight each = =-
r a
of the body becomes zero.
\ Total gravitational potential at
GM GM
6. Here, VB = and V A = 3V = 3Gm -3 3Gm
R 4R G = -3 ´ = .
a a
V A 3V
\ = = 1/ 2 \ VB = 6 V L
VB VB A B
10.
7. Escape velocity,
2GM 2G 4 3 8p O
Ve = = ´ pR r = R Gr L L
R R 3 3

Vr Rr 2Re
\ = = =2 \ Vr = 2Ve
Ve Re Re D C
L
8. From Kepler’s third law of planetary motion,
T 2 a R3 Þ T a R 3/ 2 AC = BD = L 2
3/ 2 L
T2 æ R2 ö \ OA = OB = OC = OD =
\ = 2
T1 çè R1 ÷ø
Total P.E. of the system

æR ö
3/ 2
æ 2.5R + R ö
3/ 2 æ Gmm ö æ -Gmm -Gm ´ m ö
= ç- + +
Þ T2 = T1 ç 2 ÷ = 24 ç
è 6R + R ÷ø è AB ÷ø çè BC AC ø
÷
è R1 ø
æ Gm ´ m -Gm ´ m -Gm ´ m ö
æ1ö
3/ 2 +ç- + + ÷
è AD BD CD ø
= 24 ç ÷ = 6 2 hour
è2ø
9. From figure, AB = BC = CA = a 4Gm2 2Gm2 2Gm2 æ 1 ö
=- - =- ç2+ ÷
L 2 L è 2ø
2
æ aö 3a 2 3
Median = AD = BE = CF = a2 - ç ÷ = = a
è 2ø 4 2 Total Gravitational potential at
A median is divided into 2 : 1 ratio at the centroid. 4Gm -4Gm Gm
O=V=– = = -4 2 .
1 3 a OA L/ 2 L
\ GD = GE = GF = ´ a= ;
3 2 2 3
a a
AG = BG = CG = 2 ´ =
2 3 3

¿¿¿
9 Mechanical Properties
C ha p t e r of Solids

ELASTICITY AND PLASTICITY


The property of a material by which
it tries to regain its original configuration after the removal of the deforming force applied to it is called elasticity. When a material
is deformed, the atoms or molecules are displaced from their equilibrium position causing a change in interatomic distance but
the interatomic forces try to bring them back to the equilibrium position thereby regaining its shape and size. And if after the
removal of applied force body three retains its deformed shape then this property is called plasticity.
According to degree of elasticity, solids are of three types. The degree of elasticity depends on the intermolecular force.
(i) Perfectly elastic solids : These are solids which come back to their original configuration after the removal of deforming
force immediately and completely e.g. quartz, phosphor, bronze, steel etc.
(ii) Perfectly inelastic/plastic solids : These are solids which do not come back to their original configuration at all after the
removal of deforming force. They have a permanent set in their configuration e.g. bakelite, mud and Putty. Putty and mud
are close to ideal plastics.
(iii) Partly elastic solids: They partly regain their original configuration after the removal of deforming force. Mostly all solids
are partly elastic.
ELASTIC BEHAVIOUR OF SOLIDS
In a solid, each atom or molecule is surrounded by neighbouring atoms or molecules. These are bonded together by interatomic and
intermolecular forces and stay in a stable equilibrium position. The force between atoms of an element is called interatomic force. The
force between molecules of a compound (or element) is called intermolecular force. These forces are electrical in nature. Depending on
the distance between the atoms, this force may be attractive or repulsive in nature. These forces are responsible for the definite size
or shape of a solid.

If any ball from its equilibrium position is displaced, the spring system tries to restore the ball back to its original position.

STRESS AND STRAIN


Stressess
When a deforming force is applied on a body, it changes the configuration of the body by changing the normal positions of the
molecules or atoms of the body. As a result, an internal restoring force comes into play, which tends to bring the body back to its
original configuration. This internal restoring force acting per unit area of a body is called stress.
i.e., Stress = Restoring force/ Area
Its SI unit is N/m2.
226 Physics
Types of Stress
(i) Normal or longitudinal stress : If area of cross-section of a rod is A and a deforming force F is applied along the length of the
rod and perpendicular to its cross-section, then in this case stress produced in the rod is known as normal or longitudinal
stress.
Fn
Longitudinal stress =
A

Longitudinal stress is of two types


(a) Tensile stress : When length of the rod is increased on application of deforming force over it, then stress produced in rod
is called tensile stress.
(b) Compressive stress : When length of the rod is decreased on application of deforming force, then the stress produced is
called compressive stress.
(ii) Volumetric stress : When a force is applied on a body such that it produces a change in volume and density and shape
remaining same
1. at any point, the force is perpendicular to its surface.
2. at any small area the magnitude of force is directly proportional to its area.
Then force per unit area is called volumetric stress.
Fv
Volumetric stress =
A

(iii) Shearing or tangential stress : When the force is applied tangentially to a surface, then it is called tangential or shearing
stress.
Ft
Tangential stress =
A
It produces change in shape, volume remaining same.
Strain
When a deforming force is applied on a body, there is a change in the configuration of the body. The body is said to be strained or
deformed. The ratio of change in configuration to the original configuration is called strain.

i.e., Strain =
Change in configuration
Original configuration

Strain being the ratio of two like quantities, has no units and dimensions.
Types of Strain
(i) Longitudinal strain : It is defined as the change in length per unit original length of the body under deformation by the external
force. Thus,
Change in length l
Longitudinal strain = F F
Original length

Dl
or e=
l
It is of two types :
(a) Tensile strain : If on applying a deforming force, there is an increase of Dl in length of a rod, then strain produced in the

rod is called tensile strain. F F


1+ Dl
(b) Compressive strain : If on applying a deforming force there is decrease of Dl in length of a rod, then strain produced in
l
the rod is called compressive strain. F F
1– Dl
Mechanical Proper ties of Solids 227

(ii) Volumetric strain : It is defined as the change in volume per unit original volume of the body under deformation by the external
force.

Change in volume
Volumetric strain =
Original volume

DV
or ev =
V
x
(iii) Shearing strain : This type of strain is produced when a shearing stress is present.
It is defined as the angle in radians through which a plane perpendicular to the fixed surface q
q
x Lq
of the cubical body is turned under the effect of tangential force, f =
L
Materials behave differently under stress. When dropped, a glass tumbler shatters into pieces, a rubber ball
deforms then bounces back and a metal suffers dents.
Illustration 1 :
A wire is made of a material of density 10 g/cm3 and breaking stress 5 × 109 N/m2. What length of the wire will break under its own
weight when suspended vertically?
Sol. Let L be the required length. Then weight of wire = LArg where r is the density of the material.

force 5 ´ 109
Stress = = Lrg = 5 × 109 N/m2 \ L = = 5 × 104 m
area 10 ´ 103 ´ 10

Illustration 2 :
A bar of cross section A is subjected to equal and opposite tensile forces at its ends. Consider a plane section of the bar whose
normal makes an angle q with the axis of the bar.

(a) What is the tensile stress on the plane ?


(b) What is the shearing stress on this plane ?
(c) For what value of q is the tensile stress maximum ?
(d) For what value of q is the shearing stress maximum ?
Sol. (a) The resolved part of F along the normal is the tensile force on this plane and the resolved part parallel to the plane is the
shearing force on this plane.
force F cos q F 2
\ Tensile stress = = = cos q
area A sec q A
( area of cross section = A sec q)
force F cos q = F sin 2q
(b) Shearing stress = =
area A sec q 2A
(c) Obviously tensile stress on the plane is maximum when cos2q is maximum, that is, cos q = 1 or q = 0°.
(d) Obviously shearing stress is maximum when sin 2q is maximum, that is sin 2q = 1 or 2q = 90° or q = 45°
228 Physics
HOOKE’S LAW
Elastic limit is the upper limit of deforming force up to which, if deforming force is removed, the body regains its original form
completely and beyond which if deforming force is increased, the body loses its property of elasticity and gets permanently
deformed.
Within the elastic limit, the extension of an elastic body is directly proportional to the force or stress is proportional to strain,
i.e., Stress µ strain or, stress = E × strain
stress
or, = constant = E
strain
This constant E is known as modulus of elasticity or coefficient of elasticity. It depends upon the nature of the materials.
On the removal of the deforming force even within the elastic limit, only very few solids regain their original condition. They take
some time to regain the original state. This delay in recovering back the original condition, on the removal of the deforming force is
called elastic after-effect. Quartz, silver, gold and phosphor bronze are exceptions. Glass has a large elastic after-effect.
Elastic materials get tired or fatigued when subjected to repeated strain. Lord Kelvin observed that the vibrations of a wire which was
kept vibrated continuously for a longer time died away much faster than that of a fresh wire. The continuously vibrated wire gets
tired or fatigued and this is called elastic fatigue. A knowledge of elastic fatigue is essential in estimating the stresses that the
various parts of machinery can bear, from the point of view of safety.
STRESS-STRAIN CURVE
The stress-strain graph of a ductile metal is shown in figure. Initially, the stress-strain graph is linear and it obeys the Hooke’s law
upto the point P called the proportional limit. After the proportional limit the graph is non-linear but it still remains elastic upto the
yield point Y where the slope of the curve is zero. At the yield point the material starts deforming under constant stress and it behaves
like a viscous liquid.

Elastic Proportional limit


zone Plastic zone
U
108
Y B
P Fracture point
Stress Proportional
–2 point limit
(Nm )
Elastic
behaviour
0.01 0.2 Strain

The yield point is the beginning of the plastic zone. After the yield point, the material starts gaining strength due to excessive
deformation and this phenomenon is called strain hardening. The point shows the ultimate strength of the material. It is the
maximum stress that the material can sustain without failure. After the point the curve goes down towards the breaking point because
the calculation of the stress is based on the original cross-sectional area whereas the cross-sectional areas of the sample actually
decrease. There are some materials which break as soon as the stress is increased beyond the elastic limit. They are called brittle, e.g.
glass, ceramics etc.
Elastic limit: The maximum stress upto which a body can regain its original configuration after the removal of deforming force
is called elastic limit or yield point.
Elastic hysteresis : During loading unloading of a material the stress-strain curve do not superimpose itself. Some energy is lost
as heat during the loading-unloading process. The phenomenon of the stress-strain curve not retracing its path while unloading
is called elastic hysteresis.
Mechanical Proper ties of Solids 229

Loading
B

Unloading
A

Stress
C

O Strain
Elastomers - Substances like tissue of aorta, rubber etc. which can be stretched to cause large strains are called elastomers.

ELASTIC MODULUS
The modulus of elasticity or coefficient of elasticity of a body is defined as the ratio of the stress to the corresponding strain
produced.
Types of Modulus of Elasticity
(i) Young’s modulus of elasticity: It is the ratio of longitudinal stress to longitudinal strain within the elastic limit.
longitudinal stress (s )
Young's modulus Y=
longitudinal strain ( e )
s F/A Fl
or Y= = =
e Dl / l ADl
Its S.I. unit N/m2
(ii) Bulk or volume modulus of elasticity: It is the ratio of hydraulic stress to volume strain.
hydraulic stress DP
Bulk modulus B = Þ B=-
volume strain DV / V
The reciprocal of the bulk modulus is called compressibility and is denoted by K. It is defined as the fractional change
in volume per unit increase in pressure.
1 æ 1 ö æ DV ö
Compressibility K = = -ç ´
è DP ÷ø çè V ÷ø
B
Also B (gases) < B (liquids) < B (solids)
(iii) Shear modulus or modulus of elasticity: It is the ratio of tangential stress to shearing strain.
tangential stress ss F / A F FL
Modulus of rigidity or shear modulus γ < = = = = .
shearing strain es q Aq ADL

Y
Modulus of rigidity h is generally less than Young’s modulus. For most materials, h »
3
Relation between Y, B, h and s
(i) Y = 3B (1–2s)
(ii) Y = 2h (1 + s)
3B - 2h
(iii) s =
2h + 6B

9 1 3
(iv) = +
Y B h
Elastic Potential Energy
1
It is the work done in stretching a wire = ´ stress ´ strain ´ vol.
2

1 1 1 (stress)2
\ Elastic P.E./Vol = ´ stress ´ strain = ≥ Young 's modulus ≥ (strain) 2 = ´
2 2 2 Young's modulus
230 Physics
Poisson's Ratio
The ratio of lateral to longitudinal strain is called Poisson's ratio.

lateral strain (b ) Dd / d
Poisson’s ratio, s = =
longitudinal strain (a ) Dl / l l
l + Dl
Value of s lies between 0 and 0.5

Illustration 3 :
A 4 m long copper wire with diameter 0.5 mm is used to support a 5 kg weight. Determine the elongation in the wire. Young's
modulus for copper is 1 × 1011 N/m2 and g = 9.8 m/s2
Sol. Here, F = mg = 5 × 9.8 N; l = 4m , Y = 1 ´ 1011 N/m2
Fl \ Fl
Y= Dl =
ADl AY
2
pd 2 æ 0.5 ´10 -3 ö
A= pr2 = = 3.14 × ç ÷
4 ç 2 ÷
è ø
5 ´ 9.8 ´ 4
\ Dl = = 9.99 × 10–3 m
2
æ 0.5 ´ 10-3 ö 11
3.14 ´ ç ÷ ´ 1 ´ 10
è 2 ø
Illustration 4 :
A composite wire of uniform diameter 3 mm consisting of a copper wire of length 2.2 m and a steel wire of length 1.6m stretches
under a load by 0.7 mm. Calculate the load, given that the Young's modulus for copper is 1.1 × 1011 Pa and for steel is 2 ×
1011 Pa.
3
Sol. Given: r = mm = 1.5 × 10–3m, lCu = 2.2 m, lsteel = 1.6 m
2
DlCu + Dlsteel = 0.7 mm = 0.7 × 10–3 m .... (1)
YCu = 1.1 × 1011Pa, Ysteel = 2 × 1011 Pa
Force (F) and area of cross-section (A) is same for both
F
\ Stress = is same for both.
A
F Y Dl Y Dl Y Dl
Q = \ Cu Cu = steel steel
A l lCu lsteel
DlCu Y l 2 ´ 1011 2.2 5
\ = steel ´ Cu = ´ = .... (2)
Dlsteel YCu lsteel 1.1 ´ 1011 1.6 2
Solving (1) and (2) we get,
D lCu = 5 × 10–4m and Dlsteel = 2 × 10–4 m
Y ´ A ´ Dl Y ´ pr 2 ´ D l 22 2 ´10 -4
\ F= = = 2 × 1011 × × (1.5 ×10–3)2 × = 176.8 N
l l 7 1. 6
Illustration 5 :
A square lead slab of side 50 cm and thickness 5 cm is subjected to a shearing force on its narrow face of magnitude 9 × 104
N. The lower edge is riveted to the floor. How much is the upper edge displaced if the shear modulus of lead is 5.6 × 10 9 pa?
F Q
P P¢ Q'

50 cm
q

S R
Mechanical Proper ties of Solids 231

Sol. Given :
F = 9 × 104N, h = 5.6 × 109 Pa, A = 50 × 10– 2 × 5 × 10 –2 = 250 × 10–4 m2
F/A F 9 ´ 104
h= Þ q= = Þ q = 6.4 × 10–4 radian
q Ah 250 ´ 10-4 ´ 5.6 ´ 109
PP '
From the fig. q is small, [ q = tanq = Þ PP' = PS × q
PS
\ Displacement of the upper edge = PS × q = 50 × 10–2 × 6.4 × 10–4 = 3.2 × 10–4 m = 0.32 mm
Illustration 6 :
When the load on a wire is increased slowly from 3 to 5 kg wt., the elongation increases from 0.61 to 1.02 mm. How much
work is done during the extension of the wire? Find the value of the Young's modulus of the material of the wire, if it is 1
m long and has a cross-sectional area 0.4 mm2.
1
Sol. Work done during the extension of the wire = W = ( F1Dl1 - F2 Dl2 )
2
1
= [5 × 9.8 × 1.02 × 10– 3 – 3 × 9.8 × 0.61 × 10–3] = 1.61 × 10–2 J.
2
Fl 3 ´ 9.8 ´ 1
Y= = = 1.2 × 1011 N/m2
ADl -6
0.4 ´ 10 ´ 0.61´ 10 -3

APPLICATIONS OF ELASTIC BEHAVIOUR OF MATERIALS


(1) The bridges are designed in such a way that they do not bend much or break under the load of heavy traffic, force of strongly
blowing wind and its own weight. So the beams used in constructions of bridges, as supports are made in shape of I.
(2) The metallic parts of the machinery are never subjected to a stress beyond elastic limit, otherwise they will get permanently
deformed.
(3) The thickness of the metallic rope used in the crane in order to lift a given load is decided from the knowledge of elastic limit of
the material of the rope and the factor of safety.
232 Physics
Mechanical Proper ties of Solids 233

Textbook Exercises
9.1 A steel wire of length 4.7 m and cross-section 3 × 10– The graphs are drawn to the same scale.
5 m2 stretches by the same amount as a copper wire of (a) Which of the materials has the greater Young’s
length 3.5 m and cross-section 4 × 10–5 m2 under a given modulus?
load. What is the ratio of the Young's modulus of steel (b) Which of the two is the stronger material ?
to that of copper? Stress
Sol. (a) Young’s modulus =
Sol. For steel wire, A1 = 3 × 10–5 m2, l1 = 4.7m ; Strain
= Slope of stress-strain curve
For copper wire, A2 = 4 × 10–5 m2, l2 = 3.5m Here slope is greater for A than B.
As Dl1 = Dl2 = Dl and F1 = F2 = F Hence A has greater Young’s modulus.
(b) For stronger material, the stress required for fracture
F1l1 F 4.7 is greater. Hence A is stronger than B.
\ Y1 = = ´ ;
A1Dl1 -5 Dl 9.4 Read the following two statements below carefully and
3 ´ 10
state, with reasons, if it is true or false.
F2l2 F ´ 3.5 (a) The Young’s modulus of rubber is greater than that
Y2 = =
A2 Dl2 4 ´ 10-5 Dl of steel.
(b) The stretching of a coil is determined by its shear
Y1 4.7 ´ 4 ´ 10-5 18.8 modulus.
\ = = » 1.8
Y2 3.5 ´ 3 ´ 10 -5 10.5 Sol. (a) False, because for a given amount of strain, the stress
required will be greater for steel than rubber.
18 9 (b) True, because a coil can be stretched only by the
\ Y1 : Y2 = =
10 5 application of a tangential stress.
9.2 Figure shows the strain-stress curve for a given material. 9.5 Calculate the elongation of the steel and brass wire in the
What are (a) Young’s modulus and (b) approximate yield adjacent figure. Unloaded length of steel wire is 1.5 m and
strength for this material? of brass wire is 1m, diameter of each wire = 0.25 cm.
Young's modulus of steel is 2 × 1011 Pa and that of brass
is 0.91 × 1011 Pa.
300
1.5 m Steel
250
Stress (10 Nm )
–2

200
4 kg
6

150 1 m Brass
100
6 kg
50
Sol. For steel wire: Total force = F1 = (4 + 6) kgwt. = 10 kg wt
0 = 10 × 9.8N
O 0.001 0.002 0.003 0.004
0.25
Strain l1 = 1.5m , r1 = cm = 0.125 × 10–2 m;
2
Sol. (a) Young’s modulus
Y1 = 2 × 1011 Pa, Dl1 = ?
150 ´ 106 Nm-2
= = 75 ´109 Nm-2 For brass wire, F2 = 6 kg wt. = 6 × 9.8N,
0.002 r2 = 0.125 × 10–2m
(b) From the graph, yield strength
= 300 ´ 106 Nm–2 Y2 = 0.91 × 1011 Pa, l2 = 1m
= 3 ´ 108 Nm–2
9.3 The stress-strain graphs for materials A and B are shown Fl Fl Fl
Q Y= \ Dl = =
in Figure. ADl AY pr 2Y
For steel,

A B F1l1 10 ´ 9.8 ´ 1.5 ´ 7


Dl1 = =
Stress

Stress

pr12Y1 22 ´ (0.125 ´ 10 -2 ) 2 ´ 2 ´ 1011


= 1.49 × 10–4 m
For Brass,
Strain Strain
234 Physics
9.9 A steel cable with a radius of 1.5 cm supports a chairlift
F2l2 6 ´ 9.8 ´ 1 ´ 7 at a ski area. If the maximum stress is not to exceed 108
Dl2 = =
pr2 2Y2 22 ´ (0.125 ´ 10 - 2 ) 2 ´ 0.91´ 1011 N/m–2, what is the maximum load the cable can support?
Sol. Maximum load
= 1.3 × 10–4 m
= Maximum Stress × Area of cross-section
9.6 The edge of an aluminium cube is 10 cm long. One face
22
of the cube is firmly fixed to a vertical wall. A mass of = 10 8 ´ (1.5 ´10 - 2 ) 2 = 7.07 × 104 N
100 kg is then attached to the opposite face of the cube. 7
The shear modulus of aluminium is 25 G Pa. What is the 9.10 A rigid bar of mass 15 kg is supported symmetrically by
vertical deflection of this face? (1Pa = 1N/m 2 ) three wires each 2 m long. The wires at each end are of
(g = 10 m/s2). copper and middle one is of iron. Determine the ratio of
their diameters if each has the same tension. Young's
Sol. Given: modulus of elasticity for copper and steel are 110 × 109
A = 0.1 × 0.1 = 10–2 m2 F = mg = 100 × 10N; Shearing N/m2 and 190 × 109 N/m2 respectively.
Sol. Each wire has same tension so each wire will have same
DL Shearing stress
strain = = extension. Also as they have same length, each wire will
L Shear modulus have same strain
DL F / A FL Fl Fl 4Fl 1
\ = Þ DL = Y= = = 2
\ D µ
L h Ah ADl 2
p( D / 2) Dl pD 2 Dl Y
100 ´10 ´ 0.1 DCu Y 190 ´109 19
= -2 9
Þ DL = 4 × 10–7 m \ = Steel = = = 1.31
10 ´ 25 ´10 DSteel YCu 110 ´10 9 11
9.7 Four identical hollow cylindrical columns of mild steel 9.11 A 14.5 kg mass, fastened to the end of a steel wire of
support a big structure of mass 50,000 kg. The inner and unstretched length 1m, is whirled in a vertical circle with
outer radii of each column are 30 cm and 60 cm an angular velocity of 2 rev/s at the bottom of the circle.
respectively. Assuming the load distribution to be uniform, The cross-sectional area of the wire is 0.065 cm2 .
calculate the compressional strain in each column. The Calculate the elongation of the wire when the mass is at
Young's modulus of steel is 2 × 1011 Pa. the lowest point of its path.
Ysteel = 2 × 1011 N/m2
mg 50,000 ´ 9.8 Sol. Here,
Sol. Load on each column F = = N
m = 14.5 kg, l = r = 1m,
4 4
w = 2rps = 2 × 2p rad/s
22 A = 0.065 × 10–4 m2
A = p (r22 – r12) = [(0.60)2 – (0.30)2]
7 Tension in the wire at the lowest position on the vertical
circle = F = mg + mrw2
stress F/ A F 2
Compressional strain = = = æ 22 ö
Y Y AY = 14.5 × 9.8 + 14.5 × 1 × 4 × ç ÷ × 4
è 7 ø
50,000 ´ 9.8 = 142.1 + 2291.6 = 2433.7 N
= Fl Fl 2433.7 ´ 1
22 Y= Þ Dl =
4 ´ ´ [(0.60) 2 - (0.30) 2 ] ´ 2 ´1011 ADl
=
7 AY 0.065 ´ 10 - 4 ´ 2 ´ 1011
–3
= 1.87 × 10 m = 1.87 mm
= 7.21× 10–7
9.12 Compute the bulk modulus of water from the following
9.8 A piece of copper having a rectangular cross- section 15.2 data : Initial volume = 100 litre, pressure increase = 100
mm × 19.1 mm is pulled in tension with 44,500 N force, atmosphere, final volume = 100.5 litre (1 atmosphere = 1.013
producing only elastic deformation. Calculate the × 105 Pa)
resulting strain. Young’s modulus of elasticity of copper Sol. Here,
is 120 × 109 N/m2. DV = 100.5 – 100 = 0.5 litre = 0.5 × 10–3 m3;
Sol. Given: P = 100 atm = 100 × 1.013 × 105 Pa
V = 100 litre = 100 × 10–3m3
A = 15.2 × 19.1 mm2 = 15.2 × 19.1 × 10–6 m2; F = 44,500
N, Y = 120 × 109 N/m2 -P PV
Bulk modulus = B = =
DV / V DV
F
Shearing strain =
AY 100 ´ 1.013 ´ 10 5 ´ 100 ´ 10 -3
=
0.5 ´ 10 -3
44500 Þ B = 2.026 × 109
Pa
= = 0.127 ´10-2
15.2 ´ 19.1´10 -6
´120 ´10 9 9.13 What is the density of ocean water at a depth where the
pressure is 80 atm, given that its density at the surface
is 1.03 × 103 kg/m3? Compressibility of water = 45.8 ×
1011/Pa. Given: 1 atm = 1.013 × 105 Pa.
Mechanical Proper ties of Solids 235

Sol. P = 80 atm = 80 × 1.013 × 105 Pa Sol. Here pressure,


1 P = 10 atm = 10 × 1.013 × 105 Pa,
Compressibility = = 45.8 × 10–11/ Pa; Density of water B = 37 × 109 N/m2
B
at the surface = r = 1.03 × 103 kg/m3 stress P
Vol. strain = =
Let r' = density of water at a given depth; bulk modulus B
V' = Vol. of water of mass M at given depth
V = Vol. of same mass of water at the surface 10 ´ 1.013 ´ 10 5
= = 2.74 × 10–5
M M 37 ´ 109
\ V= and V ' = DV
r r' \ Fractional change in volume = = 2.74 × 10–5
V
æ1 1 ö 9.15 Determine the volume contraction of a solid copper cube,
Change in Vol. = DV = V – V' = M çç - ÷÷
è r r' ø 10 cm on an edge when subjected to a hydraulic pressure
of 7 × 106 Pa. Bulk modulus of copper = 140 GPa
æ1 1 ö Sol. Given:
Mç - ÷ L = 10 cm = 0.1 m; P = 7 × 106 Pa; B = 140 GPa = 140
DV èr r'ø r
Vol. strain = = M = 1- × 109 Pa
V r'
r P PV PL3
B= = =
DV / V DV DV
DV 1.03 ´ 10 3
Þ =1– PL3 7 ´ 10 6 ´ ( 0.1) 3
V r' Þ DV = = = 5 × 10–8 m3
B 140 ´ 10 9
P DV P
Also B = \ = 9.16 How much should the pressure on a litre of water be
DV / V V B changed to compress it by 0.1%. (Bulk modulus of elasticity
= 80 × 1.013 × 105 × 45.8 × 10–11 = 3.712 × 10–3 of water = 2.2 × 109 N/m2)
1.03 ´ 10 3 DV 0. 1
\ 1- = 3.712 × 10–3 Sol. From question, V = 1 litre = 10–3 m3, = = 10–3
r' V 100
1.03 ´ 10 3 PV DV
Þ r' = = 1.034 × 103 kg/m3 Bulk Modulus, B= Þ P=B
1 - 3.712 ´ 10 -3 DV V
9.14 Compute the fractional change in volume of a glass slab, = 2.2 × 109 × 10–3 = 2.2 × 106 N/m2
when subjected to a hydraulic pressure of 10 atmosphere.
Bulk modulus of elasticity of glass = 37 × 109 N/m2 and
1 atm = 1.013 × 105 Pa.

9.17 Anvils made of single crystal of diamond, with the shape 9.18 A rod of length 1.05 m having negligible mass is
as is shown in the adjacent fig. are used to investigate supported at its ends by two wires of steel (wire A) and
behaviour of materials under very high pressures. Flat aluminium (wire B) of equal lengths as shown in the
faces at the narrow end of the anvil have a diameter of adjacent figure. The cross-sectional area of wires A and
B are 1 mm2 and 2 mm2 respectively. At what point along
0.5 mm, and the wide ends are subjected to a compressional
the rod should a mass m be suspended in order to produce
force of 50, 000 N. What is the pressure at the tip of the (a) equal stresses and
anvil? (b) equal strains in both steel and aluminium wires.

A B
Diamond
Metal cones 1.05 m
gasket

m
Sol. Given Ysteel = 2 × 1011 N/m2 and
Sol. Here, D = 0.5 mm = 0.5 × 10–3m, F = 50,000 N YAluminium = 7 × 1010 N/m2.
F F 4 ´ 50,000 ´ 7 For steel wire A, A1 = 1 mm2 = 1 × 10–6m2,
P= = = Y1 = 2 × 1011 N/m2
22 ´ (5 ´ 10 - 4 ) 2
2 2
pr pD / 4 For aluminium wire B, A2 = 2 mm2 = 2 × 10–6m2,
11
= 2.5 × 10 Pa Y2 = 7 × 1010 N/m2
l1 = l2 = l (for both A and B)
236 Physics
(a) Let the mass m be suspended at a distance x from (expanding binomially and neglecting higher powers of
A. Let F1 and F2 are tensions in A and B that produce
x 2 / l 2 as x << l)
equal stress in both.
F1 F2 x2 x2
= Þ Dl = 2l + ´ 2l - 2l =
\ 2l 2 l
A1 A2

F1 A1 1´ 10 -6 1 Dl x 2 / l x 2
Þ = = = Strain = = = 2
2l 2l 2l
F2 A2 2 ´ 10 -6 2
Let T be the tension in the string
For equilibrium of the rod, the moments of forces
about the point of suspension of the mass should be \ 2 T cos q = mg Þ T = mg
equal. 2 cos q
\ F1x = F2 (1.05 – x)
x x x
1.05 - x F 1 cos q = = =
Þ = 1= l 2 + x2 æ x 2 ö1/ 2 æ x2 ö
x F2 2 l ç1 + 2 ÷ l ç1 + 2 ÷
è l ø è 2l ø
Þ 2.1 – 2x = x Þ x = 0.7m
(b) For equal strain in both the wires,
x2 x
F1 F2 F1 AY
As x << l , 1 + »1 \ cos q =
2 l
= Þ = 11 2l
A1Y1 A2Y2 F2 A2Y2 T mgl
\ mg mgl
11 T= = and stress = =
1 2 ´ 10 10 2( x / l ) 2 x A 2 Ax
= ´ =
2 7 ´ 1010 7
stress mgl 2l 2 mgl 3
Equating the moments again, Y= = × 2 =
strain 2Ax x Ax3
1.05 - x
F1x = F2 (1.05 – x) Þ 1/ 3 1/ 3
x é mg ù é 0.1 ´10 ù
\ x=lê ú = 0.5 ê ú
ë YA û 11 - 6
F1 10 ë 2 ´10 ´ 0.5 ´10 û
= = Þ 10x = 7.35 – 7x = 1.074 × 10–2 m
F2 7
9.20 Two strips of metal are riveted together at their ends by
Þ x = 0.4324 m four rivets, each of diameter 6 mm. What is the maximum
9.19 A mild wire of steel of length 1 m and cross-sectional area tension that can be exerted by the riveted strip if the
0.5 × 10–2 cm2 is stretched, well within its elastic limit, shearing stress on the rivet is not to exceed 6.9 × 107 Pa?
horizontally between two pillars. A mass of 100 g is Assume that each rivet is to carry one quarter of the load.
suspended from the mid point of the wire. Calculate the 6
depression of the mid point. g = 10 m /s2, Y = 2 × 1011N/m2 Sol. Here, r = = 3mm = 3 × 10–3m;
Sol. Let x be the depression of the mid point 2
Max. stress = 6.9 × 107 Pa
\ From the figure, AC = CB = l = 0.5 m; Max. load on a rivet
m = 100 g = 0.1kg
= Max. stress × area of cross section
CD = x, AD = BD = l 2 + x2 22
= 6.9 × 107 × × (3 × 10–3)2
7
l C l \ Max. tension = 4 × max. load
A B
x 22
= 4 × 6.9 × 107 × × 9 × 10–6
7
T q q T = 7.8 × 103 N
D 9.21 The Marina Trench is located in the Pacific ocean and
at one place it is nearly eleven km beneath the surface
of water. The water pressure at the bottom of the trench
mg is about 1.1 × 108 Pa. A steel ball of initial volume
0.32 m3 is dropped into the ocean and falls to the bottom
of the trench. What is the change in the volume of the
Increase in length, Dl = AD + DB – AB
ball when it reaches to the bottom? Bulk modulus for steel
= 2AD – AB = 1.6 × 1011 N/m2
Dl = 2 l 2 + x 2 – 2l Sol. From question,
P = 1.1 × 108 Pa, V = 0.32 m3, B = 1.6 × 1011Pa;
1/ 2
æ x2 ö æ x2 ö Change in volume,
= 2l ç1 + ÷ - 2l = 2l ç1 + 2 ÷ – 2l
è l2 ø è 2l ø PV 1.1 ´ 10 8 ´ 0.32
DV = = = 2.2 × 10–4m3.
B 1.6 ´ 1011
Mechanical Proper ties of Solids 237

Practice Questions

Very Short Answer Questions [1 Mark Qs.] 2. When a wire is stretched by a certain force, its elongation
is y. If a second wire of the same material but having length
1. Why do a spring balance show wrong measure after long 4 times and radius double of the first wire is stretched by
use? the same force, what will be its elongation?
2. Which is more elastic water or air and why? 3. The length of a metal is l1 when the tension in it is T1 and
is l2 when the tension is T2. Find the original length of
3. Why are springs made of steel and not of copper? the wire.
4. When the pressure on a sphere is increased by 80 4. A wire stretches by a certain amount under a given load.
atmospheres then its volume decreases by 0.01% Find the If the load and diameter are both increased to three times,
Bulk modulus of elasticity of the material of the sphere. find the stretch produced in the wire.
5. A spherical ball contracts in volume by 0.01% when
5. What is the Bulk modulus for an incompressible liquid? subjected to a normal uniform pressure of 100 atmosphere.
6. What is the value of shear modulus of a liquid? What is the bulk modulus of the material of the ball?
6. Two rods of different materials, having coefficient of linear
7. The length of a wire is reduced to half of its initial value.
expansions a1 and a2 and Young's modulii Y1 and Y2 are
What will be the effect on the increase in its length under fixed between two rigid supports. The rods are heated to
a given load? the same temperature. There is no bending of the rods. If
8. A wire is replaced by another wire of same length and a1 : a2 = 3 : 5, find Y1 : Y2, when equal thermal stress is
material but of double the diameter. developed in the two rods.
7. Steel is more elastic than rubber. Explain.
(i) What will be the effect on the increase in its length
under a given load? 8. A metallic wire is suspended by putting weight to it. If a
is the longitudinal strain and Y its Young's modulus of
(ii) What will be the effect on the maximum load which elasticity, show that the elastic P.E per unit volume is given
it can bear?
Ya2
by .
9. The breaking force for a wire is F. What will be the 2
breaking forces for: 9. A thick rope of density r and length L is hung from a rigid
(i) Two parallel wires of same size support. The Young's modulus of the material of the rope
is Y. What is the increase in length of the rope due to its
(ii) For a single wire of double radius?
own weight?
10. The Young's modulus of a wire of length L and radius r 10. One end of a uniform rod of mass M and cross-sectional
is Y. If the length is reduced to half, what will be its area A is suspended from a rigid support and an equal mass
Young's modulus? M is suspended from the other end. What is the stress
11. What is the Young’s modulus for a perfect rigid body ? at the mid-point of the rod?
12. What is the Bulk modulus for a perfect rigid body ? 11. Two identical solid balls, one of ivory and the other of wet-
clay, are dropped from the same height on the floor. Which
13. Identical springs of steel and copper are equally stretched.
one will rise to a greater height after striking the floor and
On which, more work will have to be done ?
why?
14. Is stress a vector quantity ? 12. To what depth must a rubber ball be taken in
15. Is elastic limit a property of the material of the wire ? deep sea so that its volume is decreased by 0.1%. (The bulk
16. Stress and pressure are both forces per unit area. Then modulus of rubber is 9.8 × 108 Nm–2; and the density of sea
in what respect does stress differ from pressure ? water is 103 kg m–3.)
13. Elasticity has a different meaning in physics than that in daily
Short Answer Questions [2 or 3 Marks Qs.] life. Comment.
1. A metal rod of length L and cross-sectional area A, is rigidly
Long Answer Questions [5 Marks Qs.]
fixed between two walls. The Young's modulus of its
material is Y and the coefficient of linear expansion is a. 1. Describe stress-strain relationship for a loaded steel wire
The rod is heated so that its temperature is increased from and hence explain the terms elastic limit, permanent set,
0°C to q°C. Find the force exerted at the ends of the rod. yield point and breaking stress.
238 Physics
2. Explain, with examples, ductile materials, brittle materials 5. The length of a wire increases by 8 mm, when a weight of 5
and elastomers on the basis of stress-strain curves. kg is hung. If the conditions are the same, but the radius of
3. What do you understand by potential energy of a stretched the wire is doubled, what will be the increase in length?
wire? Find a relation for it and hence determine the elastic 6. Two parallel steel wires A and B are fixed to rigid support at
potential energy per unit volume of the wire. the upper ends and subjected to the same load at the lower
end. The lengths of the wires are in the ratio 4 : 5 and their
4. (i) Prove that the work done in stretching a wire per unit
radii are in the ratio 4 : 3. The increase in the length of the
volume is 1/2 ´ tension ´ extension.
wire A is 1 mm. Calculate the increase in the length of the
(ii) Prove that the work done per unit volume in stretching wire B.
a wire for every type of strain = 1/2 ´ stress ´ strain 7. Two wires of equal cross-section but one of steel and the
oth er of copper are join ed end by end. When the
Numerical Questions [3 or 5 Marks Qs.] combination is kept under tension the elongation in the two
1. What force is required to str etch a steel wire wires are found to be equal. Given, Young’s moduli of steel
1 sq. cm in cross section to double its length? and copper are 2 × 1011 N/m2 and 1.1 × 1011 N/m2. Find the
[Ysteel = 2 × 1011 N/m2] ratio between the lengths of steel and copper wires.
2. A load of 2 kg produces an extension of 1 mm in a wire of 3 8. A metal cube of side 10 cm is subjected to a shearing stress
m in length and 1 mm in diameter. Calculate Young’s modulus of 104 N/m2. Calculate the modulus of rigidity if the top of
of elasticity of wire. the cube is displaced by 0.05 cm with respect to its bottom.
3. A copper wire 2 m long and 2 mm in diameter hangs vertically 9. A steel wire of 4 m is stretched through 2 mm. The cross-
with its upper end fixed to a rigid support. Find the increase sectional area of the wire is 2 mm2. If Young’s modulus of
in length, when 3 kgwt is attached to its lower end. Young’s steel is 2 × 1011 N/m2. Find (i) the energy density of the wire,
modulus for copper is 12.5 × 1011 dyne/cm2. (ii) the elastic potential energy stored in the wire.
4. A stress of 1 kgwt/sq mm is applied to a wire of which 10. Two wires made of the same material are subjected to forces
Young’s modulus is 1011 N/m2. Find the percentage increase in the ratio of 1 : 4. Their lengths are in the ratio 8 : 1 and
in length. diameter in the ratio 2 : 1. Find the ratio of their extensions.
Mechanical Proper ties of Solids 239

HOTS/Exemplar Questions
Very Short Answer Questions [1 Mark Qs.] 3. Two identical solid balls, one of ivory and the other of wet-
clay, are dropped from the same height on the floor. Which
1. A steel ring of radius r and cross sectional area A is fitted one will rise to a greater height after striking the floor and
onto a wooden disc of radius R (R > r). If the Young’s modulus why? [Exemplar]
of steel is Y, then find the force with which the steel ring is 4. A truck is pulling a car out of a ditch by means of a steel
expanded. [HOTS] cable that is 9.1 m long and has a radius of 5 mm. When the
2. A metallic rod breaks when strain produced is 0.2%. The car just begins to move, the tension in the cable is 800 N.
How much has the cable stretched? (Young’s modulus for
Young’s modulus of the material of the rod is 7 × 10 9 N/m2.
steel is 2 × 1011 Nm–2.) [Exemplar]
What should be its area of cross-section to support a load
of 104 N ? [HOTS] Long Answer Questions [5 Marks Qs.]
3. The Young’s modulus for steel is much more than that for 1. (a) The bulk modulus of rubber is 9 × 108 N/m2. To what
rubber. For the same longitudinal strain, which one will have depth below the surface of sea should the rubber ball be
greater tensile stress? [Exemplar] taken as to decrease its volume by 0.1%.
4. Identical springs of steel and copper are equally stretched. (b) A steel rail is 20 m long and has an area of cross section
One which, more work will have to be done? [Exemplar] 40sq. cm. Between summer and winter its length changes
Short Answer Questions [2 or 3 Marks Qs.] by 1 cm. If it is laid in winter, what force parallel to its length
is necessary to keep it from increasing the length in the
1. There are two wire of same material and same length while summer? (Y = 19 × 1010 N/m2). [HOTS]
the diameter of second wire is two times the diameter of first 2. A steel rod of length 2l, cross sectional area A and mass M
wire, then what will be the ratio of extension produced in is set rotating in a horizontal plane about an axis passing
the wires by applying same load ? [HOTS] through the centre. If Y is the Young’s modulus for steel,
2. The length of a metal is l1 when the tension in it is T1 and find the extension in the length of the rod. (Assume the rod
is l2 when the tension is T2. What is the original length of
is uniform). [Exemplar]
the wire ? [HOTS]

CHAPTER TEST
Time : 45 min. Max. Marks : 24
Directions : (i) Attempt all questions.
(ii) Questions 1 to 5 carry 1 mark each.
(iii) Questions 6,7 and 8 carry 3 marks each
(iv) Questions 9 and 10 carry 5 marks each.

1. Write copper, steel, glass and rubber in the order of increasing greatest length of aluminium wire that can hang vertically
Young’s modulus of elasticity. without breaking. Density of aluminium is 2.7 × 103 kg/m3.
2. What is the value of Young’s modulus of a perfectly rigid 9. (a) A rubber cord catapult has a cross-sectional area 1
body? mm2 and total unstretched length 10cm. It is stretched
3. A wire stretches a certain amount under a load. If the load to 12 cm and then released to project a stone of mass 5
and diameter both increase to three times, find the stretch gm. Taking Young’s modulus for rubber 5 × 108 N/m2,
caused in the wire. find the velocity of projection of the stone.
4. What is the order of strain within elastic limit? (b) Identical springs of steel and copper are equaly
5. A wire is suspended from a roof but no weight is attached stretched. On which more work will have to be done?
10. (a) Two rods of different materials, having coefficient of
to the wire. Is the wire under stress?
linear expansions a1 and a2 and Young’s modulus Y1
6. The length of a metal rod is l1 when the tension in it is T1 and Y2, are fixed between two rigid walls. The rods are
and is l2 when the tension is T2. Find the original length of heated to the same temperature. There is no bending
the wire. of rods If a1 : a2 = 2 : 3, find Y1 : Y2 when equal thermal
7. A spherical ball contracts in volume by 0.01% when stress is developed in the two rods.
subjected to a normal uniform pressure of 100 atmosphere. (b) A steel wire of diameter 2 mm has a breaking strength
What is the bulk modulus of its material in cgs units ? of 4 × 105 N. What is the breaking strength of similar
8. The breaking stress of Aluminium is 7.5×107 N/m2. Find the steel wire of diameter 1 mm ?
240 Physics

Solutions
PRACTICE QUESTIONS 16. Pressure is the external force per unit area, while stress is
the internal restoring force which comes into play in a
Very Short Answer Questions deformed body acting transversely per unit area of the
1. After long use the spring gradually loses its elastic nature body.
and doesn't come back to its initial configuration Short Answer Questions
completely after the weight is withdrawn.
2. Water is more elastic than air because bulk modulus of Fl YADl
1. From Y = Þ F= , Also Dl = l a q
elasticity is the reciprocal of compressibility and air is more ADl l
compressible than water. YAl aq
3. Under a particular deforming force, the amount of stretch \ F= = YAaq
l
produced in steel is less than that in copper. Steel is more Fl Fl
elastic than copper, so, it comes back to its initial 2. Young’s modulus, Y = = ,
ADl pr 2Dl
configuration faster than copper after the removal of the
F ´ 4l Fl
deforming force. Also Y = =
4. From question,
2
p (2r ) Dl ' pr 2 Dl '
P = 80 atm = 80 × 1.013 × 105 N/m2 Comparing, we get Dl = Dl '
-DV 0.01 \ Elongation will be same
= 3. Let l be the initial length of the wire. A be the area of cross-
V 100
section of the wire.
-P 80 ´1.013 ´105 \ Change in length in the 1st case = l1 - l and change
Bulk modulus = B = =
DV / V 0.01 / 100
in length in the 2nd case = l2 - l
= 8.1 ×1010 N/m2
Fl T1l T2l
5. Infinity. Q Y= \ Y= =
6. Since a liquid has no definite shape, so it's shear modulus ADl A(l1 - l ) A(l2 - l )
is zero. \ T1(l2 - l ) = T2 (l1 - l )
l
7. From question, l ' = Q Dl µ l Þ T1l2 - T1l = T2l1 - T2l
2
\ Increase in length will also be half. Þ T2l - T1l = T2l1 - T1l2
8. (i) Young’s modulus T l -Tl
Þ (T2 - T1) l = T2l1 - T1l2 Þ l = 2 1 1 2
Y = ( F / A) Þ Dl = Fl = Fl T2 - T1
(Dl / l ) AY pr 2Y 4. As we knows,
1 Fl Fl 4Fl
\ Dl µ Y= = =
r2 ADl p( d / 2) 2 .Dl pd 2 Dl
Increase in length will be reduced to 1/4th, of the
3F .4l 4Fl
original increase. In the 2nd case, Y = =
(ii) Q Breaking force µ Area µ pr2
2
p (3d ) Dl ' pd 2Dl
\ Breaking load will increase by four times.
Dl ¢ 1 Dl
9. (i) Breaking force will be double as two wires are of same \ = Þ Dl ¢ =
size. Dl 3 3
\ Total breaking force = F + F = 2F DV 0.01
(ii) Q Breaking force µ area µ pr2 5. From question, – = 0.01% = ,
V 100
\ As r becomes double, breaking force will increase
P = 100 atmosphere = 100 × 1.01 × 105 N/m2
4 times.
10. Young's modulus will remain same as it is constant for a
\ B=
-P 100 ´1.01´ 105
particular material and doesn't depend on the physical =
DV / V 0.01 / 100
dimension of the wire.
= 101 × 109 N/m2 = 1.01 × 1011 N/m2
11. Infinite
6. Due to thermal expansion of the rod, increase in length is
12. Infinite
13. Copper given by Dl = a l q [where l is the initial length and q is
14. No the increase in temperature]
15. No. It also depends on the radius of the wire.
Mechanical Proper ties of Solids 241

Dl Long Answer Questions


stress stress æ ö
Y= = çèQ Dl = al q, \ = a q ÷ø .
strain aq l 1. When a wire of elastic material is subjected to an increasing
tensile stress, the strain changes as shown in the figure.
1
\ Yµ [Stress and q are constant] Proportional limit
a Yield point/Elastic limit
Y1 a 2 5

Stress
\ = = \ Y1 : Y2 = 5 : 3 E
Y2 a1 3 B P
A
7. Under a given load, extension of steel is much less than
rubber. Also steel rod will go back to its initial configuration
easily and faster than rubber, as intermolecular force of steel
is more stronger than that of rubber. Therefore steel is
more elastic than rubber.

8. Stress \ Stress = Y × strain = Ya O O'


Y= Strain
Strain The figure shows that upto point A, the relation between
1 stress and strain is linear, i.e., the material obeys Hooke's
Elastic P.E/unit volume = × Stress × Strain =
2 law. The wire returns to its original length when the stress
1 Ya 2 is removed. OA represents the the proportional region. The
´ Ya ´ a = point A represents the proportional limit of the material.
2 2
9. Let A = cross-sectional area of the rope, Weight of the rope In the region from A to B, stress and strain are not
proportional. Nevertheless, the body still returns to its
= F = Vol. × density × g = ALrg
original dimension when the load is removed. The point
The weight acts from the centre of gravity of the rod i.e. B is known as yield point.
at a distance L/2 from the support. If the stress on the wire is increased beyond the value
corresponding to the elastic limit, the wire acquires a
Stress ALrg / A L2rg
Y= = Þ DL = . permanent set (OO’); Hooke's law is not obeyed. If the
Strain D L / ( L / 2) 2Y str ess is now removed, the strain doesn't vanish
10. The rod is uniform, the mid-point will lie at half of its height. completely. If the process is carried on for some more time,
Tension at the mid point of the rod = weight of the suspended a stage is reached when the wire is found to undergo a
relatively large strain with practically no increase in stress.
mass + weight of the half of the rod
The stress at which this large increase in strain occurs is
Mg 3Mg called yield point. (point P in the figure)
= Mg + = Still further increase in stress ultimately results in the
2 2
breaking of the wire. The corresponding point E on the
\ Stress at the mid-point stress-strain curve is called breaking point. The maximum
F 3Mg / 2 3Mg stress that can be applied to a material before it breaks is
= = = called its breaking stress.
A A 2A
2. From the variation of stress and strain in different elastic
11. As the ivory ball is more elastic than the wet-clay ball, it
materials, they are classified into three types.
will tend to retain its shape instantaneously after the
collision. Hence, there will be a large energy and momentum Elastic limit
transfer compared to the wet clay ball. Thus, the ivory ball Proportional or yield point
will rise higher after the collision. limit
12. Let the depth be h then the pressure is D
Su E
P = rgh = 103 × 9.8 × h Fracture
Sy
C point
P DV AB Plastic behaviour
Now, =B Þ P=B = 9.8 ´ 108 ´ 0.1 ´ 10 -2
DV V V
Stress

Elastic behaviour
9.8 ´ 108 ´ 0.1 ´ 10-2
\ h= = 102 m
9.8 ´ 103
13. In daily life, a body is said to be elastic if a large deformation
or strain is produced on applying a given stress on it. In Permanent set
physics, elasticity is the property of the material of a body by O Strain 30%
virtue of which it opposes any change in its size or shape when <1%
a stress is applied on it. Thus a body will be more elastic if a
A typical stress-strain curve for a metal
small strain is produced on applying a given stress on it.
242 Physics
(a) Ductile materials : Materials having large plastic \ Work done per unit volume
range beyond elastic limit are called Ductile materials. 1
Breaking point is far apart from the point of elastic = stress ´ strain
limit in the stress-strain graph for these materials. i.e. 2
points D & E are far apart. Ex. copper, iron, aluminium. work done 1 æ F ö æ Dl ö
or = ç ÷ .ç ÷
(b) Brittle materials: Materials having small plastic range volume 2 è Aø è l ø
beyond elastic limit are called brittle materials. 1
Breaking point is very close to the point of elastic = F ´ Al
2
limit in the stress-strain curve. i.e. point D & E are
close. Ex: glass, cast-iron. = Tension ´ extension
(c) Elastomers: These materials doesn't even obey (ii) The type of stress does not matter as it will be the
Hooke's law even within elastic limit. They don't have variable force involved at varying extension leading
any plastic range. Breaking point is very close to the to the work done
elastic limit. Ex: Rubber, elastic tissue of the artery 1
carrying blood from heart. ò F.dx = 2 stress × strain × volume
3. When a wire is stretched, some work is done against the
internal restoring forces acting between the molecules of Numerical Questions
the wire. This work done is stored as elastic potential
energy in the wire. Fl
1. As we know, Y =
Let us consider, l = initial length of the wire; A = area of ADl
cross-section of the wire; F = stretching force applied on
the wire; Dl = increase in length of the wire. YADl 2 ´ 1011 ´ 1 ´ 10 -4 l
Þ F= = = 2 × 107 N.
Initially, the restoring force is zero, but when the length l l
has increased by Dl, the restoring force is equal to the [Q Dl = 2l - l = l ]
applied force F.
2. From question,
0+F F
\ Average restoring force = = F = Mg = 2 × 9.8 N; Dl = 1 mm = 1× 10–3 m, l = 3 m; A =
2 2
\ Work done on the wire = W = Average force × 2
æ 1´ 10-3 ö
F pç ÷
increase in length Þ W = ´ Dl ç 2 ÷
2 è ø
1 1 F Dl Fl 2 ´ 9.8 ´ 3
\ Elastic P.E = ´ F ´ Dl = ´ ´ ´ Al Y= = = 7.48 × 1010 N/m2
2 2 A l ADl -3 ö 2
22 æ 10
´ ´ 10-3
1
= ´ stress × strain × vol. of the wire 7 çè 2 ÷ø
2
3. Here, l = 2 m, Dl = ?,
1
\ Elastic P.E./vol. = ´ Stress ´ Strain
2 pd 2 3.14 ´ (2 ´ 10 -3 ) 2 2
A= = m
4. (i) Consider a wire of length l, area of cross-section A 4 4
made of material of Young’s modulus of elasticity Y.
Let a force F be applied and at any instant, x be the F = 3 × 9.8 N,
extension associated (x < L), where L is the maximum
extension. At this instant. Y = 12.5 ´ 1011 dyne / cm2
AY .x
F= 12.5 ´ 1011
l = N / m 2 = 12.5 × 1010 N/m2
Since force is a variable with x, 105 ´ 10 -4
Work done to stretch
L Fl
Using the formula Dl = , Dl = 1.5 ´ 10 -4 m
W = ò Fdx AY
0
1 AY 2 Dl
W= .L 4. Percentage increase in length = × 100
2 l l
1 æ Y .L öæ L ö
W = ( Al ) ç ÷ç ÷ F 1 ´ 9.8 ´ 100
2 è l øè l ø = ´ 100 = -6 = 0.0098%
AY 10 ´ 1011
1
= ´ volume ´ stress ´ strain
2
Mechanical Proper ties of Solids 243

(ii) Elastic P.E. = Energy density × vol. of the wire = 2.5 ×


Fl Fl 1
5. Change in length, Dl = = 2 \ Dl a 104 × (2 × 10–6) × 4 = 0.2 J
AY pr Y r2 10. Young’s modulus

Dl1 r2 2 F l Fl
\ = Y= . or Dl = 2
2 Dl
Dl2 r12 pr pr Y
Both wires are of same material, so their Y is same.
r12 r 2
Þ Dl2 = Dl ´ = 8´ = 2 mm Dl1 F1 l1 r22 1 8 1
r2 2 (2r )2 \ = . . = ´ ´ = 1: 2
Dl2 F2 l2 r12 4 1 4
6. For wire A, A1 = p(4r )2 , l1 = 4l , Y1 = Y ,
HOTS/EXEMPLAR QUESTIONS
Dl1 = 1 mm, F1 = F
Very Short Answer Questions
2
For wire B, A2 = p(3r ) , l2 = 5l , Y2 = Y , F2 = F , 1. Let T be the tension in the ring, then
Dl2 = ? T.2 p r Tr
Y= = Y A (R - r )
\ T=
A. 2p (R - r ) A (R - r) r
\ F1 l F l
Y= ´ 1 = 2´ 2
A1 Dl1 A2 Dl2 2. Maximum possible strain = 0.2/100

F 10 4 ´ 100
F A l
Þ Dl2 = 2 ´ 1 ´ 2 ´ Dl1 \ A= = = 7 .1 ´ 10 - 4 m2
F1 A2 l1 Y ´ strain (7 ´ 10 9 ) ´ 0 .2
3. Steel
2 4. Copper
æ4ö 5 20
\ Dl2 = 1 ´ ç ÷ ´ ´ 1 = mm = 2.22 mm.
è3ø 4 9 Short Answer Questions

1. Q Both wires are of same materials so both will have


F ls F l
7. For steel, Ys = ´ for copper, Ycu = ´ cu same Young’s modulus, and let it be Y.
A Dls A Dlcu
stress F
Y= = , F = applied force
\
Ys l Dl l [Q Dlcu = Dls ] strain A.(ΔL/L)
= s ´ cu = s
Ycu lcu Dls lcu
A = area of cross-section of wire

ls 2 ´ 1011 20 FL FL
\ = =
l cu 1.1 ´ 1011 11 Now, Y1 = Y2 Þ =
(A1 )(DL1 ) (A 2 )(DL 2 )
8. Given, D L = 0.05 cm, L = 10 cm, Shearing stress = 104 N/m2 Since load and length are same for both
Shearing strain = D L/L = 0.05/10 = 0.005 2
æ DL ö æ r ö
Þ r12 DL1 = r2 2 DL 2 , çç 1 ÷÷ = çç 2 ÷÷ = 4
shearing stress 10 4 è DL 2 ø è r1 ø
Modulus of rigidity = =
shearing strain 0.005
DL1 : DL2 = 4 :1
= 2 × 106 N/m2
2. If l is the original length of wire, then change in length of
1
9. (i) Energy density = × stress × strain first wire, Dl 1 = (l 1 - l)
2
Change in length of second wire, Dl 2 = (l 2 - l)
2
1 1 æ Dl ö
= × Y × (strain)2 = × Y × ç ÷ l l
2 2 è l ø T1 T
Now, Y= ´ = 2´
A Dl 1 A Dl 2
-3 2
1 11 é 2 ´ 10 ù or
T1 T
= 2 or
T1
=
T2
= ´ 2 ´ 10 ´ ê ú = 2.5 × 104 J/m3. l1 - l l 2 - l
2 Dl 1 Dl 2
ëê 4 ûú
244 Physics
3. As the ivory ball is more elastic than the wet-clay ball, it will tend
d (d)
to retain its shape instantaneously after the collision. Hence, there
will be a large energy and momentum transfer compared to the
\
dr
=
1 mw 2 2 2
YA 2
l -r ( )
wet clay ball. Thus, the ivory ball will rise higher after the collision.
4. Let the increase in length be Dl, then

800
\ d (d) =
1 mw 2 2 2
YA 2
l - r dr ( )
11
= 2 ´ 10
(p ´ 25 ´ 10 ) / ( Dl / 9.1)
-6
1 mw 2
l
l 2 - r 2 dr( )
YA 2 ò
\ d=
9.1 ´ 800 0
\ Dl = m
p ´ 25 ´ 10-6 ´ 2 ´ 1011
1 mw2 é 3 l3 ù 1 2 3 1 2 2
= YA 2 êl - 3 ú = 3YA mw l = 3YA mw l
; 0.5 ´10 m –3
êë úû
Long Answer Questions
2
The total change in length is 2d = mw2l2
3YA
ævö æ v ö 0.1
1. (a) DP = Bç ÷ Here ç ÷ =
èVø è V ø 100
\ DP = 9 × 105
Atmospheric pressure at sea leavel = 10 m of water l
dr
column.
Let the ball is taken to a depth h. Then DP = (h – 10) r
rg
9 × 105 = (h – 10) (1000) (10)
\ h = 100 m

1cm
= 0.5 ´ 10 -3 CHAPTER TEST
(b) Strain =
20 ´ 100cm
1. Rubber, glass, copper, steel.
Stress = Y × strain = (19 × 1010) (0.5 × 10–3) 2. Infinite, since a perfectly rigid body does not undergo any
= 9.5 × 107 N/m2 deformation. So, its strain is zero.
Force = Area of cross section × stress Fl
3. Young’s modulus, Y=
= (40 × 10–4) × (9.5 × 107) = 3.8 × 105 Newton. ADl

or T1 l 2 – T1 l = T2 l 1 – l T2 or l =
T2 l 1 - T1l 2 Þ Dl = Fl Þ Dl µ F
T2 - T1 AY A
Dl ' 3F A 1
2. Consider an element at r of width dr. Let T(r) and T(r + dr) be the \ = ´ =
tensions at the two edges. Dl F 9A 3
– T (r + dr) + T(r) = mw2rdr where m is the mass/length 4. 10–3/cm
dT 5. Yes, the weight of the wire itself acts as the deforming force.
- dr = mw2 rdr
dr 6. Let l and A be the original length and area of cross - section
At r = l T = 0 of the wire.
mw 2l 2 \ Change in length in the 1st case = l1 - l
Þ C=
2
Change in length in the second case = l2 - l

\ T(r) =
mw 2 2 2
2
l -r ( ) \ Y=
T1
´
l T
= 2´
l
A l1 - l A l2 - l
Let the increase in length of the element dr be d (d)
or T1l2 - T1l = T2l1 - T2l

Y=
(mw / 2)(l2 2
)
- r2 / A
or l (T2 - T1) = T2l1 - T1l1
d ( d)
dr T2l1 - T1l1
Þ l=
T2 - T1
Mechanical Proper ties of Solids 245

7. Here, (b) We know that the Young’s modulus of steel is greater


than of copper. So, in order to produce same
DV 0.01 extension, a larger force will have to be applied on
– = and P = 100 × 1.01 × 106 dyne/sq.cm
DV 100 the steel spring than copper spring. This shows that
more work will be done on steel spring.
- PV 100 ´ 1.0 ´ 106
K= = = 1012 dyne/sq cm. 10. (a) Increase in length due to thermal expansion = DL =
DV 0.01/100 aLq where q = increase in temperature
8. Mass of the wire = m = vol. density = ( Al ) ´ r
mg Arlg DL
\ Breaking stress = = = lrg \ Strain = = aq
A A L
= 7.5 × 107 1
Q Y = stress = stress \Yµ
7.5 ´ 10 7 strain aq a
\ l= 3
= 2.83 ´ 103 m
2.7 ´ 10 ´ 9.8 as stress and q are constants
9. (a) Here, elastic potential energy = K.E. of stone
\ 1/2 × Y × (strain)2 × volume of cord = 1/2 mv2 Y1 a 2 3 Þ Y : Y = 3 : 2
\ = = 1 2
Y2 a1 2
ìï Y ´ (strain) 2 ´ volume of cord üï
or v = í ý (b) Breaking strength (F)
ïî m ïþ
= breaking stress × area of cross-section

Given that strain = 12 - 10 = 1 , Y = 5 ´ 108 N / m2 pD 2


10 5 = stress × \ F µ D2
4
Volume of the cord = (1 × 10–6) × 10 × 10–2 = 1 × 10–7 m3
Mass m of the stone = 5 × 10–3 kg F2 D2 2 æ 1 ö 1
2
\ = 2 =ç ÷ =
Hence F1 D1 è 2 ø 4

ì (5 ´ 108 ) ´ (1/ 5) 2 ´ (1´10-7 ) ü 1 1 1


v= í ý = ´ (10000) Þ F2 = F1 ´ = 4 × 105 × = 105 N
î 5 ´ 10-3 þ 5 4 4
1
= ´ 100 = 20m / s
5
¿¿¿
246 Physics

10 Mechanical Properties
of Fluids
C ha p t e r

Liquids and gases can flow and are therefore called fluids. Fluids are different from solids. A solid has a fixed shape whereas a fluid has
no definite shape. Fluids offer very little resistance to shear stress about million times smaller than that of solids.
For liquids at rest (static equilibrium) only forces normal to the surface of liquid will act. The net force acting on any area of the liquid
is called thrust.
PRESSURE
If a uniform force is exerted normal to an area (A), then average pressure (pav) is defined as the normal force (F) per unit area.
F
i.e., pav =
A
DF
In limiting sense, pressure p = lim Pressure is a scalar quantity..
DA®0 DA

SI unit : pascal (Pa), 1 Pa = 1 N/m2


Practical units: atmospheric pressure (atm), bar and torr
1 atm = 1.01325 × 105Pa = 1.01325 bar = 760 torr = 760mm of Hg column pressure
1 bar = 105 Pa
1 torr = pressure exerted by 1 mm of mercury column = 133 Pa.
PASCAL'S LAW
Pascal's law is stated in following ways :
• The pressure in a fluid at rest is same at all the points if gravity is ignored.
• A liquid exerts equal pressures in all directions.
• If the pressure in an enclosed fluid is changed at a particular point, the change is transmitted to every point of the fluid and to
the walls of the container without being diminished in magnitude.
Applications of Pascal's law F2
Hydraulic machines, lifts, presses and brakes, are based on the Pascal's law.
For the hydraulic lift :
F1
F1
Pressure applied on a piston of small cross-section area A1 is, A A1 A2
1
F2
Pressure transmitted to area A2 is, = A
2
F1 F2
Q Pressures A = A
1 2
F1 A2
Upward force on A2 is F2 = A A 2 = A ´ F1
1 1
Therefore, by changing the force at A1 piston of cross section area A2 can be moved up and down.
Variation of Pressure with Depth
Consider a vessel containing liquid. As the liquid is in equilibrium, so every volume element of the fluid is also in equilibrium. Consider
one volume element in the form of a cylindrical column of liquid of height h and of area of cross section A.
Mechanical Proper ties of Fluids 247

The various forces acting on the cylindrical column of liquid are :


(i) Force, F1 = P1A acting vertically downward on the top face of the column. P1 is the pressure of the liquid on the top face of the
column and is known as atmospheric pressure.
(ii) Force, F2 = P2A acting vertically upward at the bottom face of the cylindrical column. P2 is the pressure of the liquid on the
bottom face of the column.
(iii) Weight, W = mg of the cylindrical column of the liquid acting vertically downward. Since the cylindrical column of the liquid is
in equilibrium, so the net force acting on the column is zero.
i.e., F1 + W – F2 = 0 F1
Þ P1A + mg – P2A = 0
Þ P1A + mg = P2A
mg h
\ P2 = P1 + ...... (i)
A
Mass of the cylindrical column of the liquid
m = volume × density of the liquid F2
= area of cross section × height × density = Ahr
\ Substituting this value of mass in equation (i)

Ahrg
P2 = P1 +
A
or, P2 = P1 + hrg ......... (ii)
P2 is the absolute pressure at depth h below the free surface of the liquid. Equation (ii), shows that the absolute pressure at depth
h is greater than the atmospheric pressure (P1) by an amount equal to hrg.
(P2 – P1) = hrg which is the difference of pressure between two points separated by a depth h. It is called gauge pressure.
Types of Pressure
Pressure is of three types :
(i) Atmospheric pressure (P0)
(ii) Gauge pressure (Pgauge)
(iii) Absolute pressure (Pabs)
(i) Atmospheric pressure : Force exerted by air column on unit cross-section area of sea level is called atmospheric pressure (P 0)
F
P0 = = 101.3 kN / m2
A
Barometer is used to measure atmospheric pressure which was discovered by Torricelli.
p0 =0

rm
p0 p0 h
A B

Atmospheric pressure varies from place to place and at a particular place from time to time.
(ii) Gauge pressure: Excess pressure (P– Pa) measured with the help of pressure measuring instrument is called Gauge pressure.
Pgauge = hrg or Pgauge µ h.
Gauge pressure is always measured by "manometer".
(iii) Absolute pressure : Sum of atmospheric and gauge pressure is called absolute pressure.
Pabs = Patm + Pgauge Þ Pabs = P0 + hrg
The pressure which we measure in our automobile tyres is gauge pressure.
248 Physics
Density
Mass per unit volume is called density.
mass(M)
Density (P)= Its SI unit is Kgm–3 . The density of water at 4°C (277 K) is 1.0 × 103 Kgm–3. Relative density of a substance
Volume(V)
is the ratio of its density to the density of water at 4°C.
Illustration 1 :
A base of rectangular vessel measures 10 cm × 18 cm. Water is poured in to depth of 4 cm. What is the pressure on the base?
What is the thrust on the base? [g = 10m/s2.]
Sol. Area of base = 10 cm × 18 cm = 180 cm2 = 180 × 10–4 m2
Depth of water, h = 4 cm = 4 × 10–2 m
Density of water, r = 103 kg/m3
\ Pressure on the base, P = rhg = 103 × 4 × 10–2 ×10 = 400 Pa
Thrust on the base, F = P × A = 400 × 180 × 10–4 = 7.2N
Illustration 2 :
A girl weighs 50 kg and wearing high heel shoes balances on a single heel. The heel is circular with a diameter 1cm. What is
the pressure exerted by the heel on the horizontal floor?
Sol. Force = 50 × 9.8 N

pD 2 p(1´10 -2 ) 2 3.14 ´ 10 -4 m 2
Area, A = = =
4 4 4
F 50 ´ 9.8 ´ 4
\ Pressure, P = = = 6.24 × 106 N/m2
A 3.14 ´ 10 -4

Illustration 3 :
A hydraulic automobile lift is designed to lift cars with a maximum mass of 3000 kg. The area of cross-section of the piston
carrying the load is 425 cm2. What maximum pressure would the smaller piston have to bear?
Sol. Area of cross-section of bigger piston, A = 425cm 2 = 425 × 10–4 m2
Maximum load on the bigger piston, F = 3000 × 9.8N
F 3000 ´ 9.8
\ Pressure on the bigger piston, P = = = 6.92 × 105 N/m2
A 425 ´10 - 4
According to the Pascal's law, the smaller piston would bear the same pressure as on the bigger piston equal to 6.92 × 105 N/m2.
Illustration 4 :
A hydraulic press with a larger piston of diameter 30 cm at a height of 1.0 m above the smaller piston of diameter 5 cm is shown
in figure. The mass on the smaller piston is 10 kg. What is the force exerted on the load by the larger piston? The density of
oil in the press is 750 kg/m3.
Sol. Force on the smaller piston = mg = 10 × 9.8 = 98N
2 2
æD ö pD2
Area of smaller piston = p ç 2 ÷ =
è 2 ø 4

mg 4 ´ 98
\ Pressure on the smaller piston = = = 49936.3 Pa 1.0 m
pD2 / 4 2 3.14 ´ 0.0025

Let F = force exerted on load by larger piston

4´ F 4F
\ Pressure by larger piston = = = 14.15 F pa
pD1 2 3.14 ´ 0.09

Now pressure difference = rhg = 750 × 1 × 9.8 = 7350 Pa


Now 7350 = 49936.3 – 14.15 F or F = 3009.6N
Mechanical Proper ties of Fluids 249

STREAMLINE AND TURBULENT FLOW


Streamline Flow
When a liquid (fluid) flows, such that each particle of the liquid passing a point moves along the same path and has the same
velocity as its predecessor then the flow is called stream line flow. It is also called laminar flow.
The path of a particle is called a streamline. Streamlines represent the trajectories of fluid particles. It is a curve (or a straight line) drawn
such that the tangent to it at a point gives the direction of flow at that point. Two streamlines will never cross each other. If they cross
then this means that the fluid particle flowing through the point of intersection can flow in two different directions at the same time,
which is impossible. The closer the streamline, the greater the liquid velocity and vice versa.
Turbulent Flow
When the velocity at a point in the liquid changes with time the flow is called unsteady flow. The unsteady flow is called turbulent
when there are bends in the path of a fast moving liquid. The velocity of liquid changes continuously and haphazardly both in
magnitude and direction. The turbulent flow is characterized by small, erratic whirlpool-like circles called eddy current or eddies which
absorb a large amount of energy.
Q
Principle of Continuity v2
Let us consider the steady flow of an incompressible liquid. Then
m1 = m2 P A2
i.e. v1A1r1 = v2 A2r2 .......... (1)
As we have considered the fluid incompressible thus, v1
r1 = r2
\ v 1 1 = v2 A2
A .......... (2)
Equations (1) and (2) are said to be as equation of continuity.
It is statement of the conservation of mass. A1
BERNOULLI’S PRINCIPLE
Bernoulli’s principle states that the sum of pressure energy, kinetic energy and potential energy per unit volume of an incompressible,
non-viscous fluid in a streamlined irrotational flow remains constant along a streamline.
1
Mathematically, it can be expressed as P + rv 2 + r gh = constant
2
Proof. Consider a non-viscous and incompressible fluid flowing steadily between the sections A and B of a pipe of varying cross-
section. Let a, be the area of cross section at A, v1 the fluid velocity, P1 the fluid pressure, and h1 the mean height above the ground
level. Let a2, v2, P2 and h2 be the values of the corresponding quantities at B.

B' r 2a 2
v 2 Dt
B
v 1 Dt A'
A

r 1a 1 h2

h1

Let r be the density of the fluid. As the fluid is incompressible, so whatever mass of fluid enters the pipe at section A in time Dt,an equal
mass of fluid flows out at section B in time Dt. This mass is given by
m = Volume × density
= Area of cross-section × length × density
or m =a1 v1 Dt r = a2 v2 Dt r ...(1)
a1 v1 = a2 v2 ...(2)
\ Change in K.E. of the fluid
= K.E. at B. – K.E. at A
1 1
= m(v 22 – v12 ) = a1 v1 Dt r (v 22 – v12 ) [Using (1)]
2 2
250 Physics
Change in P.E. of the fluid
= P.E. at B – P.E. at A
= mg (h2 – h1) = a1 v1 Dt r g (h2 – h1) [Using (1)]
Net work done on the fluid
= Work done on the fluid at A – Work done by the fluid at B
= P1 a1 × v1 Dt – P2 a2 × v2 Dt
= P1 a1 v1 Dt – P2 a1 v1 Dt [Using (2)]
= a1 v1 Dt (P1 – P2)
By conservation of energy,
Net work done on the fluid
= Change in K.E. of the fluid + Change in P.E. of the fluid
1
\ a1 v1 Dt (P1 – P2) = a1 v1 Dt r (v22 – v12 ) + a1 v1Dtrg (h 2 - h1 )
2
Dividing both sides by a1 v1 Dt, we get
1 2 1 2
P1 - P2 = rv2 - rv1 + rgh 2 - rgh1
2 2
1 1
or P1 + r v12 + rgh1 = P2 + r v 22 + rgh 2
2 2
1
or P + rv 2 + rgh = constant ...(3)
2
This proves Bernoulli’s principle according to which the total energy per unit volume remains constant.
Applications of Bernoulli’s Principle
(1) Dynamic lift :
(i) Wings of aeroplane : The wings of the aeroplane are having tapering as shown in figure. Due
to this specific shape of wings when the aeroplane runs, air passes at higher speed over it as
compared to its lower surface. This difference of air speeds above and below the wings, in
accordance with Bernoulli's principle, creates a pressure difference, due to which an upward
force called 'dynamic lift' acts on the plane. If this force becomes greater than
the weight of the plane, the plane will rise up.
(ii) Ball moving without spin: The velocity of fluid (air) above and below the ball at corresponding points is the same resulting
in zero pressure difference. The air therefore, exerts no upward or downward force on the ball.
(iii) Ball moving with spin: A ball which is spinning drags air along with it. If the surface is rough more air will be dragged. The
ball is moving forward and relative to it the air is moving backwards. Therefore, the velocity of air above the ball relative to
it is larger and below it is smaller. The streamlines thus get crowded above and rarified below.
This difference in the velocities of air results in the pressure difference between the lower and upper faces and there is a net
upward force on the ball. This dynamic lift due to spinning is called Magnus effect.
(2) Venturimeter :
It is used for the measurement of rate of flow of fluid through a tube. The
working of venturimeter is based on Bernoulli’s principle. Rapidly moving
fluids sustain less pressure than slowly moving fluids.
2 gh
v = A1 A2
A12 - A22
(3) Velocity of Efflux (Torricelli’s theorem):
If a liquid is filled in a vessel up to height H and a hole is made at a depth h
below the free surface of the liquid as shown in figure, then taking the level of h C v = 2 gh
hole as reference level (i.e., zero point of potential energy) and applying
Bernoulli's principle to the liquid just inside and outside the hole (assuming H r
the liquid to be at rest inside) we get
1
( P0 + hrg ) + 0 = P0 + rv 2 or v = 2 gh
2 x
which is the same speed that an object would acquire in falling from rest through a distance h and is called 'velocity of efflux' or
velocity of flux (Torricelli’s theorem). From this expression it is clear that:
(i) The speed of the liquid coming out of the orifice is independent of the nature and quantity of liquid in the container or the
area of the orifice, it depends on shape of orifice.
Mechanical Proper ties of Fluids 251

(ii) Greater is the distance of the hole from the free surface of liquid greater will be the velocity of efflux (i.e., v µ h ). This is
why liquid gush-out with maximum velocity from the orifice which is at maximum vertical distance from the free surface of
the liquid.
(iii) As the vertical velocity of liquid at the orifice is zero and it is at a height (H – h) h C
from the base, the time taken by the liquid to reach the base-level v

2( H - h) H
t= H-h
g
Now during this time liquid is moving horizontally with constant velocity , x
A B
so it will hit the base level at a horizontal distance x (called range) as shown in figure such that
x = vt = 2 gh ´ [2( H - h) / g = 2 h( H - h)

Illustration 5 :
A liquid is flowing through a non-sectional tube with its axis horizontally. If two points X and Y on the axis of tube
has a sectional area 2.0 cm3 and 25 mm2 respectively then find the flow velocity at Y when the flow velocity at
X is 10 m/s.
Sol. According to principle of continuity
vxAx = vyAy
v x Ax 10(m/s) ´ 2(cm 2 )
therefore vy = A = = 80 m/s
y 25 ´ 10 -2 (cm 2 )
Therefore, the flow velocity at y is 80 m/s.
Illustration 6 :
A container of large uniform cross sectional area holds two immiscibe, non-viscous and incompressible liquids of densities r
and 4r, each of height H/2. Determine the position where small hole be punched, as that the heavier liquid comes out with a
maximum range R initially.
Sol. Let the hole be punched at a height h above the base level, so that the heavier liquid gushes out of it (as shown in Fig.)
First of all, let us find out the velocity of efflux. Applying Bernoulli’s theorem between two points close to the hole one inside and
the other just outside the hole,
1 1
P1 + rv12 = P2 + rv22
2 2
H/2

r
(reduced form of Bernoulli’s equation for horizontal flow)
H æH ö 1 4r
Þ P0 + rg + ç – h÷ 4rg = P0 + (4r)v2
H/2

2 è 2 ø 2 h
(v1 = 0 and the vessel is open to the atmosphere)
R
2 1
Þ g [( H / 2) + 2 H – 4h] = 2v Þ v= g (5 H – 8h)
2
Now, since the vertical component of velocity of liquid is zero initially, so the time taken by it to reach the ground is t = 2h / g .

Illustration 7 :
Water flows through a tunnel a reservoir of dam towards the turbine installed in the power plant. The is situated h m below the
reservoir. If the ratio of cross-sectional areas of the tunnel at the reservoir and power station is h, find the speed of the water
entering into the turbine.
Sol. Applying Bernoulli’s theorem at reservoir and power plant for the flowing water, we obtain
1 1
P0 + rgh1 + rv 2 = P0 + rgh2 + rv22 Þ v22 = v12 + 2g (h1 – h2).
2 1 2
Putting (h1 – h2) = h1 we obtain v2 = h1 + h2 .......(1)

Equation of continuity yields A1v1 = A2v2 .......(2)


Eliminating v1 from equation (1) and (2), we obtain
252 Physics

2
æ A2 ö 2 gh
v2 = ç v2 ÷ + 2 gh Þ v2 = 2
è A1 ø æA ö
1- ç 2 ÷
è A1 ø
2 gh
Putting A1/A2 = h,we obtain v2 = h
h2 - 1
Illustration 8 :
A plane is in level flight at constant speed and each of its two wings has an area of 25 m2. If the speed of the air is
180 km/h over the lower wing and 234 km/h over the upper wing surface, determine the plane’s mass. (Take air density to be
1 kg m–3).

Sol. Here, V1 = 234 ´ 5 = 65 ms -1


18
5
V2 = 180 ´ = 50 ms - 1 , A = 25 m2
18

Applying Bernoulli’s theorem above and below the wings


1 1
P1 + rv12 = P2 + rv22
2 2

1 1
( P2 - P1 ) = r (v12 - v22 ) = ´ (652 - 50 2 )
2 2
1
= (65 - 50) (65 + 50) = 862.5 Nm -2
2
\ Upward thrust on the two wings = 862.5 × (25 × 2)
This force supports the weight of the aeroplane
i.e., m × 9.8 = 862.5 × 50
862.5 ´ 50
or m= = 4400 kg
9.8

VISCOSITY
"The property of a fluid by virtue of which it opposes the relative motion between its different layers is known as viscosity and the
force that is into play is called the viscous force".
Consider the slow and steady flow of a fluid over a fixed horizontal surface. Let v be the velocity of a thin layer of the fluid at a distance
x from the fixed solid surface. Then according to Newton, the viscous force acting tangentially to the layer is proportional to the area
of the layer and the velocity gradient at the layer. If F is the viscous force on the layer, then
F µ A, where A is the area of the layer
dv
Also, F µ -
dx
The negative sign is put to account for the fact that the viscous force is opposite to the direction of motion.
dv
Þ F = -h A
dx
where h is a constant depending upon the nature of the liquid and is called the coefficient of viscosity.
dv
is called the velocity gradient.
dx
dv
i.e., velocity gradient =
dx
dv
If A = 1 and = 1 , we have F = – h
dx
Mechanical Proper ties of Fluids 253

Thus the coefficient of viscosity of a liquid may be defined as the viscous force per unit area of the layer where velocity gradient is
unity.
The coefficient of viscosity has the dimensions [ML–1T–1] and its S.I.unit is newton second per square metre
(Nsm–2) or kilogram per meter per second (kgm–1s–1). In CGS the unit of viscosity is poise.
Effect of Temperature and Pressure on Viscosity
Effect of temperature : On increasing temperature viscosity of a liquid decreases.
Effect of pressure : On increasing pressure viscosity of a liquid increases except water whose viscosity decreases with pressure ries.
STOKE’S LAW
When a solid moves through a viscous medium, its motion is opposed by a viscous force depending on the velocity and shape and
size of the body. The energy of the body continuously decreases in overcoming the viscous resistance of the medium. This is why
cars, aeroplanes etc. are shaped streamline to minimize the viscous resistance on them.
The viscous drag on a spherical body of radius r, moving with velocity v, in a viscous medium of viscosity h is given by
Fviscous = 6phrv
This relation is called Stokes' law. .
Importance of Stoke’s law:
(i) It is used in the determination of electronic charge with the help of Milikan’s experiment.
(ii) It accounts the formation of clouds.
(iii) It accounts why the speed of rain drops is less than that of a body falling freely with a constant velocity from the height of clouds.
(iv) It helps a man coming down with the help of a parachute.
TERMINAL VELOCITY
It is the maximum constant velocity acquired by the body while falling freely in a viscous medium.
Let a body be falling in a viscous medium
Fb Fv
Three types of forces act on it
(i) Fb , the bouyant force, vertically upward
Motion
(ii) FV , the viscous force, vertically upward
(iii) W , the weight, vertically downward
W
Fb + FV = W .......... (1)
4 3
Fb = upward buoyant force = pr r0g , FV = upward viscous darg = 6 p h r v
3
4 3
W = weight of the body acting vertically downward = pr rg.
3
Here r = density of the body and r0 = density of the liquid
By putting these values in equation (1) and on solving it we get
2r 2 (r - r0 ) g
Terminal velocity, v = iscosity
9h
Critical velocity : The critical velocity is that velocity of liquid flow, upto which its flow is streamlined and above which its flow
Kh
becomes turbulent. It is given by vC =
rr
REYNOLD’S NUMBER
The stability of laminar flow is maintained by viscous forces. It is observed, however that laminar or steady flow is disrupted if the rate
of flow is large. Irregular, unsteady motion, turbulence, sets in at high flow rates.
Reynolds defined a dimensionless number whose value gives an approximate idea, whether the flow rate would be turbulent. This
rvD
number, called the Reynolds number Re =
h
Where r is the density of the fluid flowing with a speed v. The parameter D stands for the typical dimension of the obstacle or
boundary to fluid flow.
It is found that flow is streamline or laminar for Re less than 1000. The flow is turbulent for Re > 2000. The flow becomes unsteady for
Re between 1000 and 2000. The critical value of Re (known as critical Reynolds number), at which turbulence sets, is found to be the
254 Physics
same for the geometrically similar flows. For example when oil and water with their different densities and viscosities, flow in pipes of
same shapes and sizes, turbulence sets in at almost the same value of Re.
Illustration 9 :
A metal plate 100 cm2 in area rests on a layer of castor oil (h = 15.5 poise) 0.2 cm thick. Calculate the horizontal force
required to move the plate with a speed of 3 cm/s.
dv
Sol. As we know, F = -h A where h = 15.5 poise, A = 100 cm2
dx
dv 3
= = 15 s -1
dx 0.2
F = 15.5 ´ 100 ´15 = -23250 dyne = – 0.233 N

Illustration 10 :
(a) What is the largest average velocity of blood flow in an artery of radius 2 × 10–3 m if the flow must remain laminar?
(b) What is the corresponding flow rate? Take coefficient of viscosity of blood to be 2.084 × 10 –3pa s.
Sol. (a) The maximum value of Reynold's number for laminar flow is 2000. Hence, the critical velocity is
R eh 2000 ´ 2.084 ´ 10 -3
vc = = = 0.98 m/s
rD 1.06 ´ 10 3 ´ 4 ´ 10 -3
22
(b) Volume of blood flowing per second = avc = pr2vc = ´ (2 ´10 -3 ) 2 ´ 0.98 = 1.23 × 10–5m3
7
SURFACE TENSION
It is the property of liquid at rest due to which the free surface of liquid tends to have minimum area.
It is measured as the force experienced per unit length of an imaginary line drawn on the surface of the liquid i.e., . S = F/l
It is also defined as work done in increasing the area of a liquid surface by unity against the force of surface tension at a constant
temperature. Its S.I. unit is N/m.
Molecular theory of surface tension. In figure, PQRS is the surface film of a liquid. Consider the molecule A well inside the liquid. It is
attracted equally in all directions by the molecules lying in its sphere of influence. Net force on such a molecule is zero.
Now consider molecule B lying inside the surface film. Its sphere of influence lies partly
outside. This molecule experiences less force upward and more force downward by the
molecules in its sphere of influence. For molecule C, half its sphere of influence lies C B
above the surface. The resultant downward force on such a molecule is maximum. Due to P Q
this downward force, the potential energy of the molecules of the surface film is higher S R
than those lying well inside the liquid. For a system to be stable, potential energy must
be minimum. For the surface film to have minimum energy, the number of molecules in it
must be minimum. Thus the surface film tends to have minimum surface area. As a result,
the free surface of a liquid at rest behaves like an elastic stretched membrane.
A
Examples of surface tension
(i) Raindrops are spherical in shape.
(ii) The hair of a shaving brush cling together when taken out of water.
(iii) Oil spread on cold water but remains as a drop on hot water etc.
SURFACE ENERGY
According to molecular theory of surface tension the molecules in the surface have some additional energy due to their position.
This additional energy per unit area of the surface is called surface energy.
Work done
i.e., Surface energy = Increase in surface area
Its SI unit is Jm–2
Surface Energy and Surface Tension A
Let a liquid film be formed on a wire frame and a straight wire of length l can slide on this
wire frame as shown in figure. The film has two surfaces and both the surfaces are in
contact with the sliding wire and hence, exert force of surface tension on it. If T be the F
surface tension of the solution, each surface will pull the wire parallel to itself with
a force Tl. Thus, net force on the wire due to both the surface is 2Tl. Apply an B
external force F equal and opposite, to keep the wire in equilibrium. Thus, F = 2Tl
Now, suppose the wire is moved through a small distance dx, the work done by the force is, dW = F dx = (2Tl) dx
But (2l) (dx) is the total increase in area of both the surface of the film. Let it be dA.
Mechanical Proper ties of Fluids 255

Then, dW = T dA
dW
Þ T =
dA
Thus, the surface tension T can also be defined as the work done in increasing the surface area by unity. Further, since there
is no change in kinetic energy, the work done by the external force is stored as the potential energy of the new surface.
dU
T= [as dW = dU]
dA
Angle of Contact (q)
The angle enclosed between the tangent plane at the liquid surface and the tangent plane at the solid surface at the point of
contact inside the liquid is termed as the angle of contact.
The angle of contact depends on the nature of the solid and liquid in contact.
Shape of liquid surface : When a liquid is brought in contact with a solid surface, the surface of the liquid becomes curved near
the place of contact. The shape of the surface (concave or convex) depends upon the relative magnitudes of the cohesive force
between the liquid molecules and the adhesive force between the molecules of the liquid and the solid.
The free surface of a liquid which is near the walls of a vessel and which is curved because of surface tension is known as
meniscus. The cohesive force acts at a n angle 45° from liquid surface whereas the adhesive force acts at right angles to the solid
surface.
For liquids that wet the solid surface e.g., glass-water, the angle of contact qc < 90° (acute angle) and the shape of meniscus is concave.
And for liquids that does not wet the solid surface e.g., glass-mercury, the angle of contact qc > 90° (obtuse angle and the shape of
meniscus is convex.
Drops and Bubbles
Excess pressure inside a liquid drop. Consider a spherical liquid drop of radius R. Let s be the surface tension of the liquid. Due to its
spherical shape, there is an excess pressure p inside the drop over that on outside. This excess pressure acts normally outwards. Let
the radius of the drop increase from R to R + dR under the excess pressure p.
Initial surface are = 4p R2
Final surface area
= 4p (R + dR)2 = 4p (R2 + 2 R + dR2)
= 4p R2 + 8p R dR
2
dR is neglected as it is small.
Increase in surface area dR
= 4p R2 + 8p R dR – 4p R2 = 8p R dR p R
Work done in enlarging the drop
= Increase in surface energy
= Increase in surface area × Surface tension = 8p R dR s
But work done = Force × Distance
= Pressure × Area × Distance = p × 4p R2 × dR
Hence, p × 4p R2 × dR = 8p R dR s
2s
Excess pressure, p = .
R
Excess pressure inside a soap bubble. Proceeding as in the case of a liquid drop in the above question, we obtain
Increase in surface area = 8p R dR
But a soap bubble has air both inside and outside, so it has two free surfaces.
\ Effective increase in surface area
= 2 × 8p R dR = 16p R dR
Work done in enlarging the soap bubble
= Increase in surface energy
= Increase in surface area × Surface tension
= 16p R dR s
But, Work done = Force × Distance
= p × 4p R2 × dR
Hence
p × 4p R2 × dR = 16p R dR s
4s
\ p= .
R
256 Physics
Excess pressure inside an air bubble inside a liquid. An air bubble inside a liquid is similar to a liquid drop in air. It has only one free
spherical surface.
Hence excess pressure is given by
2s
p= .
R
Capillary Rise
A glass tube with fine bore and open at both ends is known as capillary tube. The property by virtue of which a liquid rise or
fall in a capillary tube is known as capillary rise or capillary fall or capillarity. Rise or fall of liquid in tubes of narrow bore
(capillary tube) is called capillary action. Rise of kerosene in lanterns, rise of ink in fountain pen etc. are due to capillary action.
Let the radius of the meniscus is R and the radius of the capillary tube is r. The angle of contact is q, surface tension is T, density
of liquid is r and the liquid rises to a height h.
Now let us consider two points A and B at the same horizontal level as shown. By Pascal's law
PA = PB Þ PA = PC + rgh
Þ PA – PC = r gh (Q PB = PC + rgh)
Now, the point C is on the curved meniscus which has PA and PC as the two pressures on its concave and convex sides
respectively.
2T 2T
Þ PA – PC = =
R r / cos q
2T
Þ = rgh Þ 2T cos q = rr gh ....(i)
r / cos q

2T cos q
Þ h=
rr g
Height of liquid rise in capillary tube
Thus the height of rise of liquid in a capillary tube is inversely proportional to the radius of the capillary tube.
1
If T, q, r and g are constant, h µ or rhh = constant.
r
This is Zurin’s law. It implies that liquid will rise more in capillary tube of less radius and vice versa.
Illustration 11 :
The length of a needle floating on water is 2.5 cm. Calculate the added force required to pull the needle out of water. Given :
T = 7.2 × 10–2 N/m
Sol. The force due to surface tension, F = T × 2l
(Q two free surfaces are there)
Þ F = 7.2 × 10–2 × 2 × 2.5 × 10–2 = 3.6 × 10–3 N
Illustration 12 :
Calculate the excess pressure within a bubble of air of radius 0.1 mm in water. If the bubble had been formed 10 cm below the water
surface on a day when the atmospheric pressure was 1.013 × 105 Pa, then what would have been the total pressure inside the bubble?
Surface tension of water = 73 × 10–3 N/m
Sol. Excess pressure,
2T 2 ´ 73 ´ 10-3
Pexcess = = = 1460 Pa
r 0.1 ´ 10-3
The pressure at a depth d, in liquid is P = hdg. Therefore, the total pressure inside the air bubble is
2T
Pin = Patm + hdg +
r
= (1.013 × 105) + (10 × 10–2 × 103 × 9.8) + 1460
= 101300 + 980 + 1460 = 103740 = 1.037 × 105 Pa
Mechanical Proper ties of Fluids 257

Illustration 13 :
Calculate the work done against surface tension in blowing a soap bubble from a radius 10 cm to 20 cm if the surface tension of soap
solution is 25 × 10–3 N/m. Then compare it with liquid drop of same radius.
Sol. (i) For soap bubble : Extension in area

= 2 ´ 4pr22 - 2 ´ 4 pr12 = 8p [(0.2) 2 - (0.1) 2 ] = 0.24 p m2


Work done W1 = surface tension × extension in area
= 25 × 10–3 × 0.24p = 6p × 10–3 J
(ii) For liquid drop: In case of liquid drop only one free surface, so extension in area will be half of soap bubble
W1
\ W2 = = 3p ´ 10-3 J
2

Illustration 14 :
Calculate the height to which water will rise in a capillary tube of diameter 1 × 10–3 m.
[Given: Surface tension of water = 0.072 N m–l, angle of contact = 0°, acc. due to gravity = 9.8 m s–2 and density of water = 1000
kg m–3]
Sol. Height of capillary rise

2T cos q 2 ´ 0.072 ´ cos 0°


h= = = 2.94 × 10–2 m
rrg 5 ´ 10-4 ´1000 ´ 9.8

DETERGENT AND SURFACE TENSION


Cleansing action of detergents: Water does not remove greasy stains on dirty clothes because water does not wet greasy dirt i.e.,
there is very little area of contact between them. To remove greasy stains we add detergent or soap to water. The molecules of
detergents are hairpin shaped with one end attracted to water and the other to molecules of grease oil or wax thus tending to form
water-oil interface and hence surface tension is greatly reduced. The greasy dirt is held suspended and when the clothes are rinsed in
water, the greasy dirt is washed away by running water.
258 Physics
Mechanical Proper ties of Fluids 259

Textbook Exercises
10.1 Explain why
SLA
(a) The blood pressure in humans is greater at the feet
than at the brain? Liquid SSA
(b) Atmospheric pressure at a height of about 6 km
decreases to nearly half its value at the sea level, Solid SSL
though the 'height' of the atmosphere is more than
100 km (b) For mercury – glass, angle of contact is obtuse. In
(c) Hydrostatic pressure is a scalar quantity even though order to achieve this obtuse value of angle of contact,
pressure is force divided by area. the mercury tends to form a drop. In case of water –
Sol. (a) The height of the blood column in the human body is glass, the angle of contact is acute. To achieve this
more at feet than at the brain. That is why, the blood acute value of angle of contact, the water tends to
exerts more pressure at the feet, than at the brain.
spread.
(Q pressure = h r g) (c) Surface tension of liquid is the force acting per unit
(b) We know that the density of air is maximum near the length on a line drawn tangentially to the liquid surface
surface of the earth and decreases rapidly with height
at rest. Since, this force is independent of the area of
and at a height of about 6 km it decreases to nearly half
liquid surface, therefore, surface tension is also
its value at the sea level. Beyond 6 km height, the
density of air decreases very slowly with height. Due independent of the area of the liquid surface.
to this reason, the atmospheric pressure at a height of (d) We know that the cloth has narrow spaces in the form
about 6 km decreases to nearly half of its value at sea of capillaries. The rise of liquid in a capillary tube is
level. directly proportional to cosq. If q is small cosq will be
(c) Since due to applied force on liquid, the pressure is large. Due to which capillary rise will be more so the
transmitted equally in all directions inside the liquid. detergent will penetrate more in cloth.
That is why, there is no fixed direction for the pressure (e) In the absence of external forces, the surface of the
due to liquid. Hence hydrostatic pressure is a scalar liquid drop tends to acquire the minimum surface area
quantity. due to surface tension. Since, for a given volume, the
10.2. Explain why surface area of sphere is least, hence the liquid drop
(a) The angle of contact of mercury with glass is obtuse, takes the spherical shape.
while that of water with glass is acute? 10.3 Fill in the blanks using the words from the list appended
(b) Water on a clean glass surface tends to spread out with each statements:
while mercury on the same surface tends to form (a) Surface tension of liquids generally ___________
drops? with temperature. (increases/decreases)
(c) Surface tension of liquid is independent of the area of (b) Viscosity of gases __________ with temperature,
the liquid surface? whereas viscosity of liquids ____________ with
(d) Detergents should have small angles of contact? temperature. (increases/decreases)
(e) A drop of liquid under no external forces is always (c) For solids with elastic modulus of rigidity, the
spherical in shape. shearing force is proportional to ____________
Sol. (a) When a small quantity of liquid is poured on solid, while for fluids it is porportional to __________ .
three interfaces namely liquid – air, solid – air and solid (shear strain/rate of shear strain)
– liquid are formed. The surface tensions (d) For a fluid in steady flow, the increases in flow speed
corresponding to these three layers are SLA, SSA and at a constriction follows from ___________ while
SSL respectively. Let q be the angle of contact between the decrease of pressure there follows from
the liquid and solid. The molecules in the region, where ____________ . (conservation of mass/
the three interfaces meet are in equilibrium. It means Bernoulli’s principle)
net force acting on them is zero. For the molecule at O (e) For the model of a plane in a wind tunnel, turbulence
to be in equilibrium, we have occurs at a _________ speed than the critical speed
for turbulence for an actual plane.(greater/smaller)
SSA , SSL
SSL + SLA cosq = SSA or cosq = Sol. (a) decreases
SLA (b) increases; decreases
In case of mercury – glass, SSA < SSL, therefore cosq is (c) shear strain; rate of shear strain
negative or q > 90° i.e obtuse. In case of water – glass, (d) conservation of mass; Bernoulli’s principle
SSA > SSL, therefore, cos q is positive q < 90° i.e. acute. (e) greater.
260 Physics
10.4 Explain why
0.76≥13.6≥103 ≥9.8
(a) To keep a piece of paper horizontal, you should blow or h= = 10.5 m.
over, not under, it. 984≥9.8
(b) When we try to close a water tap with our fingers, 10.7 A vertical off shore structure is built to withstand a
fast jets of water gush through the openings between maximum stress of 109 Pa. Is the structure suitable for
our fingers. putting upon top of an oil well in Bombay High? Take the
(c) The size of a needle of a syringe controls flow rate depth of the sea to be roughly 3 km, and ignore the ocean
better than the thumb pressure exerted by a doctor currents.
while administering an injection. Sol. Here, maximum stress = 109Pa, h = 3km = 3 × 103m;
(d) A fluid flowing out of a small hole in a vessel results P (water) = 103 kg/m3 and g = 9.8 m/s2. The structure will be
in a backward thurst on the vessel. suitable for putting upon top of an oil well provided the
(e) a spinning cricket ball in air does not follow a pressure exerted by sea water is less than maximum stress it
parabolic trajectory. can bear.
Sol. (a) When we blow over the piece of paper, the velocity of Pressure due to sea water, P = h r g = 3 × 103 × 103 × 9.8
air increases. As a result, the pressure on it decreases = 2.94 × 107 Pa
in accordance with the Bernoulli’s theorem whereas Since, the pressure of sea water is less than the maximum
the pressure below remains the same (atmospheric stress of 109Pa, the structure will be suitable for putting
pressure). Thus, the paper remains horizontal. upon tap of the oil well.
(b) By doing so the area of outlet of water jet is reduced, 10.8 A hydraulic automobile lift is designed to lift cars with a
so velocity of water increases according to equation maximum mass of 3000 kg. The area of cross - section of
of continuity av = a constant.
the piston carrying the load is 425 cm2. What maximum
(c) For a constant height, the Bernoulli’s theorem is
pressure would smaller piston have to bear?
expressed as
Sol. The maximum force, which the bigger piston can bear, F =
1 3000 kg, f = 3000 × 9.8 N
P + rv 2 = constant
2 Area of piston, A = 425 cm2 = 425 × 10–4 m2
In this equation, the pressure P occur with a single \ Maximum pressure on the bigger piston,
power whereas the velocity occurs with a square power. F 3000≥9.8
P= = = 6.92 × 105 Pa
Therefore, the velocity has more effect compared to A 425≥10,4
the pressure. It is for this reason that needle of the Since, the liquid transmits pressure equally,
springe controls flow rate better than the thumb \ the maximum pressure the smaller piston can bear is
pressure exerted by the doctor. 6.92 × 105 Pa.
(d) This is because of principle of conservation of 10.9 A U tube contains water and methylated spirit separated by
momentum. While the flowing fluid carries forward mercury. The mercury columns in the two arms are in
momentum, the vessel gets a backward momentum. level with 10.0 cm of water in one arm and 12.5 cm of spirit
(e) A spinning cricket ball would have followed a parabolic in the other, what is the relative density of spirit?
trajectory has there been no air. But because of air the Sol. For water column in one arm of U tube, h1 = 10.0 cm;
Magnus effect take place. Due to the Magnus effect r1 (density) = 1g cm–3
For the spirit column in the other arm of U tube, h 2 = 12.5
the spinning cricket ball deviates from its parabolic
cm; r2 = ?
trajectory.
As the mercury columns in the two arms of U tube are in
10.5 A 50 kg girl wearing high heel shoes balances on a single
level,
heel. The heel is circular with a diameter 1 cm. What is
\ pressure exerted by each is equal.
the pressure exerted by the heel on the horizontal floor?
Hence, h1r1g = h 2r2g
D 1 1
Sol. Here, m = 50 kg; r = = cm = m h1θ1 10 ≥1
2 2 200 or P2 = = = 0.8 g cm–3
h2 12.5
force mg
Pressure = = 2 θ2 0.8
area πr \ Relative density of spirit = =
θ1 1
10.10 In Q.10.9, if 15.0 cm of water and spirit each are further
50 ≥9.8
= = 6.24 × 106 Nm–2 poured into the respective arms of the tube, what is the
(22 / 7) ≥ (1/ 200) 2 difference in the levels of mercury in the two arms? (Specific
10.6 Torricelli's barometer used mercury. Pascal duplicated it gravity of mercury = 13.6)
using French wine of density 984 kgm–3. Determine the Sol. On pouring 15.0 cm of water and spirit each into the
height of the wine column for normal atmospheric respective arms of the U tube, the mercury level will rise in
pressure. the arm containing spirit. Let h be the difference in the levels
Sol. P = 0.76 × (13.6 × 103) × 9.8 = h × 984 × 9.8 of mercury in the two arms of U tube and let r be the
density of mercury.
Mechanical Proper ties of Fluids 261

\ The pressure exerted by h cm of mercury column =


P1 1 P 1
difference in pressure exerted by water and spirit. + gh + v12 = 2 + gh + v22
\ h r g = h1 r 1 g – h2 r 2 g θ 2 θ 2
Here, h = ?; r = 13.6 g cm–3; h1 = 15 + 10 = 25 cm; 1
P1 P
r 1 = 1g cm–3; h2 = 15 + 12.5 = 27.5 cm; \ – 2 = (v22 – v12 )
θ θ 2
r 2 = 0.8 g cm–3
Putting values in (i), we get 1 1
h × 13.6 × g = 25 × 1 × g – 27.5 × 0.8 g = 3 g or r1 – r2 = r (v22 – v12 ) = × 1.3 [(70)2 – (63)2]
2 2
3 = 605.15 Pa
or h= = 0.22 cm.
13.6 The difference of pressure provides the lift to the aeroplane.
10.11Can Bernoulli’s equation be used to describe the flow of So, lift on the aeroplane = pressure difference × area of
water throught a rapid motion in a river? Explain.
wings = 605.15 × 2.5 = 1512.875 N = 1.51 × 103 N.
Sol. Bernoulli’s theorem is applicable only for the ideal fluids in
10.15 Figures (a) and (b) refer to the steady flow of a (non-
streamlined motion. Since the flow of water in a river in a
viscous) liquid. Which of the two figures in incorrect?
rapid way cannot be treated as streamlined motion, the Why?
theorem cannot be used.
10.12 Does it matter if one uses gauge instead of absolute
pressure in applying Bernoulli’s equation? Explain.
Sol. No, it does not matter if one uses gauge instead of absolute
pressures in applying. Bernoulli’s equation, provided the
atmospheric pressure at the two points where Bernoulli’s
equation is applied are significantly different.
(a) (b)
10.13 Glycerine flows steadily through a horizontal tube of length
1.5 m and radius 1.0 cm. If the amount of glycerine collected Sol. Figure (a) is incorrect. It is because of the fact that at the
per second at one end is 4.0 × 10–3 kg s–1, what is the kink, the velocity of flow of liquid is larrge and hence using
pressure difference between the two ends of the tube? the Bernoulli’s theorem the pressure is less. As a result, the
(density of glycerine = 1.3 × 103 kg m–3 and viscosity of water should not rise higher in the tube where there is a kink
glycerine = 0.83 Pa. (i.e., where the area of cross-section is small).
10.16 The cylindrical tube of a spray pump has a cross - section
Sol. Here, l = 1.5 m; r = 1.0 cm = 10–2 m; P = 1.3 × 103 kg/m3;
of 8.0 cm2 one of which has 40 fine holes each of diameter
h = 0.83 Pa 1.0 mm. If the liquid flow inside the tube is 1.5 m per minute,
Mass of glycerine flowing per sec, M = 4 × 10–3 kg/s what is the speed of ejection of the liquid through the holes?
Volume of glycerine flowing per second, Sol. Area of cross-section of tube, a1 = 8.0 cm2 = 8 × 10–4 m2,
No. of holes = 40; Diameter of each hole, D = 1mm = 10–3 m
M 4 ≥10,3 3 –1
V= = ms D 1
P 1.3≥103 \ Radius of hole, r = = × 10–3m = 5 × 10–4 m
2 2
If P is the difference of pressure between two ends of the
Area of cross-section of each hole = p r2 = p (5 × 10–4)2 m2
tube, then using Poiseuille's formula we have
Total area of cross section of 40 holes,
οθr 4 V ≥8γL a2 = 40 × p (5 × 10–4)2 m2
V= or p=
8γL οr 4 1.5
Speed of liquid inside the tube, v1 = 1.5 m/min = ms–1
60
æ 4 ´10-3 ö 8≥ 0.83≥1.5 If v2 is the velocity of the ejection of the liquid through the
\ P= ç 3÷
× = 975.37 Pa
è 1.3 ´10 ø 3.142 ≥ (10,2 ) 4
holes, then a1 v1 = a2 v2
(8≥10,4 ) ≥1.5
10.14 In a test experiment on a model aeroplane in a wind tunnel, or v2 = Þ v2 = 0.637 ms–1
the flow speeds on the upper and lower surfaces of the wing 60≥ 40≥ο≥ (5≥10,4 ) 2
10.17 A U-shaped wire is dipped in a soap solution an removed
are 70 ms–1 and 63 ms–1 respectively. What is the lift on thin soap film formed between the wire and a light slider
the wing if its area is 2.5 m2? Take the density of air 1.3 kg supports a weight of 1.5 × 10–2 N (which includes the small
m–3. weight of the slider). The length of the slider is 30 cm.
Sol. Let v1, v2 be the speeds on the upper and lower of the wing What is the surface tension of the film?
of aeroplane, and P1 and P2 are the on upper and lower Sol. Since, soap film has two free surfaces, so total length of the
surfaces of the wing respectively. Then v1 = 70 ms–1; v2 = film = 2 l = 2 × 30 = 60 cm = 0.6 m.
63 ms–1, r = 1.3 kg m–3. Total force on the slider due to surface tension,
From Bernoulli's theorem F = S × 2 l = 5 × 0.6 N
In equilibrium, F = mg
262 Physics
\ S × 0.6 = 1.5 × 10–2 10.19 What is the pressure inside a drop of mercury of radius 3
,2 mm at room temperature? Surface tension of mercury at
1.5≥10
\ S= = 2.5 × 10–2 Nm–1 that temperature (20° C) is 4.65 × 10–1 Nm–1. The
0.6 atmospheric pressure is 1.01 × 105 Pa. Also give the excess
10.18. Figure (a) below shows a thin film supporting a small
pressure inside the drop.
weight = 4.5 × 10–2 N. What is the weight supported by a
Sol. Given, r = 3mm = 0.3 m, S = 4.65 × 10–1 Nm–1, P =1.01 × 105 Pa
film of the same liquid at the same temperature in Fig. (b)
Excess of pressure inside the drop of mercury is given by, p
and (c)? Explain your answer physically.
2S 2 ´ 4.65 ´ 10 -1
= = = 310 Pa
r 3 ´ 10 -3
10.20 What is excess pressure inside a bubble of soap solution
of radius 5 mm, given that the surface tension of soap
solution at the temperature (20°C) is 2.5 × 10–2 Nm–1? If
an air bubble of the same dimension were formed at a depth
of 40 cm inside a container containing the soap solution (of
relative density 1.2), what would be the pressure inside the
40 cm 40 cm bubble? (1 atm = 1.01 × 105 Pa)
a b c
40 cm Sol. Given, S = 2.5 × 10–2 Nm–1, r = 5 mm = 5 × 10–3 m
Sol. (a) Here, length of the film supporting the weight = 40 cm = Density of soap solution, p = 1.2 × 103 kg m–3
0.4 m. Excess pressure inside the soap bubble,
Total weight supported (or force) = 4.5 × 10 –2 N
4S 4 ´ 2.5 ´ 10 - 2
Film has two free surfaces, p= = = 20 Pa.
r 5 ´ 10 - 3
4.5 ´ 10-2 Excess pressure inside the air bubble,
\ Surface tension, S = = 5.625 ´ 10 -2 Nm -1.
2 ´ 0.4
2S 2 ´ 2.5 ´ 10 - 2
Since the liquid is same for all the cases (a), (b) and (c), and p' = = = 10 Pa.
temperature is also same, therefore surface surface tension r 5 ´ 10 - 3
for cases (b) and (c) will also be the same = 5.625 × 10–2 . In Total pressure inside the air bubble at depth h in soap
Fig. (a), (b) and (c), the length of the film supporting the solution,
weight is also (a), hence the total weight supported in each = p' + atmospheric pressure + h r g
case is 4.5 × 10–2 N. = 10 + 1.01 × 105 + 0.4 × 1.2 × 103 × 9.8 = 1.06 × 105 Pa.

10.21 A tank with a square base of area 1 m2 is divided by vertical 10.22 A manometer roads the pressure of a gas in an enclosure
partition in the middle. The bottom of the partition has a as shown in Fig. (a). When a pump removes some of the
small hinged door of area 20 cm2. The tank is filled with gas, the manometer reads as in fig. (b). The liquid used in
water and an acid (of relative density 1.7) in the other, both the manometers is mercury and the atmospheric pressure
to a height of 4 m. Compute the force necessary to keep the is 76 cm of mercury.
door closed.
Sol. For compartment containing water, h1 = 4 m,
r = 103 kg m–3
To pump
Pressure exerted by water at the door at the bottom,
p1 = h1r1 g = 4 × 103 × 9.8 = 3.92 × 104 Pa
For compartment containing acid, 20 cm
h2 = 4 m, P2 = 1.7 × 103 kg/m3 18 cm
The pressure exerted by acid at the bottom provided by at
the bottom,
p2 = h2 r2g = 4 × 1.7 × 103 × 9.8 = 6.664 × 104 Pa
[ Difference of pressure = P2 – P1
= 6.664 × 104 – 3.92 × 104 = 2.774 × 104 Pa (a) (b)
Given, area of door, A = 20 cm2 = 20 ×10–4 m2
Force on the door = difference in pressure × area (a) Give the absolute and gauge pressure of the gas in
= (P2 – P1) × A the enclosure for cases (a) and (b), in units of cm of
= (2.774 × 104) × (20 × 10–4) = 54.88 N » 55 N mercury.
[ Th e force equal to 55 N needs to be applied (b) How would the levels change in case (b) if 13.6 cm of
horizontally on the door, to keep the door closed. water (immiscible with mercury) are poured into the
Mechanical Proper ties of Fluids 263

right limb of the manometer? Ignore the small change become more important as the fluid velocity increase?
in the volume of the gas. Discuss qualitively.
Sol. The atmospheric pressure, P = 76 cm of mercury Sol. (a) If dissipative forces are present, then some forces in
(a) From figure (a), liquid flow due to pressure difference is spent against
Pressure head, h = 20 cm of mercury
dissipative forces. Due to which the pressure drop
\ Absolute pressure = p + h = 76 + 20 = 96 cm of
mercury becomes large.
Also, Gauge pressure = h = 20 cm of mercury (b) The dissipative forces become more important with
From figure (b), increasing flow velocity, because of turbulence.
pressure head, h = – 18 cm of mercury 10.26 (a) What is the largest average velocity of blood flow in
\ Abolute pressure = p + h = 76 + (–18) = 58 cm of an artery of radius 2 × 10–3 m if the flow must remain
mercury laminar?
Also, Gauge pressure = h = – 18 cm of mercury. (b) What is the corresponding flow rate? Take viscosity
(b) When 13.6 cm of water is poured into the right limb of of blood to be 2.084 × 10–3 pa-s. Density of blood is
the manometer of figure (b), the, using the relation: 1.06 ×103 kg/m3.
Pressure = rgh = r¢g¢h¢
Sol. Given, r = 2 × 10–3 m, D = 2r = 2 × 2 × 10–3 m = 4 × 10 m –3
rh 1 ´ 13.6 h = 2.084 × 10–3 Pa-s, r = 1.06 × 103 kgm–3
We get h ¢ = = = 1 cm of mercury [P’ =
r¢ 13.6
2000 ´ (2.084 ´10 -3 )
density of mercury] (a) Now, Ve = NR h /rD =
therefore, pressure at the point B (1.06 ´ 10 -3 ) ´ (4 ´10 -3 )
PB = P + h’ = 76 + 1 = 77 cm of mercury. = 0.98 m/s
If h¢¢ is the difference in the mercury levels in the two
(b) Volume flowing per second
limbs, then taking PA = PB
Þ 58 + h¢¢ = 77 Þ h¢¢ = 77 – 58 = 19 cm of mercury. 22
´ (2 ´ 10-3 ) 2 ´ 0.98 = 1.23 ´10 -5
= pr2Ve =
10.23. Two vessels have the same base area but different shapes. 7
The first vessel takes twice the volume of water that the 10.27 A plane is in level flight at constant speed and each of its
second vessel requires to fill up to a particular common wings has an area of 25m3. If the speed of the air is 180
height. Is the force exerted by the water on the base of the km/h over the lower wing and 234 km/h over the upper
vessel the same in the two cases? If so, why do the vessels wing surface, determine the plane's mass. [Take air density
filled with water to that same height give different readings = 1kgm–3, g = 9.8 ms–2.]
on a weighing scale? Sol. Given, v1 = 180 km/h = 50 m/s, v2 = 234 km/h = 65 ms–1, A =
2 × 25 = 50 m2, r = 1kgm–3.
Sol. Pressure (and therefore force) on the two equal base areas
P1 – P2 =1/2r (v22 – v12) = 1/2 × 1 × (652 – 502)
are identical. But force is exerted by water on the sides of
Upward force = (P1 – P2)A = 1/2 [652 – 502] × 50N
the vessels also, which has a non-zero vertical component As the plane is in level flight, mg = (P1 – P2)A
when sides of the vessel are not perfectly normal to the
base. This net vertical component of force by water on the (P1 - P2 )A 1´ [652 - 50 2 ] ´ 50
or, m = = = 4.4 × 103 N
sides of the vessel is greater for the first vessel than the g 2 ´ 9. 8
second. Hence, the vessels weigh different even when the 10.28 In Millikan's oil drop experiment, what is the terminal
force on the base is the same in the two cases. speed of a drop of radius 2 × 10–5 m and density 1.2 × 103
10.24 During the blood transfusion the needle is inserted in a kgm–3? Take the viscosity of air at the temperature of the
vein where the gauge pressure is 2000 Pa. At what height experiment to be 1.8 × 10–5 Nsm–2. How much is the viscous
must the blood container be placed so that blood may just force on the drop at that speed? Neglect buoyancy of the
drop due to air:
enter the vein? Density of whole blood = 1.06 × 103 kgm–3.
Sol. Given, r = 2 × 10–5 m, r = 1.2 × 103 kgm–3,
P 2000 h = 1.8 × 10–5 Nsm–2, r 0 = 0
Sol. h = = = 0.1925 m
θg 1.06 ´ 103 ´ 9.8 Terminal velocity,
The Blood may just enter the vein if the may just container 2r 2 (r - r0 )g 2 ´ (2 ´10 -5 ) 2 (1.2 ´103 - 0)
v= = ´ 9.8
height at which the blood kept must be slightly greater than 9h 9 ´ 10.8 ´ 10 -5
0.1925 m » 0.2 m. = 5.8 cms–1
10.25 In deriving Bernoulli’s equation, we equated the work Viscosity force on the drop,
done on the fluid in the tube to its change in the potential
22
and kinetic energy. (a) What is the largest average velocity F = 6phrv = 6 ´ ´ (108 ´ 10 -5 ) ´ (2.0 ´ 10 -5 ) ´ (5.8 ´ 10 - 2 )
of blood flow in an artery of diameter 2 × 10 –3 m if the 7
flow must remain laminar? (b) Do the dissipative forces = 3.93 × 10– 10 N
264 Physics
10.29 Mercury has an angle of contact equal to 140° with (b) A large He balloon of volume 1425 m3 is used to lift
sodalime glass. A narrow tube of radius 1 mm made of this a payload of 400 kg. Assume that the balloon
glass is dipped in a trough containing mercury. By what maintains constant radius as it rises. How high does
amount does the mercury dip down in the tube relative to it rise? [Take y0 = 8000 m and rHe = 0.18 kg m–3]
the mercury surface outside? Surface tension of mercury Sol. (a) We know that rate of decrease of density r of air is
at the temperature of the experiment is 0.465 Nm–1. Density directly proportional to the height y. It is given as
of mercury = 13.6 × 103 kgm–3.
dr r
Sol. Given, q = 140°, r = 1 × 10–3 m, s = 0.465 Nm–1; =- ,
dy y0
r = 13.6 × 103 kg m–3, cos 140° = – cos40° = – 0.766
where y is a constant of proportionality and –ve sign
2s cos q 2 ´ 0.465 ´ cos 140° signfies that density is decreasing with increase in
Now, h = = -3 = – 5.34 mm
rrg 10 ´13.6 ´103 ´ 9.8 height. on integration, we get
Negative value indicates that mercury level is depressed in r y
dr 1
the tube. ò r
= -ò
y0
dy
10.30 Two narrow bores of diameters 3 mm and 6 mm are joined r0 0
together to form U shaped tube open at both ends. If the U
tube contains water, what is the difference in its levels in y
r é yù
the two limbs of the tube? Surface tension of water at the Þ [log r]r = - ê ú , where, r0 = density of air at
temperature of the experiment = 7.3 × 10–2 Nm–1. Take the
0
ë y0 û0
angle of contact tube zero, and density of water = 103 kgm– sea level i.e., y = 0
3 (g = 9.8 ms–2).
y
-
Sol. Given: S = 7.3 × 10–2 Nm–1, r = 103 kgm–3, q = 0° r y
or log e = = r0 e y0
or r
For narrow tube, 2r1 = 3mm = 3 × 10–3m or, r1 = 1.5 × 10–3 m r0 y 0
For wider tube, 2r2 = 6 mm = 6 × 10–3 m or, r2 = 3 × 10–3 m Hence dimensions and units of constant y0 are same
Let h1, h2 = heights to which water rise in narrow tube and as of y.
wider tube respectively. (b) Here volume of He,ballon, V = 1425 m3 , mass of
2Scos q 2Scos q payload,m = 400 kg
\ h1 = and h 2 = y0 = 8000 m, density of He rHe = 0.18 kgm–3
r1rg r2 rg
Mean den sity of balloon,
\ Difference in levels of water in two limbs of U tube is,
Total mass of balloon m + V × rme
2Scos q é 1 1 ù r= =
h1 - h 2 = ê - ú Volume V
rg ë r1 r2 û
400 + 1425 ´ 0.18
= = 0.4608 = 0.46 kgm -3
2 ´ 7.3 ´10-2 cos 0° é 1 1 ù 1425
= ê - ú
2
10 ´ 9.8 ë1.5 ´10 -3
3 ´ 10-3 û As density of air at sea level r0 = 1.25 kg m–3. The
balloon will rise up to a height y where density of air =
= 1.49 × 10–5 [0.33] × 103 = 4.917 × 10–3m
density of balloon r =0.46 kgm–3
10.31 (a) It is known that density r of air decreases with height
y y0
y as r = r0 e - y/ y0 -
y0 r
As r = r0 e or 0 = e y
Where r0 = 1.25 kg m–3 is the density at sea level, r
and y0 is a constant. This density variation is called
the law of atmospheres. obtain this law assuming æ r0 ö y0 y0 8000
that the temperature of atmosphere remains a \ loge ç ÷ = or y = =
è rø y ær ö æ 1.25 ö
log e ç 0 ÷ log e ç
constant (isothermal conditions). Also assume that è rø è 0.46 ÷ø
the value of g remains constant.
= 8002 m or 8.0 km.
Mechanical Proper ties of Fluids 265

Practice Questions

Very Short Answer Questions [1 Mark Qs.] depth of kerosene oil of relative density 0.8 must be poured
into the tube so that the plate just falls off?
1. When a vessel containing a liquid is falling freely, does 6. A piece of ice, with a stone frozen inside it, is floating in
Archimedes' principle hold good in it? water in a beaker . What will happen to the level of water in
2. A block of wood floats in a bucket of water in a lift. Will the beaker if the whole ice melts?
there be any change in the equilibrium of wood if the lift 7. Establish a relation between surface tension and surface
starts accelerating upwards? energy.
3. A piece of ice is floating in a vessel containing water and A x A'
inside the ice there is an air bubble. What will be the effect Q P
on the level of water, when the ice melts?
4. A body is just floating in a liquid whose density is equal to
that of the body. What will happen to the body if it is slightly S F
pressed and released?
5. Why is the water reservoir of a dam thicker at the bottom?
6. What is the value of surface tension of a liquid at critical R S
temperature? B B'
7. When a liquid drop splits into large number of small drops, 8. Derive an expression for the excess pressure inside a soap
what is the change in the potential energy of small drops bubble.
and the potential energy of the bigger drop?
8. What will be the effect on the angle of contact of a liquid if
the temperature increases? dR
9. Surface tension of lubricating oils and paints is generally P
low. Why?
R
10. The terminal velocity of a sphere of radius r in a viscous
liquid is V. What is the terminal velocity of a sphere of radius
2r and of the same density in a given viscous fluid?
11. An incompressible fluid flows steadily through a cylindrical 9. On what factors does the angle of contact depend ? Where
pipe which has radius 2R at point A and radius R at point B, is the angle of contact obtuse, acute or, two degree?
along the flow direction. If the velocity of liquid flow at A is 10. An air bubble in water rises from bottom to top with growing
v, what will be its velocity at B? size. Explain
12. Two liquids of densities d1 and d 2 and coefficient of
11. Still water runs deep. Explain.
viscosities γ1 and γ2 are found to flow through a capillary 12. Two row boats moving parallel to each other and near by,
tube at the same rate. Find the ratio of γ1 and γ2 ? are pulled towards each other. Explain.
13. A hotter liquid flows faster than a colder one. Explain. 13. Why is pressure of water reduced when it flows from wide
14. One flask containing glycerine and other water is shaken pipe to narrow pipe.
vigorously and then kept on a table. Which liquid will come 14. Two capillary tubes of equal length and inner radii 2r and 4r
to rest first? respectively are added in series and a liquid flows through
15. If instead of fresh water, sea water is filled in the tank, will it. If the pressure difference between the ends of the whole
the velocity of efflux will change? system is 8.5 cm of mercury, find the pressure difference
between the ends of the first capillary tube.
Short Answer Questions [2 or 3 Marks Qs.] 15. Two equal drops of water falling through air with a steady
velocity V. If the two drops coalesce, what will be the new
1. When equal volume of two metals are mixed together, the steady velocity?
specific gravity of alloy is 4. When equal masses of the 16. Two capillaries of same length and radius in the ratio 1 : 2
same two metals are mixed together, the specific gravity of are connected in series and a liquid flows through this
the alloy is 3. What is the specific gravity of each metal? system under streamline conditions. If the pressure across
2. A liquid drop of diameter d breaks up into 27 small equal the two extreme ends of the combination is 1 m of water,
drops. If S be the surface tension of the liquid, calculate the
what is the pressure difference across the (i) first capillary
increase in potential energy.
(ii) second capillary?
3. A sphere of relative density ρ and diameter d has concentric
17. 16 cm3 of water flows per second through a capillary tube of
cavity of diameter D. What is the ratio of d/D if it just floats
on water in a tank? radius r cm and of length l cm, when connected to a pressure
4. A block of wood, specific gravity 0.6 and mass 90 g is floating head of h cm of water. If a tube of same length but radius r/
in water. A hole is drilled in it removing 8 g of wood and 2 is connected to the same pressure head, what is the volume
filled with lead of density 11.34 g/cm3. What will be effect of water flowing per second through the tube.
on the equilibrium of the block? 18. Under a constant pressure head, the rate of streamline flow
5. A glass plate of negligible mass and thickness is held of a liquid through a capillary tube is V. If the length of the
against the end of a tube and pushed 10 cm under the surface tube is doubled and the diameter is halved, find the rate of
of water. When released, the plate does not fall off. What flow of liquid through the capillary tube.
266 Physics
19. A capillary tube is attached horizontally to a constant head 12. Calculate the work done in blowing out a soap bubble of
arrangement. If the radius of the capillary tube is increased diameter 1 cm. Given that the surface tension of soap solution
by 10% then by what percent the rate of flow of liquid will is 28 × 10– 3 N/m.
change? 13. Find the work done in breaking a water drop of radius 1mm
20. The cylindrical tube of a spray pump has a radius R, one into 1000 droplets.
end of which has n fine holes, each of radius r. If the speed
[surface tension of water = 72 × 10– 3 N/m]
of the liquid in the tube is V, find the speed of liquid coming
out through the holes. 14. Calculate the work done in breaking a drop of water of 2 mm
21. In a test experiment on a model aeroplane in a wind tunnel, diameter into one thousand million droplets all of the same
the flow speeds on the lower and upper surfaces of the size. Surface tension of water is 72 × 10–3 M/m.
15. Soap bubble is blown to a diameter of 7 cm. If 36960 ergs of
wing are v and v 2 respectively. If the density of air is d
work is done in blowing it further, find the new radius, if
and surface area of the wing is A, what is the dynamic lift on surface tension of the soap solution is 40 dynes/cm.
the wing of aeroplane.
16. What amount of energy will be liberated if 1000 droplets
22. A rectangular tank is filled with water upto its brim. When a
each of diameter 10–6 cm coalesce to from a bigger drop.
hole at its bottom is unplugged, the tank is emptied in time
T. Calculate the time to empty the half filled tank? Surface tension of water is 75 × 10–3 N/m.
17. The air pressure inside a soap bubble of diameter 3.5 mm is
Long Answer Questions [5 Marks Qs.] 8 mm of water above the atmosphere. Calculate the surface
tension of soap solution.
1. State and prove Bernoulli's theorem.
2. State and prove equation of continuity. 18. Find the depth at which an air bubble of radius 0.7 mm will
3. Derive an expression for terminal velocity in case of a sphere remain in equilibrium in water given surface tension of water
falling through a liquid. = 7 × 10–2 N/m, g = 10 m/s2.
19. What would be the radius of the capillary tube so that water
Numerical Questions [3 or 5 Marks Qs.] will rise to a height of 8 cm in it?
1. If the water pressure gauge shows the pressure at ground Surface tension of water = 70 × 10–3 N/m.
floor to be 270 k Pa, how high would water rise in the pipes 20. Calculate the diameter of a capillary tube in which mercury
of a building? is depressed by 1.21 cm. Given surface tension for mercury
2. A column of water 60 cm high supports a 32 cm column of is 540 × 10–3 N/m. The angle of contact of mercury with
an unknown liquid. What is the density of the liquid? glass is 140° and density of mercury is 13.6 × 103 kg/m3.
3. What will be the length of mercury column in a barometer 21. Water rises to a height of 10 cm in a certain capillary tube. If
tube, when the atmospheric pressure is 75 cm of mercury in the same tube, level of Hg is depressed by 3.42 cm,
and tube is inclined at an angle of 60° with horizontal compare the surface tension of water and Hg. Specific gravity
direction? of Hg is 13.6, the angle of contact for water is zero and that
4. A water tank is 20 m deep. What is the pressure at the for Hg is 135°.
bottom of the tank in atmosphere? 22. Find the difference in levels of mercury in the two limbs of a
5. The volume of an air bubble becomes 8 times the original U tube, if the diameter of bore of one limb is 1 mm and of the
volume in rising from the bottom of a lake to its surface. If other 4 mm. The surface tension of mercury is 544 × 103 and
the barometric height is 0.76 m of Hg (density of Hg is 13.6 angle of contact is 130°.
g/cm3 and g = 9.8 m/s2), what is the depth of the lake? 23. Water rises in a capillary tube to a height of 2 cm. In another
6. A cylinder has a radius 10 cm. To what height should it be capillary tube whose radius is one third of it, how much the
filled with water so that the thrust in its walls is equal to that water will rise? If the first capillary tube is inclined at an
on its bottom? angle of 60° with the vertical then what will be the position
7. A piece of iron floats in mercury. Given that the denisty of of water in the tube.
iron is 7.8 × 103 kg/m3 and that of mercury is 13.6 × 103 kg/ 24. A flat plate of side 20 cm moves over another similar plate
m3. Calculate the fraction of the volume of iron piece that with a thin layer of 0.4 cm of a liquid between them. If a force
remains outside the mercury. of one kg wt. moves one of the plates uniformly with a
8. A piece of brass weighs 12.9 g in air. Brass is an alloy of zinc velocity of 1m/s, calculate the coefficient of viscosity.
and copper. When completely immersed in water it weighs 25. Water is flowing through a horizontal tube 4 cm in diameter
11.3 g. What is the mass of copper contained in the alloy? and 4 km in length at the rate of 20 litres per second.
Specific gravity of zinc and copper are 7.1 and 8.9 Assuming only viscous resistance, calculate the pressure
respectively. to maintain the flow. Coefficient of viscosity of water is
9. A cubical block of wood of 10 cm side floats on a side at the 0.001 Pa-s.
interface between oil and water, with its lower surface 26. Water is flowing out of a cylinder through a horizontal tuble
of length 50 cm and diameter 0.4 mm. The tube is 50 cm
horizontal and 4 cm below the interface. What is the mass of
below the water level in the cylinder. How much water will
the block? The density of the oil is 0.6 g/cm3. flow out per minute? Coefficient of viscosity of water = 4 ×
10. A body floats in water with 40% of its volume out side the 10–3 Pa-s
water. When the body floats in oil, 60% of its volume remains 27. Two capillary tubes of lengths 15cm and 5cm and radii 0.06
outside oil. What is the relative density of oil? cm and 0.02 cm respectively are conected in series. If the
11. A square wire frame of side 10 cm is dipped in a liquid of pressure difference across the end faces is equal to pressure
surface tension 28 × 10– 3 N/m. On taking out, a membrane is of 15 cm high water column, then find the pressure difference
formed. What is the force acting on the surface of wire frames. across the (a) first tube and (ii) second tube
Mechanical Proper ties of Fluids 267

28. Eight rain drops of radius 1 mm each falling downwards velocity of flow of water at a point where the diameter is 4
with a terminal velocity of 5 cm/s coalesce to form a bigger cm.
drop. Find the terminal velocity of bigger drop. 32. What should be the average velocity of water in a tube of
29. An air buble of 10 mm radius is rising at a steady rate of diameter 0.01m, so that the flow is laminar ? The viscosity of
2mm/s through the liquid of density 1.47× 103 kgm–3 . water is 0.001 pa–s.
Calculate the coefficient of vivcosity of liquid (g = 9.8 m/s2). 33. Water flow along a horizontal pipe of which the cross-
Neglect the density of air. section is not uniform. The pressure is 30mm of Hg where
30. Water flows at a speed of 6 cm/s through a tube of radius 1 the velocity is 0.02 m/s . Find the pressure at a point where
cm. Coefficient of viscosity of water at room temperature is the velocity is 1.2 m/s .
0.01 poise. Characterize the flow. 34. Water flows at the rate of 4 liter per second through an
31. Water flow through a horizontal pipe of varying cross orifice at the bottom of lake which contains water 720 cm
deep. Find the rate of escape of water if additional perssure
1
section at the rate of litre per second. Determine the of 16 kg/cm2 is applied at the surface of water.
2

HOTS/Exemplar Questions

3. If a drop of liquid breaks into smaller droplets, it results in


Very Short Answer Questions [1 Mark Qs.]
lowering of temperature of the droplets. Let a drop of
1. An egg when placed in ordinary water sinks but floats when radius R, break into N small droplets each of radius r.
Estimate the drop in temperature. [Exemplar]
placed in brine. Why? [HOTS]
2. How is an airplane able to fly upside down? [HOTS] Long Answer Questions [5 Marks Qs.]
3. A barge filled with scrap iron in a canal lock. If the iron is 1. (a) There is a 1 mm thick layer of glycerine between a flat
thrown overboard, does the water level at the side of the plate of area 100 cm2 and a big plate. If the coefficient
lock rise, fall, or remain unchanged? Explain. [HOTS] of viscosity of glycerine is 1.0kg/ (m-sec), then how
much force is required to move the plate with a velocity
4. How does the volume of a completely submerged object
of 7 cm/sec. [HOTS]
compare with the volume of water displaced? [HOTS]
(b) Two equal drops of water are falling through air with a
5. Is surface tension a vector? [Exemplar] steady velocity of 10 cm/s. If the drops recombine to
6. Iceberg floats in water with part of it submerged. What is form a single drop, what would be their terminal
the fraction of the volume of iceberg submerged if the density
velocity? [HOTS]
of ice is ri = 0.917 g cm–3 ? [Exemplar]
2. (a) Pressure decreases as one ascends the atmosphere. If
Short Answer Questions [2 or 3 Marks Qs.] the density of air is r, what is the change in pressure
dP over a differential height dh?
1. How does water pressure one meter below the surface of a (b) Considering the pressure p to be proportional to the
small pond compare to water pressure one meter below the density, find the pressure p at a height h if the pressure
surface of a huge lake? [HOTS] on the surface of the earth is r0.
2. Suppose an office party is taking place on the top floor of a (c) if p0 = 1.03 ×105 Nm–2, r0 = 1.29 Kgm–3 and g = 9.8 ms
–2 , at what height will the pressure drop to (1/10) the
tall building. Carrying an iced soft drink, you step on the
elevator, which begins to accelerate downward. What value at the surface of the earth?
happens to the ice in the drink? Does it rise farther out of (d) This model of the atmosphere works for relatively small
the liquid? Sink deeper into the liquid? Or is it unaffected by distances. Identify the underying assumption that
the motion? [HOTS] limits the model. [Exemplar]
268 Physics

CHAPTER TEST

Time : 45 min. Max. Marks : 24

Directions : (i) Attempt all questions


(ii) Questions 1 to 5 carry 1 mark each.
(iii) Questions 6, 7 and 8 carry 3 marks each.
(iv) Questions 9 and 10 carry 5 marks each.

1. What will be the acceleration of a body falling through a viscous medium with terminal velocity.
2. What is the work done in blowing a soap bubble of radius r and surface tension S?
3. The excess pressure inside a soap bubble is thrice the excess pressure inside a second soap bubble. What is the ratio
between the volume of the first and the second bubble?
4. Why the nib of a pen is split?
5. Why does the clouds float in the sky?

6. Water is flowing with a speed of 2m/s in a horizontal pipe with cross -sectional area decreasing from 2≥10,2 m2 to 0.01m2
at pressure 4 × 104 pascal. What will be the pressure at smaller cross-section?
7. Distinguish between streamline and turbulent flow of a liquid. List two properties of streanilne flow.
8. Derive an expression for excess pressure in a liquid drop.

DR

P
R

9. Explain surface tension on the basis of molecular theory.


10. What is capillarity? Derive a formula for the height of liquid rise in a capillary tube?
Mechanical Proper ties of Fluids 269

Solutions
When equal masses of two metals are mixed, then
PRACTICE QUESTIONS m1 = m2 = m
m m
Very Short Answer Questions [ V1 < and V2 <
θ1 θ2
1. No - because in fall Buoyant force doesn't exist.
2. No - because equilibrium of floating bodies remain m∗m 2θ1θ2
unaffected by variation of acceleration due to gravity. [ θ< < =3
3. The level of water will not change because for a floating m m
∗ P1 ∗ P2
ice, the weight of ice is equal to the weight of the displaced θ1 θ2
3 3
water by the immersed part of ice. When the ice melts, the [ θ1 2 < (θ1 ∗ θ 2 ) = ≥ 8 = 12
θ ...(2)
water so formed will be equal in volume which was displaced 2 2
by the immersed part of the ice. Solving (1) and (2) we get r 1 = 2 and r 2 = 6
4. The body will sink to the bottom. 2. Vol. of the big drop = 27 × vol. of each small drop
5. Because the pressure of water increases with increase in
4 æç d ö÷
3
4 d
depth. Þ ο ç ÷÷ < 27 ≥ οr 3 Þ r<
6. Zero ç
3 è2ø 3 6
7. Change is positive as on splitting the surface area increases Increase in P.E. = S.T × increase in surface area
and hence the work done to increase the surface area is é æ d ö2 ù
stored as potential energy of small drops. So P.E. increases. < S êê 27≥ 4οr 3 , 4ο çç ÷÷÷ úú
8. Surface tension of liquid decreases with increase in êë çè 2 ø ú
û
temperature. So, the curvature of the meniscus decreases é 2ù
æ ö2
æ ö 2d 2
< 4οS êê 27 ≥çç ÷÷÷ , çç ÷÷÷ úú < 4οS≥
and the angle of contact increases. d d
< 2οd 2S
9. So that it can spread over a large area. êë çè 6 ø èç 2 ø ú 4
10. Terminal velocity (VC) µ r2 û
3. Let r ↑ density of the sphere, r ' ↑ density of water..
2 The sphere will float if weight of the sphere = weight of the
VC ' (2r)
[ < [ VC' = 4 VC displaced water
VC r2
[ Terminal velocity will increase 4 times. 4 éêæç d ÷ö æç D ö÷ ùú
3 3
4 ædö
3
Þ ο êç ÷ , ç ÷ ú θg = οçç ÷÷ θ 'g
11. From equation of continuity, a1v1 = a2v2 Þ οR 2 ≥ v1 3 êçè 2 ÷ø èç 2 ø÷ ú 3 çè 2 ÷ø
ë û
= ο(2R)2 v θ
Þ (d3 – D3) < d3
[ v1 = 4v θ1
[ Velocity at B is 4 times the velocity at A. Þ (d 3 – D3)
ρ = d3 Þ d3( ρ – 1) = ρ D3
ο Pr 4 οhdgr 4 d æç ρ ö÷
1/ 3
12. By Poiseuille's formula, V < < Þ <ç ÷
8 γl 8γl D çè ρ,1ø÷
d γ1 d
[ is constant [ < 1 M 90
γ γ2 d 2 4. Volume of the block = < < 150 cm3 ;
D 0.6
13. As the temperature of the liquid increases, its coefficient of
viscosity will decrease. So the flow of liquid increases. 8 40
14. Glycerine will come to rest earlier as its viscosity is larger Volume of the hole = < cm3
0.6 3
than water, so the relative motion will be destroyed early in
glycerine. 40
[ Volume of lead = cm3
15. No, because it is independent of the density of the liquid. 3
Short Answer Questions 40
[ Mass of lead = V × D = × 11.34 = 151.2 g
1. Let m1, m2 – masses of two metals, V1, V2 – volumes of two 3
metals, θ ↑ density of the mixture. Mass of the block with lead = (90 – 8) + 151.2 = 233.2 g;
Volume of water displaced by the block = 150 cm3
θ = m1 ∗ m 2
[
weight of water displaced = 150 × 1 = 150 gm wt.
V1 ∗ V2 Q Weight of water displaced is less than the weight of
When equal volumes of two metals are mixed, then the block
V1 = V2 = V [ The block will sink.
[ m1 = r 1V and m2 = r 2V 5. Let A ↑ Area of the base of the plate,
θ1v ∗ θ 2 v θ1 ∗ θ2 h ↑ height of kerosene column for which the plate just
[ θ< < falls off.
V∗V 2 [ Downward thrust of h cm of kerosene = upthrust of 10
[ θ1 ∗ θ2 = 2 θ = 2 × 4 = 8 ... (1) cm of water
270 Physics
Þ h × 0.8 × g × A = 10 × 1 × g × A (c) independent of the inclination of the solid to liquid
surface.
10
Þ h< = 12.5 cm. The angle of contact is zero degree for pure water glass
0.8 tube. The liquid which wets the surface of the container
6. Let m1 ↑ mass of ice , m2 ↑ mass of stone, r ↑ density and having a concave menisus has acute angle of contact.
of water Liquids which do not wet the walls of the container and
having a convex meniscus will have obtuse angle of contact.
m1 ∗ m 2
[ Volume of water displaced by the ice < V < 10. A fluid always moves from a region of higher to lower
θ pressure and liquid pressure increases with depth. The
m pressure at the top of water surface is less than at the
When the ice melts, volume of water produced = 1 bottom. So the bubble rises up. As it is coming up pressure
r
If r s ↑ density of stone then the volume of water displaced decreases so by Boyle's law volume increases. Therefore it
grows in size.
m2 11. According to equation of continuity, av = constant.
by the stone =
θs a ↑ cross – sectional area of the liquid, v ↑ velocity of
liquid flow.
m1 m 2 For still water, v is very small , a becomes extremely large.
[ Total vol. of water displaced = V ' < ∗
θ θs So still water runs deep.
12. From Bernoulli's theorem, when velocity of liquid increases
1 1 , pressure decreases.
Q rs > r \ < \ V' < V
rs r When two boats move parallel very close to each other, the
velocity of stream of water between the boats increases. So
[ The level of water will decrease.
pressure exerted by the water on the boats beocmes less
7. Consider a rectangular metallic frame PQRS having a sliding than the pressure of water out side the boats. So the boats
wire AB at its one side. are pulled towards each other.
Dip it in soap solution and a thin soap film AQRB is formed. 13. By equation of continuity av = constant. So when water
This film has 2 free surfaces of which water and air are in goes to narrow pipe its velocity increases. According to
contact with each other . The force of surface tension acts Bernoulli's theorem, when velocity increases, pressure
tangentially inwards and perpendicular to the free surfaces decreases. So pressure in narrow pipe is less.
of the film. Let the area of the film has increased by moving 14. Let P1 ® Pressure at the joint of two tubes, P2 ® Pressure
AB onwards to A' B'. at the end of second tube
[ Increase in area of this film = AA'B'B =a = 2 ( l × x ) P ® Pressure at the beginning of the first tube.
Total force due to surface tension = F = S × 2 l Q the tube are in series., [ by Poiseuille's formula,
( [ there are two free surfaces )
[ work done against the force of surface tension = W p(P - P1 ) (2r)4 p(P 1 - P2 ) (4r)4
= Þ P – P1 = 16 (P1 –
= Force × displacement 8h l 8hl
= S × 2l × x = S × a [ [ a = 2 ( l × x ) ] P2) Þ P = 17P1 – 16P2 ...........(i)
This work done is stored as surface energy. Also given that, P – P2 = 8.5 ..........(ii)
[ Surface energy = E = S × a 8.5
8. Let us consider a soap bubble of radius R. Solving (i) & (ii), we get, P1 – P2 = = 0.5 cm of Hg.
17
S ↑ surface tension of soap solution, P ↑ Excess pressure
[ Pressure difference between the ends of the first tube
inside the bubble. = P – P1
dR ↑ increase in radius due to excess pressure soap = (P – P2) – (P1 – P2) = 8.5 – 0.5 = 8 cm of Hg
bubble has two free surfaces where air is in contact with the
soap solution. Work done by the excess pressure = 2 r 2 (θ ,ρ)g
15. Terminal velocity VT =
P ≥ 4οR 2 χR 9 γ
Increase in area of the bubble < 2[4ο(R ∗ χR)2 , 4οR 2 ] [ VT µ r2
< 2≥ 4ο[R 2 ∗ 2RχR ∗ (χR) 2 , R 2 ] < 16οR(χR) 4 4
If R is the radius of the bigger drop, then p R3 = 2 × p r3
[ Neglecting ( dR ) 2as it is very small ] 3 3
Þ R = (2)1/3r
Increase in P.E. = Increase in surface area × surface tension
=16οR(χR)≥S VT r2 V r2
[ = Þ =
VT' R2 VT' (2)2 / 3 r 2
[ Increase in P.E. = Work done
[ V'T = (2)2/3 V
4S 16. According to the problem,
16οR(χR)≥S < p≥ 4οR 2χR Þ P <
R π P2 (2r)4
V < οP1r / 8ηl <
4
9. Angle of contact depends on the following factors : Þ P1 = 16P2
(a) nature of the liquid and solid in contact . 8ηl
(b) nature of the medium above the free surface of the Given that, P1 + P2 = 1 Þ 16P2 + P2 = 1 Þ 17P2 = 1
liquid.
Mechanical Proper ties of Fluids 271

1 16 When the tank is half filled Vwater becomes half & h


Þ P2 = m and P1 = m. h
17 17 becomes
17. From Poiseuille's formula, V µ r4 2

æ r ö4 Vwater / 2 T
[
16 ≥çç ÷÷÷ Time = =
V1 r14 (r2 )4 çè 2 ø g.h / 2 2
[ = Þ V2 = V1 = = 1 cm3 A.
4 2
V2 r24 (r1 ) 4 r
Long Answer Questions
π P r4 r4 Bernoulls’s theorem:
18. From, V = [ Vµ 1. Statement : For an ideal liquid in streamline flow, the total
8ηl l
energy per unit mass remains same at every cross-section
V1 r4 2l 32 of the flow. Total energy is the sum of pressure energy,
Þ = × = potential energy and kinetic energy.
V2 (r / 2) 4 l 1
P 1 2
1 1 i.e., + gh + v = constant
[ V2 = V1 = V r 2
32 32
Rate of flow will reduce 32 times. Proof : Let P1, V1, a1 are pressure, velocity of the liquid and
area of cross-section of the pipe at A respectivelyand, P2,
π P r4 V2, a2 are the corresponding quantities at B
19. As we know, V = h1 and h2 are heights of the ends A & B from a reference
8ηl
level.
10 B
New radius = r1 = r + r = 1.1 r
100 v2
4 4 P2
π P (1.1r) 1.464 π P r A
New volume of liquid, V' = =
8ηl 8ηl
v1
= 1.464 V h2
V ', V h1 P1
% increase in rate of flow = × 100
V
Reference Level
æ1.464 V , V ö÷
= çç ÷ 100 = 46.4%
çè V ø÷ Q Liquid is flowing from A to B,
20. Area of cross section of the tube = p R2; Area of cross [ P >P
1 2
section of each hole = p r2 By equation of continuity, a1 V1 = a2V2 = V (say)
Let v ® speed of liquid coming out through the holes [ Work done/second on the liquid at A = Force ×
[ By equation of continuity, p R2V = (p r2v) n displacement = P1a1V1
Work done/s on the liquid at B = P2a2V2
V R2
Þ v= [ Net work done/s on the liquid = P1 a1 V1 – P2 a2 V2 = P1
n r2 V – P2 V
1 1 1 1
21. By Bernoulli's theorem, P1 + d v 2 = P2 + dv22 Change in K.E of the liquid = mV22 – m V12 [V2 > V1
2 1 2 2 2
Q a2 < a1]
1 1 1
P1 – P2 = d (v22 – v12) = d (2v2 – v2) = d v2 Change in P.E of the liquid = mgh2 – mgh1
2 2 2 According to work-energy theorem, Work done = Change
1 2 in energy
[ Force of dynamic lift = (P1 – P2) × A = d v ×A
2 1 1
22. Let h ® height of the tank P1V – P2V = mV22 – mV12 + mgh2 – mgh1
2 2
Velocity of efflux = V = 2gh 1 1
[ P1V + mgh1 + mV12 = P2V + mgh2 + m V22
When the hole is at the top V = 0 2 2
V∗0 gh 1
[ Average velocity of efflux = = Þ PV + mgh + mV2 = constant
2 2 2
Volume of water V PV 1
Time = = water Þ + gh + V2 = constant
Area of the water ≥ Velocity of efflux gh m 2
A.
2
272 Physics

P 1 2 r 2 (θ ,ρ)g
Þ + gh + V2 = constant VT =
m/V 2 9 γ
P 1
Þ + gh + V2 = constant Numerical Questions
θ 2
1. Given P = 270 kPa = 270 × 103 Pa
P
Where is called pressure energy/unit mass, gh is called P 270×103
θ Using pressure, P = hrg Þ h = = = 27.6 m
potential energy/unit mass
rg 103 ×9.8

1 2 h1r1 0.6×103
and V is K.E/unit mass. 2. h1r1g = h2r2g Þ r2 = = = 1.875 × 103 kg/m3
2 h2 0.32
2. For an incompressible, non-viscous liquid in streamline flow
the product of cross section of the tube and the velocity of 3. h = l sin 60° \ l = h/sin 60° = 75 /( 3 / 2) = 86.6 cm
the liquid through that cross section is constant throughout 4. Pressure at the bottom of the tank = atmospheric pressure +
the flow. pressure due to the liquid column
i.e. aV = constant 20rg
æ ö
P = P0 + hrg = 1 + hrg atm = 1 + 5 = 3atm
a2 çè ÷
P0 ø 10
v2 5. Since the volume of the bubble becomes 8 times on reaching
the surface, therefore the pressure on the surface of the
v1 B lake is 1/8 times of the pressure at the bottom.
a1 \ By Boyle’s law, pressure at the bottom of the lake = 8 ×
P where P is the pressure at the surface.
\ Pressure due to column of liquid = 8P – P = 7P = 7 ×
A 0.76 × 13.6 × 103 × 9.8 N/m2
7×0.76×13.6×103 ×9.8
Let a1, a2 ® area of cross section of the tube at A and B; v1, \ h=
rg
v2 ® Velocity of the liquid at A and B.
r1, r2 ® density of liquid at A & B.
7×0.76×13.6×103 ×9.8
[ Mass of the liquid entering/s at A = a1v1r1 = = 72.35 m
mass of the liquid leaving/s at B = a2v2r2 103 ×9.8
Q there is no liquid remaining in the tube and the flow is 6. Let r = denisty of water, h = height of water in cylinder
steady, Þ a1v1r1 = a2v2r2 \ Thrust at the bottom = F = pressure × area = (hrg)pr2
Q the liquid is incompressible, Thrust on the walls = Average pressure × area
[ r1 = r2
rhg ö
[ a1 v1 = a2 v2 Þ av = constant. Þ F' = çæ (2prh) = prh 2rg
3. When a sphere falls through a viscous liquid, viscous forces è 2 ÷ø
acting on it are :
When F = F' then hrg × pr2 = prh2rg or h = r = 10 cm
(a) Weight of the sphere acting downwards
7. Let V = Total volume of the piece of iron, v = the volume of
4 3 the portion of iron outside the mercury.
W1 = pr rg [r = radius of the sphere,
3 The piece of iron will float if wt. of iron piece = wt. of mercury
r = density of the material of the sphere] displaced by the immersed portion of iron
(b) Weight of the displaced liquid acting upwards Þ V ≥ 7.8≥103 ≥ g < (V , v) ≥13.6≥103 ≥g
4 3 Þ v/V = 0.43
W2 = pr sg [s = density of the liquid]
3 8. Let m = mass of copper in the alloy
(c) Viscous force = F = 6 phrV [h = coefficient of viscosity [ mass of zinc in the alloy = (12.9 – m)g
of the liquid, V = velocity of the liquid] [ Volume of copper in the alloy = m/8.9
Initially, W1 is greater than the forces acting upwards.
12.9 , m
As there is acceleration in the downward direction Volume of zinc in the alloy =
velocity of the sphere in that direction increases. So, 7.1
the viscous force also increases. When the downward m 12.9 , m
force becomes equal to the total upward force, net [ Total volume of the alloy = ∗
8.9 7.1
force on the sphere becomes zero. So, it falls with a [ Apparent loss of weight of alloy = 12.9 – 11.3 = 1.6 g wt.
constant velocity called terminal velocity (VT).
1.6
4 3 4 [ Vol. of water displaced by alloy = = 1.6cm 3
Under that condition, p r r g = p r3s g + 6 p h r VT 1
3 3
m 12.9 - m
4 3 \ + = 1.6 Þ m = 7.61 g.
p r (r - s) g = 6 p h r VT [ 8.9 7.1
3
Mechanical Proper ties of Fluids 273

9. Volume of the block = 10 × 10 × 10 = 1000 cm3. 16. Vol. of the big drop = 1000 × vol. of each small drop
Volume of the block in water = 10 × 10 × 4 = 400 cm3 4 4
[ Volume of the block in oil = 1000 – 400 = 600 cm3 οR 3 < 1000 ≥ οr 3 Þ R3 = (10r)3 Þ R = 10r Þ r =
3 3
Q the block floats. [ weight of the block = wt. of
0.5 × 10– 8 m, R = 10 × 0.5 × 10–8m
the water displaced + wt. of oil displaced
Energy released = S[1000 × 4pr2 – 4pR2] = 75 × 10–3 × 4 ×
Þ mg = 400 × 1 × g + 600 × 0.6 × 6 = 400 + 360 = 760 g
10. Weight of the body is equal to the upthurst. 22
[1000 ×(0.5 × 10–8)2 – (0.5 × 10–7)2]
Let, V = volume of the body, r = density of the liquid, 7
r0 = density of water. = 2.12 × 10– 14 J.
(100 , 40)Vθ0 g (100 , 60) 4S
[ Weight of body = < V θg 17. Pressure p < Here, p = 8 mm of water = 8 × 10– 3 × 103 ×
100 100 R
3.5≥10,3
θ 0.6 0.8 N/m2 r = m
or < < 1.5 = Relative density of oil 2
θ0 0.4
p ≥ r 8≥9.8≥3.5≥10,3
11. The surface film has two free surfaces. Perimeter of the squre [ S< = = 0.0343 N/m
= 4l [ l = side of the square] 4 2≥ 4
[ Total lenght of the film = 8l 18. The bubble will remain in equilibrium at a depth h if
2s
< hθg
[ Force due to surface tension = 8l × S = 8 × 0.1 × 28 × r
10–3 = 0.0224 N 2s 2 ≥ 7 ≥10,2
12. Work done = surface tension × increase in surface area Þ h< = = 2 × 10–2m = 2 cm
rθg 0.7 ≥10,3 ≥103 ≥10
æ D ö2 2s cos π
or W = S × 4p ≥çç ÷÷÷ ≥ 2 19. As we know, h = 2s cos π Þ r =
çè 2 ø rθg hθg

2 ≥ 70 ≥10,3 ≥ cos 0↓
22 æç1≥10,2 ö÷÷
2
= 28≥10 ,3
≥ 4≥ ≥çç = = 1.785 × 10 m–5
÷ ≥ 2 = 1.76 × 10–5 J. 8≥10,2 ≥103 ≥ 9.8
7 èç 2 ø÷÷
2s cos π
13. If R and r are the radii of the big drop and the small drop 20. Radius of the capillary tube, r <
hθg
4 3 4
respectively, pR = 1000 × pr3
3 3 4s cos π 4≥540 ≥10,3 ≥ cos140↓
R3 = 1000 r3 = (10r)3 [ D = 2r = =
hθg (,1.21≥10,2 ) ≥ (13.6 ≥103 )9.8
R = 10 r [ –3
= 1.026 × 10 m
R 1≥10,3 21. For water s1 = surface tension of water = ?
r< < < 10,4 m
10 10 2s1 cos π1
Work = surface tension × increase in surface area = S[1000 h1 = 10 cm. q1 = 0°, h1 =
× 4p r2 – 4pR2] rθ1g
22 For Hg, s2 = surface tension = 7
= 72 × 10–3[1000 × (10–4)2 – (10–3)2] × 4 × = 72 × 10–3 4 × 2s 2 cos π 2
7 h2 = – 3.42 cm., q 2 = 135°, h2 =
rθ 2 g
22
[10–5 – 10–6]
7 h1 s cos π1θ 2 s h θ cos π 2
[ < 1 Þ 1< 11
22 é 1ù 22 9 h 2 s 2 cos π 2 θ1 s2 h 2 θ 2 cos π1
= 72 × 10–8 × 4 × ê1, ú = 72 × 10–8 × 4 × ×
7 êë 10 úû 7 10 s1 10≥1≥ cos135↓ 10(,0.7071)
= 8.14 × 10–6 J Þ < = = 0.152
s2 ,3.42≥13.6 ≥ cos 0↓ ,3.42 ≥13.6
14. Vol. of the big drop = vol of one thousand million droplets
22. Here, r1 = 1mm = 1 × 10–3m. r2 = 4 mm = 4 × 10–3 m.
4 çæ 2≥10,3 ÷÷ö
3
4 3 S = 544 × 10 –3 N/m, q = 130°, r = 13.6 × 103 kg/m3
Þ ç
οç ÷÷ < οr ≥10 Þ r = 10 m
9 –6
Let h1 and h2 be the heights to which mercury rises in narrow
ç
3 è 2 ÷ø 3 and wide tube respectively then
22 2s cos π 2s cos π
Work done = S [4pr2 × 109 – 4pR2] = 72 × 10–3 × 4 × [(10– h1 < and h 2 <
7 r1θg r2 θg
6)2 × 109 – (1 × 10–3)2] = 9.042 × 10–4J
[ Difference in levels of mercury in the two limbs of the
15. Let R and r be the radii of the new bubble and the original
radius of the bubble respectively. 2s cos π éê 1 1 ùú
[ Work done = surface tension × increase in area U-tube = h2 – h1 = ,
θg êë r2 r1 úû
Þ 36960 = 40 × [4pR2 – 4p (7/2)2] × 2
On solving we get, R = 7 cm.
274 Physics

2≥544≥10,3 ≥ cos130↓ é 1 1 ù 2 r 2 (θ , r)g


= ê , ú [ Terminal velocity VT <
13.6≥10 ≥ 9.8
3 ê
ë 4≥10
,3
1≥10,3 úû 9 γ
= 0.00787 m] V2 R 2
23. Height h = 2s cosq/rrg Þ hr = 2s cosq /rg = const. [ VT × r 2 [ <
V1 r2
h1r1 hr æRö æ 2r ö2
[ h 1 r1 = h 2 r2 Þ h 2 < = < 3h = 3 × 2 = 6 cm.
Þ V2 < V1 çç ÷÷÷ Þ V2 < 5≥çç ÷÷÷ = 20 m/s
r2 r /3 çè r ø çè r ø
If h3 be the height of liquid in the capillary in the inclined
position, the vertical height will remain the same. 2 r 2 θg
29. Terminal Velocity VT < [neglecting ρ ]
[ h3 = h/cos 60° = 2/(1/2) = 4 cm. 9 γ
24. Area of the plate = 20× 20 = 400 sq cm = 400 × 10–4 sq m,
F = 1 kg wt = 1 × 9.8 N, dv = 1m/s, dx = 0.4cm 2 r 2 θg 2≥ (10 ≥10,3 ) 2 ≥1.47≥103 ≥9.8
[ γ< = = 160
F 9.8≥ (0.4 ≥10,2 ) 9 VT 9≥ (2≥10,3 )
[ γ< = = 0.98 Pa-s
A(dv / dx) 400 ≥10,4 ≥1 pa– s
25. Given: r = 2cm = 2 × 10–2m, l = 4 km = 4000 m, θVc D 1≥ 6≥ (2 ≥1)
30. Reynold’sno. = NR = < = 1200 < 2000
V = 20 × 10–3 m3/s γ 0.01
οpr 4 8Vγl [ Flow is streamline
V< Þ P<
8γl οr 4 l
31. From question, V < = 500cm3/s, 2r = 4 cm Þ 2r = 4, r =
s
8≥ (20 ≥10,3 ) ≥ 0.001≥ 4000
Þ P= = 1.27 × 106 N/m2 V 500
22 ,2 4 2 cm; Vc < = < 39.77 cm/s
≥ (2≥10 ) οr 1
22 2
7 ≥2
7
οpr 4 32. Given: D = 0.01m , γ < 0.001 Pa - s, VC = ?
26. Vol. of water flowing per unit time = v <
8γl For laminar (streamline flow), Nr = 2000; V c =
[ vol. of liquid flowing per 1 minute
N R γ 2000≥ 0.01
οθr 4 < = 0.2 m/s
v ≥60 < ≥60 θD 103 ≥ 0.01
8γ l 33. Here, P1 = 30mm of Hg = 3 cm of Hg = 3 × 13.6 ×980
3.14≥ (0.50 ≥10 ≥9.8) ≥ (0.2 ≥10
3 ,3 4
) ≥ 60 θ1 < 13.6g / cc , V1 = 0.20 m/s, P2 = ?;
= From Bernoulli’s theorem
8≥ 4 ≥10,3 ≥ 0.50
= 9.24 × 10 m3
–8 1 1
θ1 ∗ θV12 < θ2 ∗ θV22 Þ on solving we get
27. Rate of flow of liquid in each tube connected in series will 2 2
be same. Let h be the height of water column at the junction 32984
of two tubes connected in series, r 2 = 3298 dyne/cm2 = = 2.475 cm of Hg
13.6 ≥980
ο(15 , h)θg ≥ r14 ο(hθg)≥ r24
[ V< < 34. Given: h = 720 cm, V = 2gh = 2 ≥980 ≥ 720 m/s; V = 4
8 γ l1 8γ l 2
liters/s
(15 , h)r14 hr24 Additional pressure = 16 kg/cm2 = 16000 g/cm2
Þ <
l1 l2 16000 ≥ 980
Now, 16000 g/cm2 = cm of water = 16000 cm of
(0.06) 4
h ≥ (0.02)4 980
Þ (15 , h) < water
15 5
[ Pressure head = h1 = 16000 + 720 = 16720 cm
Þ h = 14.464 cm of water column
(b) Pressure difference across the second tube = 15 – h [ New velocity = v1 = 2gh1 = 2≥980≥16720 cm/s
= 15 – 14.464 = 0.536 cm of water column. [ V = av and V1 = av1
[ Pressure difference across second tube = 14.464 cm of v1
V1 < V 16720
water column [ = < 19.28 l/s
28. Let R and r be the radii of the big drop and each small drop v 720
respectively
HOTS/EXEMPLAR QUESTIONS
4
Vol of the large drop = vol of 8 small drops Þ οR
3
[ Very Short Answer Questions
3
4 1. The density of brine is greater than that of ordinary water.
= 8≥ οR Þ R 3 < 8r 3 Þ R = 2r
3
3 Due to its high density brine exerts an upthrust which
can balance the weight of the egg.
Mechanical Proper ties of Fluids 275

2. An airplane is able to fly upside down by using Bernoulli’s


principle. When it turns upside down, the air pressure 3T æ 1 r 2 ö
\ Dq = ç - N÷
below the wings of the plane is more than that above it rs çè R R 3 ÷ø
which keeps the plane floating.
3. When the iron is thrown overboard, then the mass of the 3T æ 1 r 2 R 3 ö 3T æ 1 1 ö
ç - ÷= -
barge will decrease, so its weight (acting downwards) will =
rs çè R R 3r 3 ÷ø rs çè R r ÷ø
decreases. But the buoyant force remains the same. Now
upward force becomes more than the downward force so Long Answer Questions
the water level will rise.
4. When an object is completely submerged in water it dv
1. (a) F = hA
displaces some water. The volume of the displaced water dx
is equal to the volume of the object. Here h = 1.0 kg /(m-sec),
5. No
A = 100 cm2 = 10–2 metre2,
6. Let the volume of the iceberg be V. The weight of the iceberg is
riVg. If x is the fraction submerged, then the volume of water dv = 7 × 10–2 m/sec and dx = 1 mm = 10–3 m
displaced is xV. The buoyant force is rwxVg where rw is the
density of water. 1.0 ´ 10 -2 ´ (7 ´ 10 -2 )
\ F= = 0.7 Newton
ri Vg = rw xVg 10 -3
(b) Terminal velocity v a r2
ri Let v1 and v2 be the terminal velocities of drops
\ x= = 0.917
rw before and after combination respectively.
Short Answer Questions v1 r2
Hence = ......(i)
1. Liquid pressure is given by the formula, v2 R 2
Liquid pressure = weight density of the liquid × height (where R = radius of combined drop)
Liquid pressure is independent of the area of the liquid at æ4 ö 4
a particular depth. So water pressure 1 m below the surface Further, 2 ´ ç pr 3 ÷ = pR 3
of a small pond will be the same as that due to water è3 ø 3
pressure 1 m below the surface of a huge lake. 2
2. The acceleration of the elevator is equivalent to a change æ rö 1
i.e., ç ÷ = ......(ii)
in the gravitational field, according to the principle of R
è ø (2) 2 / 3
equivalence. If the elevator accelerates downward, one From equation (i) and (ii), we get
might be tempted to say that the effect is the same as if
gravity decreases-the weight of the ice cube decreases, 2
æRö
causing it to float higher in the liquid. Recall, however v 2 = v1 ç ÷ = 10 ´ (2) 2 / 3 = 15.9cm / sec .
è r ø
that the magnitude of the buoyant force is equal to the
weight of the liquid displaced by the ice cube. The weight 2. (a) Consider a horizontal parcel of air with cross section A and
of the liquid also decreases with the effectively decreased height dh. Let the pressure on the top surface and bottom surface
be p and p+dp. If the parcel is in equilibrium, then the net
gravity. Because both the weight of the ice cube and the
upward force must be balanced by the weight.
buoyant force decrease by the same factor, the level of
the ice cube in the liquid is unaffected.
3. R3 = Nr3 p
R
Þ r=
N1/ 3
p + dp
(
DU = 4pT R 2 - Nr 2 ) i.e., (p + dp)A – pA = – PgAdh
Suppose all this energy is released at the cost of lowering the Þ dp = – r gdh.
temperature. If s is the specific heat then the change in temperature (b) Let the density of air on the earth’s surface be ro, then
would be, p r r
= Þ r= p

Dq =
2
DU 4pT R - Nr
=
2
( ) p o ro
ro g
po
, where r is the density. dp = - pdh
ms 4 3 \
pR rs po
3
p h
dp r g dp r g
Þ
p
= - o dh
po
Þ ò p
= - o ò dh
po
po o
276 Physics

æ r g ö (b) The tengent drawn at a particular point on a streamline


p r g
Þ ln =- o h Þ p = po exp ç - o h ÷ gives the direction of velocity of the liquid particle at
po po è po ø that point.
1 r g 8. Let S = surface tension of liquid, p = excess pressure inside
(c) ln = - o ho the drop
10 po SR = Increase in radius due to excess pressure
po 1 po [ Work done by excess pressure = force × displacement =
\ ho = - ln = ´ 2.303 excess pressure × area × displacement
ro g 10 ro g
= p≥ 4οR ≥SR
2
1.013 ´ 105
= ´ 2.303 = 0.16 × 105 m = 16 × 103 m
1.29 ´ 9.8
(d) The assumption p µr is valid only for the isothermal case
which is only valid for small distances.
P
CHAPTER TEST O SR
1. Since velocity is costant so the acceleration of the body R
falling throgh the viscous medium is zero
2. Work done = surface tension × change in surface area,
W < S≥ 4οr 2 Increase in surface area of the drop, 4ο(R ∗ SR )2 , 4οR 2
4S = 4ο[R ∗ 2RSR ∗ SR ] < 8οRSR
2 2
3. Here, P2 < and P1 < 3P2
R2
4S 3≥ 4S [Neglecting (SR)2 which is very small]
Þ < Þ R1 < R 2 [ Increase in energy = Increase in area × surface tension
R1 R2 3
= 8οRSR ≥S
4
οR13 R 32 Q Work done = Increase in energy
V1
[ <3 = 27 <
1 2S
V2 4
οR 23 R23 27 [ p≥ 4οR 2SR < 8οRSR ≥S Þ p <
R
3
9. Consider a liquid molecule A, well within the liquid surface.
[ V1 : V2 = 1 : 27 It is equally attracted by all other molecules. So, the net
4. For Capillary action
force on A is zero. Consider another molecule B which is
within the surface film of liquid KLMN. This molecule will
5. Clouds float in the sky because they have zero terminal
be attracted more on the downwards direction.
velocity.
6. According to Bernoulli’s theorem,
1 1 C
P1 ∗ θV12 < P2 ∗ θV22 K L
2 2
B M
N
Here, V1 = 2 m/s, a1 = 2 ×10–2m2
P1 = 4 × 104 Pa, a2 = 0.01 m2, P2 = ?
a1v1 2≥10,2 ≥ 2 A
a1v1 = a2v2 Þ v 2 < = = 4 m/s
a2 0.01

1 Now, let us consider a third molecule C on the surface of the


1
P2 < P1 ∗ θ(v12 , v22 ) = 4≥104 ∗ ≥103 (22 , 42 ) liquid which will be attracted in the downward direction
2 2 with maximum force. So, to take a molecule from within the
= 3.4 ≥10 4 pa liquid to the surface, work has to be done against the
7. Streamline flow of a liquid is an orderly flow of a liquid attractive force of other molecules (cohesive force). This
where the velocity of all liquid particles at a particular point work done is stored as the surface potential energy.
in the path is constant. More number of molecules means more surface potential
Turbulent flow of a liquid is a disorderly flow where the energy. But a system is said to be stable if it has minimum
velocity of different liquid particles crossing a particular potential energy. Therefore, there will be a minimum number
point is different. of molecules on the surface. Hence, there will be a minimum
Properties of streamline flow: surface area at the free surface, which is the cause of surface
(a) No two streamlines will cross each other. tension.
Mechanical Proper ties of Fluids 277

10. The phenomenon of rise or fall of liquid in a capillary tube is 2S 2S


called capillarity. [ hrg= Þ h=
R Rθg
C(P) A (P) R ® Radius of the meniscus.
h
I
R
D(P) B æç P - 2s ö÷ q
è Rø r
q

Pressure at points C & D one above and another below the


flat surface of the liquid is same as atmospheric pressure
(P). Inside the capillary tube, pressure at A above the liquid r
From Fig. = cosq
2S R
surface is P but just below the liquid meniscus is P – .
R r
So, there is a pressure difference between B and D which [ R=
cos π
are at the same level. Since pressure at B is less, so liquid
rises through the capillary upto a height h. r ® radius of the capillary tube, q ® angle of contact.
2S cosθ
2S [ Capillary rise, h =
Then P = P – +hrg rρg
R

¿¿¿
278 Physics

11 Thermal Properties
C ha p t e r of Matter

How we feel hot and cold? When we touch a piece of ice, heat flows from our body to ice and we feel cold. Similarly when we
touch a cup of hot tea, heat flows from hot cup of tea towards our body and we feel hot. Hot and cold are relative terms.
TEMPERATURE AND HEAT
Temperature is the degree of hotness or coldness of a body and it is the condition of the body which determines the direction
of flow of heat. The S.I. unit of temperature is kelvin (K).
Heat is a form of energy which flows from a body to another body or its surroundings because of the temperature difference
between the two bodies.
Heat is a measure of total thermal energy of a body which is the sum of kinetic energies of all molecules of the body due to
their translational, rotational and vibrational motions.
The S.I unit of heat is joule. Another unit of heat is Calorie. 1 Calorie is the amount of heat required to increase the temperature
of 1 gm of water by 1°C.
1 Calorie = 4.18 J
MEASUREMENT OF TEMPERATURE
To meaures the temperature of a body we use a device called thermometer.
Principle of a thermometer: A thermometer makes use of some measurable property of a substance like volume, changes linearly
with temperature.
A common thermometer - the liquid-in-glass type, mercury and alcohol are the most common used liquid.
Different Scales of Temperature
For the definition of any standard scale, two fixed reference points are needed. The ice point (lower fixed point) and steam point of
water (upper fixed point) are two convenient fixed points.
(i) Celsius or centigrade scale :
Lower fixed point = 0°C; Upper fixed point = 100°C; No. of divisions = 100
(ii) Fahrenheit scale :
Lower fixed point = 32°F; Upper fixed point = 212°F; No. of divisions = 180
(iii) Absolute scale of temperature or Kelvin scale :
Lower fixed point = 0 K = –273.15°C; Upper fixed point = 100 K = – 173.15°C
No. of divisions = 100
Relations between Celsius (C), Fahrenheit (F) and Kelvin (K) scale :
C F - 32 K - 273.15
= =
100 180 100
Conversion of temperature from Celsius to Kelvin scale :
t K = t°C + 273.15
Conversion of temperature from Kelvin to Celsius scale :
t ° C = t K – 273.15
IDEAL-GAS EQUATION AND ABSOLUTE TEMPERATURE
The behaviour of a given quantity of gas is described by variables like pressure (P), volume (V) and temperature (T)
The equation, PV = nRT is known as ideal-gas equation
here, n = number of moles in the sample of gas
R = universal gas constant; (its value is 8.31 J mol–1 K–1),
It is the combination of following three laws
(i) Boyle's law : When temperature is held constant, the pressure is inversely proportional to volume.
1
i.e., P µ ( at constant temperature)
V
Ther mal Proper ties of Matter 279

(ii) Charle's law : When the pressure is held constant, the volume of the gas is directly porportional to the absolute temperature.
i.e., V µ T (at constant pressure)
(iii) Avogadro's law : When the pressure and temperature are kept constant, the volume is directly proportional to the number of
moles of the ideal gas in the container.
i.e., V µ n (at constant pressure and temperature)
Absolute Temperature
The lowest temperature of –273.16 °C at which a gas is supposed to have zero volume and zero pressure and at which entire
molecular motion stops is called absolute zero temperature. A new scale of temperature starting with –273.16°C by Lord Kelvin as
zero. This is called Kelvin scale or absolute scale of temperature.
T(K) = t°C + 273.16
Illustration 1 :
The normal temperature of a human body is 98.4°F. Convert it into Celsius scale.
Sol. Given, F = 98.4°F
C F - 32
Using, =
100 180
C 98.4 - 32 Þ 66.4
[ = C= ´ 100 = 36.9°C
100 180 180
Illustration 2 :
The pressure and temperature of a gas are 2 atm and 27ºC respectively. Find
(a) number of moles per cm3
(b) number of molecules per m3
Sol. (a) PV = nRT.
2 ´ 105 ´ V = n ´ 8.3 ´ 300
n 2 ´ 103
Þ = moles / m3.
V 8.3 ´ 3

2 ´ 103
\ Number of moles per cm3 = ´ 10–6 = 8.03 ´ 10-5 moles/cm3
8.3 ´ 3

2 ´ 103
(b) Number of moles per m3 = moles / m3
8.3 ´ 3
\ Number of molecules per m3

2 ´ 103
= ´ 6 ´ 1023 = 4.8 ´ 1025 molecules / m3.
8.3 ´ 3

THERMAL EXPANSION
The increase in the dimensions of a body due to the increase in its temperature is called thermal expansions.
Thermal expansion of solids is of three types :
(i) Linear expansion
(ii) Superficial expansion
(iii) Cubical expansion
(i) Linear expansion is the increase in length of a solid on heating.
change in length DL
Coefficient of linear expansion (a) = Þa=
initial length ´ increase in temperature LDT
Final length of the solid L2 = L1 [1 + a (T2 – T1)]
(ii) Superficial expansion is the increase in surface area of a solid on heating.
change in area DS
Coefficient of superficial expansion (b) = Þ b=
original area ´ increase in temperature S DT
Final surface area of the solid S2 = S1 [1 + b (T2 – T1)]
280 Physics
(iii) Cubical expansion is the increase in volume of a solid on heating.
change in volume DV
Coefficient of cubical expansion ( g ) = Þ g=
initial volume ´ increase in temperature V DT
Final volume of the solid V2 = V1 [1 + g (T2 – T1)]
The S.I unit of a, b and g is /K (per kelvin)
b g
Relation between a, b and g a = =
2 3
a, b and g are characteristics of the substance.
Water exhibits an anomalous behaviour. It contracts on heating between 0°C and 4°C. Water has a maximum density at 4°C. This
property has an important environmental effect. The bottom of lakes and ponds does not freeze. This allows marine animals to
remain alive and move freely near the bottom.

1.04343 × 10 3

V
(in m3)

–3
1.00 × 10
4°C T
Thermal Stress
If the thermal expansion of a rod is prevented by fixing its ends rigidly, thermal stress sets up in the rod. The corresponding
compressive strain is given by
Dl
= aDT
l

DF æ Dl ö
Thermal stress, =Yç ÷
A è l ø
Some applications of thermal expansion in solids
(i) Bimetallic strips (ii) Effect of temperature on pendulum clock.
Illustration 3 :
Find the length of a steel rod which would have the same difference in length with a copper rod of length 24 cm at all
temperatures.
(acopper= 18× 10–6K–1, asteel = 12 × 10–6K–1).
Sol. We know that, Dl = l0a.DT
According to the quesiton,
lsteel . asteel.DT = lcopper. acopper.DT
l copper.a copper 24 ´ 18 ´ 10-6
lsteel = = = 36 cm
a steel 12 ´ 10-6

Illustration 4 :
A steel rod of length 50 cm has a cross-sectional area of 0.4 cm2. What force would be required to stretch this rod by the same
amount as the expansion produced by heating it through 10ºC? (a = 10–5 k–1 and Y = 2 × 1011 N/m2)
Sol. We have,
Force = YA a.Dq = 2 × 1011 × 0.4 × 10–4 × 10–5 × 10 = 0.8 × 103 = 800 N
SPECIFIC HEAT CAPACITY
The change in temperature of a substance, when a given quantity of heat is absorbed or rejected by it is called heat capacity.
DQ
Heat capacity , S = = mc
DT
Ther mal Proper ties of Matter 281

Specific heat capacity of a substance is the amount of heat required to increase the temperature of unit mass of the substance
through unit degree.
S Q
Specific heat capacity c = = .
m m Dt
Its S.I. unit J/kgK
The specific heat capacity of water is 4186.0 Jkg–1K–1 ; 4200 Jkg–1K–1
Molar specific heat capacity is the amount of heat required for 1 mole of a substance to increase its temperature by one degree.
Q
Molar specific heat capacity (C) = molar mass × specific heat = MC =
M DT
where M is the number of moles. The S.I. unit of molar specific heat capacity is J mol–1 K–1
Specific heat of a gas : There are two specific heats of a gas
(i) Specific heat at constant volume (CV) is defined as the amount of heat required to raise the temperature of 1 gram of a
gas by 1°C when the volume of the gas is kept constant.
(ii) Specific heat at constant pressure (CP) is defined as the amount of heat required to increase the temperature of 1 gram
of a gas by 1°C when the pressure of the gas is kept constant.
Molar specific heat of a gas at constant pressure (CP) = mCP
Molar specific heat of a gas at constant volume (CV) = mCV
The ratio of two specific heats of a gas is a constant for a particular gas and depends on the atomicity of the gas.
Cp
= g Also CP – CV = R as CP > CV
Cv
Water equivalent of a body is defined as the mass of water which absorbs the same amount of heat as the body itself when
heated to the same temperature.
Water equivalent = heat capacity of a body = mass × specific heat ( W = mc )
CALORIMETRY
The branch of physics that deals with the measurement of heat is called calorimetry. A device in which heat measurement can be done
is called calorimeter.
When a hot body is put in contact with a cold one, the former gets colder and the latter warmer. From this observation it is natural to
conclude that a certain quantity of heat has passed from the hot body to the cold one.
Principle of Calorimetry or Law of Mixture
When two bodies (one being solid and other liquid or both being liquid) at different temperatures are mixed, heat will be transferred
from body at higher temperature to a body at lower temperature till both acquire same temperature. The body at higher temperature
released heat while body at lower temperature absorbs it, Then according to the principle of calorimetry, Heat lost = heat gained
Heat lost or gained by a body is given by,
Q = m × C × (q2 – q1)
Principle of calorimetry represents the law of conservation of heat energy.
Temperature of mixture (T) is always ³ lower temperature (TL) and £ higher temperature (TH),
TL £ T £ TH
The temperature of mixture can never be lesser than lower temperature. (as a body cannot be cooled below the temperature of cooling
body) and greater than higher temperature. (as a body cannot be heated above the temperatureof heating body). Furthermore usually
rise in temperature of one body is not equal to the fall in temp. of the other body though heat gained by one body is equal to the heat
lost by the other.
CHANGE OF STATE
Any state of a substance (solid/ liquid/ gas) can be changed into another by heating or cooling. The transition of a substance from
one state to another is called a change of state.
Some common changes of states of matter
(i) Melting : When heat is supplied, solid substance changes into liquid, this change of state of substance is called melting.
Heat
Solid ¾¾ ¾
® Liquid
The temperature at which the solid and the liquid states of a substance coexist in thermal equilibrium with each other is called its
melting point.
(ii) Freezing : When heat is released, liquid changes into solid, this change of state of substance is called freezing.
Cool
Liquid ¾¾¾
® Solid
282 Physics
(iii) Condensation : When vapour is cooled, it changes into liquid, this change of state is called condensation.
Cool
Vapour ¾¾¾ ® liquid
(iv) Evaporation : Conversion of liquid into gaseous state at all the temperatures is called evaporation or boiling.
Heat
Liquid ¾¾¾
® Vapour
The temperature at which the liquid and vapour states of a substance coexsist in thermal equilibrium with each other is
called its boiling point.
It is a phenomenon that occurs at the surface of liquids. The rate of evaporation increases with rise in temperature. Heat
required to change unit mass of liquid into vapour at a given temperature is called heat of evaporation at that temperature.
(v) Sublimation : It is the conversion of a solid directly into vapours.
Heat
Solid ¾¾¾ ® Vapour
Sublimation takes place when boiling point is less than the melting point. A block of ice sublimates into vapours on the
surface of moon because of very-very low pressure on its surface. Heat required to change unit mass of solid directly into
vapours at a given temperature is called heat of sublimation at that temperature.
Heating Curve
If to a given mass (m) of a solid, heat is supplied at constant rate and a graph is plotted between temperature and time, the graph
is as shown in fig. and is called heating curve. From this curve it is clear that :

(a) In the region OA temperature of solid is changing with time so, Q = mcS DT
DT
But as is the slope of temp/time curve
Dt
cS µ (1/slope of line OA)
i.e., specific heat (or thermal capacity) is inversely proportional to the slope of temp/time curve.
(b) In the region AB temperature is constant, so it represents change of state, i.e., melting of solid with melting point T1. At A melting
starts and at B all solids is converted into liquid. So between A and B substance is partly solid and partly liquid. If LF is the latent
heat of fusion
or Q = mLF
or LF µ length of line AB
i.e., Latent heat of fusion is proportional to the length of line of zero slope.
(c) In the region BC temperature of liquid increases so specific heat (or thermal capacity) of liquid will be inversely proportional to
the slope of line BC.
i.e., cL µ (1/slope of line BC)
(d) In the region CD temperature is constant, so it represents change of state, i.e., boiling with boiling point T2. At C all substance
is in liquid state while at D is vapour state and between C and D partly liquid and partly gas. The length of line CD is proportional
to latent heat of vaporisation.
i.e., LV µ length of line CD
(e) The line DE represents gaseous state of substance with its temperature increasing linearly with time. The reciprocal of slope of
line will be proportional to specific heat or thermal capacity of substance in vapour state.
Regelation : The phenomenon in which ice-melts when pressure is increased and again freezes when pressure is removed is
called regelation.
LATENT HEAT OR HIDDEN HEAT
When state of a body changes, change of state takes place at constant temperature [m.pt. or b.pt.] and heat released or absorbed is
given by, Q = mL where L is latent heat.
Ther mal Proper ties of Matter 283

Heat is absorbed, if solid converts into liquid (at m.pt.) or liquid converts into vapours (at b.pt.) and heat is released, if liquid, converts
into solid or vapours convert into liquid.
Latent heat of fusion : The quantity of heat (in kilocalories) required to change1 kg mass from solid to liquid state at its melting point.
For ice, latent heat of fusion = 80 kilocal/kg.
Latent heat of vaporization : The quantity of heat required to change 1 kg mass from liquid to vapour state at its boiling point.
For water, latent heat of vaporisation = 536 kilocal/kg.
Triple Point
It is the state at which three phases of a substance namely ice, liquid and vapour are equally stable and co-exist in equilibrium
is calld triple point. It occurs at a specific temperature and pressure. Triple point of water, at temperature Ttr = 273.16K and pressure,
Ptr = 0.46 cm of Hg.

Illustration 5 :
The temperature of a silver bar rises by 10.0°C when it absorbs 1.23 kJ of energy by heat. The mass of bar is 525 g. Determine
the specific heat capacity of silver.
Sol. Silver is far down in the periodic table, so we expect its specific heat to be on the order of one-tenth of that of water, or some
hundreds of J/kg°C.
We find its specific heat from the definition, which is contained in the equation
Q = mcsilver DT for energy input by heat to produce a temperature change.
Q 1.23 ´ 103 J
Csilver = = = 234 J/kg° C
mDT (0.525 kg) (10.0°C )

Illustration 6 :
Steam at 100ºC is passed into 1.1 kg of water contained in a calorimeter of water equivalent 0.02 kg at 15ºC till the temperature
of the calorimeter and its contents rises to 80ºC. What is the mass of steam condensed? Latent heat of steam = 536 cal/g.
Sol. Heat required by (calorimeter + water)
Q = (m1c1 + m2c2) Dq = (0.02 + 1.1 × 1) (80 – 15) = 1.12 × 65 = 72.8
If m is mass of steam condensed, then heat given by steam
Q = mL + mc Dq = m × 536 + m × 1 × (100 – 80) = 556 m
\ 556 m = 72.8
72.8
\ Mass of steam condensed , m = = 0.130 kg
556
Illustration 7 :
5g ice at 0ºC is mixed with 5g of steam at 100ºC . What is the final temperature?
Sol. Heat required by ice to raise its temperature to 100ºC,
Q1 = m1L1 + m1c1Dq1 = 5 × 80 + 5 × 1 × 100 = 400 + 500 = 900 cal
Heat given by steam when condensed,
Q2 = m2L2 = 5 × 536 = 2680 cal
As Q2 > Q1.
This means that whole steam is not even condensed.
Hence temperature of mixture will remain at 100ºC.
Illustration 8 :
5g of ice at 0ºC is dropped in a beaker containing 20 g of water at 40ºC. What will be the final temperature ?
Sol. Let final temperature = q
Heat taken by ice = m1L + m1c1Dq1 = 5 × 80 + 5 × 1(q – 0) = 400 + 5q
Heat given by water at 40ºC = m2c2Dq2 = 20 × 1 × (40 – q) = 800 – 20q
According to principle of calorimetry,
Heat given = heat taken
400
800 – 20q = 400 + 5q Þ 25q = 400 \ q= = 16ºC
25
284 Physics
HEAT TRANSFER
There are three processes by which heat can be transferred from one body to another.
(i) Conduction, (ii) Convection, and (iii) Radiation
(i) Conduction is the process of transfer of heat from one part of the body to the other without any actual movement of the
particles. It generally takes place in solids except mercury.
When a solid rod is heated, the molecules at a particular cross-section of the rod which is getting heat directly starts vibrating
more vigorously after gaining energy. As a result, they collide with the molecules at the next cross-section and transmit-
some energy to them. So, the temperature of this cross-section of the rod starts increasing, which is called Variable state.
After some time, the temperature of each cross-section becomes same as there is no more absorption of heat. The heat that
reaches to a particular cross-section is totally transmitted to the molecules of the next cross-section and a small part is lost
due to radiation. This is called Steady state.
Thermal conductivity is a measure of the ability of a solid to conduct heat.
Coefficient of thermal conductivity of a solid is defined as the rate of flow of heat per unit area per unit temperature
gradient across the solid. It depends on the nature of the solid.
rate of conduction of heat DQ / Dt
Thermal conductivity = or k=
area of cross-section × temperature gradient A ´ DT / Dx
The S.I unit of k is watt/mK.
Metals are generally good conductors of heat and non-metals are poor conductors of heat.
(ii) Convection is the process of transfer of heat from one part of a fluid to other with actual movement of the particles of
the fluid land breeze and sea breeze are based on natural convection.
(iii) Radiation is the process of transfer of heat from the source to the receiver without any actual movement of both the source
and the receiver and without heating the intervening medium.
Radiation is the fastest mode of transmission of heat, same as the speed of light c = 3 × 108 m/s.
Thermal radiations : The electromagnetic radiations emitted by a body because of its temperature.
Properties of thermal radiations :
(i) It travels in a straight line with the speed of light.
(ii) It doesn't need a medium for propagation.
(iii) It doesn't heat the intervening medium.
(iv) It obeys inverse square law.
(v) It obeys the laws of reflection and refraction of light.
(vi) It shows interference, diffraction and polarisation like light.
Reflectance, Absorptance and Transmittance of a Solid
When thermal radiations fall on a solid body a part of it is reflected, some other part is transmitted and the rest is absorbed.
The reflectance or reflecting power (r) of a solid body is the ratio of the amount of thermal energy reflected by the body in a
given time to the amount of energy incident on the body in the same time.
The absorptance or absorbing power (a) of a solid body is the ratio of the amount of thermal energy absorbed by the body in
a given time to the amount of energy incident on the body in the same time.
The transmittance or transmitting power (t) of a solid body is the ratio of the amount of thermal energy transmitted by the body
in a given time to the amount of energy incident on the body in the same time.
r+a+t=1
Monochromatic emittance of a body corresponding to a particular wavelength at a particular temperature is defined as the amount
of energy radiated per unit area per unit time of the body within a range of unit wavelength around the particular wavelength
concerned.
(a) Emissive power of a body is the total amount of radiation emitted by a body per unit time per unit area for all possible
wavelengths at a particular temperature.
(b) Emissivity of a body at a particular temperature is the ratio of emissive power of the body to the emissive power of perfect
black body at that temperature.
Perfect black body is a body which absorbs all radiations falling on it completely.
Kirchhoff ’s Law
The ratio of the emissive power ( el ) to the absorptive power ( al ) at a particular temperature, corresponding to a particular
wavelength is constant for all bodies. The constant is equal to emissive power of the perfectly black body ( El ) at that temperature,
corresponding to the same wavelength.
Ther mal Proper ties of Matter 285

el
i.e., = El = constant
al
Stefan’s Law
The amount of heat energy (E) radiated per second per unit area of a perfect black body is directly proportional to the fourth
power of its absolute temperature (T)
i.e., E = s T 4 s = Stefan's constant = 5.67 × 10–8 W/m2K4
Wien’s Displacement Law
The wavelength of maximum intensity of emission (lm) of black body radiations is inversely proportional to the absolute temperature
(T) of the black body.
1 b
lm µ Þ lm = ; b = Wien's constant = 2.892 × 10–3 mK.
T T
NEWTON’S LAW OF COOLING
Newton’s law of cooling state that the rate of cooling (or rate of loss of heat) of a body is directly proportional to the temperature
difference between the body and its surroundings, provided the temperature difference is small.
Mathematical expressions for Newton’s law of cooling. Consider a hot body at temperature T. Let T0 be the temperature of its
surroundings. According to Newton’s law of cooling,
Rate of loss of heat µ Temperature difference between the body and its surroundings
dQ dQ
or - µ (T - T0 ) or - = k(T - T0 ) ...(1)
dt dt
where k is a proportionality constant depending upon the area and nature of the surface of the body.
Let m be the mass and c the specific heat of the body at temperature T. If the temperature of the body falls by small amount
dT in time dt, then the amount of heat lost is
dQ = mc dT
dQ dT
\ Rate of loss of heat is given by = mc
dt dt
dT
Combining the above equations, we get - mc = k(T - T0 )
dt
dT k
or =- (T - T0 ) = - K(T - T0 ) ...(2)
dt mc
where K = k/mc is another constant. The negative sign indicates that as the time passes, the temperature of the body decreases.
dT
The above equation can be written as = -K dt
T - T0

1
On integrating both sides, we get ò T - T0 dT = -K ò dt
or loge (T – T0) = – Kt + c ...(3)
or T – T0 = e– Kt + c
or T = T0 + ec e–Kt
or T = T0 + C e–Kt ...(4)

where c is a constant of integration and C = ec. Equations (1), (2), (3) and (4) are the
different mathematical representations for Newton’s law of cooling. Using equation (4),
one can calculate the time of cooling of a body through a particular range of temperature.
If we plot a graph by taking different values of temperature difference DT = T – T0 along
y-axis and the corresponding values of t along x-along, we get a curve of the form shown
DT(°C)

in Fig. It clearly shows that the rate of cooling is higher initially and then decreases
as the temperature of the body falls.
Experimental verification of Newton’s law of cooling. The experimental set-up used for
verifying Newton’s law of cooling is shown in Fig. The set-up consists of a double walled
vessel (V) containing water in between the two walls. A copper calorimeter (C) containing
hot water is placed inside the double walled vessel. Two thermometers through the corks Time (minute)
are used to note the temperatures T of hot water in calorimeter and T0 of water in between
the double walls respectively.
286 Physics

Stirrer
T T0
Temperature of hot water in the calorimeter is noted after fixed intervals
of time, say after every one minute of stirring the water gently with a stirrer. Tap water
Continue noting its temperature till it attains a temperature about 5°C above Calorimeter
that of surroundings. Plot a graph between loge (T – T0) and time (t). The Hot water
nature of the graph is observed to be a straight line, having a negative C
Double
slope, as shown in Fig. This verifies newton’s law of cooling. V walled vessel
Illustration 9 :
An ice box made of 1.5 cm thick styrofoam has dimensions 60 cm × 30cm. It contains ice at 0ºC and is kept in a room at 40ºC.
Find the rate at which the ice is melting. Latent heat of fusion of ice = 3.36 × 105 J/kg and thermal conductivity of styrofoam
= 0.4 W/mºC.
Sol. The total surface area of the walls = 2 (60 cm × 60 cm + 60 cm × 30 cm + 60 cm × 30 cm) = 1.44 m 2
The thickness of the wall = 1.5 cm = 0.015m
The rate of heat flow into the box
2
DQ KA(q1 - q 2 ) (0.04W / m - º C)(1.44m )(40º C)
= = = 154 W.
Dt x 0.015m

Dq 154W
The rate at which the ice melts = = 5 = 0.46 g/s
Dt × k 3.36 ´ 10 J / kg

Illustration 10 :
Compare the rate of loss of heat from a metal sphere of the temperature 827°C, with the rate of loss of heat from the same
sphere at 427 °C, if the temperature of surroundings is 27°C.
Sol. Given : T1 = 827 °C = 1100 K,
T2 = 427 °C = 700 K and T0 = 27 °C = 300 K
According to Steafan's law of radiation,
dQ
= s Ae (T4 – T04)
dt

æ dQ ö
çè ÷ø
dt 1 (T14 - T04 ) [(1100)4 - (300)4 ]
= = æ dQ ö æ dQ ö
\ æ dQ ö (T24 - T04 ) [(700) 4 - (300)4 ] = 6.276 or çè ÷ :ç = 6.276 :1
çè ÷ø dt ø 1 è dt ÷ø 2
dt 2

Illustration 11 :
A copper sphere cools from 62°C to 50°C in 10 minutes and to 42°C in the next 10 minutes. Calculate the temperature of the
surroundings.
q1 - q 2 é q1 + q 2 ù
Sol. By Newton's law of cooling, = -k ê 2 - q0 ú ....(1)
t ë û

62 - 50 é 62 + 50 ù
A sphere cools from 62°C to 50°C in 10 min. = -k ê - q0 ú ....(2)
10 ë 2 û

50 - 42 é 50 + 42 ù
Now, sphere cools from 50°C to 42°C in next 10 min. = -k ê - q0 ú ....(3)
10 ë 2 û

56 - q0 1.2
Dividing eqn. (2) by (3) we get, 46 - q = 0.8
0

or, 0.4q0 = 10.4 \ q0 = 26°C


Ther mal Proper ties of Matter 287
288 Physics

Textbook Exercises
11.1 The triple points of neon and carbon dioxide are 24.57 (Substituting the value of a in the equation
K and 216.55 K respectively. Express these temperatures
(123.4 - 101.6) ´ 327.34
on the Celsius and Fahrenheit scales. \ T2 = + 273.16
Sol. If temperature in Kelvin scale is K and that in Celsius scale 63.9
is C then, C = K – 273.15 = 111.67 + 237.16 = 348.83 K
\ For Neon, C = 24.57 – 273.15 = – 248.58°C; For CO2, 11.4 Answer the following :
C = 216.55 – 273.15 = – 56.60°C (a) The triple point of water is a standard fixed point
If temperature in Fahrenheit scale is F then, in modern thermometry. Why? What is wrong in
taking the melting point of ice and the boiling point
F - 32 K - 273.15 of water as standard fixed points (as was originally
=
180 100 done in the Celsius scale)?
180 (b) There were two fixed points in the original Celsius
For neon, F = (24.57 – 273.15) + 32
100 scale as mentioned above which were assigned the
= – 415.44°F number 0°C and 100°C respectively. On the absolute
180 scale, one of the fixed points is the triple point of
For CO2, F = (216.55 – 273.15) + 32 water, which on the Kelvin absolute scale is assigned
100
the number 273.16 K. What is the other fixed point
= – 69.88°F on this (Kelvin) scale?
11.2 Two absolute scales A and B have triple points of water (c) The absolute temperature (kelvin scale) T is related
defined to be 200 A and 350 B. What is the relation to temperature tc on the Celsius scale by tc = T –
between TA and TB ? 273.15. Why do we have 273.15 in this relation, 0
Sol. According to the question, triple point of water = 200 A and not 273.16?
= 350 B = 273.16 K (d) What is the temperature of the triple point of water
[Triple point of water in Kelvin scale = 273.16 K] on an absolute scale whose unit interval size is equal
273.16 273.16 to that of the Fahrenheit scale?
\ 1A = K and 1 B = K Sol. (a) Triple point of water has a unique value (273.16 K)
200 350
at fixed values of pressure and volume. But melting
If TA and TB represent the triple point of water on scales
point of ice and boiling point of water do not have
A and B then,
unique value as they depend both on pressure and
273.16 temperature.
T A = 273.16 T B (b) The other fixed point is the absolute zero.
200 350
(c) On Celsius scale, 0°C corresponds to melting point
TA 200 4 4 of ice at normal pressure and the same temperature
Þ = = \ TA = T
TB 350 7 7 B in Kelvin scale is 273.15 K. The triple point of water
in Kelvin scale is 273.16 K.
11.3 The electrical resistance in ohms of a certain
\ By the relation, tc = T – 273.15,
thermometer varies with temperature according to the the triple point of water on Celsius scale= 273.16
approximate law : – 273.15 = 0.01°C
R = R0 [1 + a (T – T0)]. The resistance is 101.6 W at (d) Relation between temperature in Fahrenheit scale and
the triple point of water 273.16 K, and 165.5 W at the absolute scale is
normal melting point of lead 600.5 K. What is the
temperature when the resistance is 123.4 W? F - 32 K - 273.15
= ..............(i)
Sol. R0 = 101.6 W, T 0 = 273.16 K, R 1 = 165.5 W, 180 100
T1 = 600.5 K, R2 = 123.4 W, T2 = ? For another set of temperature,
Using the relation, R = R0 [1 + a (T – T0)] F'- 32 K '- 273.15
165.5 = 101.6 [1 + a (600.5 – 273.16)] = .............(ii)
180 100
= 101.6 [1 + a × 327.34] Subtracting (i) from (ii), we get
165.5 - 101.6 F '- F K '- K
Þ a= . =
101.6 ´ 327.34 180 100
Also, 123.4 = 101.6 [1 + a (T2 – 273.16)])
180
é 63.9 ù Þ F' – F = × (K ' - K)
or, 123.4 = 101.6 ê1 + (T2 - 273.16) ú 100
ë 101.6 ´ 327.34 û
If K' – K = 1K, then
Ther mal Proper ties of Matter 289

180 9 100.0216
F' – F = ×1= = × 63 = 63.0136 cm
100 5 100
Also, length of the same rod at 27°C
\ The triple point of water (273.16 K) in the new scale = 63 × 1 = 63 cm
9 11.7 A large steel wheel is to be fitted on to a shaft of the same
is 273.16 × = 491.69. material. At 27°C, the outer diameter of the Shaft is
5
11.5 Two ideal gas thermometers A and B use oxygen and 8.70cm and the diameter of the central hole in the wheel
hydrogen respectively. The following observations are is 8.69cm. The shaft is cooled using dry ice. At what
made: temperature of the shaft does the wheel slip on the shaft?
Assume, coefficient of linear expansion of the steel to be
Temperature Pressure Pressure
constant over the required temperature range,
thermometer A thermometer B
asteel = 1.20 × 10–5 K –1 .
Triple point 1.250 × 105 Pa 0.200 × 105 Pa Sol. T1 = 27°C = 273+27 = 300 K, Length
of water at temperature T1 = l1= 8.70 cm
Normal melting 1.797 × 10 5 Pa 0.287 × 105 Pa Length at temperature T2 = l2 = 8.69cm, Change in length
point of Sulphur
= Dl = l2 – vl1= l1× a × DT
(a) What is the absolute temperature of normal melting Þ 8.69 – 8.7 = 8.7 × 1.2 × 10–5 (T2 – 300)
point of sulphur as read by thermometers A and B?
(b) What do you think is the reason behind the slight - 0.01
Þ T2 - 300 = = - 95.8
difference in answers of thermometers A and B? (The 8.7 ´ 1.2 ´ 10 -5
thermometers are not faulty). What further procedure is Þ T2 = 300 – 95.8 = 204.2 K = – 68.8°C
needed in the experiment to reduce the discrepancy 11.8 A hole is drilled in a copper sheet. The diameter of the
between the two readings ? hole is 4.24 cm at 27.0°C. What is the change in
Sol. (a) Triple point of water = 273.16 K, the diameter of the hole when the sheet is heated to
Let T be the melting point of sulphur. 227°C ? Coefficient of linear expansion of copper =
For thermometer A, 1.70 × 10–5 K–1
P 1.797 ´ 105 Sol. Area of the hole at 27°C
T= × 273.16 = × 273.16
Pw 1.250 ´ 105 pD12 p
= S1 = = ´ (4.24)2 cm 2
= 392.69 K [ Q P µT] 4 4
For thermometer B,
pD 22
0.287 ´ 105 Area of the hole at 227°C = S2 =
T= × 273.16 = 391.98 K 4
0.200 ´ 105
(b) The cause of this slight difference in the two b = 2a = 2×1.70×10–5 = 3.4 × 10–5 / °C.
temperatures is that oxygen and hydrogen gases are Increase in area = S2 – S1 = S1bDT
not perfectly ideal.
To reduce the discrepancy, the readings should be \ S2 = S1 + S1bDT = S1 (1 + bDT)
taken at a very low pressure where the gases
approach to the ideal gas behaviour. pD 22 p
= ´ (4.24) 2 [1 + 3.4 × 10–5 × (227 – 27)]
11.6 A steel tape 1 m long is correctly calibrated for a 4 4
temperature of 27.0°C. The length of a steel rod
measured by this tape is found to be 63.0 cm on a hot D22 = (4.24)2 × 1.0068 Þ D2 = 4.2544 cm
day when the temperature is 45.0°C. What is the actual \ Change in diameter
length of the steel rod on that day? What is the length = 4.2544 – 4.24 = 0.0144 cm
of the same steel rod on a day when the temperature is 11.9 A brass wire 1.8 m long at 27°C is held taut with little
27.0°C? Coefficient of linear expansion of steel = 1.20 tension between two rigid supports. If the wire is cooled
× 10–5K–1. to a temperature of –39°C, what is the tension developed
Sol. L1 = 100 cm & T1 = 27°C in the wire, if its diameter is 2.0 mm? Coefficient of
At 45°C, L2 = L1 + a L1DT linear expansion of brass = 2.0 × 10–5 K–1, Young’s
= 100 + 1.2 × 10–5 × 100 × (45 – 27) modulus of brass = 0.91 × 1011 Pa.
= 100.0216 cm Sol. Here, l = 1.8 m, T1 = 27°C, T2 = – 39°C,
\ Length of 1 cm mark at 27°C on this scale becomes r = 1 mm = 10–3m
100.0216 a = 2 × 10–5/°C Y = 0.91 × 1011 N/m2
at 45°C = cm.
100 Fl Fl
\ Actual length of the steel rod on that day Q Y= Þ Dl =
A Dl AY
290 Physics
11.13 A copper block of mass 2.5 kg is heated in a furnace
Fl
Also, D l = a l DT \ = alDT to a temperature of 500°C and then placed on a large ice
AY block. What is the maximum amount of ice that can melt?
\ F = AYaDT = p r2Ya(T2 – T1) Specific heat of copper is = 0.39 Jg–1 K–1, heat of fusion
22 of water = 335Jg–1
F= × (10–3)2 × 0.91 × 1011 × 2 Sol. Mass of copper = m1 = 2.5kg = 2500g,
7
× 10–5(– 39 – 27) Change in temperature = Dt = 500 – 0 = 500°C
= – 3.77 × 102 N Specific heat of copper = c1= 0.39 J/g/°C,
The negative sign indicates that the force is inwards Latent heat of fusion of ice = L = 335 J/g
leading to contraction of the wire. Let mass of ice melted = m2
11.10 A brass rod of length 50 cm and diameter Q Heat lost by copper = Heat gained by ice
3.0 mm is joined to a steel rod of the same length and Þ m1c1Dt = m2L
diameter. What is the change in length of the combined
m1c1Dt 2500 ´ 0.39 ´ 500
rod at 250°C, if the original lengths are at 40.0°C? Is m2 = =
there a thermal stress developed at the function ? The L 335
ends of the rod are free to expand (coefficient of linear = 0.1455 × 104 = 1455g
expansion of brass 11.14 In an experiment on the specific heat of a metal, a 0.20
= 2.0 × 10–5 K–1, steel = 1.2 × 10–5 K–1) kg block of a metal at 150°C is dropped in a copper
Sol. Dl 1 = l 2 a 1 DT = 50 × (2.1 × 10 –5 ) × (250 – 40) calorimeter (of water equivalent 0.025 kg) containing 150
= 0.2205 cm cm3 of water at 27°C. The final temperature is 40°C.
Dl 2 = l 2 a 2 DT = 50 × (1.2 × 10 –5 ) × (250 – 40) Compute the specific heat of the metal . If heat losses
= 0.126 cm to the surroundings are not negligible, is your answer
\ Chan ge in length of the combined rod greater or smaller than the actual value for specific heat
= D l1 + Dl2= 0.2205 + 0.126 = 0.3465 cm. of the metal?
11.11 The coefficient of volume expansion of glycerine is 49 Sol. Mass of the metal m1 = 0.2kg = 200g
× 10–5 K–1. What is the fractional change in its density Fall in temperature = Dt1=150 – 40 = 110°C
for a 30°C rise in temperature? Heat lost by the metal = DQ1 = mcDt1
Sol. g = 49 × 10–5/°C, DT = 30°C [c = specific heat of metal]
V2 = V1 + DV = V1 (1 + gDT) = 200 × c ×110 .......(1)
Þ V2 = V1 (1+ 49 × 10–5 × 30) = 1.0147 V1 3
Volume of water = 150cm ; Mass of water = m2 = 150g
M M Water equivalent of calorimeter
Q r1 = & r2 = = w = 0.025kg = 25g
V1 V2
Rise in temperature of water and calorimeter
M = Dt2= 40 – 27 = 13°C
= = 0.9855 r1
1.0147V1 Heat gained by water and calorimeter
\ Fractional change in density = DQ2 = (m2 + w) Dt2 = (150 + 25) × 13
r -r (r1 - 0.9855 r1 ) = 175 × 13 .......(2)
= 1 2 = = 0.0145 Q Heat lost = heat gained,
r1 r1
200 × c × 110 = 175 × 13
11.12 A 10 kW drilling machine is used to drill a bore in a
small aluminium block of mass 8.0 kg. How much is the 175≥13
rise in temperature of the block in 2.5 minutes, assuming c< < 0.1
200≥110
50% of power is used up in heating the machine itself
The value of c is less than its actual value, as some heat
or lost to the surroundings. Specific heat of aluminium
= 0.91J/g–1 K–1 is lost to the surroundings.
Sol. Total energy = P× t = 104W ×150 s = 15 × 105J 11.15 Given below are observations on molar specific heats at
As 50% of the heat is lost, room temperature of some common gases.
Gas Molar specific heat (Cv)
50
\ Energy available = ≥15≥105 (cal mol–1 K–1)
100
Hydrogen 4.87
= 7.5 × 105 J
Nitrogen 4.97
Q Q = m C D T = 8 × 103 × 0.91 × DT
Oxygen 5.02
\ 7.5 × 105 = 8 × 103 × 0.91 × DT
Nitric oxide 4.99
7.5≥105 Carbon monoxide 5.01
Þ DT = < 103↓C
8≥103 ≥ 0.91 Chlorine 6.17
Ther mal Proper ties of Matter 291

The measured molar specific heats of these gases are (a) CO2 at 1 atm pressure and temperature – 60 °C is
markedly different from those for monatomic gases. compressed isothermally. Does it go through a liquid
Typically, molar specific heat of a monatomic gas is 2.92 phase?
cal/mol K. Explain this difference. What can you infer (b) What happens when CO2 at 4 atm pressure is cooled
from the somewhat larger (than the rest) value for from room temperature at constant pressure ?
chlorine ? (c) Describe qualitatively the changes in a given mass
of solid CO2 at 10 atm pressure and temperature
Sol. The gases are diatomic, and have other degrees of – 65 °C as it is heated up to room temperature at
freedom (i.e. have other modes of motion) possible besides constant pressure.
the translational degrees of freedom. To raise the (d) CO 2 is heated to a temperature 70 °C and
temperature of the gas by a certain amount, heat is to be compressed isothermally. What changes in its
supplied to increase the average energy of all the modes. properties do you expect to observe?
Consequently, molar specific heat of diatomic gases is Sol. (a) No, vapour condenses to solid directly.
more than that of monatomic gases. It can be shown that (b) It condenses to solid directly without passing
if only rotational modes of motion are considered, the through the liquid phase.
molar specific heat of diatomic gases is nearly (5/2) R (c) It turns to liquid phase and then to vapour phase.
which agrees with the observations for all the gases listed The fusion and boiling points are where the
in the table, except chlorine. The higher value of molar horizontal line on P-T diagram at the constant
pressure of 10 atm intersects the fusion and
specific heat of chlorine indicates that besides rotational
vaporization curves.
modes, vibrational modes are also present in chlorine at
(d) It will not exhibit any clear transition to the liquid
room temperature. phase, but will depart more and more from ideal gas
11.16 Answer the following questions based on the P-T phase behaviour as its pressure increases.
diagram of carbon dioxide : 11.18 A child running a temperature of 101°F is given an
antipyrin (i.e. a medicine that lowers fever), which causes
P an increase in the rate of evaporation of sweat from his
(atm)
body. If the fever is brought down to 98°F in 20 min what
73.0
is the average rate of extra evaporation caused by the
56.0 drug? Assume the evaporation mechanism to be the only
Liquid
way by which heat is lost. The mass of the child in 30
Solid kg. The specific heat of human body is approximately the
C
same as that of water and latent heat of evaporation of
5.11 water at that temperature is about 580 cal g–1.
1.0 Vapour Sol. Decrease in temperature = 101 – 98 = 3°F
5 5
–78.5 –56.6 20 31.1 T (°C) = 3≥ < ↓C ; Mass of child = m = 30 kg
9 3
(a) At what temperature and pressure can the solid, Specific heat of water = specific heat of human body
liquid and vapour phases of CO 2 co-exist in = c = 1000 cal/kg/°C
[ Heat lost by the child
equilibrium ?
(b) What is the effect of decrease of pressure on the 5
= mcDt = 30 × 1000 × = 5000 cal.
fusion and boiling point of CO2? 3
(c) What are the critical temperature and pressure for Let m ' be the mass of water evaporated then m ' L = mcDt
CO2? What is their significance? 5000
(d) Is CO2 solid, liquid or gas at (a) –70 °C under 1 or m ' = = 8.620
580
atm, (b) – 60 °C under 10 atm, (c) 15 °C under 56
[ Average rate of extra evaporation
atm ?
Sol. (a) At the triple point temperature = –56.6°C and 8.62
= = 0.431 g/min.
pressure = 5.11 atm. 20
(b) Both the boiling point and freezing point of CO2 11.19 A ‘thermacole’ icebox is a cheap and efficient method for
decrease if pressure decreases. storing small quantities of cooked food in summer in
(c) The critical temperature and pressure of CO2 are 31.1 particular. A cubical icebox of side 30 cm has a thickness
°C and 73.0 atm respectively. Above this temperature, of 5.0 cm. If 4.0 kg of ice is put in the box, estimate the
CO2 will not liquefy even if compressed to high amount of ice remaining after 6 h. The outside
pressures. temperature is 45 °C, and co-efficient of thermal
(d) (a) vapour (b) solid (c) liquid conductivity of thermacole is 0.01 J s–1 m–1 K–1. [Heat
of fusion of water = 335 × 103 J kg–1]
11.17 Answer the following questions based on the P-T phase
Sol. Length of each side = l = 30 cm = 0.3 m, Thickness = Dx
diagram of CO2.
= 5 cm = 0.05 m
292 Physics
Total surface area of the box, Energy radiated by the hot iron when it is in the
A = 6l2 = 6 × 0.3 × 0.3, surroundings of temperature To is
Temperature difference = DT = 45 – 0 = 45°C
k = 0.01 J/s/m/°C, E ' < ρ(T 4 , T04 )
Time = Dt = 6 hrs = 6 × 60 × 60 s Q Pyrometer works on the principle that the
Latent heat of fusion = L = 335×103J/kg brightness of a surface depends on its
If m is the mass of ice melted, then temperature.
[ It gives a low value for the temperature, of iron
æ DT ö KA æ DT ö in the open.
mL = kA ç Dt = m = ç ÷ Dt
è Dx ÷ø L è Dx ø (d) The infrared radiation received by the earth during
the day from the sun is trapped by the atmosphere.
0.01≥0.54≥45≥6≥60≥60 The lower layers of the atmosphere reflect the
= = 0.313 kg
0.05≥335≥103 infrared radiation back to the surface of the earth. So
[ Mass of ice left = 4 – 0.313 kg = 3.687 kg if the atmosphere in not there, the earth will be too
11.20 A brass boiler has a base area of 0.15 m2 and thickness cold to live in.
1.0 cm. It boils water at the rate of 6.0 kg/min, when (e) Steam at 100°C contains more heat than water at
placed on a gas stove. Estimate the temperature of the 100°C as latent heat is given to water to convert it
part of the flame in contact with the boiler. Thermal to steam. So heating system based on circulation
conductivity of brass = 109 J s–1 m–1 k –1 . Heat of of steam are more efficient than system based on
vaporisation of water = 2256 × 103 J k/g–1 circulation of water.
Sol. A = 0.15 m2, Dx = 1 cm = 0.01m 11.22 A body cools from 80°C to 50°C in 5 minutes. Calculate
the time it takes to cool from 60°C to 30°C. The
Q 6 kg 1
Transfer of heat = = kg/s temperature of the surroundings is 20°C.
t min 10
Sol. From Newton’s law of cooling,
ΧQ æ ΧT ÷ö
< kA çç
Q Χt çè Χx ÷÷ø dT
< ,k (T , T0 ), where T and T are the temperatures
dt 0
2256 ´ 103 t , 100
Þ = 6.09 × 0.15 × of the body and the surrounding respectively
10 0.01
If the temperature of the body decreases from T1 to T2
Þ 2256
t , 100 < < 24.7 in time t,
609≥ 0.15
t < 124.7↓C T2
dT
t
11.21 Explain, why : ò T - T0
= - ò kdt
(a) a body with large reflectivity is a poor emitter. T1 0
(b) a brass tumbler feels much colder than a wooden
tray on a chilly day. T
Þ log(T – T0) |T12 = – kt
(c) an optical pyrometer (for measuring high
temperatures), caliberated for an ideal black body
radiation, gives too low a value for the temperature T2 , T0
Þ loge < ,kt
of a red hot iron piece in the open, but gives a correct T1 , T0
value for the temperature when the same piece is
in a furnace. T2 , T0
(d) the Earth without its atmosphere would be Þ 2.303log10 < ,kt
T1 , T0
inhospitably cold.
(e) heating system based on circulation of steam are T1 , T0
more efficient in warming a building than those Þ 2.303log10 < kt
T2 , T0
based on circulation of hot water.
Sol. (a) A body with large reflectivity will absorb very less
2.303 T ,T
amount of heat, so, it will be a poor emitter too. Þ t< log10 1 0
(b) When we touch a brass tray on a chilly day heat k T2 , T0
flows from our body to the tray very fast as brass Here, T1 = 80°C, T2 = 50°C, T0 = 20°C;
is a good conductor of heat, so, it appears colder,
whereas wood is a bad conductor of heat, so, heat t = 5 min = 5 × 60 = 300 sec
does not flow to wood from our body and we feel 2.303 80 , 20
it warm. [ 5≥ 60 < log10
k 50 , 20
(c) By Stefan’s law, energy radiated when the hot iron
piece is in the open is E = sT4.
Ther mal Proper ties of Matter 293

2.303 2.303
= log10 (2) ...(1) = log10 (4) ...(2)
k k
Also, if T1 = 60°C, T2 = 30°C, T0 Dividing eq. (2) by (1),
= 20°C, t = ? t log10 (4) 0.6012
< < <2
2.303
log10
60 - 20 5≥ 60 log10 (2) 0.3010
t=
k 30 - 20 t = 5 × 60 × 2 = 10 × 60 s = 10 min

Practice Questions
Very Short Answer Questions [1 Mark Qs.] 4. Why is it hotter at the same distance over the top of a
fire than in front of it?
1. What do you mean by triple point ? 5. If a car is parked in sun with all the windows closed, it
2. Why the triple point of water is considered as a standard is found that it becomes hotter than the air outside. Explain,
fixed point in thermometry? why?
3. What is the specific heat of a gas in an isothermal process? 6. A cloudy night is hotter than a night with clear sky. Explain,
4. What is the specific heat of gas in an adiabatic process? why?
5. Two rods A and B are of equal lengths. Each rod has the 7. Heat is generated continuously by an electric heater, but
ends at temperatures T1 and T2. What is the condition that its temperature doesn't increase after some time. Explain.
8. Explain how convection current maintains temperature at
will ensure equal rates of flow of heat through the rods
the core of the earth?
A and B?
9. What are the basis requirements of a cooking utensil in
6. What is the thermal conductivity of a perfect heat respect of specific heat, thermal conductivity and
conductor and a perfect heat insulator? coefficient of expansion?
7. The temperature of a black body is increased from 200 K 10. Derive Newton’s law of cooling from Stefan-Boltzman law.
to 400 K. How will the ratio of emission of energy change? 11. Two rods of the same area of cross-section, but of lengths
8. Thr ee stars appear as green, red and blue l1 and l2 and conductivities K1 and K2 are joined in series.
in colour. Which has maximum temperature? Show that the combination is equivalent of a material of
9. Why a clinical thermometer should not be sterilized by l1 + l2
boiling? conductivity K =
æ l1 ö æ l2 ö
10. Explain why a beaker filled with water at 4°C overflows if
çè K ÷ø + çè K ÷ø
the temperature is decreased or increased? 1 2
11. Two identical rectangular strips of copper, and the other 12. Calculate the stress developed inside a tooth cavity filled
of steel are riveted to form a bimetallic strip. What will with copper when hot tea at temperature of 57°C is drunk.
happen on heating? You can take body (tooth) temeprature to be 37° C and
12. Why must telephone or power lines necessarily sag a little? a = 1.7 × 10 –5 /°C, bulk modulus for copper
13. Which object will cool faster when kept in open air, the = 140 × 109 N/m2.
one at 300°C or the one at 100°C Why? 13. A rail track made of steel having length 10 m is clamped
14. Why a body with large reflectivity is a poor emitter? on a railway line at its two ends (Fig.). On a summer day
15. Can the temperature of a body be negative on the Kelvin due to rise in temperature by 20° C, it is deformed as
scale? shown in figure. Find x (displacement of the centre) if asteel
16. The earth without its atmosphere would be inhospitably = 1.2 × 10–5 / °C.
cold. Explain.
17. Heating systems based on circulation of steam are more L
efficient in warming a building than those based on x
circulation of hot water. 1/2 (L + D L)
Short Answer Questions [2 or 3 Marks Qs.]
Long Answer Questions [5 Marks Qs.]
1. Why gases have two specific heats?
1. Derive a relation between the three coefficients of thermal
2. Why does density of a solid, liquid or gas change on expansion - linear (a) area (b) volume (g).
heating? 2. Derive an expression for the thermal conductivity of a solid
3. Why specific heat of a gas at constant pressure is greater slab.
than that at constant volume?
294 Physics
3. Using Newton’s law of cooling, calculate the time of 8. At what temperature the Fahrenheit and Kelvin scale will
cooling of a body. give the same reading?
4. Discuss briefly energy distribution of black body radiation. 9. The brass scale of a barometer gives correct reading at 0°C.
Hence deduce Wien’s displacement law and Stefan’s law. Coefficient of linear expansion of brass is 20 × 10–6/°C. The
Numerical Questions [3 or 5 Marks Qs.] barometer reads 75 cm at 27°C. What is the atmospheric
pressure at 27°C?
1. A metal plate 4 mm thick has a temperature difference of 10. A sheet of brass is 50 cm long and 10 cm broad at 0°C. The
32°C between its faces. It transmits 200 k cal/h through an area of the surface increases by 1.9 cm2 at 100°C. Find the
area of 5 cm2. Calculate thermal conductivity of the material coefficient of linear expansion of brass.
of the plate. 11. A clock with an iron pendulum keeps correct time at 20°C.
2. Calculate the difference in temperatures between two sides How much will it lose or gain per day if temperature
of an iron plate 20 mm thick, when heat is conducted at the changes to 40°C? Coefficient of cubical expansion of iron
rateof 6 × 105 cal/min m2. (K for the metal is 0.2 cal/s cm K.) = 36×10–6/°C.
3. Calculate the temperature at which a perfectly black body 12. How many gram of ice at – 14°C are needed to cool 200 gram
radiates at the rate of one watt/cm2. Stefan’s constant = 5.67 of water from 25°C to 10°C. Take specific heat of ice = 0.5
cal/g/°C and latent heat of ice = 80 cal/g.
× 10–5 erg/s/cm2/K4.
13. A piece of iron of mass 100 g is kept inside a furnace for a
4. The surface temperature of a hot body is 1227°C. Find the long time and then put in a calorimeter of water equivalent
wavelength at which it radiates maximum energy. Given, 10 g containing 240 g of water at 20°C. The mixture attains
Wien’s constant = 0.2898 cm K. an equilibrium temperature of 60°C. Find the temperature of
5. The temperature of a body is increased from 27°C to 127°C. the furnace; specific heat capacity of iron = 470 J/kg/C.
By what factor would the radiation emitted by it increase? 14. Calculate CP for air, given that Cv = 0.162 cal/g/K and density
6. The solar constant on the surface of earth is 1400 N/m2. of air at N.T.P. is 0.001293 g/cm3.
What will be its value on the surface of Jupiter, which is 15. Calculate two specific heats of a gas from the following
about 5.2 A.U, away from sun? C
data : d = p = 1.51 and density of gas at N.T.P. is 1.234 g/
7. A Celsius and a Fahrenheit thermometer are put in a hot Cv
bath. The reading on Fahrenheit thermometer is just three litre, J = 4.2 × 107 erg/cal
times the reading on Celsius thermometer. What is the
temperature of the bath?

HOTS/Exemplar Questions
Very Short Answer Questions [1 Mark Qs.] 2. Which has the greater amount of internal energy an iceberg
or a cup of hot coffee? Explain. [HOTS]
1. Why will a watermelon stay cool for a longer time than 3. In winter, why do road surfaces on bridges tend to be more
sandwiches when both are removed from a cooler on a hot icy than the road surfaces on the land at either side?
day? [HOTS]
[HOTS]
2. Is cold the opposite of thermal energy or the lack of it? 4. During summers in India, one of the common practice to
[HOTS] keep cool is to make ice balls of crushed ice, dip it in flavoured
3. Does a substance that heats up quickly have a high or a low sugar syrup and sip it. For this a stick is inserted into crushed
ice and is squeezed in the palm to make it into the ball.
specific heat capacity? [HOTS]
Equivalently in winter in those areas where it snows, people
4. Is the bulb of a thermometer made of diathermic or adiabatic make snow balls and throw around. Explain the formation of
wall? [Exemplar] ball out of crushed ice or snow in the light of P-T diagram of
5. Why does a metal bar appear hotter than a wooden bar at water. [Exemplar]
the same temperature? Equivalently it also appears cooler
than wooden bar if they are both colder than room Long Answer Questions [5 Marks Qs.]
temperature. [Exemplar]
1. (a) A brass rod of length 50 cm and diameter 3.0 mm is
Short Answer Questions [2 or 3 Marks Qs.] joined to a steel rod of the same length and diameter.
What is the change in length of the combined rod at
1. What is the reason for ice being less dense than water? 250ºC, if the original lengths are at 40.0ºC? Is there a
[HOTS]
Ther mal Proper ties of Matter 295

thermal stress developed at the junction? The ends of 2. A thin rod having length L0 at 0°C and coefficient of linear
the rod are free to expand. expansion a has its two ends maintained at temperature q1
(Coefficient of linear expansion of brass = 2 × 10–5 ºC– and q2, respectively. Find its new length. [Exemplar]
1, and that of steel =1.2×10–5 ºC–1). 3. Calculate the stress developed inside a tooth cavity filled
(b) A metal plate 16 mm thick has a temperature difference with copper when hot tea at temperature of 57°C is drunk.
of 32 ºC between its opposite faces. It transmits 400 You can take body (tooth) temperature to be 37°C and
kcal in 2 hours through an area of 5 cm2. Calculate the a = 1.7 × 10–5/°C, bulk modulus for copper = 140 × 109 N/m2.
coefficient of thermal conductivity. [Exemplar]
(c) Explain why a beaker filled with water at 4ºC overflows
if the temperature is decreased or increased?
[HOTS]

CHAPTER TEST

Time : 45 min. Max. Marks : 24

Directions : (i) Attempt all questions


(ii) Questions 1 to 5 carry 1 mark each.
(iii) Questions 6, 7 and 8 carry 3 marks each.
(iv) Questions 9 and 10 carry 5 marks each.

1. How many calories of heat are required for external work when one gram mole of a gas is heated by 1°C at constant
pressure?
2. Two copper balls having masses 5 g and 10 g collide with a target with the same velocity. If the total energy is used in
heating the balls. Which ball will attain higher temperature?
3. A sphere, a cube and a thin circular plate, all of same material and same mass are initially heated to same high temperature.
Which will cool fastest?
4. A faulty thermometer has its fixed points marked as 5° and 95°. The temperature of a body as measured by the faulty
thermometer is 59°. Find the correct temperature of the body on Celsius scale.
5. Two large holes are cut in a metal sheet. If the sheet is heated, how will the diameters of the holes change?

C B

6. How much should the temperature of a brass rod be increased so as to increase its length by 1% ? Given a for brass is
0.00002 / °C
7. Calculate difference in specific heats of 1 g of air at N.T.P. Given density of air at N.T.P. is 1.293 g/l, J = 4.2 × 107 erg/cal.
8. Distinguish between conduction, convection and radiation.
9. Assume that the thermal conductivity of copper is four times that of brass. Two rods of copper and brass of the same
length, and cross-section are joined end to end. The free end of the copper rod is kept at 0°C and the free end of the brass
rod at 100°C. Calculate the temperature at the junction of the two rods at equilibrium. Ignore radiation losses.
10. (a) How much heat must be absorbed by ice of mass m = 720 g at – 10°C to take liquid state at 15°C ?
(b) If we supply the ice with a total energy of only 210 kJ (as heat), what then are the final state and temperature of the
water?
296 Physics

Solutions
12. The sag is allowed for contraction in winter. If no sag is
PRACTICE QUESTIONS
allowed, the wire may snap in extremely cold weather.
Very Short Answer Questions 13. The object at 300°C will cool faster than the object at
1. It is a particular point in the phase diagram, representing 100°C. This is in accordance with Newton’s law of cooling,
a particular pressure and temperature at which a substance Rate cooling of an object µ Temperature between the
can coexist in all three phases in solid, liquid and gas. object and its surroundings
2. Triple point of water is fixed and occurs only for particular 14. A body whose reflectivity is large would naturally absorb
values of pressure = 0.46 cm of Hg and temperature less heat. So, a body with large reflectivity is a poor emitter.
= –273.16 K.
15. No. This is because the absolute zero on the Kelvin scale
Q is the minimum possible temperature.
3. Specific heat c =
m DT 16. The atmosphere serves purpose of a blanket over the earth
For isothermal process, DT = 0 and it does not allow earth’s heat to be radiated during
\ C=¥ night.
4. For adiabatic process, Q = 0 17. This is due to fact that steam contains more heat in the
\ C = 0. form of latent heat (540 calories/gram) than water.
5. Q Rate of flow of heat is equal Short Answer Questions
DQA DQB 1. Since, gases can be heated under different conditions of
\ =
Dt Dt pressure and volume, so, amount of heat required to raise
the temperature of unit mass of a gas by one degree under
k1 A1 (T1 - T2 ) k A (T - T ) different conditions will not be same.
Þ = 2 2 1 2
L1 L2 2. When a given mass of a solid, liquid or gas is heated,
it expands, so its volume increases. Density of a substance
A1 k2 is inversely proportional to volume, so, if volume increases,
Q L1 = L2 \ = k density decreases.
A2 1
Let, V = initial vol. of a given mass of a substance
This is the required condition.
6. For perfect heat conductor, thermal conductivity is infinity V ' = New vol. of the same mass of the substance for a
whereas for perfect insulators, thermal conductivity is zero. temperature increase DT.
g = coefficient of cubical expansion of the substance
7. By Stefan's law, E µ T4
Here, temperature becomes double. Then V ' = V (1 + gDT )

\ E will increase 24 or 16 times. M


Initial density = r = and final density
1 V
8. From Wien's displacement law, lm µ
T M M
M and
= r' = \ V= V'=
lm for blue is minimum. So, its temperature will be V' r r'
maximum.
\
M M
9. The range of clinical thermometer is usually from 95° F to = (1 + gDT )
110° F and the boiling point of water is 212°F. So on r' r
sterilization by boiling, the capillary of thermometer will r
burst due to thermal expansion of mercury in the capillary. Þ r' = = r(1 + gDT )-1
1 + gDT
10. It is because of the anomalous expansion of water. Water
has a maximum density at 4°C. Therefore, water expands \ r ' = r(1 - gDT ) \ r'; r
whether it is heated above 4°C or cooled below 4°C. \ Density decreases on heating
3. When the volume of the gas is kept constant, heat is
11. Since a for copper is more than a for steel, hence on
required only to increase the temperature of the gas. No
heating, the bi-metallic strip will bend in such a way that
heat is used to expand the gas. But when the heat is given
the copper strip remains on outer or convex side. to the gas at constant pressure, some heat is used in the
Ther mal Proper ties of Matter 297

expansion of the gas in addition to the heat used in 11. Since they are in series, the rate of flow of heat energy
increasing its temperature. So, more heat is required when is the same. But the sum of the difference in temperatures
one gram mole of a gas is heated to increase its temperature in the difference across their free ends.
by 1° at constant pressure. So, CP > CV. q
4. In front of the fire, heat is received only by the process
q1 K1 K1 q2
of radiation, but at a point above the fire, heat is received
both by convection and radiation. l1 l2
5. By Wien's displacement law. The wavelength of the \(q1 - q) + (q - q 2 ) = (q1 - q 2 )
radiation emitted is inversely proportional to the absolute Q l Q l Q (l1 + l2 )
temperature of the source. The infrared radiation coming i.e., . 1 + . 2 = .
t K1 A t K 2 A t K eq . A
from the sun will be transmitted through the glass windows
of the car as it is of shorter wavelength but the infrared l1 l l +l
Þ + 2 = 1 2
radiation emitted by the bodies inside the car will be of K1 K 2 K eq
longer wavelength, so, they will not be able to come out
l1 + l2
through the window & remains trapped inside increasing \ K eq = .
the temperature inside the car. Since, Sun is at a very high æ l1 l2 ö
temperature than the objects inside the car, so the çè K + K ÷ø
1 2
wavelength of infrared radiations coming from the sun will DV
be very less than that emitted by the objects inside the 12. Stress = K × strain = K = K (3a) Dt
V
car.
6. Since, in a cloudy night, heat radiated by the earth's surface = 140 × 109 × 3 × 1.7 × 10–5 × 20
is radiated back by the clouds on the earth. So, it remains = 1.428 × 108 N/m2
hotter. This is about 103 times atmospheric pressure.
7. Because, after a certain time, the rate of heat generation 2 2
becomes equal to the rate of loss of heat by radiation. This æ L DL ö æ Lö 1
13. From Question, x = çè + ÷ - çè ÷ø » 2 L DL
is called thermal equilibrium. So, the temperature remains 2 2 ø 2 2
constant.
8. Temperature at the core of the earth is high. The molten L
DL = aL Dt \x » 2a D t
magma rises up passing the heat to higher layers and 2
thereby the heavy higher layers flow into the core and get » 0.11 m ® 11 cm
heated up. Thus a current of molten liquid is made and Long Answer Questions
the temperature is maintained at a moderate level.
9. A cooking utensil should be of suitable material which 1. Consider a solid cube of side L, surface area of the face
possesses small value of specific heat (S), large value S and volume V.
of thermal conductivity (K) and low coefficient of \ S = L2 and V = L3
expansion (a). Increase in length = DL = a LDT
10. For a non-black body at temperature T surrounded by an Increase in surface area = DS = bSDT
enclosure T0 (where T > T0), Increase in volume = DV = gVDT
We have E = e s (T4 – T04) New surface area of the cube
where e is a constant, and its value depends upon the
= S + DS = (L + DL)2
physical nature of the emitting surface.
\ E = e s (T2 + T02) (T2 – T02)
= e s (T2 + T02) (T + T0) (T – T0)
= e s (T – T0) (T + T0) (T2 + T02)
= e s (T – T0) (T 3 + T T02 + T0 T 2 + T03)
If the temperature difference (T – T0) between the body L
and the surrounding is very small, then T = T0 L
\ T3 = T03, T2 T0 = T03, T T02 = T03
\ E = e s (T – T0) (4T03) = K (T – T0). L

where K = e s T03 is another constant or E µ (T - T0 ). Þ S + bSDT = (L + aLDi)2


Thus the rate at which a body loses heat is directly Þ S (1+ bDT) = L2 (1 + aDT)2
proportional to the temperature difference between the Þ 1 + bDT = 1 + 2aDT + a2DT2 [Q S = L2]
body and its surrounding provided the temperature Þ 1 + bDT = 1 + 2aDT
difference is not very large. This statement is called [a2DT2 is neglected as it is very small]
Newton’s law of cooling.
298 Physics
Þ bDT = 2aDT Integrating, loge (T2 – T1) = – Kt + c
Þ b = 2a .... (i) \ T2 = T1 + c¢ e–Kt where c¢ = ec
New volume = V + DV = (L + DL)3 The above equation enables us to calculate the time of
Þ V + Vg DT = (L + aLDT)3 = L3 (1 + aDT)3 cooling of a body through a particular range of temperature.
Þ V (1 + g DT) = L3 (1 + a DT)3 4. A perfectly black body is that which absorbs completely
Þ 1 + g DT = 1 + 3 a DT + 3 a2DT2 the radiations of all wavelengths incident on it.
+ a3DT3 [Q V = L3] As a perfectly black body neither reflects nor transmits any
radiation, therefore the absorbtance or absorbing power of
Þ g DT = 3a DT [a2 DT2 and a3 DT3 are neglected
a perfectly black body is unity.
as they are very small]
We know that the colour of an opaque body is the colour
Þ g = 3a ..... (ii)
(i.e., wavelength) of radiation reflected by it. As a black
b g body reflects no wavelength, it appears black whatever be
\ From (i) and (ii), a = = the colour of radiation incident on it.
2 3
2. Let, DT = Temperature difference between two opposite When a perfectly black body is heated to a suitable high
faces of the slab. temperature, it emits radiations of all possible wavelengths.
A = Area of cross-section of the face, The radiations given out by a perfectly black body are
Dx = distance between the two faces called plain body radiations or full radiations or total
DQ = amount of heat conducted through the slab in time radiations.
Dt.
y
Dx
1664 K
T + DT
T
E1 0K
A 145
0K
125
DQ 0K
110

DQ x
\ Rate of conduction of heat = l
Dt
DQ 1 A perfectly black body cannot be realised in practice. Fery
From experiment, µ A µ DT µ designed a perfectly black body which is most commonly
Dt Dx
used. It consists of a hollow double walled metallic sphere
DQ DT DQ DT having a narrow opening O on the side and a conical
\ µA Þ =KA
Dt Dx Dt Dx projection P inside just opposite to it. The inside of the
K = constant called coefficient of thermal conductivity of sphere is coated with lamp black. Any radiation entering
the solid. the sphere through the opening O suffers multiple
DQ / Dt reflections at its inner walls and about 97% of it is absorbed
\ K= by lamp black at each reflection. Therefore, after a few
A(DT / Dx)
reflections, almost entire radiation is absorbed. The
3. According to Newton’s law of cooling, Rate of loss of projection helps in avoiding any direct reflection which
heat a difference of temperature even otherwise is hardly possible because of the small size
dQ of the opening O. When this body is placed in a bath at
\ - = K (T2 - T1 )
dt fixed temperature, the heat radiations come out of the hole.
when K is the constant of proportionality. The opening thus acts as a black body radiator. It should
Suppose a body of mass m and specific be remembered that only the opening (and not the walls)
heat capacity S is at temperature T 2 . If the acts as a black body radiator.
temperature falls by a small amount dT2 in time dt, then the (i) At a given temperature of black body, the energy is
amount of heat lost is dQ = ms dT2 not distributed uniformly amongst all wavelength.
(ii) The energy emitted is maximum corresponding to a
dT2
\ - ms = K (T2 - T1 ) certain wavelength to l and it falls on either side
dt of it.
dT2 K K (iii) With rise in temperature of black body total energy
Þ =- dt = – Kdt where K = emitted increases rapidly for any given wavelength.
T2 - T1 ms ms
Ther mal Proper ties of Matter 299

(iv) The wavelength corresponding to which energy b 2.898≥10,3


emitted is maximum is shifted towards shorter lm = =
T 1500
wavelength side, i.e., l). decreases with rise in
temperature. = 1.932 × 10–6 m = 19320 Å
(v) The area enclosed by each curve with x· axis E2 T24
increases with rise in temperature of the black body. 5. From Stefan’s law, =
E1 T14
Stefan's law. The amount of energy radiated per second
from unit area of a body is proportional to the fourth power
(127 ∗ 273) 4 (400) 4 256
of its absolute temperature. = < <
\ Energy radiated = eAs T4 per second, where e is a (27 ∗ 273) 4
(300) 4 81
characteristic of the body and is called emissivity of the 6. Given: E1 = 1400 W/m2, r1 = 1A.U, E2 = ?,
surface (Ranges from 0 to 1 and is a surface parameter.)
r2 = 5.2 A.U
According to Wien's law, the product of the wavelength
corresponding to maximum intense radiation and the E2 r12 r2
absolute temperature is a constant, i.e., lmT = constant. = 2 [ E2 = 1 ´ E1
E1 r2 r22
Numerical Questions
1. Given: Dx = 4 mm = 4 × 10–3 m, æ 1 ö2
= çç ÷÷÷ ≥1400 = 51.77 W/m2
DQ èç 5.2 ø
DT = 32°C, = 200 k cal/h
Dt
TC TF - 32
DQ 200≥1000 ≥ 4.2
7. We know, = .
= J / s = 233.33 J/s 100 180
Dt 60≥ 60 Here TF = 3TC
A = 5cm2 = 5 × 10– 4 m2, K = ?
TC 3T - 32
DQ æ ΧT ö÷ [ = C Þ 9TC = 5(3TC – 32)
= KA ççç ÷ 100 180
Dt è Χx ø÷
Þ 15TC – 9TC = 5 × 32 Þ 6TC = 160
,3
DQ / Dt 233.33≥ 4≥10 160
Þ K= = Þ TC = = 26.6°C
A(DT / Dx) 5≥10,4 ≥32 6
= 58.33 W/m/°C 8. Here, TF = TK = x
2. According to question, Dx = 20 mm = 20 × 10–3 m.
TF - 32 TK - 273.16
DQ =
= 6 × 105 cal min m2 180 100
Dt
x - 32 x - 273.16
0.2≥4.2 Þ =
K = 0.2 cal s cm K = J/s/m/K, DT = ? 9 5
10,2 Þ 5x – 160 = 9x – 273.16 × 9
DT (DQ / Dt ) Þ 9x – 5x = 273.16 × 9 – 160
( DQ / Dt )Dx
= Þ DT =
Dx KA KA 273.16≥ 9 ,160
Þ x= = 574.6°C
4
5 -3 -2
Þ DT = 6 ´ 10 ´ 4.2 ´ 20 ´ 10 ´ 10 = 10°C 9. As we know l2 < l1 (1 ∗ Χt )
60 ´ 0.2 ´ 4.2
= 75 (1 + 20 × 10–6 × 27) = 75.0405 cm.
3. Given: E = 1 watt/cm2 = 104 W/m2 T = ?
10. Given: S0 = 50 × 10 = 500 cm2,
s = 5.67 × 10–5 erg/s cm2 K4 DS = 1.9 cm2, DT = 100°C , a = ?
= 5.67 × 10–8 J/s m2 K4
From Stefan’s law, E = sT4 DS 1.9 α
b= = = 3.8 × 10–5/°C and  <
S DT 500 ´ 100 2
E 10 4
Þ T = 3.8≥10,5
4
= = 1.764 × 1011
s 5.67 ≥10,8 = = 1.9 × 10–5/°C
2
T = (1.764 × 1011)1/4 = 6.48 × 102 K
l l
4. Given: T = 1227°C = (1227 + 273)K 11. Time period, T = 2p Þ 2 = 2p 20
g g
= 1500 K, lm = ? b = 0.2898 cm K
= 2.898 × 10–3 mK l40
and T ' = 2p
From wein’s displacement law, g
300 Physics

T' l40 r Pv P(1/ r)


[ = = [1 + a(40 – 20)]1/2 Cp – Cv = = = 1
2 l20 J TJ TJ

é 36 ù
1/ 2 1.013≥106
= ê1 + ´ 10-6 ´ 20ú =
ë 3 û 1234 ≥10,3 ≥ 273≥ 4.2 ≥107
1 Cp – Cv = 0.0715
= 1 + × 12 × 20 × 10–6 = 1 + 0.000120
2 1.51Cv – Cv = 0.0715 Þ 0.51 Cv = 0.0715
T ' = 2 (1 + 0.00012) = 2 + 2 × 0.00012 0.0715
Þ Cv =
[ Loss in 2 seconds = 2 + 2 × 0.00012 – 2 0.51
= 2 × 0.00012 s. Þ Cv = 0.1404 cal/g/K
2 Þ Cp = 1.51 Cv = 1.51 × 0.1404 = 0.2119 cal/g/K
[ Loss per day = × 0.00012 × (24 × 60 × 60)
2
HOTS/EXEMPLAR QUESTIONS
= 10.368s
Very Short Answer Questions
12. Heat given out by water = Q1
1. Watermelon having higher specific heat will enhance its
= mcDT = 1 × 200(25 – 10) = 3000 cal. temperature slowly compared to the sandwiches which has
Let m be the mass of ice required. Heat taken in by ice = Q2 lower specific heat.
= (mcDT) + mL + (mcDT) 2. Cold is the lack of thermal energy. Cold is not a thing in
itself, but the result of reduced thermal energy.
Q2 = 0.5 × m × 14 + m × 80 + 1 × m × 10 = 97 m
3. A substance that heats up quickly has a low specific heat
Q Q2 = Q1 [ 97 m = 3000 Þ m = 31 g capacity. Higher the specific heat capacity of a substance,
lower is the increase in its temperature.
13. Given: m1 = 100 g, W = 10 g, m2 = 240 g;
4. Diathermic
T1 = 20°C, T = 60°C, T2 = Temp. of furnace = ? 5. Due to difference in conductivity, metals having high
conductivity compared to wood. On touch with a finger,
Q Heat gained by water and calorimeter
heat from the surrounding flows faster to the finger from
= Heat lost by iron metals and so one feels the heat. Similarly, when one touches
a cold metal the heat from the finger flows away to the
[ (m2 + W) (T – T1) = m1c (T2 – T) surroundings faster.
470(T2 , 60) Short Answer Questions
Þ (240 + 10) (60 – 20) = 100 ×
4.2≥103
1. Ice is less dense than water due to its crystalline structure.
250≥ 40≥ 4.2≥10 3 Ice has a crystalline structure with open structured crystals.
Þ T2 – 60 = = 893.6 Water molecule in this structure occupy a greater volume
100≥ 470
compared to the volume occupied in the liquid form for a
Þ T2 = 893.6 + 60 = 953.6°C given mass. Hence density of ice is less than water.
14. Given: Cv = 0.162 cal/g K and r = 0.001293 g/cm3 , Cp = ? 2. Internal energy of a substance is the sum of kinetic and
potential energies of its submicroscopic particles. Iceberg
R/n r PV P (1/ r) has greater amount of internal energy as the number of
Cp – Cv = = = = molecules / atoms in an iceberg is very-very large compared
J J TJ TJ
to the number of molecules in a coffee cup. If they have equal
P/θ number of molecules, internal energy of coffee cup will be
Cp = Cv +
TJ more, because its temperature is greater than the iceberg.
3. Energy radiated by roads on land is partcialy replenished
1.013≥106 by heat conducted from the warmer ground below the
= 0.162 +
0.001293≥ 273≥ 4.2≥10 7 pavement. But there's an absence of thermal contact
between the road surfaces of bridges and the ground, so
= 0.162 + 0.068 = 0.230 cal/g/K
they receive very little if any replenishing energy conducted
15. Here, Cp = 0.2119 cal/g K and Cv = 0.1404 cal/g K from the ground. This is why road surfaces on bridges get
colder than roads on land, which increases the chance of
Cp ice formation. Understanding heat transfer can make you a
= 1.51 Þ Cp = 1.51 Cv
Cv safer driver!
Ther mal Proper ties of Matter 301

4. Refer to the P.T diagram of water and doubled headed arrow. New length of small element of length dx0
Increasing pressure at 0°C and 1 atm takes ice into liquid dx = dx o (1 + aq )
state and decreasing pressure in liquid state at 0°C and 1
atm takes water to ice state. é x ù
= dx o + dx o a ê q1 + L ( q2 - q1 )ú
P ë o û
(atm)
Now ò dx o = L o and ò dx = L : new length
Liquid
218 dx

solid gas
q1 q2
1 x
0.006 Integrating
( q2 - q1 ) a
220 0.01 374 T(K) \ L = Lo + Lo aq1 +
Lo ò x dx o
When crushed ice is squeezed, some of it melts, filling up o L
gap between ice flakes. Upon releasing pressure, this water æ 1 ö 1
L o ç1 + a (q 2 + q1 )÷ as ò x dx = L20
freezes binding all ice flakes making the ball more stable. è 2 ø 2
0
3. Stress = K × strain
Long Answer Questions
DV
1. (a) Here Lb = 50cm, Ls = 50cm =K
V
DT = 250 – 40= 210º C
= K (3a ) Dt
-5 o -1 -5 o -1 = 140 × 109 × 3 × 1.7 × 10–5 × 20
µ b = 2 ´10 C , µ b = 1.2 ´ 10 C .
= 1.428 × 108 N/m2
Total change in length is
This is about 103 times atmospheric pressure.
DL = DLb + DLs
CHAPTER TEST
= Lb µ b DT + Ls µ s DT
1. From the definition heat required for external work when
= L (µ b + µ s )DT 1 gm mole of a gas is heated by 1°C at constant pressure =
= 50 (2×10–5+1.2×10–5)×210 = 50×3.2×10–5×210 Cp – Cv = R = 2cal
= 0.336 cm 2. Rise in temperature will be same for both the balls.
(b) Here, L = 16mm = 16 × 10–3 m 3. Plate will cool fastest as area exposed is greatest among
q1 – q2 = 32ºC sphere, cube and thin circular plate.
Q = 400 × 103 × 4.2J
4. Let C be the correct temperature on the celcius scale then
t = 2 × 60 × 60sec, A = 5 × 10–4 m2
C - 0 temp. in faulty scale , lower fixed pt.
from formula =
100 upper fixed point , lower fixed pt.
KA (Q1 - Q 2 ) Q=
Q.1
Q= 59 - 5 54 3
L
, A ( Q1 - Q 2 ) t Þ
C
= = =
100 95 - 5 90 5
400 ´ 10 3 ´ 4.2 ´ 16 ´ 10 -3 300
K= = 233.3 W m–1 0C–1. Þ C< < 60↓
5 ´ 10 - 4 ´ 32º C ´ 60 ´ 60 5
1/ 4
é 5.67 ´10 4 ù 1/ 4 5. When a body is heated, the distance between any of its two
=ê ú = é1012 ù = 103 K points increases. Hence the diameters AB and CD of the
-8 ë û
êë 5.67 ´10 úû two holes will increase.
(c) The reason is the anomalous expansion of water. The DL 1
maximum density of water occurs at 4ºC. So the water 6. From question, = 1% = , a = 0.00002/°C, D T = ?
L 100
expands whether it is heated above 4ºC or cooled
below 4ºC. DL 1
DL = a LDT Þ DT = .
2. Temperature q at a distance x from one and (that at q1) is L a
x 1 1 105
given by q = q1 + ( q2 - q1 ) : linear temperature gradient. Þ DT = ´ = = 500°C
Lo 100 0.00002 2 ´ 102
302 Physics

R / n r PV P(1/ θ) P = 1.013 × 106 dyne/cm2, T = 6273 K


7. As we know, Cp – Cv = < < <
J J TJ TJ 1.013 ´ 106
[ C p - Cv =
1.293 ´ 10-3 ´ 273 ´ 4.2 ´ 107
Here, r = 1.293 g/l = 1.293 × 10– 3 g/cm3 = 0.068 cal /g/K

8. Conduction Convection Radiation


1. Material medium is required Material medium is required No material medium is required
2. It occurs in solids without It occurs in fluids by actual It heats the matter without heating the
actual motion of matter motion of matter intervening medium.
3. It does not obey the laws of It does not obey the laws of It obeys the laws of reflection and refraction.
reflection and refraction reflection and refraction

9. Let the thermal conductivity of brass be K ice to liquid water. The heat Q2 needed to melt all
the ice is given by (Q = Lm). Here L is the heat of
\ Thermal conductivity of copper = 4K fusion LF, with the value given, we find.
Copper Brass Q2 = LFm = (333 kJ/kg)(0.720 kg) = 239.8 kJ.
Step 3 Now we have liquid water at 0°C. The next key idea
is that the energy transferred to the liquid water as
heat now can only increase the temperature of the
0°C (4 K) O (K) 100°C liquid water. The heat Q3 needed to increase the
Let the area of each rod = A ; temperature of the water from the initial value
Ti = 0°C to the final value Tf = 15°C is given by (with
Let length of each rod = x the specific heat of liquid water Cliq):
Let q be the temperature of the junction of the two rods in Q3 = Cliqm (Tf –Ti) = (4190 J/kg.K)(0.720 kg)(15°C –
0°C) = 45,252 J = 45.25 kJ.
equilibrium. Rate of flow of heat energy through brass =
The total required heat Qtot is the sum of the amunts
rate of flow of heat energy through copper.
required in the three steps:
Qtot = Q1 + Q2 + Q3 = 15.98 kJ + 239.kJ + 45.25kJ
K . A.(100 - q) 4 K . A.(q - 0)
= = 300 kJ.
x x
Note that the heat required to melt the ice is much
or (100 – q) = 4 (q – 0) = 4q greater than the heat required to raise the temperature
of either the ice or the liquid water.
100 (b) From step 1, we know that 15.98 kJ is needed to raise
or, 5q = 100 or q = = 20°C the temperature of the ice to the melting point. The
5
remaining heat Qrem is then 210 kJ – 15.98 kJ, or about
10. (a) The heating process is accomplished in three steps. 194 kJ.
Step1 The key idea here is that the ice cannot melt at a From step 2, we can see that this amount of heat is
temperature below the freezing point-so initially, any insufficient to melt all the ice. Then this key idea
energy transferred to the ice as heat can only becomes important. Because the melting of the ice is
increase the temperature of the ice. The heat Q1 incomplete, we must end up with a mixture of ice and
needed to increase that temperature from the initial liquid ; the temperature of the mixture must be the
value Ti = – 10°C to a final value T f = 0°C (so that freezing point, 0°C.
the ice can then melt) is given by (Q = cmDT). Using We can find the mass m of ice that is melted by the
the specific heat of ice cic gives us available energy Qrem by using .
Q1 =cicem(Tf – Ti) = (2220 J/kg K)(0.720kg) [0°C – Q rem 194 kJ
(–10°C)] = 15,984J = 15.98 kJ. m = L = 333kJ / kg = 0.583kg = 580g
F
Step 2 The next Key Idea is that the temperature cannot
increase from 0°C until all the ice melts-so any energy Thus, the mass of the ice that remains is 720g – 580 g,
transferred to the ice as heat now can only change or 140g , and we have 580 g water and 140g ice, at 0°C.

¿¿¿
12 Thermodynamics
C ha p t e r

Thermodynamics is the branch of physics that deals with the concepts of heat and temperature and the inter-conversion of heat and
other forms of energy. It is a macroscopic science, deals with bulk systems but not with the molecular constitution of matter.
THERMAL EQUILIBRIUM
Two bodies A and B are said to be in thermal equilibrium with each other if there is no transfer of heat between them i.e., they are
said to be at same temperature. The energy that flows between two objects or systems due to a temperature difference between them
is called heat.
ZEROTH LAW OF THERMODYNAMICS
When two thermodynamic systems A and B are separately in thermal equilibrium with a third thermodynamic system C, then the
systems A and B are in the thermal equilibrium with each other. i.e., If TA = TC and TB = TC then TA = TB.
HEAT, INTERNAL ENERGY AND WORK
Heat
It is energy in transit between two objects or systems due to temperature difference between them. It exists in the form of translational,
rotational and vibrational motion of molecules of a substance. It depends on processes.
For constant pressure process Q = nCP DT
Constant volume process Q = nCv DT and for any other process Q = nC DT,
where C is called molar specific heat for that process.
Internal Energy
Internal energy of a system is the energy possessed by the system due to molecular motion and molecular configuration. The energy
due to molecular motion is called internal kinetic energy (Uk) and that due to molecular configuration is called internal potential energy
(Up). dU = dUk + dUp
If there are no intermolecular forces, then dUp = 0
\ dU = dUk = mcv dT
where m is mass of system, cv specific heat at constant volume and dT is the change in temperature. If Cv is molar heat capacity (= Mcv,
M being molecular weight), then for µ-moles of ideal gas
m
dU = µC vdT = Cv dT
M
It is obvious that internal energy in the absence of inter-molecular forces is the simply the function of temperature and state only, and
it is independent of path followed.
dU = Uf – Ui
where Ui and Uf are internal energies in initial and final states.
Work
It is the energy transfer brounght about by other means such as moving the piston of a cylinder containing the gas by raising or
lowering some weight connected to it.
Heat and work in the thermodynamics are not state variables. They are modes of energy transfer to a system resulting in its internal
energy which is a thermodynamic state variable. Its value depends only on the state of system not on the path taken to arrive at that
state.
FIRST LAW OF THERMODYNAMICS
When some quantity of heat (dQ) is given to a system capable of doing external work, then the quantity of heat absorbed by the
system is equal to the sum of increase in its internal energy (dU) due to increase in temperature and the external work (dW) done
by it in expansion.
304 Physics
i.e., dQ = dU + dW
Since force is pressure times area, and area times displacement is volume, work done by the system against a constant pressure P is
dW = PdV
\ dQ = dU + PdV
Limitations of first law of thermodynamics:
(i) The direction of change cannot be found.
(ii) The extent of change cannot be calculated.
(iii) The information about the source of heat cannot be obtained.

Illustration 1
At 0°C and normal atmospheric pressure, the volume of 1g of water increases from 1cm3 to 1.091 cm3 on freezing.
What will be the change in its internal energy? Normal atmospheric pressure is 1.013×10 5 N/m2 and latent heat of fusion of
ice = 80 cal/ g.
Sol.
Heat released by water on freezing
Q = – mL
= – 1× 80 = – 80 cal = – 336 J
During freezing water expands against the atmospheric pressure. Hence work done in the process
W = PDV = (1.013 ´ 105 ) ´ (1.091 - 1) ´ 10-6
= 0.092 J
By first law of thermodynamics
Q = DU + W
or –336 = DU + 0.092
\ DU = –336.092 J
Illustration 2
1.0 m3 of water is converted into 1671m3 of steam at atmospheric pressure and 100°C temperature. The latent heat of
vapourisation of water is 2.3×106 J/ kg. If 2.0 kg of water be converted into steam at atmospheric pressure and 100°C
temperature, then how much will be the increase in its internal energy? Density of water is 1.0×103 kg/m3, atmospheric
pressure = 1.01×105 N/m2.
Sol. Heat given to water to change into steam
Q = ML = 2.0 ´ 2.3 ´ 106 = 4.6 × 106J
Mass 2.0
Volume of water V = density = 103 = 2.0×10–3 m3
Volume of steam formed will be =2.0×10–3×1671= 3342 ×10–3m3
The change in volume in the process DV = V = 3342 ´10-3 - 2.0 ´ 10-3 = 3340 ×10–3m3
The work done against the atmospheric pressure
W = PDV
= (1.01×105)×(3340×10–3) = 0.337 ×106J
By first law of thermodynamics
Q = DU + W
\ DU = Q – W
= 4.6×106 – 0.337×106 = 4.263×106 J`
Here positive value of DU indicates that internal energy in the process increases.
SPECIFIC HEAT CAPACITY
It is the amount of heat required to raise the temperature of unit mass of substance through 1 degree.
It may be expressed in cal/gºC, kcal/kgºC, BTU/1bºf, J/kgK.
Q
Specific heat capacity, c =
m ´ Dt
For gases : During the isothermal process
Dt = 0 Þ c = ¥ (infinite)
During the adiabatic process : Q = 0 Þ c = 0 (zero)
Ther modynamics 305

Specific heat of gas : The specific heat of a gas varies from zero to infinity. It may have any positive or negative value.
Following of two specific heats are more significant.
(a) Molar specific heat at constant volume (CV) :
It is the amount of heat required to raise the temperature of 1 mole of a gas through 1K or 1°C at constant volume.
1 æ dQ ö
CV = ç ÷
n è dT ø
(b) Molar specific heat at constant pressure (CP) :
It is the amount of heat required to raise the temperature of 1 mole of a gas through 1K or 1°C at constant pressure.
1 æ dQ ö
CP = ç ÷
n è dT ø
Mayers relation : CP – CV = R
where R is a gas constant. (R = 8.314 Jmol–1K–1)
Thermodynamic State Variables
Thermodynamic state variables describe equilibrium states of systems. The connection between the state variables is called the
equation of state. For an ideal gas, the equation of state is ideal gas equation PV = nRT
(i) Extensive variables indicate the ‘size’ of the system. i.e. volume.
(ii) Intensive variables are the variables that do not indicate the size of the system such as pressure and temperature.
THERMODYNAMIC PROCESSES
Thermodynamic process is said to take place if some change occurs in the state of a thermodynamic system, i.e. the thermodynamic
variables of the system
Sign conventions for the study of thermodynamic processes:
The following sign conventions are adopted in the study of thermodynamic process :
(i) Heat gained by a system is taken as positive while that lost by a system is taken as negative.
(ii) The work done by a system is taken as positive while that done on the system is taken as negative.
(iii) Increase in the internal energy of a system is taken as positive. Decrease in the internal energy of system is taken as
negative.
DIFFERENT TYPES OF THERMODYNAMIC PROCESSES
Quasi-static Process
It is infinitely slow. So its variables (P, V, T) remains in thermal and mechanical equilibrium with its surroundings.
Isothermal Process
It is a thermodynamic process in which the pressure and volume of system change but temperature remains constant.
In an isothermal process, the exchange of heat between the system and the surroundings is allowed. In other words, as isothermal
process is carried out by either supplying heat to the substance or by extracting heat from it.
A process has to be extremely slow to be isothermal.
Equation of state : PV = constant = µRT (since T is constant)
Consider µ moles of an ideal gas, enclosed in a cylinder, at absolute temperature T, fitted with a frictionless piston.
Suppose that gas undergoes an isothermal expansion from the initial state (P1, V1) to the final state (P2, V2).
V2 µRT
\ Work done W = òV1 V
dV

é V2 ù
or W = 2.303µRT log10 ê V ú
ë 1û
é P1 ù
or W = 2.303µRT log10 ê P ú [Q P1 V1 = P2 V2.]
ë 2û
Form of first law of thermodynamics in isothermal process : Since there is no change in temperature and internal energy of the
system, so, dU = 0.
é V2 ù
dQ = 2.303 µRT log10 ê V ú
ë 1û
Whole of the heat energy supplied to the system is utilized by the system in doing external work. There is no change in
the internal energy of the system.
306 Physics
Slope of the isothermal curve : For an isothermal process, PV = constant
P

Differentiating, PdV + VdP = 0 Isotherm


Pi
PV = constant
or VdP = – PdV
dP P Hyperbola
or =- Pf
dV V
V
æ dP ö P Vi Vf
\ Slope of isothermal, çè dV ÷ø = - V Isothermal expansion of an
iso
ideal gas
Examples of Isothermal Process :
(1) The temperature of a substance remains constant during melting. So, the melting process is an isothermal process.
(2) When a substance boils, its temperature remains constant. So, boiling is an isothermal process.
(3) An ideal gas enclosed in a conducting cylinder fitted with a conducting piston.
Let the gas be allowed to expand very-very slowly. This shall cause a very slow cooling of the gas. But heat will be
conducted into the cylinder from the surrounding. Thus, the temperature of the gas remains constant.
If the gas is compressed very-very slowly, heat will be produced. But this heat will be conducted to the surroundings. So,
the temperature of the gas shall remain constant.
(4) If sudden changes are executed in a vessel of infinite conductivity then they will be isothermal.
Adiabatic Process
It is that thermodynamic process in which pressure, volume and temperature of the system change but there is no exchange
of heat between the system and the surroundings.
A process has to be sudden and quick to be adiabatic.
Equation of state : PV = µRT
Equation for adiabatic process PVg = constant
Let initial state of system is (P1, V1, T1) and after adiabatic change final state of system is (P2, V2, T2) then we can write,
P1 V1g = P2 V2g = K (here K is const.)
V2 1
W = [ P1V1 - P2V 2 ]
Work done W =
òV1 PdV or
( g - 1)

mR
W = (T1 - T2 ) (Q PV = µRT)
( g - 1)
Form of first law of thermodynamics in adabatic process :
dU = – dW
The work done by an ideal gas during adiabatic expansion (or compression) is proportional to the fall (or rise) in the temperature of the
gas.
If the gas expands adiabatically, work is done by the gas.
So, Wadia is positive.
\ T1 > T2. So, the gas cools during adiabatic expansion.
If the gas is compressed adiabatically, work is done on the gas. So, Wadia is negative.
\ T1 < T2. So, the gas heats up during adiabatic compression.
Slope of the adiabatic curve :
For an adiabatic process, PVg = constant
Differentiating, Pg Vg – 1 dV + Vg dP = 0
1 Monatomic P
P Diatomic
or Vg dP = – g PVg – 1 dV 2
3 Polyatomic
g -1
dP g PV gP
or =- =- 3
= 1.33
Isothermal
dV Vg V 2 = 1.4
1
æ dP ö gP = 1.67 Adiabatic
Slope of adiabatic curve, çè dV ÷ø =-
adia V V V
Adiabatic expansion of mono, Adiabatic and isothermal
æ dP ö æ Pö æ dP ö dia and polyatomic gases
expansion of an ideal gas
ç ÷ = g ç- ÷ = g ç ÷
è dV øadia è Vø è dV øiso
Ther modynamics 307

Slope of adiabatic changes


=g
Slope of isothermal changes
But g is always greater than one.
So, slope of adiabatic is greater than the slope of isothermal.
Thus, we conclude that an adiabatic curve is steeper than an isothermal curve.
Examples of Adiabatic Process :
(1) A gas enclosed in a thermally insulated cylinder fitted with a non-conducting piston. If the gas is compressed suddenly by
moving the piston downwards, some heat is produced. This heat cannot escape the cylinder. Consequently, there will be an
increase in the temperature of the gas.
(2) If a gas is suddenly expanded by moving the piston outwards, there will be a decrease in the temperature of the gas.
(3) Bursting of a cycle tube.
(4) Propagation of sound waves in a gas.
(5) Expansion of hot gases in internal combustion engine. In diesel engines burning of diesel without spark plug is done due to
adiabatic compression of diesel vapour and air mixture.
(6) Expansion of steam in the cylinder of a steam engine.
Adiabatic Relation Between V and T for Ideal Gas
TVg – 1 = constant
If V1, T1 be the initial and V2, T2 be the final volumes and temperatures respectively of the gas for an adiabatic change, then
T1V1g – 1 = T2V2g – 1
Adiabatic Relation Between P and T for Ideal Gas
Tg P1 – g = constant
If P1, T1 be the original and P2, T2 be the final pressures and temperatures respectively of gas for an adiabatic change, then
T1g P11 – g = T2g P21 – g
Isochoric or Isometric Process
It is a thermodynamic process that takes place at constant volume of the system, but pressure and temperature varies for change in
state of the system.
Equation of state : P = const. × T
Vf
This is the process in which volume remains constant i.e. DV = 0 therefore, work done DW = ò PdV = 0
Vi
Form of first law of thermodynamics in isochoric process : dQ = dU
Whole of heat supplied is utilized for change in internal energy of the system. dQ = dU = µ Cv dT
Isobaric Process
It is a thermodynamic process that takes place at constant pressure, but volume and temperature varies for change in state of the
system.
Equation of state : V = const. × T
This is the process in which pressure remains constant, i.e.
DP = 0
Vf
\ Work done W = ò PdV = P (V f - Vi )
Vi
Form of first law of thermodynamics in isobaric process :
dQ = dU + P (Vf – Vi)
or µ Cp dT = µ Cv dT + P(Vf – Vi)
Heat supplied to the system is utilized for :
(a) Increasing internal energy and
(b) Work done against the surrounding atmosphere.
Cyclic Process
In a cyclic process, the system returns to its initial state. Since internal energy is a state variable, Du = 0 for a cyclic process.
From first law of thermodynamics, DQ = DU + DW \ DQ = DW (Q DU = 0)
Illustration 3 :
Some quantity of air at 27°C and atmospheric pressure is suddenly compressed to half its original volume. Find the final
pressure and temperature. Given g for air = 1.42.
308 Physics

1
Sol. Given: T1 = 27 + 273 = 300K, P1 = 1 atm, V2 = V , P = ?, T2 = ?, g = 1.42
2 1 2
The process is adiabatic.
γ
æV ö
[ g
P2 V2 = P1V1 g [ P2 = P1 ç 1 ÷ = 1 ´ (2)1.42 = 2.675 atm
èV ø2
g -1
æ V1 ö
Q T2V2 g–1 = T1V1 g–1 [ T2 = T1 ç ÷ = 300(2)1.42 -1 = 401.3 K
èV ø2
T2 = 401.3 K = 401.3 – 273 = 128.3°C
Illustration 4 :
One gram mole of a gas at 127°C expands isothermally until its volume is doubled. Find the amount of work done and heat
absorbed.
Sol. Given: T = 273 + 127 = 400K. Let V1 = V and V2 = 2V
V2 2V
Work done = W = 2.303 RT log10 = 2.303 × 8.3 × 400 × log10
V1 V
3
= 2.303 × 8.3 × 400 × 0.3010 = 2.3× 10 J
W 2.3≥103
Amount of heat absorbed H = = = 548 cal.
J 4.2
Illustration 5 :
Two samples of a gas initially at same temperature and pressure are compressed from a volume V to V/2. One sample is
compressed isothermally and the other adiabatically. In which sample is the pressure greater?
Sol. Let initial volume, V1 = V and final volume, V2 = V/2
Initial pressure, P1 = P ; final pressure, P2 = ?
For isothermal compression Adiabatic compression
g g
P1V1 PV æ V1 ö æ V ö
P2V2 = P1V1 or P2 = V = V / 2 = 2P P2' V2g = P1V1g or P2' = P1 ç ÷ = P ç
2 è V2 ø è V / 2 ÷ø
or P2´ = 2gP
Since g > 1 \ 2g > 2 \ P2' > P2
Pressure during adiabatic compression is greater than the pressure during isothermal compression.
HEAT ENGINES
A heat engine is a device which converts continuously heat energy into mechanical energy in a cyclic process.
Heat engine has the following three parts.
(i) Source. It is a heat reservoir at higher temperature T1. It is supposed to have infinite Hot reservoir
thermal capacity so that any amount of heat can be drawn from it without changing its or Source
temperature.
(ii) Sink. It is a heat reservoir at a lower temperature T2. It has also infinite thermal Q1= Heat absorbed
capacity so that any amount of heat can be added to it without changing its from source
temperature.
(iii) Working substance. Working substance is any material (solid, liquid or gas) which Heat Work done
performs mechanical work when heat is supplied to it. engine W= Q1 – Q2
Working. In every cycle of operation, the working substance absorbs a definite amount
o heat Q1 from the source at higher temperature T1, converts a part of this heat energy into
mechanical work W and rejects the remaining heat Q2 to the sink at lower temperature T2. Q1= Heat rejected
The work done W in a cycle is transferred to the environment by some arrangement e.g., the to sink
working substance may be in a cylinder withh a moving piston that transfers mechanical
energy to the wheels of a vehicle via a shaft. Cold reservoir
or Sink
The efficiency of heat engine is given by
Work done by engine (output) W Q1 - Q2 Q
h = = = or h = 1 - 2 .
Heat absorbed from the source (input) Q1 Q1 Q1
Efficiency of a heat engine is always less than unity. Clearly, when Q2 = 0, h = 1 or 100%.
Ther modynamics 309

The efficiency of a steam engine varies from 12 to 16%. The maximum efficiency of a petrol engine is 26% and that of a diesel engine
is 40%.
REFRIGERATORS AND HEAT PUMPS
A refrigerator is the reverse of heat engine. In refrigerator working substance extracts heat Q 2 from sink T2 or low temperature
reservoir, some external work W is done on it and heat Q1 is released to the source T1 or high temperature reservoir.
W

T1 source or high Q1 Q2 T2 sink or low


temperature reservoir temperature reservoir

Heat pump to pump heat into a portion of space is the same as a refrigerator.
Working of a refrigerator
In a refrigerator the working substance (usually, in gaseous form) goes through the following steps :
(a) sudden expansion of the gas from high to low pressure which cools it and converts it into a vapour-liquid mixture,
(b) absorption by the cold fluid of heat from the region to be cooled converting it into vapour,
(c) heating up of the vapour due to external work done on the system, and
(d) release of heat by the vapour to the surroundings, bringing it to the initial state and completing the cycle.
Coefficient of Performance of a refrigerator
Q2 Q2
a= =
W Q1 - Q 2
[QW = Q1 - Q2 ]
Second Law of Thermodynamics
1. Kelvin-Planck statement : It is impossible for an engine working between a cyclic process to extract heat from a reservoir and
convert completely into work. In other words, 100% conversion of heat into work is impossible.
2. Clausius statement : It is impossible for a self-acting machine, unaided by any external agency to transfer heat from a cold to hot
reservoir. In other words heat itself cannot flow from a colder to a hotter body.
Entropy
It is a measure of molecular disorder of a system. Greater is disorder, greater is entropy.
The change in entropy,
Heat absorbed dQ
dS = i.e., dS =
Absolute temperature T
dQ = T dS
This is the mathematical form of second law of thermodynamics.

REVERSIBLE AND IRREVERSIBLE PROCESSES

Reversible Process
Any process which can be made to proceed in the reverse direction by variation in its conditions such that any change occurring
in any part of the direct process is exactly reversed in the corresponding part of reverse process is called a reversible process.
Examples: (i) An infinitesimally slow compression and expansion of an ideal gas at constant temperature.
(ii) The process of gradual compression and extension of an elastic spring is approximately reversible.
(iii) A working substance taken along the complete Carnot’s cycle.
(iv) The process of electrolysis is reversible if the resistance offered by the electrolyte is negligibly small.
A complete reversible process is an idealised concept as it can never be realised because dissipative forces cannot be
completely eliminated.
Irreversible Process
Any process which cannot be retraced in the reverse direction exactly is called an irreversible process. Most of the processes
occurring in the nature are irreversible processes.
Examples: (i) Diffusion of gases.
(ii) Dissolution of salt in water.
310 Physics
(iii) Rusting of iron.
(iv) Sudden expansion or contraction of a gas.
CARNOT ENGINE
Carnot devised an ideal engine which is based on a reversible cycle of four operations in succession : isothermal expansion,
adiabatic expansion, isothermal compression and adiabatic compression.

A Isothermal
Q1 expansion

c o m pre s s io n
T1

Ad ia b a tic

exp a n s io n
Ad ia b a tic
B
P D
T2
Isothermal C
Q2
compression
V1 V V2

Main parts of Carnot's engine are :


Source of heat : It is a hot body of high temperature T1 from which the heat can be drawn. It is a hot body of very large heat capacity
kept at a constant high temperature T1 K. Its upper surface is perfectly conducting so that working substance can take heat.
Working substance : A cylinder whose walls are perfectly non-conducting but base perfectly conducting fitted with non-
conducting piston which can move without any frictional loses. Ideal gas enclosed in these system as a working substance.
Heat sink : A cold body at low temperature T2 K to which the heat can be rejected. It is a body of large heat capacity. Its upper
surface is highly conducting so that working substance can reject heat to it.
Stand : It is made by perfectly insulating material so that when cylinder placed on it working substance can expended or
compressed adiabatically .
Working of Carnot’s engine :
To get maximum work from this type of ideal engine there is a set of reversible processes through them working substance is
taken back to initial condition.
Complete Carnot cycle is divided in four steps.
Processes of Carnot's cycle can be denoted by an indicator diagram as shown. The variation of pressure (P) and volume (V) of
working substance (ideal gas), are plotted. A
P1 Q1
(a) First process : Isothermal expansion A ® B
Initially the cylinder is taken to be in thermal equilibrium with the high T1
temper ature T1 represented by point A P P B
2
(P1, V1, T1) This is initial state of working substance
P4 D T2
Then the piston is allowed to move outward slowly. With the movement of
the piston, the temperature of the gas tends to fall. The process is very slow P3 Q2
so that it is isothermal. Heat from reservoir flows into the gas and temperature C
of the gas remains T1. In this expansion gas receive heat Q1 from source and
V1 V4 V2 V3
gets state B (P2, V2, T1) V
The total heat input Q1 to the gas occurs over the path from A to B and comes from a large reservoir of heat (source) at
temperature T1 and utilized for doing work W1.
Over the path from A to B the heat input to the gas equals the work done against the external pressure.
V2 V2 µRT1 æV ö
Q1 = ò PdV = ò dV = µRT1 In ç 2 ÷
V1 V1 V èV ø 1

æV ö
W1 = µRT1 In ç 2 ÷ (\ W1 = Q1)
è V1 ø
(b) Second process : Adiabatic expansion B ® C
Now the contact of cylinder with source is removed and cylinder is put in contact with a non-conducting stand. The piston is
allowed to move outward the gas now expands adiabatically because no heat can enter in or leave out. The temperature falls to
T2 and gas describes the adiabatic BC to point C (P3, V3, T2) during which more work is done (W2) at the expense of the internal
energy.
gR
Work done over this adiabatic path BC W2 = (T1 - T2 )
g -1
Ther modynamics 311

(c) Third process : Isothermal compression C ® D


The gas cylinder is placed in contact with sink at temperature T2. The piston is moved slowly inward so that heat produced
during compression passes to the sink. The gas is isothermally compressed to point D. (P4, V4, T2)
The heat rejected Q2 to the cold reservoir (sink) at T2 occurs over the path from C to D.
The amount of work done on the gas W3 is equal to the amount of heat rejected to the sink W3 = Q2 and
æV ö æV ö
W3 = µRT2 In ç 4 ÷ or Q2 = µRT2 In ç 4 ÷
è 3ø
V è V3 ø
(d) Fourth process : Adiabatic compression D ® A
The cylinder is removed from the sink and is put in contact with insulating stand and piston moves inward. Heat is not allowed
to go out and it increases the internal energy of the system. Then work done on the gas during adiabatic compression from state
D to initial point A (P1, V1, T1).
No heat exchanges occur over the adiabatic path.
gR
Work done on the system is, W4 = (T1 - T2 )
g -1
This cycle of operations is called a Carnot cycle. In first two steps work is done by engine (W1 and W2 are positive) and in last
two steps work is done on gas (W3 and W4 are negative). The work done in complete cycle W is equal to the area of the closed
part of the P–V cycle.
W = W1 + W2 + W3 + W4
æ V2 ö gR æV ö gR
or W = µRT1 In ç V ÷ + (T1 - T2 ) + µRT2 In ç 4 ÷ + (T2 - T1 )
è 1ø g -1 è 3ø
V g -1

æV ö æV ö
= µRT1 In ç 2 ÷ + µRT2 In ç 4 ÷
è V1 ø è V3 ø
Efficiency of Carnot engine,
æV ö æV ö
µRT1 In ç 2 ÷ + µRT2 In ç 4 ÷
W è V1 ø è V3 ø
h=
Q1 = æV ö
µRT1 In ç 2 ÷
è V1 ø
The points B and C are connected by an adiabatic path as are the points D and A. Hence, using this eq n. and the adiabatic gas
eqn.
T1V2(g – 1) = T2 V3(g – 1) and T1V1(g – 1) = T2 V4(g – 1).
V2 V3
Combination of the above eqns. gives V = V , and,
1 4

T1 - T2 Q1 - Q2
h= =
T1 Q1
Q2 T
or, h = 1- =1- 2 .
Q1 T1

Efficiency of Cyclic Process


When a system is subjected to a cyclic process, heat is supplied during some part of the process, while heat is abstracted, during
other part.
Evidently, the net heat supplied will be the work done by the system. (Q DQ = DW). However, the gross heat supplied will be more
than that of the net heat.
Efficiency (h) of a cycle, is defined as the ratio of the work performed (net heat given) to the gross heat supplied to the system,
per cycle.
work done per cycle
Thus h=
gross heat supplied per cycle
312 Physics
The percentage efficiency of Carnot’s engine:
T1 - T2 Q - Q2
h= ´ 100% or, h = 1 ´ 100%
T1 Q1
It can be shown that the efficiency for the Carnot engine is the best that can be obtained for any heat engine and eq n. gives an
upper limit to the efficiency of any heat engine operating between temperatures T1 and T2.
The efficiency depends upon the temperatures T 1 and T2 which approaches unity only when the temperature of the cold
reservoir approaches absolute zero. A steam engine using steam at 373 K and with the cold reservoir at 273 K has a best possible
efficiency of h @ 1 – (273/373) @ 27%.
The efficiency of a Carnot engine is never 100% because it is 100% only if temperature of sink T2 = 0 which is impossible.
Q2 T2 Q1 Q2
In a Carnot cycle, = or =
Q1 T1 T1 T2
Carnot Theorem
No irreversible engine (I) can have efficiency greater than Carnot reversible engine (R) working between same hot and cold
reservoirs. i.e., hR > hI

Illustration 6 :
Assuming that a domestic refrigerator can be regarded as a reversible engine working between the temperature of melting ice and
that of the atmosphere (17°C), calculate the energy which must be supplied to freeze one 1 kilogram of water already at 0°C. [Latent
heat of fusion = 80 cal/g]
Sol. Here, T2 = 0°C = 273°K, T1 = 17°C = 17 + 273 = 290 K
Heat required to freeze 1 kg of water at 0°C = Q2 = mL = 1× 80000 cal [L = 80cal/g = 80,000 cal/kg]
= 80,000 × 4.2J
[ Coefficient of performance of the refrigerator, a Q 2 = T2
W T1 - T2
Q 2 (T1 - T2 ) 80000 ≥ 4.2(290 , 273)
[ Energy supplied W = = < 2.092 ≥104 J
T2 273

Illustration 7 :
The temperature T1 and T2 of two heat reservoirs in an ideal Carnot engine are 1500°C and 500°C. Which of these increasing
T1 by 100°C or decreasing T2 by 100°C, would result in greater improvement of the efficiency of the engine?
T2 T1 - T2
Sol. Efficiency of engine, h = 1 - =
T1 T1

(1500 ∗ 100) , 500 1100


Increasing T1 by 100°C, efficiency γ1 < < = 59%
(1500 ∗ 100 ∗ 273) 1873

1500 , (500 ,100) 1100


Decreasing T2 by 100°C, efficiency γ2 < < = 67%
(1500 ∗ 273) 1773
[ Efficiency in the 2nd case is better..
Illustration 8 :
A Carnot engine takes 1000 K cal of heat from a source at 627°C and rejects heat to the sink at 27°C. What is its efficiency?
What is the useful work done/cycle by the engine?
Sol. Here, Q1 = 1000 Kcal = 106 cal
T1 = 627 + 273 = 900K, T2 = 27 + 273 = 300K
æ T ö æ 300 ö 2
Efficiency, h = ç1 - 2 ÷ ´ 100 = ç1 - ÷ ´ 100 = 100≥ < 66.67 %
è T1 ø è 900 ø 3

Q2 T2 300 1
Work done/cycle = W = Q1 – Q2 also = = =
Q1 T1 900 3

Q1 106
[ Q2 = = cal.
3 3
Ther modynamics 313
314 Physics

Textbook Exercises
12.1 A geyser heats water flowing at the rate of (d) In a harbour town, the relative humidity is more than in
3.0 liter/min from 27°C to 77°C. If the geyser operates on a desert town. Hence, the climate of a harbour town is
a gas burner, what is the rate of consumption of the fuel, if without extremes of hot and cold.
its heat of combustion is 4.0 × 104 J/g? 12.4 A cylinder with a movable piston contains
Sol. Given, volume of water heated = 3 liter/min. 3 moles of hydrogen at standard temperature and pressure.
Mass of water heated, m = 3000 g/min The walls of the cylinder are made of a heat insulator, and
the piston is insulated by having a pile of sand on it. By
Rise in temperature, ΧT = 77 – 27 = 50°C, Specific heat of
what factor does the pressure of the gas increase, if the
water, c = 4.2 Jg–1°C–1
gas is compressed to half its original volume?
Amount of heat used, ΧQ = mc ΧT Sol. The process is adiabatic, as no heat in exchanged
= 3000 × 4.2 × 50 = 63 × 104 J/min g
Heat of combustion = 4 × 104 J/g; P2 æ V1 ö
[ P2V2g = P1V1g [ =
Rate of combustion of fuel P1 çè V2 ÷ø
Given V2 = 1/2 V1
63≥10 4
= < 15.75 g/m 1.4
4 ≥10 4 P2 æ V1 ö
[ = = 2.64
12.2 What amount of heat must be supplied to P1 çè V1 / 2 ÷ø
2.0 × 10–2 kg of nitrogen at room temperature to raise its 12.5 In changing state the of a gas adiabatically from an
equilibrium state A to another equilibrium state B, an
temperature by 45°C at constant pressure? (Given
amount of work equal to 22.3 J is done on the system. If the
molecular mass of N2 = 28
gas is taken from state A to B via a process in which the net
R = 8.3 J mol–1K–1).
heat absorbed by the system is 9.35 cal, how much is the
Sol. Given, m = mass of gas = 2 × 10–2 kg = 20 g; Rise in
net work done by the system in the latter case? (Take 1 cal
temperatureDT = 45°C
= 4.19 J)
Molecular mass, M = 28
Sol. For adiabatic process, DQ = 0, DW = – 22.3 J
Heat required,
Now DU = DQ – DW = 0 – (– 22.3) = 22.3 J
æ mö æ 20ö æ 7 ö Secondly, DQ = 9.35 cal = 9.35 × 4.2 = 39.3 J
DQ = nCp DT = çè ÷ø C p DT = çè ÷ø çè R÷ø (45) DW = DQ – DU = 39.3 – 22.3 = 17J
M 28 2
12.6 Two cylinders A and B of equal capacity are connected to
20 æ 7 ö each other via stopcock. A contains gas at standard
= ççç ≥ 8.3÷÷÷ 45 < 933.4J
28 è 2 ø temperature and pressure, B is completely evacuated. The
entire system is thermally insulated. The stopcock is
12.3 Explain why
suddenly opened. Answer the following:
(a) Two bodies at different temperatures T1 and T2, if
(a) What is the final pressure of the gas in A and B?
brought in thermal contact do not necessarily settle
(b) What is the change in internal energy of the gas?
T1 + T2 (c) What is the change in the temperature of the gas?
to the mean temperature ?
2 (d) Do the intermediate states of the system (before
(b) The coolant in a chemical or a nuclear plant (i.e. the settling to the final equilibrium state) lie on its P-V-T
liquid used to prevent the different parts of a plant surface?
from getting too hot) should have high specific heat. Sol. (a) If the stopcock opens suddenly, volume of gas would
(c) Air pressure in a car tyre increases during driving. become double at 1 atmospheric pressure, therefore,
(d) The climate of a harbour town is more temperate than pressure would become half (i.e. 1/2 atmosphere).
that of a town in a desert at the same latitude. (b) As no work is done on the system by the gas, there
Sol. (a) In thermal contact, heat from the body at higher will be no change in internal energy.
temperature transfers to the body at lower temperature, (c) Gas does not do any work in expanding, thereby, no
till temperature of both becomes equal. Final change in temperature of gas would occur.
(d) No, the process of the expansion is rapid and cannot
T +T
temperature can be mean temperature æç 1 2 ö÷ only be controlled. The intermediate state would be a non-
è 2 ø equilibrium state, which does not follow gas equation.
when thermal capacities of two bodies are equal. In due course, gas will come back to equilibrium state.
(b) Because heat absorbed µ specific heat of substance. 12.7 A steam engine delivers 5.4 × 108 J of work per minute and
(c) Temperature of air inside the tyre increases due to services 3.6 × 109 J of heat per minute from its boiler.
motion during driving. Air pressure therefore, increase What is the efficiency of the engine? How much heat is
inside the tyre (by Charle's law, P µ T). wasted per minute?
Ther modynamics 315

Sol. Useful work done per min (Out put) = 5.4 × 108 J; Heat absorbed Its volume is then reduced to the original value from E to F by
per min (Input) = 3.6 × 109 J an isobaric process. Calculate the total work done by the gas
from D to E to F.
Output 5.4 ≥108
Efficiency = < ≥ 0.15 = 15% Sol. Work done from D to E
Input 3.6≥109
1
Heat energy wasted/min. = Heat absorbed/min – useful work = WDE = × (5 – 2)(600 – 300) + (5 – 2)(300)
2
done/minute
= 3.6 × 109 – 5.4 × 108 = 3.06 × 109 J = 450 + 900 = 1350 J
12.8 An electric heater supplies heat to a system at a rate of Work done from E to F
100W. If system performs work at a rate of 75 joules per = WEF = 300(2 – 5) = – 900 J
second. At what rate is the internal energy increasing? \ Work done from D to E to F
Sol. Given, DQ = 100W = 100 J/s = WDEF
Useful work done, DW = 75 Js–1 = WDE + WEF
Now, DU = DQ – DW = 100 – 75 = 25 J/s. = (1350 – 900) J = 450 J
12.9 A thermodynamic system is taken from an original state 12.10 A refrigerator is to maintain eatables kept inside at 9°C.
to an intermediate state by the linear process shown in If room temperature is 36°C. Calculate the coefficient of
Fig. performence.
Sol. Given T1 = 36°C = 36 + 273 = 309 K;
D T2 = 9°C = 10 + 273 = 282°K
600
Coefficient of performance
T2 282
= = = 10.45
T1 - T2 309 - 282
Pressure, P(N/m2)

300 E
F

2.0 5.0
Volume, V(m3)

Practice Questions
Very Short Answer Questions [1 Mark Qs.] 9. If on giving 40 joule of heat to a system, work done on
the system is 10 joule, what will be the change in internal
1. Is the heat supplied to a system always equal to the energy of the system?
increase in its internal energy? 10. In a thermodynamical process, 300 joule of heat is supplied
2. If a drop of water falls on a very hot iron, it takes long to a gas and 200 joule of work is done by the gas. What
time to evaporate. Explain. is the change in internal energy of the system?
3. Can temperature of a body be raised without heating it? 11. In summer, when the valve of a bicycle tube is removed,
Is the converse true? the escaping air appears cold, why?
4. Can water be made to boil without heating? 12. When air of the atmoshpere rise up, it cools why?
5. Which molecules, ice at 0°C or water have greater potential 13. Do the internal energy of ideal gas depend upon volume ?
energy and why? 14. Explain why it is impossible to design a heat engine with
6. When an ideal gas is compressed at a constant temperature, 100% efficiency ?
will its internal energy increase or decrease? 15. What thermodynamical variable is defined by (a) Zeroth
7. Does the internal energy of an ideal gas change in an law (b) First law ?
isothermal process? In an adiabatic process? 16. If an inflated tyre bursts, the air escaping out is cooled.
8. A sample of an ideal gas in a cylinder is compressed Why ?
1 17. An engine working under isothermal conditions can
adiabatically to rd of its volume. Will the final pressure produce no useful work. Explain.
3
18. What does second law of thermodynamics imply about
be more or less than 3 times the initial pressure?
the efficiency of a heat engine ?
316 Physics
19. Which thermodynamic variables are involved in an 3. Discuss pressure-temperature phase diagram. What is
indicator diagram ? triple point? Explain.
20. Is Carnot’s cycle a cyclic process ?
Numerical Questions [3 or 5 Marks Qs.]
Short Answer Questions [2 or 3 Marks Qs.]
1. A quantity of air at 27°C and atmospheric pressure is
1. The volume of an ideal gas is V at a pressure P. On suddenly compressed to half its original volume. Find the
increasing the pressure by D P, the change in volume of final pressure and temperature. Given g for air = 1.42
the gas is ( D V1) under isothermal conditions and ( D V2) 2. Calculate the final volume of a gram molecule of a gas after
under adiabatic conditions. Is D V1 > D V2 or vice-versa an isothermal expansion at 127°C, if the original volume is
and why? 400 cc. Given the amount of work done by a gram molecule
2. Can a room be cooled by opening the door of a refrigerator of a gas during expansion is 2302.6 J. R = 8.3J/moleK.
in a closed room? 3. Calculate the work done if one mole of an ideal gas is
3. If an electric fan be switched on in a closed room, will the compressed isothermally at a temperature 27°C from volume 5l
air of the room be cooled? If not, why do we feel cold? to 1l. [R = 8.31 J/moleK]
4. A thermoflask containing tea is shaken vigorously.
4. At 27°C, two moles of an ideal monoatomic gas occupy a
Considering tea as a system
(a) has heat been added to tea? volume V. The gas expands adiabatically to a volume 2V.
(b) has work been done on it? Calculate (a) final temperature of the gas (b) change in its
(c) has its internal energy changed? internal energy and work done by the gas during the
(d) does its temperature rise? process. Take g = 5/3, R = 8.31 J/moleK
5. 200 joule of work is done on a gas to reduce its volume 5. Calculate the work done to compress isothermally 1 g of
by compressing it. If this change is done under adiabatic hydrogen gas at N.T.P. to half its initial volume. Find the
conditions, find out the change in internal energy of the amount of heat evolved and change in internal energy. Given
gas and also the amount of heat absorbed by the gas?
R = 8.31J/moleK.
6. Derive an expression for work done in the isothermal
expansion of a gas. 6. Water of 1 kg mass at 373 K is converted into steam at the
7. Derive an expression for work done in the adiabatic same temperature. On boiling, 1 cc. of water takes a volume
expansion of a gas. of 1671 cc. Calculate the change in internal energy of the
8. Derive the equation of state for adiabatic process. system, taking the heat of vaporisation to be 540 cal/g.
9. Derive the relation between two principle specific heats 7. An ideal gas heat engine operates in a Carnot cycle between
CP and Cv of a gas
227°C and 127°C. It absorbs 6 × 104 calories at the higher
10. What is a heat engine? Obtain an expression for its
efficiency. temperature. How much work per cycle is done by the
11. Obtain an expression for work done by a gas in an engine?
isothermal expansion. 8. In a Carnot engine, the temperature of the source and sink
are 500 K and 375 K respectively. If the engine consumes
Long Answer Questions [5 Marks Qs.] 600 × 103 cals/cycle, find (i) the efficiency of the engine
1 Briefly describe a Carnot engine and derive an expression (ii) heat rejected/cycle (iii) workdone/cycle.
for efficiency of this engine. 9. A refrigerator whose coefficient of performance is 5 extracts
2. Explain the principle and working of a refrigerator and heat from the cooling compartment at the rate of 250J/cycle.
calculate an expression for its coefficient of performance. How much electric energy is spent per cycle? How much
heat per cycle is discharged to the room?

HOTS/Exemplar Questions
Very Short Answer Questions [1 Mark Qs.]
P
1. Suppose a body spends its mechanical energy to overcome
friction. Will it be a reversible process? [HOTS] 1
P0
2. If friction could be totally removed from a Carnot engine,
would it be 100% efficient ? [HOTS] 2
3. A gas is expanded from volume V0 to 2V0 under three different
processes. Process 1 is isobaric process, process 2 is isothermal 3
and process 3 is adiabatic. Let DU1, DU2 and DU3 be the V
change in internal energy of the gas in these three V0 2V 0
processes.Then arrange U1, U2 and U3 in descending order.
[HOTS]
Ther modynamics 317

4. If a refrigerator’s door is kept open, will the room become Long Answer Questions [5 Marks Qs.]
cool or hot? Explain. [Exemplar]
5. Can a system be heated and its temperature remains 1. Figure shows a cycle ABCDA undergone by 2 moles of an
constant? [Exemplar] ideal diatomic gas.
The curve AB is a rectangular hyperbola and T1 = 300 K and
Short Answer Questions [2 or 3 Marks Qs.] T2 = 500 K.
1. It is a sweltering summer day and your air conditioning Determine the following : [HOTS]
system is not operating. In your kitchen, you have a work- (a) Work done by the gas in the process A ® B.
ing refrigerator, and an ice chest full of ice. Which should (b) Heat given to the gas in the process A ® B.
you open and leave open to cool the room more effectively? (c) Molar heat capacity of the gas in the process A ® B.
[HOTS]
2. A box contains five gas molecules spread throughout the V
box. At some time, all five are in one half of the box. Does C
D
this violate the second law of thermodynamics ? Is the sec-
ond law valid for this system ? [HOTS]
3. Consider a cycle tyre being filled with air by a pump. Let V A
be the volume of the tyre (fixed) and at each stroke of the
pump DV (<<V) of air is transferred to the tube adiabatically. B
What is the work done when the pressure in the tube is
increased from P1 to P2? [Exemplar] T
T1 T2
4. A person of mass 60 kg wants to lose 5 kg by going up and
down a 10 m high stairs. Assume he burns twice as much fat
2. Consider that an ideal gas (n moles) is expanding in a
while going up than coming down. If 1 kg of fat is burnt on
expending 7000 kilo calories, how many times must he go up process given by P = f(V), which passes through a point
and down to reduce his weight by 5 kg? [Exemplar] (V0, P0). Show that the gas is absorbing heat at (P0, V0) if
the slope of the curve P = f(V) is larger than the slope of the
adiabat passing through (P0, V0). [Exemplar]
318 Physics

CHAPTER TEST

Time : 45 min. Max. Marks : 24

Directions : (i) Attempt all questions


(ii) Questions 1 to 5 carry 1 mark each.
(iii) Questions 6, 7 and 8 carry 3 marks each.
(iv) Questions 9 and 10 carry 5 marks each.

1. Which thermodynamic variables are defined by the zeroth 9. When a system is taken from state a to state b, in fig. along
law and first law of thermodynamics? the path a ® c ® b, 60 J of heat flow into the system, and 30
2. An ideal gas undergoes four different process from the J of work are done :
same initial state [Fig.]. Four processes are adiabatic, (a) How much heat flows into the system along the path a
isothermal, isobaric and isochoric. Out of 1, 2, 3 and 4 ® d ® b if the work is 10 J.
which one is a adiabatic? (b) When the system is returned from b to a along the
curved path, the work done by the system is –20 J.
Does the system absorb or liberate heat, and how
4 much?
P 3
2 (c) If, Ua = 0 and Ud = 22 J, find the heat absorbed in the
1 process a ® d and d ® b.
V P

3. Is the equation PV = RT valid for isothermal and adiabatic c b


processes?
4. Can two isothermal curves intersect?
5. Cooling is produced when a gas at high pressure suddenly a d
expands. Why?
6. Which one among a solid, liquid and gas of same mass V
and at the same temperature has the greatest and least 10. A perfect Carnot engine utilizes an ideal gas. The source
internal energy and why? temperature is 500 K and sink temperature is 375 K. If the
7. Ice at 0°C is converted into steam at 100°C. State the engine takes 600 K cal per cycle from the source, compute :
isothermal changes in this process. (a) the efficiency of the engine.
8. Calculate difference in efficiencies of a Carnot engine (b) work done per cycle.
working between (i) 400 K and 350 K (ii) 350 K and 300 K (c) heat rejected to the sink per cycle.
Ther modynamics 319

Solutions
PRACTICE QUESTIONS \ Either T2 = 0 or T1 = ¥ both of which cannot be
attained. Hence it is impossible to design heat engine
Very Short Answer Questions with 100% efficiency.
1. From 1st law of thermodynamics, dQ = dU + dW 15. (a) Temperature T
If dW = 0, then dQ = dU. i.e. if heat is supplied at constant (b) Internal energy U
volume so that the work done is zero then the heat 16. When the tyre bursts, adiabatic expansion takes place in
supplied will be equal to the increase in the internal energy which work is done by the escaping air.
of the system. Hence, dU = Q – W
2. When a drop of water falls on a very hot iron, it gets Þ dU = Q – W [Q q = 0]
separated from the iron by a thin layer of water vapour As W > 0, dU < 0 hence internal energy decreases due to
which is a bad conductor of heat. So, heat is conducted which temperature falls.
from the iron to the water drop very slowly so that it 17. Because, the efficiency of an engine working between
evaporates in a long time. same temperature of source and sink is zero. i.e.
3. Yes, in adiabatic compression. Converse is also true, e.g. T
h = 1 - = 1 -1 = 0
when a solid melts, temperature does not change, though T
heat is increased. 18. According to second law of thermodynamics, the
4. Yes, by reducing pressure on water, boiling point can be efficiency of a heat engine cannot be equal to 1.
reduced, even at room temperature. 19. Indicator diagram is a P-V graph, hence pressure P and
5. Potential energy of water molecule at 0°C is more, because volume V are involved in it.
heat spent in melting is used in increasing the potential 20. Yes, in carnot’s cycle the working substance regains its
energy. original state.
6. No effect since internal energy of gas depends only on
Short Answer Questions
temperature of gas.
7. In isothermal process, T = constant, dT = 0
DP
\ dU = 0. 1. In isothermal conditions, Ki = = P;
In an adiabatic process, Q = constant, dQ = 0 DV1 / V
\ dU = – dW ¹ 0
DP
8. Change in pressure would be more than 3 times the initial In adiabatic condition, K a = = gP
pressure. DV2 / V

g
P2 K a DV1
P æV ö g \ g >1 \ >3 Now = = g , DV1 > DV2 (Q g > 1 )
Since 2 = ç 1 ÷ = 3 P1 Ki DV2
P1 è V2 ø
2. No, when door of refrigerator is closed, refrigerator is
9. Internal energy of the system would increase by (40 + 10) rejecting heat to the room, so the room temperature
= 50 joule. increases gradually. When door is open, heat rejected by
10. Internal energy of the system would increase by (300 – refrigerator would be more than, heat taken from the room,
200) = 100 joule. which is equal to the work done by the compressor.
11. This occurs due to adiabatic expansion of the air of the Therefore, temperature of room will increase at a slower
tube of the bicycle. rate compared to 1st case.
12. When air rises up, it expands due to decrease in the 3. No, air in room will be heated due to motion of the fan
atmospheric pressure. Therefore, temperature falls. and speed of air molecules will increase. However we feel
13. No, for an ideal gas the internal energy depends only on cold due to evaporation of our sweat.
temperature i.e. U = f (T). 4. (a) No, heat is not transfered from the surrounding i.e.
14. For 100% efficiency, h = 1 dQ = 0.
T2 (b) Yes, work is done in shaking against the viscous
For a heat engine, h = 1 - force, i.e. dW = –ve
T1
(c) Yes, internal energy of tea increases, as dU = dQ –
T2 dW = 0 – dW, i.e. dW is negative,
Now h = 1 Þ 1 - =1 \ dU = +ve
T1
(d) Yes, temperature of tea rises due to increase in
T2 internal energy.
Þ =0
T1
320 Physics
5. In adiabatic change, dQ = 0 Let P 1 , V 1 , T 1 be the initial pressure, volume and
\ from dQ = dU + dW = 0 temperature of the gas and P2, V2, T2 be its final pressure,
Þ dU = – dW = – (20 0) = 200 J volume and temperature.
Internal energy increases by 200 J. Heat absorbed is zero.
If A be the area of cross section of the piston, then the force
6. Consider 1gm mole of an ideal gas in a cylinder with perfectly
exerted by the gas on the piston is F = P × A
conducting walls and fitted with a perfectly conducting and
frictionless piston. \ Work done by the gas for a small displacement dx of the
piston = dW = Fdx = PA dx = PdV
\ Total work done by the gas in expanding from volume V1
to volume V2 = W
V2

1 gm mole of
= ò dW = ò PdV .
V1
ideal gas Equation for adialatic change is PVg = K

K
Let P1, V1, T be the initial pressure, volume and \ P= = V -g K
g
temperature of the gas and P2, V2, T be the final pressure, V
volume and temperature of the gas. When the gas is V2
expanded, volume increases, so, V2 > V1 but pressure
òV
-g
\W = KdV
decreases, so, P2 < P1. For isothermal process, temperature
V1
remains constant. Let A be the cross-sectional area of the
piston then the force exerted by the gas on the piston is F =
V
P×A éV -g + 1 ù 2 K
[V 1-g ]V2
V
Work done by the gas for a small expansion = Kê ú =
dx = dW = Fdx = P × Adx = PdV êë -g + 1 úû 1- g 1
V1
[Adx = dV = change in volume of the gas]
Total work done by the gas in expanding it from volume V1 K
= [V21–g – V11–g]
V2 1- g
ò
to V2 is W = dW =
ò PdV 1
V1 or, W = [KV21–g – KV11–g]
RT 1- g
From ideal gas equation, PV = RT, P =
V 1
= [P V g.V 1-g – P1V1g V11-g]
V2 V2 1- g 2 2 2
RT dV
ò ò
\ V
W= dV = RT = RT [log e V ]V2
V V 1 [Q PVg = K, \ P1V1g = P2V2g = K]
V1 V1
1
V2 Þ W= [P V – P1V1]
\ W = RT [ loge V2 - loge V1 ] = RT log e 1- g 2 2
V1
Also, from ideal gas equation for 1 mole PV = RT
V2 \ P1V1 = RT1 and P2V2 = RT2
\ W = 2.303 RT log10
V1
1 R
7. Consider 1 gm mole of an ideal gas in a cylinder with perfectly \ W= [RT2 – RT1] = [T – T ]
1- g 1 - g 2 1
non-conducting walls and fitted with a perfectly non-
8. Consider 1 gm mole of an ideal gas in a cylinder with perfectly
conducting frictionless piston.
non-conducting walls and fitted with a perfectly frictionless
and non-conducting piston.
Let P, V, T be the pressure, volume and temperature of the
gas at any instant.
If a small amount of heat (dQ) is given to the gas, it will be
spent in two ways.
1 gm mole of (a) In increasing its temperature by dT at constant volume.
ideal gas
Heat used for that process = 1. CV. dT
[Q n = 1 mole]
Ther modynamics 321

(b) In the expansion of the gas by a small volume dV. Heat Þ (CP – CV) dT = PdV
used in that process = PdV From ideal gas equation for 1 mole, PV = RT
\ dQ = CV dT + PdV [ Differentiating both side, PdV = RdT
But for adiabatic change, no heat is given or taken out [ (C – C ) dT = RdT Þ CP – CV = R
P V
from the system. 10. A heat engine is a device which converts heat energy into
\ dQ = 0 \ CVdT + PdV = 0 mechanical energy. It consists of three basic parts,
From the ideal gas equation of 1 mole PV = RT (a) Source of heat
\ Differentiating both sides, (b) Working substance
PdV + VdP = RdT (c) Sink of heat
PdV + VdP Source at
\ T1 > T2
dT = temperature T1
R
Q1
æ PdV + VdP ö
\ CV ç ÷ø + PdV = 0
è R Working
Substance W
Þ CVPdV + CVVdP + RPdV = 0
Þ (CV + R) PdV + CVVdP = 0
Q2
[Q Cp – CV = R; \ Cp = CV + R]
Þ Cp PdV + CVVdP = 0 Sink at
temperature T2
Dividing both sides, by CVPV
CP PdV CV VdP The working substance, generally a gas, absorbs an amount
Þ . + . =0 of heat Q1, from the source at temperature T1, converts a
CV PV CV PV
part of it into work (W) and rejects the rest (Q2) to the sink at
dV dP CP temperature T2.
Þ g. + = 0 [Q =g]
V P CV [ Net amount of heat absorbed by the working substance
= dQ = Q1 – Q2
dV dP Q The working substance comes back to its initial state
Intergrating both sides, g . ò V +ò P
=C
after the operation, so there is no change in its internal
= constant Þ g logeV + logeP = C energy.
[ dU = 0
Þ loge Vg + logeP = C Þ loge PVg = C
[ By 1st law of thermodynamics
Þ PV = e = K = constant Þ PVg = K
g C
dQ = dU + dW
9. Consider 1 gm mole of an ideal gas in a cylinder fitted with [ dQ = dW = Q – Q
a perfectly frictionless piston. Let P, V, T be pressure, volume, 1 2
[ Efficiency of the heat engine
and temperature of a gas. When the gas is heated at constant
volume by a small temperature dT, amount of heat energy Net work done
=
given to the gas, dQ = 1.CV.dT [Q n = 1 mole] Heat supplied
As the volume remains constant, dW Q - Q2 Q
[ dV = 0 Þ h= = 1 =1- 2
Q1 Q1 Q1
[ dW = PdV = 0
As Q2 < Q1 [ h<1
By 1st law of thermodynamics, dQ = dU + dW
[ CvdT = dU 11. Consider 1 gm mole of an ideal gas enclosed in a cylinder
with perfectly conducting walls and fitted with a perfectly
When the gas is heated at constant pressure by the same
frictionless and conducting piston.
temperature dT, amount of heat given to the gas dQ' =
Let, P1 , V1 and T be the initial pressure, volume and
1.CP.dT
temperature of a gas.
If the increase in volume in this case is dV then work done Also, P2, V2, and T be the final pressure, volume and
by the gas in the expansion temperature of the gas.
= dW = PdV. Small work done during expansion of the gas
By 1st law of thermodynamics, dQ ¢ = dU ¢ + dW = dW = F × dx = P × A × dx = PdV
[ C dT = dU ¢ + PdV where dv = Adx = small increase in volume of the gas.
p
Since, the increase in temperature (dT) is same in both cases, V2

ò PdV
[ dU = dU' = C dT [ Total work done = W =
V
[ C dT = C dT + PdV
P V V1
322 Physics

RT
Q PV = RT \ P= Y
V A (V1 , P1 )
V2 V2 Q1

ò
RT
V
dV = RT ò
dV
V
= RT loge ( )
V2
V1
B(V 2 , P2 )
T1

Pressure (P)
V1 V1 D(V 4 , P4 )
W = RT [logeV2 – logeV1] = RT loge V2/V1
C(V3 , P3 )
V2 Q2
T2
[ W = 2.3026 RT log 10 V1 X
O KL M N
Volume (V)
Long Answer Questions

1. A Carnot engine is an ideal heat engine and consists of the Work done in expanding the gas
following parts. isothermally from A to B = W1
I. Source of heat: It is at a constant temperature T1 and V2
V
of infinite thermal capacity. So, when the working
substance draws heat from the source its temperature
= Q1 = ò P.dV = RT1 loge V12 ... (1)
V1
does not change. = Area (ABMKA)
(b) Adiabatic expansion: The cylinder is removed from
II. Sink of heat: It is also at a constant temperature T2 (T2
the source and is placed on the insulating pad. The
< T1) and of infinite thermal capacity. So, when the gas is allowed to expand from B to C in the P–V
heat is rejected to the sink its temperature doesn't diagram. Since the gas is thermally insulated from the
change. surroundings, so, no exchange of heat takes place.
\ The expansion is adiabatic and the temp. drops to
T2.
V3
1 gm mole of
ideal gas
Work done by the gas = W2 =
ò PdV
V2

Source at Insulating Sink at R (T2 - T1 )


= ..... (2)
temp. T1 pad temp. T2 1- g
= Area (BCNMB)
III. Working substance: It is an ideal gas (1 mole) in a (c) Isothermal compression: The cylinder is removed from
cylinder with perfectly non-conduting walls but insulating pad and placed on the sink at temperature
perfectly conducting base and fitted with a perfectly T2. The gas is now compressed from C (V3, P3) to
non-conducting and frictionless piston. D (V4, P4) in the P-V diagram. The heat produced
during compression will go to the sink and temperature
IV. Insulating pad: The cylinder is placed on it during
will remain constant at T2. So, the process is isothermal.
adiabatic operations so that no heat exchange can take Work done in this process
place.
V4
Theory: The Carnot cycle consists of 4 steps: V4
(a) Isothermal expansion
= W3 = Q2 = ò PdV = RT2 log e V3
V3
(b) Adiabatic expansion.
(Work done is negative as it is done on the gas)
(c) Isothermal compression
(d) Adiabatic compression V3
= – RT2 loge ..... (3)
(a) Isothermal expansion: The cylinder should be placed V4
on the source and the gas be allowed to expand slowly. = – Area CDLNC
Heat is absorbed by the gas through the base of the (d) Adiabatic compression: The cylinder is again placed
cylinder so that the temperature of the gas remains on the insulating pad. The gas is compressed from D
(V4, P4) to A (V1, P1). As the gas is insulated from all
constant. AB represents the isothermal expansion in
sides, its temperature will increase to T1.
the P-V diagram. Work done in this process = W4
Ther modynamics 323

V1 Sink at
- R (T2 - T1 )
= ò - PdV =
1- g
..... (4) temperature T2
V4
Q2
= – Area DAKLD
\ Total work done by the gas in the complete cycle = W Working Electric
Substance Motor
= (W1 + W2) – (W3 + W4) W
[Q W3 and W4 are work done on the gas, so they are
negative] Q1
But W2 = W4 in magnitude Source at
\ W = W1 – W3 = Q1 – Q2 temperature T1
or, W = Area (ABCDA) Working :
Expression for efficiency After completing one cycle of operation, the working
substance comes back to its initial state.
W Q1 - Q2 Q \ Change in internal energy = dU = 0
Efficiency = h = = = 1- 2 dQ = Q2 – Q1 and dW = – W
Q1 Q1 Q1
\ By using 1st law of thermodynamics,
Using the equation of state for the four steps dQ = dU + dW
Þ Q2 – Q1 = 0 – W
We get, P1V1= P2V2 (isothermal expansion)
W = Q1 – Q2
P2V2g = P3V3g (adiabatic expansion) Coefficient of performance of a refrigerator
P3V3= P4V4 (isothermal compression) quantity of heat absorbed from the sink
b=
P4V4g = P1V1g (adiabatic compression) energy spent/given on it
Multiplying the four equations we get,
(P1V1) (P2V2g) (P3V3) (P4V4g ) Q2 Q2
\ b= =
= (P2V2) (P3V3g) (P4V4) (P1V1g) W Q1 - Q2
V1 V2g V3 V4g = V2 V3g V4 V1g
Q2 Q2
V2g–1 V4g–1 = V3g–1 V1 g–1
Q1 Q1
= =
Þ (V2V4)g–1 = (V1V3)g–1 Þ V2V4 = V1V3 Q1 Q2 Q
- 1- 2
Q1 Q1 Q1
V2 V3 æV ö æV ö Q2 T2
Þ = Þ log ç 2 ÷ = log ç 3 ÷ But Q = T
V1 V 4 è V1 ø è V4 ø 1 1

T2 / T1 T2
Q RT2 log e (V3 / V4 ) T2 b= \ b=
Also, 2 = = 1 - T2 / T1 T1 - T2
Q1 RT1 log e (V2 / V1 ) T1 3. In a pressure-temperature phase diagram, each phase (like
solid, liquid and gas) corresponds to a region or area. The
T2 boundary between any two areas is a curve on which the
\ h = 1-
T1 two phase co-exists in equilibrium.

2. Principle of a refrigerator:
In a refrigerator, the working substance absorbs certain
Ice e
amount of heat (Q2) from the sink which is at a lower Water lin
lin am
Pressure (P)

temperature T2 and reject a larger amount of heat (Q1) to the e Ste


source which is at a higher temperature T1. This is possible Steam
with the help of an external agency which supplies necessary Ice
Hoarfrost line
energy to the system.

Temperature (T)
324 Physics
The adjacent figure shows the P-T diagram for water.
æV ö
There are 3 curves in the diagram. W = 2.3026 RT log10 ç 2 ÷ w = ?
I. Vaporisation/steam line: It represents variation of è V1 ø
boiling point of the substance (in liquid state) with
æ 1ö
pressure. On this line, the substance coexists in liquid = 2.3026 × 8.31 × (273 + 27) log çç ÷÷÷
çè 5 ø
and vapour phases.
II. Fusion/Ice line: It represents variation of melting point = 2.3026 × 8.31 × 300 (log 1 – log 5)
of the substance with pressure. On this line, the = 2.3026 × 8.31 × 300 ( 0 – 0.6990 = – 4012.5 J
substance coexists in solid and liquid phases. 4. (a) using, T1V1g–1 = T2V2 g – 1
III. Sublimation line/Hoarfrost line: It represents the 5
g -1 -1
variation of pressure with temperature at which a solid æV ö æ 1ö 3
T2 = T1 ç 1 ÷ = 300 ç ÷ = 189K
directly changes into vapour. At all points above this èV ø
2 è 2ø
line, the substance exists only in solid state whereas
below this line it only exists in vapour state. nR DT
(b) dU = ncv DT =
When the three lines are produced they meet at a point g -1
P called the triple point of the substance where the
substance can coexist in all three phases. Therefore, 3
= 2≥ ≥8.31(189 , 300) = – 2767.23 J.
the coordinate of the point P represents that particular 2
temperature and pressure at which the substance exists Q dQ = dU + dW but dQ = 0
in all three phases. [ dW = – dU = 2767.23J.
Numerical Questions æ mö æV ö
5. Work done, W = 2.3026 ç ÷ RT log10 ç 2 ÷
èMø è V1 ø
1. Here, T1 = 27 + 273 = 300 K, P1 = 1 atm.
1 1 æ1 ö
= 2.3026≥ ≥8.31≥ 273log çççè ø÷÷÷
V 2 = V1 , T2 = ?, P2 = ?, g = 1.42
2 2 2
g
æ V1 ö [ W = – 786.2 J
Q P1V1 g= P2V2 g
Þ P2 = P1 ç ÷
èV ø2 W 786.2
= 1(2)1.42 = 2.67 atm Heat evolved = = = 187.2 cal.
J 4.2
g -1
æ V1 ö Since the change is isothermal, so, temperature remains
Also, T1V1g-1 = T2V2g-1 Þ T2 = T1 ç ÷ constant.
èV ø 2
[ Internal energy doesn’t change, so change in internal
1.42 - 1 energy is zero.
é V ù
Þ T2 = 300 ê = 300 (2)0.42 6. Given: mass
ë V / 2 úû m = 1kg = 103 g, initial vol. V1 = 103 cc.
= 300 × 1.338 K = 300 × 1.338 – 273 = 128.37°C Final vol V2 = 1671 × 103 cc, P = 1 atm
2. Here, = 1.013 × 106 dyne /cm2
T = 127 + 273 = 400 K, V1 = 400cc. V2 = ?, Work done, dW = 2.303 nrT log (v2/v1)
W = 2302.6 J, R = 8.3 J/moleK æ1000 ÷ö æ1671≥103 ö÷
= 2.303≥çç çç ÷÷
÷ ≥1.98≥ 373 ≥ log ç
æ V2 ö çè 18 ÷ø çè 103 ø÷÷
Q W = 2.3026 RT log 10 ç ÷
è V1 ø = 304.54 kcal
Heat supplied, dQ = mL = (103 × 540) cal
æV ö = 540 k cal
Þ 2302.6 = 2.3026 × 8.3 × 400 log10 ç 2 ÷
è V1 ø [ Change in inernal energy, dU = dQ – dW
= 540 – 304.54 = 235.46 k cal.
æV ö 2302.6 7. Here,
Þ log10 ç 2 ÷ =
è 1ø
V 2.3026 ´ 8.3 ´ 400 T1 = 227°C = (227 + 273) = 500 K;
T2 = 127°C = (127 + 273) = 400 K
= 0.3012 = log102 Q1 = 6 × 104 cal, work done per cycle, W = ?
V2 Q1 T1 T
[ = 2 Þ V2 = 2V1 = 2 × 400 = 800 cc. Q = Þ Q2 = Q1 2
V1 Q2 T2 T1
3. Given: V1 = 5l, V2 = 1l
[ Q = 6 ´ 104 ´ 400 = 6 ´ 104 ´ 4
T = 27 + 273 = 300K 2
500 5
Work done,
Ther modynamics 325

passed through the refrigeration system and transferred


4
[ Work done per cycle, = W = Q1 – Q2 = 6 × 104 – 6 × 104 × right back into the air. The result would be that the kitchen
5 would become warmer, due to the addition of the energy
4æ 4ö 4 1
coming in by electricity to run the refrigeration system. If
= 6≥10 ççç1, ÷÷÷ < 6≥10 ≥ the ice chest were opened, energy in the air would enter
è 5ø 5
the ice, raising its temperature and causing it melt. The
= 1.2 × 104 cal = 1.2 × 4.2 × 104 = 5.04 ×104J
transfer of energy from the air would cause its temperature
8. Given:
to drop. Thus, it would be more effective to open the ice
T1 = 500 K, T2 = 375 K, Q1 = 600 × 103 cal.
chest.
T2 375 3 1 2. Strictly speaking, this situation does violate the second
(i) h =1- =1- 1 , < < 25%
T1 500 = 4 4 law of thermodynamics. In response to the second
question, however, the second law is not for small numbers
Q2 T2 T2 of particles. The second law is based on collections of
(ii) Q Q = T Þ Q2 = Q1 T huge numbers of particles for which disordered states have
1 1 1
astronomically higher probabilites than ordered states.
375 Because the macroscopic world is built from these huge
= 6 ≥105 ≥ = 4.5 × 105 cals
500 numbers of particles, the second law is valid as real
(iii) W = Q1 – Q2 = 6 × 105 – 4.5 × 105 processes proceed from order to disorder. In the five-
= 1.5 × 105 cals = 1.5 × 105 × 4.2J = 6.3 × 105J molecule system, the general idea of the second law is
9. Here, heat extracts, Q2 = 250 J/cycle, valid, in that more disordered states occur than ordered
coefficient of performance = 5 ones, but the relatively high probability of the ordered
Q2 states results in their existence from time to time.
Q Coefficient of performance = =5 g
W 3. P ( V + Dv ) = ( P + Dp ) V g
Q2 250
[ W = = = 50J é Dv ù æ Dp ö
5 5 P ê1 + g ú = P ç1 + ÷
ë Vû è P ø
[ Heat discharged/ cycle to the room
= Q1 = Q2 + W = 250 + 50 = 300 J Dv Dp dv V
g = ; =
V P dp gp
HOTS/EXEMPLAR QUESTIONS
P2 P2
V ( P2 - P1 ) V
Very Short Answer Questions W.D. = ò p dv = ò p gp dp = g
1. When a body is taken from one position to another, it P1 P1
spends mechanical energy to overcome friction. This 4. Here, 5 × 7000 × 103 × 4.2J = 60 × 15 × 10 × N
energy is dissipated as heat. Now, if the body is allowed
to go round to its path in reverse direction, its energy 21´ 7 ´ 10 6 147
\ N= = ´103 = 16.3 ´103 times
spent is not recovered but instead is further spent in the 900 9
reverse path. Hence the process is not reversible.
2. No. Friction is one factor that reduces the efficiency of an Long Answer Questions
engine, but even a frictionless engine is restricted to an 1. Evidently, for the process A ® B
efficiency less than or equal to that of the Carnot engine,
which is less than 100%. 1
Vµ or VT = constant (say, k)
3. Since volume is same in all three process therefore T
temperature will be least having least pressure. \
DU1 > DU 2 > DU3 VdT
or TdV + VdT = 0 or dV = -
4. Here heat removed is less than the heat supplied and hence T
the room, including the refrigerator (which is not insulated (a) Now, work done by the gas in a process is given by
from the room) becomes hotter.
5. If the system does work against the surroundings so that it W = ò PdV or
compensates for the heat supplied, the temperature can
remain constant. nRTdV
WAB = ò = ò (- nRdT)
V
Short Answer Questions
500
1. The high-temperature reservoir for your kitchen refrigerator
is the air in the kitchen. If the refrigerator door were left
WAB = ò -nRdT = – (2 mol) (8.314 J/mol-K) [500 –300]
300
open, energy would be drawn from the air in the kitchen
K = –3.326 kJ
326 Physics
(b) Heat given to the gas in the process A ® B 7. The isothermal changes are :
Q AB = WAB + DU AB (i) conversion of ice at 0°C to water at 0°C
5 ´ 8.314J (ii) conversion of water at 100°C to steam at 100°C
= -3.326kJ + (2mol) (200K) = 5kJ
2mol - K T2
8. Efficiency of a Carnot engine = h = 1 -
é mRDT ù T1
êQ DU = ú
ë g -1 û 350 50 1 100
γ ' < 1, = < < = 12.5 %
(c) Molar heat capacity C is given by 400 400 8 8
Q 5 ´ 103 J 300 50 1 100
C= = = 12.5 J / mol - K γ ' < 1, < < < % < 14.3%
nDT (2mol) (200K) 350 350 7 7
2. Slope of P = f(V), curve at (V0, P0) [ Difference in efficiencies
= f(V0)
Slope of adiabat at (V0, P0) = γ ',γ = 14.3 – 12.5 = 1.8%

= k ( -g ) V0-1-g = -gP0 / V0 9. For the path a® c® b


Now heat absorbed in the process P = f(V) dU = dQ – dW = 60 – 30 = 30 J or Ub – Ua = 30 J
dQ = dV + dW (a) Along the path a®d®b
= nCv dT + PdV dQ = dU + dW = 30 + 10 = 40 J
Since T = (1/nR) PV = (1/nR) V f(V) (b) Along the curved path b®a
dT = (1/nR) (f(V) + Vf¢ (V)] dV
dQ = (Ua – Ub) + W = (–30) + (–20) = –50 J,
Thus
heat flows out the system
dQ CV
= éf ( V0 ) + V0 f ¢ ( V0 )ùû + f ( V0 ) (c) Qad = 32 J; Qdb = 8 J
dV V =V0 R ë
10. (a) Here, T1 = 500 K, T2 = 375 K
é 1 ù V f ¢ ( V0 ) Q1 = Heat absorbed per cycle = 600 K cal
= ê + 1ú f ( V0 ) + 0 \ (a) Using the relation,
ë g -1 û g -1
T2
g V h =1- , we get
= P0 + o f ¢ ( V0 ) T1
g -1 g -1
T1 - T2 500 - 375 125
Heat is absorbed when dQ /dV > 0 when gas expands, that is when h= = = = 0.25
T1 500 500
gP0 + V0 f ¢ ( V0 ) > 0
\ h% = 0.25 × 100 = 25%
f ¢ ( V0 ) > -gP0 / V0
(b) Let W = work done per cycle
CHAPTER TEST \ Using the relation
1. Zeroth law defines temperature and first law defines internal W
energy. h= , we get
Q1
2. Process 2 is adiabatic.
3. Yes, the equation PV = RT is valid for all types of the W = hQ1
thermodynamically processes.
4. No, if two isothermal curves intersect, then this would mean = 0.25 × 600 k cal
that the pressure and volume of a gas are the same at two = 150 k cal
different temperatures. This is not possible. = 150 × 103 × 4.2 J
5. During the expansion. The gas does work against high = 6.3 × 105J.
pressure. This decreases the internal energy and hence the
(c) Let Q2 = heat rejected to the sink
temperature of the gas.
6. A gas has highest value of internal energy because negative \ Using the relation
potential energy of its molecules is smallest. Internal energy W = Q1 – Q2, we get
of a solid is the least because negative potential energy of Q2 = Q1 – W = 600 – 150 = 450 k cal
its molecules is maximum.
¿¿¿
13 Kinetic Theory
C ha p t e r

Kinetic theory explains the behaviour of gases on the basis of fact that the gas consists of rapidly moving atoms or molecules
as the interatomic forces in gas in comparison to solids and liquids are negligible.

MOLECULAR NATURE OF MATTER


Matter is made of atoms. About 200 years ago John Dalton suggested that the smallest constituents of an element are atoms.
Atoms of each element combine to form a molecule. Hence Dalton’s atomic theory also referred to as the molecular theory of
matter. The size of an atom is about an angstrom (1Å = 10–10 m). In solids and liquids, the separation between atoms is about
the same ( @ 2Å) but in liquids the atoms are not as rigidly fixed as in solids. This enables a liquid to flow. In gases distance
between atoms i.e., interatomic distance is approximately five times of solids or liquids. The gases atoms are much freer and can
travel long distances without colliding.
The average distance a molecule can travel without colliding with other molecules within the matter is known as the mean free
path. For gases it is ~1000 Å.

BEHAVIOUR OF GASES
Ideal Gas
A gas that satisfies the perfect gas equation exactly at all conditions of temperature and pressure is called an ideal gas.
An ideal or perfect gas has two characteristics:
(i) Size of the molecule is zero i.e. the molecules are point masses with no dimensions
(ii) There is no attraction or repulsion between the molecules of the gas.
At high temperature or low pressure the real gas approaches ideal behavior.
Ideal/perfect gas equation is an equation relating the pressure, volume and temperature of the gas.
i.e., PV = nRT
P = Pressure, V = Volume, T = Temperature, n = No. of moles of the gas,
R = Universal gas constant = 8.31 J/molK.
Ideal gas equation in terms of density : PM = rRT
where M is the molecular mass of the gas and r is the density.
Avogadro’s Law
Equal volumes of all gases at equal temperature and pressure have the same number of molecules.
PV
1 1 PV
i.e., = 2 2 = constant = k B
N1T1 N 2T2
Here kB is called Boltzmann constant and has a value 1.38 × 10–23 JK–1.
If P, V and T are same, then N is also same for all gases.
Boyle’s Law
It states that the volume of a given mass of a gas is inversely proportional to its pressure when the temperature of the gas
remains constant.
1
i.e., Vµ Þ PV = k = constant \ P1V1 = P2V2
P
328 Physics
Thus, P-V graph in an isothermal process is a rectangular hyperbola, or PV versus P or V graph is a straight line parallel to P or V axis.
P PV

T=constant
T=constant

V P or V
Charles’ Law
It states that the volume of a given mass of a gas increases or decreases by 1/273.15 of its volume at 0°C for each 1°C increase
or decrease in temperature when the pressure of the gas remains constant.
æ t ö æ 273.15 + t ö
Vt = V0 ç1 + = V0 ç
è 273.15÷ø è 273.15 ÷ø
æ V0 ö
273.15 + t = TK = Temperature in Kelvin scale \ Vt = ç T
è 273.15 ÷ø K
Charles’ law can also be stated as, volume of a given mass of a gas is directly proportional to its absolute temperature
when pressure is constant
i.e., V µ T
V V1 V2
or = K = constant or =
T T1 T2

V
Thus, V-T graph in an isobaric process is a straight line passing through origin, or versus V or T graph is a straight line parallel
T
to V or T axis.
V V
T
P=constant
P=constant

T(in K) V or T

Gay Lussacs’ Law


It states that, the pressure of a given mass of a gas increases or decreases by 1/273.15 of its pressure at 0°C, for 1°C increase
or decrease in temperature when the volume of the gas is constant.
æ t ö æ 273.15 + t ö
Pressure at t°C = Pt = P0 ç1 + = P0 ç
è 273.15 ÷ø è 273.15 ÷ø
T Pt P0 P
Pt = P0 ´ ; \ = \ = constant
T0 T T0 T
\ P µ T when volume of the gas is constant; \ Gay Lussac’s law can also be stated as that the pressure of a given
mass of a gas is directly proportional to its absolute temperature when volume of the gas is constant.
P
Thus P-T graph in an isochoric process is a straight line passing through origin or versus P or T graph is a straight line parallel
T
to P or T axis.
P P
T

V=constant
V=constant

T(in K) P or T
Kinetic Theory 329

Dalton’s Law of Partial Pressure


Partial Pressure : The pressure exerted by an individual gas in a mixture of non-interacting gases at the same conditions of
temperature and volume is called its partial pressure.
Dalton’s law of Partial Pressure : The total pressure of a mixture of ideal gases is the sum of the partial pressures exerted by individual
gas.
i.e., P = P1 + P2 + .........
Let us consider a mixture of gases occupying a volume V. Suppose the first gas contains N1 molecules, each of mass m1 having mean-
2 2
square-speed v rms1 . The second gas contains N2 molecules each of mass m2 and mean-square-speed v rms2 , and so on. Let P1,
P2,........Pn are the partial pressures of the gases. Each gas fills the whole volume V. According to kinetic theory, we have
1 2 1 2
P1V = m1N1 vrms , P2 V = m 2 N 2 v rms2 , and so on.
3 1
3
1 2 2 2
Adding, we get (P1 + P2 + ....) V = (m1N1v rms1 + m 2 N 2 v rms2 ... + m n N n v rmsn ) ......(i)
3
Now, the whole mixture is at the same temperature.
1 2 1 2 1 2
\ m1v rms = m 2 v rms ...... = mvrms (say).
2 1
2 2
2
Substituting this result in eqn. (i) we have
1 2
(P1 + P2 + ......) V = (N1 + N 2 + .... + N n ) mv rms
3
The mixture has a total number of molecules (N1 + N2 + ....+ Nn). Hence the pressure P exerted by the mixture is given by
1 2
PV = ( N1 + N 2 + ......) mvrms
3
i.e., P = P1 + P2 .......+ Pn.
This is Dalton's law of partial pressures.
KINETIC THEORY OF GASES
The theory is based on following assumptions as regards to the motion of molecules and the nature of the gases.
(1) All the molecules of a gas are identical . The molecules of different gases are different.
(2) The molecules are rigid and perfectly elastic spheres of very small diameter.
(3) Gas molecules occupy very small space. The actual volume occupied by the molecule is very small compared to the total volume
of the gas. Therefore volume of the gas is equal to volume of the vessel.
(4) The molecules of gases are in a state of random motion, i.e., they are constantly moving with all possible velocities lying between
zero and infinity in all possible directions.
(5) Normally no force acts between the molecules. Hence they move in straight line with constant speeds.
(6) The molecules collide with one another and also with the walls of the container and change there direction and speed due to
collision. These collisions are perfectly elastic i.e., there is no loss of kinetic energy in these collisions.
(7) The molecules do not exert any force of attraction or repulsion on each other except during collision. So, the molecules do not
posses any potential energy. Their energy is wholly kinetic.
(8) The collisions are instantaneous i.e., the time spent by a molecule in a collision is very small as compared to the time elapsed
between two consecutive collisions.
(9) Though the molecules are constantly moving from one place to another, the average number of molecules per unit volume of the
gas remains constant.
(10) The molecules inside the vessel keep on moving continuously in all possible directions, the distribution of molecules in the
whole vessel remains uniform.
(11) The mass of a molecule is negligibly small and the speed is very large, there is no effect of gravity on the motion of the molecules.
If this effect were there, the density of the gas would have been greater at the bottom of the vessel.
330 Physics
Expression for the Pressure Exerted by a Gas
A gas exerts pressure on the walls of the containing vessel due to continuous collisions of the Y
molecules against the wall. B F
Consider a gas enclosed in a cube shape container having each side length l.
The area of each face of the cube is A = l2. A E
vy v
The volume of the cube is V = l3.
vz m vx G
Let the total number of molecules of the gas inside the cube = N C X
The mass of each molecule = m O
D
Suppose that the three intersecting edges of the cube are along the rectangular co-ordinate H
axes X, Y, and Z with the origin O at one corner of the cube. Z
Consider a molecule which has a velocity
r
v = v iˆ + v ˆj + v kˆ
x y z
If this molecule collides the face EFGH, it will rebound with its x component of velocity (–vx). There will be no effect on vy or vz and
after collision molecule will move with constant velocity along straight path along –ve X-axis.
\ Change in momentum of the molecule = final momentum – initial momentum = –mvx – (mvx) = –2mvx
Since the total momentum is conserved, the momentum imparted by the molecule to the wall of the container in this impact to the face
EFGH = 2mvx.
\ Change in momentum of the surface EFGH = +2mvx
Some time later, the molecule strikes the opposite wall ABCD and eventually returns to the wall EFGH.
Between two successive collisions with the same face EFGH, the molecule covers a distance 2l.
2l
Therefore, the time between two successive collisions at the same face EFGH = v .
x
vx
Hence in one second, the number of collisions of the molecule with face EFGH = .
2l
Change in momentum of surfaces EFGH per second due to this molecule
= [change in momentum in one collision] × [number of collisions per second]
v x mv2x
= 2mv x ´ =
2l l
To find the total change in momentum per second at the wall, we add the contributions of all these molecules.
m 2
Change in momentum per second = (v x1 + v x2 2 + vx3
2
+ ...... + v 2xN )
l
By Newton's second law of motion the change in momentum per second is equal to the force.
m 2 2 2 2
\ Fx = (v x1 + v x 2 + v x3 + ..... + v xN )
l
So, pressure on this wall
Force Fx m 2 2 2 2
Px = = 2 = 3 (v x1 + v x2 + vx3 + ...... + vxN )
Area l l
Similarly, on the walls normal to Y and Z axes
m
Py = 3 (v 2y1 + v2y2 + ...... + v 2yN )
l
m 2 2 2
and Pz = 3 (vz1 + vz2 + ...... + vzN )
l
Since molecule are in random motion so they exert same pressure on all the phases of cubical vessel. As the choice of the axes has been
arbitrary, so Px = Py = Pz = P
Px + Py + Pz
Therefore, P =
3
m é(v 2x1 + v 2y1 + v2z1 ) + (v 2x 2 + v 2y2 + v 2z2 ) ù
= ê ú
3l3 ëê + ....(v 2xN + v 2yN + v 2zN ) ûú

mN é v12 + v 22 + ..... + v2N ù Nm 2


P= ê ú = v rms
3l3 êë N úû 3V
Kinetic Theory 331

1 2 1 M 2
Q PV = Nm.vrms or P= v (v2rms = v2 )
3 3 V
1 2 Nm
P = rvrms (Q = r density of gas)
3 V
Relation between Pressure and Kinetic Energy
From kinetic theory of gases,
1 Nm 2 2 1
P= v rms or PV = N ( mv 2rms )
3 V 3 2
æ1 ö 3
\ N ç mv2rms ÷ = PV
è2 ø 2

1
But, mv 2rms = average K.E. of a gas molecule.
2
1 2 3
\ Total K.E. of a gas E = N ( mvrms ) = PV
2 2
3
\ K.E. per unit volume of the gas E = P
2
2
The pressure exerted by a gas is numerically equal to rd of the kinetic energy of the molecules present per unit volume of the gas.
3
DIFFERENT TYPES OF SPEEDS OF GAS MOLECULES
Root Mean Square Speed (R.M.S. speed) vrms
The square root of the average of the squares of the velocities of gas molecules is called the R.M.S. velocity.

v12 + v 22 + ...... + v 2N
v rms = = v2
N
1 M 2 2 3PV
P= v or 3PV = M v 2 or v = [M = mass of gas]
3 V M
3P æ Mö
v2 = r= ÷
r çè Vø

3PV 3P 3RT 3kT


vrms = = = M = (Q R = NA k and Mw = m . NA)
M r w m
(a) For the molecules of a given gas the root mean square velocity is proportional to the square root of its temperature in Kelvin
scale.
T
v rms µ T ; v rms =
Mw
v rms2 T2
For two different temperatures of a gas v =
rms1 T1

v rms2 T2 M w1
(b) For two different gases v = ´
rms1 T1 M w 2
(c) For given temperature lighter the gas, larger the root mean square velocity of the gas molecules.
1
v rms µ
m
(d) If vrms is equal or greater than escape velocity ve then gas will escape from earth or any other planet and so a planet or satellite
will have atmosphere only and only if v rms < ve (= 2gR )
In earth's atmosphere hydrogen molecules acquire this velocity due to elastic collisions with other molecules and escape,
so free hydrogen is not present in atmosphere.
332 Physics
Most Probable Speed vmp
It is the speed which maximum number of molecules in a gas have at constant temperature.
2RT æ 2 ö
v mp = = v rms = 0.816 v rms
M w çè 3 ÷ø

Average Speed vav


It is the arithmetic mean of the speeds of molecules in a gas at a given temperature.
v1 + v 2 + v3 + .... + v N
i.e., v av =
N
and according to kinetic theory of gases,
8RT æ 8 ö
v av = = v rms = 0.92 v rms
pM w çè 3p ÷ø
i.e., vrms > vav > vmp

Kinetic Interpretation of Temperature :


The mean kinetic energy of a molecule is directly proportional to the kelvin (or absolute) temperature of a gas. When the temperature
of the gas is increased, the mean kinetic energy of the molecule increases. On the other hand, when heat is withdrawn from a gas, the
mean kinetic energy of the molecules decreases. Thus, the temperature of a gas is a measure of the mean translational kinetic energy
per molecule of the gas.
Effect of gas temperature on pressure of the gas :
The pressure of a gas is the rate of change of momentum of molecules of gas striking the walls of container per unit area. When the
temperature increases, the root mean square speed of gas molecules also increases. This causes an increase in change in momentum
of the molecules as well as the frequency of collisions. Consequently the increase in temperature results in an increase of rate of
change of momentum of striking molecules per unit area and hence an increase in pressure.

Illustration 1 :
Two gases occupy two containers A and B the gas in A, of volume 0.10m3, exerts a pressure of 1.40 MPa and that in B of volume
0.15m3 exerts a pressure 0.7 MPa. The two containers are united by a tube of negligible volume and the gases are allowed to
intermingle. Then if the temperature remains constant, find the final pressure in the container (in MPa).
Sol. We know that, PAVA = nART, PBVB = nBRT
and Pf (VA + VB) = (n A + nB) RT
Pf (VA + VB) = PAVA + PBVB
æ PA VA + PB VB ö 1.4 ´ 0.1 + 0.7 ´ 0.15
\ Pf = ç V + V ÷ø = MPa = 9.8MPa
è A B 0.1 + 0.15
Illustration 2 :
A litre of air at S.T.P. weighs 1.2 gram. Determine the mass of air required to produce a pressure of 3 atmospheres, in a volume
100 cm³ at –23°C.
Sol. Let us reduce the volume 100 cm³ of air to an equivalent volume corresponding to S.T.P.
P1V1 P2 V2
Using, =
T1 T2

(3atm) (100cm3 ) (1atm) (V2 )


or, =
250K 273K
or, V2 = 327.6 cm³
Since, 1 × 10³ cm³ of air at S.T.P. weighs 1.2g
so, 327.6 cm³ of air at S.T.P. weighs
æ 327.6 cm3 ö
(1.2g) ç 3 3÷
= 0.393 g
è 1 ´ 10 cm ø
Kinetic Theory 333

Illustration 3 :
Calculate the root mean square velocity of the molecules of hydrogen at a temperature of 0°C and a pressure of 76 cm of
mercury column when the density of hydrogen at NTP is 0.00009 g cm–3.
Sol. Pressure, P = 76 cm of mercury column = 76 × 13.6 × 981 dyne/cm2
é 1 ù
Q P = rú v2rms
Density, r = 0.00009 gm/cm3 êë 3 û

3P 3 ´ 76 ´ 13.6 ´ 981
rms velocity, v rms = = cm/s = 1.838 × 105 cm/s.
r 0.0009

DEGREES OF FREEDOM
The number of degrees of freedom of a dynamical system is defined as the total number of co-ordinates required to describe
completely the position and configuration of the system.
For monoatomic gases, the molecule consists of only one atom and can have only translational motion along 3 coordinate axes.
So, it has three degrees of freedom.
The molecules of diatomic gases has two atoms in it. The molecule can have translational motion about its centre of mass. It
can have rotational motion about two axes. So, a diatomic moleculer has three degrees of freedom due to translational motion
and two, due to rotational motion. So, it has five degrees of freedom in total.
A triatomic linear molecule has seven degrees of freedom and a triatomic non-linear molecule has six degrees of freedom.
Degree of freedom (f) in different gas molecules
Molecules Translational Rotational Vibrational
Monatomic 3 0 0
Diatomic 3 2 0
Polyatomic 3 2 (linear molecule) 2
3 (non-linear molecule) 0
LAW OF EQUIPARTITION OF ENERGY
For a dynamical system in thermal equilibrium, the energy of the system is equally distributed amongst the various degrees of
1
freedom and the energy associated with each degree of freedom per molecule is kT
2
1 3
Translational kinetic energy of a molecule , mv2rms = kT
2 2
1 2 1 2 1 2 3
we have mv rmsx + mv rmsy + mv rmsz = kT
2 2 2 2
The molecular motion is a random one and as no direction of motion is a preferred one, the average kinetic energy corresponding to
each degree of freedom is the same.
1 2 1 2 1 2 1
i.e. mvrmsx = mvrmsy = mvrmsz = kT
2 2 2 2
1
Mean kinetic energy per molecule per degree of freedom = kT
2
1
In each degree of freedom, energy of one mole of an ideal gas = RT
T
2
f
Thus, if f be the number of degrees of freedom, the internal energy of 1 mole of the gas will be RT
2
µ
or internal energy of µ moles of the gas will be f RT..
2
SPECIFIC HEAT CAPACITY FOR A GAS
When gases are heated, small change in temperature is accompanied with considerable change in both, volume and pressure. Hence
it is necessary to define two specific heats of gases.
Specific Heat of a Gas at Constant Volume "CV"
It is the quantity of heat required to raise the temperature of unit mass of a gas through 1°C or 1K at constant volume.
334 Physics
Molar specific heat at constant volume "CV" : The amount of heat required to raise the temperature of 1 mole of gas through 1 K
(or 1°C) at constant volume.
It is denoted by Cv.
If Mw is the molecular weight of the gas in gram, then Cv = Mw . cv
At constant volume no external work is done by gas and total heat given to it is utilized for increase in internal energy.
So, dU = µ Cv dT.
Specific Heat of a Gas at Constant Pressure "CP"
It is the quantity of heat required to raise the temperature of unit mass of a gas through 1°C or 1 K at constant pressure.
Molar specific heat at constant pressure “Cp”: The amount of heat required to raise the temperature of 1 gram-mole of gas by 1 K (or
1°C) at constant pressure. It is denoted by Cp
Cp = Mw . cp
The S.I unit of both Cp and Cv : J K–1 mol–1.
If a gas is heated at constant volume, the gas does no work against external pressure. In this case, the whole of the heat energy
supplied to the gas is spent in raising the temperature of the gas.
If a gas is heated at constant pressure, its volume increases. In this case, heat energy is required for the following two purposes :
(i) To increase the volume of the gas against external pressure.
(ii) To increase the temperature of 1 mole of gas through 1 K.
Thus, more heat energy is required to raise the temperature of 1 mole of a gas through 1 K when it is heated at constant pressure than
when it is heated at constant volume.
Hence, Cp > Cv
According to Mayer's formula CP – CV = R

RELATION BETWEEN Cv, Cp AND g FOR IDEAL GAS


Internal energy for µ moles,
f
U = µ RT (where f is the degree of freedom)
2
f dU f
For 1 gm. mole gas U = RT \ = R
2 dT 2
dU 1 1
dU = µ Cv dT ; If µ = 1 mole then C v = so, Cv = fR and hence Cp = Rf + R
dT 2 2
1
Cp f R+R 2 2
Now g = = 2 so, g = 1 + and f =
Cv 1 f g -1
fR
2
R gR
CV = ; CP =
g -1 g -1
5
(1) For a monatomic gas, C v = 3R , C p = 5R and g = = 1.67
2 2 3
5R 7R 7
(2) For a diatomic gas, C v = , Cp = and g = = 1.4
2 2 5
4
(3) For a triatomic or polyatomic gas, Cv = 3R, Cp = 4R and g = = 1.33
3
Specific heat capacity of solids: For one mole of solid, total energy of vibration of its atoms,
æ 1 ö
U = 3 × ç 2 × k BT × N A ÷ = 3k BT × N A
è 2 ø
DQ DU
Now at constant pressure DQ = DU + P DV = DU , since for a solid DV is negligible. Hence, C = = = 3R.
DT DT
DQ DU
Specific heat capacity of water : If we treat water like a solid, then U = 9RT and C = = = 9R
DT DT
Specific heat of all substances approach zero as T ® 0.
As T ® 0, degrees of freedom get frozen and ineffective.
Kinetic Theory 335

MEAN FREE PATH


The distance covered by the molecules between two successive collisions is called the free path.
Mean free path : The average distance covered by the molecules between two successive collisions is called the mean free path.
1 K BT
i.e., l= 2
=
2 . pnd 2pd 2 P
where, n = number of molecules per unit volume
d = diameter of each molecule
KB = Boltzmann’s constant
T = temperature
P = pressure
Mean free path depends on the diameter of molecule (d) and the number of molecules per unit volume n.
At N.T.P., l for air molecules is 0.01 µm.

Illustration 4 :
Calculate Cv for H2, given Cp = 6.85 cal (g mol)–1, density of H2 at N.T.P. = 0.0899 g litre–1 and J = 4.18 × 107 erg cal–1.
Sol. Given, Cp = 6.85 cal (g mol)–1, J = 4.18 × 107 erg cal–1, Density of H2 at NTP = 0.0899 g litre–1
1 103
\ Volume for 1 gm of H2 = = cc
0.0899 0.0899
\ r=
PV 166 ´ 13.6 ´ 980 ´ 103
Now, PV = rT = = 4.1 × 107 erg (g mol)–1°C–1.
T 273 ´ 0.0899
R
Mayer's relation, Cp – Cv = [where, R = universal gas constant = r × molecular mass of H2]
J
\ R = 2 × 4.1 × 10 erg (g mol)–1 °C–1
7

R 2 ´ 4.1 ´ 107
\ Cv = C p - = 6.85 – = 4.87 cal (g mol)–1
J 4.18 ´ 107
Illustration 5 :
Calculate the ratio of specific heat for N2, given that the specific heat at constant pressure is 0.235 and density at N.T.P.
is 1.234 g litre–1.
Sol. Given, Cp = 0.235; Density of N2 at NTP = 1.234 g litre–1
1 103 r
Volume for 1 gm of N2 = litre = cc By Mayer's relation, C p - Cv =
1.234 1.234 J
r 3 ´ 10 6 0.235
\ Cv = C p - = 0.235 – = 0.163 \ Ratio of specific heat = = 1.4
J 4.2 ´ 10 7 0.163
Illustration 6 :
Calculate the values of the principle specific heats of H2 if their ratio is 1.41 and the density of H2 is 0.089 g litre–1
at 0°C and 106 dyne cm–2 pressure [J = 4.18 × 107 erg cal–1]
Sol. Given, g = 1.41 J = 4.18 × 107 erg cal–1
1 10 3
Volume of H2 for unit mass = litre = cc [Q Density = 0.089 gm litre–1]
0.089 0.089
PV 106 ´103
Now, PV = rT \ r= = = 4.11 × 107 erg g–1°C–1
T 273 ´ 0.089
4.11 ´ 10 7
By Mayer's equation, Cp – Cv = r/J = = 0.98 cal g–1 °C–1.
4.18 ´ 10 7
Cp
But, g = = 1.41 \ Cp = 1.41 Cv or, 1.41Cv – Cv = 0.98
Cv
0.98
\ Cv = = 2.39 cal g–1°C–1 \ Cp = 0.98 + Cv = 0.98 + 2.39 = 3.37 cal g–1°C–1
0.41
336 Physics
Kinetic Theory 337

Textbook Exercises
13.1 Estimate the fraction of molecular volume to the actual (b) As the temperature decreases, the deviation from
volume occupied by oxygen gas at STP. Take the ideal behaviour increases.
diameter of an oxygen molecule to be 3Å. Hence as deviation for T2 is more than T1,
Sol. Given, Diameter, d = 3Å, r = d/2 = 1.5Å Hence, T1 > T2
= 1.5 × 10–8 cm PV
(c) Required value of = nR
4 3 T
V = molecular volume = pr N (Where N-Avogadro's
3
number) 10-3 ´ 103
= ´ 8.314 JK–1= 0.26 JK–1
32
4 æ 22 ö
= ´ ç ÷(1.5 ´ 10 -8 ) 3 ´ ( 6.023 ´ 10 23 ) = 8.52cc (d) No, because the no. of moles for hydrogen will be
3 è 7 ø different from that for oxygen.
V¢ = actual volume occupied by 1 mole of O2 at STP The required mass of hydrogen can be found from
= 22400cc m ´ 103
´ 8.314 = 0.26
V 8.52 2
\ = = 3.8 ´ 10-4
V ' 22400 0.26 ´ 2
13.2 Molar volume is the volume occupied by 1 mole of any Þ m= ´ 10 -3 kg = 6.3 × 10–5 kg
8.314
ideal gas at standard temperature and pressure (STP) 13.4 An oxygen cylinder of volume 30 litres has an initial
which are 1 atmospheric pressure and 0°C. Show that gauge pressure of 15 atm and a temperature of 27°C.
it is 22.4 litres.
After some oxygen is withdrawn from the cylinder, the
Sol. For one mole of an ideal gas, PV = RT gauge pressure drops to 11 atm and its temperature
RT drops to 17°C. Estimate the mass of oxygen taken out
\V = [Where, R = 8.31J mol–1K–1,
P of the cylinder. (R = 8.31 J mol–1k–1, molecular mass
T = 273 K, P = 1 atmosphere = 1.013 × 105Nm–2] of O2 = 32cc)
8.31´ 273 Sol. Initially in the O2 cylinder,
= = 0.0224 m3 = 22.4 litres. V1 = 30 litre = 30 × 10–3 m3
1.013 ´ 105
13.3 Figure shows plot of PV/T versus P for P1 = 15 atm = 15 × 1.01 × 105 Pa, T1 = 27 + 273 = 300°K
1.00 × 10 –3 kg of oxygen gas at two different If n1 be the moles of O2 gas in the cylinder,
temperatures. P1V1 = n1RT1
y
(15 ´1.01´105 ) ´ (30 ´10 -3 )
\ n1 = = 18.253
8.3 ´ 300
M = molecular weight of O2 = 32g; Initial cylinder mass
PV (J K–1)
T T1 = m1 = n1M = 18.253 × 32 = 584.1g
T2 Let, n2 – moles of O2 left in the cylinder
P2V2
\ n2 = [Where, V2 = 30 × 10–3m3,
RT2
P x
P2 = 11 × 1.01 × 105 Pa, T2 = 17 + 273 = 290°K]
(a) What does the dotted plot signify ?
(b) Which is true : T1 > T2 or T1 < T2 ? (11´ 1.01´105 )(30 ´ 10 -3 )
= = 13.847
(c) What is the value of PV/T where the curves meet 8.3 ´ 290
on the y-axis ? \ m2 = final mass of O2 in cylinder = 13.847 × 32 = 453.1g
(d) If we obtained similar plots for 1.00 × 10–3 kg of \ Mass of Oxygen taken out of the cylinder.
hydrogen, would we get the same value of PV/T at m1 – m2 = 584.1 – 453.1 = 131g
the point where the curves meet on the y-axis ? If 13.5 An air bubble of volume 1 cm3 rises from the bottom of
not, what mass of hydrogen yields the same value a lake 40 m deep at a temperature of 12°C. To what volume
of PV/T (for low pressure high temperature region does it grow when it reaches the surface, which is at a
of the plot)? (Molecular mass of H2 = 2.02 u, of temperature of 35°C? Given 1 atm = 1.01 × 105 Pa.
O2 = 32.0 u, R = 8.31 J mol–1K–1.) Sol. Given, V1 = 1.0 cm3 = 1 × 10–6 m3,
T1 = 12°C = 285 K, P1 = 1 atm + h1rg
PV
Sol. (a) For dotted plot, = constant which is true for = 1.01 × 105 + 40 × 103 × 9.8 = 493000 Pa
T
Let, V2 = volume of bubble at the surface of the lake
ideal gas.
338 Physics

P1V1 P2V2 3kT


= As vrms =
T1 T2 m
1
PV T \ vrms µ , therefore, at a given temperature, rms
\ V2 = 1 1 2 [Where, T2 = 35°C = 308K, m
T1 P2
speed of molecules will not be the same in the three cases.
P2 = 1atm = 1.01 × 105 Pa] As neon has the smallest mass, therefore, rms speed will
be the largest in case of neon.
( 493000) ´ (1.0 ´ 10 -6 ) ´ 308
= = 5.275 × 10–6m3 13.9 At what temperature is the root mean square speed of
285 ´ 1.01 ´ 10 5 an atom in an argon gas cylinder equal to the rms speed
13.6 Estimate the total number of air molecules (inclusive of of a helium gas atom at – 20°C? (Atomic mass of Ar
O2, N2, water vapour and other constituents) in a room = 39.9 u and He = 4u).
of capacity 25m3 at a temperature of 27°C and 1 atm Sol. Given, M = 39.9, M' = 4,
pressure. [Boltzmann constant = 1.38 × 10–23 JK–1]
T' = – 20 + 273 = 253°K
Sol. Given, V = 25m3, T = 27 + 273 = 300K, k = 1.38 ×
Let C, C' be the rms velocity of Ar and He at temperatures
10–23 JK–1.
T and T' K respectively.
Now, PV = nRT = n(NK)T = N'kT
where, nN = N' = total no. of air molecules in the given 3RT 3RT
gas Now, C = =
M 39.9
PV (1.01 ´ 10 5 ) ´ 25
N'= = = 6.10 × 1026. 3RT ' 3R ´ 253
kT (1.38 ´ 10 - 23 ) ´ 300 and C ' = =
M' 4
13.7 Estimate the average thermal energy of a helium atom
at (i) room temperature (27°C) (ii) the temperature of Since, C = C',
the surface of the sun (6000K) (iii) the temperature of
3RT 3R ´ 253
10 million kelvin (the typical core temperature in the \ =
case of a star). 39.9 4
Sol. (i) Given, T = 27°C = 300K or, T = 2523.7 K
Average thermal energy = 3/2 kT 13.10 Estimate the mean free path and collision frequency of
a nitrogen molecule in a cylinder containing nitrogen
3
= ´ 1.38 ´ 10 - 23 ´ 300 = 6.2 × 10–21J at 2 atm and temperature 17°C. Take the radius of a
2 nitrogen molecule to be roughly 1Å. Compare the
(ii) At T = 6000K; Avg. thermal energy collision time with the time, the molecule moves freely
3 3 between two successive collisions. (Molecule mass of
= kT = ´ (1.38 ´ 10-23 ) ´ 6000 nitrogen = 28u)
2 2
Sol. Given, P = 2 atm = 2 × 1.013 × 105 Nm–2;
= 1.24 × 10–19 J
(iii) At T = 10 million K, T = 107K T = 17°C = 290 K, s = 2 × 1 = 2Å
= 2 × 10–10m
3 k = 1.38 × 10–23 J molecule–1K–1;
Avg. thermal energy = kT
2 M = 28 × 10–23 kg
3
= ´ (1.38 ´10 - 23 ) ´10 7 = 2.1 × 10–16 J. kT
2 Mean free path, l =
2ps2 P
13.8 Three vessels of equal capacity have gases at the same
temperature and pressure. The first vessel contains neon
(monoatomic), the second contains oxygen (diatomic), and 1.38 ´10 -23 ´ 290
=
the third contains uranium hexafluoride (polyatomic). Do 1.414 ´ 3.14 ´ (2 ´ 10-10 ) 2 ´ (2.026 ´10 5 )
the vessels contain equal number of respective molecules? Þ l = 1.11 × 10–7m
Is the root mean square speed of molecules the same in
the three cases? If not, in which case is vrms the largest? 3RT 3 ´ 8.31´ 290
Sol. At same temperature and pressure, all the vessels have vrms = = = 508.24m/s
M 28 ´10 -3
same volume.
By Avogadro's law, three vessels contain equal number Collision frequency = no. of collisions per second =
of molecules 508.24
Avogadro's no. N = 6.023 × 1023 = 4.58 × 109
1.11´10 -7
Kinetic Theory 339

13.11 A metre long narrow bore held horizontally (and closed r2 = diffusion rate of unknown gas = 7.2cm3s–1
at one end) contains a 76 cm long mercury thread, which M1 = molecular wt. of H2 = 2u
traps a 15 cm column of air. What happens if the tube
is held vertically with the open end at the bottom ? M2
\ 28.7 = Þ M2 = 32
Sol. In horizontal position, the bore will leave an air-column 7. 2 2
of 9 cm as shown in figure (a) so as to balance the
atmospheric pressure (= 76 cm of Hg) As M2 = 32, which is the molecular mass of O2 gas hence
15 cm 76 cm the gas is Oxygen.
13.13 A gas in equilibrium has uniform density and pressure
throughout its volume. This is strictly true only if there
are no external influences. A gas column under gravity,
for example, does not have uniform density (and
91 cm 9 cm pressure). As you might expect, its density decreases
Fig (a) with height. The precise dependence is given by the so-
called ‘law of atmospheres’
\ P1 = 76 cm &
Volume of air column = 15 cm3 n2 = n1 exp [ mg(h2 - h1 ) / kBT ] where n2, n1 refer to
assuming area of cross section of tube = 1 cm 3
number density at heights h2 and h1 respectively. Use this
Total length of air column in the tube = 24 cm.
relation to derive the equation for sedimentation
equilibrium of a suspension in a liquid column :
24 + h
n2 = n1 exp[- mg N A (r - P )( h2 - h1 ) /(rRT )] where r is
100 cm

the density of the suspended particle, and r¢ that of


76 – h surrounding medium. [NA is Avogadro’s number, and R
the universal gas constant.]
[Hint : Use Archimedes principle to find the apparent
weight of the suspended paricle.]
Fig (b) Sol. According to Archimedes’ Principle :
In vertical position as shown in figure (b) if h¢ cm of Apparent weight of the suspended particle (mg)
mercury flows out to balance the atmospheric pressure
= Actual wt – (weight of an equal volume of the liquid)
then new pressure on mercury column = 76 – (76 – h)=
h cm of Hg
æm ö
The new volume of air column = 24 + h. (mg)¢ = mg - ç r ' g ÷
Since the temperature remains constant during the èr ø
transition from horizontal to vertical position. (Q Volume of particle = m/r and weight = Vol. × density
\ P1 V1 = P2 V2 × g)
76 × 15 = h × (24 + h)
(mg¢) = mg (1 – r¢/r) ....(1)
or h2 + 24h – 1140 = 0
Þ h = 23.8 cm Given equation is
Hence, 23.8 cm of mercury flows out from the tube in
vertical position. é mg ù
n2 = n1 exp ê - (h2 - h1 )ú ...(2)
13.12 From a certain apparatus, the diffusion rate of H2 has ë kT û
an average value of 28.7 cm3s–1. The diffusion of another Replacing mg by the apparent weight (mg)¢
gas under the same condition is measured to have an
average rate of 7.2 cm3s–1. Identify the gas. From equations (1) and (2), we get
[Hint : Use Graham’s law of diffusion :
R1/R2 = (M2/M1)½, where R1, R2 are diffusion rates of gases é - mgN A æ r ' ö ù
n2 = n1 exp ê çè1 - r ø÷ (h2 - h1 )ú
1 and 2, and M1 and M2 their respective molecular masses. ë RT û
The law is a simple consequence of kinetic theory. ]
(Q h = R/NA)
r M2 This is the required equation.
Sol. According to Graham's law of diffusion, 1 = ,
r2 M1 13.14 Given below are densities of some solids and liquids. Give
rough estimates of the size of their atoms :
where, r1 = diffusion rate of H2 = 28.7 cm3s–1
340 Physics
Substance Atomic Density (10 3 4 3
Mass (u ) –3 Sol. For carbon, Atomic mass = pR ´ density
kg m ) 3

Carbon (diamond) 12.01 2.22 æ 4 3ö


çQ Volume = pR ÷
Gold 197.00 19.32 è 3 ø

Nitrogen (liquid) 14.01 1.00 4


Þ 12.01 × 1.67 × 10–27 kg = × 3.14 × R3 × 2.22 ×
Lithium 6.94 0.53 3
103 kg m–3
Fluorine (liquid) 19.00 1.14 \ R3 = 2.15 × 10–30 m3
Þ R = 1.29 × 10–10 m = 1.29 Å
[Hint : Assume the atoms to be ‘tightly packed’ in a solid
Similarly for Gold , R = 1.59Å
or liquid phase, and use the known value of Avogadro’s
number. You should, however, not take the actual Liquid N2, R = 1.77Å
numbers you obtain for various atomic sizes too literally. Lithium, R = 1.73Å
Because of the crudeness of the tight packing Liquid fluorine, R = 1.88Å
approximation, the results only indicate that atomic sizes
are in the range of a few Å].

Practice Questions
Very Short Answer Questions [1 Mark Qs.] 13. What is the relation between ratio of specific heats g and
degree of freedom f of an ideal gas ?
1. For ideal behavior of a gas, state the condition in terms
14. At what temperature does all molecular motion cease ?
of density of the gas.
15. Compare the mean kinetic energy of one mole of Helium
2. The velocities of three molecules A,B,C of a gas are 3V, and Nitrogen at the same temperature.
4V, and 5V respectively. Calculate the R.M.S velocity? 16. At 0K, which of the following properties of a gas will be zero.
3. The ratio of vapour densities of two gases at the same K.E., P.E., Vibrational energy, Density ?
temperature is 4 : 9. Calculate the ratio of their r.m.s 17. If in an isothermal process, pressure is decreased then what
velocities of molecules. will be its effect on mean free path ?
4. A container filled with Helium gas at 300 K is heated to
600K. The coefficient of thermal expansion of the container Short Answer Questions [2 or 3 Marks Qs.]
is negligible. Calculate the change in the average K.E. of 1. State the postulates of the kinetic theory of gases.
the helium atoms. 2. The rms speed of hydrogen molecule at a certain
5. The pressure of a gas at –173°C is 1 atmosphere, what temperature T is V. If the temperature of the gas becomes
should be the temperature of the gas when the pressure double and the molecule dissociates into atoms, what will
is 2 atmosphere if the volume of the gas is kept constant? be the speed of atomic hydrogen?
6. The absolute temperature of a gas is increased 2 times. 3. When a gas filled in a closed vessel is heated through 2°C,
What will be the increase in r.m.s velocity of the gas its pressure increases by 0.2%. What is the initial
molecules? temperature of the gas.
7. Nitrogen and hydrogen gas are taken at the same 4. The volume of vessel A is 4 times the volume of vessel
temperature. What will be the ratio of the kinetic energies B and both of them are filled with the same gas. If the
of nitrogen and hydrogen molecule when nitrogen is 14 pressure and temperature of gas A is 4 times as that of
times heavier than hydrogen? gas B, then calculate the ratio of no. of molecules of A and
8. 2 cm3 of oxygen and 2 cm3 of nitrogen gas are taken at B.
N.T.P. Which will have larger number of molecules? 5. Derive a relation between pressure and kinetic energy of
9. There are N no. of molecules of a gas in a container. It a gas.
is increased to 2N. Explain, with reason, how the pressure, 6. Show that average K.E. of each molecule of a gas doesn't
total energy and r.m.s speed of the gas will change? depend on the mass of the molecule, but depends on the
10. When a scooter runs for a long time, the air pressure in temperature of the gas.
its tyres increases slightly.
7. Derive Boyle's law on the basis of kinetic theory.
11. On what factors does the average kinetic energy of gas
molecules depend : Nature of the gas, temperature, volume? 8. Derive Charles’ law on the basis of kinetic theory.
12. Obtain the demensional formula for R used in the ideal gas 9. Derive Gay Lussac’s law on the basis of kinetic theory.
equation PV = RT. 10. Derive Avogadro’s law from kinetic theory of gases.
Kinetic Theory 341

11. Two ideal gases at absolute temperature T1 and T2 are 5. Calculate the expression for mean free path of an ideal gas.
mixed. There is no loss of energy. Find the temperature
of the mixture if the masses of the molecules are m1 and Numerical Questions [3 or 5 Marks Qs.]
m2 and the no. of molecules are n1 and n2 respectively. 1. A litre of dry air weighs 1.293 g at S.T.P. Find the temperature
12. Draw P – V curves showing deviations from ideal at which a litre of air will weigh one gram when the pressure
behaviour for a given mass of a gas for two different is 72 cm of mercury.
temperatures.
2. A mass of a gas exerts a pressure of 72 cm of mercury at
13. When air is pumped into a cycle tyre the volume and
27°C. It is heated at constant volume and the pressure after
pressure of the air in the tyre both are increased. What
about Boyle’s law in this case ? sometime is 90 cm of mercury. Calculate temperature.
14. Why temperature less than absolute zero is not possible 3. Find the volume of one gramme of CO2 at 107°C and half of
? the standard pressure when one ml of CO2weighs 0.0019 g
15. A box contains equal number of molecules of hydrogen at S.T.P.
and oxygen. If there is a fine hole in the box, which gas 4. Calculate the r.m.s velocity of air molecules at N.T.P. given
will leak rapidly ? Why? that 100 l of air at N.T.P. weighs 0.129 kg.
16. Two gases, each at temperature T, volume V and pressure 5. At what temperature, pressure remaining unchanged, will
P are mixed such that the temperature and volume of the
the r.m.s velocity of hydrogen be double of its velocity at
mixture are T and V respectively. What would be the
N.T.P ?
pressure of the mixture ? Justify your answer on the basis
of kinetic theory. 6. Calculate kinetic energy of a gram molecule of oxygen at
127°C. k = 1.381 ×10–23 J/k., N = 6.022 × 1023 gm-mol.
17. Calculate the value of the gas constant for one mole of an
ideal gas. Given that one mole of a gas at S.T.P. has a volume 7. Calculate the temperature at which r.m.s velocity of a gas
of 22.4 L. molecule is the same as that of a molecule of another gas at
18. The pressure of a gas increases on increase of its 27°C. Molecular weight of first and second gases are 64 and
temperature and also on decrease of its volume. How are 32 respectively.
these cases different on the basis of kinetic theory of gases? 8. If three molecules have velocities 0.5, 1 and 2 km/s
respectively, calculate the r.m.s speed.
Long Answer Questions [5 Marks Qs.]
9. When a gas filled in a closed vessel is heated through 1°C,
1. Derive an expression for the pressure due to an ideal gas. its pressure increases by 0.4%. What is the initial temperature
2. From kinetic theory of gases, explain kinetic interpretation of the gas ?
of temperature and absolute zero of temperature. 10. The volume of vessel A is twice the volume of another vessel
3. Explain the meaning of mean speed, rms speed and most B and both of them are filled with the same gas. If the gas in
probable speed of a gas. A is at twice the temperature and twice the pressure in
4. Calculate the specific heats of mono, di and triatomic gases camparison to the gas in B, what is the ratio of number of
from the law of equipartition of energy. gas molecules in A and B.

HOTS/ExemplarQuestions

Very Short Answer Questions [1 Mark Qs.] P

1. Will the temperature of gas in a container increase when we


put the container on a moving train? Explain. [HOTS]
1
2. A gas mixture consists of 2.0 moles of oxygen and 4.0 moles
of neon at temperature T. Neglecting all vibrational modes, 2
calculate the total internal energy of the system. (Oxygen
has two rotational modes.) [Exemplar] 3
3. Two molecules of a gas have speeds of 9 × 106 ms–1 and
T2
1 × 106 ms–1, respectively. What is the root mean square 4
T3
speed of these molecules? [Exemplar] 5
T4
Short Answer Questions [2 or 3 Marks Qs.] V

1. The figure here shows five paths traversed by a gas on a 2. When air is pumped into a cycle tyre the volume and pressure
p-V diagram. Rank the paths according to the change in of the air in the tyre both are increased. What about Boyle’s
internal energy of the gas, greatest first. [HOTS] law in this case? [Exemplar]
342 Physics

3. An insulated container containing monatomic gas of molar 3. Consider a rectangular block of wood moving with a velocity
mass m is moving with a velocity v0. If the container is v0 in a gas at temperature T and mass density r. Assume the
suddenly stopped, find the change in temperature. velocity is along x-axis and the area of cross-section of the
[Exemplar] block perpendicular to v0 is A. Show that the drag force on
Long Answer Questions [5 Marks Qs.] kT
the block is 4rAv0 , where m is the mass of the gas
m
1. Although the velocity of air molecules is nearly 0.5 kms–1
molecule. [Exemplar]
yet the smell of scent spreads at a much slower rate. Explain
why? [HOTS]
2. Explain why
(a) there is no atmosphere on moon?
(b) there is fall in temperature with altitude? [Exemplar]

CHAPTER TEST

Time : 45 min. Max. Marks : 24

Directions : (i) Attempt all questions


(ii) Questions 1 to 5 carry 1 mark each.
(iii) Questions 6, 7 and 8 carry 3 marks each.
(iv) Questions 9 and 10 carry 5 marks each.

1. Can you increase the temperature of a gas keeping its pressure and volume constant?
2. The ratio of vapour densities of two gases at the same temperature is 8 : 9. Compare the r.m.s velocities of their molecules.
3. Two cylinders contain helium at 2 atmosphere and argon at 1 atmosphere respectively. If both the gases are filled in one
of the cylinders, then what would be the pressure ?
4. What is average velocity of the molecules of an ideal gas?
5. What are the two factors on which the degrees of freedom of a gas depends ?
6. The pressure of a given mass of a gas is halved at constant temperature. What will be the volume of the gas in comparison
to its initial volume ?
7. On which of the following factors does the average kinetic energy of gas molecules depend ? (i) Nature of the gas, (ii) Absolute
temperature, (iii) Volume. What will be its value at the absolute zero ?
8. As an air bubble rises from the bottom of a lake to the surface, its volume is doubled. Find the depth of the lake. Atmospheric
pressure = 76 cm of Hg.
9. A closed container of volume 0.02 m3 contains a mixture of neon and argon gases at 27°C temperature and 1.0 × 105 Nm–2
pressure. If the gram-molecular weights of neon and argon are 20 and 40 respectively, find the masses of the individual
gases in the container, assuming them to be ideal. (R = 8.314 J mole –1 K –1). Total mass of the mixture = 28 g.
10. What is an ideal gas? Why do the real gases show deviations from ideal behaviour ? Show these deviations graphically.
Kinetic Theory 343

Solutions
PRACTICE QUESTIONS It doesn't depend on the no. of molecules. So, it remains
unchanged.
Very Short Answer Questions 10. On running the scooter, the work done against friction is
1. For ideal gas, P = rRT converted to heat. Q P µ T , so, as the temperature of the
For ideal behavior, r should be low. tyre increases, pressure will also increase.
11. The average kinetic energy of the molecules of a gas
C A2 + CB2 + CC2 depends only on temperature i.e.,
2. C= Eav = f (T).
3
12. Since PV = RT
(3V ) 2 + (4V )2 + (5V )2 50 PV F V [MLT -2 ][ L3 ]
= = V = 4.08 V So R= = or R =
3 3 T AT [ L2 ][ K ]
2 -2 -1
C1 M2 d2 9 3 or R = [ML T K ]
3. = = = =
C2 M1 d1 4 2 2
\ C1 : C2 = 3 : 2 13. g = 1+
f
3 14. Absolute zero.
4. Average K.E. = kT 15. Since mean kinetic energy per mole depends only on
2
\ temperature, hence it will be same for Helium and Nitrogen.
Change in average K.E.
16. Since K.E. µ T and vibrations are excited only at high
3 3 temperature, hence K.E. of vibrational energy will be zero
= k (600 - 300) = ´ 300 × 1.38 × 10–23 Joule = 6.21
2 2 at 0K.
× 10–21 Joule 1
5. When V is constant, P µ T 17. As mean free path l µ [at constant T]
P
P1 P2 Hence on decreasing pressure, l will increase.
\ =
T1 T2 Short Answer Questions
P2 T1 2 ´ (273 - 173) 1. Postulates of kinetic theory : –
Þ T2 = = = 200 K
P1 1 (i) A gas consists of a very large number of molecules
= (200 – 273)°C = – 73°C which are perfect elastic spheres and are identical in
all respects for a given gas and are different for different
6. C µ T \ New r.m.s velocity will be 2C gases.
\ Increase in r.m.s velocity = 2C - C (ii) The molecules of a gas are in a state of continuous,
rapid and random motion.
= ( 2 - 1)C = (1.414 – 1) C = 0.414C (iii) The size of the gas molecules is very small as compared
3 to the distance between them. So, volume of all
7. Q K.E/molecule = kT molecules of the gas is negligible than the volume of
2
\ Both will have the same K.E. and ratio will be 1 : 1. the gas.
8. Both will have the same number of molecules as (iv) The molecules do not exert any force of attraction or
Avogadro's hypothesis says equal volumes of all gases repulsion on each other, except during collision.
contain equal number of molecules.
(v) The collisions of the molecules with themselves and
Q 1 1 mn 2 with the walls of the container are perfectly elastic.
9. Pressure = P = rC 2 = C
3 3 V (vi) Molecular density is uniform throughout the gas.
\ P µ no. of molecules.
As it becomes double, (vii) A molecule moves along a straight line between two
\ Pressure will also be doubled. successive collisions.
(viii) The collisions are almost instantaneous.
1
K.E. = mnC 2
2
3RT
\ It will also become double. 2. R.M.S speed, = C = C = =V
M
R.M.S speed, C µ T
344 Physics

3R2T R
New r.m.s speed = C' = where, k = = Boltzmann's constant.
M /2 N
[Mass of atomic hydrogen is half that of its molecular mass] \ K.E µ T
7. From kinetic theory, pressure due to an ideal gas,
3RT
C' =2 = 2V 1 1M 2 1
M P = rC 2 = C Þ PV = MC 2
\ R.M.S speed becomes double. 3 3V 3
Þ PV µ C2
æ 0.2 ö
3. P2 = ç P + P and T2 = (T + 2)
è 100 ÷ø But C µ T (from kinetic interpretation of temperature)
\ PV µ T
0.2 When T is constant, PV = constant
P1 P2 P+ P
P 100
\ = Þ =
T1 T2 T T +2 \ P µ 1 or V µ 1 when T is constant, which is
V P
P P (1 + .002) Boyle's law.
Þ = Þ T + 2 = 1.002 T
T T +2 8. From kinetic theory, pressure due to an ideal gas,
Þ 2 = 1.002 T – T = 0.002T 1 1M 2 1M 2
P = rC 2 = C ÞV= C
2 3 3V 3 P
Þ T= = 1000 K
0.002 V µ C 2 for a given mass of a gas at constant pressure.
4. VA = 4 VB, TA = 4TB and PA = 4PB
PAV A But C 2 µ T (from kinetic theory)
From ideal gas equation, = n A R and \ V µ T at constant pressure, which is Charles’ law.
TA
1 1M 2
PB VB 9. From kinetic theory, P = rC 2 = C
= nB R 3 3V
TB For a given mass of a gas (i.e. M = constant) and at
n P V T constant volume (V = constant)
Dividing we get, A = A A ´ B ;
nB TA PB V B P µ C2 but C2 µ T (from kinetic theory) \ PµT
which is Gay Lussac's law.
nA 4 ´ 4 10. Consider two gases A and B at same temperature T,
= =4 \ nA : nB = 4 : 1
nB 4 pressure P and having same volume V.
5. From kinetic theory, pressure of a gas Let MA, MB = Total mass of gas A and B; CA, CB = R.M.S
1 velocities of gas A and B
P = rC 2 , where r = density of the gas, Mean K.E/unit mA, mB = mass of each molecule of A and B; nA, nB =
3
1 no. of molecules of A and B respectively.
volume of the gas = E = rC 2 \ MA = nAmA and MB = nBmB
2 From kinetic theory, pressure of gas A and B are,
[Q Mass/unit volume = density = r] \
P 1/ 3rC 2
2 2 1 MA 2 1 MB 2
= = \ P= E P= C A and P = CB
E 1/ 2rC 2 3 3 3 V 3 V
6. Consider one mole of an ideal gas at absolute temperature 1 MA 2 1 MB 2
\ P= CA = CB
T, volume V and molecular weight M. 3 V 3 V
If m be the mass of each molecule and N is the Avogadro's
number then M = m × N Þ M AC A2 = M B CB2 ..... (1)
1 1M 2 Q Both the gases are at same temperature T. \ Average
Pressure, P = rC 2 = C \
3 3V K.E./molecule of both the gases will be equal.
1 1 1
PV = MC 2 \ m AC A2 = mB CB2 Þ m AC A2 = mB CB2
3 2 2
From ideal gas equation PV = RT ..... (2)
1 1 3
\ MC 2 = RT \ MC 2 = RT M AC A2 M B CB2
3 2 2 Dividing (1) by (2), =
1 3R m AC A2 mB CB2
1 3
Þ mNC 2 = RT Þ mC 2 = T
2 2 2 2N nA mA nB mB
Þ = Þ nA = n B
1 mA mB
\ K.E/molecule = mC 2
2 \ All gases contain equal no. of molecules under same
3æ Rö 3 condition of pressure, volume and temperature, which
= ç ÷ T = kT , is Avogadro's law.
è
2 N ø 2
Kinetic Theory 345

3 18. When the volume of a gas is decreased, the space for the
11. Average K.E./molecule of a gas = kT . Before mixing, given number of molecules of the gas decreases and no. of
2
molecules per unit volume increases. So, more moelcules
the average K.E. of the molecules of the two gases =
collide with the walls of the container per second and a
3 3 larger momentum is transferred, to the walls per second.
kn1T1 + kn2T2 , where n1 and n 2 are no. of molecules
2 2 Therefore, pressure of the gas increases.
of gas 1 and 2 respectively.
When the temperature of the gas is increased, the molecules
After mixing, the average K.E. of both the gases =
gain more kinetic energy. Hence they start moving faster
3 and collide more frequently with the walls of the container.
k ( n1 + n2 )T
2 Energy and hence transfer of monentum per second to the
T is the absolute temperature of the mixture. wall of the container increases. Hence, pressure increases.
Q There is no loss of energy. \
3 3 3 Long Answer Questions
k ( n1 + n2 )T = kn1T1 + kn2T2
2 2 2 1. Let us consider, an ideal gas in a cubical container of side
n1T1 + n2T2 a.
\ T = V = volume of the container = a3, n = no. of molecules,
(n1 + n2 )
m = mass of each molecule
12. T1 > T2 Total mass = M = m × n
C1, C2 , C3 = velocities of A1, A2, ....., An
Ideal Gas
P (x1, y1, z1), (x2, y2, z2), ......., (xn, yn, zn) are the components
T1
T1 Real Gas of C1, C2, ......, Cn along x, y, z axes respectively.
T2
Y Y1
T2
C1
V
13. When air is pumped into a cycle tyre, a large number of A1 X1
molecules are pumped in, and Boyle’s law is applicable in
a situation in which the number of molecules remain Z1 A2
constant. C2
14. According to the kinetic interpretation of temperature, Cn
Absolute temperature µ average K.E. of molecules. X
An a
As the heat is removed, the temperature falls and velocity
of molecules decreases. At absolute zero, the molecular
motion eases i.e., the kinetic energy becomes zero. As Z
kinetic energy cannot be negative, so no further decrease
in kinetic energy is possible. Hence temperature cannot be Initial momentum of A1 along OX = mx1;
decreased below 0K. Final momentum of A 1 after collision along
OX = – mx1
1
15. As vrms µ Change in momentum = – mx1 – mx1 = – 2mx1, According
M to law of conservation of mome-ntum. Momentum
Hence hydrogen gas will leak more rapidly because of its transferred by the wall to A1= 2mx1
smaller molecular mass. \ Time between two successive collisions
1M 2 2a 1 x
16. From kinetic theory, P = v = ; No. of collisions/s = = 1
3V x1 t 2a
MT Momentum transferred in one second by
But v 2 µ T \ P µ
V x mx 2
As both T and V remain unchanged but mass M is A1 = 2mx1 ´ 1 = 1
2a a
doubled, so the pressure of mixture gets doubled i.e., it Force exerted by molecule A1 on the wall
is equal to 2P.
17. Volume of 1 mole of a gas at S.T.P. = 22.4 L mx12
= f1 =
= 22.4 × 10–3 m3 a

R=
PV mx22 mx2 mx2
Q \ f2 = , f3 = 3 ,....., f n = n
T a a a
0.76≥13.6≥103 ≥9.8≥ 22.4≥10,3 \ Total force on the wall by all the molecules
=
273 = Fx = f1 + f2 + ......+ fn
= 8.31 J/mole / K
346 Physics
3. Mean speed is the average speed with which a molecule
mx12 mx22 mx 2
= + + ....... + n of a gas moves.
a a a If v 1 , v 2 , ....., v n are the individual speeds of
=
m 2
a 1
(2 2
x + x2 + ..... + xn ) v + v2 + .... + vn
the molecules then, mean speed = v = 1
Fx n
Pressure exerted on the wall = Px = 8kT
a2 = ;
m mp
= 3
( x12 + x22 + ..... + xn2 ) ; r.m.s speed is the square root of the mean of the squares
a

Py =
m
a 3 ( y12 + y22 + ..... + yn2 ) ; of the random velocities of the individual molecules of a
gas.
C12 + C22 + ....Cn2
Pz =
a
m
3 ( z12 + z22 + ..... + zn2 ) Crms =
n
=
3kT
m
Q The molecular density is uniform. Most probable speed of the molecules of a gas is that speed
\ Px = Py = Pz = P Þ Px + Py + Pz = 3P with which the maximum fraction of total no. of molecules
moves.
P + Py + Pz
\ P= x 2kT
3 Most probable speed = v max =
1ém 2 m
P= ê
3 ëa 3 ( m
x1 + x22 + ..... + xn2 + 3
a
) 4. For monoatomic gases, there is only three degree of
freedom.

( y12 + y22 + ..... + yn2 ) +


m
a 3 ( z12 + z22 + ..... + zn2 )ùúû K.E/molecule/degree of freedom =
1
kT (From law of
2
equipartition of energy)
=
m é 2
3a
2
(
2 + x2 + y2 + z2
3 ë x1 + y1 + z1 2 2 2 ) ( )
\
3
kT
For three degrees of freedom, K.E. =
2
(
+ .....+ xn2 + yn2 + zn2 ùû ) \ Total K.E. of 1 gm mole of the gas
3 3
= kT ´ N = RT , where R = kN
=
m
(
C 2 + C2 2 + ........ + Cn 2
3V 1
) 2 2
If dQ is the small amount of heat given to increase the
temperature by dT of 1 gm mole of a gas at constant
mn æ C12 + C22 + ........ + Cn 2 ö M 2 volume, then dQ = 1.Cv.dT
= ç ÷= C
3V è n ø 3V d æ3
dQ ö 3
\ Cv = = çè RT ÷ø = R
Where C = R.M.S velocity dT dT 2 2
Q Cp – Cv = R Þ Cp = Cv + R
C12 + C2 2 + ... + Cn 2 1
= Þ P = rC2 , 3 5
n 3 = R+ R = R
2 2
M C p (5 / 2) R 5
where, = r = density of the gas \ g= = =
V Cv (3 / 2)r 3
2. From Kinetic Theory of gases, Pressure of 1 mole of an For diatomic gases, there are 5 degrees of freedom.
1M 2 \ Energy associated with one gm mole of a diatomic gas
ideal gas = P = C 1 5
3V = E = 5 ´ kT ´ N = RT
2 2
1 [Q PV = RT ] d æ5 ö 5
Þ PV = MC 2 \ Cv =
dQ
= \ Cp =
3 çè RT ÷ø = R
dT dT 2 2
1 3RT 5 7
Þ RT = MC 2 Þ C2 = Cv + R = R + R = R
3 M 2 2
C p (7 / 2) R 7
Þ C2 µ T Þ C µ T \ g= = =
When T = 0, C = 0 Cv (5 / 2) R 5
\ Absolute zero is that temperature at which the r.m.s For triatomic gases with linear molecule, there are seven
speed of the molecules become zero. degrees of freedom.
\ The molecular motion ceases at that temperature. \ Total energy associated with 1 gm molecule of such
Kinetic Theory 347

1 7 1
a gas = 7 ´ kT ´ N = RT = ml
2 2 0.0019
d æ7 ö 7 PV PV PV T
\ Cv = ç RT ÷ = R \ \ 1 1 = 2 2 Þ V2 = 1 1 2
dT è 2 ø 2 T1 T2 T1 P2
7 9
C p =Cv + R = R + R = R 76 ´ (1/ 0.0019) ´ (273 + 107)
2 2 Þ V2 =
273 ´ (76 / 2)
C p (9 / 2) R 9 = 1465.2 ml
\ g= = =
Cv (7 / 2) R 7 1/ 2
For non linear triatomic molecule, there are six degrees of Pressure, P =
1M 2 æ 3PV ö
4. C ÞC = ç
freedom. 3V è M ÷ø
1/ 2
\ Total energy associated with one é 3 ´ (76 ´ 13.6 ´ 980) ´ (100 ´ 1000) ù
gm molecule of such a gas =ê úû
ë 129
1 6 = 48532 cm/s = 485.3 m/s
= 6 ´ kT ´ N = RT = 3RT
2 2
3RT
d 5. As we know, rms speed C =
\ Cv = (3RT ) = 3R M
dT
\ Cp = Cv + R = 3R + R = 4R C t + 273
=
C 4R 4 C' 273
\ g= p = =
Cv 3R 3 t ∗ 273
5. Let us consider a gas possessing n-molecules per unit Þ 2< Þ t = 819°C.
volume. If d is the diameter of each molecule, then the 273
moving molecule will collide with all those molecules whose 3 3
centres be within the distance d from its centre. 6. K.E/gm-mol = NkT = × (6.022 × 1023) × (1.381× 10–23)
2 2
(273 + 127) = 4989.8 J.
2d 7. Here, temperature T = 27 + 273 = 300k
3RT 3R(27 + 273)
vt =
M1 M2
Number of collisions made by a moving molecule in 1 sec
= pd 2vn M2 300
\ Mean free path l Þ =
M1 t + 273
Total distance traversed in 1 sec
= M2 32 1
No of collisions suffered in 1 sec But = =
M1 64 2
v 1
= = 300 1
pd vn pd 2 n
2 [ = Þ 600 = t + 273
Maxwell made correction to the term and found t + 273 2
Þ t = 600 – 273 = 327°C
1
l= . 8. Here, C1 = 0.5 km/s
2pd 2 n
C2 = 1 km/s
Numerical Questions C3 = 2 km/s
1. At S.T.P, Volume of 1 g of dry air C12 + C2 2 + C3 2
r.m.s speed = C =
1000 3
V= cc
1.293
(0.5)2 ∗ 12 ∗ 22
PV PV PV T = = 1.3228 km/s
Q 1 1 = 2 2 Þ T2 = 2 2 1 3
T1 T2 P1V1 9. According to question,
72 ´ 1000 ´ 273 æ 0.4 ö
Þ T2 = P2 = ç P + P÷ and T2 = T + 1
76 ´ (1000 /1.293) è 100 ø
= 334.4 K = 334.4 – 273 = 61.4°C 0.4
P+ P
P2 P1 P 90 P2 100 . But P2 = P
2. Using = Þ T2 = T1 ´ 2 = (273 + 27) = 375 K = Þ =
T2 T1 P1 72 T2 T +1 T2 T
(375 –273)°C =102°C 0.4
P+ P
3. At S.T.P, volume of 1 gm of CO2 [ P 100
= , on solving, T = 250 K
T T +1
348 Physics
10. Here, VA = 2VB; TA = 2TB, PA= 2PB Long Answer Questions
PAV A P V 1. () Gas molecules do not travel uninterrupted. They have
Þ = n A R and B B = nB R
TA TB random motion. The scent vapour molecules undergo
a number of collision and trace a zig-zig path. This is
nA PAV ATB (2 PB ) ´ (2VB ) ´ TB why their effective displacement per unit time is low
\ = = = 2, and spreads is at a much slower rate.
nB PBVB TA PB ´ VB ´ (2TB )
2. (a) The moon has small gravitational force and hence the
\ nA : nB = 2 :1 escape velocity is small. As the moon is in the proximity
of the Earth as seen from the Sun, the moon has the
HOTS/EXEMPLAR QUESTIONS same amount of heat per unit area as that of the Earth.
Very Short Answer Questions The air molecules have large range of speeds. Even
though the rms speed of the air molecules is smaller
1. According to kinetic interpretation of temp., the average than the escape velocity on the moon, a significant
kinetic energy per molecule is proportional to absolute number of molecules have speed greater than escape
temp. But the temp. of a gas is determined by the total velocity and they escape. Now rest of the molecules
translational kinetic energy measured with respect to the arrange the speed distribution for the equilibrium
centre of mass of the gas. Therefore the motion of centre temperature. Again a significant number of molecules
of mass of the gas does not affect the temp. Hence in this escape as their speeds exceed escape speed. Hence,
case there will be no increase or decrease of gas temp. over a long time the moon has lost most of its
2. O2 has 5 degrees of freedom. Therefore, energy per mole atmosphere.
At 300 K
5
= RT
2 3kT 3 ´1.38 ´ 10-23 ´ 300
\ For 2 moles of O2, energy = 5RT v rms = = = 1.7km / s
m 7.3 ´10-26
3
Neon has 3 degrees of freedom \ Energy per mole = RT Vesc for moon = 4.6 km/s
2
(b) As the molecules move higher their potential energy
3
For 4 mole of neon, energy = 4 ´ RT = 6RT increases and hence kinetic energy decreases and
2 hence temperature reduces.
\ Total energy = 11RT. At greater height more volume is available and gas
expands and hence some cooling takes place.

(9 ´106 ) + (1´106 )
2 2 3. n = no. of molecules per unit volume
v 2 + v 22
3. v rms = 1 = vrms = rms speed of gas molecules
2 2 When block is moving with speed v0, relative speed of
molecules w.r.t front face = v + v0

=
( 81 + 1) ´ 1012 = 41 ´ 106 ms -1
Coming head on, momentum transferred to block per
2 collission = 2m(v + v0), where m = mass of molecule.
1
Short Answer Questions No. of collision in time Dt = ( v + v0 ) nDtA , where A =
2
1. The change in temperature is the greatest in case of the area of cross section of block and factor of 1/2 appears due
path (5), hence the change in internal energy of the gas is to particles moving towards block.
maximum in ths case. \ Momentum transferred in time Dt = m(v + v0)2 nADt from
For the paths (1) (2) (3) and (4) taken by the gas the change front surface
in temperature is the same so the change in internal energy Similarly momentum transferred in time Dt = m(v – v0)2
is the same in each case. nADT from back surface
2. When air is pumped, more molecules are pumped in. Boyle’s \ Net force (drag force) = mnA [(v + v0) – (v – v0)2] from
law is stated for situation where number of molecules remain front
constant. = mnA (4vv0) = (4mnAv)v0
1 = (4rAv)v0
Loss in K.E of the gas = DE = ( mn ) v 0
2
3. 1 1
2 2
where n = no. of moles. We also have mv = kT (v - is the velocity along
2 2
If its temperature changes by DT, then x-axis)
3 1 kT
n RDT = mnv 20 Therefore, v =
2 2 m
mv 20 kT
\ DT = Thus drag = 4rA v0
m
3R
Kinetic Theory 349

CHAPTER TEST 9. Let mass of neon gas = mg


1. No, temperature cannot be changed by keeping both Then mass of argon gas = (28 – m) g
pressure and volume constant. m
Number of moles of neon, n1 =
20
C1 M2 r2 9
2. Ratio of rms speed, = = = = 3:2 2
C2 M1 r1 8 (28 - m)
Number of moles of argon, n2 =
40
3. According to Dalton’s law of partial pressure,
P = P1 + P2 = 2 + 1 = 3 atmosphere. Now (P1 + P2)V = (n1 + n2)RT
4. Since the molecules of an ideal gas move in random \ 1 × 105 × 0.02
directions, hence for each molecule moving in a specific
direction we will find a molecule moving in opposite æ m 28 - m ö
= çè + ÷ × 8.314 × (27 + 273)
direction. Hence the average velocity of all molecules is 20 40 ø
zero.
5. The degree of freedom of a gas depends on æ 2m + 28 - m ö
or 2 × 103 = 2494.2 çè ÷ø
(i) atomicity of the gas i.e. monoatomic, diatomic etc and 40
(ii) temperature of the gas.
= 62.355(m + 28)
1M 2 1M 2 1 or m + 28 = 32.07 or m = 4.07 g
6. P= C ÞV = C \V µ
3V 3 P P \ Mass of neon = 4.07 g
é 1 2 ù Mass of argon = 28 – 4.07 = 23.93 g.
êëQ 3 MC = constant at constant temperature úû 10. Ideal gas. A gas which obeys the ideal gas equation,
[ If P becomes P/2, V becomes 2V PV = mRT at all temperatures and pressures is called an ideal
gas. In an ideal gas
7. The average kinetic energy of a gas molecule
(i) the size of the molecule is negligibly small and (ii) there
(i) is independent of the nature of the gas is no force of interaction among the molecules of the gas.
(ii) depends on the temperature of the gas But no real gas possesses these conditions and the
1 3 behaviour of real gas differs from that of an ideal gas. But
Q mC 2 < kT at high temperature and low pressure some real gasses do
2 2 behave like an ideal gas.
1 Graphical representation
[ mC2 µ T
2
(iii) is independent of the volume of the gas.
Ideal Gas
At absolute zero, r.m.s velocity of the gas molecules
becomes zero. PV T1
mT
[ Mean K.E/molecule of the gas becomes zero at absolute T2
zero temperature.
T3
8. When the volume of the air bubble becomes double on
reaching the surface of lake, the pressure becomes half by T1 >T2 > T3
Boyle's law. P
Q Pressure on the surface of the lake = 1 atmospheric PV
pressure. The given figure shows a graph between and P for
mT
[ Pressure of air inside the bubble at the depth of lake = there temperatures T1, T2 and T3, (T1) > (T2) > (T3). From
2 atmospheric pressure = atmospheric pressure + the graph we find that for an ideal gas.
pressure due to column of water h.
PV
[ 2 × 76 cm of Hg = 76 cm of Hg + h cm of water = 8.31 J mol–1 K–1
mT
Þ 76 cm of Hg = h cm of water
But here we find departures from ideal gas behaviour of
Þ Depth of lake = 76 cm of Hg = 76 × 13.6 cm of water three gases. But we find that all of them approach the ideal
= 1033.6 cm = 10.34 m. gas behaviour for low pressures and high temperatures.

¿¿¿
14 Oscillations
C ha p t e r

If a body is given a small displacement from the position, a force comes into play which tries to bring the body back to the equilibrium
point giving rise to oscillations or vibrations.
PERIODIC OR OSCILLATORY MOTION
The motion of a body in which a body moves to-and-fro or back and forth repeatedly about a fixed point in a definite interval
of time is called oscillatory or vibratory or harmonic motion. A periodic motion is a motion of a body which repeats itself over
and over again after a regular interval of time.
A special type of periodic motion in which a particle moves to-and-fro about a mean position is called simple harmonic motion (SHM).
Every oscillatory motion or SHM is periodic but every periodic motion need not be oscillatory.
The motion of the pendulum of a wall clock is an example of oscillatory motion. And circular motion is a periodic motion, but
it is not oscillatory motion.
Harmonic oscillation can be expressed in terms of single harmonic function i.e., y = a sinwt or, y = a coswt
But non-harmonic oscillation cannot be expressed in terms of single harmonic function. It is a combination of two or more than
two harmonic functions i.e., y = a sinwt + b sin2wt.
Time Period and Frequency
Time Period (T) : Time required for one complete oscillation. Its S.I. unit is second.
1
Frequency (v) : No. of oscillations complete in one second i.e., u = . Its S.I. unit is hertz (Hz).
T
SIMPLE HARMONIC MOTION
It is the simplest form of oscillatory motion. When a particle always moves towards the mean position such that its acceleration
is directly proportional to its displacement from mean position but it is directed away from displacement it is said to execute SHM.
i.e., a µ – x.
For simple harmonic motion, the displacement x(t) of the particle from a certain chosen origin or mean position is found to vary
with time(t) as,
Phase

x(t) = A cos (w t + f )

Displacement Amplitude Angular Phase


frequency constant

This equation of motion represents simple harmonic motion.


2p
Angular frequency w = = 2pu where T = time period, u = frequency
T
Restoring Force is the force acting on a particle executing SHM that tries to bring the particle back towards the mean position.
This is directly proportional to negative of displacement.
F µ - x or F = – kx = – mw2x.
k = Restoring force constant = mw2
Phase : It represent the state of the particle (i.e. position and direction of motion) at any instant while executing SHM.
Phase Constant: If the initial position of the particle is not the mean-position of the maximum displacement (A) then a phase
constant f is added to the displacement equation.
x = a cos (wt ± f) or x = a sin (wt ± f)
Amplitude: It is the magnitude of the maximum displacement of the particle in either direction.
Oscillations 351

Displacement
In case of SHM displacement is represented by x = a sinwt or x = a coswt
where ‘a’ is the amplitude or maximum displacement from the mean position and w is angular frequency.
1
Mathematically, we can represent displacement as a function of time as in x = ut + gt2. In case of periodic motion, the displacement
2
is a function of time such that it is periodic as in x = f (t) = a sin wt or x = f (x) = a cos wt.
X X
x = - a cos w t
a a
x = a sin w t
wt wt
a a

Illustration 1 :
A harmonic oscillation is represented by x = 0.34 cos (3000 t + 0.74)
Where x and t are in mm and sec. respectively. Calculate (a) amplitude (b) the frequency and angular frequency, (c) the
period and the initial phase.
Sol. Equation given, x = 0.34 cos (3000 t + 0.74) and also, equation of SHM is x = a cos (wt + f)
Comparing the two equations, we get, amplitude, a = 0.34 mm, angular frequency, w = 3000 Hz.
w 3000 1500
\ Frequency, n = = = Hz; (Q w = 2pn)
2p 2p p
1 p
Time period, T = = s
n 1500
Initial phase, f o = 0.74 radian.

Illustration 2 :
In what time after its motion begins, will a particle oscillating according to the equation, y = 7 sin 0.5 pt move from the
mean position to maximum displacement.
Sol. Given, y = 7sin 0.5 pt
Comparing with the standard eqn. y = a sinwt
a = 7, w = 0.5 p
If t be the time taken by the particle to move from mean position to maximum position
ymax = 7 sin 0.5 pt
ymax = a = 7 \ 7 sin 0.5 pt = 7
p 1
sin 0.5pt = 1 = sin Þ t= = 1s.
2 2 ´ 0.5
Illustration 3 :
A particle executes SHM of amplitude 25 cm and time period 3s. What is the minimum time required for the particle
to move between two points 12.5 cm on either side of the mean position?
Sol. Given: a = 25 cm, T = 3s, x(t) = 12.5 cm.
2pt
x(t) = a sin wt = a sin
T
2 pt 2 pt 12.5 1 p
\ 12.5 = 25 sin Þ sin = = = sin
3 3 25 2 6
1 2 pt p
t= = 0.25s or = from the mean position.
4 3 6
\ Time taken by the particle to move between the two points on either side of mean position
= 0.25 + 0.25 = 0.5 s.
352 Physics
SIMPLE HARMONIC MOTION AND UNIFORM CIRCULAR MOTION
If a particle is moving with uniform speed along the circumference of a circle then the straight line motion of the foot of perpendicular
drawn from the particle on the diameter of the circle is called Simple Harmonic Motion.
Description of SHM based on circular motion :
Draw a circle of radius A equal to the amplitude of the particle performing SHM. Suppose particle is moving with constant angular
velocity w along the circle. Perpendicular from particle position on vertical and horizontal diameter shows SHM. After time t radius
vector turns by wt. Y
So, if the projection of P is taken on y-axis then from the figure
N PAt t = t
2p A
y = a sin wt = a sin t = a sin 2pnt y
T t
X' O X
(i) y = a sin wt when the time is noted from the instant x M At t = 0
when the vibrating particle is at mean position.
(ii) y = a cos wt when the time is noted from the instant
when the vibrating particle is at extreme position. Y'
(iii) y = a sin(wt ± f) when the vibrating particle is f phase leading or lagging from the mean position.
If the projection of P is taken on X-axis then equations of S.H.M. can be given as
æ 2p ö
x = a cos(wt ± f) = a cos ç t ± f ÷ = a cos(2pnt ± f)
è T ø
–a +a –a +a

x = – a sin wt x = a sin wt x = – a cos wt x = a cos wt


Direction of displacement is always taken away from the equilibrium position, particle either is moving away from or is coming
towards the equilibrium position.
VELOCITY AND ACCELERATION IN SIMPLE HARMONIC MOTION
Velocity
Velocity is the rate of change of displacement
dx
Displacement, x = a sin (wt ± f) Þ = v = aw cos(wt ± f)
dt
x2 2 2
v = aw 1 - sin 2 (wt ± f) = aw 1 - = w a -x
a2
vmax = aw at the mean position.
vmin = 0 at the extreme position.
Acceleration
The acceleration is the rate of change of velocity.
dv d
A= = (aw cos wt ) = -w 2 x ; A = 0 at x = 0; amax = – w2a, amin = 0 in mean position or at x = ± a.
dt dt
FORCE LAW FOR SIMPLE HARMONIC MOTION
According to force law for simple harmonic motions when a particle is subject to a force which is proportional to the displacement
of the particle and is directed towards the mean position, the particle executes SHM and the system is called linear harmonic
oscillator.
x m
From this definition, F = ma = – kx Þ - =
a k
4p 2 2p x m
ma = – mw2x = - m x [w = ] \ T = 2p = 2p
T 2 T a k

displacement inertial factor


Time period = 2p or 2p
acceleration spring factor

1 acceleration 1 spring factor


Frequency = or
2p displacement 2 p inertial factor
Oscillations 353

ENERGY IN SHM
K.E. Þ Energy stored in the particle due to its motion i.e. kinetic energy.
P.E. Þ Work done in overcoming restoring force is stored as P.E. i.e. potential energy
x
1 2
W = – F .dx = – ò ( - kx ) dx = kx
2
0
1 2 1 1
P.E. == kx mw 2 a 2 =
sin 2 wt mw 2 x 2 T.E. = Constant
2 2 2
P.E.
1 1 1
K.E. Þ mv 2 =
= mw 2 (a2 - x 2 ) mw 2 a 2 cos 2 wt
2 2 2 K.E.

1 1 1
T.E. = K.E. + P.E. = mw 2 a2 sin 2 wt + mw 2a 2 cos 2 wt = mw 2 a2 a O
2 2 2 a
Total energy of the particle executing SHM is a constant.
Illustration 4 :
A particle executing a SHM has a maximum displacement of 4 cm its acceleration at a distance of 1 cm from its mean
position is 3 cm/s2. What will be its velocity when it is at a distance of 2 cm from its mean position?
Sol. Acceleration at a point = w2y; a = 3 cm/s2, y = 1 cm

3 = w2 × 1 Þ w = 3 rad/s

Velocity at any point, v = w a 2 - y 2

w= 3 , a = 4, y = 2,

\ velocity, v = 3 × 4 2 - 2 2 = 6 cm/s

Illustration 5 :
A small body of mass 100 g is undergoing SHM of amplitude 100 cm and period 0.2 s. What is the max. value of the force
acting on the body? What is the phase difference between the acceleration of the body and its displacement?
Sol. Here, mass m = 100 g = 0.1 kg, a = 100 cm = 1 m, T = 0.20 s.
2
æ 2p ö 4p2 ma
Max. force, F = maw2 = [mass × max. acceleration] Þ F = ma ç ÷ =
èT ø T2
4 ´ (3.14) 2 ´ 0.1´ 1
F= = 98.596 N.
(0.20) 2
Illustration 6 :
æ pö
A body of mass 1 kg is executing SHM given by, x = 6 cos ç 100t + ÷ cm. What is the (a) velocity, (b) acceleration and
è 4ø
(c) maximum kinetic energy?

æ pö dx æ pö
Sol. Given, x = 6 cos ç 100t + ÷ ; Velocity = = – 600 sin ç 100t + ÷ ;
è 4ø dt è 4ø

d2x æ pö
Acceleration = = – 60000 cos ç 100t + ÷
2
dt è 4ø
K.E. is maximum when P.E. is minimum (= 0) and the total energy is constant.

1 1
Max. K.E. = mw2 a 2 = ´1´ (100) 2 ´ (0.06) 2 = 18J.
2 2
354 Physics
SYSTEMS EXECUTING SIMPLE HARMONIC MOTION
Simple Pendulum
A bob of mass ‘m’ suspended at free end of an unstretchable massless string of length ‘l’ while the other end is fixed, constitutes
a simple pendulum.
P

l q
Tm
B A
O
mg sin q mg cos q
mg
The pendulum oscillates about the equilibrium/mean position O under the influence of tangential component of weight which acts
opposite to the displacement of the bob and tries to bring it back to the mean position. Thus for small angles of displacement
(ÐOPA) the motion of simple pendulum can be considered as SHM with time period given by
I l
T = 2p = 2p
mgl g
where, I = ml2 = moment of inertia.
A second's pendulum is a simple pendulum with time period 2 seconds.
Oscillations due to a Spring
Let us find out the time period of a spring-mass system oscillating on a smooth horizontal
surface as shown in the figure. k Smooth
At the equilibrium position the spring is relaxed. When the block is displaced through a m
distance x towards right, it experiences a net restoring force F = – kx towards left.
The negative sign shows that the restoring force is always opposite to the x
displacement. That is, when x is positive, F is negative, the force is directed to the left. k
When x is negative, F is positive, the force always tends to restore the block to its equilibrium m
position x = 0.
F = – kx
Applying Newton’s second law,
d2 x d2 xæ kö
F=m = - kx or +ç ÷ x = 0
dt 2
dt è mø
2

d2 x k m
Comparing the above equation with, a = 2
= -w 2 x we get w 2 = or T = 2p
dt m k
Time period is independent of the amplitude. For a given spring constant, the period increases with the mass of the block that means
more massive block oscillates slowly. For a given block, the period decreases as k increases. A stiffer spring produces quicker
oscillations.

DAMPED SIMPLE HARMONIC MOTION


Amplitude decreases with time and oscillation stops after some time due to dissipative forces like air friction. The total energy
decreases with time.
The equation of a particle undergoing damped oscillations is written as
x (t) = ae–bt/2m cos (w¢t + f)

k b2
where b is damping factor, the angular frequency w ' = -
m 4 m2
1 2 -bt / m
and total energy E = ka e
2
Undamped Oscillations: The amplitude of the oscillation remains same and hence the total energy is constant with time.
FORCED OSCILLATIONS AND RESONANCE
Forced Oscillation
The vibrations in which a body oscillates under the effect of an external periodic force, whose frequency is different from the
natural frequency of oscillating body are called forced oscillations. In forced vibrations the oscillating body vibrates with
the frequency of external force and amplitude of oscillations is generally small.
Oscillations 355

If an external driving force is represented by


F (t ) = F0 cos w d t
The motion of particle is under combined action of (i) restoring force (–kx) (ii) damping force (–bv), and (iii) driving force F(t)
Now, ma = – kx - bv + F0 cos w d t

¶2x kx b ¶x F0 cos w0 t
or =- - +
¶t 2 m m ¶t m
F0 / m
The solution of this equation gives x = x0 sin(w d t + f ) with amplitude x0 =
2
æ bw ö
(w 02 - w d2 ) + ç ÷
è mø

w 02 - w d2 k
tan q = and w 0 = = natural frequency
bw d / m m
Resonant Vibrations
It is a special case of forced vibrations in which frequency of external force is exactly same as the natural frequency of
oscillator. As a result the oscillating body begins to vibrate with a large amplitude leading to the resonance phenomenon to
occur. Resonant vibrations play a very important role in music and tunning of station/ channel in a radio/ TV.
Illustration 7 :
A mass M attached to a spring oscillates with a period of 2 sec.If the mass is increased by 2 kg, the period increases by one
second. Find the initial mass M assuming that Hooke's law is obeyed.
æ Mö
Sol. We know that T = 2p çè ÷ø , where k = spring constant.
k

æ Mö
In first case, 2 = 2p çè ÷ø ... (1)
k

æ M + 2ö
In second case, 3 = 2p çè ÷ ... (2)
k ø
Squaring eq. (1) and (2) and then dividing (2) by (1), we have
9 M+2 2
= = 1+
4 M M
Solving we get, initial mass M = 1.6 kg.
Illustration 8 :
For the damped oscillator, the mass m of the block is 200 g, k = 90 N m–1 and the damping constant b is 40 g s–1. Calculate
(a) the period of oscillation, (b) time taken for its amplitude of vibrations to drop to half of its initial value and (c) the time taken
for its mechanical energy to drop to half its initial value.
Sol. (a) We see that km = 90 × 0.2 = 18 kg N m–1 or kg2 s–2; therefore, km = 4.243 kg s–1, and b = 0.04 kg s–1.
i.e., b is much less than km .

k b2 m 0.2 kg
Hence, the time period T from eq. w ¢ = - is given by T = 2p k = 2p = 0.3 s
m 4m 2 90 Nm -1

(b) Now, from eq. x(t) = Ae - bt / 2m cos (w ¢t + f) , the time, T1/2, for the amplitude to drop to half of its initial value is given by,
ln (1/ 2) 0.693
T1/2 = = ´ 2 ´ 200 s = 6.93 s .
b / 2m 40
(c) For calculating the time, t1/2 for its mechanical energy to drop to half its initial value we make use of eq.
1
E(t) = kA 2 e - bt /m . From this equation we have,
2
1
E(t1/2 ) / E(0) = exp ( - bt1/ 2 / m) or = exp (- bt1/2 / m)
2
1 0.693
ln = -(bt1/2 / m) or t1/ 2 = -1
´ 200g = 3.46 s
2 40gs
356 Physics
Oscillations 357

Textbook Exercises
14.1 Which of the following examples represent periodic Sol. (a) Non-periodic motion
motion? (b) Periodic motion, the particle repeats its entire motion
(a) A swimmer completing one (return) trip from one after an interval of 2 seconds hence period = 2s.
bank of a river to other and back. (c) Non-periodic motion as the position is not retraced
(b) A freely suspended bar magnet displaced from its in the fixed interval.
N-S direction and released. (d) Periodic motion with a period of 2s.
(c) A hydrogen molecule rotating about its centre of 14.4 Which of the following functions of time represent
mass. (a) simple harmonic
(d) An arrow released from a bow. (b) periodic but not simple harmonic, and
Sol. (a) Not a periodic motion as he does not take a definite (c) non-periodic motion ?
time to repeat his motion Give period for each case of periodic motion (w is any
positive constant) :
(b) Is a periodic motion as it oscillates about N-S
(a) sinwt – coswt
direction.
(b) sin3wt
(c) Is a periodic motion.
(d) Not a periodic motion. æp ö
(c) 3cos ç - 2w t ÷
14.2 Which of the following examples represent (nearly) simple è4 ø
harmonic motion and which represent periodic but not (d) cos wt + cos 3wt + cos 5wt
simple harmonic motion?
2 2
(a) the rotation of earth about its axis. (e) e - w t
(b) motion of an oscillating mercury column in a U-tube. (f) 1 + wt + w2t2
(c) motion of a ball bearing inside a smooth curved bowl,
when released from a point slightly above the lower é 1 1 ù
Sol. (a) sin wt – cos wt = 2 ê sin wt - cos wt ú
most point. ë 2 2 û
(d) general vibrations of a polyatomic molecule about its
é p pù
equilibrium position. = 2 êsin wt.cos - cos wt.sin ú
ë 4 4û
Sol. (a) It is periodic but not SHM because it is not a to and
fro motion about a fixed point. æ pö
=2 sin ç wt - ÷
(b) S.H.M. è 4ø
(c) S.H.M. It is an SHM and time period is 2p/w
(d) It is periodic but not S.H.M. 1
14.3 Figure depicts four x- t plots for linear motion of a (b) sin3wt = [3sinwt – sin3wt]
4
particle. Which of the plots represent periodic motion? 2p
What is the period of motion (in case of periodic motion)? It is only periodic but not SHM, T =
w
x
æp ö æ pö
(c) 3cos ç - 2wt ÷ = 3cos ç 2wt - ÷
è4 ø è 4ø
(a)
t(s) [Q cos( -q) = cos q]
x
2p
It is a periodic motion but not SHM, T =
w
(b) (d) cos wt + cos 3wt + cos 5wt
–3 –1 0 1 3 t(s)
= cos wt + cos 5wt + cos 3wt
x
æ wt + 5wt ö æ wt - 5wt ö
= 2cos ç ÷ .cos ç ÷ø + cos 3wt
(c) è 2 ø è 2
1 4 7 10 13 t(s) = 2cos (2 wt).cos (–2wt) + cos 3wt
x = cos 3wt(1 + 2cos 2wt);
It is periodic with T = 2 p /w.
(d)
–3 –2 –1 0 1 2 3
t(s)
358 Physics
2 2 dx
(e) e -w t is an exponential function which does not Velocity, v = = – Aw sin (wt + f)
repeat.
dt
\ w = – Aw sin (p × 0 + f) Þ 1 = – Asin f
\ It is non-periodic.
(f) 1 + wt + w2t2 is also non-periodic. or Asin f = – 1 ... (2)
Squaring and adding (1) and (2)
14.5 A particle is in linear simple harmonic motion between two A2 (cos2 f + sin2 f) = 1 + 1 = 2
points A and B, 10 cm apart. Take the direction from A to
Þ A2 = 2 or A = 2 cm
B as the positive direction and give the signs of velocity,
Dividing (2) by (1),
acceleration and force on the particle when it is
(a) at the end A, 3p 7p
tan f = – 1 or f = or
(b) at the end B, 4 4
(c) at the mid-point of AB going towards A, For x = B sin (wt + a),
(d) at 2 cm away from B going towards A, Solving similarly we get B = 2 cm
(e) at 3 cm away from A going towards B, and p 5p
(f) at 4 cm away from B going towards A. and phase angle, a = or
4 4
Sol. 14.8 A spring balance has a scale that reads from 0 to 50 kg.
S.No. Position of the Sign Sign of Sign of The length of the scale is 20 cm. A body suspended from
particle Velocity Acceler force this balance, when displaced and released, oscillates with
a period of 0.6s. What is the weight of the body?
= ation on the Sol. Here, m = 50 kg, max. extension = y = 20 – 0
particle = 20 cm = 0.2 m, T = 0.6 s
a At the end A 0 (Zero) + + Force = F = mg = 50 × 9.8 N
b At the end B 0 (Zero) – –
F 50 ´ 9.8
At the mid point \ k= = = 2450 N/m
c – (Max) 0 0 y 0. 2
O going towards
A m T 2k (0.6) 2 ´ 2450
At 2 cm from B T = 2p Þ m= = = 22.36 kg
d – – – k 4p 2 4 ´ (3.14) 2
going towards A
Weight of the body = mg = 22.36 × 9.8 = 219.1 N
At 3 cm from A 14.9 A spring having with a spring constant 1200 Nm–1 is
e + + +
going towards B mounted on a horizontal table as shown in Fig. A mass of
At 4 cm from A 3 kg is attached to the free end of the spring. The mass
f + + +
going towards A is then pulled sideways to a distance of 2.0 cm and released.
14.6 Which of the following relationships between the
acceleration a and displacement x of a particle involve
SHM?
(a) a = 0.7 x, (b) a = – 200x2
(c) a = – 10x (d) a = 100 x3
Sol. In SHM, a µ – x
\ (b) and (c) represent SHM. Fig
14.7 The motion of a particle executing SHM is described by Determine (i) the frequency of oscillations, (ii) maximum
the displacement function, acceleration of the mass, and (iii) the maximum speed of
x (t)= A cos (wt + f). the mass.
If the initial (t = 0) position of the particle is 1 cm and Sol. Given, k = 1200 N/m; m = 3kg; a = 2cm = 0.02 m
the initial velocity is w cm/s, what are its amplitude and
1 1 k
initial phase angle? The angular frequency of the particle (i) Frequency = v = =
T 2p m
is ps–1. If instead of the cosine function, we choose the
sine function to describe the SHM : x = B sin (wt + a), 1 1200
= = 3.2 s–1
what are the amplitude and initial phase of the particle 2 ´ 3.14 3
with the above initial conditions. k
Sol. Here, t = 0, x = 1 cm, v = w cm/s, (ii) Acceleration = w2y = y;
m
f = ?; w = p/s ka 1200´ 0.02
Max. acceleration = = = 8 m/s2
x = A cos (wt + f) Þ 1 = A cos m 3
(iii) Max. speed = aw
(p × 0 + f) = Acos f ...(1)
Oscillations 359

k
Obtain the corresponding simple harmonic motions of
1200
= a = 0.02 ´ = 0.4 m/s the x-projection of the radius vector of the revolving
m 3
14.10 In Exercise 14.9, let us take the position of mass when particle P in each case.
Sol. (a) Let the radius vector OA is projected at point B on
the spring is unstreched as x = 0, and the direction from
the diameter of circle and makes an angle q with
left to right as the positive direction of x-axis. Give x initial position P.
as a function of time t for the oscillating mass if at the Thus ÐPOA = q = ÐBAO
moment we start the stopwatch (t = 0), the mass is y
(a) at the mean position,
(b) at the maximum stretched position, and
(c) at the maximum compressed position.
In what way do these functions for SHM differ from each
other, in frequency, in amplitude or the initial phase ? T = 2s O
B x
k 1200 -1 3 cm
Sol. a = 2cm, w = = s = 20s -1 q q
m 3
(a) Since time is measured from mean position, x = a A
sin wt = 2 sin 20t P (t = 0)
(b) At the maximum stretched position, the body is at
In D OBA
p OB = OA sin q
the extreme right position. The initial phase is .
2 or – x = 3sin q = 3sin wt
æ pö 2p 2p
\ x = a sin çè wt + 2 ÷ø = a cos wt = 2 cos 20 t é ù
or x = -3sin t êw = T and T = 2s ú
2 ë û
(c) At the maximum compressed position, the body is
x = – 3 sin pt
3p (b) Let the radius vector OA is projected at point B on
at the extreme left position. The initial phase is .
2 the diameter of circle and makes an angle q with
æ 3p ö initial position.
\ x = a sin ç wt + ÷ = - a cos wt = -2 cos 20t
è 2ø Thus, ÐAOB = q
The function neither differ in amplitude nor in frequency. In, D OBA
The function differ only in initial phase. y
14.11 Following figures correspond to two circular motions.
The radius of the circle, the period of revolution, the initial
position, and the sense of revolution (i.e., clockwise or anti-
clockwise) are indicated on each figure. OB T = 4s
y = cos q P B O
OA x
(t = 0) q
OB = OA cos q
2p A
or – x = 2cos t
T = 2s x 4
3 cm
p ( T = 4)
x = –2cos t
2
14.12 Plot the corresponding reference circle for each of the
P(t = 0)
following simple harmonic motions. Indicate the initial
(a) (t = 0) position of the particle, the radius of the circle,
y and the angular speed of the rotating particle. For
simplicity, the sense of rotation may be fixed to be anti-
clockwise in every case:
(x is in cm and t is in s)
(a) x = –2 sin (3t + p/3)
P 2m T = 4s (b) x = cos (p/6 – t)
x (c) x = 3 sin (2pt + p/4)
(t = 0)
(d) x = 2 cos pt
æ pö æ p pö
Sol. (a) x = -2sin ç 3t + ÷ = 2cos ç 3t + + ÷
è 3ø è 3 2ø
Radius of the reference circle,
(b) r = amplitude of SHM = 2 cm
360 Physics
(d) x = 2 cos pt
p -2 3 Radius of reference circle, r = 2 cm and at t = 0,
At t = 0, x = –2 sin = = - 3 cm
3 2 x = 2 cm
Also wt = 3t \ w = 3 rad/s \ wt = pt, or w = p rad/s
3 cos f0 = 1, f0 = 0
At t = 0 cos f0 = - Þ f0 = 150º The reference circle is plotted below.
2
The reference circle is, thus, as plotted below.
w = 3 rad/s
t=0
150º
t=0
r = 2 cm
- 3 O
2 cm

14.13 Figure (a) shows a spring of force constant k clamped


rigidly at one end and a mass m attached to its free end.
æp ö æ pö A force F applied at the free end stretches the spring.
(b) x = cos ç - t ÷ = cos ç t - ÷
è6 ø è 6ø Figure (b) shows the same spring with both ends free
[Q cos (– q) = cos q] and attached to a mass m at either end. Each end of the
Radius of circle, r = amplitude of spring in Fig. (b) is stretched by the same force F.
SHM = 1 cm
p 3
At t = 0, x = cos = cm k m
6 2
F
3 p
cos f0 = , f0 = -
2 6 (a)
Also wt = 1t Þ w = 1 rad/s
The reference circle is, thus as plotted below m k m
F F
w = 1 rad/s
(b)
O (a) What is the maximum extension of the spring in
30º the two cases?
1 cm t=0 (b) If the mass in Fig. (a) and the two masses in Fig.
(b) are released, what is the period of oscillation in
each case?
æ p pö Sol. (a) Let y be the maximum extension produced in the
(c) x = -3cos ç 2pt + + ÷
è 4 2ø spring in Fig. (a)
æ pö F
= 3sin ç 2pt + ÷ [Q sin q = cos(90° + q)] Then F = ky (in magnitude) \ y =
è 4ø k
Here, radius of reference circle, r = 3 cm and at
In Fig. (b), the force on one mass acts as the force
p 3 of reaction due to the force on the other mass.
t = 0, x = 3 sin = cm
4 2 m
wt = 2pt Þ w = 2p rad/s k O k m
F F
3
2 1 p Therefore, each mass behaves as if it is fixed with
At t = 0; cos f0 = = Þ f0 =
3 2 4 respect to the other.
Therefore, the reference circle is being shown below.
F
Therefore, F = ky Þ y =
k
w = 2p rad/s (b) In Fig. (a), F = – ky
-3
k
2 Þ ma = - ky Þ a = - y
O 45º m
k k
3 cm t=0 \ w2 = i.e., w = [Q a = w2y]
m m
Oscillations 361

Sol. (a) In case of a spring, k does not depend upon m.


2p m However, in case of a simple pendulum, k is directly
Therefore, period T = = 2p
w k m
In Fig. (b), we may consider that the centre of the proportional to m and hence the ratio is a
k
system is O and there are two springs each of length constant quantity.
l (b) The restoring force for the bob of the pendulum is
attached to the two masses, each m, so that K¢ given by
2 F = – mg sin q
is the spring factor of each of the springs. If q is small, then
Then, K¢ = 2k
y mg
sin q = q = \ F=– y
m = 2p m l l
\ T = 2p
K' 2k i.e., the motion is simple harmonic and time period
l
14.14 The piston in the cylinder head of a locomotive has a is T = 2p .
g
stroke (twice the amplitude) of 1.0 m. If the piston moves
with SHM with an angular frequency of 200 rad/min, Clearly, the above formula is obtained only if we
What is its maximum speed? apply the approximation sin q » q. For large angles,
this approximation is not valid and T is greater than
1
Sol. Here, a = m , w = 200 rad/min l
2 2p .
\ vmax = aw = 100 m/min g
100 5 -1 (c) The wristwatch uses an electronic system or spring
= = ms = 1.67 ms-1 system to give the time, which does not change with
60 3 acceleration due to gravity. Therefore, watch gives
14.15 The acceleration due to gravity on the surface of the the correct time.
moon is 1.7 m/s2. What is the time period of a simple (d) During free fall of the cabin, the acceleration due to
pendulum on the surface of moon if its time period on gravity is zero. Therefore, the frequency of
the surface of earth is 3.5 s? (g = 9.8 m/s2) oscillations is also zero i.e., the pendulum will not
Sol. Given: gm = 1.7 m/s2, ge = 9.8 m/s2, vibrate at all.
Tm = ?, Te = 3.5 s 14.17. A simple pendulum of length l and having a bob of mass
M is suspended in a car. The car is moving on a circular
l Tm ge track of radius R with a uniform speed v. If the pendulum
T = 2p \ = ;
g Te gm makes small oscillations in a radial direction about its
equilibrium position, what will be its time period?
9.8 Sol. In this case, the bob of the pendulum is under the action
Tm = 3.5 = 8.4 s. of two accelerations.
1.7
(i) Acceleration due to gravity ‘g’ acting vertically
14.16. Answer the following questions: downwards.
(a) Time period of a particle in SHM depends on the
force constant k and mass m of the particle. v2
(ii) Centripetal acceleration ac = acting along the
R
m horizontal direction.
T = 2p . A simple pendulum executes SHM
k
approximately. Why then is the time period of a \ Effective acceleration, g ' = g 2 + ac2
pendulum independent of the mass of the
2 v4
pendulum? or g ' = g +
(b) The motion of a simple pendulum is approximately R2
simple harmonic for small angle oscillations. For Now time period,
larger angles of oscillation, a more involved
analysis shows that T is greater than l l
T ' = 2p = 2p
g v4
l g2 +
2p . Think of a qualitative argument to R2
g
14.18. A cylindrical piece of cork of density of base area A and
appreciate this result.
(c) A man with a wristwatch on his hand falls from the
height h floats in a liquid of density rl . The cork is
depressed slightly and then released. Show that the cork
top of a tower. Does the watch give correct time
oscillates up and down simple harmonically with a period
during the free fall?
(d) What is the frequency of oscillation of a simple hr
pendulum mounted in a cabin that is freely falling T = 2p
r g
under gravity? l
where r is the density of cork. (Ignore damping due to
viscosity of the liquid).
362 Physics
Sol. Let initially in equilibrium y height of cylinder is inside
the liquid. Then,
Weight of the cylinder = upthrust due to liquid displaced P Q P Q¢
\ Ahrg = Ayrl g y Q 2y
h P¢ h

h
y Consider the mercury contained in a vertical U-tube upto
y´+D y the level r and Q in its two limbs.
Let r = density of the mercury.
L = Total length of the mercury column in both the
limbs.
When the cork cylinder is depressed slightly by Dy and A = internal cross-sectional area of
U-tube.
released, a restoring force, equal to additional upthrust,
m = mass of mercury in U-tube = LAP.
acts on it. The restoring force Assume, the mercury be depressed in left limb to P¢ by
F = A(y + Dy) rl g – Ayrlg = A gDy a small distance y, then it rises by the same amount in
\ Acceleration, the right limb to position Q ¢.
F Arl gDy r l g \ Difference in levels in the two limbs
a= = = . Dy and the = P¢Q¢ = 2y.
m Ahr hr \ Volume of mercury contained in the column of length
acceleration is directed in a direction opposite to Dy. 2 y = A ´ 2y
Obviously, as a a – Dy, the motion of cork cylinder is \ m = A ´ 2y ´ r
SHM, whose time period is given by If W = weight of liquid contained in the column of length 2y.
Then W = mg = A ´ 2y ´ r ´ g
displacement This weight produces the restoring force (F) which tends
T = 2p
acceleration to bring back the mercury to its equilibrium position.
\ F = – 2 Ayrg = – (2Arg)y
Dy = 2p hr If a = acceleration produced in the liquid column, Then
= 2p
a rg
l a=
F
=-
( 2 Arg ) y = g = -2 g y
(Q L = 2h)
14.19. One end of a U-tube containing mercury is connected m LAr 2h
to a suction pump and the other end to atmosphere. A Where h = height of mercury in each limb. Now from eqn.
small pressure difference is maintained between the two (i), it is clear that a a y and –ve sign shows that it acts
columns. Show that, when the suction pump is removed, opposite to y, so the motion of mercury in U-tube is
the column of mercury in the U-tube executes simple simple harmonic in nature having time period (T) given
harmonic motion. by
Sol. The suction pump creates the pressure difference, thus y 2hr hr
mercury rises in one limb of the U-tube. When it is T = 2p = 2p = 2p
a 2rg rg
removed, a net force acts on the liquid column due to
the difference in levels of mercury in the two limbs and h
hence the liquid column executes S.H.M. which can be T = 2p
g
expressed as:

14.20. An air chamber of volume V has a neck area of cross A


section a into which a ball of mass m just fits and can m C
y
move up and down without any friction (Fig.). Show that
when the ball is pressed down a little and released, it D
executes SHM. Obtain an expression for the time period V
of oscillations assuming pressure-volume variations of
air to be isothermal.
Air
Oscillations 363

Sol. Given an air chamber of volume V with a long neck of 14.21. You are riding in an automobile of mass
uniform area of cross-section A, and a frictionless ball 3000 kg. Assuming that you are examining the
of mass m fitted smoothly in the neck at position C. Fig. oscillation characteristics of its suspension system. The
The pressure of air below the ball inside the chamber is suspension sags 15 cm when the entire automobile is
equal to the atmospheric pressure. Increasing the pressure placed on it. Also, the amplitude of oscillation decreases
on the ball by a little amount p, so that the ball is depressed by 50% during one complete oscillation. Estimate the
to position D, where CD = y. Then there will be a decrease values of (a) the spring constant k and (b) the damping
in volume and hence increase in pressure of air inside the constant b for the spring and shock absorber system of
chamber. The decrease in volume of the air inside the one wheel, assuming that each wheel supports 750 kg.
chamber = DV = Ay [g = 10 m/s2].
Sol. (a) Here, mass, M = 300 kg, displacement,
changein volume
Volumetric Strain = x = 15 cm = 0.15 m, g = 10 m/s2.
original volume There are four spring systems. If k is the spring
DV Ay constant of each spring, then total spring constant
=
=
V V of all the four springs in parallel is
\ Bulk Modulus of elasticity E, will be K p = 4k \ M g = k p x = 4kx
stress ( or increase in pressure )
E= Mg 3000 ´ 10
volumetric strain Þ K= = = 5 ´ 104 N
4x 4 ´ 0.15
-p - pV (b) For each spring system supporting
= =
Ay / V Ay 750 kg of weight,
Here, negative sign shows that the increase in pressure
will decrease the volume of air in the chamber. m 750
T = 2p = 2 ´ 3.14 ´ = 0.77 sec.
k 5 ´ 104
- E Ay
Now, p=
V -
bt
Due to this excess pressure, the restoring force acting \ Using x = x0 e 2m ,
on the ball is we get
- E Ay - E A2 b ´ 0.77 0.77b
F = p´ A = .A = y ...(i) 50 -
V V x0 = x0 e 2´750 or e 1500 = 2
Since F µ y and negative sign shows that the force is 100
directed towards equilibrium position. If the applied Taking logarithm of both sides,
increased pressure is removed from the ball, the ball will 0.77b
start executing linear SHM in the neck of chamber with = log e 2 = 2.303 log 2
1500
C as mean position.
In SHM, the restoring force, 1500
\ b= ´ 2.303 ´ 0.3010
F = – ky ...(ii) 0.77
Comparing (i) and (ii), we have = 1350.4 kg s–1
Spring factor, k = EA2/V 14.22. Show that for a particle in linear SHM the average
Here, inertial factor = mass of ball = m. kinetic energy over a period of oscillation equals the
average potential energy over the same period.
inertial factor Sol. Let the particle executing SHM starts oscillating from its
Time Period, T = 2 p
spring factor mean position. Then displacement equation is
x = A sin wt
m 2p mV \ Particle velocity, v = Aw cos wt
= 2p = \ Instantaneous K.E.,
2 A E
EA / V
1 1
K = mv 2 = mA2 w2 cos 2 wt
1 A E 2 2
\ Frequency, v = = \ Average value of K.E. over one complete cycle
T 2p mV
T
If the ball oscillates in the neck of chamber under 1 1 2 2
K av = ò 2 mA w cos 2 wt dt
isothermal conditions, then E = P = pressure of air inside T
0
the chamber, when ball is at equilibrium position. If the
T
ball oscillates in the neck of chamber under adiabatic mA2 w 2 2
conditions, then E = gP, where g = Cp/Cv. =
2T ò cos wt dt
0
364 Physics

mA2 w2
T
(1 + cos 2wt ) dt I 4p 2 I
Þ a= 2
=
2T ò 2
T = 2p
a T
0
2
mA2w2 é sin 2wt ù
T æ 22 ö 1 10 ´ (0.15)2
= = 4´ç ÷ ´ ´ = 1.97 Nm/rad.
4T êt + 2w ú è 7 ø 2 (1.5) 2
ë û0
14.24 A body describes SHM with an amplitude of 5 cm and
mA2 w 2 é æ sin 2 wT - sin 0 ö ù a period of 0.2 s. Find the acceleration and velocity of
= ê(T - 0) + çè ÷ø ú the body when the displacement is (a) 5 cm, (b) 3 cm,
4T ë 2w û
(c) 0 cm
1 Sol. Given: a = 5 cm = 0.05 m, T = 0.2 s,
= mA2 w2 ...(i)
4 2p
Again instantaneous P.E., w= = 10p rad/s
T
1 1 1 (a) When y = 5 cm = 0.05 m [a = – w2 y]
U = kx 2 = mw2 x 2 = mw2 A2 sin 2 wt
2 2 2 a = – (10p)2 × 0.05 = – 5p2 m/s2;
\ Average value of P.E. over one complete cycle
T
v = 10p ( 0.05) 2 - (0.05) 2 = 0
1 1
U av = ò mw 2 A2 sin 2 wt
T
0
2 v = w a2 - x2
(b) When y = 3 cm = 0.03 m
T a = – (10p)2 × 0.03 = – 3p2 m/s2; v = 10p
mw 2 A2
sin 2 wt dt
2T ò
=
0 ( 0.05) 2 - (0.03) 2 = – 0.4p m/s
(c) When y = 0
mw 2 A2
T
(1 - cos 2wt ) dt
=
2T ò 2 a = – w2 y = 0; v = 10p × (0.05) 2 - 0
0
T
= 0.5p m/s.
mw2 A2 é sin 2wt ù 14.25. A mass attached to a spring is free to oscillate, with
= êt - 2w ú
4T ë û0 angular velocity w, in a horizontal plane without friction
or damping. It is pulled to a distance x0 and pushed
mw 2 A2 é (sin 2wT - sin 0) ù towards the centre with a velocity v0 at time t = 0.
= ê( T - 0 ) - ú Determine the amplitude of the resulting oscillations in
4T ë 2w û terms of the parameters w, x0 and v0.
1 [Hint : Start with the equation x = a cos
= mw2 A2 ...(ii) (wt + q) and note that the initial velocity is negative.]
4
Simple comparison of (i) and (ii), shows that Sol. x = a cos (wt + q)
1 dx
K av = U av = mw 2 A2 v= = - aw sin ( wt + q )
4 dt
14.23 A circular disc of mass 10 kg is suspended by a wire dx
attached to its centre. The wire is twisted by rotating When t = 0, x = x0 and = - v0
dt
the disc and released. The period of torsional \ x0 = a cos q ...(i)
oscillations is found to be 1.5 s. The radius of the disc and – v0 = – a w sin q
is 15 cm. Determine the torsional spring constant of the v0
wire. or a sin q = ...(ii)
w
(Torsional spring constant a is defined by the relation
Squaring and adding (i) and (ii), we get
J = – aq, where J is the restoring couple and q the angle
v02
( )
of twist).
Sol. Here, mass m = 10 kg, R = 15 cm = 0.15 m, T = 1.5 s, a 2 cos2 q + sin 2 q = x02 +
a=? w2
1
mR 2 v02
Moment of inertia of the disc = or a= x02 +
2 w2
1
= ´10 ´ (0.15) 2
2
Oscillations 365

Practice Questions
Very Short Answer Questions [1 Mark Qs.] 2. Why soldiers are asked to break their steps while crossing
a temporary bridge of rope?
1. Which of the following conditions is not sufficient for
3. The length of a seconds pendulum on the surface of earth
SHM and why?
is 100 cm. What will be the length of a seconds pendulum
(a) Acceleration µ displacement
on the surface of moon?
(b) Restoring force µ displacement
4. The maximum acceleration of a simple harmonic oscillator
2. A particle is executing SHM. Identify the positions of the is a0 and maximum velocity is v0.What is its amplitude?
particle where,
5. The bob of a vibrating simple pendulum is made of ice.
(i) K.E. of the particle is zero
How will the period of swing change, when the ice starts
(ii) P.E. is zero melting?
1 6. What are the free, damped and maintained oscillations.
(iii) P.E. is th of total energy
4 Give examples.
(iv) P.E. and K.E. are equal 7. The periodic time of a mass suspended by a spring of force
3. How will the time period of a simple pendulum change constant K is T. If the spring is cut into three equal parts,
when its length is doubled? what will be the force constant of each part? If the same
4. Will a pendulum gain or lose time when taken to the top mass be suspended from one piece, what will be its periodic
of a mountain? time?
5. Why a point on a rotating wheel cannot be considered as 8. Two simple harmonic motions are represented by:
executing SHM?
6. A spring of force constant k is broken into n equal parts æ pö
x1 = 10 sin ç 4 pt + ÷ and x2 = 5(sin 4pt + 3 cos 4pt ) .
(n > 0). What will be the spring factor of each part? è 4ø
7. Two clocks, one working with oscillating pendulum and
What is the ratio of the amplitudes?
the other with spring are given. Which one will give correct
time in a satellite? 9. Find the ratio of the frequencies in the following
8. How would the period of a spring mass system change arrangement of springs:
when it is made to oscillate horizontally and then vertically?
9. Two simple pendulum of equal lengths cross each other
from opposite directions at mean position. What is their k1
phase difference?
10. When is the tension maximum in the spring of a simple k1 k2
pendulum?
(a) (b) k2
11. m
k1 k2 m
m
In the above arrangement, if the block of mass m is 10. The amplitude of an oscillating simple pendulum is
displaced, what is the frequency of oscillation? doubled. What will be its effect on the (a) periodic time,
12. Two identical springs of springs constant k are attached
(b) total energy and (c) maximum velocity .
to a block of mass m and to fixed supports as shown in
Fig. When the mass is displaced from equilibrium position 11. A block rests on a horizontal table which is executing SHM
by a distance x towards right,find the restoring force in a horizontal plane with an amplitude A. What will be
the frequency of oscillation when the block will just start
m to slip? Coefficient of friction = µ.
k k
12. A simple pendulum performs SHM about x = 0 with an
amplitude a and time period T. What is the speed of the
13. Show that for a particle executing S.H.M. velocity and pendulum at x = a/2?
displacement have a phase difference of p/2. 13. Define force constant of a spring. Give its S.I unit.
14. What is the ratio of maximum acceleration to the maximum 14. What is a simple pendulum? Find an expression for the time
velocity of a simple harmonic oscillator? period and frequency of a simple pendulum.
Short Answer Questions [2 or 3 Marks Qs.] 15. Define an expression for the time period of a SHM in terms
inertia factor and spring factor.
1. A girl swinging suddenly stands up on the swing. What 16. A particle is executing S.H.M in a straight line. When the
is the influence on the time period and frequency? distance of the particle from equilibrium position has values
366 Physics
x1 and x2, the corresponding velocities are u1 and u2. Show 4. A body oscillates with S.H.M. according to the equation, x
that the time period of vibration is = 6 cos (3pt + p / 3) m. What is (a) amplitude and (b) the
éx 2 -x 2 ù
1/ 2 velocity at t = 2s ?
T = 2p ê 2 1
ú 5. A body oscillates with S.H.M., according to the equation x
2 2
ëê 1
u - u 2 ûú pù
é
= (5.0 m) cos ê (2 p rad / s )t + ú
ë 4û
Long Answer Questions [5 Marks Qs.]
At t = 1.5s, calculate the (a) displacement,
1. What is the effect on the time period of a simple pendulum (b) speed and (c) acceleration of the body.
when : 6. When the displacement is one half of the amplitude, what
(i) it is immersed in a liquid of density s. fraction of the total energy is kinetic and what fraction is
(ii) the temperature of wire through which the bob is potential in SHM.? At what displacement is the energy half
suspended is increased? kinetic and half potential ?
2. Derive the differential equation of Simple Harmonic 7. A particle starts oscillating simple harmonically from its
Oscillation. equilibrium position. What is the ratio of K.E. and P.E. of the
3. Discuss the effect of driving force frequency on the driven particle at time T/12s?
frequency. How does friction affect the oscillation of an 8. A particle executes SHM with an amplitude
oscillator? 4 cm. Find the position of a point where its speed is half its
4. Draw displacement-time, velocity-time and acceleration-time maximum speed. At what displacement is potential energy
graphs for a particle executing simple harmonic motion. equal to kinetic energy?
Discuss their phase relationship. 9. A spring is of spring constant K = 15 N/cm. It is cut into 3
5. Find the time period and frequency of oscillations of a equal parts, which are joined in parallel. What is the spring
cylinder floating in a liquid. constant of the combination?
6. Derive expressions for displacement, velocity, acceleration 10. A force of 1 N is required to stretch a spring by 1.5 cm. If the
and time period in SHM. Also find the expressions for K.E, spring is cut into three equal parts, find the force required
P.E and total energy. to stretch one part by 3 cm.
Numerical Questions [3 or 5 Marks Qs.] 11. Two springs of equal lengths and equal cross sectional
area are made of materials whose Young's moduli are in the
1. A particle executes S.H.M of period 12 seconds and ratio of 2 : 3. They are suspended and loaded with the same
amplitude 8 cm. Find the time it takes to travel 3 cm from the mass. When stretched and released they oscillate. Find the
positive extremity of its oscillation. ratio of the time period of oscillation.
2. A body is vibrating with S.H.M of amplitude 15 cm and 12. If the length of a second's pendulum is increased by 1%
frequency 4 Hz. Compute the maximum values of the how many seconds will it lose or gain in a day?
acceleration and velocity. 13. A pendulum 1m long makes 20 vibrations in 40 s. Find the
3. A particle is executing S.H.M given by time taken to make 30 vibrations, if its length is increased to
4 m.
2pt
x = 5 cos æç + j÷
ö 14. A particle is executing SHM according to the equation x = 5
è T ø sin pt where x is in cm. How long will the particle take to
The period of vibration is 20s. At t = 0, the particle is move from the equilibrium position to the position of
displaced + 2 units. Determine (i) its initial phase and (ii) the maximum displacement?
phase angle corresponding to displacement of + 3 units.

HOTS/Exemplar Questions
Very Short Answer Questions [1 Mark Qs.] Short Answer Questions [2 or 3 Marks Qs.]
1. A grandfather clock depends on the period of a pendulum 1. A horizontal platform executing S.H.M. up and down about
to keep correct time. Suppose a grandfather clock is a mean position. It's time period of oscillation is 2 p seconds.
calibrated correctly and then a mischievous child slides the A mass m is resting on it. Determine the greatest amplitude,
bob of the pendulum downward on the oscillating rod. Does the platform can have so that the mass resting on it may not
the grandfather clock run (a) slow, (b) fast, or (c) correctly? leave it ? [HOTS]
2. A body of mass m is situated in a potential field
[HOTS] U(x) =U0 (1 – cos ax) when U0 and a are constants. Find the
2. What are the two basic characteristics of a simple harmonic time period of small oscillations. [Exemplar]
motion? [Exemplar] 3. Consider a pair of identical pendulums, which oscillate with
3. When will the motion of a simple pendulum be simple equal amplitude independently such that when one pendulum
harmonic? [Exemplar] is at its extreme position making an angle of 2° to the right
Oscillations 367

with the vertical, the other pendulum makes an angle of 1° to 2. A cylinderical log of wood of height h and area of cross-
the left of the vertical. What is the phase difference between section A floats in water. It is pressed and then released.
the pendulums? [Exemplar] Show that the log would execute S.H.M. with a time period.
[Exemplar]
Long Answer Questions [5 Marks Qs.]
m
1. A solid ball of mass m is made to fall from a height H on a T = 2p
Arg
pan suspended through a spring of spring constant k. If the
where m is mass of the body and r is density of the liquid.
ball does not rebound and the pan is massless, find the
3. A tunnel is dug through the centre of the Earth. Show that a
amplitude of oscillation. [HOTS]
body of mass ‘m’ when dropped from rest from one end of
the tunnel will execute simple harmonic motion. [Exemplar]

CHAPTER TEST

Time : 45 min. Max. Marks : 24

Directions : (i) Attempt all questions


(ii) Questions 1 to 5 carry 1 mark each.
(iii) Questions 6, 7 and 8 carry 3 marks each.
(iv) Questions 9 and 10 carry 5 marks each.

1. Can the motion of a satellite round a planet be taken as SHM ? Why.


2. What is the frequency of variation of P.E. or K.E. when the frequency of oscillation is f ?
3. Displacement versus time curve for a particle executing SHM is shown in Fig. Identify the points marked at which
(i) velocity of the oscillator is zero, (ii) speed of the oscillator is maximum.
displacement

C E G
A B D F H time (s)

4. Two simple pendulums of equal length, cross each other at mean position. What is their phase difference?
5. A simple pendulum is inside a space craft. What should be its time period of vibration?
6. Two exactly indentical pendulums are oscillating with amplitudes 2 cm and 6 cm. Calculate the ratio of their energies of
oscillation.
7. All trigonometric functions are periodic, but only sine or cosine functions are used to define SHM Why?

8. A body executing SHM makes 100 vibrations per minute. If its speed at mean position is 15 m/s, calculate its path length.
What is its velocity when it is half way between its mean position and extreme position.
9. Find the time period and frequency of the oscillations of a ball in the neck of an air chamber.
10. One end of a V-tube containing mercury is connected to a suction pump and the other end to atmosphere. The two arms of
the tube are inclined to horizontal at an angle of 45º each. A small pressure difference is created between two columns when
the suction pump is removed. Will the column of mercury in V-tube execute simple harmonic motion? Neglect capillary and
viscous forces. Find the time period of oscillation.
368 Physics

Solutions
PRACTICE QUESTIONS 1 k1 + k 2
f =
2p m
Very Short Answer Questions 12. 2kx towards left.
1. Condition (a) is not sufficient as the direction of 13. Let the displacement of a particle executing SHM be x =
acceleration is opposite to displacement and that needs a sin wt
to be mentioned. dx
the particle velocity will be = aw cos wt
2. (i) At both the extreme positions dt
(ii) At mean position æ pö
= aw sin ç wt + ÷
è 2ø
1
(iii) P.E. = m w2 ( a 2 - x 2 )
4 p
Thus there is a phase difference of between particle
2
1 1
T.E. = mw2 a 2 Given, P.E. = T.E. displacement and velocity.
2 4
14. + w, For a particle executing SHM maximum velocity
1 1 v max = aw
mw 2 ( a 2 - x 2 ) = m w 2 a 2
2 2 and maximum acceleration amax = aw2
3
Þ x= a amax
2 \ =w
vmax
1
(iv) K.E. = mw2 x 2 , P.E. = K.E.
2 Short Answer Questions

1 1. The girl can be considered as an extended body. As the


1 2 2 2
Þ mw2 x 2 = mw (a - x ) girl stands up on the swing the separation 'd' between the
2 2 point of suspension and the centre of gravity decreases

a since the time period is inversely proportional to d , time


Þ x=
2 period increases and frequency decreases
2. Resonance may occur where the frequency of vibration
l of the stepping of soldiers becomes equal to that of the
3. T = 2p When l is doubled, T will increase 2 time. bridge, so the bridge will vibrate with larger amplitude and
g
may collapse.
4. On the top of a mountain, acceleration due to gravity 'g'
decreases. l
3. Time period, T = 2p
\ Time period increases. So it loses time. g
5. It is not a to-and-fro motion about a fixed point. So, it is On moon's surface g' = g/6
only periodic but not oscillatory. \ l¢ = l/6 = 1/6 m [l = 100 cm = 1 m]
4. v0 = Aw, a0 = w2A = v0w Þ w = a0/v0
6. The spring factor of each part is nk.
\ A = v0/w = v0/(a0/v0) = v02/a0
7. The clock with spring because the simple pendulum will 5. Time period remains unchanged till the centre of gravity
1 of the bob remains fixed from the point of suspension. If
have infinite Time period in space as T µ and centre of gravity goes up after melting of ice, length of
g
pendulum decreases and the time period also decreases.
g = 0 in the satellite. If center of gravity goes down, time period increases.
8. Time period of a spring is independent of 'g'. So, no 6. Refer to theory.
change will take place. 7. When it is cut into 3 parts it can be considered as a
combination of 3 springs in series, each of spring
9. p radian
constant k.
10. At the mean position.
1 1 1 1
11. Since restoring force is equal on both the springs, \ Effective spring constant, = + +
K k k k
frequency of the system
Oscillations 369

k
Þ K= \ k = 3K
3 l
q
T
m m 3m
T = 2p = 2p = 2p
K k /3 k
x
Time period of each individual spring, T1 = 2p
m mg sin q mg cos q
k mg
T1 1 T
\ = Þ T1 = Restoring force = F = – mg sin π Þ sin π
T 3 3
8. Amplitude of x1 = 10, Amplitude of x
=π = [for small π , sin π » π ]
l
x2 = 5 2 + (5 3 ) 2 = 10; [ F = – mg π = – mgx/l Þ F = – kx
Ratio of amplitudes = 1 : 1
mg inertial factor
9. (a) Restoring force = – k1x – k2x; \ k= \ T = 2p
l spring factor
1 k1 + k 2
keq = k1 + k2 \ f =
2p m m l
T = 2p = 2p
(b) Here, extensions are different. Total extension = x mg / l g
= x1 + x2
1 1 g
1 1 1 frequency = f = =
= + T 2p l
keq k1 k2 15. Acceleration of a body executing S.H.M
1 k1k 2 = A = – w2 x
\ f =
2p m(k1 + k 2 ) Restoring force = mA = –m w2 x
Also F = kx
l 1
10. Time period = T = 2p ; Total energy = E = mw 2 a 2 [ kx = m w2 x
g 2
and vmax = aw k
[ w2 = k/m Þw=
When the amplitude is doubled m
(a) There is no change in T as it is independent of
amplitude. 2p m inertial factor
\ T= = 2p = 2p
w k spring factor
(b) E µ a2 \ It will become 4 times.
(c) vmax µ a \ It will become double. 16. using, v 2 = w2 (a 2 - y 2 )
11. The block will start to slip when
Restoring force on the block = mw2a = µmg [ u12 = w2 (a 2 - x12 ) and u22 = w2 (a 2 - x12 )
Acceleration of the block = µg
Subtracting, u12 – u22 = w2 (x22 – x12)
\ Frequency of oscillation
1 acceleration 1 µg
= n= Þ n= 4p 2
2p displacement 2p A = (x22 – x12)
T2
12. Speed = v = w a 2 - x 2 1/ 2
é 2 2ù
2p 2 3ap [ Time period of vibration, T = 2p ê x2 - x1 ú
= a - a2 / 4 = 2 2
ëê u1 - u2 ûú
T T
13. Force constant of a spring is defined as the force required
Long Answer Questions
to produce unit extension or compression in the spring.
S.I unit is N/m 1. (i) Let r = density of the material of the pendulum bob.
14. A simple pendulum consists of a heavy point mass s =density of liquid in which the pendulum bob is
suspended by a weightless, inextensible, perfectly flexible immersed such that r > s. Let it is made to oscillate
string from rigid support about which it can oscillate freely. with a time period T. The effective value of ‘g’ in the
liquid is given by
æ sö æ r - sö
g ' = g ç1 - ÷ = g ç
è rø è r ÷ø
370 Physics

l l dx (t )
T ' = 2p law of motion of x (t) is the displacement then is the
\ = 2p (r - s) dt
g' g
r d 2 ( x )t
velocity and is the acceleration if the oscillating
r æ l ö dt
=T çQ T = 2p g ÷ particle. Now
r-s è ø
Force = mass × acceleration
where T = time period of the simple pendulum when
is oscillates in air. d 2 x(t )
so F = m = - k x (t )
T' r dt 2
or =
T r-s
d 2 x (t ) k
\ r > s or r – s > 0 or 2
=– x (t )
dt m
r T'
\ >1 or >1 or T ¢ > T
r-s T d 2 x(t ) k
or + x(t ) = 0 ...(i)
i.e. its time period increases. dt 2 m
(ii) When the temperature of the wire with which of bob of
pendulum is suspended increases, the length of wire d 2 x (t ) 2
or + w x (t ) = 0 ...(ii)
increases due to its thermal expansion. As T µ l , dt 2
Equation (i) and (ii) are known as the differential equations
so T will also increase as shown below.
of simple harmonic oscillator, w is the angular velocity given
Let l = original length of the wire by
d q = rise in its temperature
2p
a = coefficient of linear expansion of the material of w= = 2pv
the wire T
If l’ = new length k
Also, w =
and dl= increase in its length = l '- l, then m
dl µ l So, the time period of oscillation of the oscillator is given by
µ dq or dl µ ldq 2p k m
or dl = a l dq = or T = 2p
T m k
or l '- l = a l dq 3. Driving force is a time dependent force represented by
\ l ¢ = l (1 + a dq) F (t) = f0 cos 2pnt ....(1)
If T ¢ = its period of oscillation, then where n = frequency of driving force. This frequency is
different from the natural frequency of the oscillation nn
l' l (1 + ad q) of the oscillator. nn is also called driven frequency. Thus
T ¢ = 2p = 2p
g g motion depends on driving force.
Thus the motion of the particle is now under the action
æ lö
= T (1+ad q) çQ T = 2p ÷
of : (a) linear restoring force, (b) time dependent (or
è g ø periodic) force given in equaiton (1) so that using Newton’s
1 law, we have for the oscillator :
æ 1 ö ma(t) = – k x (t) + f0 cos2pvt ....(2)
T' = T (1 + ad q) 2 = T ç 1 + ad q÷
è 2 ø
1 k
and vn2 = = ....(3)
T ad q T 2
4p2 m
= T '- T = (Using Binomial expansion)
2 where m = mass of oscillator, k = force constant,
i.e. Increase in Time period v = frequency of the driving force, w = 2pvt,
2. When an oscillator is displaced, say in vn = natural frequency of the oscillator.
x-direction, the restoring force tends to bring it in its Let the displacement be given by, x = C cos wt,
where C = constant,
equilibrium position. Since the restoring force is directly
proportional to the displacement, thus F µ – x or F = – k dx
\ = - C w sin wt
x (t) dt
where, the negative sign indicates that the force always
d2x
acts towards equilibrium position and opposes displacement. = a ( t ) = - C w 2 cos wt
k is known as the force constant. According to Newton’s dt 2
Oscillations 371

\ From (2), we get Using the above relations, we determine the values of
– Cw2m coswt = – k C coswt + f0 cos wt displacement, velocity and acceleration at various instant
t for one complete cycle as illustrated below.
f0 f0
or C= =
k - mw 2
m ( w02 - w2 ) Time, t 0 T/4 T/2 3T/4 T

f0 2p
Phase angle, w t = t 0 p /2 p 3p/2 2p
=
( )
2 T
4p m v02 -v 2
Displacement, x (t) +A 0 –A 0 +A
max. min. max. min. max.
f 0 cos 2pvt Velocity, v (t) 0 – wA 0 + wA 0
x (t ) =
(
4 p2 m v02 - v 2 ) ....(4)
Acceleration, a (t)
min. max. min. max. min.
– w A 0 + w 2A 0 – w 2A
2

This shows that the mass m will executes S.H.M. whose max. min. max. min. max.
frequency is equal to the frequency of the driving force.
The amplitude of oscillation too depends on the frequency We have plotted separately the x-versus t, v versus t and
of the driving force and independent of time. If there is a versus t curves for a simple harmonic motion.
large difference between v and vn, then the amplitude of
oscillation will be too small. But if vn » v, the amplitude
of oscillation will become infinite, which does not actually

Displacement
happens as some sort of unaccounted force breaks the +A
oscillations down. But if v = vn, the amplitude becomes 0 3T/4 t
very large. This phenomenon is called resonance. There T/4 T/2 T
–A
are several examples of resonance in daily life such as T
playing of musical instruments, marching of troops in steps (a)
etc. + wA
Effect of force of friction on S.H.M. : The real oscillations
T/2
Velocity

of almost all oscillators are known as damped oscillations. 0 t


T/4 3T/4 T
The energy of the system is continuouusly dissipated but
nevertheless oscillations remain periodic. This dampens – wA
(b)
oscillations. It is an additional restoring force which is
proportional to the velocity of the particle rather than its +wA
2
Acceleration

displacement i.e. F = –bv(t), where b is a positive constant


called damping constant. The oscillator is now under the T/4 t
0
T/2 3T/4 T
combined action of F(t) and f (t) so that
–wA
2
ma(t) = – kx(t) – bv(t) ...(5) (c)
The solution of this equation gives
bt Relation between velocity, displacement and acceleration
-
x ( t ) = Ae 2m cos ( wt + f ) ...(6) in S.H.M.

2
k æ b ö When pushed
where w = 2pv = -ç ÷ ...(7)
m è 2m ø 5.

4. Inter-relationship between particle displacement, velocity


and acceleration in S.H.M. If a particle executing S.H.M.
passes through its positive extreme position (x = + A) at time
t = 0, then displacement equaiton can be written as l
x (t) = A cos wt y+ l
Velocity,

dx æ pö
v (t ) = = - w A sin wt = wA cos ç wt + ÷ mg Thrust
dt è 2ø

dv
Acceleration, a(t) = -w 2 A cos wt
dt Let a cylinder of mass ‘m’ & length ‘L’ area of cross-
= w2A cos (wt + p) section A & density ‘r’ is floating in a liquid of density
372 Physics
‘s’. Naturally (s < r). is the expression for velocity
Let ‘l’ be the length of the part of cylinder immersed in Acceleration = A
liquid under equilibrium. Then under equilibrium weight of
the liquid displaced by the immersed part = weight of the dv d
= = (aw cos wt ) = -w2 a sin wt
whole cylinder. dt dt
i.e. A l s g = mg Þ m = Als A = – w 2y
Let the cylinder be pushed through a height ‘y’, then
additional liquid will be displaced & weight of that liquid 2p
Time period = T =
provides the restoring force ‘F’. w
Restoring force = F = – (F2 – mg)
= – {A( l + y)sg – A l sg} = – Aysg y displacement
= 2p = 2p ... (iii)
= – (Asg)y = – ky A acceleration
\ Spring factor = k = Asg; Inertial factor
is the expression for time - period.
= m = Alr
m 1 2 1
Alr Lr K.E. = mv = m (aw cos wt )2
T = 2p = 2p = 2p ; Frequency, 2 2
k Asg sg
1 1 sg 1
=
n = = mw2 a 2 cos 2 wt
T 2p Lr 2
Q m = Alr
1 2 1
Al s l 1 g = ka cos2 wt = ka 2 (1 - sin 2 wt )
\ T = 2p = 2p & n= 2 2
Asg g 2p l
6. Let a particle be executing SHM along a circular path of 1 2æ y2 ö 1
= ka ç1 - ÷ = k (a 2 - y 2 )
radius a with angular frequency w and time period T with a 2 ç a2 ÷ 2
è ø
velocity ‘v’. At any instant of time ‘t’ let the particle be at P
making angle q with its initial position at t = 0 1
= mw2 (a 2 - y 2 ) ... (iv)
Y 2
is the expression for K. E.
M P
Restoring force = F = mass × acceleration
a
y
q =w = -mw2 y
X¢ X
O x N t=0 Work done against the restoring force for a small
displacement dx = dW = – Fdy
2
= – (-mw2 y )dy = mw ydy
Y¢ [ Total workdone in displacing the particle from the mean
The position of the particle at any instant ‘t’ can be given position to a displacement
by y
2 1 2
In DOPM , sin q = OM = y
ò
y = w = mw ydy =
2
mw y ... (v)
0
OP a

Þ y = asin π = asin w t ... (i) is the expression for P.E.


Total energy = K.E. + P.E.
is the expression for displacement.
1 1
dy d = mw2 (a 2 - y 2 ) + mw2 y
Velocity = v = = ( a sin wt ) = aw cos wt 2 2
dt dt
1
= mw2 a 2 = constant ... (vi)
Þ v = aw 1 - sin 2 wt 2

y2 is the expression for total - energy.


= aw 1 - = w a 2 - y 2 ... (ii)
2
a
Oscillations 373

Numerical Questions dx æ pö
(b) Velocity, v = = – 5.0 × 2p sin ç 2pt + 4 ÷
1. Here, T = 12s, a = 8 cm. displacement, x = 8 – 3 = 5 cm, t = ? dt è ø
2p 2p pö
Q x = a cos wt = a cos t Þ 5 = 8 cos t æ 22 p
Þ v = –5.0 × 2p sin ç 2 p ´ 1.5 +
4 ÷ø
T 12 =5×2× sin
è 7 4
2p 5
Þ cos = = 0.625 = cos 51.3° 22
6 8 =5× 2× × 0.707 = 22.22 m/s
7
pt 51.3↓≥ 6
Þ = 51.3° Þ t = = 1.31s
d 2x
6 180↓
(c) Acceleration, a =
15 dt
2. Given, a = 15 cm = m, v = 4 Hz.
100 æ pö
= – 5.0 × 2 p ´ 2 p ´ cos ç 2 pt +
2
Maximum acceleration = w a = 4 p v a 2 2 è 4 ÷ø

15 æ pö
= 4 × (3.14)2 × 42 ×
100
= 94.8 m/s2 = -5.0 ´ 4p2 ´ cos ç 2p´ 1.5 + ÷
è 4ø
Maximum value of velocity = aw = 2pa
2
p æ 22 ö 1
15 = +5.0 ´ 4p2 cos = 5´ 4´ç ÷ ´
= 2 × 3.14 × 4 × = 3.77 m/s 4 7
è ø 2
100
= 139.56 m / s 2 .
Here, displacement, x = 5 cos æç ö
2pt
3. + f÷
è T ø 6. Fraction of total energy which is kinetic
(i) When t = 0, x = + 2 units
1
mw2 (a2 - b2 )
[ 2 = 5 cos æç 2p ´ 0 + fö÷ K .E 2
è T ø = =
T .E 1
mw2 a 2
2 2
Þ cos f = = 0.4 a2
5 2 2 a2 - 2
a -y 4 = 3a / 4 = 3 th
\ f = 66.4° Þ f = cos–1(0.4) = 66.4° = =
a2 a2 a2 4
æ 2pt ö
(ii) Phase angle = ç + f÷ = 53.1° Fraction of total energy which is potential
è T ø
4. Here, displacement, x = 6 cos (3pt + p / 3) 1
mw2 y 2
P.E. 2 y2

= = =
dx æ T .E 1 2
Velocity = dt = 6( - 3p ) sin ç 3pt + 3 ÷ mw2 a2 a
è ø 2
æ pö
= -18 p sin ç 3p t + a2 / 4 1 é aù
è 3 ÷ø = 2 =th êQ y = 2 ú
a 4 ë û
(i) Amplitude, a = 6 m
When the total energy is half kinetic and half potential with
pö K.E. = P.E.
(ii) Velocity at t = 2s, V = -18 p sin æç 3p t +
è 3 ÷ø 1 1
mw2 (a 2 - y 2 ) = mw2 y 2 Þ a2 – y2 = y2 Þ a2 = 2y2
2 2
22 p
= – 18 × ´ sin = – 48.99 m/s
7 3 2 a2
Þ y = Þ y = a/ 2
5. Given; w = 2p rad/s, T = 1s, t = 1.5s 2

p 2p
(a) Displacement, x = (5.0) cos éê 2 p ´ 1.5 + ùú 7. Displacement, y = a sin t . Here t = T/12
ë 4û T
2p T p a
æ pö p y = a sin . = a sin =
Þ x = 5.0 cos ç 3p + ÷ = – 5.0 cos T 12 6 2
è 4 ø 4
Þ x = – 5.0 × 0.707 = – 3.535 m 1
[ K.E. of the particle = mw2 ( a 2 - y 2 )
2
374 Physics

1 æ a2 ö m ml 1
= mw2 ´ ç a2 + ÷ T = 2p = 2p \ Tµ
2 ç 4 ÷ø k YA Y
è
3 2 2
K.E. = mw a T1 Y2 3
8 \ = =
T2 Y1 2
1 1 a2 1
[ P.E. = mw2 y 2 = mw2 = mw2 a 2 12. According to question,
2 2 4 8
1 101
l2 = l + l= l; Q Ta l
K .E 3 / 8mw2 a 2 3 100 100
[ = =
P.E 1/ 8mw2 a2 1 1/ 2
l2 101 æ 1 ö
\ T2 = T =2 = 2 ç1 + ÷
[ Ratio = 3 : 1 l 100 è 100 ø
æ 1 ö
∗ ....÷÷÷ = 2 +
1
8. Here, a = 4 cm, v = w a 2 - y 2 = 2 ççç1 ∗
è 200 ø 100
aw a2 [ Increase in time period
Þ = w a2 - y 2 Þ = a2 - y2 1 1
2 4 = T2 – T = 2 + –2= s
100 100
a 2 3a 2 Increase in time period means there will be loss in time.
Þ y2 = a2 - =
4 4 [ Loss in time per day
1 1
3a < ≥ × 24 × 60 × 60 = 432s
Þ y= 100 2
2
40
4 13. Time period, T = = 2s; QTµ l
Þ y = 3´ = 2 3 cm 20
2
l 4
As, P.E. = K.E. Q T1 = T 1 = 2 = 4s.
l 1
[
1 1
mw2 y 2 = mw2 (a 2 - y 2 ) [ Time for 30 vibrations = 30 × 4 = 120s
2 2 14. According to question, x = 5 sin pt;
Þ y2 = a2 – y2 Þ y2 = a2 Maximum displacement = a = 5 cm
At time t = 0, x = 5 sin p× 0 = 0
a 4 (equilibrium position)
Þ y= = = 2 2 cm
2 2 [ Time taken by the particle to move from x = 0 to x = 5 cm
9. Let k be the spring constant of each part. Before the spring is 5 = 5 sin pt
was cut, these parts were connected in ser ies. [ Þ sin pt = 1 = sin p / 2
1 1 1 1 3 p 1
< ∗ ∗ < Þ pt = Þ t = = 0.5 s
K k k k k 2 2
[ k = 3K
When the three parts are connected in parallel, the effective HOTS/EXEMPLAR QUESTIONS
spring constant k' Very Short Answer Questions
k' = k + k + k = 3k = 3 × 3K = 9K = 9 × 15 = 135 N/cm
F 1 1. With a longer length, the period of the pendulum increases.
10. Spring constant, K = = N/cm. Thus, it takes longer to execute each swing, so that each
x 1.5
second according to the clock takes longer than an actual
Let k be the spring constant of each of the part. second. Thus, the clock runs slow.
[ 1 1 1 1 3 1 2. (a) Acceleration is directly proportional to displacement.
= + + = Þ k = 3K = 3 ´
K k k k k 1.5 (b) Acceleration is directed opposite to displacement.
3. When the bob of the pendulum is displaced from the mean
Þ k = 2N/cm
[ Force required, F = kx' = 2×3 = 6 N position so that sin q @ q
F l Short Answer Questions
11. Using, Y = ´ .
A Dl
1. When the platform reaches at the highest point, the
F YA acceleration is maximum which acts towards the mean
Force constant, k = =
Dl l position. At highest point, R – mg = – ma i.e.,
R = mg – ma
Oscillations 375

The maximum acceleration of platform


a = w2A (A = amplitude) 1 m2g 2 1 2
Þ m.2gH + = kA
\ R = mg – mw 2A 2 2k 2
For the object to rest on the platform, g should be less
than a. The maximum value of a will give R = 0. 2mgH m 2 g 2 m 2g 2 æ 2Hk ö
or A2 = + 2 = ç1 + mg ÷ø \
0 = mg – m w2A k k k2 è

g 9.8 ´ T 2 9.8 ´ 4p 2 1/ 2
\ A= = = = 9.8 metre. mg æ 2Hk ö
w 2 2
(4p ) 2 A= 1+
4p k çè mg ÷ø
2. U = U0 (1 – cosax)
2. Let the log be pressed and let the vertical displacement at
-dU -d
F= = ( U 0 - U0 cos ax ) the equilibrium position be x0.
dx dx At equilibrium
= -U 0a sin ax mg = Buoyant force
= Ax0rg
; -U 0aax (for small ax, sinax ~ ax) When it is displaced by a further displacement x, the buoyant
= -U 0a 2 x force is A(x0 + x)rg.
Net restoning force
We know that F = – kx
= Buoyant force – weight = A(x0 + x) rg – mg
So, k = U0 a 2 = (Arg)x. i.e., proportional to x.

m m
T = 2p \ T = 2p
U 0a 2 Arg

3. q1 = q0 sin ( wt + d1 ) g.x
3. Acceleration due to gravity at P = , where g is the
R
q2 = q0 sin ( wt + d2 ) acceleration at the surface.

For the first, q = 2°, \ sin ( wt + d1 ) = 1 mgx mg


Force = = -k.x , k =
R R
For the 2nd, q = – 1°, \ sin ( wt + d2 ) = -1/ 2 Hence, motion will be SHM with time period
\ wt + d1 = 90° , wt + d 2 = -30°
m R
T= = 2p
\ d1 - d2 = 120° K g
Long Answer Questions
CHAPTER TEST
1. Velocity of ball just before collision, u = 2gH . 1. The motion of a satellite round a planet is a periodic motion
but not oscillatory. So, it is not an S.H.M.
As pan is massless, just after collision velocity of “pan +
2. Q K.E. or P.E. µ sin2wt or cos2wt and
mass” system remains the same.
Now the new system has equilibrium position mg/k below w = 2pf
the natural length. \ Frequency of variation of K.E. and P.E.
is 2f.
3. (i) (A), (C), (E), (G)
(ii) (B), (D), (F), (H)
k k 4. Phase difference = 180° or p radian.
5. Inside a spacecraft, g = 0.
Therefore,
N.L.
mg/k l l
u T = 2p = 2p =¥
E.P. g 0
6. Total energy of the bob of a simple pendulum
We can write, energy of SHM as 1
E= mw 2 a 2 \ E µ a 2
2
2
1 1 æ mg ö 1
mu 2 + k ç ÷ = kA 2 E1 a12 æ 2 ö 1
2
2 2 è k ø 2 \ = =ç ÷ =
E2 a22 è 6 ø 9
376 Physics
7. All trigonometric functions are periodic. The sine and cosine The PE of the left column
functions can take value between –1 and +1 only. So they
can be used to represent a bounded motion like S.H.M. But
the functions such as tangent, cotangent, secant and dx
cosecant can take value between 0 and ¥ (both positive
and negative). So those functions cannot be used to h1 x l l h2
represent bounded motion like S.H.M. 45º 45º
100
8. n = 100 vibrations/min = vibrations/s
60 h1

60 3
= ò Argxdx
T = = s 0
100 5
2ο h1
h2 Argl 2 sin 2 45º
vmax = 15 m/s = a w = a x2
T = Arg = Arg 1 =
2 2 2
15 ´ T 15 ´ 3 7 0
Þa = = ´ = 1.432 m
2p 2 ´ 5 22 Similarly, P.E. of the right column
Path length = 2a = 2× 1.432 = 2.864 m h22 Argl 2 sin 2 45º
= Arg =
a 2 2
For x =
2 h1 = h2 = l sin 45º where l is the length of the liquid in
one arm of the tube.
2 a2 1 15 3
v= w a - = aw 1 - = = 13m/s
4 4 2 Argl 2
9. Let V = volume of the air chamber, A = uniform cross- Total P.E. = Argh2 = Argl2sin 45º =
2
sectional area of the neck, m = mass of the ball If the change in liquid level along the tube in left side in
y, then length of the liquid in left side is l – y and inteh
right side is l + y.
y Total P.E. = Arg(l – y)2 sin245º + Arg(l + y)2 +sin2 45º
Change in PE = (PE)' – (PE)
Arg é
= ( l - y )2 + ( l + y )2 - l 2 ùúû
2 êë
Arg é 2
Pressure below the ball inside the chamber = Atmospheric = l + y 2 - 2 ly + l 2 + y 2 + 2ly - l 2 ù
pressure = P, Increase in pressure = p 2 ë û
y = displacement of the ball
Decrease in volume of air inside the chamber = DV = Ay = Arg é y 2 + l 2 ù
ë û
DV Ay 1
Volume strain = = Change in K.E. = Ar2ly 2
V V 2
- EAy
Restoring force = F = p × A = , Change in total energy = 0
V
p p p D ( P.E ) + D ( K .E ) = 0
[Q E = = =
Volume strain æ DV ö æ Dy ö
ç ÷ ç ÷ Arg él 2 + y 2 ù + Arly 2 = 0
EAy è V ø èV ø ë û
Þ p= ]
V Differentiating both sides w.r.t. time.
- EA 2 y
F = pA= é dy ù
V Arg ê0 + 2 y ú + 2 Arlyy = 0
F = – ky \ k = EA2/V ë dt û
m m 2p mV 2Argy + 2Arly = 0
T = 2p = 2p =
k 2 A E .. .. g
EA / V g 2
l y + gy = 0 , y+ y =0, w =
1 A E l l
and v = =
T 2 p mV l
g T = 2p
10. Consider the liquid in the length dx. It’s mass is Ardx at w=
l g
a height x.
PE = Ardx gx
¿¿¿
15 Waves
C ha p t e r

WAVE
Wave is a continuous disturbance which transfers energy and information from one point to another without the actual transfer
of matter as a whole. Water waves are familiar examples. when a pebble is dropped into a pond, it disturbs the surface of the
water. Ripples on the surface of the pond travel away from the spot where the pebble landed.
Types of Waves
There are mainly three types of waves :
(i) Mechanical waves : They can exist only within a material medium and are governed by Newton’s laws. e.g. water waves,
sound waves and seismic waves.
(ii) Electromagnetic waves : They do not require any medium to propagate and hence can propagate through vaccum. e.g.
light waves, radio waves, X-rays, microwaves etc.
(iii) Matter waves : They exist within the matter as they are associated with the fundamental particles of matter like electrons,
protons, neutrons etc. and even atoms and molecules.

TRANSVERSE AND LONGITUDINAL WAVES


Transverse Waves
These are the waves in which the individual particles of the medium oscillate perpendicular to the direction of wave propagation.
Consider a horizontal string with its one end fixed to a rigid support and other end held in the hand fig (a). If we give its free
end an upward jerk, an upward pulse is created there which travels along the string towards the fixed end. Each part of the string
successively undergoes a disturbance about its mean position as shown in fig. (b). If we continuously give up and down jerks
to the free end of the string, a number of sinusoidal waves begin to travel along the string.

Sinusoidal wave
v
Pulse
v C C C
Motion Motion

Ý of the (b)
of the (a)
particles particles
T T T

Each part of the string vibrates up and down while the wave travels along the string. So the waves in the string are transverse
in nature.
The points (C, C, ....) of maximum displacement in the upward direction are called crests. The points (T, T, ....) of maximum displacement
in the downward direction are called troughs. One crest and one trough together form one wave.
Longitudinal Waves
These are the waves in which the individual particles of the medium oscillate along the direction of wave propagation.
Consider a long hollow cylinder AB closed at one end and having a movable piston at the other end. If we suddenly move the
piston rapidly towards right, a small layer of air just near the piston-head is compressed and after being compressed, this layer
moves towards right and compresses the next layer and soon the compression reaches the other end. Now if the piston is suddenly
moves towards left, the layer adjacent to it is rarefied resulting in the fall of pressure. The air from the next layer moves in the
restore pressure. Consequently the next layer is rarefied. In this way a pulse of rarefaction moves towards right.
378 Physics

C R C RC R C R C

A B
If we continuously push and pull the piston in a simple harmonic manner, a sinusoidal sound wave travels along the cylinder in the
form of alternate compressions and rarefactions, marked C, R, C, R, etc. As the oscillations of an element of air are parallel to direction
of wave propagation, the wave is a longitudinal wave. Hence sound waves produced in air are longitudinal waves.
Some Important Terms Connected With Wave Motion
Wavelength (l) : Distance travelled by the wave during the time, anyone particle of the medium completes one vibration about its
mean position. We may also define wavelength as the distance between any two nearest particles of the medium, vibrating in the
same phase.
Frequency (n) : Number of vibrations (number of complete wavelengths) completed by a particle in one second.
The S.I unit of frequency is hertz (Hz).
Time period (T) : Time taken by wave to travel a distance equal to one wavelength.
1
T=
n
Amplitude (A) : It is the maximum displacement of vibrating particle from its equilibrium position.
Angular frequency (w) : It is defined as the rate of change of phase.
2p
i.e., w= = 2pn
T
Wave velocity or phase velocity : The velocity with which the disturbance, or planes of equal phase (wavefront), travel through the
medium is called wave (or phase) velocity.
distance (l ) é 1ù
Wave velocity (v) = or, v = ul êQ v = ú
time (T) ë T û
This is the relation between wave velocity, frequency and wavelength.
Angular wave number or propagation constant (k) : It is defined as phase change per unit path difference.

2p
i.e., k=
l
Wave number (k) : It is the reciprocal of wavelength(l).
1 k
i.e., k= = = number of waves in a unit length of the wave pattern.
l 2p

PROGRESSIVE WAVES
A wave that travels from one point of the medium to another is called a progressive wave. A progressive wave may be transverse
or longitudinal.
Y
Displacement Relation in a Progressive Wave +A
Suppose a simple harmonic wave starts from the origin O and travels
along the positive direction of X-axis with speed v. Let the time be y Wave velocity = v
measured from the instant when the particle at the origin O is passing

O
through the mean position. Taking the initial phase of the particle to be P X
x
zero, the displacement of the particle at the origin O ( x = 0) at any
instant t is given by
y (0, t ) = A sin wt … (i) –A l
where T is the periodic time and A the amplitude of the wave.
A simple harmonic wave
Consider a particle P on the X-axis at a distance x from O. The disturbance starting from the origin O will reach P in
x/v seconds. This means the particle P will start vibrating x/v seconds later than the particle at O. Therefore,
Waves 379

Displacement of the particle at P at any instant t


= displacement of the particle at O at a time x/v seconds earlier
= displacement of the particle at O at time (t – x/v).
Thus the displacement of the particle at P at any time t can be obtained by replacing t by (t – x/v) in the equation (i). It is given by
æ xö æ w ö
y ( x, t ) = A sin w ç t - ÷ = A sin ç w t - x÷
è vø è v ø
w 2p v 2p
But = = =k
v v l
The quantity k = 2p/l is called angular wave number or propagation constant. Hence
y ( x, t ) = A sin(w t - kx ) …(ii)
This equation represents a harmonic wave travelling along the positive direction of the X-axis. It can also be written in the following
forms :
æ 2p 2p ö
y ( x, t ) = A sin ç t - x÷
è T l ø
æ t xö
or y ( x, t ) = A sin 2p ç - ÷ … (iii)
èT lø
2p æ x ö
= A sin çt - T ÷
T è l ø
l
But =v
T
2p æ x ö
\ y ( x, t ) = A sin çt - ÷
T è vø
2p æ l ö
Also y ( x, t ) = A sin çè t - x÷ø …(iv)
l T
2p
or y ( x, t ) = A sin ( vt - x) … (v)
l
Equations (ii), (iii), (iv) and (v) are the various forms of plane progressive wave. If the initial phase of the particle at O is f0 , then the
equation of wave motion will be

Displacement Amplitude Phase


y(x, t) = A sin (w t – k x + f 0)

Angular
frequency
Time Initial
Angular wave phase
number angle
Position
A harmonic wave travelling along negative direction of X-axis can be written as
y ( x, t ) = A sin(w t + kx + f0 )
Phase, Phase Difference and Path Difference
Phase : Phase is a quantity which contains all information related to any vibrating particle in a wave.
For equation y = A sin (wt – kx), (wt – kx) is the phase.
Phase difference and path difference : At any instant t, if f1 and f2 are the phase of two particles whose distances from the origin
are x1 and x2 respectively, then
f1 = (wt - kx1 ) and f2 = (wt - kx2 )
Þ f1 – f2 = k ( x2 - x1 )
2p
Þ phase difference Df = (Path difference Dx )
l
380 Physics
Relation between phase difference, path difference and time period :
Df Dl DT
Þ = =
2p l T
æ l ö
Þ Path difference = ç ÷ Phase difference
è 2p ø

p 3p 5p
Phase (f) 0 p 2p 3p
2 2 2
l l 3l 5l 3l
Wavelength (l) 0 l
4 2 4 4 2
T T 3T 5T 3T
Time period (T) 0 T
4 2 4 4 2

Speed of a Transverse Wave in a Stretched String


The speed of any mechanical wave, transverse or longitudinal, depends on both an Dl
inertial property of the medium (to store kinetic energy) and an elastic property of the
medium (to store potential energy). Dq R
T
Consider a transverse pulse produced in a taut string of linear mass density µ. O
Consider a small segment of the pulse, of length Dl , forming an arc of a circle of radius R.
A force equal in magnitude to the tension T pulls tangentially on this segment at each end.
Let us set an observer at the centre of the pulse which moves along with the pulse towards rights. For the observer any small length
dl of the string as shown will appear to move backward with a velocity v..
Now the small mass of the string is in a circular path of radius R moving with speed v. Therefore, the required centripetal force is
provided by the only force acting, (neglecting gravity) is the component of tension along the radius.
The net restoring force on the element is
Dl
F = 2T sin(Dq) » T(2Dq) = T
R
The mass of the segment is Dm = mDl
v2
The acceleration of this element towards the centre of the circle is a = , where v is the velocity of the pulse.
R
Dl æ v2 ö T
Using second law of motion, T = (mDl) ç ÷ or v=
R è Rø m

Speed of Longitudinal (Sound) Waves


According to Newton’s formula,
E
Velocity of longitudinal wave in medium, Vmedium =
r
where, E = elastic coefficient of medium and
r = density of medium
Y
(a) For solid medium : Vsolid = (E = Y = Young's modulus)
r
Velocity of sound in iron is 5150 ms–1 (approx)
B
(b) For liquid medium : Vliquid = here E = B, (Bulk modulus)
r
For water VWater = 1450 m/s
(c) For gas medium : The formula for velocity of sound in air was first obtained by Newton. He assumed that sound propagates
through air, temperature remains constant. (i.e. the process is isothermal)
So, isothermal elasticity = P
\ Vair = ( P / r)
Waves 381

At NTP for air, P = 1.01 × 105 N/m2 and r = 1.3 kg/m3

1.01x105
So, Vair = = 279 m/s
1.3
However, the experimental value of sound in air is 332 m/s which is much higher than given by Newton's formula.
Laplace Correction
In order to remove the discrepancy between theoretical and experimental values of velocity of sound, Laplace modified Newton's
formula assuming that propagation of sound in air is adiabatic process.
i.e. Adiabatic elasticity = g P
\ vair = ( gP / r )
i.e., vair = 1.41 × 279 = 331.3 m/s [as gair = 1.41]
Which is in good agreement with the experimental value (332 m/s). This in turn establishes that sound propagates adiabatically
through gases.
The velocity of sound in air at NTP is 332 m/s which is much lesser than that of light and radio-waves (3 × 108m/s) .
In case of gases
E Ef gP
vsound = = =
r r r

gPV é mass Mù gmRT


i.e. vsound = êë as r = volume = V úû or vsound = [as PV = mRT]
M M

gRT mass M
or vsound = MW as m = = (Mw = molecular weight)
MW MW
And from kinetic-theory of gases
Vrms = ( 3RT / M W )
1/2
vsound g ég ù
So, = . i.e., vsound = ê ú vrms
vrms 3 ë3û
1/ 2
éγù
i.e., velocity of sound in a gas is ê ú times rms speed of gas molecules (vsound ~ vrms)
ë3û
gRT
As velocity of sound is, vsound = Mw
Velocity of sound in case of gases at constant temperature depends on the nature of gas i.e., its atomicity (g) and molecular weight
(Mw).
Illustration 1 :
A wave is described by y = (2.00 cm) sin (kx – wt), where k = 2.11 rad/m, w = 3.62 rad/s, x is in metres, and t is in seconds.
Determine the amplitude, wavelength, frequency, and speed of the wave.
Sol. The wave function is a moving graph. The position of a particle of the medium, represented by y, is verying all the time and from
every point to the next point at each instant. But we can pick out the parameters that characterize the whole wave and have
constant values.
We compare the given wave function with the general sinusoidal wave equation
y = A sin (kx – wt + f)
Its functional equality to y = (2.00 cm) sin (kx – wt) reveals that the amplitude is A = 2.00 cm
The angular wave number is k = 2.1 rad/m so, wavelength
l = 2p/k = 2.98 m
The angular frequency is w = 3.62 rad/s so, frequency f = w/2p – 0.576 Hz
The speed of wave v = fl = (0.576 s–1) (2.98 m) = 1.72 m/s
382 Physics
Illustration 2 :
A progressive wave of frequency 500 Hz is travelling with a velocity of 360 m/s. How far apart are two points 60o out of phase.
Sol. We know that for a wave v = f l

v 360
So, l = = = 0.72 m
f 500
Now as in a wave, path difference is related to phase difference by the relation
Phase difference Df = 60o = (p/180) x 60 = (p/3) rad

l 0.72 p é p ù
Path difference Dx =
2p
(Df) =
2p 3
= 0.12 m êëQ Df = 60º 3 radúû

Illustration 3 :
An observer standing at sea coast observes 54 waves reaching the coast per minute. If the wavelength of the waves is 10m, find
the velocity. What type of waves did he observe ?

54 9
Sol. As 54 waves reaches the shore per minute, frequency f = = Hz
60 10
9
And as wavelength of waves is 10 m velocity v = f l = × 10 = 9 m/s
10
The waves on the surface of water are combined transverse and longitudinal called 'ripple'. In case surface waves the particles
of medium move in elliptical paths in a vertical plane so that vibrations are simultaneously back and forth and up and down.

Illustration 4 :
A uniform rope of length 12m and mass 6 kg hangs from a rigid support. A block of mass 2 kg is attached to the free end of the
rope. A transverse pulse of wavelength 0.06 m is produced at the lower end of the rope. What is the wavelength of the pulse when
it reaches the top of the rope ?
Sol. Tension at the lower end, T1 = 2g N, tension at the upper end, T 2 = 8gN
If v1 and v2 are the speeds at the lower and the upper ends respectively, then

v2 T2 8
= = =2
v1 T1 2

Further, if l1 and l 2 are the wavelengths at the lower and the upper ends, respectively, then, since the frequency is constant,

l 2 v2
= = 2 or l = 2l = 2 ´ 0.06 = 0.12 m
l1 v1 2 1

Illustration 5 :
Determine the change in volume of 6 litres of alcohol if the pressure is decreased from 200 cm of Hg to 75 cm. [velocity of
sound in alcohol is 1280 m/s, density of alcohol = 0.81 g/cc, density of Hg = 13.6 g/cc and g = 9.81 m/s2].
Sol. For propagation of sound in liquid

v= ( B / r) , i.e., B = v2r
DP DP V( -DP)
But by definition B= –V so, –V = v2r, i.e., DV =
DV DV rv 2
Here DP = H2rg – H1rg = (75 – 200) ´ 13.6 ´ 981 = –1.667 ´ 106 dynes/cm2

So DV =
(6 ´ 10 )(1.667 ´ 10 ) = 0.75 cc
3 6

0.81 ´ (1.280 ´ 10 ) 5 2
Waves 383

PARTICLE VELOCITY AND ACCELERATION


Particle Velocity
The particle velocity V is different from the wave velocity v. It is the velocity with which the particles of the medium vibrate
about their mean positions.
The displacement relation for a harmonic wave travelling along positive X-direction is
y (x, t) = A sin (wt – kx) ... (1)
Differentiating (1) w.r.t. time t, and taking x constant, we get the particle velocity
dy
V= = ωAcos(ωt - kx) ... (2)
dt
V = ωAsin[(wt - kx) +p/ 2] ... (3)
It may be noted that
(i) While the wave velocity (v = nl) remains constant, the particle velocity changes simple harmonically with time.
(ii) The particle velocity is ahead of displacement in phase by p/2 radian.
(iii) The maximum particle velocity or the velocity amplitude is
2π 2π
V0 = ωA = A = times the displacement amplitude A.
T T
(iv) If we differentiate equation (1) w.r.t. position x, we get
dy
= - kAcos(ωt - kx) ... (4)
dx
From equations (2) and (4), we get
V ωAcos(ωt - kx) ω 2πv dy
= =- =- = - ν or V = -n .
dy / dx - kAcos(ωt - kx) k 2π / λ dx
\ Particle velocity at a point = – Wave velocity × slope of displacement curve at that point.
Particle Acceleration
If we differentiate equation with respect to time t, we get the particle acceleration
dV
a= = - ω2 Asin (ωt - kx) = - ω2 y
dt
or a = w2 A sin [(wt – kx) + p]
It may be noted that
(i) The maximum value of particle acceleration or the acceleration amplitude is

æ 2π ö2 æ 2π ö2
a 0 = ω2 A = ç
ç ÷ ÷ A = ç
ç ÷ ÷ times the displacement amplitude.
èT ø èTø
(ii) The particle acceleration is ahead of the particle displacement in phase by p radian.
PRINCIPLE OF SUPERPOSITION OF WAVES
The principle of superposition of waves states that when a number of waves travel through a medium simultaneously, the resultant
displacement of any particle of the medium at any given time is equal to the algebraic sum of the displacements due to the
individual waves.
If y1, y2, y3, ....., yn are the displacements due to waves acting separately, then according to the principle of superposition
the resultant displacement, when all the waves act together is given by the algebraic sum
y = y1 + y2 + y3 + ..... + yn
= f1 (vt – x) + f2 (vt – x) + ..... + fn (vt – x)
n
= å fi (vt - x)
i=1
The superposition of two waves may lead to following three different effects:
(i) When two waves of the same frequency moving with the same speed in the same direction in a medium superpose on
each other, they give rise to effect called interference of waves.
(ii) When two waves of same frequency moving with the same speed in the opposite directions in a medium superpose on
each other, they produce stationary waves.
(iii) When two waves of slightly different frequencies moving with the same speed in the same direction in a medium superpose
on each other, they produce beats.
384 Physics
REFLECTION OF WAVES
An incident wave yi(x, t) = a sin (kx – wt)
(a) at a rigid boundary is reflected with a phase change of p
yr(x, t) = – a sin (kx + wt)
(b) at an open boundary is reflected without any phase change.
yr(x, t) = a sin (kx + wt)
STATIONARY OR STANDING WAVES
A stationary wave is formed when two identical waves travelling in the opposite directions along a straight line interfere. The
superposition of the two waves will give rise to a stationary wave. Formation of stationary wave is possible only in bounded
medium.
Analytical method for stationary waves
(i) From rigid end : y = – 2a sin kx cos wt
This is equation of stationary wave reflected from rigid end.
dy
Velocity of particle Vp = = 2a w sin kx sin wt
dt
dy
Strain, = – 2ak cos kx cos wt
dx
Stress dp
Elasticity E = =
Strain dy
dx
dy
Change in pressure dp = E
dx
(ii) From free end : y = 2 a sin wt cos kx
This is equation of stationary wave reflected from free end.
dy
Velocity of particle = = 2aw cos wt cos kx
dt
dy
Strain = = – 2ak sin wt sin kx
dt
dy
Change in pressure dp = E
dx
Stationary Transverse Waves
In this waves ends are always nodes
A N l
First harmonic = L Þ l = 2L
2
L
(a)
N 2l
Second harmonic or first overtone N A A N = L Þ l=L
2
L
(b)
ANANA
N N
3l 2L
Third harmonic or second overtone = LÞ l=
L 2 3
(c)
pl 2L
For pth harmonic =L Þ l=
2 p
Waves 385

Laws of Vibration of Stretched String


(i) Law of length:
1
n µ (T and m are constant)
l
(ii) Law of tension :
n µ T (l and m are constant)
(iii) Law of mass :
1
n µ (l and T are constant)
m
Stationary Longitudinal Waves
When two longitudinal waves of same frequency and amplitude travel in a medium in opposite directions then by superposition,
standing waves are produced.
These waves are produced in air columns in cylindrical tube of uniform diameter. These sound producing tubes are called organ pipes.
1. Vibration of air column in closed organ pipe :
If l is length of pipe and l be the wavelength and v be the velocity of sound in organ pipe then,

l v v
Case (a) L = Þ l = 4Lk Þ n1 = = 4L
4 l
Fundamental frequency or first harmonic. A
A N A
3l 4L v 3v l3/4
Case (b) L = Þl= Þ n2 = = l2/4
4 3 l 4L N
L l3/2 A
First overtone or third harmonic l1/4 N
5l 4L v 5v l2/2 A
Case (c) L = Þl= Þ n3 = = l3/2 A
4 5 l 4L N N
Second overtone or fifth harmonic. N
When closed organ pipe vibrate in mth overtone then (a) (b) (c)

l 4L v
L = (2m + 1) so, l = ( 2 m + 1) Þ n = (2m + 1)
4 4L
Hence frequency of overtones is given by
n1 : n2 : n3.......= 1 : 3 : 5....
2. Vibration of air column in open organ pipe : If l is length of the pipe and l be the wavelength and v is velocity of sound in organ
pipe then,
l v v
Case (a) L = Þ l = 2L Þ n1 = =
2 l 2L
Fundamental frequency or first harmonic.
2l 2L v 2v
Case (b) L = Þ l= Þ n2 = =
2 2 l 2L
First overtone or second harmonic.
3l 2L v 3v
Case (c) L = Þ l= Þ n3 = =
2 3 l 2L
Second overtone or third harmonic.
Hence frequency of overtones are given by the relation
n1 : n2 : n3 ...............= 1 : 2 : 3.........
When open organ pipe vibrate in mth overtone then
l 4L v
L = (m + 1) so, l = Þ n = (m + 1) wo waves may have equal or unequal amplitu
4 m +1 2L
386 Physics
BEATS
When two sound waves of nearly same frequency are produced simultaneously, then the intensity of resultant sound wave
increases and decreases with time. This change in the intensity of sound is called as the phenomenon of ‘beats’.
The time interval between two successive beats is called beat period and the number of beats per second is called the beat
frequency.
If f1 and f2 are the frequencies (f1 > f2) of the two waves, then the beat frequency b = f1 – f2
Tunning fork :
Frequency of tuning fork
ætö E
n µ çç ÷÷
è l2 ø d
where,
t = thickness of tuning fork
l = length of arm of fork
E = coefficient of elasticity for the material of fork
d = density of the material of fork.
Frequency of tuning fork decreases with increase in temperature.
Illustration 6 :
For a certain organ pipe, three successive resonance frequencies are observed at 425, 595 and 765 Hz respectively. Taking the
speed for sound in air to be 340 m/s (a) Explain whether the pipe is closed at one end or open at both ends (b) Determine the
fundamental frequency and length of the pipe.
Sol. (a) The given frequencies are in the ratio 425 : 595 : 765, i.e., 5 : 7 : 9
And as clearly these are odd integers so the given pipe is closed at one end.
(b) It is clear that the frequency of 5th harmonic (which is third overtone) is 425 Hz
so, 425 = 5 fc or fc = 85 Hz
v v 340
Further as fc = , \ Length of the pipe L = 4f = = 10 m
4L c 4 ´ 85

Illustration 7 :
A column of air and a tuning fork produce 4 beats per second when sounded together. The tuning fork gives the lower note. The
temperature of air is 15ºC. When the temperature falls to 10ºC, the two produce 3 beats per second. Find the frequency of the
fork
Sol. Let the frequency of the tuning fork be n Hz
Then frequency of air column at 15ºC = n + 4
Frequency of air column at 10ºC = n + 3
From, v = nl, we have
v15 n+4
v15 = (n + 4)l and v10 = (n + 3)l \ v10 = n + 3
The speed of sound is directly proportional to the square-root of the absolute temperature.
v15 15 + 273 288
\ v = =
10 10 + 273 283
1/ 2
n+4 288 æ 5 ö 1 5 5
\ = = ç1 + ÷ Þ1+ = 1 + 1/2 × =1+
n+3 283 è 283ø n+3 283 566
1 5
Þ = Þ n + 3 = 113 Þ n = 110 Hz
n + 3 566
Illustration 8 :
Two tuning forks A and B sounded together give 6 beats per second. With an air resonance tube closed at one end, the two
forks give resonance when the two air columns are 24 cm and 25 cm respectively. Calculate the frequencies of forks.
Sol. Let the frequency of the first fork be f1 and that of second for be f2.
v v
Then we have, f1 = and f2 =
4 ´ 24 4 ´ 25
Waves 387

We also see that f1 > f2


\ f1 – f2 = 6 …(i)
f1 24
and = …(ii)
f 2 25
Solving eq. (i) and (ii), we get f1 = 150 Hz and f2 = 144 Hz

DOPPLER EFFECT
Acoustic Doppler effect (Doppler effect for sound waves) :
The apparent change in the frequency of sound when the source of sound, the observer and the medium are in relative motion
is called Doppler effect.
While deriving the expressions for apparent frequency we make the following assumptions :
(i) The velocity of the source, the observer and the medium are along the line joining the positions of the source and the
observer.
(ii) The velocity of the source and the observer is less than velocity of sound.
Case I : When source is moving and observer and medium are at rest :
(i) When source is moving towards stationary observer:
v - vs
Apparent wavelength λ' =
n
v v æ v ö
Apparent frequency, n' = = = n çç ÷
λ æ v - vs ö è v - vs ÷
ø
çç ÷÷
è n ø
So, apparent frequency n’ becomes less than actual frequency n (n' < n)
Case II : When observer is moving and source and medium are at rest :
(i) When observer is moving towards stationary source :
Apparent frequency, n' = actual wave (n) + additional waves frequency (Dn)
v v0 v + v0 æ v + v0 öæ vö
n' = + = = nç
ç ÷ç
÷ çQ λ = ÷÷
λ λ (v / n) è v øè nø
Hence apparent frequency is greater than actual frequency (n’ > n)
(ii) When observer is moving away from stationary source :
Apparent frequency n’=actual wave (n) – reduction in number of waves (Dn)
v v0 v - v0 v - v0 æ v - v0 ö æ vö
n' = + = = = çç ÷÷ n ççQ λ = ÷÷
λ λ l ( v / n) è v ø è nø
Hence apparent frequency in this case is less than actual frequency (n’ < n).
év±v ù

General formula for doppler effect n' = n ê .........(i)
êëv m vs úû
If medium (air) is also moving with vm velocity in direction of source and observer, then velocity of sound relative to
observer will be v ± vm (–ve sign, if vm is opposite to sound velocity). So,
æv ± v ± v ö
n' = n çç m 0÷
è v ± v m m v ÷

[On replacing v by v ± vm in equation (i)]
Case - III : When both source and observer are moving and medium is at rest.
(i) When observer and source are moving away from each other.
æv - v ö
0
Then, n' = çç v + v ÷÷ n
è sø
Hence apparent frequency is less than actual frequency (n’ > n).
(ii) If both source and observer are moving towards each other.
æv+ v ö

Then, n' = çç ÷n
è v - v sø
Therefore actual frequency is less than apparent frequency.
388 Physics
(iii)If source is moving towards observer which is moving away from source then,
æv - v ö
ç 0÷
Apparent freuquency n' = ç v - v ÷ n
è sø
If v0 > vs then, n' > n
v0 > vs then, n' = n
v0 > vs then, n' < n
Doppler's Effect in Reflection of Sound (Echo)
æ v + uö
If the source is at rest and reflector is moving towards the source with speed u, then apparent frequency heard n1 = ç n
è v ÷ø
Now this frequency n1 acts as a source so that apparent frequency received by observer
æ v ö æ v ö æ v + uö æ v + uö
n2 = ç n = ´ n=ç n
è v - u ÷ø 1 çè v - u ÷ø çè v ÷ø è v - u ÷ø
-1 2
æ uö æ uö æ uö æ 2u ö
If u << v then, n 2 = n çè1 + ÷ø çè1 - ÷ø = n ç1 + ÷ = n ç1 + ÷
è vø è
v v vø
æ 2u ö
Beat frequency Dn = n2 – n 1==n ç ÷ n
è vø
v æ Dn ö
So speed of the source u = ç ÷
2è n ø
Conditions When Doppler's Effect is not Observed for Sound-waves
(i) When the source of sound and observer both are at rest.
(ii) When the source and observer both are moving with same velocity in same direction.
(iii) When the source and observer are moving mutually in perpendicular directions.
(iv) When the medium only is moving.
(v) When the distance between the source and observer is constant then doppler effect is not observed.
Illustration 9 :
Two metallic strings A and B of different materials are connected in series forming a joint. The strings have equal cross-
sectional area. The length of A is l A = 0.3m and that of B is l B = 0.75m . One end of the combined string is tied with a supportt
rigidly and the other end is loaded with a block of mass m passing over a frictionless pulley. Transverse waves are set up in the
combined string using an external sources of variable frequency. Calculate:
(a) Lowest of frequency for which standing waves are observed such that the joint is a node.
(b) The total number of antinodes at this frequency.
The densities of A and B are 6.3 ´ 103 kg/m3 and 2.8 ´ 103 kg/m3 respectively..
Sol. (a) Let p and q be the number of loops formed in A and B
p T q T
\ nA = and nB =
2l A ArA 2l B ArB

n A p l B rB p l A rA 0.03 6.3 5
\ = . . =1 [Q nA = nB ] \ q . l . r = 0.75 2.8 = 3
n B q l A rA B B

p 3 6 9
\ the ratio of the number of loops = , , ,....
q 5 10 5
p 3
For the lowest frequency, =
q 5
this means that A will have 3 loops and B will have 5 loops. Hence the required minimum frequency
3 T 5 T
f min = =
2rA ArA 2rB ArB

3 mg m
or f min = =
2 ´ 0.3 A ´ 6.3 ´ 103 25.7A
(b) The total number of antinodes at this minimum frequency is 3 + 5 = 8
Waves 389

Illustration 10 :
When 0.98 m long metallic wire is stressed, an extension of 0.02 m is produced. An organ pipe 0.5 m long and open at both
ends, when sounded with this stressed metallic wire, produces 8 beats in its fundamental mode. By decreasing the stress in the
wire, the frequency of beats is found to decrease. Find the Young’s modulus of the wire. The density of metallic wire is 104 kg/
m3 and speed of sound in air is 292 m/s.
Sol. Frequency of the transverse vibration of the stretched string
1 T
f1 = …(i)
2 ( L + DL ) m

T L YADL 1 YDL
Here, Y = , or T = , and m = Ar \ f1 = …(ii)
A DL L 2 ( L + DL ) Lr

When the stress in the wire is decreased, f1 will decrease, consequently the beat frequency will decrease if f1 < f pipe

1 YDL v
\ f1 - f pipe = 8 or - =8
2 ( L + DL) Lr 2l
Substituting the value, we get
1 Y ´ 0.02 292
- =8
2 ( 0.98 + 0.02) 0.98 ´ 104 2 ´ 0.5
Simplifying, we get, Y = 1.764 ´ 1011 N/m 2
Illustration 11 :
A train, standing at the outer signal of a railway station blows a whistle of frequency 400 Hz in still air. What is the frequency of
the whistle for a platform observer when the train (a) approaches the platform with a speed of 10 m s–1,
(b) recedes from the platform with a speed of 10 m s–1? The speed of sound in still air can be taken as 340 m s–1.
Sol. Here air is still and listener is standing on the platform so vm = 0, v0 = 0
æ v + vm - v0 ö æ v+ ö
(a) From formula n¢ = ç v + v - v ÷ n ; n¢ = ç v - v ÷ n
è m sø è sø
–1 –1
n = 400 Hz, vs = 10 ms , v = 340 ms .
æ 340 ö
n¢ = ç 400 = 412.12 Hz
\ è 340 - 10 ÷ø
(b) When the train recedes from the platform the frequency of the whistle
æ v+ ö æ 340 ö
n¢ = ç n=ç 400 = 388.57 Hz
è v + vs ÷ø è 340 + 10 ÷ø

Illustration 12 :
In a car race sound signals emitted by the two cars are detected by the detector on the straight track at the end point of the race.
Frequency observed are 330 Hz and 360 Hz and the original frequency is 300 Hz of both cars. Race ends with the separation
of 100m between the cars. Assume both cars move with constant velocity and velocity of sound is 330 m/s. Find the time taken
by winning car.
Sol. Let the velocities of car 1 and car 2 be V1 m/s and V2 m/s
\ Apparent frequencies of sound emitted by car 1 and car 2 as detected at end point are
V V V 2t
f1 = f 0 and f2 = f0
V - V1 V - V2 V1 t
1 2
330 330
Þ 330 = 300 and 360 = 300 330 - V A B
330 - V1 2 100m
Þ V1 = 30 m/s and V2 = 100 m/s
The distance between both the cars just when the 2nd car reach point B
100m = V2 t - V1t Þ t = 4 sec
390 Physics
Waves 391

Textbook Exercises

15.1 A string of mass 2.50 kg is under a tension of 200N.


The length of the stretched string is 20m. If the rRT
Sol. Assume ideal gas law : P = , where r is the
transverse jerk is struck at one end of the string, how M
long does the disturbance take to reach the other end? density, M is the molecular mass, and T is the
Sol. Given, M = 2.5 kg, T = 200N, l = 20m ; Mass/unit
M gRT
2.5 temperature of the gas. This gives V= . This shows
Length, m = = = 0.125 kg/m M
l 20
that velocity of sound
T 200
Velocity, v = = = 40 m/s (a) is independent of pressure.
m 0.125 (b) increases as the temperature rises.
Time taken by disturbance to reach the other end,
(c) increases with humidity as the molecular mass of
l 20
t= = = 0.5s water (18) is less than that of N2 (28) and O2 (32).
v 40
15.2 A stone dropped from the top of a tower of height 300 Therefore as humidity increases, the effective
m high splashes into the water of a pond near the base 1
of the tower. When is the splash heard at the top? [Speed molecular mass of air decreases and v µ
M
of sound in air = 340 m/s and g = 9.8 m/s2] hence v increases.
Sol. Here, h = 300m, g = 9.8 m/s2, v = 340 m/s
Let t1 = time taken by stone to strike the surface of water 15.5 You have learnt that a travelling wave in one dimension
is represented by a function
1 2
Using s = ut1 + gt1 y = f(x, t) where x and t must appear in the combination
2
1 x – nt or x + n t, i.e., y = f(x ± nt). Is the converse true
Þ 300 = 0 + ´ 9.8 ´ t12 ? Examine if the following functions for y can possibly
2
represent a travelling wave:
300
\ t1 = = 7.82s. (a) (x – vt)2
4.9 (b) log [(x + vt)/x0]
And, t2 = time taken by sound to reach to top of tower
(c) 1/(x + vt)
h 300 Sol. The converse is not true. For any function to represent
= = = 0.88s.
v 340 a travelling wave it should be finite everywhere and at
\ Total time after which the splash is heard = t1 + t2 all times. Only function (c) satisfies this condition, the
= 7.82 + 0.88 = 8.70s. remaining functions cannot possibly represent a
15.3 A steel wire has a length of 12 m and a mass of 2.1 kg. travelling wave.
What should be the tension in the wire so that the speed 15.6 A bat emits ultrasonic sound of frequency 1000 kHz in
of a transverse wave on the wire equals the speed of air. If the sound meets a water surface, what is the
sound in dry air at 20°C = 343 m/s.
wavelength of (a) the reflected sound, (b) the transmitted
Sol. Given, l = 12m, M = 2.1 kg, T =?v = 343 m/s;
sound? [Speed of sound in air = 340 m/s and in water
M 2.1 = 1486 m/s]
Mass/per unit length(m) = =
l 12 Sol. According to question,
= 0.175 kg/m n =1000 kHz = 106 Hz, va = 340m/s,
T vw = 1486 m/s
Q v= (a) Wavelength of reflected sound
m
Þ T = v × m = (343)2 × 0.175
2 va 340
= 2.06 × 104 N = la = = = 3.4 × 10–4m
n 106
gP (b) Wavelength of transmitted sound
15.4 Use the formula n = to explain why the speed of
r vw 1486
= lw = = = 1.486 × 10–3 m
sound in air n 106
(a) is independent of pressure, 15.7 A hospital uses an ultrasonic scanner to locate tumours
(b) increases with temperature, in a tissue. What is the wavelength of sound in the tissue
(c) increases with humidity. in which the speed of sound is 1.7 km/s? The operating
frequency of the scanner is 4.2 MHz.
392 Physics
Sol. Here, v = 1.7 km/s = 1700 m/s, n = 4.2MHz 2 p 2p p
= 4.2 × 106 Hz Þ T= = = s
w 36 18
v 1700
Wavelength, l = = æ pö
n 4.2 ´10 6 for x = 2; y(2, t) = 3.0 sin ç 36t + 0.036 + ÷
è 4ø
–3
= 0.405 × 10 m = 0.405mm
15.8 A transverse harmonic wave on a string is described by æ pö
for x = 4 ; y(4, t) = 3.0 sin ç 36t + 0.072 + ÷
y(x, t) = 3sin (36t + 0.018x + p/4), where x, y are in è 4ø
cm and t in sec. The positive direction of x-axis is from For x = 0, putting various values of ‘t’ in eq. (i) we get
left to right.
(i) Is this a travelling or stationary wave? If it is T 2T 3T 4T 5T 6T 7T
t 0 T
travelling, what are the speed and direction of its 8 8 8 8 8 8 8
propagation? 3 3 -3 -3 3
y 3 0 -3 0
(ii) What are its amplitude and frequency? 2 2 2 2 2
(iii) What is the initial phase at the origin?
which can be plotted as
(iv) What is the least distance between two successive
crests in the wave? 3
Sol. Wave equation given, 3/ 2
y(x, t) = 3sin (36t + 0.018x + p/4)
0

y(cm)
The general equation of a plane progressive wave is y(x, T/8 2T/8 3T/8 4T/ 8 5T/8 6T/8 7T/8 T

é 2p ù 3/ 2
t) = a sin ê (vt + x ) + f0 ú –3
ël û t(s)
Comparing we get, Similarly the graphs for x = 2 and
(i) The given wave is a travelling wave moving from x = 4 can be plotted.
right to left. The amplitude and frequency of these waves are same
Equating coefficient of t
36 i.e., 3.0 cm and 36 s - 1 but they differ in phase given
Velocity, v = = 2000 cm/s 2p
0.018 p p p
(ii) Amplitude, a = 3cm by , + 0.036, + 0.072
4 4 4
2p
= 0.018 15.10 For the travelling harmonic wave y(x, t) = 2 cos 2p [10t
l
2p – 0.0080x + 0.35], where x and y are in cm and t in s.
Þ l = wavelength = = 349 cm What is the phase difference between oscillatory motion
0.018
at two points separated by a distance of (i) 4m
v 2000 (ii) 0.5 m (iii) l/2 (iv) 3l/4
Frequency, n = = = 5.73 Hz.
l 349 Sol. From question,
y = 2cos [2p (10t – 0.0080x) + 2p × 0.35]
p
(iii) Initial phase = f0 = radian Comparing it with the standard equation
4
(iv) Least distance between two successive crests = é 2p ù
wavelength = 349 cm. ; 3.5 m y = a cos ê ( vt - x ) + f o ú
ë l û
15.9 For the wave described in Exercise 15.8, plot the
displacement (y) versus (t) graphs for x = 0, 2 and 4 cm. Path difference
What are the shapes of these graphs ? In which aspects 2p
does the oscillatory motion in travelling wave differ from = 2 p ´ 0.0080
l 1
one point to another : amplitude, frequency or phase ? Intial phase = f0 = 2p × 0.35 Þ l =
Sol. For the given wave 0.008
2p
p Now phase difference = ´x
y(x, t) = 3.0 sin (36t + 0.018x + ) l
4
(i) When x = 4m = 400 cm;
æ pö Phase difference
for x = 0; y (0, t) = 3.0 sin ç 36t + ÷ ...(i) f = 2p × 0.0080 × 400 = 6.4p rad.
è 4ø
(ii) When x = 0.5 m = 50 cm;
p Phase difference
Þ a = 3 cm, f0 = & w = 36 = f = 2p × 0.0080 × 50 = 0.8p rad.
4
Waves 393

2p l (a) frequency
(iii) When x = l/2, f = ´ = p rad (b) phase
l 2
(c) amplitude ?
2p 3l 3p Explain your answers.
(iv) When x = 3l/4, f = ´ = rad.
l 4 2 (ii) What is the amplitude of a point 0.375m away from
15.11 The transverse displacement of a string (clamped at its one end ?
both ends) is given by Sol. (i) The transverse displacement is given by
æ 2p ö 2p
y (x, t) = 0.06 sin ç x÷ cos (120pt) where x, y are y(x, t) = 0.06 sin x cos 120pt.
è 3 ø 3
(a) Yes. The time dependent harmonic function cos 120pt
in m and t in s. The length of the string is 1.5 m and of the stationary wave represents its frequency. As
its mass is 3 × 10–2 kg. Answer the following: this function does not depend on x, so frequency
(a) Does the function represent a travelling or a of oscillation of all points on the string is same.
stationary wave? (b) Yes. The phase of all the points on the string is same
(b) Interpret the wave as a superposition of two waves for the reasons similar to (a).
travelling in opposite directions. What is the (c) No. The amplitude of stationary wave is given by
wavelength, frequency and speed of propagation of
2p
each wave? a = 0.06 sin x
3
(c) Determine the tension in the string. [time dependent part]
2 px As a depends on x, amplitude of all the points on
Sol. (a) The equation given, y (x, t) = 0.06 sin cos
3 the string is not same]
120pt ..... (1) (ii) Now, amplitude at a point 0.375 m away from one
Q This equation is both harmonic functions of x and end is given by
t separately
2p
\ It represents a stationary wave, a = 0.06 sin × 0.375 = 0.06 sin 0.7854
3
(b) A stationary wave is formed when a wave
= 0.06 × 0.707 = 0.042 m.
2p 15.13 Given below are some functions of x and t to represent
y1 = a sin (vt – x) travelling along +ve direction of the displacement transverse or longitudinal of an elastic
l
wave. State which of these represent (i) a travelling wave,
x and another reflected wave, (ii) a stationary wave or (iii) none at all:
2p (a) y = 2 cos 3x sin 10t
y2 = – a sin (vt + x) travelling along –ve direction
l (b) y = 2 x - vt
of x, superpose on each other. (c) y = 3 sin (5x – 0.5t) + 4 cos (5x – 0.5t)
\ Resultant wave = y = y1 + y2 (d) y = cos x sint + cos 2x sin 2t
2px 2p Sol. (a) It represents a stationary wave as the wave is a
= -2a sin cos vt ..... (2) products of functions of x and t separately.
l l (b) It doesn't represent any wave.
Comparing equations (1) and (2), 2p = 2p Þ l = 3m (c) It represents a progressive wave.
l 3 (d) This equation represents the sum of two stationary
waves.
2pv 15.14 A wire stretched between two rigid supports vibrates in
= 120 p Þ v = 60l = 60 × 3 = 180 m/s
l its fundamental mode with a frequency of 45 Hz. The
mass of the wire is 3.5 × 10–2 kg and its linear mass
v 180 density is 4 × 10–2 kg/m. What is (a) the speed of the
Frequency = n = = = 60Hz.
l 3 transverse wave on the string and (b) the tension in the
Both waves have same wavelength, frequency and speed. string?
T Sol. (a) Given, n = 45Hz , M = 3.5 × 10–2 kg,
(c) Velocity, v =
m m = 4 × 10–2 kg/m
\ tension
M 3.5 ´ 10 -2 7
3 ´10 -2 \ l= = = m
T = v2 × m = (180) 2 ´ = 648N m 4 ´ 10 -2 8
1.5
15.12 (i) For the wave on a string described in Exercise As it is vibrating in fundamental mode
15.11, do all the points on the string oscillate with
the same l 7 7
=l= \ l= = 1.75 m.
2 8 4
394 Physics
Speed = v = nl = 45 × 1.75= 78.75 m/s v n ´ 340
nn = n ´ = = 430
T
2L 2 ´ 0.2
(b) v= Þ T = v2 × m
m 430 ´ 2 ´ 0.2
\ n= = 0.5
= (78.5)2×4× 10–2 = 248.06 N 340
15.15 A metre-long tube open at one end, with a movable piston Q n should be an integer, therefore it is not in
at the other end, shows resonance with a fixed frequency resonance with the source.
source (a tuning fork of frequency 340 Hz) when the 15.18 Two sitar strings A and B playing the note ‘Ga’ are
slightly out of tune and produce beats of frequency 6
tube length is 25.5 cm or 79.3 cm. Estimate the speed
Hz. The tension in the string A is slightly reduced and
of sound in air at the temperature of the experiment.
the beat frequency is found to reduce to 3Hz. If the
The edge effects may be neglected. original frequency of A is 324 Hz, what is the frequency
Sol. One end of the tube is open and the other end is closed of B?
by the piston, so it behaves as a closed organ pipe, which Sol. Let original frequency of A and B are nA and nB
produces only odd harmonics. respectively.
\ The pipe is in resonance with the fundamental mode n A - n B = ±6 Þ n B = nA ± 6 = 324 ± 6
and the 3rd harmonic (Q 79.3 » 3 × 25.5) = 330 or 318 Hz
l When the tension is reduced, frequency is also reduced
For fundamental mode, = l1 = 25.5
4 as n µ T
\ l = 4 × 25.5 = 102 cm = 1.02m As the no. of beats decreases
Speed = v = nl = 340 × 1.02 = 346.8 m/s \ n B = 324 - 6 = 318 Hz.
15.16 A steel rod 100 cm long is clamped at its middle. The 15.19 Explain why (or how) :
fundamental frequency of longitudinal vibrations of the (a) in a sound wave, a displacement node is a pressure
rod are given to be 2.53 kHz. What is the speed of sound antinode and vice versa,
in steel? (b) bats can ascertain distances, directions, nature,
Sol. When the rod is clamped at the middle, a node is forrmed and sizes of the obstacles without any “eyes”,
at the middle and 2 antinodes are forrmed at the two ends. (c) a violin note and sitar note may have the same
L frequency, yet we can distinguish between the two
notes,
A A (d) solids can support both longitudinal and transverse
N waves, but only longitudinal waves can propagate
in gases, and
l /4 l/4
(e) the shape of a pulse gets distorted during
Here, L = 100 cm = 1m, n = 2.53 kHz = 2.53 × 103 Hz propagation in a dispersive medium.
Sol. (a) As displacement node is a point of zero
l l l displacement the pressure variation is maximum at
\ L= + = Þ l = 2L = 2m
4 4 2 this point.
v = nl = 2.53 × 103 × 2 = 5.06 × 103 m/s Hence it is a pressure antinode.
15.17 A pipe 20 cm long is closed at one end. Which harmonic (b) Bats have the ability to produce and detect
mode of the pipe is resonantly excited by a 430 Hz ultrasonic waves (n > 20 kHz) so they can detect
source? Will the same source be in resonance with the the distance of the object by the interval between
pipe, if both ends are open. Speed of sound = 340 m/s their producing the waves and receiving the echo.
Sol. Given, L = 20 cm = 0.2 m, From the intensity of the echo they detect the nature
Natural frequency of the pipe and size of the object and from the small interval
n n = 430Hz , v = 340m/s between their receiving the echo by their two ears,
they can ascertain the direction of the object.
v 340 (c) The quality of the sound produced depends on the
nn = (2n - 1) Þ 430 = (2n – 1)
4L 4 ´ 0.2 overtones produced by the instruments. The Sitar
430 ´ 4 ´ 0.2 and Violin though may have the same fundamental
Þ 2n – 1 = = 1.02 frequency but they produce different overtone
340
which is an integral multiple of fundamental
Þ 2n = 2.02 Þ n = 1.01 frequency and hence we can differentiate between
\ It is 1st normal mode of vibration. the sound of the two.
In a pipe, with both ends open (d) Gases do not have shear elasticity so only the
longitudinal waves can travel through them whereas
solids have both volume and shear elasticity and
Waves 395

hence both longitudinal and tranverse waves can platform? Is the situation exactly identical to the case
travel through them. when the air is still and the observer runs towards the
(e) It is the property of a dispersive medium that waves yard at a speed of 10 ms–1 ? The speed of sound in still
of different wavelength travel with different speeds air can be taken as 340 ms–1
in different directions or with different velocities. Sol. n = 400Hz , vm = 10 m/s, v = 340 m/s
Thus when a sound pulse which is a combination When the wind is blowing in the direction of sound
of waves of different wavelengths travels through \ Effective speed of sound
it gets distorted. = v + vm = 340 + 10 = 350 m/s
15.20 A train, standing at the outer signal of railway station As the source and listener both are at rest, therefore
blows a whistle of frequency 400 Hz in still air. (i) What frequency remains unchanged, i.e. 400 Hz.
is the frequency of the whistle for a platform observer Wavelength of sound for the stationary observer.
when the train
(a) approaches the platform with a speed of 10 m/s v + vm 350
l'= = = 0.875m
(b) recedes from the platform with a speed of 10 m/s? n 400
(ii) What is the speed of sound in each case? In case the air is still but the observer is running towards
[Speed of sound in still air = 340 m/s] the yard.
Sol. (i) Given, n = 400Hz , v = 340 m/s When the observer moves towards the stationary engine
(a) When the train approaches the platform vs = 10 m/s (source) in still air,
v 340 ´ 400 vo = – 10 m/s, vs= 0
n' = ´n = = 412.12 Hz
v - vs 340 - 10
v - vo 340 + 10
v´n n' = ´n' = ´ 400
(b) When the train recedes n' = v - vs 340 - 0
v + vs
340 ´ 400 350
= = 388.6 Hz. = ´ 400 = 411.8 Hz.
340 + 10 340
(ii) The speed of sound remains same in both cases i.e. As wavelength of sound waves is not affected by motion
340 m/s. of the observer, it remains unchanged.
15.21 A train, standing in a station-yard, blows a whistle of
Speed of sound relative to the observer,
frequency 400 Hz in still air. The wind starts blowing
in the direction from the yard to the station with a speed v¢¢ = 340 + 10 = 350 ms–1.
of 10 m/s. What are the frequency, wavelength and speed Thus the situations in both the cases are not exactly
of sound for an observer standing on the station’s identical.

15.22 A travelling harmonic wave on a string is described by


12
a = 7.5cm, n = cm s–1
æ pö 0.0050
y( x , t ) = 7.5 sin ç 0.0050 x + 12t + ÷
è 4ø 2p
(i) What are the displacement and velocity of and = 0.0050 cm –1
l
oscillation of a point at x = 1 cm, and t = 1s ? Is (i) At x = 1cm and t = 1s, displacement is
this velocity equal to the velocity of wave y = 7.5 sin (0.0050 × 1 + 12 × 1 + p/4)
propagation ? = 7.5 sin 12.79
(ii) Locate the points of the string, which have the same = 7.5 × 0.2222 = 1.67cm.
transverse displacement and velocity as the x = Velocity of oscillation of the particle is
1cm point at t = 2s, 5s and 11s.
Sol. Given, y (x, t) = 7.5 sin (0.0050x + 12t + p/4) dy d
u= = [7.5 sin (0.0050x + 12t + p/4)]
dt dt
é ì 12 ü ù = 7.5 × 12 cos (0.0050x + 12t + p/4)
= 7.5 sin ê0.0050 í 0.0050 t + x ý + p / 4 ú ...(1)
ë î þ û
æ pö
Comparing it with standard equation of travelling wave = 90 cos ç 0.0050 x + 12t + ÷
è 4ø
é 2p ù At x = 1cm and t = 1s,
y = a sin ê {vt + x} + fú ...(2)
ël û u = 90 cos (0.000050 × 1 + 12 × 1 + p/4)
Comparing equations (1) and (2), we get = 90 cos 12.79
396 Physics
= 90 × 0.9751 = 87.76 cm s–1
w
Velocity of wave propagation is As v =
k
12
n= = 2400 cm s–1
0.0050 w 16.1 ´ 102
\ k= = = 4.84 rad/m
\ Velocity of oscillation of a point is not equal to v 3.32 ´ 102
the velocity of wave propagation. As the wave propagates along positive x-axis, so the
2p displacement equation is
(ii) As = 0.0050 y = a sin (wt – kx)
l
or y = 0.05 sin (16.1 × 102t – 4.84 x), x and y are in
2p metre.
\ l= = 1256.64 cm 15.25 A SONAR system fixed in a submarine operates at a
0.0050
frequency 40 kHz. An enemy submarine moves towards
All points located at distance n l (where n is an
the SONAR with a speed of 360 km/h. What is the
integer) from the point x = 1cm have the same
transverse displacement and velocity. frequency of sound reflected by the submarine? [Speed
15.23 A narrow sound pulse (for example, a short pip by a of sound in water = 1450 m/s]
whistle) is sent across a medium. Sol. Frequency of SONAR, n = 40 kHz
(a) Does the pulse have a definite : = 40 × 103 Hz
(i) frequency, (ii) wavelength, (iii) speed of Speed of observer = vo = 360 km/h = 100m/s
propagation? Speed of sound = v = 1450 m/s.
(b) If the pulse rate is 1 after every 20s, (that is the Q The source is at rest and observer is moving
whistle is blown for a split of second after every towards the source
20 s), is the frequency of the note produced by the \ Apparent frequency received by the submarine =
whistle equal to 1/20 or 0.05Hz? (v + vo )n
Sol. (a) A narrow sound pulse such as a short pip by a n' =
v
whistle does not have a definite wavelength or
frequency. But being a sound wave, it has a definite (1450 + 100) ´ 40 ´ 103
speed (in a non-dispersive medium). = = 4.276 × 104 Hz
1450
(b) No, the frequency of the note produced by the
This frequency is reflected by the enemy submarine and
whistle is not equal to 1/20 or 0.05 Hz, it is only the
is observed by SONAR.
frequency at which the pulse gets repeated.
15.24 One end of a long string of linear mass density 8.0 × \ vs = 360 km/s = 100 m/s, vo = 0
10–3 kg/m is connected to an electrically driven tuning v´n
fork of frequency 256 Hz. The other end passes ove a \ Apparent frequency = n " =
v - vs
pulley and is tied to a pan containing a mass of 90 kg.
The pulley end absorbs all the incoming energy so that 1450 ´ 4.276 ´10 4
reflected waves at this end have negligible amplitude. = = 4.59 × 104 Hz.
1450 - 10
At t = 0, the left end (fork end) of the string x = 0 has
zero transverse displacement (y = 0) and is moving along 15.26 Earthquakes generate sound waves inside the earth.
positive y-direction. The amplitude of the wave is 5.0cm. Unlike a gas, the earth can experience both transverse
Write down the transverse displacement y as function (S) and longitudinal (P) sound waves. Typically, the speed
of x and t that describes the wave on the string. of S wave is about 4 km/s, and that of P wave is 8
Sol. Tension in the string, km/s. A seismograph records P and S waves from an
T = 90 × 9.8 = 882 N earthquake. The first P wave arrives 4 min before the
Mass per unit length of the string. first S wave. Assuming the waves travel in straight line,
m = 8.0 × 10–3 kg/m at what distance does the earthquake occur?
The velocity of the transverse wave along the string is Sol. Let v1, v2 = speeds of S and P waves respectively
t 1 , t 2 = time taken by S and P waves to travel to
T 882 seismograph.
v= = = 3.32 ´ 102 m/s
m -3 L = distance of seismograph from the point of earthquake
8 ´10
Frequency of the wave, Then L = v1t1 = v2 t 2
v = 256 Hz v1 = 4 km/s and v2 = 8 km/s \ 4t1 = 8t2
Amplitude of the wave, t1 = 2t2
a = 5.0 cm = 0.05 m Also t1 – t2 = 4min = 240s.
Angular frequency,
w = 2pv = 2 × 3.142 × 256 L L 4L L
Þ - = 240 Þ = 240 Þ = 240
= 16.1 × 102 rad/s 4 8 32 8
L = 240 × 8 = 1920 km
Waves 397

15.27 A bat is flitting about in a cave, navigating via ultrasonic


beeps. Assume that the sound emission frequency of the 40 ´103
=
bat is 40 kHz. During one fast swoop directly toward 0.97
a flat wall surface, the bat is moving at 0.03 times the This is reflected by the wall and is received by the bat.
speed of sound in air. What frequency does the bat hear \ vs = 0, vB = 0.03v
reflected off the wall? \ Apparent frequency (bat hear)
Sol. Frequency of sound emitted by the bat, n = 40 kHz.
Velocity of bat (source), v = 0.03vs v + vB (v + 0.03v ) 40 ´103
= n' = ×
[vs = velocity of sound] v v 0.97
Apparent frequency striking the wall, = 42.47 × 103 Hz = 42.47 kHz.
v v
n' = n= × 40 × 103
v - vs v - 0.03v
398 Physics

Practice Questions
Very Short Answer Questions [1 Mark Qs.] 9. If y = 3 sin(36t + 0.18x + p/4) m, find the amplitude and
velocity of the wave?
1. What type of mechanical wave exists in air and inside water? 10. If density is made four times, what will be effect on the
2. What is the phase difference between the wave y = a velocity of sound?
cos(wt + kx) and y = a sin (wt + kx)? 11. The speed of sound waves depends on temperature but speed
3. What characteristics of a medium determine the speed of of light waves does not. Explain.
sound waves though it? 12. All harmonics are overtones but all overtones are not
4. What is the effect on the velocity of waves in a string harmonics. How ?
if only 1/4th of the original length is used with the same 13. A sonometer wire resonates with a tuning fork. If the
tension? length of the wire between the bridges is made twice even
5. Why should the difference between the frequencies be then it can resonate with the same tuning fork. How ?
less than 10, to produce beats? 14. An organ pipe is in resonance with a tuning fork. What
6. Velocity of sound in air at N.T.P. is 332 m/s. What will change will have to be done in the length L to maintain
be the velocity, when pressure is doubled and temperature the resonance, if (i) the temperature increases, (ii) hydrogen
is kept constant? is filled in place of air and (iii) pressure becomes higher?
7. Why is it possible to detect the approach of a train by
placing the ear upon the rails of track? Long Answer Questions [5 Marks Qs.]
8. Is the phenomenon of beats observable in case of two light
waves of nearly equal frequencies? 1. What are stationary waves? Write some characteristics of
9. Why is sound heard in water more intense in comparison stationary wave. How are they formed in strings? Draw
to sound heard in air? the various modes of vibration in them.
10. How is the vibration of the air column in a flute different 2. State Newton's formula for the velocity of sound in air.
from that of a string? Point out the error and hence discuss the Laplace's
11. What is an echo ? What should be the minimum distance correction.
between the source of sound and the reflector for hearing 3. Establish a relation between particle velocity and wave
a distinct echo ? velocity.
12. Sometimes, in a stringed instrument, a thick wire is wrapped 4. Discuss the formation of stationary waves in open organ
by a thin wire. Why? pipe analytically and show that all overtones are present
13. If oil of density higher than the density of water is used in the sound.
in a resonance tube, how will the frequency change? 5. What is meant by beats? Derive a formula for beat
14. Why are there so many holes in a flute? frequency of two waves.
6. What is Doppler effect in sound. Derive expressions for
15. An open pipe makes a good musical instrument, in
apparent frequency when
comparison to a closed pipe. Why?
(i) source is moving towards stationary observer.
16. What is the difference between a tone and a note? (ii) observer is moving towards stationary source.
17. As in sound, can beats be observed by two light sources? (iii) both source and observer are in motion.
18. A violin note and a sitar note may have the same frequency
and yet we can distinguish between the two notes. Explain, Numericals Questions [3 or 5 Marks Qs.]
why it is so.
1. A progressive wave of frequency 500 Hz is travelling with a
Short Answer Questions [2 or 3 Marks Qs.] velocity of 360 m/s. How far apart are two points 60° out of
phase?
1. Why transverse waves cannot propagate through liquids
and gases? 2. The distance between two points on a stretched string is
2. Differentiate between Stationary waves and Progressive 20 cm. A progressive wave of frequency 400 Hz travels on
waves. the string with a velocity of 100 m/s. Calculate phase
3. Write basic conditions for formation of stationary waves. difference between the points.
4. Distinguish between sound waves and radio waves of same 3. A sound wave of frequency 10 kHz is travelling in air with a
frequency. speed of 340 m/s. Find the minimum separation between
5. What is the difference between interference and stationary two points where the phase difference is 60°?
waves ? In which phenomenon, out of the two, energy 4. A tuning fork of frequency 220 Hz produces sound waves
is not propagated ? Why there is no energy at interference of wavelength 1.5 m in air at N.T.P. Calculate the increase in
minimum ? wavelength, when temperature of air is increased to 27°C?
6. Derive a relation between speed of sound in a gas and root 5. At what temperature will the speed of sound be double its
mean square velocity of the molecules of that gas. velocity at 0°C?
7. Where will a man hear a louder sound at the node or at 6. Speed of sound in air is 332 m/s at N.T.P What will be its
the antinode in case of a stationary wave ? value in hydrogen at N.T.P, if density of hydrogen at N.T.P.
8. Why is the sound produced in air not heard by a person is 1/16th that of air?
deep inside the water ?
Waves 399

7. At what temperature will the velocity of sound in hydrogen (a) 2nd harmonic if it is closed at one end
be the same as in oxygen at 100°C? Density of oxygen is 16 (b) Second harmonic if it is open at both ends.
times the density of hydrogen. (c) Second overtone if it is closed at one end.
9. The equation of a transverse wave travelling along the x- (d) Seocnd overtone if it is open at both ends.
axis is given by 15. A policeman blows a whistle of frequency
y = 10 sin p (0.01x – 2.00t) 300 Hz as a car speeds past him with a velocity of 18 km/
where y and x are expressed in cm and t in sec. Find the hour. Find the change in frequency as heard by the driver
amplitude, frequency, velocity and wavelength of the wave. of car just as he passes the policeman. (Velocity of sound =
10. The equation of a wave travelling in x direction on a string 340 m/s)
is 16. Two engines pass each other in opposite directions with a
y = (3.0 cm) sin [(3.14 cm–1) x – (314 s–1)t] velocity of 60 kmph. One of them is emitting a note of
(a) Find the maximum velocity of a particle of the string. frequency 540 Hz. Calculate the frequencies heard in the
(b) Find the acceleration of a particle at x = 6 cm and at other engine before and after they have passed each other.
time t = 11 s. Velocity of sound = 316.67 ms–1.
11. A string 50 cm long and weighing 1 g produces a note of 17. Two closed organ pipes when sounded together produe 12
100 Hz on plucking. What is the tension in the string? beats in 4s at a temperature of 24°C. Find the temperature at
which the number of beats produced is 16 during the same
12. Write down the equation for a wave propagating with period.
velocity 330 m/s and having frequency 110 Hz. The 18. A train moves towards a stationary observer with velocity
amplitude is 0.05 m. 1
13. Two tuning forks A and B produce 4 beats/s. On loading B of th of the velocity of sound. The whistle of the engine
30
with wax, 6 beat/s are heard. If quantity of wax is reduced, regularly blows after one second. What is the interval
the number of beats per second again becomes 4. Find the between successive sounds of whistle as heard by the
frequency of B if the frequency of A is 256 Hz. observer ?
14. If the fundamental frequency of an organ pipe is 150 Hz, 19. A stone is dropped into a well in which water is 78.4 m deep.
what is the frequency of After how long will the sound of splash be heard at the
top? Take velocity of sound in air = 332 m/s

HOTS/Exemplar Questions
Very Short Answer Questions [1 Mark Qs.] (a) Calculate the time at which the second curve is plotted.
(b) Mark nodes and antinodes on the curve.
1. In mechanics, a simplification model of massless strings is (c) Calculate the distance between A¢ and C¢.
often adopted. Why is this not a good model when 3. Show that when a string fixed at its two ends vibrates in
discussing waves on strings? [HOTS] 1 loop, 2 loops, 3 loops and 4 loops, the frequencies are in
2. A sonometer wire is vibrating in resonance with a tuning the ratio 1 : 2 : 3 : 4. [Exemplar]
fork. Keeping the tension applied same, the length of the
wire is doubled. Under what conditions would the tuning Long Answer Questions [5 Marks Qs.]
fork still be in resonance with the wire? [Exemplar]
3. A sitar wire is replaced by another wire of same length and 1. You send a travelling wave along a particular string by
material but of three times the earlier radius. If the tension in oscillating one end. If you increase the frequency of the
the wire remains the same, by what factor will the frequency oscillations, do (a) the speed of the wave and (b) the
change? [Exemplar] wavelength of the wave increase, decrease, or remain the
same? If, instead, you increase the tension in the string, do
Short Answer Questions [2 or 3 Marks Qs.] (c) the speed of the wave and (d) the wavelength of the
wave increase, decrease, or remain the same? [HOTS]
1. Suppose you run past your stereo speakers, from right to
2. In the given progressive wave [ Exemplar]
left, or left to right. If you run rapidly enough and have
y = 5 sin (100 pt – 0.4 px)
excellent pitch discrimination, you may notice that the music
where y and x are in m, t is in s. What is the
that is playing seems to be out of tune when you are between
(a) amplitude
the speakers. Why? [HOTS] (b) wavelength
2. The pattern of standing waves formed on a stretched string (c) frequency
at two instants of time are shown in Fig. The velocity of two (d) wave velocity
waves superimposing to form stationary waves is 360 ms–1
(e) particle velocity amplitude
and their frequencies are 256 Hz. [Exemplar]

B D
displacement

A x in m
A¢ C C¢ E
t=0

t=? x in m
400 Physics

CHAPTER TEST

Time : 45 min. Max. Marks : 24

Directions : (i) Attempt all questions


(ii) Questions 1 to 5 carry 1 mark each.
(iii) Questions 6, 7 and 8 carry 3 marks each.
(iv) Questions 9 and 10 carry 5 marks each.

1. How does velocity of sound in air change when temperature rises by 1°C?
2. In which medium, do the sound waves travel faster, solids, liquids or gases ? Give reason.
3. If the pressure of a gas at constant temperature is increased four times, how the velocity of sound in the gas will be affected ?
4. A streel wire 70 cm long has a mass of 7g. If the wire is under a tension of 100 N, what is the speed of transverse wave in the
wire?
5. In a sound wave, a displacement node is a pressure antinode and vice versa. Why?
6. Explain why waves on strings are always transverse?
7. The speed of sound in moist air is greater than that in dry air, why ? Will the speed of sound in moist hydrogen be greater than
that in dry hydrogen ?
8. Waves are generated on water surface. Calculate the phase difference between two points A and B when
(i) A and B lie on the same wave front at a distance 2l.
(ii) A and B lie on successive crests separated by 1 m.
(iii) A and B lie on successive troughs separated by 1.5 m.
9. A drop of water 2 mm in diameter falling from a height of 50 cm in a bucket generates sound which can be heard from 5 m
Distance. Take all gravitational energy difference equal to sound energy, the transformation being spread in time over 0.2s.
Deduce the average intensity. Take g = 10 m/s2.
10. Describe analytically how standing waves are formed in closed organ pipes. Also, discuss normal modes of vibration
of the pipes.
Waves 401

Solutions
Very Short Answer Questions 15. In open pipe, all harmonics are present. So, the sound is
of higher quality than a closed pipe where only odd
1. Longitudinal waves exist in both air and inside the water.
harmonics are present.
2. Phase difference is 90°
16. A sound of single frequency is called a tone. A
3. The speed of sound waves in a medium is determined (i)
combination of tones of different frequencies is called a
elasticity and (ii) density of the medium.
note.
T 17. No, to observe beats by two light-sources the phase
4. As we know, velocity v = difference between the sources should change regularly.
m
In light sources, phase difference changes at random
Q Tension (T) and mass/unit length (m) are constants because the light-source consists of a large number of
so velocity will not change. atoms and each atom emits wave independently, so we
5. Human ear cannot identify any change in intensity of cannot observe the beats.
sound in less than 1/10 of a second due to persistence 18. Two instruments of same fundamental frequency produce
of hearing. So, the difference should be less than 10. different overtones and hence have different sounds.
gP Therefore, we can differentiate between the Sitar and Violin
6. Velocity of sound = . At constant temperature PV notes.
r
P Short Answer Questions
= RT = constant Þ constant
r
1. Liquids and gases cannot sustain shearing stress. So,
\ V = g ´ constant Þ velocity will remain the same.
formation of crests and troughs involving the change in
7. The speed of sound is more in solids than in air. So, the shape is not possible.
sound of train approaches faster through rails than air. 2. S tationary waves Progress ive waves
8. No, as the phase difference of two independent light (i)
sources changes randomly and round only at a given The disturbance
The disturbance
position. travels forward,
remains confin ed
being handed over
9. This is because intensity of sound increases with increase to a particular
from one particle to
in density of the medium and the density of liquid is more region and there is
the neighbouring
than that of air. no onward motion.
particle.
10. The nodes in a sitar string are replaced by the antinodes
in a flute. (ii) Energy is
There is no trans fer
trans ferred in the
11. Echo is the phenomenon of repetition of sound due to its of energy in th e
medium along the
reflection from the surface of a large obstacle. medium.
waves .
If s be the distance between the source and reflector, v
(iii) The amplitude of
the velocity of sound and t be the total time taken by
vibration of
sound to reach the listener after the reflection, then The amplitude of
particles varies
vibration of each
vt 340 1 from zero at nodes
s= = ´ = 17 m particle is s ame.
2s = vt or to maximum at
2 2 10
antinodes .
12. This increases mass per unit length and hence helps in
(iv)
obtaining a desired low frequency. The particles of the No particle of the
13. The frequency of vibration depends on the length of the medium at nod es medium is
air column. The liquid surface only causes the reflection are permanently at permanently at
rest. rest.
of waves. Hence frequency will change if oil of density
higher than that of water is used in the resonance tube.
3. The basic conditions for formation of stationary waves are
14. The flute is basically an open organ pipe. The location listed below :
of the open end can be changed by keeping the one hole (i) The direct and reflected waves must be travelling
open and closing the other holes. Thus the frequency of along the same line.
the note produced by the flute can be changed. (ii) For stationary wave formation, the superposing
waves should either be longitudinal or transverse. A
longitudinal and a transverse wave cannot
superpose.
402 Physics
(iii) For formation of stationary waves, there should not 9. Amplitude = 3cm.
be any relative motion between and oppositely w 36
travelling waves. Velocity of wave, v = = = 200 m/s
(iv) Amplitude and period of the superposing waves k 0.18
should be same. 1
10. As we know velocity v µ so if density becomes 4
4. r
Sound Waves Radio Waves
times, the velocity will become half.
Sound waves cann ot travel Radio waves can travel
1. 1. 11. Sound wave is a mechanical wave and its velocity is given
through free space. through free space.
2.
Velocity of soun d waves in
2.
Velocity of radio waves in
by g P . As the temperature changes, pressure and
air = 332 m/s free space = 3×108 m/s
r
Velocity of soun d increases Velocity of radio wavs is not density of the medium change, so velocity will also
3. 3.
with increase temperature. affected by in temperature.
change.
Sound waves within Light is an electromagnetic wave which doesn't need a
Radio waves cannot be
4. audible frequen cy ran ge 4. medium to propagate. Even if there is a medium, the speed
d etected by ear.
can be heard. doesn't depend on the temperature of the medium.
5. Superposition of two waves close to each other travelling 12. All overtones have frequency. Only the frequencies which
in the same direction produces interference. The energy are integral multiple of the fundamental, are called
gets redistributed. It is minimum of zero at points of harmonics rest are not called as harmonics. Hence all
destructive interference and maximum at points of overtones are not harmonics but all harmonics are
constructive interference. It is to be noted that the overtones.
interference minima may not be points of zero energy unless
13. Initially, the fundamental frequency of the sonometer wire
the frequencies and amplitudes of the superposing waves
are exactly equal. is equal to the frequency n of the tuning fork. When the
The superposition of two similar waves (waves length of the wire is made twice, its fundamental frequency
having same amplitude and period) travelling in opposite becomes n/2. But it now produces a first overtone of
direction produce stationary wave. The nodes have no frequency n, vibrating in two segments. Hence the wire
vibration of particles but antinodes have maximum can still vibrate in resonance with the tuning fork.
amplitude of vibration. 14. Suppose the organ pipe has both ends open. Then its
fundamental frequency of vibration will be
gP
6. Speed of sound waves, v = v 1 gP
r n= =
2 L 2L r
From kinetic theory of gases, (i) As the temperature increases, the velocity of sound
1 2 3P increases, so to fix n constant, the value of L must
Pressure = P = θC or C < be increased.
3 θ (ii) If hydrogen is filled in the pipe in place of air, the
value of density r decreases. To keep n constant,
v g g
[ = or v = ×C must be increased.
C 3 3 (iii) The pressure has no effect on the velocity of sound
7. The sound is heard due to variation in pressure, which provided the temperature is constant. It does not
is given by affect resonance.
DP = – Elasticity × Strain
At the antinodes, amplitude is maximum but strain is Long Answer Questions
minimum. At the nodes, the amplitude is minimum but 1. When two sets of progressive waves either both
strain is maximum. So variation in pressure is maximum at longitudinal or both transverse having the same amplitude
the nodes. Hence a loud sound is heard at node not at and same frequency travel with the same speed along the
antinode. same straight line but in opposite directions superpose
8. The speed of sound in water is nearly four times the speed each other, a new set of wave is formed, which is called
of sound in air. From Snell’s law of refraction, stationary or standing waves.
sin i va 1 Characteristics of stationary wave:
m= = = = 0.25 I. They do not propagate in any direction. Also, they
sin r vw 4 do not transfer any energy in the medium.
For refraction, rmax = 90°, so (sin i)max = 0.25. Hence imax II. There are certain points in the medium where the
= 14°. Consequently, most of the sound produced in air particles are permanently at rest, these are called
and incident at Ð i > 14° gets reflected back in air and nodes.
very small amount is refracted into water. Hence a person III. There are some other points, which vibrate about
deep inside water cannot hear the sound produced in air. their mean positions with largest amplitude. These are
called antinodes.
Waves 403

When a string under tension is set into vibrations, mode or first harmonic.
transverse harmonic wave propagates along its length. When the string vibrates in two segments, 3 nodes and
When both the ends of the string are fixed, reflected and 2 antinodes are formed and first overtone or second
the incident waves superimpose on each other, resulting harmonic is produced.
in the formation of transverse stationary wave. When the string vibrates in three segments, 4 nodes and
Let y1 be the incident wave represented by 3 antinodes are formed.
2. Newton assumed that the propagation of sound in air is
2p an isothermal process. The heat generated in compression
y1 = a sin (vt - x)
l is dissipated in rarefaction. So, the temperature of the
Then the reflected wave will be medium remains constant. So, Boyle's law is valid.
According to Newton : If P, V are initial pressure and
2p volume of the gas and dP, dV are the changes in pressure
y2 = - a sin (vt + x )
l and volume due to propagation of sound then
as the phase change of p takes place on reflection from a PV = (P + dP) (V – dV)
rigid boundary. PV = PV – PdV + VdP – dP.dV
The resultant wave will then be represented by the Product of dP.dV is small and is neglected.
superposition principle V dP dP
\ PdV = VdP Þ P = =
y = y1 + y2 dV dV / V
æ 2p 2p ö vol.stress
= çè sin (vt - x) - sin (vt + x) ÷ =
l l ø vol.strain
\ P = Bulk modulus = B
æ 2p ö æ 2vt ö æ 2p ö æ 2 x ö Velocity of sound in a gaseous medium
2a cos ç ÷ ç × sin ç ÷ ç - ÷
= è l ø è 2 ÷ø è l øè 2 ø B P
= v= =
2p 2p r r
y = - 2 a cos
vt sin x
l l At N.T.P, velocity of sound comes out to be
This is the wave function for a standing wave. For this
760 ´ 10 -3 ´ 13.6 ´10 3 ´ 9.8
2p 2L v = = 280 m/s but the
wave, if L = np or l = the position of nodes and 1.29
l n experimental value is 332 m/s
hence wavelength and frequency of set up wave can be Laplace pointed out the errors as,
predicted. (a) Air is a bad conductor of heat,
(b) Compressions and rarefactions occur so rapidly that
the heat generated during compression cannot be
dissipated in rarefaction. So, the temperature of the
N N medium doesn't remain constant. Therefore, the
A process is not isothermal but adiabatic.
\ PVg = constant
Fig. (a) Þ PVg = (P + dP) (V – dV)g
L = l1 / 2 dP
Simplifying we get, gP = =B
dV / V

N gP
A \ speed of sound v = ; g = 1.4 for air, and the
N N r
A
value of v is found to be correct.
Fig. (b) 3. The equation of a plane progressive wave travelling with
a velocity v along positive x direction is given by
L = l2
é 2p ù
y ( x, t ) = a sin ê (vt - x) + fo ú
ël û
Its initial phase, fo = 0 then
N N
A N A N A é 2p ù
y(x, t) = a sin ê (vt - x )ú
ël û
Fig. (c)
L = 3l 3 / 2 d
Velocity of the particle = [ y ( x, t )]
dt
When the string vibrates as a whole in one segments, 2
d é ì 2p üù
nodes are formed at 2 fixed ends and an antinode is formed = ê a sin í (vt - x) ý ú
at the middle, and the string is said to vibrate fundamental dt ë îl þû
404 Physics

ì 2p ü 2pv where A = 2a cos p( n1 - n2 )t = amplitude of the resultant


= a cos í (vt - x )ý ´
îl þ l wave.
Wave velocity = v A will be maximum when
2p ì 2p ü cos p( n1 - n2 )t = ±1 = coskp
\ Particle velocity = a cos í (vt - x)ý × wave
l îl þ p(n1 - n 2 )t = kp
velocity
k
4. In open organ pipe always two antinodes are formed at Þ t= ; k = 0, 1, 2, .....
the two open ends as the air can vibrate with maximum n1 - n 2
amplitude. Time interval between successive maxima
(a) 1st normal mode of vibration:
1
=
n1 - n 2
A A
N \ Frequency of maxima = n1 - n 2

L = l1 / 2 'A' will be minimum when cos p( n1 - n2 )t = 0


\ Fundamental frequency of vibration or 1st p
= cos(2 k + 1)
v v 2
harmonic = n1 = =
l1 2 L p 2k + 1
(b) 2nd mode of vibration: p(n1 - n2 )t = (2 k + 1) t=
2 2( n1 - n2 )
Time interval between successive minima
A A A 1
N N =
n1 - n 2
L = l2
\ Frequency of minima = n1 - n 2
\ Frequency of 2nd harmonic or 1st overtone
v \ Beat frequency = n1 - n 2
v
= n2 = Þ n2 = = 2n1 6. The phenomenon of the change in apparent frequency of
l2 L sound due to relative motion between the source of sound
(c) 3rd mode of vibration: and the observer is called Doppler effect. Let S and O be
the positions of source and observer respectively and v
and n be the velocity and frequency of the source of
A A A A sound, respectively.
N N N
v
L = 3l 3 / 2
O
Frequency of 3rd harmonic or 2nd overtone S
v 3v vs
= n3 = = O
l3 2 L S S¢
v – vs
\ n 3 = 3n1
\ The harmonics are all integral multiples of the (i) Source (S) in motion and observer (O) stationary –
fundamental frequency When S is at rest, it will emit waves in one second and
\ All overtones are present. these will occupy a space of length v in one second.
5. When two sound waves of slightly different frequencies, If l = wavelength of these waves, then
travelling in a medium along the same direction, superpose
on each other, the intensity of the resultant sound at a v
l= ...(i)
particular position rises and falls alternately with time. This n
is called beats formation and the phenomenon of waxing Let vs = velocity of source moving towards O at rest and
and waxing of sound is called beats. let S reaches to S¢ in one second. Thus the sound waves
Let the equations of the two superposing waves are y1 will be crowded in length (v – vs). So if l¢ be the new
= a sinw1t = a sin 2pn1t wavelength,
and y2 = a sinw2t = a sin 2pvf2t v - vs
According to the principle of superposition Then l'=
The resultant displacement, y = y1 + y2 n
= a sin 2pn1t + a sin 2 pn 2t if n¢ be the apparent frequency, then
= 2a cos p ( n1 - n2 ) t. sin p ( n1 + n2 ) t v v
n' = = n
= A sin p ( n1 - n2 ) t. l ' v - vs
Waves 405

\ n ' > n i.e. when S moves towards O, the apparent Numerical Questions
frequency of sound waves is greater than the actual
frequency. v 360
1. Here, wavelength, l = = = 0.72 m
(ii) Observer (O) in motion and source (S) stationary n 500
Let vo = velocity of observer moving towards S at 2p
rest. Q Phase difference = × path difference [
l
v
2p
O Df = Dx
l
S
l 0.72 p
O¢ v0 or, Dx = Df = ´ = 0.12 m
O 2p 2p 3
S
v0 v 100
2. Wavelength, l = = = 0.25 m ;
As the observer moves towards S at rest, the velocity n 400
of sound waves w.r.t. the observer is v + vo. 2p 2p æ 20 ö
Then apparent frequency Df = Dx = ´ç ÷ = 1.6p rad
l 0.25 è 100 ø
v + vo v + v0
n² = = v v 340
l v 3. Wavelength, : l = = = 0.034 m = 3.4 cm;
n 10 ´ 103
It can be seen that n² > n
(iii) Both source, S and observer O are moving l 3.4 p
Dx = Df = ´ = 0.57 cm
We know that 2p 2p 3
When S moves towards stationary O 4. Velocity, v0 = v l0 = 220 × 1.5 = 330 m/s;
v
Then n¢ = n
v - vS T 273 + 27
vt = v0 = 330 = 345.93 m/s
and when O moves towards stationary S. T0 273
v + vo vt 345.93
Then n² = n lt = = = 1.57 m;
v n 220
Case I : When both S and O are moving towards each other l t - l 0 = 1.57 – 1.5 = 0.07 m
Due to the motion of observer the frequency will
5. Let the velocity (vt) be double at T°C
change from n² to n²¢ so
T T
v + vo vt = v0 or 2v0 = v0
n¢¢¢ = n¢ S O T0 273
v

æ v + vo ö T
v + vo v or 2= or T = 4 × 273 = 1092 K
= ´ n=ç ÷n 273
v v - vS è v - vS ø or 1092–273 = 819°C
Case 2: Both S and O move in same direction i.e. vo can be
replaced by –vo in above relation gP
6. Speed of sound, v = ; For H2, P and g are same as that
r
æ v - vo ö S O
\ n²¢ = ç v - v ÷ n vH ra ra
è Sø S O of air, = = =4
Case 3 : Both S and O are moving away from each other.
va rH (1/16) ra
When source moves away from O or, vH = 4 va = 4 × 332 = 1328 m/s
7. Let at temperature t the velocity of sound in (H2) be the
æ v – vo ö
same in oxygen at 100°C
n¢ = ç v + v ÷ n S O
è ø
S S
When observer moves away from S the frequency n²
O
Q (vH ) = (vO )100
2 2
[

will change to n²¢


æ v – vo ö
( v H )0
2
273 + t
273
= vO2( )0 273 + 100
273
v - vo n
\ n²¢ = n¢ = ç
v è v + vS ÷ø æ vH ö 273 + 100 r0 2 16 4
Þ ç
2
÷ = = = =
è vO2 ø0 273 + t rH2 1 1

373 373
[ =4 Þ = 16
273 + t 273 + t
Þ t = – 249.7°C
406 Physics
8. According to question, y = 10 sin p (0.01 x – 2.00 t); (c) In a pipe closed at one end, the frequency of second
2p overtone = 5n1 = 5 × 150 = 750 Hz.
y = a sin (x – vt) (d) In a pipe open at both ends frequency of 2nd overtone
l
= vf2 + 3vf1
2p 15. Before passing, the listener moves towards the source,
Amplitude a = 10 cm; and = 0.01p
l ( v + vL ) (340 ∗ 5)
n' = ´n = ≥ 300 Hz
2 v 340
Þ l= = 200 cm
0.01 After passing, the listner moves away from the sound
2p (v + vL ) æ 340 - 5 ö
v = 2p Þ v = l = 200 cm/s ; n" = ´n=ç ´ 300 Hz
Velocity
l v è 340 ÷ø
Frequency, [ µ ' – µ " = 8.82 Hz
v 200 16. Here, vs = v0 = 60 kmph
n= = = 1s–1
l 200 5
10. Displacement equation, y = 3 sin [3.14x – 314t] = 60 ×= 16.67 ms–1
18
dy n = 540 Hz
v= = 3 cos[3.14x – 314t] × 314 n ¢= ?
dt v = velocity of sound = 316.67 ms–1
Þ vmax = 3 × 314 × 1 = 9.4 m/s. Case A : Before passing each other :
dv v = 316.67 ms–1
acceleration = a = = – 3 sin [3.14x – 314t] × (314)2 vs = v0 = 16.67
dt
n ¢= ?
= – 3(314)2 sin [3.14 × 6 – 314 × 0.11] Using the relation
= – 3(314)2 sin (6p – 11p) = – 3(314)2 sin (– 5p) v + v0 , gives
= 3(314)2 sin (4p + p) = 3(314)2 sin p = 0 n' = n
v – vs
1≥10,3 316.67 + 16.67 333.34
11. Mass per unit length, m = kg/m n' = ´ 540 = ´ 540 = 600 Hz
50 /100 316.67 - 16.67 300
Þ m = 2 × 10–3 kg/m Case B : After passing each other :
vs = – 16.67 ms–1
1 T
Frequency = v = Þ T = 4L2 n 2m v0 = – 16.67 ms–1
2l m v = 316.67 ms–1
æ 1 ö2 v - v0
Þ T = 4≥çç ÷÷ ≥(100)2 ≥ 2≥10,3 < 20 N \ n' = n, gives
èç 2 ø÷ v + vs

æ xö 316.67 - 16.67
12. Displacement, y = 0.05 sin 2p ç110t ± ÷ ± f0 n' = ´ 540
è 3ø 316.67 + 16.67
v = 330 m/s, n = 110 Hz 300
= ´ 540 = 485.92 Hz = 486 Hz
v 330 333.34
[ l= = = 3m and a = 0.05 m
n 110 17. Let n1 and n2 be the frequencies of sound produced by
[ The wave quation is the two organ pipes.
ì 2p 2p ü v0 = velocity of sound at 27°C
y = a sin í vt ± x ± fo ý and vt = velocity of sound at t°C
îl l þ l1, l2 be their lengths respectively.
é æ xö ù v0 v
= 0.05 sin ê 2 p ç110t ± ÷ ± f o ú \ n1 = and n2 = 0
ë è 3 ø û 4l1 4l2
[f0 is the initial phase of the wave]
v0 æ 1 1 ö
Possible frequncies of B are 256 ° 4 = 260 or 252. \ n1 - n2 = -
13. 4 çè l1 l2 ÷ø
As no. of beats is 4 when B is loaded with wax therefore
final frequency must be less than the initial frequency. 12
[ Initial frequency of B = 260 Hz Here, n1 - n2 = = 3 (given)
4
14. (a) Only odd harmonics exist in organ pipe closed at one
end. v0 æ 1 1 ö
\ 3= - ...(i)
[ 2nd harmonic does not exist. 4 çè l1 l2 ÷ø
(b) n2 = 2 n1 = 2 × 150 = 300 Hz.
Waves 407

vt æ 1 1 ö 16 Short Answer Questions


Similarly, - = = 4 ...(ii)
4 çè l1 l2 ÷ø 4
1. When you are between the speakers, you are running
(ii ) vt 4 away from one of them and toward the other. Thus, there
gives v =
(i ) 0 3 is a Doppler shift downward for the sound from the speaker
behind you and a Doppler shift upward for the sound
vt 273 + t 4 273 + t
Also = or = from the speaker ahead of you. As a result, the sound
v0 273 + 27 3 300
from the two speakers will not be in tune. A calculation
16 shows that a world-class sprint runner could run fast
or 273 + t = ´ 300
9 enough to generate about a semitone difference in the
1600 1600 - 819 sound from the two speakers.
or t = - 273 = 2. (a) 9.8 × 10 –4s. (b) Nodes-A, B, C, D, E. Antinodes-A¢, C¢.
3 3 (c) 1.41 m.
781 nv
or t= = 260.3°C
3 3. From the relation, v = , the result follows.
2L
18. Actual time interval between successive blows of whistle
= 1 sec.
\ Actual frequency of whistle, n = 1 Hz Long Answer Questions
Let v = velocity of sound 1. (a) The speed of the wave is independent of the fre-
v quency hence the speed remains the same even if the
\ velocity of source, vs =
30 frequency is increased.
\ apparent frequency of the whistle is
v
v v (b) As l = , so the wavelength decreases with increase
n' = ´n= ´1 f
v – vs v
v– in frequency.
30 (c) Velocity of wave in a string is given by
30 T
or n' = Hz v=
29 . The speed of wave increases when the ten-
m
\ Apparent time interval between two successive sounds
of whistle is sion (T) in the string is increased.
(d) With the increase in the tension (T) in the string the
1 29 speed of wave increases hence from the relation
t= = s
n ' 30
19. Let t1 be the time taken by the stone to go down and touch v
l= , the wavelength also increases.
the water surface in the well. f
1 2 1 2. (a) 5m, (b) 5 m, (c) 50 Hz, (d) 250 ms–1, (e) 500 p ms–1.
[ h = ut + gt Þ 78.4 = 0 + × 9.8 × t12
2 2 CHAPTER TEST
78.4 ≥ 2
Þ = t12 Þ t1 = 4s. 1. Velocity of sound in air increases by 0.61 m/s when
9.8 temperature rises by 1°C
If t2 be the time taken by the sound to come up then t2 2. Sound waves travel in solids with highest speed. This is
distance 78.4 because the coefficient of elasticity (E) of solids is much
= < = 0.25 greater than coefficient of elasticity of liquids and gases.
speed 332
E
78.2 Vsound =
[ Total time = t1 + t2 = 4 + = 4 + 0.2 = 4.28s r
332
gP
HOTS/EXEMPLAR QUESTIONS 3. Unchanged as, v = , any increase in ‘P’ produces
r
Very Short Answer Questions P
corresponding increase in ‘r’ so that = constant. Hence
r
1. A wave on a massless string would have an infinite speed of
the velocity of sound in a gas is independent of pressure.
propagation.
,3
= m = 7 ≥10
2. Wire of twice the length vibrates in its second harmonic. Thus if Mass
4. = 10–2 kg/m;
the tuning fork resonaters at L, it will resonate at 2L. Length 0.7
1 T 100
3. 1/3. Since frequency a m = pr 2 r =
m T = 100 N, \ frequency, v = = 100 m/s
m 10 -2
408 Physics
5. Displacement node is a point of zero displacement and the 4
pressure variation is maximum at this point. ´10-9 ´103 ´10 ´ 0.5 1
Hence, it is a pressure antinode. =3 = ´ 10 -6
0.2 ´ 25 ´ 4 3
6. A string is nonstretchable, so compressions and
rarefactions cannot be produced in strings. So, longitudinal 10
waves in strings are not possible. Strings do have elasticity, = ´ 10-7 = 3.33 × 10–7 Wm–2
3
so, the waves on strings are transverse.
10. In a closed organ pipe, the air inside the pipe vibrates
7. The density of water vapour is less than that of air. So the longitudinally and the wave is reflected from the closed
density of air mixed with water vapour (moist air) is less end. When the incident and the reflected waves
than that of dry air. Hence the speed of sound in moist air is superimpose on each other, standing wave is formed.
greater than that in dry air ( v µ 1/ r ). However, the density (a) 1st mode of vibration:
of water vapour is more than that of hydrogen, so the When the air vibrates in such a way that a node is
density of moist hydrogen is more than that of dry formed at the closed end and an antinode is formed
hydrogen. Consequently, the speed of sound in moist at the open end then L = l1/4
hydrogen is less than that in dry hydrogen. x=0 x=L
8. (i) On the same wavefront phase difference = 0
whatever be the linear distance between the points.
N A
(ii) Path difference between two successive crests = l
[ Phase difference = 2p radian. L = l1 / 4
(iii) Path difference between two successive troughs = l Frequency of this mode also called fundamental mode
[ Phase difference between A and B of vibration is
= 2p radian
9. Here, r of water = 103 kg/m3 v v
n1 = =
Diameter D = 2 mm l1 4 L
2 (b) 2nd mode of vibration: Here two nodes and two
\ Radius, r = = 1mm = 10–3 m antinodes are formed
2
4 3
Mass m = pr r
3 N A A
h = 50 cm = 0.50 m N
g = 10 m/s2
d=5m L = 3l 2 / 4
A = area in which sound is heard = 4pd2 v 3v
I = intensity of sound = ? Frequency = n2 = = = 3n1
l2 4 L
energy transfered
Now I = This is called the third harmonic or first overtone.
tim e × area
(c) 3rd mode of vibration: Here, three nodes and three
P.E. of drop mg h antinodes are forrmed.
= = ....(i)
time × area t × A
4
Now, mgh = pr3rgh N A A A
3 N N
A = 4 pd2
= 4 × p × 52 ....(ii) L = 5l 3 / 4
t = 0.2 s v 5v
\ From (i) and (ii), we get intensity of sound, n3 = = = 5 n1
l3 4L
4
p(10-3 )3 ´103 ´10 ´ 0.5 This is the 5th harmonic or 2nd overtone.
I= 3
Therefore only, odd harmonics are present.
0.2 ´ 4p(5) 2

¿¿¿
VALUE BASED QUESTIONS
1. A small hair piece has fallen into the eye of Mahesh. It 5. An old woman, holding a purse was crossing the road. She
caused itching sensation in his eyes seeing that Hari, who was lean and weak and was unable to walk. Suddenly a pick
is a friend of Mahesh, took him to the eye specialist. The pocket snatches away her purse. A school student of class
Doctor removed it. Mahesh expressed his gratitude to Hari. IXth having seen this incident tries to help that old lady. He
(a) According to you, what values are possessed by Hari. informs the police inspector who stands nearby. The
inspector collects the purse from the pickpocket and hand it
(b) A student measure the thickness of a human hair by
over to the old lady.
looking at it through a microscope of magnification
100.He makes 20 observations and finds that the (a) According to you what values were displayed by the
student?
average width of the hair in the field of view of the
microscope is 2.7 mm. What is the estimate on the (b) The police inspector in a jeep is chasing the pickpocket
thickness of hair? on a straight road. The jeep is going at its maximum
speed 'v'. The pickpocket rides on the motorcycle of
2. Once upon a time, Mrs. Sharma was playing the game of
his friend when the jeep is at a distance 'd' away and
carrom board with her son arnav. She decided to give him
the motorcycle starts with a constant acceleration 'a'.
an idea of different types of motion. She told him that the
motion of the carrom coin is motion in a plane. She asked Show that the pickpocket will be caught if v ³ 2ad .
arnav to observe the motion in his day to day life. To explain 6. Inter schools sports competition was scheduled to be held
motion in a straight line, she gave her the example of a ball in akash's school. With the intention to win in the competition
thrown vertically up. akash practiced high jump every day for about a month. He
(a) What according to you, are the values displayed by participated and won Ist position in the interschool sports.
Mrs. Shrama ? (a) Comment upon the values Akash possesses.
(b) Differentiate between distance and displacement. (b) Why does an athlete run some steps before taking a
3. Aditi a student of class IX was reside in hostel. She was jump?
suffering from malaria. The hostel area was full of 7. Saurabh the grandson of Mrs. Kaushik was visiting her in
mosquitoes. She was not having mosquito net. Her friend his summer holidays. Her royal estate had many sprawling
Needhi has an extra net. She gave it to Aditi. Also she took laws. One day, just for fun, he started pushing and pulling a
Aditi to a Doctor. After a week Aditi became normal. lawn roller. He felt that it was easier to pull a lawn roller than
(a) According to you what, qualities displayed by Needhi? to push it. He asked Mr. Thomas, the estate officer, the
(b) The mosquito net over a 7 m × 4m bed is 3m high. The reason of easier pull and difficult push. Mr. Thomas was
surprised at this observation of Saurabh. He talked to the
net has a hole at one corner of the bed through which
gardeners but they knew nothing. Finally, he approached
a mosquito enters the net. It flies and sits at the
the Physics teacher of Science college run by Mrs. Kaushik.
diagonally opposite upper corner of the net(i) Find the
magnitude of the displacement of the mosquito The Physics teacher explained to Thomas and Thomas
(ii)Taking the hole as the origin, the length of the bed explained this fact to Saurabh. Saurabh was overjoyed and
as the X-axis, its width as the Y-axis and vertically up thanked Thomas.
as the Z-axis, write the components of the displacement (a) What according to you, are the values displayed by
vector. Thomas ?
4. Once Sachin and Saurabh went to visit an ancient fort. (b) Why is it easier to pull a roller than to push it ?
Having seen a big stone falling from the top of a dome of the 8. Arav went to market with his father. They decided to visit
fort. Sachin pulled his friend Saurabh away. The stone hit Big Bazar to purchase certain goods. There he has noticed
Saurabh slightly and he got hurt. But he was saved from a an old lady struggling with her shopping. Immediately he
major accident. showed her the lift and explained to her how it carries the
(a) What values do you find in Sachin? load from one floor to the next. Even then the old lady was
not convinced. Then suraj took her in the lift and showed
(b) From the top of a tower 100 m in height, a ball is dropped
her how to operate it.
and at the same time another ball is projected vertically
upwards from the ground with a velocity of 25 m/s.Find That old lady was very happy and thanked Arav.
when and where the two balls meet. Take g = 9.8 m/ (a) According to you, what values were possessed by
sec2 . Arav?
410 Physics
(b) An elevator can carry a maximum load of 1800 kg is mechanism. Her mother told her the actual mechanism of
moving up with a constant speed of 2 m/s , The gaining speed in a swing.
frictional force opposing the motion is 4000 N. Find the (a) What values were displayed by Kamini?
total downword force on the elevator.
(b) Specify which energy is maximum and which is minimum
9. During monsoon, due to heavy rains and high speed winds when Mohini's swing was at its lowest position?
a tree broke and fell on the road and disturb the
(c) Define energy. What is its SI unit?
communication. To remove that broken part of the tree
Ramesh and Mahesh who are local shop owner came out. 13. Niyati, a student of class IX, was struggling to understand
But as the log was too heavy they were unable to remove it. the universal law of gravitation. Then her friend Rajiv who
A passer by came to their help with a few solid heavy sticks. came to him and explained how the planet moves around
Consequently the broken part of the tree was put aside and the sun obeying Kepler's law of planetary motion and
the communication on the road began as usual. universal law of gravitation.
(a) According to you what values were displayed by (a) What qualities of Rajiv impressed you?
passer by?
(b) State the universal law of gravitation
(b) Name the energy stored when the heavy log of the tree
14. Reena and Veena were classmates. Reena was a bright girl
was lifted to some height.
and had a quick grasping power. One day Reena noticed
If it is lifted to 80 cm from the earth surface then that her classmate Veena felt burdened while carrying her
determine the energy stored in heavy load. (Assume
school bag in her hand or on her shoulders although it was
the mass of the log is 1.2 tonne)
as heavy as it was her own. Reena then told to Veena to use
10. Mohan Lal lives in a village. He is a farmer. One day he school bag having a broad and thick strap instead of strap
decided to meet his son who live in a town which is 200 km made of thin and strong string.
from his village. Mohan Lal was waiting for a bus at a bus
(a) What values were displayed by Reena?
stand with his heavy luggage bag in his hands. He was
tiring and used to shift his luggage bag from one hand to (b) Why Reena suggested Veena to use school bag having
another and so on. Kapil a student of class IX was seeing a broad and thick strap?
this. He felt uneasy and requested Mohan Lal to put his (c) Define pressure and give its SI unit.
luggage on the floor of bus stand near him so that he may 15. Navneet saw gardner planting seeds in the land. Gardener
be free. Initially Mohan Lal looked suspiciously towards does not know methods of growing plants. Then he decided
Kapil but on persuation he agreed and put his luggage on to make gardener aware of this. He explained the importance
the floor. Now he was relieved and peaceful. of ploughing the land before planting the seeds. Gardener
(a) What values were shown by Kapil? is convinced with his ideas. He planted accordingly. The
(b) Was the Mohan Lal doing any work while holding the plants grown successfully.
bag in his hand. Why? (a) What can you say about Navneet?
11. Kanika a student of class VIII was preparing a chart for a (b) What is the utility of ploughing a field? Does it help
science project. Her mother who was preparing food for the soil to retain moisture?
guests asked Kanika to help her in kitchen. Kanika took 2-3
16. Madhu’s younger sister Rama was crying. Then she took a
minutes and joined her mother to help kitchen work. Again
piece of camphor and put it in water. By seeing the camphor
after finishing kitchen work, she continued her chart work.
piece dancing on the surface of water, Rama stopped crying.
By the time Kanika completed chart she felt tired and hungry.
She took her dinner and went to bed. (a) What can you say about the qualities of Madhu?

(a) What values were displayed by Kanika? (b) Why do small pieces of camphor dance on the surface
of water?
(b) Was Kanika actually doing work form physics point of
views while preparing chart for a science project? Give 17. Sarita was surprised to see oil spreading on to the surface
reason. of water and asked her brother to explain why oil spreads on
to the surface of water. Her brother explained to Sarita the
12. Kamini live with her 10 years old daughter Mohini in a Arya
reason behind it. By going through the explanation she
Samaj Society. These is a well maintained society park.
thought of learning more about the other scientific
Mohini was swinging on a swing in her society park. She
phenomenon also.
wanted to increase the speed of the swing. For that she
started pushing the ground backward harder with her feet. (a) What qualities do you find in Sarita?
When her swing was at its lowest position during its (b) Why does oil spreads over the surface of water but
oscillation. She enjoyed it but did not show the actual water does not spread over the surface of oil?
Value Based Questions 411

18. The king of sicily suspected that the goldsmith had mixed (a) According to you, what are the values displayed by
some silver in his crown and cheated him. Without Raj ?
destroying the crown he wanted to know the truth. (b) Write a note on the concept of work, heat and internal
Archimedes was asked to find out whether the crown was energy.
made of pure gold as was alleged or whether it had been
22. Ram has to attend to an interview . He was not well. He took
adulterated with silver or any other metal.
the help of his friend Shyam. On the way office Ram felt
(a) What values were displayed by Archimedes? giddy, He vomited on his dress. Shyam washed his shirt. He
(b) How Archimedes find out whether the crown was made made Ram to drink enough amount of water. In spite of
of pure gold or adulterated with silver? doing ,a foul smell was coming from the shirt. Then Shyam
(c) What is Archimedes principle? purchased a scent bottle from the nearby cosmetics shop
and applied on Ram. Ram attended the interview, Performed
19. Having found his mother suffering from fever Raj took her
well. Finally he was selected.
to the doctor for treatment. While checking the status, the
doctor used a thermometer to know the temperature of the (a) According to you, what values are possessed by
body. He kept the thermometer in the mouth of the patient Shyam?
and noted the reading as 103° F. Doctor gave the necessary (b) The velocity of air molecules is nearly 600m/s. But the
medicines. After 3-4 hours while coming home, Raj asked smell of scent spreads very slowly, Why?
his mother, who is a science teacher, why mercury is used in 23. Amit a student of class XI has found the factors on which
a thermometer. Then his mother explained the reason. the time period of oscillation of a pendulum depends and
(a) What, according to you, were the values displayed by
Raj and his mother. l
arrived at the expression T = x . He wants to know how
g
(b) A newly designed thermometer has its lower fixed point
and upper fixed point marked at 4°Cand 96°C the length of the pendulum gets affected on the surface of
respectively. Calculate the temperature on this scale the moon for the same pendulum and arrived at the
corresponding to 50°C conclusion that it is l/6.
20. Raman noticed that his grandfather is suffering from fever. (a) What values does Amit possesses?.
He took him to the doctor. The doctor gave him some (b) If the length of a seconds' pendulum on the surface of
medicines. When the medicines were used he sweated much, the earth is 1m. What will be the length on the surface
after some time became normal.Raman enquired the Doctor of the moon?
about how his grandfather became normal .
24. While performing an experiment, to verify laws of reflection
(a) According to you what, values are possessed by of sound, Parth made experimental set up as shown below.
Raman? His classmate Kunj came there and by mistake he turned
(b) A child running a temperature of 101°F is given an tube T1 by 15° towards ON. Parth then turned tube T 2 by 15°
Antipyria which causes an increase in the rate of towards ON and asked Kunj to hear sound by placing ear at
evaporation of the body. If the fever is brought down T2.
to 98°F in 20 mts, what is the amount of heat lost by the
O Reflecting
body? . The mass of the child is 30 kg.
Surface
21. Sanaya was feeling uncomfortable because of the harsh
cold weather. On an impulse, she started rubbing her hands.
To her utter surprise, she found that her hands became warm.
T1 T2
She shared this observation with her elder brother Raj. Raj
started rubbing his hands and he also found that his hands
Sound Ear
became warm. Raj decided to find out as to how rubbing of Source
hands produces heat. Raj questioned his science teacher N
about this observation. The teacher explained to him that
when he rubs his hands, he does mechanical work. Due to (i) What values were displaced by Parth?
friction between the hands, some of the mechanical work (ii) Why Parth turned tube T2 by 15° towards ON after
gets converted into heat energy. Raj explained this to his Kunj turned tube T1 by 15° towards ON?
younger sister Sanaya who was thrilled with the knowledge 25. One day Subhash a student of class VIII and his father Mr.
of conversion of work into heat. She thanked her elder Ramesh was waiting to go across a railway crossing. Subhash
brother. jumped over the barrier and curiously put his ear on the
railway track to hear the sound. His father opposed Subhash
412 Physics
to do so and pulled him away from the railway track. Mr. (i) According to you what values were displayed by Ria?
Ramesh explained about the propagation of sound in different (ii) Why did Ria and Jiya get disturbed?
media.
(iii) Write the principal on which megaphone works.
(i) What values were displayed by Ramesh?
(iv) What is an echo? What should be the minimum distance
(ii) Can sound travel through vacuum? Why? between the source of sound and the reflector for
(iii) Two waves A and B are propagated in the same medium hearing a distinct echo?
Let frequency of the wave B be 6660Hz. Study the 27. Rohit went to Shimla with his uncle Pradeep. While walking
following figure and find out the velocity of waves A through the hills of Shimla, Rohit asked his uncle Pradeep if
and B. it was possible to measure the height of a distant hill. Pradeep
never thought of this earlier. But he decided to find out the
A possibility of measuring the height of a distant hill. He talked
B
to many of his friends but could not find a satisfactory
answer. Then he went to an engineering college and
Displacement

0 1 2 3 4 consulted a teacher. The teacher explained the method of


Distance
(in cm) measurement to Pradeep. Pradeep came home and explained
the same to Rohit. He became very happy and thanked his
uncle.
26. Ria and Jiya were classmates. They were preparing for their (a) What according to you, are the values displayed by
annual examination at Ria's residence. There came a hawker Pradeep ?
who was shouting through a mega phone to sell his products.
(b) How can we measure the height of a distant hill?
Both Riya and Jiya get disturbed. Ria enquired her father
about the instrument being used by the hawker. Her father
told them that it was a megaphone also known as speaking
tube or loud hailer.
Value Based Questions 413

SOLUTIONS
1. (a) Hari has presence of mind, a caring attitude towards ax2 + bx + c = 0
his friend, & helping nature towards others.
(b) The estimated thickness of hair = 2.7/100 –b ± b 2 – 4ac
from t =
= 0.027mm 2a
2. (a) Mrs. Sharma is a knowledgable lady. She likes her son 1
v ± v2 – 4× a ´ d
to learn through play. She inparts knowledge in a very t= 2
1
simple and intelligent manner. 2× a
2
(b) Displacement can be zero, negative and positive. But
distance is always positive. v ± v 2 – 2ad
t=
a
3. (a) Needhi has a caring attitude, and concern for others.

{( )}
This will be possible if v2 ³ 2ador, v ³ 2ad .
2 2 2
(b) (i) 74 m i.e., 7 +4 +3
6. (a) Akash has determination, he Plans and executes his
plan accordingly.
(ii) The components of the vector are 7m,4m, and 3m
(b) An Athlete runs some steps before taking a jump to
.
gain some initial momentum, which helps him to jump
4. (a) More observation, presence of mind and concern. more?
(b) In the first case h = ½gt2 For the second case 100-h = 7. (a) Thomas is a man of great confidence and great
determination and his problem solving ability is beyond
25 t - ½ gt2 where h is the height at which the two balls
doubt.
meet.
(b) There is more friction at the time of pushing as compared
i.e., 100 – h = 25t – h; 25t = 100; or t= 100/25 to pulling the roller.
= 4 sec. ; h=78.4m 8. (a) Suraj is sympathetic also has the attitude of helping
5. (a) The student is sympathetic towards others, helping, others and he has patience.
and applies his presence of mind in solving the (b) The downward force on the elevator is
problems, knows how to use public services. F = mg + Ff = (1800 × 10) + 4000 = 22000N
(b) Distance covered by the this time interval 9. (a) Helpful, social, concern for others
t=s+d
(b) Potential energy.
Now distance covered = velocity × time
Given: height h = 80 cm = 0.8 m
v ± v 2 – 2ad mass m = 1.2 tonne = 1.2 × 1000
Then, t =
a = 1200 Kg
The pickpocketter will be caught if t is real and positive Potential energy stored in the log
st , d = vt P.E. = mgh = 1200 × 10 × 0.8
1 2 = 9600 Joule.
Using, s = ut + at
2 10. (a) Open mindedness, empathy, concern for others.
Q u=0 (b) No, as work is said to be done, only if the applied force
1 2 moves a body through some distance along the
s= at direction of the force.
2
s+ d = vt ® s = vt – d …(i) Here displacement is zero.
1 2 \ Work done = Force × displacement = 0
s= at …(ii) 11. (a) Helpful, concern for her mother, dedication.
2
from eqn (i) and (ii) (b) No, in physics work is defined if force applied on object
displaces the object in direction of force.
1 2
vt –d = at Work (w) = force × displacement in the direction of
2 force.
12. (a) Kamini shares her knowledge with her daughter and
1 2
at – vt + d = 0 wants to improve her knowledge, concern towards her
2 daughter.
414 Physics
(b) When Mohini's swing was at its lowest position Kinetic gold.
energy is maximum and potential energy is minimum.
Sp. Weight
(c) Energy is defined at the capacity of doing work. Its SI
unit is same as that of work i.e. joule.
Weight of solid in air
13. (a) Rajiv shares his knowledge with his friends and wants =
Weight of an equal volume of water at 4°C
to improve his knowledge in the subject, and concern
towards his friends. (c) Archimedes’ principle: When a body is immersed in a
(b) Every two objects in the universe attract each other fluid wholly or partially, its weight is equal to the weight
and the force of attraction between the two bodies is of the fluid, displaced by the immersed part of the body.
directly proportional to the product of their masses 19 (a) Mother has interest in educating her son and explained
and inversely proportional to the square of distance that Mercury has got the following properties for being
used in thermometers
between the two masses.
(i) The expansion of Mercury is fairly regular and uniform.
14. (a) Helpful, caring nature, concern for others,
(ii) It is opaque and shining, hence can be easily seen
knowledgeable.
through the glass tube.
Force (iii) Mercury is a good conductor of heat ad has low thermal
(c) As, Pressure = capacity,
Area
(iv) Mercury does not wet the sides of the glass tube in
So School bag having broader and thicker strap i.e., which it is filled.
greater area exerts less pressure on the shoulders. (b) Let q be the temperature on the scale corresponding to
(c) Pressure is defined as the force acting per unit area of 50°C, then (q – 4)/(96 – 4) =(C – 0)/(100 – 0) =C/100 or q
the surface. Its SI unit is Newton per metre2 (N/m2) or = 50°C. Thus, the required temperature on the scale of
Pascal (Pa). the designed thermometer is 50°C.
15. (a) Naveet has good knowledge of agriculture. He is very 20 (a) Raman is responsible and he has concern for others,
much interested in putting his ideas into practice, uses inquisitiveness in gaining the knowledge
his knowledge to convince gardener. (b) Heat lost by the body = mDst = 30kg × 1000cal/kg/°C ×
(b) When the field is ploughed, the capillaries are broken. 1.67 °C = 50100cal
So water cannot rise to the surface and the soil is able where loss in temperature = (101 – 98) °C
to retain its moisture. =3 × 5/9 °F = 1.67°C.
16. (a) Madhu is responsible, helps her mother in looking after 21. (a) Raj is a curious boy and has great self confidence. He
her younger sister. is a seeker of knowledge. He has lots of love and
(b) When camphor is dissolved in water, the surface affection for his younger sister.
tension of water is decreased. Since camphor has (b) Work : It is the energy transfer brounght about by
irregular shape therefore it may dissolve more at one other means such as moving the piston of a cylinder
end than at the other end. This produces an unbalanced containing the gas by raising or lowering some weight
force due to which it moves .When it reaches a different connected to it.
region, the same process is repeated.
Heat : It is energy in transit between two objects or
17. (a) She has inquisitiveness, she wants know the scientific systems due to temperature difference between them.
reason behind the phenomena. It exists in the form of translational, rotational and
(b) The surface tension of the water is more than that of vibrational motion of molecules of a substance.
oil, therefore when oil is poured over water, the greater Internal Energy :Internal energy of a system is the
value of surface tension of water, pulls the oil in all energy possessed by the system due to molecular
directions. On the other hand, when water is poured motion and molecular configuration. The energy due
over oil, it does not spread over it because surface to molecular motion is called internal kinetic energy
tension of oil is less than that of water. (Uk) and that due to molecular configuration is called
18. (a) Research mindedness, accept the challenge, problem internal potential energy (Up).
solver. dU = dUk + dUp
(b) Archimedes find out it by comparing the weight of the 22 (a) He has presence of mind, serves others in need.
crown with the weight of the displaced water which (b) This is because the air molecules can travel only along
gave him the specific weight which, if less than that of a zigzag path due to frequent collisions i.e., brownian
gold, would indicate that the crown is adulterated with motion. Consequently, the displacement per unit time
is considerably very small or negligible.
silver if it is equal to that of gold, it is made of pure
Value Based Questions 415

23. (a) Amit is hardworking, thinks logically,having scientific = 6660 × 0.04


temper, and he is able to find solutions with patience. = 266.4 ms-1
(b) Since' l 'is proportional to' g ' the length of the pendulum As the medium same, therefore velocity of wave B =
on the surface of the moon will be 1/6m. velocity of wave A = 266.4ms-1.
24. (i) Understanding, dedication, Calm nature. 26. (i) Curiosity and dedication.
(ii) As sound waves after striking the reflecting surface (ii) Ria and Jiya were preparing for their examination. The
turned back with the same angle as that of the angle of loud sound produced through the megaphone breaking
incidence of the sound wave. Hence to hear clear sound their concentration hence they were not comfortable.
Parth turned T2 by 15° towards ON.
(iii) Reflection of sound.
25. (i) Caring and sharing knowledge, concern for his son,
(iv) Echo is the phenomenon of repetition of sound due to
awareness.
its reflection from the surface of a large obstacle. The
(ii) No, sound require material medium to propagate minimum distance between the source of sound and
propagation of sound is possible only in that medium the reflector for hearing a distinct echo is 17 metre.
which have elasticity and inertia.
27. (a) Pradeep is a man of great enthuseasm and strong
(iii) Frequency of B. fB = 6660 Hz
determination. He shows lots of confidence.
Wavelength of wave B, lB = 4 cm = 0.04m
(b) By applying method of Echo i.e., reflection of sound.
Velocity of wave B, VB = fB × lB
IMPORTANT FORMULAE, TERMS
AND DEFINITIONS
The SI system : It is the international
s1 + s2 + s3
system of units. At present internationally Average speed, Vav =
accepted for measurement. In this system t1 + t 2 + t 3
UNITS AND MOTION IN A
MEASURE- there are seven fundamental and two a1t1 + a 2 t 2
supplementary quantities and their STRAIGHT Average acceleration, aav =
MENTS LINE t1 + t 2
corresponding units are:
The area under the velocity-time curve
is equal to the displacement and slope gives
Quantity Unit Symbol acceleration.
1. Length metre m If a body falls freely, the distance covered by it in each
2. Mass kilogram kg subsequent second starting from first second will be in the
3. Time second s ratio 1 : 3 : 5 : 7 etc.
4. Electric current ampere A If a body is thrown vertically up with an initial velocity u, it
takes u/g second to reach maximum height and u/g second
5. Temperature kelvin K to return, if air resistance is negligible.
6. Luminous intensity candela cd If air resistance acting on a body is considered, the time
7. Amount of substance mole mol taken by the body to reach maximum height is less than the
Supplementary time to fall back the same height.
a
1. Plane angle radian rad For a particle having zero initial velocity if s µ t , where
2. Solid angle steradian sr a > 2 , then particle’s acceleration increases with time.
a
Dimensions : These are the powers to which the fundamental For a particle having zero initial velocity if s µ t , where
units are raised to get the unit of a physical quantity. a < 0 , then particle’s acceleration decreases with time.
Uses of dimensions Kinematic equations :
(i) To check the correctness of a physical relation. v = u + at (t) ; v2 = u2 + 2at (s)
(ii) To derive relationship between different physical quantities. 1 a
(iii) To convert one system of unit into another.
S = ut + at (t)2; Sn = u + (2 n - 1)
2 2
r
n 1u 1 = n 2u 2 applicable only when | a t | = a t is constant.
a b c at = magnitude of tangential acceleration, S = distance
n1[M1a Lb1T1c ] = n2 [M 2 L 2T2 ]
If acceleration is variable use calculus approach.
Significant figures : In any measurement, the reliable digits r r r
plus the first uncertain digit are known as significant Relative velocity : v BA = vB - vA
figures. If T is the time of flight, h maximum
Error : It is the difference between the measured value and height, R horizontal range of a projectile, a
true value of a physical quantity or the uncertainty in the its angle of projection, then the relations
measurements. among these quantities.
MOTION IN A
Absolute error : The magnitude of the difference between the gT 2
true value and the measured value is called absolute error. PLANE h= ...... (1);
8
Da1 = a - a1 , Da2 = a - a2 , Dan = a - an
gT2 = 2R tan a ....... (2);
R tan a = 4h ....... (3)
Mean absolute error
n 2u sin q u 2 sin 2 q
| Da1 | + | Da2 | +.....+ | Dan | 1 T= ;h =
Da =
n
= å
n i =1
| Dai | g 2g

Relative error : It is the ratio of the mean absolute error u 2 sin 2q u2


R= ; Rmax = when q = 45°
to its true value g g
Da For a given initial velocity, to get the same horizontal range,
or relative error = there are two angles of projection a and 90° – a.
a
Percentage error : It is the relative error in per cent. The equation to the parabola traced by a body projected
horizontally from the top of a tower of height y, with a
æ Da ö velocity u is y = gx2/2u2, where x is the horizontal distance
Percentage error = ç ´ 100%
è amean ÷ø covered by it from the foot of the tower.
(ii) PHYSICS
Work due to kinetic force of friction between two contact
gx 2 surfaces is always negative. It depends on relative
Equation of trajectory is y = x tan q - , which is
2u 2 cos 2 q displacement between contact surfaces. WFK = -FK (Srel ) .
parabola. S W = S DK , S W Þ total work due to all kinds of forces,
Equation of trajectory of an oblique projectile in terms of S DK Þ total change in kinetic energy..
æ xö SWconservative = -S DU ; SWconservative Þ Total work due
range (R) is y = x tan q ç1 - ÷
è Rø to all kinds of conservative forces.
Maximum height is equal to n times the range when the SDu Þ Total change in all kinds of potential energy.
projectile is launched at an angle q = tan–1(4n). velocity of separation
Coefficient of restitution e =
In a uniform circular motion, velocity and acceleration are velocity of approach
constants only in magnitude. Their directions change. The total momentum of a system of particles is a constant in
In a uniform circular motion, the kinetic energy of the body the absence of external forces.
r r r The centre of mass of a system of particles is defined as the
is a constant. W = 0, a ¹ 0, P ¹ constant, L=constant
point whose position vector is R =
å mi ri
v2
2 M
Centripetal acceleration, a r = w r = = wv (always
r The angular momentum of a system of n
applicable) n
r r r particles about the origin is L = å ri ´ p i ;
ar = w ´ v SYSTEMS OF
PARTICLES AND i =1
Newton’s first law of motion or law of ROTATIONAL L = mvr = Iw
inertia : It is resistance to change. MOTION The torque or moment of force on a
Newton’s second law : system of n particles about the origin is
LAWS OF r r r r
=F ma, = F dp / dt
MOTION t= å ri ´ Fi
2 i
r r The moment of inertia of a rigid body about an axis is defined
Impulse : Dp = FDt, p2 - p1 = ò F dt
1 by the formula I = å mi ri 2
r r
Newton’s third law : F12 = - F21
1
The kinetic energy of rotation is K = Iw 2
Frictional force fs £ (fs ) max = ms R ; fk = m k R 2
Circular motion with variable speed. For complete circles, The theorem of parallel axes : I'z = Iz + Ma2
the string must be taut in the highest position, u ³ 5ga .2
Theorem of perpendicular axes : Iz = Ix + Iy
Circular motion ceases at the instant when the string For rolling motion without slipping vcm = Rw. The kinetic
becomes slack, i.e., when T = 0, range of values of u for energy of such a rolling body is the sum of kinetic energies
which the string does go slack is 2ga < u < 5ga . 1 2 1
of translation and rotation : K = mvcm + Iw 2
2 2
Conical pendulum : w = g / h where h is height of a point A rigid body is in mechanical equilibrium if
of suspension from the centre of circular motion. (a) It is translational equilibrium i.e., the total external force
The acceleration of a lift on it is zero : S Fi = 0.
(b) It is rotational equilibrium i.e., the total external torque
actual weight - apparent weight on it is zero : S ti = Sri × Fi = 0.
a=
mass If a body is released from rest on rough inclined plane, then
If ‘a’ is positive lift is moving down, and if it is negative the
lift is moving up. n
for pure rolling m r ³ tan q (Ic = nmr2)
On a banked road, the maximum permissible speed Vmax n +1
æ ö1/2 æ n ö
çç R u s ∗ tan π ÷÷ Rolling with sliding 0 < ms < çè ÷ tan q ;
= çç g ÷ n + 1ø
è 1, u tan π ø÷
s
Work done W = FS cosq g sin q
< a < g sin q
Relation between kinetic energy E and n +1
WORK, momentum, P = 2mE Newton’s universal law of gravitation
ENERGY AND K.E. = 1/2 mV2; P.E. = mgh Gm1m 2
POWER Gravitational force F =
If a body moves with constant power, r2
its velocity (v) is related to distance GRAVITATION
travelled (x) by the formula v µ x3/2. Nm 2
G = 6.67 × 10 –11
W kg 2
Power P = = F.V
t
PHYSICS (iii)
The acceleration due to gravity. length (or surface energy per unit area) acting in the plane
(a) at a height h above the Earth’s surface of interface.
Stokes’ law states that the viscous drag force F on a sphere
GM E æ 2h ö
g(h) = = g ç1 - ÷ for h << RE of radius a moving with velocity v through a fluid of viscosity h
2
(R E + h) è RE ø F = – 6phav.
æ 2h ö 2 r 2 (r – σ)g
g(h) = g(0) ç 1 - GM E Terminal velocity VT =
where g(0) = h
è R E ÷ø
9
R 2E The surface tension of a liquid is zero at boiling point. The
(b) at depth d below the Earth’s surface is surface tension is zero at critical temperature.
GM E æ d ö æ d ö If a drop of water of radius R is broken into n identical
g(d) = ç1 - R ÷ø = g (0) çè1 - R ÷ø drops, the work done in the process is 4pR2S(n1/3 – 1) and
2 è
RE E E
3T 1 1
(c) with latitude l g1 = g – Rw2 cos2l fall in temperature Dq = ,
J r R
GM Two capillary tubes each of radius r are joined in parallel.
Gravitational potential Vg = –
r The rate of flow is Q. If they are replaced by single capillary
GM tube of radius R for the same rate of flow, then
Intensity of gravitational field I = 2 R = 21/4 r.
r 2s cos ε
The gravitational potential energy Ascent of a liquid column in a capillary tube h =
rθg
Gm1m 2
V=- + constant F
r Coefficient of viscosity, n = -
The escape speed from the surface of the Earth is æ dv ö
Aç ÷
2GM E è dx ø
ve = = 2gR E and has a value of 11.2 km s–1.
RE Velocity of efflux V = 2 gh
GM E Relation between different temperature
Orbital velocity, vorbi = = gR E scales :
RE
C F , 32 K , 273
A geostationary (geosynchronous communication) satellite THERMAL = <
moves in a circular orbit in the equatorial plane at a 100 100 100
PROPERTIES The coefficient of linear expansion (al),
approximate distance of 4.22 × 104 km from the Earth’s centre. OF MATTER superficial (b) and volume expansion (a )
Kepler’s 3rd law of planetary motion. v
are defined by the relations :
T12 a13
T2 µ a3 ; = Dl DA DV
T22 a 32 = a l DT ; = bDT ; = a V DT
l A V
Hooke’s law : stress µ strain av = 3al ; b = 2a l
Young’s modulus of elasticity In conduction, heat is transferred between neighbouring
F Dl parts of a body through molecular collisions, without any
MECHANICAL Y=
PROPERTIES Al TC - TD
flow of matter. The rate of flow of heat H = KA ,
OF SOLIDS 1 L
Compressibility = where K is the thermal conductivity of the material of the bar.
Bulk modulus Convection involves flow of matter within a fluid due to
Y = 3k (1 – 2s) unequal temperatures of its parts.
Y = 2n (1 + s) Radiation is the transmission of heat as electromagnetic
If S is the stress and Y is Young’s modulus, the energy waves.
density of the wire E is equal to S2/2Y. Stefan’s law of radiation : E = sT4, where the constant s is
If a is the longitudinal strain and E is the energy density of known as Stefan’s constant = 5.67 × 10–8 wm–2 k–4.
a stretched wire, Y Young’s modulus of wire, then E is equal Wein’s displacement law : lmT = constant, where constant
is known as Wein’s constant = 2.898 × 10 –3 mk.
1 2
to Ya dQ
2 Newton’s law of cooling: = - k (T2 - T1 ) ; where T1 is
F dt
Thermal stress = = Y a Dq the temperature of the surrounding medium and T2 is the
A temperature of the body.
Pascal’s law : A change in pressure Heat required to change the temperature of the substance,
applied to an enclosed fluid is transmitted Q = mcDq
undiminished to every point of the fluid and c = specific heat of the substance
MECHANICAL the walls of the containing vessel.
Heat absorbed or released during state change Q = mL
PROPERTIES Pressure exerted by a liquid column P = hrg L = latent heat of the substance
OF FLUIDS Bernoulli’s principle Mayer’s formula cp – cv = R
P + rv2/2 + rgh = constant
Surface tension is a force per unit
(iv) PHYSICS
First law of thermodynamics: DQ = DU A particle of mass m oscillating under the influence of a
+ DW, where DQ is the heat supplied to the Hooke’s law restoring force given by F = – k x exhibits simple
system, DW is the work done by the system
and DU is the change in internal energy of k
harmonic motion with w = (angular frequency),
THERMODY- the system. m
NAMICS In an isothermal expansion of an ideal
gas from volume V1 to V2 at temperature T m
T = 2p (period)
the heat absorbed (Q) equals the work done k
(W) by the gas, each given by Such a system is also called a linear oscillator.
æV ö æ l cosq ö
Q = W = nRT ln ç 2 ÷ Time period for conical pendulum T = 2p ç where
è V1 ø è g ÷ø
In an adiabatic process of an ideal gas PVg = TVg–1 q angle between string & vertical.
Tg Cp 1
Energy of the particle E = k + u = mω2 A 2
= = constant, where g = C .
P g -1 v 2
Work done by an ideal gas in an adiabatic change of state The displacement in a sinusoidal wave y (x, t) = a sin (kx – wt +
f) where f is the phase constant or phase
nR (T1 - T2 ) angle.
from (P1, V1, T1) to (P2, V2, T2) is W = Equation of plane progressive wave :
g -1
T2 æt xö
WAVES = a sin 2p ç - ÷
The efficiency of a Carnot engine is given by h = 1 - T èT Vø
1
Equation of stationary wave : Y = 2a sin
Second law of thermodynamics: No engine operating
between two temperatures can have efficiency greater than 2p t 2 px
cos
that of the Carnot engine. T l
dQ The speed of a transverse wave on a stretched string
Entropy or disorder S =
T v = T/m.
Ideal gas equation PV = nRT Sound waves are longitudinal mechanical waves that can
Kinetic theory of an ideal gas gives travel through solids, liquids, or gases. The speed v of sound
1 wave in a fluid having bulk modulus B and density µ is
KINETIC the relation P = nmv 2 , Combined with
3 v = B/r.
THEORY the ideal gas equation it yields a kinetic The speed of longitudinal waves in a metallic bar is
interpretation of temperature.
v= Y/r
1 3 3k BT
nmv 2 = k B T , vrms = (v 2 )1/2 =
2 2 m For gases, since B = g P, the speed of sound is v = gP / r
The law of equipartition of energy is stated thus: the energy The interference of two identical waves moving in opposite
for each degree of freedom in thermal equilibrium is 1/2 (kBT) directions produces standing waves. For a string with fixed
3 ends, standing wave y (x, t) = [2a sin kx ] cos wt
The translational kinetic energy E = k B NT . This leads The separation between two consecutive nodes or antinodes
2 is l/2.
2 A stretched string of length L fixed at both the ends vibrates
to a relation PV = E .
3 1 v
Degree of freedom : Number of directions in which it can with frequencies f = .
move freely. 2 2L
The oscillation mode with lowest frequency is called the
Root mean square (rms) velocity of the gas fundamental mode or the first harmonic.
3RT 3P A pipe of length L with one end closed and other end open
C= = (such as air columns) vibrates with frequencies given by
M r
æ 1ö v
2RT 2KT f = çn + ÷ , n = 0, 1, 2, 3, ....
Most probable speed Vmp = = è 2 ø 2L
M m The lowest frequency given by v/4L is the fundamental
KT mode or the first harmonic.
Mean free path l =
2 pd 2 P V
Open organ pipe n1 : n2 : n3 ....... 1, 2, 3......., n =
Displacement in SHM : Y = a sin wt 2l
or, y = a cos wt Beats arise when two waves having slightly different
The particle velocity and acceleration frequencies, f1 and f2 and comparable amplitudes, are
OSCILLATIONS during SHM as functions of time are given superposed. The beat frequency fbeat = f1 – f2
by, The Doppler effect is a change in the observed frequency
v (t) = – wA sin (w + f ) (velocity), of a wave when the source S and the observer O moves
a (t) = – w2A cos (wt + f) = – w2x (t)
(acceleration) æ v ± v0 ö
relative to the medium. f = f0
Velocity amplitude vm =w A and acceleration amplitude çè v ± v ÷ø
s
am =w2A.
SAMPLE PAPER - 1
Time : 3 Hrs. Max. Marks : 70

GENERAL INSTRUCTIONS
(i) There are 26 questions in all. All questions are compulsory.
(ii) This question paper has five sections: Section A, Section B, Section C, Section D and Section E.
(iii) Section A contains five questions of one mark each, Section B contains five questions of two marks each, Section
C contains twelve questions of three marks each, Section D contains one value based question of four marks and
Section E contains three questions of five marks each.
(iv) There is no overall choice. However, an internal choice has been provided in one question of two marks, one
question of three marks and all the three questions of five marks weightage. You have to atempt only one of the
choices in such questions.

SECTION - A 9. A physical quantity Q is given by

1. A force of 5N acts on a body of weight 9.8 N. What is the


acceleration produced in m/s2? A2 .B3 2
Q=
C +4 D1 2
2. What will be the effect on the angle of contact of a liquid if
the temperature increases? The percentage error in A,B, C, D are 1%, 2%, 4%, 2%
respectively. Find the percentage error in Q.
3. “Friction is a self-adjusting force.” Is this statement correct.
10. A car A travelling with a speed of 60km/hr on a straight road is
4. The masses M in the two figures are identical. Which of ahead of another car B travelling with a speed of 40km/hr.
the two systems has the smaller gravitational potential What would be relative velocity of A with respect to B? Would
energy ? Why ? it be changed if B is ahead of A?

M
SECTION - C

r 11. Derive an expression for the excess pressure inside a soap


bubble.
M r O r M
M r M M 2r
(a) (b)
dR
P
5. Does the internal energy of an ideal gas change in an
isothermal process? In an adiabatic process? R

SECTION - B
6. How do we calculate work done by a force? Write any two
12. (a) A rail track made of steel having length 10 m is
conditions under which work done by a force is zero.
clamped on a railway line at its two ends (Fig.). On
7. Steel is more elastic than rubber. Explain. a summer day due to rise in temperature by 20° C,
it is deformed as shown in figure. Find x (displacement
8. Why temperature less than absolute zero is not possible ?
of the centre) if asteel = 1.2 × 10–5 / °C.

L
OR
x
Draw P – V curves showing deviations from ideal 1/2 (L + D L)
behaviour for a given mass of a gas for two different
temperatures. (b) Why is it hotter at the same distance over the top
of a fire than in front of it?
SP-2 PHYSICS
13. (a) Write basic conditions for formation of stationary 18. (a) A mass m is splaced at P a distance h along the normal
waves. through the centre O of a thin circular ring of mass M
(b) All harmonics are overtones but all overtones are not and radius r (fig.)
harmonics. How ?
14. (a) A body moves on three quarters of circle of radius r.
Find the values of the displacement and the distance r
travelled by the body.
P
C o h m

A M
O

If the mass is removed further away such that OP


B becomes 2h, by what factor the force of gravitation
(b) A particle moves along a straight line such that its will decrease, if h = r ?
displacement ‘s’ at any time ‘t’ is given by s
(b) Calculate the escape velocity of the moon. [Given :
= (t3 – 6t2 + 3t + 4) m. Find the velocity when
Radius of the moon = 1.7 × 106 m and acceleration due
acceleration is zero.
to gravity at the surface of the moon = 1.63 m/s2]
15. A body oscillates with S.H.M., according to the equation x
19. Find the ratio of the frequencies in the following
é pù arrangement of springs:
= (5.0 m) cos ê (2 p rad / s )t + ú
ë 4û

At t = 1.5s, calculate the (a) displacement,


(b) speed and (c) acceleration of the body.
k1
16. Show that the total linear momentum of an isolated system
of interacting particles is conserved. k1 k2
OR (i) (ii) k2
A rocket of initial mass 6000 kg ejects mass at a constant
rate of 16 kg/s with constant relative speed of 11 km/s. What m
m
is the acceleration of the rocket one minute after blast?
17. (a) Three particles, each of mass m, are situated at the
20. Derive Newton’s law of cooling from Stefan-Boltzman law.
vertices of an equilateral triangle PQR of side a. Find
the M.I. of the system about the line PX perpendicular 21. In a Carnot engine, the temperature of the source and sink
to PR in the plane of PQR as shown in the following are 500 K and 375 K respectively. If the engine consumes
figure. 600 × 103 cals/cycle, find (i) the efficiency of the engine
(ii) heat rejected/cycle (iii) workdone/cycle.
X
Q 22. (a) Two rods of different materials, having coefficient of
linear expansions a1 and a2 and Young's modulii Y1
and Y2 are fixed between two rigid supports. The rods
are heated to the same temperature. There is no
bending of the rods. If a1 : a2 = 3 : 5, find Y1 : Y2,
when equal thermal stress is developed in the two
P a R rods.

(b) Using expression for power P = tw and rotational (b) A spherical ball contracts in volume by 0.01% when
subjected to a normal uniform pressure of 100
1 2
K.E. = Iw , derive the relation M = Ia, where the atmosphere. What is the bulk modulus of the material
2
of the ball?
symbols have their usual meaning.
Sample Paper-1 SP-3

SECTION - D 25. What is meant by beats? Derive a formula for beat


frequency of two waves.
23. Kamini live with her 10 years old daughter Mohini in a Arya
Samaj Society. These is a well maintained society park. OR
Mohini was swinging on a swing in her society park. She
(a) Establish a relation between particle velocity and
wanted to increase the speed of the swing. For that she
wave velocity.
started pushing the ground backward harder with her feet.
When her swing was at its lowest position during its (b) Two tuning forks A and B produce 4 beats/s. On
oscillation. She enjoyed it but did not show the actual loading B with wax, 6 beat/s are heard. If quantity of
mechanism. Her mother told her the actual mechanism of wax is reduced, the number of beats per second again
gaining speed in a swing. becomes 4. Find the frequency of B if the frequency of
A is 256 Hz.
(a) What values were displayed by Kamini?
26. (a) Derive an expression for the moment of inertia of a
(b) Specify which energy is maximum and which is uniform thin circular ring about an axis passing
minimum when Mohini's swing was at its lowest through its centre and perpendicular to its plane.
position?
(b) If the earth were to shrink to half its radius what would
(c) Define energy. What is its SI unit? be the duration of the day?

SECTION - E OR

24. (a) State the postulates of the kinetic theory of gases. Derive an expression for the acceleration and the
coefficient of friction of a solid cylinder rolling without
(b) Explain the meaning of mean speed, rms speed and
slipping down an inclined plane.
most probable speed of a gas.
OR

Calculate the specific heats of mono, di and triatomic gases


from the law of equipartition of energy.
SP-4 PHYSICS

SAMPLE PAPER - 2
Time : 3 Hrs. Max. Marks : 100
GENERAL INSTRUCTIONS
(i) There are 26 questions in all. All questions are compulsory.
(ii) This question paper has five sections: Section A, Section B, Section C, Section D and Section E.
(iii) Section A contains five questions of one mark each, Section B contains five questions of two marks each, Section
C contains twelve questions of three marks each, Section D contains one value based question of four marks and
Section E contains three questions of five marks each.
(iv) There is no overall choice. However, an internal choice has been provided in one question of two marks, one
question of three marks and all the three questions of five marks weightage. You have to atempt only one of the
choices in such questions.

SECTION - A 8. State the law of conservation of momentum. Establish the


same for a ‘n’ body system.
1. Under what condition will the displacement and distance
of a moving object have the same magnitude? 9. Two simple harmonic motions are represented by:

2. An engine working under isothermal conditions can æ pö


x1 = 10 sin ç 4 pt + ÷ and x2 = 5(sin 4pt + 3 cos 4pt ) .
produce no useful work. why? è 4ø
3. Three stars appear as green, red and blue in colour. Which What is the ratio of the amplitudes?
has maximum temperature? 10. Two inclined frictionless tracks are gradual and the other
4. State the condition for rotational equilibrium of a body. steep meet at A from where two stones are allowed to slide
down from rest on each track. Will the stones reach the
5. Two identical springs of springs constant k are attached bottom at the same time and at same speed. Why?
to a block of mass m and to fixed supports as shown in
Fig. When the mass is displaced from equilibrium position A
by a distance x towards right,find the restoring force.

k m k
B C

SECTION - B SECTION - C
11. Derive the maximum angle by which a cyclist can bend while
6. Justify the statement that a uniform circular motion is an
negotiating a curved path.
accelerated motion.
12. (a) Calculate the temperature at which r.m.s velocity of a
7. Establish a relation between surface tension and surface
gas molecule is the same as that of a molecule of another
energy.
gas at 27°C. Molecular weight of first and second gases
Q A x A'
P are 64 and 32 respectively.
(b) Calculate the value of the gas constant for one mole of
S F an ideal gas. Given that one mole of a gas at S.T.P. has
a volume of 22.4 L.

R S 13. (a) Discuss the variation of ‘g’


B B'
(i) due to height (ii) due to depth.
OR
(b) The value of 'g' at the moon is 1/6th of the value of
Two equal drops of water falling through air with a steady
'g' at the surface of the earth and the diameter of the
velocity V. If the two drops coalesce, what will be the new
moon is 1/4th of the diameter of the earth. Compare
steady velocity?
the ratio of the escape velocities.
Sample Paper-2 SP-5

14. The velocity-time graph of a particle moving along a straight 17. (a) Two wires of equal cross-section but one of steel and
line is given by graph shown the other of copper are joined end by end. When the
combination is kept under tension the elongation in
the two wires are found to be equal. Given, Young’s
20 – 2t moduli of steel and copper are 2 × 1011 N/m2 and 1.1 ×
v 1011 N/m2. Find the ratio between the lengths of steel
and copper wires.
t (b) Two identical solid balls, one of ivory and the other
t 20 – t 20
of wet-clay, are dropped from the same height on the
The rate of acceleration and deceleration is same and it is floor. Which one will rise to a greater height after
equal to 4 m/s2. If the average velocity during the motion is striking the floor and why?
15 m/s and total time of motion is 20 seconds then find
18. Derive the relation between two principle specific heats
(i) the value of t CP and Cv of a gas

(ii) the maximum velocity of the particle during the journey. 19. A flywheel of mass 20 kg and radius 20 cm is rotating at an
angular speed of 50 rpm when its motor is turned off.
(iii) the distance travelled with uniform velocity. Neglecting friction at the axle, calculate the force that must
OR be applied tangentially to the wheel to bring it to rest in 10
revolutions.
Figure shows the acceleration-time graph for a particle
moving along a straight line. Find the following : (Assume 20. (a) What is a projectile? Find its horizontal range and
the particle starts from rest.) show it is maximum for an angle of 45°.

(b) In a projectile motion the range R is ‘n’ times that of its


maximum height H, find its angle of projection.
6
21. (a) A thin uniform circular disc of mass M and radius R
5 is rotating in a horizontal plane about an axis passing
through its centre and perpendicular to its plane with
4
acc. an angular velocity w. Another disc of same dimension
(m/s² ) 3 but of mass M/4 is placed gently on the first coaxially.
2 Show that the angular velocity of the system becomes
4
1 w.
5
1 2 3 4 5 6 (b) The moments of inertia of two rotating bodies X and
time(sec) Y are IX and IY (IX > IY) and their angular momenta
are equal. Which one has greater K.E.?
(i) Velocity at the end of (a) 3 sec. (b) 6sec.
22. State and prove equation of continuity.
(ii) Does the direction of motion change during the motion?
SECTION - D
15. Discuss the formation of stationary waves in open organ
pipe analytically and show that all overtones are present 23. Reena and Veena were classmates. Reena was a bright girl
in the sound. and had a quick grasping power. One day Reena noticed
that her classmate Veena felt burdened while carrying her
16. (a) What is the percentage error in measuring the density school bag in her hand or on her shoulders although it was
of a cube if the error in measuring its mass is ±1% and as heavy as it was her own. Reena then told to Veena to use
in measuring its length is ±2%? school bag having a broad and thick strap instead of strap
made of thin and strong string.
æ a ö
(b) If ç P + ÷ (V – b ) = RT , where the symbols have (a) What values were displayed by Reena?
è V2 ø
(b) Why Reena suggested Veena to use school bag
æaö having a broad and thick strap?
their usual meanings, then find the dimension of ç ÷.
èbø
(c) Define pressure and give its SI unit.
SP-6 PHYSICS
SECTION - E OR

24. (a) Draw displacement-time, velocity-time and (a) Derive a relation between the three coefficients of
acceleration-time graphs for a particle executing simple thermal expansion - linear, (a) area (b), volume (g).
harmonic motion. Discuss their phase relationship.
(b) A metal plate 4 mm thick has a temperature difference
(b) A particle is executing S.H.M in a straight line. When of 32°C between its faces. It transmits 200 k cal/h
the distance of the particle from equilibrium position through an area of 5 cm2. Calculate thermal conductivity
has values x1 and x2, the corresponding velocities are of the material of the plate.
u1 and u2. Show that the time period of vibration is
26. Explain how potential energy can be stored in a spring.
Obtain an expression for it and discuss graphically its
1/ 2
éx 2 -x 2 ù nature of variation.
T = 2p ê 2 1
ú
êë u1 - u2 2 úû
2
OR
(a) State and prove work-energy theorem.
OR
(b) What is an elastic collision? What will happen when
Derive expressions for displacement, velocity, acceleration
and time period in SHM. Also find the expressions for K.E, (i) a heavy body collides on a light mass at rest.
P.E and total energy.
(ii) a light body collides on a heavy mass at rest.
25. Discuss briefly energy distribution of black body radiation.
Hence deduce Wien’s displacement law and Stefan’s law.
SAMPLE PAPER - 3
Time : 3 Hrs. Max. Marks : 100

GENERAL INSTRUCTIONS
(i) There are 26 questions in all. All questions are compulsory.
(ii) This question paper has five sections: Section A, Section B, Section C, Section D and Section E.
(iii) Section A contains five questions of one mark each, Section B contains five questions of two marks each, Section
C contains twelve questions of three marks each, Section D contains one value based question of four marks and
Section E contains three questions of five marks each.
(iv) There is no overall choice. However, an internal choice has been provided in one question of two marks, one
question of three marks and all the three questions of five marks weightage. You have to atempt only one of the
choices in such questions.

SECTION - A Derive an expression for the time period of a SHM in terms


of inertia factor and spring factor.
1. Give an example which shows that a negative acceleration
can be associated with a speeding up object. 9. The equation of a wave travelling in x direction on a string
is y = (3.0 cm) sin [(3.14 cm–1) x – (314 s–1)t]
2. Obtain the demensional formula for R used in the ideal gas
equation PV = RT. (i) Find the maximum velocity of a particle of the string.
(ii) Find the acceleration of a particle at x = 6 cm and at
3. Define power, write its S.I. unit. time t = 11 s.
4. The variation of angular position q, of a point on a rotating 10. Calculate work done from the graph.
rigid body, with time t is shown in fig. Is the body rotating
clock-wise or anti-clockwise? A
10 –

5–
q
t1 t2 t3
F orce F

B E
O t 0 |
20 30
X
10
–5 –
C D
Displacement
–10 –

5. Two liquids of densities d1 and d 2 and coefficient of


viscosities h1 and h2 are found to flow through a capillary SECTION - C
11. Three point masses, each of mass m, are placed at the
tube at the same rate. Find the ratio of h1 and h2 ?
vertices of an equilateral triangle of side a. What is the
SECTION - B gravitational field and potential due to this system at the
centroid of the triangle?
6. Two rods of the same area of cross-section, but of lengths
l1 and l2 and conductivities K1 and K2 are joined in series. 12. Differentiate between Stationary waves and Progressive
Show that the combination is equivalent of a material of waves.

l1 + l2 13. A block of mass 1 kg placed on an inclined plane as shown


conductivity K =
æ l1 ö æ l2 ö in figure.
çè K ÷ø + çè K ÷ø
1 2
m a
7. Deduce Kepler's 3rd law from Newton's law of gravitation.
8. What is a simple pendulum? Find an expression for the time
period and frequency of a simple pendulum.
OR
SP-8 PHYSICS
(i) What must be the frictional force between block and between block and the incline is 0.1. If the block is pushed
inclined plane if the block is not to slide along the up by 10 m along the incline.
inclined plane when the inclined plane is accelerating
to the right at 3 m/s2 ? F
(ii) What is the least value of m can have for this to happen?
m
(cos 37° = 0.8 ; sin 37° = 0.6)
14. (a) A wooden block of mass 10g is dropped from the top of a
cliff 100m high. Simultaneously, a bullet of mass 10g is 30º
fired from the foot of the cliff upwards with a velocity
100ms–1. After what time, the bullet and the block meet ? Calculate

(b) A body covers 12m in 2nd second and 20 m is 4th (i) work done against gravity
second. Find what distance the body will cover in 4 (ii) work done against force of friction
second after the 5th second.
(iii) increase in potential energy
15. Explain surface tension on the basis of molecular theory.
(iv) increase in kinetic energy
16. A steel wire of 4 m is stretched through 2 mm. The cross- (v) work done by applied force.
sectional area of the wire is 2 mm2. If Young’s modulus of
steel is 2 × 1011 N/m2. SECTION - D

Find (i) the energy density of the wire, (ii) the elastic potential 23. Arav went to market with his father. They decided to visit
energy stored in the wire. Big Bazar to purchase certain goods. There he has noticed
an old lady struggling with her shopping. Immediately he
OR showed her the lift and explained to her how it carries the
A rubber cord catapult has a cross-sectional area 1 mm2 load from one floor to the next. Even then the old lady was
and total unstretched length 10cm. It is stretched to 12 cm not convinced. Then suraj took her in the lift and showed
and then released to project a stone of mass 5 gm. Taking her how to operate it.
Young’s modulus for rubber 5 × 108 N/m2, find the velocity That old lady was very happy and thanked Arav.
of projection of the stone.
(a) According to you, what values were possessed by
17. (a) A body oscillates with S.H.M. according to the Arav?
equation, x = 6 cos (3pt + p / 3) m. What is (i) amplitude
(b) An elevator can carry a maximum load of 1800 kg is
and (ii) the velocity at t = 2s ? moving up with a constant speed of 2 m/s, The
(b) A particle starts oscillating simple harmonically from frictional force opposing the motion is 4000 N. Find
its equilibrium position. What is the ratio of K.E. and the total downword force on the elevator.
P.E. of the particle at time T/12s? SECTION - E
18. (a) A particle is projected with a velocity u so that its 24. Derive an expression for the position vector of centre of
horizontal range is twice the greatest height attained. mass of a two particle system.
Find the horizontal range of it.
Y
m1 A
(b) Calculate the angular velocity of the minute’s hand of f1
a clock. r1
C.M.
r B
19. What is an ideal gas? Why do the real gases show m2
deviations from ideal behaviour ? Show these deviations r2 f2
graphically. X
O
20. Derive an expression for the thermal conductivity of a solid
slab. Z

21. Write theorem of perpendicular and parallel axes. OR

22. A block of mass 1 kg is pushed up a surface inclined to Derive an expression for the kinetic energy of rotation of
horizontal at an angle of 30º by a force of 10 N parallel a body. Hence define its moment of inertia?
to the inclined surface (Fig. ). The coefficient of friction
Sample Paper - 3 SP-9

25. What are stationary waves? Write some characteristics of 26 Briefly describe a Carnot engine and derive an expression
stationary wave. How are they formed in strings? Draw the for efficiency of this engine.
various modes of vibration in them.
OR
OR
(a) Derive an expression for work done in the adiabatic
What is Doppler effect in sound. Derive expressions for expansion of a gas.
apparent frequency when (b) At 27°C, two moles of an ideal monoatomic gas occupy
(i) source is moving towards stationary observer. a volume V. The gas expands adiabatically to a volume
2V. Calculate (i) final temperature of the gas (ii) change
(ii) observer is moving towards stationary source. in its internal energy and work done by the gas during
(iii) both source and observer are in motion. the process. Take g = 5/3, R = 8.31 J/mol K
SP-10 PHYSICS

SOLUTIONS
SOLUTIONS
SAMPLE PAPER 1 8. According to the kinetic interpretation of temperature,
Absolute temperature µ average K.E. of molecules.
SECTION - A As the heat is removed, the temperature falls and velocity
1. As weight = 9.8 N \ Mass = 1 kg of molecules decreases. At absolute zero, the molecular
motion eases i.e., the kinetic energy becomes zero. As
Force (F) 5 2 kinetic energy cannot be negative, so no further decrease
Acceleration , a = Mass (M) = 1 = 5 m/s in kinetic energy is possible. Hence temperature cannot be
decreased below 0K.
2. Surface tension of liquid decreases with increase in
temperature. So, the curvature of the meniscus decreases OR
and the angle of contact increases. T1 > T2
3. This statement is correct only so long as the friction is
static friction. Upto the limiting friction, the force of friction Ideal Gas
is equal (in magnitude) and opposite to the applied force. P T1
4. The first system has the outer most masses closer to each T1 Real Gas
other than the second system. Hence the gravitational P.E. T2
of the second system is lesser than that of the first system. T2
5. In isothermal process, T = constant, dT = 0
\ dU = 0.
In an adiabatic process, Q = constant, dQ = 0 V
\ dU = – dW ¹ 0
% error in Q = 2 æç ´ 100 ö÷ + æç ´ 100 ö÷
dA 3 dB
9.
è A ø 2è B ø
SECTION - B
6. Measurement of work done when force and displacement
æ dC ö 1 æ dD ö
are inclined to each other at an angle q. + 4ç ´ 100 ÷ + ç ´100 ÷
è C ø 2è D ø
F sinq 3 1
F = 2 ´1 + ´ 2 + 4 ´ 4 + ´ 2 = 2 + 3 + 16 + 1 = 22%
F 2 2
10. Relative velocity of A with respect to B is 20km/hr and it
q will not change if B is ahead of A.
F cosq
s SECTION - C
Fig. Work done, when force is inclined to displacement. 11. Let us consider a soap bubble of radius R.
Work done = Component of force in the direction of S ↑ surface tension of soap solution, P ↑ Excess pressure
inside the bubble.
displacement ´ magnitude of displacement
W = F cos q ´ s dR ↑ increase in radius due to excess pressure soap
bubble has two free surfaces where air is in contact with the
or W = Fs cos q soap solution. Work done by the excess pressure =
r r
or W = F . S
P ≥ 4οR 2 χR
Thus work done is th e dot product of force and Increase in area of the bubble < 2[4ο(R ∗ χR)2 , 4οR 2 ]
displacement vectors. Hence work is a scalar quantity.
Work done by a force is zero if < 2≥ 4ο[R 2 ∗ 2RχR ∗ (χR)2 , R 2 ] < 16οR(χR)
p [ Neglecting ( dR ) 2as it is very small ]
p
(i) q = Þ i.e. W = Fs cos = 0 as in the Increase in P.E. = Increase in surface area × surface tension
2 2
gravitational force of earth does no work in the moon =16οR(χR)≥S
in its orbit. [ Increase in P.E. = Work done
(ii) s = 0 i.e. applied force do not displace the object as
in the case of man pushing a brick wall. 4S
7. Under a given load, extension of steel is much less than 16οR(χR)≥S < p ≥ 4οR 2 χR Þ P <
R
rubber. Also steel rod will go back to its initial configuration
easily and faster than rubber, as intermolecular force of steel 2 2
æ L DL ö æ Lö
is more stronger than that of rubber. Therefore steel is 12. (a) From Question, x = çè + ÷ - çè ÷ø
2 2 ø 2
more elastic than rubber.
SOLUTIONS SP-11

1 æ pö
» 2 L DL = – 5.0 × 2 p ´ 2 p ´ cos ç 2 pt +
2 è 4 ÷ø
L
DL = aL Dt \x » 2a D t æ pö
2 = -5.0 ´ 4p2 ´ cos ç 2p´ 1.5 + ÷
è 4ø
» 0.11 m ® 11 cm
(b) In front of the fire, heat is received only by the 2
p æ 22 ö 1
process of radiation, but at a point above the fire, heat = +5.0 ´ 4p2 cos = 5´ 4´ç ÷ ´
is received both by convection and radiation.
4 7
è ø 2
13. (a) The basic conditions for formation of stationary = 139.56 m / s 2 .
waves are listed below :
(i) The direct and reflected waves must be travelling r
16. Consider two bodies A and B, with initial momenta p A and
along the same line.
r
(ii) For stationary wave formation, the superposing p B respectively. Let the two bodies collide, get apart and
waves should either be longitudinal or r r
transverse. A longitudinal and a transverse wave have final momenta p¢A and p ¢B respectively. By the second
cannot superpose.
(iii) For formation of stationary waves, there should law of motion :
not be any relative motion between and Change in momentum of body A,
oppositely travelling waves. r r r
pA¢ - pA = FAB Dt ...(i)
(iv) Amplitude and period of the superposing waves
should be same. r
where FAB is the force acting on A due to action of B for a
(b) All overtones have frequency. Only the frequencies
which are integral multiple of the fundamental, are time Dt.
called harmonics rest are not called as harmonics. r r
Similarly, change in momentum of body B, pB¢ - pB =
Hence all overtones are not harmonics but all
r
harmonics are overtones. FAB Dt ...(ii)
14. (a) Here, Displacement AB = r 2 + r 2 = 2r Here time Dt, the time for which two bodies A and B are in
contact and interact, is same for both the forces.
3 3pr Moreover, from third law of motion
Distance = AB = ´ 2pr =
4 2 r r
FAB = – FBA
(b) Here, s = (t 3 - 6t 2 + 3t + 4) ; Hence, adding (i) and (ii), we obtain
r r r r r r
ds dv ( p¢ A - p A ) + ( p¢ B - p B ) = FAB Dt + FBA Dt
v= = 3t 2 - 12t + 3 ; a = = 6t - 12 r r
dt dt = – FBA Dt + FBA Dt = 0
When a = 0 Þ 6t – 12 = 0 Þ t = 2s r r
r r
\ v = 3 × 22 – 12 × 2 + 3 Þ pA¢ + pB¢ = pA + pB
= 12 – 24 + 3 = – 9 m/s Which shows that the total final momentum of the isolated
15. Given; w = 2p rad/s, T = 1s, t = 1.5s system is exactly same as its initial momentum. Thus, it is
proved that total momentum of an isolated system remains
p
(a) Displacement, x = (5.0) cos êé 2 p ´ 1.5 + úù conserved.
ë 4û
OR
æ pö p dm
Þ x = 5.0 cos ç 3p + ÷ = – 5.0 cos Here, m0 = 6000 kg, = 16 kg/s , u = 11 km/s
è 4 ø 4 dt
= 11000 m/s, t = 1 min = 60 s, a = ?
Þ x = – 5.0 × 0.707 = – 3.535 m

æ dm ö
dx æ Mass left after 1 minute = m0 – ç t
(b) Velocity, v = = – 5.0 × 2p sin ç 2pt + 4 ÷ è dt ÷ø
dt è ø = 6000 – 16 × 60 = 5040 kg.
æ pö 22 p du dm
Þ v = –5.0 × 2p sin ç 2 p ´ 1.5 + ÷ = 5 × 2 × sin F= m =u
è 4ø 7 4 dt dt
22 du u (dm / dt )
=5× 2× × 0.707 = 22.22 m/s Þ a= =
7 dt m
1100 ´ 16
d 2x Þ a= = 34.92 m/s2
(c) Acceleration, a = 5040
dt
SP-12 PHYSICS
17. (a) Moment of inertia in the plane PQR T2 375 3 1
IPQR = m1r12 + m2r22 + m3r32 = m × 0 + m (i) h =1- =1- 1 , < < 25%
T1 500 = 4 4
2
æaö ma 2 5 Q2 T2 T2
× ç ÷ + m × a2 = + ma2 = ma2.
è2ø 4 4 (ii) Q Q = T Þ Q2 = Q1 T
1 1 1
(b) Power, P = rate of doing work = rate of change of
K.E. 375
= 6 ≥105 ≥ = 4.5 × 105 cals
d æ 1 2ö 1 dw 500
\ P= ç I w ÷ø = I.2w (iii) W = Q1 – Q2 = 6 × 105 – 4.5 × 105
dt è 2 2 dt
= 1.5 × 105 cals = 1.5 × 105 × 4.2J = 6.3 × 105J
é dw ù 22. (a) Due to thermal expansion of the rod, increase in
Þ P = I.wa êë a = dt úû
length is given by Dl = al q [where l is the initial
But P = tw `(given) length and q is the increase in temperature]
Comparing we get, M = Ia
18. (a) Only the horizontal component (i.e. along the line stress stress æ Dl ö
Y= = çèQ Dl = al q, \ = a q ÷ø .
joining m and O) will survive. The horizontal component strain aq l
of the force on any point on the ring changes by a
1
factor : \ Yµ [Stress and q are constant]
a
é 2r ùé m ù 4 2 Y1 a 2 5
ê 2 2 3/ 2
ú ê 2
ú
2 3/ 2 = . \ = = \ Y1 : Y2 = 5 : 3
êë (4r + r ) úû êë (r + r ) úû 5 5 Y2 a1 3
(b) Escape velocity DV 0.01
(b) From question, – = 0.01% = ,
V 100
Ve = 2 g M RM = 2 ´ 1.63 ´ 1.7 ´ 106
P = 100 atmosphere = 100 × 1.01 × 105 N/m2
= 2.354 × 103 m/s
19. (i) Restoring force = – k1x – k2x; -P 100 ´1.01´ 105
\ B= =
DV / V 0.01 / 100
1 k1 + k 2 = 101 × 109 N/m2 = 1.01 × 1011 N/m2
keq = k1 + k2 \ f =
2p m SECTION - D
(ii) Here, extensions are different. Total extension = x 23. (a) Kamini shares her knowledge with her daughter and
= x1 + x2 wants to improve her knowledge, concern towards her
daughter.
1 1 1 1 k1k 2 (b) When Mohini's swing was at its lowest position Kinetic
= + \ f =
keq k1 k2 2p m(k1 + k 2 ) energy is maximum and potential energy is minimum.
20. For a non-black body at temperature T surrounded by an (c) Energy is defined at the capacity of doing work. Its SI
enclosure T0 (where T > T0), unit is same as that of work i.e. joule.
We have E = e s (T4 – T04) SECTION - E
where e is a constant, and its value depends upon the 24. (a) Postulates of kinetic theory : –
physical nature of the emitting surface. (i) A gas consists of a very large number of molecules
\ E = e s (T2 + T02) (T2 – T02) which are perfect elastic spheres and are identical
= e s (T2 + T02) (T + T0) (T – T0) in all respects for a given gas and are different for
different gases.
= e s (T – T0) (T + T0) (T2 + T02)
(ii) The molecules of a gas are in a state of continuous,
= e s (T – T0) (T 3 + T T02 + T0 T 2 + T03)
rapid and random motion.
If the temperature difference (T – T0) between the body
(iii) The size of the gas molecules is very small as
and the surrounding is very small, then T = T0
compared to the distance between them. So,
\ T3 = T03, T2 T0 = T03, T T02 = T03
volume of all molecules of the gas is negligible
\ E = e s (T – T0) (4T03) = K (T – T0).
than the volume of the gas.
where K = e s T03 is another constant or E µ (T - T0 ). (iv) The molecules do not exert any force of attraction
Thus the rate at which a body loses heat is directly or repulsion on each other, except during collision.
proportional to the temperature difference between the (v) The collisions of the molecules with themselves
body and its surrounding provided the temperature and with the walls of the container are perfectly
difference is not very large. This statement is called elastic.
Newton’s law of cooling. (vi) Molecular density is uniform throughout the gas.
(vii) A molecule moves along a straight line between
21. Given: T1 = 500 K, T2 = 375 K, Q1 = 600 × 103 cal.
two successive collisions.
SOLUTIONS SP-13
(viii) The collisions are almost instantaneous. d æ7 ö 7
(b) Mean speed is the average speed with which a \ Cv = çè RT ÷ø = R \
molecule of a gas moves. dT 2 2
If v 1 , v 2 , ....., v n are the individual speeds of 7 9
the molecules then, mean speed = v = C p =Cv + R = R + R = R
2 2
v1 + v2 + .... + vn 8kT C p (9 / 2) R 9
= ; \ g= = =
n mp Cv (7 / 2) R 7
r.m.s speed is the square root of the mean of the For non linear triatomic molecule, there are six degrees of
squares of the random velocities of the individual freedom.
molecules of a gas. \ Total energy associated with one gm molecule of such
a gas
C12 + C22 + ....Cn2 3kT
Crms = = 1 6
n m = 6 ´ kT ´ N = RT = 3RT
Most probable speed of the molecules of a gas is that 2 2
speed with which the maximum fraction of total no. d
of molecules moves. \ Cv = (3RT ) = 3R
dT
2kT \ Cp = Cv + R = 3R + R = 4R
Most probable speed = v max =
m C 4R 4
\ g= p = =
Cv 3R 3
OR 25. When two sound waves of slightly different frequencies,
For monoatomic gases, there is only three degree of travelling in a medium along the same direction, superpose
freedom. on each other, the intensity of the resultant sound at a
1 particular position rises and falls alternately with time. This
K.E/molecule/degree of freedom = kT (From law of is called beats formation and the phenomenon of waxing
2
equipartition of energy) and waxing of sound is called beats.
3 Let the equations of the two superposing waves are y1
\ For three degrees of freedom, K.E. = kT = a sinw1t = a sin 2pn1t
2
\ Total K.E. of 1 gm mole of the gas and y2 = a sinw2t = a sin 2pvf2t
3 3 According to the principle of superposition
= kT ´ N = RT , where R = kN The resultant displacement, y = y1 + y2
2 2
If dQ is the small amount of heat given to increase the = a sin 2pn1t + a sin 2 pn 2t
temperature by dT of 1 gm mole of a gas at constant
volume, then dQ = 1.Cv.dT = 2a cos p ( n1 - n2 ) t. sin p ( n1 + n2 ) t
dQ d æ3 ö 3 = A sin p ( n1 - n2 ) t.
\ Cv = = ç RT ÷ = R
dT dT è 2 ø 2 where A = 2a cos p( n1 - n2 )t = amplitude of the resultant
Q Cp – Cv = R Þ Cp = Cv + R wave.
3 5 A will be maximum when
= R+ R = R cos p( n1 - n2 )t = ±1 = coskp
2 2
C p (5 / 2) R 5 k
\ g= = = p(n1 - n 2 )t = kp Þ t = ; k = 0, 1, 2, .....
Cv (3/ 2)r 3 n1 - n 2
For diatomic gases, there are 5 degrees of freedom.
\ Energy associated with one gm mole of a diatomic gas 1
Time interval between successive maxima =
1 5 n1 - n 2
= E = 5 ´ kT ´ N = RT
2 2 \ Frequency of maxima = n1 - n 2
dQ d æ5 ö 5
\ Cv = = ç RT ÷ = R 'A' will be minimum when cos p( n1 - n2 )t = 0
dT dT è 2 ø 2
p
5 7 = cos(2k + 1)
\ Cp = Cv + R = R + R = R 2
2 2
C p (7 / 2) R 7 p 2k + 1
\ g= = = p(n1 - n2 )t = (2k + 1) t=
Cv (5 / 2) R 5 2 2( n1 - n2 )
For triatomic gases with linear molecule, there are seven 1
degrees of freedom. Time interval between successive minima =
\ Total energy associated with 1 gm molecule of such n1 - n 2
1 7 \ Frequency of minima = n1 - n 2
a gas = 7 ´ kT ´ N = RT
2 2 \ Beat frequency = n1 - n 2
SP-14 PHYSICS

æ Rö
OR will decrease, ç R ' = ÷ M.I. will also decrease.
(a) The equation of a plane progressive wave travelling è 2ø
with a velocity v along positive x direction is given As no external torque acting on the earth, so its
by angular momentum (L = Iw) is constant.
According to the law of conservation of angular
é 2p ù
y ( x, t ) = a sin ê (vt - x) + fo ú momentum,
ël û
I1 I 2 I2
Its initial phase, fo = 0 then I1w1 = I 2 w 2 Þ = T2 = T1
T1 T2 or I1
é 2p ù
y(x, t) = a sin ê (vt - x )ú 2
ël û 2 2 æ Rö
I1 = MR 2 , I 2 = M ç ÷
d 5 5 è 2ø
Velocity of the particle = [ y ( x, t )]
dt 1
\ T1 = ´ 24 = 6 hr
d é ì 2p üù 4
= a sin í (vt - x) ý ú
dt êë î l þû
OR
Let M ® mass of the solid cylinder R ® Radius of the
ì 2p ü 2pv cylinder, q ® Angle of inclination of the plane, a - Angular
= a cos í (vt - x )ý ´ acceleration of the cylinder.
î l þ l
Wave velocity = v R
a
\ Particle velocity = F
2p ì 2p ü q
a cos í (vt - x)ý × wave velocity sin
l î l þ mg q
(b) Possible frequncies of B are 256 ° 4 = 260 or 252. mg cos q
mg
As no. of beats is 4 when B is loaded with wax therefore q
final frequency must be less than the initial frequency.
Various forces acting on the cylinder are:
[ Initial frequency of B = 260 Hz
26. (a) Let M ® mass and R ® Radius of the ring. R = mg cos q, F = frictional force acting upwards
Length of the ring = circumference = 2 p R \ Net force in the downward direction
Mass/unit length = M/2 p R = f = ma = mg sin q – F
The torque required for the rolling motion of the cylinder
X is due to frictional force which acts tangential to the
surface.
Ia
\ t = r × F Also t = Ia = [Q a = ra]
r
O dx Ia Ia
R \ = rF \ F=
r r2
Ia
\ ma = mg sin q -
r2
Y
æ I ö mg sin q
M Þ a ç m + 2 ÷ = mg sinq Þ a =
Mass of a small length element dx = .dx è r ø I
m+ 2
2 pR r
\ M.I. of this element about the axis XY For a solid cylinder, M.I. about its axis
M MR Mr 2 mg sin q 2
= dI = .dx.R 2 = .dx I= \ a= = g sin q
2 pR 2p 2 mr 2 3
\ Moment of inertia of the entire ring = m+
2
2p r
MR Ia I 2
I=
ò dI =
2p ò dx \ Frictional force = F = 2 = 2 . g sin q
r r 3
0

MR 2 pR MR MR mr 2 2 1
[ x] = (2 pR - 0) = .2 pR = . g sin q = mg sin q
2 3 3
2p 0 2p 2p 2r
\ I = MR 2 1
mg sin q
(b) Q The earth is a solid sphere. F
\ Coefficient of friction = m = = 3
2 R mg cos q
\ Its M.I. is I = MR2. If it shrinks, then its radius
5 1
Þ m= tan q
3
SOLUTIONS SP-15

OR
SAMPLE PAPER - 2
2 r 2 (θ ,ρ)g
SECTION - A Terminal velocity VT =
1. If the object is moving along a straight line. 9 γ
2. Because, the efficiency of an engine working between [ VT µ r2
same temperature of source and sink is zero. i.e. 4 4
If R is the radius of the bigger drop, then p R3 = 2 × p r3
T 3 3
h = 1- = 1 -1 = 0
T Þ R = (2)1/3r
1 VT r2 V r2
3. From Wien's displacement law, lm µ [ = Þ =
T VT' R2 (2)2 / 3 r 2
VT'
lm for blue is minimum. So, its temperature will be
[ V'T = (2)2/3 V
maximum.
8. When no external force acts on a system the momentum will
4. For rotational equilibrium of a body the vector sum of remain conserved. Consider a system of a, n bodies of
torques of all the forces acting on the body about the masses m1, m2 , m3...., mn. If p1, p2, p3, ...., pn are the
reference point must be zero. momentum associated then, the rate of change of momentum
5. 2kx towards left. with the system,
SECTION - B dp dp dp dp dp
= 1 + 2 + 3 + ..... n
6. In uniform circular motion, the speed of the body remains dt dt dt dt dt
the same but the direction of motion changes at every point. d
= (p + p + p + .... + pn)
Fig. shows the different velocity vectors at different dt 1 2 3
positions of the particle. At each position, the velocity dp
If no external force acts, =0
r r dt
vector v is perpendicular to the radius vector r .
\ p = constant, i.e., p1 + p2 + p3 + .... + pn = constant.
v 9. Amplitude of x1 = 10, Amplitude of

x2 = 5 2 + (5 3 ) 2 = 10;
r Ratio of amplitudes = 1 : 1
O
10. Let AM = h and AB = s, a = acceleration of the stone
v
falling through AB.
v A
h
Thus the velocity of the body changes continuously due \ a = g sin q = g .
s
to the continuous change in the direction of motion of the q qmg cos q
1 2 1 in
body. As the rate of change of velocity is acceleration, so s = ut + at = at 2 gs mg
Q m
2 2 h
a uniform circular motion is an accelerated motion.
q
7. Consider a rectangular metallic frame PQRS having a sliding 1 gh 2 B M C
= t [Q u = 0]
wire AB at its one side. 2 s
Dip it in soap solution and a thin soap film AQRB is formed.
This film has 2 free surfaces of which water and air are in 2
\ t=s Þ t µs
contact with each other . The force of surface tension acts gh
tangentially inwards and perpendicular to the free surfaces
of the film. Let the area of the film has increased by moving \ The stone falling through AB will take more time than
AB onwards to A' B'. that falling through AC.
v2 – u2 = 2gh
[ Increase in area of this film = AA'B'B =a = 2 ( l × x )
Total force due to surface tension = F = S × 2 l Q u = 0, \ v = 2 gh
( [ there are two free surfaces ) Q Both the stones are falling through the same vertical
height h, therefore they will reach the bottom with the
[ work done against the force of surface tension = W
same velocity.
= Force × displacement SECTION - C
= S × 2l × x = S × a [ [ a = 2 ( l × x ) ] 11. Let the cyclist OC of mass ‘m’ bends at an angle q with the
This work done is stored as surface energy. vertical. In order to take a circular turn of radius ‘r’ with
[ Surface energy = E = S × a
speed ‘v’. The horizontal component R sin q of the normal
SP-16 PHYSICS
reaction ‘R’ provides the necessary centripetal force while 14. v = 0 + ut
the vertical component R cos q balances the weight of the 1
cycle & cyclist. R Total displacement = (20 - 2t + 20) ´ 4t = 2t ( 40 – 2t)
2
In equilibrium,
Rcosq
R cos q = mg and total displacement
R sin q = mv2/r (centripetal force)
Y C Average velocity =
Rsin q total time
R sin q /R cos q = tan q = v /rg
2

mg 2t (40 - 2t )
15 =
q
20
Solvin g quadratic equation, 150 = 40t – 2t 2
O X Þ t = 5 sec. (another solution not acceptable think why!)
Maximum velocity = 4t = 4 × 5 = 20 m/s
distance travelled with uniform velocity
= (20 – 2t) V = (20 – 2 × 5) × 20 = 200m
OR
12. (a) Here, temperature T = 27 + 273 = 300k The equation for the above curve is given by : a = 2t for
3RT 3R(27 + 273) 0£t£3
= [Q Equation of a straight line passing through the origin is
M1 M2
given by y = mx where m is the slope]
M2 300 and a = 12 – 2t, for 3 < t £ 6
Þ =
M1 t + 273 (i) (a) Thus, a = 2t Þ dv = 2t dt
M2 32 1 v v
But = =
M1 64 2 Integrating both sides we have ò dv = ò 2t dt
0 0
300 1
[ = Þ 600 = t + 273
t + 273 2 ét2 ù
3
Þ t = 600 – 273 = 327°C Þ v=2ê ú Þ v = 9 m/s
(b) Volume of 1 mole of a gas at S.T.P. = 22.4 L ëê 2 ûú 0
= 22.4 × 10–3 m3 (b) Again, from a = 12 – 2t
PV Integrating both sides we have
Q R=
T v 6

0.76≥13.6≥10 ≥9.8≥ 22.4≥10


3 ,3 ò dv = ò (12 - 2t ) dt
= 9 3
273
= 8.31 J/mole / K 6
Þ v - 9 = éë12t - t ùû
2
æ 2h ö æ dö 3
13. (a) g = g çè1 - ÷ø (ii) g¢ = g çè1 - ÷ø
R R Þ v – 9 = 12 × 3 – (36 – 9) Þ v = 18 m/s
(ii) Evidently, the direction of motion does not change.
15. In open organ pipe always two antinodes are formed at
the two open ends as the air can vibrate with maximum
g amplitude.
(a) 1st normal mode of vibration:

A A
N
O R ¥
The value of g decreases both on moving up and on
L = l1 / 2
moving down from the surface of earth. This can be
shown graphically. \ Fundamental frequency of vibration or 1st
v v
2GM harmonic = n1 = =
(b) Escape speed, ve = = 2gR = g´D l1 2 L
R
(b) 2nd mode of vibration:
[D = Diameter = 2R]
(ve )moon ( g ´ D)moon 1 1 1
= = ´ = . A A A
(ve )earth ( g ´ D)earth 6 4 4.9 N N
L = l2
SOLUTIONS SP-17

\ Frequency of 2nd harmonic or 1st overtone dQ' = 1.CP.dT


v v If the increase in volume in this case is dV then work done
= n2 = Þ n 2 = = 2n1 by the gas in the expansion
l2 L
(c) 3rd mode of vibration: = dW = PdV.
By 1st law of thermodynamics, dQ ¢ = dU ¢ + dW
[ C dT = dU ¢ + PdV
A A A A p
N N N Since, the increase in temperature (dT) is same in both cases,
[ dU = dU' = C dT
L = 3l 3 / 2 V
[ C dT = C dT + PdV
Frequency of 3rd harmonic or 2nd overtone P V
Þ (CP – CV) dT = PdV
v 3v From ideal gas equation for 1 mole, PV = RT
= n3 = =
l3 2 L [ Differentiating both side, PdV = RdT
\ n 3 = 3n1 [ (C – C ) dT = RdT Þ CP – CV = R
P V
\ The harmonics are all integral multiples of the 19. Given: M = 20 kg. r = 20 cm = 20 × 10–2 m; w 0 = 50 rpm
fundamental frequency
\ All overtones are present. 50 ´ 2p 5
= = p rad / s
M M Dd DM 3DL 60 3
16. (a) Density = d = = = +
V L3 d M L q = 10 rev. = 10 × 2 p = 20 p rad
Dd æ DM 3DL ö w 2 - w 0 2 = 2aq
´100% = ç + ÷ ´100%
d è M L ø 2
æ5 ö
= ± (1% + 3 ´ 2%) = ± 7%
02 - ç p ÷ = 2 ´ a ´ 20p
è3 ø
-5 / 3p ´ 5 / 3p 5p
a Þ a= =-
(b) Energy. i.e., dimensions of are [ML2T–2] 2 ´ 20p 72
b
1
\ t = Ia = F ´ R Þ MR 2 a = F ´ R
F l F l 2
17. (a) For steel, Ys = ´ s for copper, Ycu = ´ cu
A Dls A Dlcu æ -5p ö
20 ´ 20 ´ 10-2 ´ ç
MRa è 72 ÷ø
\ F= =
Ys l Dl l [Q Dlcu = Dls ] 2 2
\ = s ´ cu = s
Ycu lcu Dls lcu
20 ´10-1 ´ 5 ´ 3.14 31.4
=- =- = -0.4 N
ls 2 ´ 1011 20 72 72
\ = =
l cu 1.1 ´ 1011 11
20. (a) An object that is in flight after being thrown or
(b) As the ivory ball is more elastic than the wet-clay ball, projected is called a projectile
it will tend to retain its shape instantaneously after Horizontal Range :
the collision. Hence, there will be a large energy and
momentum transfer compared to the wet clay ball. u 2 sin 2q
Thus, the ivory ball will rise higher after the collision. R= [Q 2sin q cos q = sin 2q]
g
18. Consider 1 gm mole of an ideal gas in a cylinder fitted with
a perfectly frictionless piston. Let P, V, T be pressure, volume, Clearly, R will be maximum when
and temperature of a gas. When the gas is heated at constant sin 2q = 1= sin 90º or 2q = 90° or q = 45°
volume by a small temperature dT, amount of heat energy Thus the horizontal range of a projectile is maximum
given to the gas, dQ = 1.CV.dT [Q n = 1 mole] when it is projected an angle of 45° with the horizontal.
As the volume remains constant, (b) Here, R = nH so, (u2/g) sin 2q = nu2 sin2q /2g,
[ dV = 0
\ tan q = 4/n, q = tan–1 (4/n)
[ dW = PdV = 0
21. (a) Initial angular momentum of the disc = L = Iw
By 1st law of thermodynamics, dQ = dU + dW
[ CvdT = dU 1 é 1 ù
= MR 2 w êQ I disc = MR 2 ú
When the gas is heated at constant pressure by the same 2 ë 2 û
temperature dT, amount of heat given to the gas Final angular momentum of the combination should
remain same as no external torque is acting on it.
SP-18 PHYSICS
M.I. of the combination SECTION - E
1 1æMö 5 24. (a) Inter-relationship between particle displacement,
= MR 2 + ç ÷ R 2 = MR 2 velocity and acceleration in S.H.M. If a particle
2 2è 4 ø 8
executing S.H.M. passes through its positive extreme
\ 5 MR 2 ´ w ' = 1 MR 2 w position (x = + A) at time t = 0, then displacement equaiton
8 2 can be written as
4 x (t) = A cos wt
\ w' = w Velocity,
5
1 2 dx æ pö
(b) Angular momentum L = I w and K.E. = Iw v (t ) = = - w A sin wt = wA cos ç wt + ÷
2 dt è 2ø
1 I 2 w 2 L2 dv
= =
2 I 2I Acceleration, = -w 2 A cos wt
a(t)
dt
1
Q L is constant. \ K.E. µ = w2A cos (wt + p)
I Using the above relations, we determine the values
as IX > IY \ (K.E.)X < (K.E.)Y
\ K.E. of Y is greater than K.E. of X. of displacement, velocity and acceleration at various
instant t for one complete cycle as illustrated below.
22. For an incompressible, non-viscous liquid in streamline flow
the product of cross section of the tube and the velocity of Time, t 0 T/4 T/2 3T/4 T
the liquid through that cross section is constant throughout 2p
the flow. Phase angle, w t = t 0 p /2 p 3p/2 2p
T
i.e. aV = constant
Displacement, x (t) +A 0 –A 0 +A
a2 max. min. max. min. max.
Velocity, v (t) 0 – wA 0 + wA 0
v2 min. max. min. max. min.
– w A 0 + w 2A 0 – w 2A
2
v1 B Acceleration, a (t)
max. min. max. min. max.
a1
We have plotted separately the x-versus t, v versus
t and a versus t curves for a simple harmonic motion.
A
Displacement

Let a1, a2 ® area of cross section of the tube at A and B; v1, +A


v2 ® Velocity of the liquid at A and B. 3T/4
0 t
r1, r2 ® density of liquid at A & B. T/4 T/2 T
[ Mass of the liquid entering/s at A = a1v1r1 –A
mass of the liquid leaving/s at B = a2v2r2 T
Q there is no liquid remaining in the tube and the flow is (a)
steady, Þ a1v1r1 = a2v2r2 + wA
Q the liquid is incompressible,
[ r1 = r2 T/2
Velocity

0 t
[ a1 v1 = a2 v2 Þ av = constant. T/4 3T/4 T
– wA
SECTION - D (b)
23. (a) Helpful, caring nature, concern for others,
+wA
2
Acceleration

knowledgeable.
T/4 t
Force 0
T/2 3T/4 T
(c) As, Pressure =
Area –wA
2
(c)
So School bag having broader and thicker strap i.e., Relation between velocity, displacement and
greater area exerts less pressure on the shoulders. acceleration in S.H.M.
(c) Pressure is defined as the force acting per unit area of (b) using, v 2 = w2 (a 2 - y 2 )
the surface. Its SI unit is Newton per metre2 (N/m2) or
Pascal (Pa). [ u12 = w2 (a 2 - x12 ) and u22 = w2 (a 2 - x12 )

Subtracting, u12 – u22 = w2 (x22 – x12)


SOLUTIONS SP-19

4p 2 1
= (x22 – x12) = mw2 a 2 cos 2 wt
2 2
T
1/ 2
é 2 2ù 1 2 1
[ Time period of vibration, T = 2p ê x2 - x1 ú = ka cos2 wt = ka 2 (1 - sin 2 wt )
êë u12 - u2 2 úû 2 2

OR
1 2æ y2 ö 1
Let a particle be executing SHM along a circular path of = ka ç1 - ÷ = k (a 2 - y 2 )
2 ç a 2 ÷ø 2
radius a with angular frequency w and time period T with a è
velocity ‘v’. At any instant of time ‘t’ let the particle be at P
1
making angle q with its initial position at t = 0 = mw2 (a 2 - y 2 ) ... (iv)
Y 2
is the expression for K. E.
M P Restoring force = F = mass × acceleration
a
y
q =w = -mw2 y
X¢ X
O x N t=0 Work done against the restoring force for a small
displacement dx = dW = – Fdy
2
= – (-mw2 y )dy = mw ydy

[ Total workdone in displacing the particle from the mean
The position of the particle at any instant ‘t’ can be given
position to a displacement
by
y
In DOPM , sin q = OM = y 2 1 2
OP a ò
y = w = mw ydy =
2
mw y ... (v)
0
Þ y = asin π = asin w t ... (i)
is the expression for displacement. is the expression for P.E.

dy d Total energy = K.E. + P.E.


Velocity = v = = ( a sin wt ) = aw cos wt
dt dt 1 1
= mw2 (a2 - y 2 ) + mw2 y
Þ v = aw 1 - sin wt 2 2 2

y2 1
= aw 1 - = w a2 - y2 ... (ii) = mw2 a 2 = constant ... (vi)
2 2
a
is the expression for velocity
is the expression for total - energy.
Acceleration = A
25. A perfectly black body is that which absorbs completely
dv d the radiations of all wavelengths incident on it.
= = (aw cos wt ) = -w2 a sin wt
dt dt As a perfectly black body neither reflects nor transmits any
A = – w 2y radiation, therefore the absorbtance or absorbing power of
a perfectly black body is unity.
2p
Time period = T = We know that the colour of an opaque body is the colour
w
(i.e., wavelength) of radiation reflected by it. As a black
y displacement body reflects no wavelength, it appears black whatever be
= 2p = 2p ... (iii) the colour of radiation incident on it.
A acceleration
When a perfectly black body is heated to a suitable high
is the expression for time - period.
temperature, it emits radiations of all possible wavelengths.
1 2 1 The radiations given out by a perfectly black body are
K.E. = mv = m (aw cos wt )2 called plain body radiations or full radiations or total
2 2
radiations.
SP-20 PHYSICS
y \ S= L2 and V = L3
Increase in length = DL = a LDT
1664 K Increase in surface area = DS = bSDT
Increase in volume = DV = gVDT
New surface area of the cube
E1 0K = S + DS = (L + DL)2
145
0K
125
0K
110

x
l L
A perfectly black body cannot be realised in practice. Fery L
designed a perfectly black body which is most commonly
used. It consists of a hollow double walled metallic sphere L
having a narrow opening O on the side and a conical Þ S + bSDT = (L + aLDi)2
projection P inside just opposite to it. The inside of the Þ S (1+ bDT) = L2 (1 + aDT)2
sphere is coated with lamp black. Any radiation entering Þ 1 + bDT = 1 + 2aDT + a2DT2 [Q S = L2]
the sphere through the opening O suffers multiple Þ 1 + bDT = 1 + 2aDT
reflections at its inner walls and about 97% of it is absorbed [a2DT2 is neglected as it is very small]
by lamp black at each reflection. Therefore, after a few Þ bDT = 2aDT
reflections, almost entire radiation is absorbed. The Þ b = 2a .... (i)
projection helps in avoiding any direct reflection which New volume = V + DV = (L + DL)3
even otherwise is hardly possible because of the small size Þ V + Vg DT = (L + aLDT)3 = L3 (1 + aDT)3
of the opening O. When this body is placed in a bath at Þ V (1 + g DT) = L3 (1 + a DT)3
fixed temperature, the heat radiations come out of the hole. Þ 1 + g DT = 1 + 3 a DT + 3 a2DT2
The opening thus acts as a black body radiator. It should + a3DT3 [Q V = L3]
be remembered that only the opening (and not the walls) Þ g DT = 3a DT [a2 DT2 and a3 DT3 are neglected
acts as a black body radiator. as they are very small]
Þ g = 3a ..... (ii)
(i) At a given temperature of black body, the energy is
not distributed uniformly amongst all wavelength. b g
\ From (i) and (ii), a= =
(ii) The energy emitted is maximum corresponding to a 2 3
certain wavelength to l and it falls on either side (b) Given: Dx = 4 mm = 4 × 10–3 m,
of it.
DQ
(iii) With rise in temperature of black body total energy DT = 32°C, = 200 k cal/h
Dt
emitted increases rapidly for any given wavelength.
(iv) The wavelength corresponding to which energy DQ 200≥1000 ≥ 4.2
= J / s = 233.33 J/s
emitted is maximum is shifted towards shorter Dt 60≥ 60
wavelength side, i.e., l). decreases with rise in A = 5cm2 = 5 × 10– 4 m2, K = ?
temperature.
DQ æ ΧT ö÷
= KA ççç ÷
è Χx ø÷
(v) The area enclosed by each curve with x· axis
Dt
increases with rise in temperature of the black body.
,3
Stefan's law. The amount of energy radiated per second DQ / Dt 233.33≥ 4≥10
from unit area of a body is proportional to the fourth power Þ K= =
A(DT / Dx) 5≥10,4 ≥32
of its absolute temperature.
= 58.33 W/m/°C
\ Energy radiated = eAs T4 per second, where e is a 26. When a spring is compressed or extended a restoring force
characteristic of the body and is called emissivity of the
is developed in it due to its elasticity in a direction
surface (Ranges from 0 to 1 and is a surface parameter.)
opposite to the applied force. Some work has to be done
According to Wien's law, the product of the wavelength
corresponding to maximum intense radiation and the against the restoring force, which is stored as potential
absolute temperature is a constant, i.e., lmT = constant. energy of the spring.
OR Let x ® compression or extension of the spring and
(a) Consider a solid cube of side L, surface area of the F ® restoring force
face S and volume V.
SOLUTIONS SP-21

F µ - x or F = – kx [k = spring constant]. \ Total work done by the force to change the


Fs = 0 velocity of the body from u to v
x=0 v
(a) é v2 u2 ù
x
W = ò dw = m ò = ê2 - 2ú
vdv m
x=0 Fs is negative u ëê úû
x is positive 1 1
= mv 2 - mu 2 = Change in K.E. of the body.
2 2
(b) (b) Elastic collision is defined as that collision in which
x x both momentum and kinetic energy before and after
Fs Fs is positive collision are conserved, If two masses m 1 and m2
x is negative moving with velocities u1 and u2 collides, velocities
(c) after collision is given by

x x ( m1 - m2 )u1 + 2m2 u2
v1 =
( m1 + m2 )
F x =F0s
s =
–k -kxm2 (m2 - m1 )u2 + 2 m1u1
x Area = and v2 =
2 (m1 + m2 )
(d) xm
O B x m1u1
(i) If m1 >> m2 and u2 = 0, v1 = = u1 and v2 = 2u1
m1
\ The first body continues to move with the same
(a) The spring force Fs is zero when
A the displacement x velocity and the second body acquires double the
from the equilibrium position is zero. initial velocity of the first.
(b) For the stretched spring x > 0 and Fs < 0 (ii) If m1 >> m2 and u1 = 0
(c) For the compressed spring x < 0 and Fs > 0. 2m2 u2 mu
(d) The plot of Fs versus x. v 1= » 0 and v 2 = - 1 2 = -u2
m1 m1
Let the spring be further displaced by a small amount dx \ After collision, the heavy body remains at rest
against the restoring force. and the lighter body rebounds with the same
\ Work done against the restoring force velocity.
= dW = – Fx.dx = kx.dx. SAMPLE PAPER - 3
\ Total work done in producing displacement x from 0 SECTION - A
x 1. An object in simple harmonic motion speeds up while
= W = ò dW = ò kxdx moving from an extreme position to the mean position but
x =0 its acceleration is negative.
1 2 1 2 2. Since PV = RT
Þ W = kx \ P.E. = kx
2 2 PV F V [MLT -2 ][ L3 ]
So R= = or R =
T AT [ L2 ][ K ]
OR 2 - 2 -1
or R = [ML T K ]
(a) According to work-energy theorem work done by a
force in displacing a body is equal to the change in 3. Power is defined as the rate of doing work or the rate of
its kinetic energy. transfer of energy.
Let m = mass of the body, u = initial velocity, work done
F = force applied in the direction of motion \ Power =
time taken
ds = small displacement of motion
Small amount of work done by the force = dW SI unit of power is watt.
uur uur 4. Positive slope indicates anticlockwise rotation which is
= F .ds = Fds cos0° = Fds = mads traditionally taken as positive.
dv ο Pr 4 οhdgr 4
dW = m .ds éQ acceleration = a =
dv ù
5. By Poiseuille's formula, V < <
dt ê dt úû
ë 8 γl 8γl
ds d γ1 d
= m.dv.v [Q =v] [ is constant [ < 1
dt γ γ2 d 2
SP-22 PHYSICS
SECTION - B
6. Since they are in series, the rate of flow of heat energy 1 1 g
frequency = f = =
is the same. But the sum of the difference in temperatures T 2p l
in the difference across their free ends. OR
q Acceleration of a body executing S.H.M

q1 K1 K1 q2 = A = – w2 x

l1 l2 Restoring force = mA = –m w2 x
Also F = kx
\(q1 - q) + (q - q 2 ) = (q1 - q 2 )
Q (l1 + l2 ) [ kx = m w2 x
Q l Q l
i.e., . 1 + . 2 = .
t K1 A t K 2 A t K eq . A k
[ w2 = k/m Þw=
l1 l l +l m
Þ + 2 = 1 2
K1 K 2 K eq 2p m inertial factor
\ T= = 2p = 2p
l1 + l2 w k spring factor
\ K eq = .
æ l1 l2 ö 9. Displacement equation, y = 3 sin [3.14x – 314t]
çè K + K ÷ø dy
1 2
7. Let m = mass of the planet, M = mass of the sun, r = radius v= = 3 cos[3.14x – 314t] × 314
dt
of the orbit of the planet w = constant angular velocity Þ vmax = 3 × 314 × 1 = 9.4 m/s.
of the planet. The centripetal force on the planet required dv
for its orbital motion is provided by the gravitational force acceleration = a = = – 3 sin [3.14x – 314t] × (314)2
dt
of attraction between the planet and the sun. = – 3(314)2 sin [3.14 × 6 – 314 × 0.11]
= – 3(314)2 sin (6p – 11p) = – 3(314)2 sin (– 5p)
GMm = 3(314)2 sin (4p + p) = 3(314)2 sin p = 0
\ mw 2 r =
r2 10. Work done = area of triangle (OAB) – area of rectangle
(BCDE)
æ 4p 2 ö GMm é 2p ù
Þ m çç 2 ÷÷ = 2 1
T r êQ w = T ú W= × 20 × 10 – 10 × 5 = 50 J
è ør ë û 2
é 4p 2 ù SECTION - C
æ 4p 2 ö 3
\ T =ç
2 2 3 êQ = constant ú 11. From figure, AB = BC = CA = a
÷r \ T µr êë GM úû
è GM ø 2
æ aö 3a 2 3
8. A simple pendulum consists of a heavy point mass Median = AD = BE = CF = a2 - ç ÷ = = a
è 2ø 4 2
suspended by a weightless, inextensible, perfectly flexible
string from rigid support about which it can oscillate freely. A median is divided into 2 : 1 ratio at the centroid.

1 3 a
l \ GD = GE = GF = ´ a= ;
q 3 2 2 3
T
a a
AG = BG = CG = 2 ´ =
x 2 3 3
mg sin q mg cos q uuur uuur uuur Gm Gm 3Gm
mg | FA | = | FB | = | FC | = 2 = =
r ( a / 3) 2
a2
Restoring force = F = – mg sin π Þ sin π Resultant of FB and FC, FBC
x
=π = [for small π , sin π » π ] = FB 2 + FC 2 + 2 FB FC cos120°
l
[ F = – mg π = – mgx/l Þ F = – kx 2 2 .2
æ 3Gm ö æ 3Gm ö æ 3Gm ö 2GM
mg inertial factor
= çè 2 ÷ø + çè 2 ÷ø + 2 çè 2 ÷ø cos120° =
\ k= \ T = 2p a a a a2
l spring factor
3Gm 3Gm
\ Resultant of FBC and FA = -=0
m l a 2
a2
T = 2p = 2p
mg / l g (as they are equal and opposite)
Gravitational potential at G due to masses at A, B and C
SOLUTIONS SP-23

1
-Gm 3Gm 20 = u + a (2 ´ 4 - 1)
each = =- 2
r a
1
\ Total gravitational potential at Þ 20 = 4 + a ´ 7 ...(ii)
2
3Gm -3 3Gm Solve (i) and (ii) to get u and then
G = -3 ´ = . we get S9– S5 = 136 m
a a
15. Consider a liquid molecule A, well within the liquid surface.
12. Stationary waves Progressive waves It is equally attracted by all other molecules. So, the net
(i) The disturbance The disturbance travels force on A is zero. Consider another molecule B which is
remains confined to forward, being handed within the surface film of liquid KLMN. This molecule will
a particular region over from one particle to be attracted more on the downwards direction.
and there is no the neighbouring particle. C
onward motion. K L
(ii) There is no transfer Energy is transferred in B M
N
of energy in the the medium along the
medium. waves.
(iii) The amplitude of The amplitude of A
vibration of particles vibration of each particle
varies from zero at is same.
Now, let us consider a third molecule C on the surface of the
nodes to maximum at liquid which will be attracted in the downward direction
antinodes. with maximum force. So, to take a molecule from within the
(iv) The particles of the No particle of the medium liquid to the surface, work has to be done against the
medium at nodes are is permanently at rest. attractive force of other molecules (cohesive force). This
permanently at rest. work done is stored as the surface potential energy.
More number of molecules means more surface potential
energy. But a system is said to be stable if it has minimum
13. N = m (g cos 37° + a sin 37°) = 1 (9.8 × 0.8 + 3 × 0.6)
potential energy. Therefore, there will be a minimum number
and mg sin 37° = ma sin 37° + f of molecules on the surface. Hence, there will be a minimum
ma cos 37° N surface area at the free surface, which is the cause of surface
f tension.
ma 1
16. (i) Energy density = × stress × strain
ma sin 37° mgsin 37° 2
mg cos 37° f 2
mg37° 1 1 æ Dl ö
= × Y × (strain)2 = × Y × ç ÷
(a) f = mg sin37° – ma sin37° 2 2 è l ø
or, f = 1 (9.8 × 0.8 – 3 × 0.8) = 5.48
-3 2
11 é 2 ´ 10 ù
f 5.48 1
(b) f = µ N \ m= = = 0.57 = ´ 2 ´ 10 ´ ê ú = 2.5 × 104 J/m3.
N 9.64 2 ëê 4 ûú
14. (a) Let the block and the bullet meet at a height x from the
ground. Considering downward direction as +ve and (ii) Elastic P.E. = Energy density × vol. of the wire = 2.5 ×
the upward direction as negative, we get 104 × (2 × 10–6) × 4 = 0.2 J
for block, u = 0, s = 100 – x, a = + g
For bullet, u = – 100 ms–1, a = + g, s = – x OR
1 2 Here, elastic potential energy = K.E. of stone
Putting in s = ut + at \ 1/2 × Y × (strain)2 × volume of cord = 1/2 mv2
2
1 2 1 ìï Y ´ (strain) 2 ´ volume of cord üï
We get, – x = – 100t + gt and 100 – x = gt2 or v = í ý
2 2 ïî m ïþ
Therefore, – x = – 100t + 100 – x
Therefore, t = 1s.
Given that strain = 12 - 10 = 1 , Y = 5 ´ 108 N / m2
1 10 5
(b) Here, s = u + a (2 ´ 2 - 1)
2 Volume of the cord = (1 × 10–6) × 10 × 10–2 = 1 × 10–7 m3
1
Þ 12 = u + a ´ 3 ...(i) Mass m of the stone = 5 × 10–3 kg
2
SP-24 PHYSICS
Hence Graphical representation

ì (5 ´ 108 ) ´ (1/ 5) 2 ´ (1´10-7 ) ü 1


v= í ý = ´ (10000)
î 5 ´ 10-3 þ 5 Ideal Gas
1 PV
= ´ 100 = 20m / s T1
5 mT
T2
17. (a) Here, displacement, x = 6 cos (3pt + p / 3)
T3
dx æ pö
Velocity = dt = 6( - 3p ) sin ç 3pt + 3 ÷ T1 >T2 > T3
è ø
æ pö P
= -18p sin ç 3p t + ÷
è 3ø
(i) Amplitude, a = 6 m PV
The given figure shows a graph between and P for
mT
(ii) Velocity at t = 2s, V = -18 p sin æç 3p t + p ö÷ there temperatures T1, T2 and T3, (T1) > (T2) > (T3). From
è ø 3
the graph we find that for an ideal gas.
22 p
= – 18 × ´ sin = – 48.99 m/s PV
7 3 = 8.31 J mol–1 K–1
2p mT
(b) Displacement, y = a sin t . Here t = T/12 But here we find departures from ideal gas behaviour of
T
three gases. But we find that all of them approach the ideal
2p T p a gas behaviour for low pressures and high temperatures.
y = a sin . = a sin = 20. Let, DT = Temperature difference between two opposite
T 12 6 2
faces of the slab.
1 A = Area of cross-section of the face,
[ K.E. of the particle = mw2 ( a 2 - y 2 )
2 Dx = distance between the two faces
DQ = amount of heat conducted through the slab in time
1 æ a2 ö
= mw2 ´ ç a2 + ÷ Dt.
2 ç 4 ÷ø
è Dx
3 2 2
K.E. = mw a T + DT
8
1 1 a2 1 T
[ P.E. = mw2 y 2 = mw2 = mw2 a 2
2 2 4 8 A
K .E 3 / 8mw2 a 2 3
[ = = DQ
P.E 1/ 8mw2 a 2 1
[ Ratio = 3 : 1
u 2 sin 2q u 2 sin 2 q \ DQ
18. (a) Range, R = 2H Þ = 2´ Rate of conduction of heat =
g Dt
2g
DQ 1
Þ 2 sin q cos q= sin2q Þ tan q = 2 From experiment, µ A µ DT µ
Dt Dx
2 1
\ sin q= and cos q= DQ DT DQ DT
5 5 \ µA Þ =KA
Dt Dx Dt Dx
2u 2 2 1 4u 2 K = constant called coefficient of thermal conductivity of
\ R2 = . ´ = the solid.
g 5 5 5g DQ / Dt
(b) Here, T = 60 min = 60 × 60 s, q = 2p rad. \ K=
A( DT / Dx)
q 2p p 21. Perpendicular Axes Theorem
w= = = rad/s
t 60 ´ 60 1800
19. Ideal gas. A gas which obeys the ideal gas equation, z
PV = mRT at all temperatures and pressures is called an ideal
gas. In an ideal gas
(i) the size of the molecule is negligibly small and (ii) there
is no force of interaction among the molecules of the gas. x
But no real gas possesses these conditions and the
behaviour of real gas differs from that of an ideal gas. But
at high temperature and low pressure some real gasses do y
behave like an ideal gas.
SOLUTIONS SP-25
If I x and I y are the moments of inertia about two r r
r dr r dr
perpendicular axes of any body, the moment of inertia I \ v1 = 1 and v 2 = 2
about a 3rd perpendicular axis (z) is the sum of two dt dt
uur uur
moments of inertia, where all 3 axes are meeting at one Let f1 = external force on m1, f2 = external force on m2.
point and x and y axes are in the plane of the body and uur uur
z-axis is perpendicular to the plane of the body. F 12 ® force on m1 due to m2, F 21 ® force on m2 due
i.e., Iz = Ix + Iy to m1
Parallel Axes Theorem r r
The moment of inertia (I) about an axis parallel to the axis Linear momentum of m1 = p1 = m1v1
through C.M., is the sum of the moment of inertia about From Newton's 2nd law, total force applied on a particle
the axis through the C.M. and the product of the total mass is equal to its rate of change of linear momentum.
(M) and the square of the perpendicular distance (r) ur
d p1 uur ur
between the two axes concerned. \ = f1 + F 12
ICD = IAB + Mr2, where AB || CD. dt
d r uur ur
A C Þ (m1 v1 ) = f1 + F 12
dt
ur
d p2 ur ur d r ur ur
and = f 2 + F 21 Þ (m2 v 2 ) = f 2 + F 21
h dt dt
O
(C.M.) d r d r
\ (m1 v1 ) + (m2 v 2 )
dt dt
ur ur ur ur
B D = f 1 + F 12 + f 2 + F 21
22. (i) Wg = mg sinq d = 1 ´ 10 ´ 0.5 ´ 10 = 50 J d r r r r r
(m1v1 + m2 v2 ) = f1 + f2 = f
(ii)Wr = m mg cosq d = 0.1 ´10 ´ 0.866 ´ 10 = 8.66 J dt
DU = mgh = 1 ´ 10 ´ 5 = 50 J
(iii) r r
[as F12 = - F21 ]
(iv)a = {F – (mg sinq + m mg cosq )}
= [10 –5.87] = 4.13 m/s2 r r
d é dr1 dr2 ù r
v = u + at or v2 = u2 + 2ad m + m = f
dt êë dt dt úû
1 2
1 1
DK = mv 2 - mu 2 = mad = 41.3J
2 2 d éd r r ù r
Þ ê (m1r1 + m2 r2 )ú = f
(v) W = F d = 100 J dt ë dt û
SECTION - D r r
23. (a) Suraj is sympathetic also has the attitude of helping d 2 é m1r1 + m1r2 ù r
Þ (m1 + m2 ) ê ú= f
others and he has patience. dt 2 ë m1 + m2 û
(b) The downward force on the elevator is
r r
F = mg + Ff = (1800 × 10) + 4000 = 22000N m1 r 1 + m2 r 2 r
If we substitute = r CM then we get
m1 + m2
SECTION - E
24. Let m1, m2 ® masses of the 2 particles of the system, r
r r d 2 r CM ur
r1 , r2 ® position vectors of the particles. (m1 + m2 ) = f which is the equation of motion of
2
r r dt
v1 , v2 ® velocities of the particles. the centre of mass.
\ Position vector of the C.M. of a 2 particle system =
Y r r
m1 A r m r + m2 r2
r CM = 1 1
f1 m1 + m2
C.M.
r1 OR
r B
m2
r2 f2 Let us consider an n particle system of masses
X m1, m2 ..............., mn at the perpendicular distances
O
r1 , r2 ..............., rn respectively, about the axis of rotation.

Z
SP-26 PHYSICS
Z III. There are some other points, which vibrate about their
mean positions with largest amplitude. These are
called antinodes.
v2 When a string under tension is set into vibrations,
r1 m2 transverse harmonic wave propagates along its length.
m1 r2 When both the ends of the string are fixed, reflected and
the incident waves superimpose on each other, resulting
v1 in the formation of transverse stationary wave.
Let y1 be the incident wave represented by
O Y
2p
y1 = a sin (vt - x)
l
Then the reflected wave will be
X
2p
The angular momentum of each point mass is same as the y2 = - a sin (vt + x )
body is rigid and all its constituent particles take same time l
to complete one revolution. as the phase change of p takes place on reflection from a
r r r rigid boundary.
If v1 , v2 ..............., vn be their linear velocities respectively, The resultant wave will then be represented by the
superposition principle
then v1 = r1w, v2 = r2w,............., vn = rn w
y = y1 + y2
1 1 1 æ 2p 2p ö
K.E. of m1 = m1v12 = (r1w 2 ) = m1r12 w 2 , K.E. of = çè sin (vt - x) - sin (vt + x) ÷
2 2 2 l l ø
1 æ 2p ö æ 2vt ö æ 2p ö æ 2 x ö
m2 = m2 r22 w 2 , = 2a cos ç ÷ ç ÷ × sin ç ÷ ç - ÷
2 è l øè 2 ø è l øè 2 ø
1 2p 2p
K.E. of mn= m n rn2 w 2 y = - 2 a cos
vt sin x
2 l l
\ Total K.E. of rotation of the body This is the wave function for a standing wave. For this
1 1 1 2p 2L
= m 1r12 w2 + m 2 r22 w2 + .......... + m n rn2 w 2 wave, if L = np or l = the position of nodes and
2 2 2 l n
hence wavelength and frequency of set up wave can be
1 2 predicted.
= w [m1r12 + m2r22 + .............. + mnrn2]
2
n
1
= w2
2 i =1
å 1
mi ri 2 = w2 .
2 N N
A

n Fig. (a)
Where I = å mi ri2 = M.I of the body L = l1 / 2
2 =1
A N
2 × K.E. of rotation N N
\ I= 2 A
ω
If w = 1, then I = 2 × K.E. of rotation. Fig. (b)
\ M.I. of the rotating body can also be defined as twice L = l2
the K.E. of rotation when the body is moving with unit
angular velocity.
25. When two sets of progressive waves either both N N
A N A N A
longitudinal or both transverse having the same amplitude
and same frequency travel with the same speed along the
Fig. (c)
same straight line but in opposite directions superpose
each other, a new set of wave is formed, which is called L = 3l 3 / 2
stationary or standing waves. When the string vibrates as a whole in one segments, 2
Characteristics of stationary wave: nodes are formed at 2 fixed ends and an antinode is formed
I. They do not propagate in any direction. Also, they at the middle, and the string is said to vibrate fundamental
do not transfer any energy in the medium. mode or first harmonic.
II. There are certain points in the medium where the When the string vibrates in two segments, 3 nodes and
particles are permanently at rest, these are called 2 antinodes are formed and first overtone or second
nodes. harmonic is produced.
SOLUTIONS SP-27
When the string vibrates in three segments, 4 nodes and Case I :
3 antinodes are formed. When both S and O are moving towards each other
OR Due to the motion of observer the frequency will
(a) The phenomenon of the change in apparent frequency of change from n² to n²¢ so
sound due to relative motion between the source of sound
and the observer is called Doppler effect. Let S and O be v + vo
n¢¢¢ = n¢ S O
the positions of source and observer respectively and v v
and n be the velocity and frequency of the source of
sound, respectively. v + vo v æ v + vo ö
v = ´ n=ç ÷n
v v - vS è v - vS ø
O Case 2:
S
vs Both S and O move in same direction i.e. vo can be
O replaced by –vo in above relation
S S¢
v – vs æ v - vo ö S O
(i) Source (S) in motion and observer (O) stationary – \ n²¢ = ç v - v ÷ n
è Sø S O
When S is at rest, it will emit waves in one second and
these will occupy a space of length v in one second. Case 3 :
If l = wavelength of these waves, then Both S and O are moving away from each other.
v When source moves away from O
l= ...(i)
n
Let vs = velocity of source moving towards O at rest and æ v – vo ö
let S reaches to S¢ in one second. Thus the sound waves n¢ = ç v + v ÷ n S O
è Sø S O
will be crowded in length (v – vs). So if l¢ be the new
wavelength, When observer moves away from S the frequency n²
v - vs will change to n²¢
Then l'=
n æ v – vo ö
if n¢ be the apparent frequency, then v - vo n
\ n²¢ = n¢ = ç
v v v è v + vS ø÷
n' = = n
l ' v - vs 26. A Carnot engine is an ideal heat engine and consists of the
\ n ' > n i.e. when S moves towards O, the apparent following parts.
frequency of sound waves is greater than the actual
frequency. I. Source of heat: It is at a constant temperature T1 and
(ii) Observer (O) in motion and source (S) stationary of infinite thermal capacity. So, when the working
Let vo = velocity of observer moving towards S at substance draws heat from the source its temperature
rest.
v does not change.
O II. Sink of heat: It is also at a constant temperature T2 (T2
S < T1) and of infinite thermal capacity. So, when the
v0 heat is rejected to the sink its temperature doesn't
O¢ O change.
S
v0
As the observer moves towards S at rest, the velocity
of sound waves w.r.t. the observer is v + vo. 1 gm mole of
Then apparent frequency ideal gas
v + vo v + v0
n² = = v Source at Insulating Sink at
l v
temp. T1 pad temp. T2
It can be seen that n² > n
(iii) Both source, S and observer O are moving III. Working substance: It is an ideal gas (1 mole) in a
We know that cylinder with perfectly non-conduting walls but
When S moves towards stationary O perfectly conducting base and fitted with a perfectly
v non-conducting and frictionless piston.
Then n¢ = n
v - vS IV. Insulating pad: The cylinder is placed on it during
and when O moves towards stationary S. adiabatic operations so that no heat exchange can take
v + vo place.
Then n² = n
v
SP-28 PHYSICS
Theory: The Carnot cycle consists of 4 steps:
(a) Isothermal expansion
(b) Adiabatic expansion.
(c) Isothermal compression 1 gm mole of
(d) Adiabatic compression ideal gas

Let P 1 , V 1 , T 1 be the initial pressure, volume and


temperature of the gas and P2, V2, T2 be its final pressure,
Y
A (V1 , P1 ) volume and temperature.
Q1 If A be the area of cross section of the piston, then the force
B( V 2 , P2 ) exerted by the gas on the piston is F = P × A
T1 \ Work done by the gas for a small displacement dx of the
Pressure (P)

D(V 4 , P4 )
piston = dW = Fdx = PA dx = PdV
\ Total work done by the gas in expanding from volume V1
C(V3 , P3 )
Q2 to volume V2 = W
T2
X V2
O KL M N
Volume (V) = ò dW = ò PdV .
V1
Equation for adialatic change is PVg = K
Expression for efficiency K
\ P= = V -g K
g
V
W Q1 - Q2 Q
Efficiency = h = = = 1- 2 V
Q1 Q1 Q1
V2 éV -g + 1 ù 2 K
[V 1-g ]V2
V
òV
-g
\ W = KdV = K ê ú =
Using the equation of state for the four steps êë -g + 1 úû 1- g 1
V1 V1
We get, P1V1= P2V2 (isothermal expansion)
P2V2g = P3V3g (adiabatic expansion) K
= [V21–g – V11–g]
P3V3= P4V4 (isothermal compression) 1- g
P4V4g = P1V1g (adiabatic compression)
1
Multiplying the four equations we get, or, W = [KV21–g – KV11–g]
1- g
(P1V1) (P2V2g) (P3V3) (P4V4g ) 1
= [P V g.V 1-g – P1V1g V11-g]
= (P2V2) (P3V3g) (P4V4) (P1V1g) 1- g 2 2 2
V1 V2g V3 V4g = V2 V3g V4 V1g [Q PVg = K, \ P1V1g = P2V2g = K]
V2 g–1 V4 g–1 = V3 g–1 V1 g–1
1
Þ W= [P V – P1V1]
Þ (V2V4 )g–1 = (V1V3)g–1 Þ V2V4 = V1V3 1- g 2 2
Also, from ideal gas equation for 1 mole PV = RT
V2 V3 æV ö æV ö \ P1V1 = RT1 and P2V2 = RT2
Þ = Þ log ç 2 ÷ = log ç 3 ÷
V1 V 4 è V1 ø è V4 ø 1 R
\ W= [RT2 – RT1] = [T – T ]
1- g 1- g 2 1
Q2 RT2 log e (V3 / V4 ) T2 (b) (i) using, T1V1g–1 = T2V2 g – 1
Also, = =
Q1 RT1 log e (V2 / V1 ) T1 5
g -1 -1
æV ö æ 1ö 3
T2 = T1 ç 1 ÷ = 300 ç ÷ = 189K
T èV ø è 2ø
\ h = 1- 2 2
T1
nR DT
OR (ii) dU = ncv DT =
g -1
(a) Consider 1 gm mole of an ideal gas in a cylinder with
3
perfectly non-conducting walls and fitted with a = 2≥ ≥8.31(189 , 300) = – 2767.23 J.
perfectly non-conducting frictionless piston. 2
Q dQ = dU + dW but dQ = 0
[ dW = – dU = 2767.23J.
100% STUDENTS GOT ABOVE 90 PERCENTILE
EVERY YEAR IN JEE (MAIN) & NEET

KHUSHBOO JAIN VERSHA SHARMA Poornima Bhati DHRUV SAINI RAHUL SINGH RAHUL BHARDWAJ LAKSHAYA BANSAL
(Tagore, Sec-3) (Tagore, Sec-3) (Tagore, Sec-3) (Vidya Mandir) (Tagore, Sec-3) (Delhi School) (MVN, Sec-17)
NEET : 175/180 (Phy) NEET : 165/180 (Phy) NEET : 160/180 (Phy) NEET : 160/180 (Phy) NEET : 160/180 JEE MAIN : 99.97 Percentile (Phy) JEE MAIN : 99.94 Percentile (Phy)
NDMC (Delhi) Kalpana Chawla, Karnal) IGMC (Shimal) SNMC (Jodhpur) GSVM (Kanpur) CS (IIT-Delhi) ECE (IIT-Roorkee)

DAKSH CHAUDHARY RAMANUJ GOEL ABHISHKE ANJALI AYUSH GOEL POONAM BHATI SHREYA CHAUDHARY
(MVN, Sec-17) (Delhi School) (MVN, Sec-17) (MVN, Sec-17) (MVN Aravali Hills) (Tagore, Sec-3) (Tagore, Sec-3)
JEE MAIN : 99.63 Percentile (Phy) JEE MAIN : 99.49 Percentile (Phy) JEE MAIN : 98.21 Percentile (Phy) JEE MAIN : 98.75 Percentile (Phy) JEE MAIN : 99.00 Percentile (Phy) NEET : 145/180 (Phy) NEET : 165/180 (Phy)
CS (IIT-BHU) CS (IIT-Delhi) Mech. (NIT-Kurukshetra) ECE (NIT-Kurukshetra) CS (YMCA) BPS (Sonipat) PGI (Rohtak)

CHIRAG GOYAL LOVELY GARG ABHINAV BHUPESH GARG VIVEK GARG KUNAL PARASHAR HARSHIT
(Tagore, Sec-3) (Aggarwal School, Blb) (Tagore, Sec-3) (Tagore, Sec-3) (Tagore, Sec-3) (MVN Aravali Hills) (DAV, Sec-14)
NEET : 175/180 (Phy) JEE MAIN : 99.23 Percentile (Phy) JEE MAIN : 97.75 Percentile (Phy) JEE MAIN : 98.38 Percentile (Phy) JEE MAIN : 99.23 Percentile (Phy) JEE MAIN : 96.25 Percentile (Phy) JEE MAIN : 97.30Percentile (Phy)
BAMS (BHU) CS (YMCA) IIIT (Vadodara) CS (YMCA) CS (YMCA) CS (YMCA) CS (YMCA)

What Students Speaks...?

You might also like